You are on page 1of 601

WEEK 1,2 I.

Doing an Insurance Business

G.R. No. 76452 July 26, 1994 PHILIPPINE AMERICAN LIFE INSURANCE COMPANY and RODRIGO DE LOS REYES, petitioners, vs. HON. ARMANDO ANSALDO, in his capacity as Insurance Commissioner, and RAMON MONTILLA PATERNO, JR., respondents. Ponce Enrile, Cayetano, Reyes and Manalastas for petitioners. Oscar Z. Benares for private respondent.

QUIASON, J.: This is a petition for certiorari and prohibition under Rule 65 of the Revised Rules of Court, with preliminary injunction or temporary restraining order, to annul and set aside the Order dated November 6, 1986 of the Insurance Commissioner and the entire proceedings taken in I.C. Special Case No. 1-86. We grant the petition. The instant case arose from a letter-complaint of private respondent Ramon M. Paterno, Jr. dated April 17, 1986, to respondent Commissioner, alleging certain problems encountered by agents, supervisors, managers and public consumers of the Philippine American Life Insurance Company (Philamlife) as a result of certain practices by said company. In a letter dated April 23, 1986, respondent Commissioner requested petitioner Rodrigo de los Reyes, in his capacity as Philamlife's president, to comment on respondent Paterno's letter. In a letter dated April 29, 1986 to respondent Commissioner, petitioner De los Reyes suggested that private respondent "submit some sort of a 'bill of particulars' listing and citing actual cases, facts, dates, figures, provisions of law, rules and regulations, and all other pertinent data which are necessary to enable him to prepare an intelligent reply" (Rollo, p. 37). A copy of this letter was sent by the Insurance Commissioner to private respondent for his comments thereon. On May 16, 1986, respondent Commissioner received a letter from private respondent maintaining that his letter-complaint of April 17, 1986 was sufficient in form and substance, and requested that a hearing thereon be conducted. Petitioner De los Reyes, in his letter to respondent Commissioner dated June 6, 1986, reiterated his claim that private respondent's letter of May 16, 1986 did not supply the information he needed to enable him to answer the letter-complaint.

On July 14, a hearing on the letter-complaint was held by respondent Commissioner on the validity of the Contract of Agency complained of by private respondent. In said hearing, private respondent was required by respondent Commissioner to specify the provisions of the agency contract which he claimed to be illegal. On August 4, private respondent submitted a letter of specification to respondent Commissioner dated July 31, 1986, reiterating his letter of April 17, 1986 and praying that the provisions on charges and fees stated in the Contract of Agency executed between Philamlife and its agents, as well as the implementing provisions as published in the agents' handbook, agency bulletins and circulars, be declared as null and void. He also asked that the amounts of such charges and fees already deducted and collected by Philamlife in connection therewith be reimbursed to the agents, with interest at the prevailing rate reckoned from the date when they were deducted. Respondent Commissioner furnished petitioner De los Reyes with a copy of private respondent's letter of July 31, 1986, and requested his answer thereto. Petitioner De los Reyes submitted an Answer dated September 8, 1986, stating inter alia that: (1) Private respondent's letter of August 11, 1986 does not contain any of the particular information which Philamlife was seeking from him and which he promised to submit. (2) That since the Commission's quasi-judicial power was being invoked with regard to the complaint, private respondent must file a verified formal complaint before any further proceedings. In his letter dated September 9, 1986, private respondent asked for the resumption of the hearings on his complaint. On October 1, private respondent executed an affidavit, verifying his letters of April 17, 1986, and July 31, 1986. In a letter dated October 14, 1986, Manuel Ortega, Philamlife's Senior Assistant Vice-President and Executive Assistant to the President, asked that respondent Commission first rule on the questions of the jurisdiction of the Insurance Commissioner over the subject matter of the letters-complaint and the legal standing of private respondent. On October 27, respondent Commissioner notified both parties of the hearing of the case on November 5, 1986. On November 3, Manuel Ortega filed a Motion to Quash Subpoena/Notice on the following grounds; 1. The Subpoena/Notice has no legal basis and is premature because: (1) No complaint sufficient in form and contents has been filed;

(2) No summons has been issued nor received by the respondent De los Reyes, and hence, no jurisdiction has been acquired over his person; (3) No answer has been filed, and hence, the hearing scheduled on November 5, 1986 in the Subpoena/Notice, and wherein the respondent is required to appear, is premature and lacks legal basis. II. The Insurance Commission has no jurisdiction over; (1) the subject matter or nature of the action; and (2) over the parties involved (Rollo, p. 102). In the Order dated November 6, 1986, respondent Commissioner denied the Motion to Quash. The dispositive portion of said Order reads: NOW, THEREFORE, finding the position of complainant thru counsel tenable and considering the fact that the instant case is an informal administrative litigation falling outside the operation of the aforecited memorandum circular but cognizable by this Commission, the hearing officer, in open session ruled as it is hereby ruled to deny the Motion to Quash Subpoena/Notice for lack of merit (Rollo, p. 109). Hence, this petition. II The main issue to be resolved is whether or not the resolution of the legality of the Contract of Agency falls within the jurisdiction of the Insurance Commissioner. Private respondent contends that the Insurance Commissioner has jurisdiction to take cognizance of the complaint in the exercise of its quasi-judicial powers. The Solicitor General, upholding the jurisdiction of the Insurance Commissioner, claims that under Sections 414 and 415 of the Insurance Code, the Commissioner has authority to nullify the alleged illegal provisions of the Contract of Agency. III The general regulatory authority of the Insurance Commissioner is described in Section 414 of the Insurance Code, to wit: The Insurance Commissioner shall have the duty to see that all laws relating to insurance, insurance companies and other insurance matters, mutual benefit

associations and trusts for charitable uses are faithfully executed and to perform the duties imposed upon him by this Code, . . . On the other hand, Section 415 provides: In addition to the administrative sanctions provided elsewhere in this Code, the Insurance Commissioner is hereby authorized, at his discretion, to impose upon insurance companies, their directors and/or officers and/or agents, for any willful failure or refusal to comply with, or violation of any provision of this Code, or any order, instruction, regulation or ruling of the Insurance Commissioner, or any commission of irregularities, and/or conducting business in an unsafe and unsound manner as may be determined by the the Insurance Commissioner, the following: (a) fines not in excess of five hundred pesos a day; and (b) suspension, or after due hearing, removal of directors and/or officers and/or agents. A plain reading of the above-quoted provisions show that the Insurance Commissioner has the authority to regulate the business of insurance, which is defined as follows: (2) The term "doing an insurance business" or "transacting an insurance business," within the meaning of this Code, shall include (a) making or proposing to make, as insurer, any insurance contract; (b) making, or proposing to make, as surety, any contract of suretyship as a vocation and not as merely incidental to any other legitimate business or activity of the surety; (c) doing any kind of business, including a reinsurance business, specifically recognized as constituting the doing of an insurance business within the meaning of this Code; (d) doing or proposing to do any business in substance equivalent to any of the foregoing in a manner designed to evade the provisions of this Code. (Insurance Code, Sec. 2[2]; Emphasis supplied). Since the contract of agency entered into between Philamlife and its agents is not included within the meaning of an insurance business, Section 2 of the Insurance Code cannot be invoked to give jurisdiction over the same to the Insurance Commissioner. Expressio unius est exclusio alterius. With regard to private respondent's contention that the quasi-judicial power of the Insurance Commissioner under Section 416 of the Insurance Code applies in his case, we likewise rule in the negative. Section 416 of the Code in pertinent part, provides: The Commissioner shall have the power to adjudicate claims and complaints involving any loss, damage or liability for which an insurer may be answerable under any kind of policy or contract of insurance, or for which such insurer may be liable under a contract of suretyship, or for which a reinsurer may be used under any contract or reinsurance it may have entered into, or for which a mutual benefit association may be held liable under the membership certificates it has issued to its members, where the amount of any such loss, damage or liability,

excluding interest, costs and attorney's fees, being claimed or sued upon any kind of insurance, bond, reinsurance contract, or membership certificate does not exceed in any single claim one hundred thousand pesos. A reading of the said section shows that the quasi-judicial power of the Insurance Commissioner is limited by law "to claims and complaints involving any loss, damage or liability for which an insurer may be answerable under any kind of policy or contract of insurance, . . ." Hence, this power does not cover the relationship affecting the insurance company and its agents but is limited to adjudicating claims and complaints filed by the insured against the insurance company. While the subject of Insurance Agents and Brokers is discussed under Chapter IV, Title I of the Insurance Code, the provisions of said Chapter speak only of the licensing requirements and limitations imposed on insurance agents and brokers. The Insurance Code does not have provisions governing the relations between insurance companies and their agents. It follows that the Insurance Commissioner cannot, in the exercise of its quasi-judicial powers, assume jurisdiction over controversies between the insurance companies and their agents. We have held in the cases of Great Pacific Life Assurance Corporation v. Judico, 180 SCRA 445 (1989), andInvestment Planning Corporation of the Philippines v. Social Security Commission, 21 SCRA 904 (1962), that an insurance company may have two classes of agents who sell its insurance policies: (1) salaried employees who keep definite hours and work under the control and supervision of the company; and (2) registered representatives, who work on commission basis. Under the first category, the relationship between the insurance company and its agents is governed by the Contract of Employment and the provisions of the Labor Code, while under the second category, the same is governed by the Contract of Agency and the provisions of the Civil Code on the Agency. Disputes involving the latter are cognizable by the regular courts. WHEREFORE, the petition is GRANTED. The Order dated November 6, 1986 of the Insurance Commission is SET ASIDE. SO ORDERED. Cruz, Davide, Jr. and Kapunan, JJ., concur. Bellosillo, J,. is on leave.

[G.R. No. 125678. March 18, 2002]

PHILAMCARE HEALTH SYSTEMS, INC., petitioner, vs. COURT OF APPEALS and JULITA TRINOS, respondents.

DECISION
YNARES-SANTIAGO, J.:

Ernani Trinos, deceased husband of respondent Julita Trinos, applied for a health care coverage with petitioner Philamcare Health Systems, Inc. In the standard application form, he answered no to the following question:

Have you or any of your family members ever consulted or been treated for high blood pressure, heart trouble, diabetes, cancer, liver disease, asthma or peptic ulcer? (If Yes, give details).[1]
The application was approved for a period of one year from March 1, 1988 to March 1, 1989. Accordingly, he was issued Health Care Agreement No. P010194. Under the agreement, respondents husband was entitled to avail of hospitalization benefits, whether ordinary or emergency, listed therein. He was also entitled to avail of outpatient benefits such as annual physical examinations, preventive health care and other out-patient services. Upon the termination of the agreement, the same was extended for another year from March 1, 1989 to March 1, 1990, then from March 1, 1990 to June 1, 1990. The amount of coverage was increased to a maximum sum of P75,000.00 per disability. [2] During the period of his coverage, Ernani suffered a heart attack and was confined at the Manila Medical Center (MMC) for one month beginning March 9, 1990. While her husband was in the hospital, respondent tried to claim the benefits under the health care agreement. However, petitioner denied her claim saying that the Health Care Agreement was void. According to petitioner, there was a concealment regarding Ernanis medical history. Doctors at the MMC allegedly discovered at the time of Ernanis confinement that he was hypertensive, diabetic and asthmatic, contrary to his answer in the application form. Thus, respondent paid the hospitalization expenses herself, amounting to about P76,000.00. After her husband was discharged from the MMC, he was attended by a physical therapist at home. Later, he was admitted at the Chinese General Hospital. Due to financial difficulties, however, respondent brought her husband home again. In the morning of April 13, 1990, Ernani had fever and was feeling very weak. Respondent was constrained to bring him back to the Chinese General Hospital where he died on the same day.

On July 24, 1990, respondent instituted with the Regional Trial Court of Manila, Branch 44, an action for damages against petitioner and its president, Dr. Benito Reverente, which was docketed as Civil Case No. 90-53795. She asked for reimbursement of her expenses plus moral damages and attorneys fees. After trial, the lower court ruled against petitioners, viz:

WHEREFORE, in view of the forgoing, the Court renders judgment in favor of the plaintiff Julita Trinos, ordering: 1. Defendants to pay and reimburse the medical and hospital coverage of the late Ernani Trinos in the amount of P76,000.00 plus interest, until the amount is fully paid to plaintiff who paid the same; 2. Defendants to pay the reduced amount of moral damages of P10,000.00 to plaintiff; 3. Defendants to pay the reduced amount of P10,000.00 as exemplary damages to plaintiff; 4. Defendants to pay attorneys fees of P20,000.00, plus costs of suit.

SO ORDERED.[3]
On appeal, the Court of Appeals affirmed the decision of the trial court but deleted all awards for damages and absolved petitioner Reverente. [4] Petitioners motion for reconsideration was denied.[5] Hence, petitioner brought the instant petition for review, raising the primary argument that a health care agreement is not an insurance contract; hence the incontestability clause under the Insurance Code[6] does not apply. Petitioner argues that the agreement grants living benefits, such as medical check-ups and hospitalization which a member may immediately enjoy so long as he is alive upon effectivity of the agreement until its expiration one-year thereafter. Petitioner also points out that only medical and hospitalization benefits are given under the agreement without any indemnification, unlike in an insurance contract where the insured is indemnified for his loss. Moreover, since Health Care Agreements are only for a period of one year, as compared to insurance contracts which last longer,[7] petitioner argues that the incontestability clause does not apply, as the same requires an effectivity period of at least two years. Petitioner further argues that it is not an insurance company, which is governed by the Insurance Commission, but a Health Maintenance Organization under the authority of the Department of Health. Section 2 (1) of the Insurance Code defines a contract of insurance as an agreement whereby one undertakes for a consideration to indemnify another against loss, damage or liability arising from an unknown or contingent event. An insurance contract exists where the following elements concur:
1. The insured has an insurable interest;

2. The insured is subject to a risk of loss by the happening of the designated peril; 3. The insurer assumes the risk; 4. Such assumption of risk is part of a general scheme to distribute actual losses among a large group of persons bearing a similar risk; and 5. In consideration of the insurers promise, the insured pays a premium.[8]

Section 3 of the Insurance Code states that any contingent or unknown event, whether past or future, which may damnify a person having an insurable interest against him, may be insured against. Every person has an insurable interest in the life and health of himself. Section 10 provides:

Every person has an insurable interest in the life and health: (1) (2) (3) of himself, of his spouse and of his children; of any person on whom he depends wholly or in part for education or support, or in whom he has a pecuniary interest; of any person under a legal obligation to him for the payment of money, respecting property or service, of which death or illness might delay or prevent the performance; and of any person upon whose life any estate or interest vested in him depends.

(4)

In the case at bar, the insurable interest of respondents husband in obtaining the health care agreement was his own health. The health care agreement was in the nature of non-life insurance, which is primarily a contract of indemnity. [9] Once the member incurs hospital, medical or any other expense arising from sickness, injury or other stipulated contingent, the health care provider must pay for the same to the extent agreed upon under the contract. Petitioner argues that respondents husband concealed a material fact in his application. It appears that in the application for health coverage, petitioners required respondents husband to sign an express authorization for any person, organization or entity that has any record or knowledge of his health to furnish any and all information relative to any hospitalization, consultation, treatment or any other medical advice or examination.[10] Specifically, the Health Care Agreement signed by respondents husband states:

We hereby declare and agree that all statement and answers contained herein and in any addendum annexed to this application are full, complete and true and bind all parties in interest under the Agreement herein applied for, that there shall be no contract of health care coverage unless and until an

Agreement is issued on this application and the full Membership Fee according to the mode of payment applied for is actually paid during the lifetime and good health of proposed Members; that no information acquired by any Representative of PhilamCare shall be binding upon PhilamCare unless set out in writing in the application; that any physician is, by these presents, expressly authorized to disclose or give testimony at anytime relative to any information acquired by him in his professional capacity upon any question affecting the eligibility for health care coverage of the Proposed Members and that the acceptance of any Agreement issued on this application shall be a ratification of any correction in or addition to this application as stated in the space for Home Office Endorsement.[11] (Underscoring ours)
In addition to the above condition, petitioner additionally required the applicant for authorization to inquire about the applicants medical history, thus:

I hereby authorize any person, organization, or entity that has any record or knowledge of my health and/or that of __________ to give to the PhilamCare Health Systems, Inc. any and all information relative to any hospitalization, consultation, treatment or any other medical advice or examination. This authorization is in connection with the application for health care coverage only. A photographic copy of this authorization shall be as valid as the original.[12](Underscoring ours)
Petitioner cannot rely on the stipulation regarding Invalidation of agreement which reads:

Failure to disclose or misrepresentation of any material information by the member in the application or medical examination, whether intentional or unintentional, shall automatically invalidate the Agreement from the very beginning and liability of Philamcare shall be limited to return of all Membership Fees paid. An undisclosed or misrepresented information is deemed material if its revelation would have resulted in the declination of the applicant by Philamcare or the assessment of a higher Membership Fee for the benefit or benefits applied for.[13]
The answer assailed by petitioner was in response to the question relating to the medical history of the applicant. This largely depends on opinion rather than fact, especially coming from respondents husband who was not a medical doctor. Where matters of opinion or judgment are called for, answers made in good faith and without intent to deceive will not avoid a policy even though they are untrue. [14] Thus,

(A)lthough false, a representation of the expectation, intention, belief, opinion, or judgment of the insured will not avoid the policy if there is no actual fraud in inducing the acceptance of the risk, or its acceptance at a lower rate of premium, and this is likewise the rule although the statement is material to the risk, if the statement is obviously of the foregoing character, since in such case the insurer is not justified in relying upon such statement, but is obligated to make further inquiry. There is a clear distinction between such a case and one in which the insured is fraudulently and intentionally states to be true, as a matter of expectation or belief, that which he then knows, to be actually untrue, or the impossibility of which is shown by the facts within his knowledge, since in such case the intent to deceive the insurer is obvious and amounts to actual fraud.[15] (Underscoring ours)
The fraudulent intent on the part of the insured must be established to warrant rescission of the insurance contract.[16] Concealment as a defense for the health care provider or insurer to avoid liability is an affirmative defense and the duty to establish such defense by satisfactory and convincing evidence rests upon the provider or insurer. In any case, with or without the authority to investigate, petitioner is liable for claims made under the contract. Having assumed a responsibility under the agreement, petitioner is bound to answer the same to the extent agreed upon. In the end, the liability of the health care provider attaches once the member is hospitalized for the disease or injury covered by the agreement or whenever he avails of the covered benefits which he has prepaid. Under Section 27 of the Insurance Code, a concealment entitles the injured party to rescind a contract of insurance. The right to rescind should be exercised previous to the commencement of an action on the contract.[17] In this case, no rescission was made. Besides, the cancellation of health care agreements as in insurance policies require the concurrence of the following conditions:

1.

Prior notice of cancellation to insured;

2. Notice must be based on the occurrence after effective date of the policy of one or more of the grounds mentioned; 3. Must be in writing, mailed or delivered to the insured at the address shown in the policy; 4. Must state the grounds relied upon provided in Section 64 of the Insurance Code and upon request of insured, to furnish facts on which cancellation is based.[18]
None of the above pre-conditions was fulfilled in this case. When the terms of insurance contract contain limitations on liability, courts should construe them in such a

way as to preclude the insurer from non-compliance with his obligation.[19] Being a contract of adhesion, the terms of an insurance contract are to be construed strictly against the party which prepared the contract the insurer.[20] By reason of the exclusive control of the insurance company over the terms and phraseology of the insurance contract, ambiguity must be strictly interpreted against the insurer and liberally in favor of the insured, especially to avoid forfeiture.[21] This is equally applicable to Health Care Agreements. The phraseology used in medical or hospital service contracts, such as the one at bar, must be liberally construed in favor of the subscriber, and if doubtful or reasonably susceptible of two interpretations the construction conferring coverage is to be adopted, and exclusionary clauses of doubtful import should be strictly construed against the provider.[22] Anent the incontestability of the membership of respondents husband, we quote with approval the following findings of the trial court:

(U)nder the title Claim procedures of expenses, the defendant Philamcare Health Systems Inc. had twelve months from the date of issuance of the Agreement within which to contest the membership of the patient if he had previous ailment of asthma, and six months from the issuance of the agreement if the patient was sick of diabetes or hypertension. The periods having expired, the defense of concealment or misrepresentation no longer lie.[23]
Finally, petitioner alleges that respondent was not the legal wife of the deceased member considering that at the time of their marriage, the deceased was previously married to another woman who was still alive. The health care agreement is in the nature of a contract of indemnity. Hence, payment should be made to the party who incurred the expenses. It is not controverted that respondent paid all the hospital and medical expenses. She is therefore entitled to reimbursement. The records adequately prove the expenses incurred by respondent for the deceaseds hospitalization, medication and the professional fees of the attending physicians.[24] WHEREFORE, in view of the foregoing, the petition is DENIED. The assailed decision of the Court of Appeals dated December 14, 1995 is AFFIRMED. SO ORDERED. Davide, Jr., C.J., (Chairman), Puno, and Kapunan, JJ., concur.

II.

Cardinal Principles in Insurance

G.R. No. L-36413 September 26, 1988 MALAYAN INSURANCE CO., INC., petitioner, vs. THE HON. COURT OF APPEALS (THIRD DIVISION) MARTIN C. VALLEJOS, SIO CHOY, SAN LEON RICE MILL, INC. and PANGASINAN TRANSPORTATION CO., INC., respondents. Freqillana Jr. for petitioner. B.F. Estrella & Associates for respondent Martin Vallejos. Vicente Erfe Law Office for respondent Pangasinan Transportation Co., Inc. Nemesio Callanta for respondent Sio Choy and San Leon Rice Mill, Inc.

PADILLA, J.: Review on certiorari of the judgment * of the respondent appellate court in CA-G.R. No. 47319-R, dated 22 February 1973, which affirmed, with some modifications, the decision, ** dated 27 April 1970, rendered in Civil Case No. U-2021 of the Court of First Instance of Pangasinan. The antecedent facts of the case are as follows: On 29 March 1967, herein petitioner, Malayan Insurance Co., Inc., issued in favor of private respondent Sio Choy Private Car Comprehensive Policy No. MRO/PV-15753, effective from 18 April 1967 to 18 April 1968, covering a Willys jeep with Motor No. ET03023 Serial No. 351672, and Plate No. J-21536, Quezon City, 1967. The insurance coverage was for "own damage" not to exceed P600.00 and "third-party liability" in the amount of P20,000.00. During the effectivity of said insurance policy, and more particularly on 19 December 1967, at about 3:30 o'clock in the afternoon, the insured jeep, while being driven by one Juan P. Campollo an employee of the respondent San Leon Rice Mill, Inc., collided with a passenger bus belonging to the respondent Pangasinan Transportation Co., Inc. (PANTRANCO, for short) at the national highway in Barrio San Pedro, Rosales, Pangasinan, causing damage to the insured vehicle and injuries to the driver, Juan P. Campollo, and the respondent Martin C. Vallejos, who was riding in the ill-fated jeep.

As a result, Martin C. Vallejos filed an action for damages against Sio Choy, Malayan Insurance Co., Inc. and the PANTRANCO before the Court of First Instance of Pangasinan, which was docketed as Civil Case No. U-2021. He prayed therein that the defendants be ordered to pay him, jointly and severally, the amount of P15,000.00, as reimbursement for medical and hospital expenses; P6,000.00, for lost income; P51,000.00 as actual, moral and compensatory damages; and P5,000.00, for attorney's fees. Answering, PANTRANCO claimed that the jeep of Sio Choy was then operated at an excessive speed and bumped the PANTRANCO bus which had moved to, and stopped at, the shoulder of the highway in order to avoid the jeep; and that it had observed the diligence of a good father of a family to prevent damage, especially in the selection and supervision of its employees and in the maintenance of its motor vehicles. It prayed that it be absolved from any and all liability. Defendant Sio Choy and the petitioner insurance company, in their answer, also denied liability to the plaintiff, claiming that the fault in the accident was solely imputable to the PANTRANCO. Sio Choy, however, later filed a separate answer with a cross-claim against the herein petitioner wherein he alleged that he had actually paid the plaintiff, Martin C. Vallejos, the amount of P5,000.00 for hospitalization and other expenses, and, in his cross-claim against the herein petitioner, he alleged that the petitioner had issued in his favor a private car comprehensive policy wherein the insurance company obligated itself to indemnify Sio Choy, as insured, for the damage to his motor vehicle, as well as for any liability to third persons arising out of any accident during the effectivity of such insurance contract, which policy was in full force and effect when the vehicular accident complained of occurred. He prayed that he be reimbursed by the insurance company for the amount that he may be ordered to pay. Also later, the herein petitioner sought, and was granted, leave to file a third-party complaint against the San Leon Rice Mill, Inc. for the reason that the person driving the jeep of Sio Choy, at the time of the accident, was an employee of the San Leon Rice Mill, Inc. performing his duties within the scope of his assigned task, and not an employee of Sio Choy; and that, as the San Leon Rice Mill, Inc. is the employer of the deceased driver, Juan P. Campollo, it should be liable for the acts of its employee, pursuant to Art. 2180 of the Civil Code. The herein petitioner prayed that judgment be rendered against the San Leon Rice Mill, Inc., making it liable for the amounts claimed by the plaintiff and/or ordering said San Leon Rice Mill, Inc. to reimburse and indemnify the petitioner for any sum that it may be ordered to pay the plaintiff. After trial, judgment was rendered as follows: WHEREFORE, in view of the foregoing findings of this Court judgment is hereby rendered in favor of the plaintiff and against Sio Choy and Malayan

Insurance Co., Inc., and third-party defendant San Leon Rice Mill, Inc., as follows: (a) P4,103 as actual damages; (b) P18,000.00 representing the unearned income of plaintiff Martin C. Vallejos for the period of three (3) years; (c) P5,000.00 as moral damages; (d) P2,000.00 as attomey's fees or the total of P29,103.00, plus costs. The above-named parties against whom this judgment is rendered are hereby held jointly and severally liable. With respect, however, to Malayan Insurance Co., Inc., its liability will be up to only P20,000.00. As no satisfactory proof of cost of damage to its bus was presented by defendant Pantranco, no award should be made in its favor. Its counterclaim for attorney's fees is also dismissed for not being proved. 1 On appeal, the respondent Court of Appeals affirmed the judgment of the trial court that Sio Choy, the San Leon Rice Mill, Inc. and the Malayan Insurance Co., Inc. are jointly and severally liable for the damages awarded to the plaintiff Martin C. Vallejos. It ruled, however, that the San Leon Rice Mill, Inc. has no obligation to indemnify or reimburse the petitioner insurance company for whatever amount it has been ordered to pay on its policy, since the San Leon Rice Mill, Inc. is not a privy to the contract of insurance between Sio Choy and the insurance company. 2 Hence, the present recourse by petitioner insurance company. The petitioner prays for the reversal of the appellate court's judgment, or, in the alternative, to order the San Leon Rice Mill, Inc. to reimburse petitioner any amount, in excess of one-half (1/2) of the entire amount of damages, petitioner may be ordered to pay jointly and severally with Sio Choy. The Court, acting upon the petition, gave due course to the same, but "only insofar as it concerns the alleged liability of respondent San Leon Rice Mill, Inc. to petitioner, it being understood that no other aspect of the decision of the Court of Appeals shall be reviewed, hence, execution may already issue in favor of respondent Martin C. Vallejos against the respondents, without prejudice to the determination of whether or not petitioner shall be entitled to reimbursement by respondent San Leon Rice Mill, Inc. for the whole or part of whatever the former may pay on the P20,000.00 it has been adjudged to pay respondent Vallejos." 3 However, in order to determine the alleged liability of respondent San Leon Rice Mill, Inc. to petitioner, it is important to determine first the nature or basis of the liability of

petitioner to respondent Vallejos, as compared to that of respondents Sio Choy and San Leon Rice Mill, Inc. Therefore, the two (2) principal issues to be resolved are (1) whether the trial court, as upheld by the Court of Appeals, was correct in holding petitioner and respondents Sio Choy and San Leon Rice Mill, Inc. "solidarily liable" to respondent Vallejos; and (2) whether petitioner is entitled to be reimbursed by respondent San Leon Rice Mill, Inc. for whatever amount petitioner has been adjudged to pay respondent Vallejos on its insurance policy. As to the first issue, it is noted that the trial court found, as affirmed by the appellate court, that petitioner and respondents Sio Choy and San Leon Rice Mill, Inc. are jointly and severally liable to respondent Vallejos. We do not agree with the aforesaid ruling. We hold instead that it is only respondents Sio Choy and San Leon Rice Mill, Inc, (to the exclusion of the petitioner) that are solidarily liable to respondent Vallejos for the damages awarded to Vallejos. It must be observed that respondent Sio Choy is made liable to said plaintiff as owner of the ill-fated Willys jeep, pursuant to Article 2184 of the Civil Code which provides: Art. 2184. In motor vehicle mishaps, the owner is solidarily liable with his driver, if the former, who was in the vehicle, could have, by the use of due diligence, prevented the misfortune it is disputably presumed that a driver was negligent, if he had been found guilty of reckless driving or violating traffic regulations at least twice within the next preceding two months. If the owner was not in the motor vehicle, the provisions of article 2180 are applicable. On the other hand, it is noted that the basis of liability of respondent San Leon Rice Mill, Inc. to plaintiff Vallejos, the former being the employer of the driver of the Willys jeep at the time of the motor vehicle mishap, is Article 2180 of the Civil Code which reads: Art. 2180. The obligation imposed by article 2176 is demandable not only for one's own acts or omissions, but also for those of persons for whom one is responsible. xxx xxx xxx Employers shall be liable for the damages caused by their employees and household helpers acting within the scope of their assigned tasks, even though the former are not engaged ill any business or industry. xxx xxx xxx

The responsibility treated in this article shall cease when the persons herein mentioned proved that they observed all the diligence of a good father of a family to prevent damage. It thus appears that respondents Sio Choy and San Leon Rice Mill, Inc. are the principal tortfeasors who are primarily liable to respondent Vallejos. The law states that the responsibility of two or more persons who are liable for a quasi-delict is solidarily. 4 On the other hand, the basis of petitioner's liability is its insurance contract with respondent Sio Choy. If petitioner is adjudged to pay respondent Vallejos in the amount of not more than P20,000.00, this is on account of its being the insurer of respondent Sio Choy under the third party liability clause included in the private car comprehensive policy existing between petitioner and respondent Sio Choy at the time of the complained vehicular accident. In Guingon vs. Del Monte, 5 a passenger of a jeepney had just alighted therefrom, when he was bumped by another passenger jeepney. He died as a result thereof. In the damage suit filed by the heirs of said passenger against the driver and owner of the jeepney at fault as well as against the insurance company which insured the latter jeepney against third party liability, the trial court, affirmed by this Court, adjudged the owner and the driver of the jeepney at fault jointly and severally liable to the heirs of the victim in the total amount of P9,572.95 as damages and attorney's fees; while the insurance company was sentenced to pay the heirs the amount of P5,500.00 which was to be applied as partial satisfaction of the judgment rendered against said owner and driver of the jeepney. Thus, in said Guingon case, it was only the owner and the driver of the jeepney at fault, not including the insurance company, who were held solidarily liable to the heirs of the victim. While it is true that where the insurance contract provides for indemnity against liability to third persons, such third persons can directly sue the insurer, 6 however, the direct liability of the insurer under indemnity contracts against third party liability does not mean that the insurer can be held solidarily liable with the insured and/or the other parties found at fault. The liability of the insurer is based on contract; that of the insured is based on tort. In the case at bar, petitioner as insurer of Sio Choy, is liable to respondent Vallejos, but it cannot, as incorrectly held by the trial court, be made "solidarily" liable with the two principal tortfeasors namely respondents Sio Choy and San Leon Rice Mill, Inc. For if petitioner-insurer were solidarily liable with said two (2) respondents by reason of the indemnity contract against third party liability-under which an insurer can be directly sued by a third party this will result in a violation of the principles underlying solidary obligation and insurance contracts. In solidary obligation, the creditor may enforce the entire obligation against one of the solidary debtors. 7 On the other hand, insurance is defined as "a contract whereby one

undertakes for a consideration to indemnify another against loss, damage, or liability arising from an unknown or contingent event." 8 In the case at bar, the trial court held petitioner together with respondents Sio Choy and San Leon Rice Mills Inc. solidarily liable to respondent Vallejos for a total amount of P29,103.00, with the qualification that petitioner's liability is only up to P20,000.00. In the context of a solidary obligation, petitioner may be compelled by respondent Vallejos to pay the entire obligation of P29,013.00, notwithstanding the qualification made by the trial court. But, how can petitioner be obliged to pay the entire obligation when the amount stated in its insurance policy with respondent Sio Choy for indemnity against third party liability is only P20,000.00? Moreover, the qualification made in the decision of the trial court to the effect that petitioner is sentenced to pay up to P20,000.00 only when the obligation to pay P29,103.00 is made solidary, is an evident breach of the concept of a solidary obligation. Thus, We hold that the trial court, as upheld by the Court of Appeals, erred in holding petitioner, solidarily liable with respondents Sio Choy and San Leon Rice Mill, Inc. to respondent Vallejos. As to the second issue, the Court of Appeals, in affirming the decision of the trial court, ruled that petitioner is not entitled to be reimbursed by respondent San Leon Rice Mill, Inc. on the ground that said respondent is not privy to the contract of insurance existing between petitioner and respondent Sio Choy. We disagree. The appellate court overlooked the principle of subrogation in insurance contracts. Thus ... Subrogation is a normal incident of indemnity insurance (Aetna L. Ins. Co. vs. Moses, 287 U.S. 530, 77 L. ed. 477). Upon payment of the loss, the insurer is entitled to be subrogated pro tanto to any right of action which the insured may have against the third person whose negligence or wrongful act caused the loss (44 Am. Jur. 2nd 745, citing Standard Marine Ins. Co. vs. Scottish Metropolitan Assurance Co., 283 U.S. 284, 75 L. ed. 1037). The right of subrogation is of the highest equity. The loss in the first instance is that of the insured but after reimbursement or compensation, it becomes the loss of the insurer (44 Am. Jur. 2d, 746, note 16, citing Newcomb vs. Cincinnati Ins. Co., 22 Ohio St. 382). Although many policies including policies in the standard form, now provide for subrogation, and thus determine the rights of the insurer in this respect, the equitable right of subrogation as the legal effect of payment inures to the insurer without any formal assignment or any express stipulation to that effect in the policy" (44 Am. Jur. 2nd 746). Stated otherwise, when the insurance company pays for the loss, such payment operates as an equitable assignment to the insurer of the property and all remedies which the insured may have for the recovery thereof. That right

is not dependent upon , nor does it grow out of any privity of contract (emphasis supplied) or upon written assignment of claim, and payment to the insured makes the insurer assignee in equity (Shambley v. Jobe-Blackley Plumbing and Heating Co., 264 N.C. 456, 142 SE 2d 18). 9 It follows, therefore, that petitioner, upon paying respondent Vallejos the amount of riot exceeding P20,000.00, shall become the subrogee of the insured, the respondent Sio Choy; as such, it is subrogated to whatever rights the latter has against respondent San Leon Rice Mill, Inc. Article 1217 of the Civil Code gives to a solidary debtor who has paid the entire obligation the right to be reimbursed by his co-debtors for the share which corresponds to each. Art. 1217. Payment made by one of the solidary debtors extinguishes the obligation. If two or more solidary debtors offer to pay, the creditor may choose which offer to accept. He who made the payment may claim from his co-debtors only the share which corresponds to each, with the interest for the payment already made. If the payment is made before the debt is due, no interest for the intervening period may be demanded. xxx xxx xxx In accordance with Article 1217, petitioner, upon payment to respondent Vallejos and thereby becoming the subrogee of solidary debtor Sio Choy, is entitled to reimbursement from respondent San Leon Rice Mill, Inc. To recapitulate then: We hold that only respondents Sio Choy and San Leon Rice Mill, Inc. are solidarily liable to the respondent Martin C. Vallejos for the amount of P29,103.00. Vallejos may enforce the entire obligation on only one of said solidary debtors. If Sio Choy as solidary debtor is made to pay for the entire obligation (P29,103.00) and petitioner, as insurer of Sio Choy, is compelled to pay P20,000.00 of said entire obligation, petitioner would be entitled, as subrogee of Sio Choy as against San Leon Rice Mills, Inc., to be reimbursed by the latter in the amount of P14,551.50 (which is 1/2 of P29,103.00 ) WHEREFORE, the petition is GRANTED. The decision of the trial court, as affirmed by the Court of Appeals, is hereby AFFIRMED, with the modification above-mentioned. Without pronouncement as to costs. SO ORDERED. Melencio-Herrera (Chairperson), Paras, Sarmiento and Regalado, JJ., concur.

MALAYAN INSURANCE CO., INC., Petitioner,

G.R. No. 194320 Present:

- versus -

RODELIO ALBERTO and ENRICO ALBERTO REYES, Respondents.

VELASCO, JR., J., Chairperson, PERALTA, MENDOZA, REYES,* and PERLAS-BERNABE, JJ. Promulgated:

February 1, 2012 x-----------------------------------------------------------------------------------------x DECISION

VELASCO, JR., J.: The Case Before Us is a Petition for Review on Certiorari under Rule 45, seeking to reverse and set aside the July 28, 2010 Decision[1] of the Court of Appeals (CA) and its October 29, 2010 Resolution[2] denying the motion for reconsideration filed by petitioner Malayan Insurance Co., Inc. (Malayan Insurance). The July 28, 2010 CA Decision reversed and set aside the Decision[3] dated February 2, 2009 of the Regional Trial Court, Branch 51 in Manila.

The Facts At around 5 oclock in the morning of December 17, 1995, an accident occurred at the corner of EDSA and Ayala Avenue, Makati City, involving four (4) vehicles, to wit: (1) a Nissan Bus operated by Aladdin Transit with plate number NYS 381; (2) an Isuzu Tanker with plate number PLR 684; (3) a Fuzo Cargo Truckwith plate number PDL 297; and (4) a Mitsubishi Galant with plate number TLM 732.[4]

Based on the Police Report issued by the on-the-spot investigator, Senior Police Officer 1 Alfredo M. Dungga (SPO1 Dungga), the Isuzu Tanker was in front of the Mitsubishi Galant with the Nissan Bus on their right side shortly before the vehicular incident. All three (3) vehicles were at a halt along EDSA facing the south direction when the Fuzo Cargo Truck simultaneously bumped the rear portion of the Mitsubishi Galant and the rear left portion of the Nissan Bus. Due to the strong impact, these two vehicles were shoved forward and the front left portion of the Mitsubishi Galant rammed into the rear right portion of the Isuzu Tanker.[5] Previously, particularly on December 15, 1994, Malayan Insurance issued Car Insurance Policy No. PV-025-00220 in favor of First Malayan Leasing and Finance Corporation (the assured), insuring the aforementioned Mitsubishi Galant against third party liability, own damage and theft, among others. Having insured the vehicle against such risks, Malayan Insurance claimed in its Complaint dated October 18, 1999 that it paid the damages sustained by the assured amounting to PhP 700,000. [6] Maintaining that it has been subrogated to the rights and interests of the assured by operation of law upon its payment to the latter, Malayan Insurance sent several demand letters to respondents Rodelio Alberto (Alberto) and Enrico Alberto Reyes (Reyes), the registered owner and the driver, respectively, of the Fuzo Cargo Truck, requiring them to pay the amount it had paid to the assured. When respondents refused to settle their liability, Malayan Insurance was constrained to file a complaint for damages for gross negligence against respondents.[7] In their Answer, respondents asserted that they cannot be held liable for the vehicular accident, since its proximate cause was the reckless driving of the Nissan Bus driver. They alleged that the speeding bus, coming from the service road of EDSA, maneuvered its way towards the middle lane without due regard to Reyes right of way. When the Nissan Bus abruptly stopped, Reyes stepped hard on the brakes but the braking action could not cope with the inertia and failed to gain sufficient traction. As a consequence, the Fuzo Cargo Truck hit the rear end of the Mitsubishi Galant, which, in turn, hit the rear end of the vehicle in front of it. The Nissan Bus, on the other hand, sideswiped the Fuzo Cargo Truck, causing damage to the latter in the

amount of PhP 20,000. Respondents also controverted the results of the Police Report, asserting that it was based solely on the biased narration of the Nissan Bus driver.[8] After the termination of the pre-trial proceedings, trial ensued. Malayan Insurance presented the testimony of its lone witness, a motor car claim adjuster, who attested that he processed the insurance claim of the assured and verified the documents submitted to him. Respondents, on the other hand, failed to present any evidence. In its Decision dated February 2, 2009, the trial court, in Civil Case No. 99-95885, ruled in favor of Malayan Insurance and declared respondents liable for damages. The dispositive portion reads: WHEREFORE, judgment is hereby rendered in favor of the plaintiff against defendants jointly and severally to pay plaintiff the following: 1. The amount of P700,000.00 with legal interest from the time of the filing of the complaint; 2. Attorneys fees of P10,000.00 and; 3. Cost of suit. SO ORDERED.[9]

Dissatisfied, respondents filed an appeal with the CA, docketed as CA-G.R. CV No. 93112. In its Decision dated July 28, 2010, the CA reversed and set aside the Decision of the trial court and ruled in favor of respondents, disposing: WHEREFORE, the foregoing considered, the instant appeal is hereby GRANTED and the assailed Decision dated 2 February 2009 REVERSED and SET ASIDE. The Complaint dated 18 October 1999 is hereby DISMISSED for lack of merit. No costs. SO ORDERED.[10]

The CA held that the evidence on record has failed to establish not only negligence on the part of respondents, but also compliance with the other requisites and the consequent right of Malayan Insurance to subrogation. [11] It noted that the police report, which has been made part of the records of the trial court, was not properly

identified by the police officer who conducted the on-the-spot investigation of the subject collision. It, thus, held that an appellate court, as a reviewing body, cannot rightly appreciate firsthand the genuineness of an unverified and unidentified document, much less accord it evidentiary value.[12] Subsequently, Malayan Insurance filed its Motion for Reconsideration, arguing that a police report is a prima facie evidence of the facts stated in it. And inasmuch as they never questioned the presentation of the report in evidence, respondents are deemed to have waived their right to question its authenticity and due execution. [13] In its Resolution dated October 29, 2010, the CA denied the motion for reconsideration. Hence, Malayan Insurance filed the instant petition.

The Issues In its Memorandum[14] dated June 27, 2011, Malayan Insurance raises the following issues for Our consideration: I WHETHER THE CA ERRED IN REFUSING ADMISSIBILITY OF THE POLICE REPORT SINCE THE POLICE INVESTIGATOR WHO PREPARED THE SAME DID NOT ACTUALLY TESTIFY IN COURT THEREON. II WHETHER THE SUBROGATION OF MALAYAN INSURANCE IS IMPAIRED AND/OR DEFICIENT.

On

the
[15]

other

hand,

respondents

submit

the

following

issues

in

its

Memorandum

dated July 7, 2011:

I WHETHER THE CA IS CORRECT IN DISMISSING THE COMPLAINT FOR FAILURE OF MALAYAN INSURANCE TO OVERCOME THE BURDEN OF PROOF REQUIRED TO ESTABLISH THE NEGLIGENCE OF RESPONDENTS. II

WHETHER THE PIECES OF EVIDENCE PRESENTED BY MALAYAN INSURANCE ARE SUFFICIENT TO CLAIM FOR THE AMOUNT OF DAMAGES. III WHETHER THE SUBROGATION OF MALAYAN INSURANCE HAS PASSED COMPLIANCE AND REQUISITES AS PROVIDED UNDER PERTINENT LAWS.

Essentially, the issues boil down to the following: (1) the admissibility of the police report; (2) the sufficiency of the evidence to support a claim for gross negligence; and (3) the validity of subrogation in the instant case. Our Ruling The petition has merit. Admissibility of the Police Report Malayan Insurance contends that, even without the presentation of the police investigator who prepared the police report, said report is still admissible in evidence, especially since respondents failed to make a timely objection to its presentation in evidence.[16] Respondents counter that since the police report was never confirmed by the investigating police officer, it cannot be considered as part of the evidence on record.[17] Indeed, under the rules of evidence, a witness can testify only to those facts which the witness knows of his or her personal knowledge, that is, which are derived from the witness own perception.[18] Concomitantly, a witness may not testify on matters which he or she merely learned from others either because said witness was told or read or heard those matters.[19] Such testimony is considered hearsay and may not be received as proof of the truth of what the witness has learned. This is known as the hearsay rule.[20]

As discussed in D.M. Consunji, Inc. v. CA,[21] Hearsay is not limited to oral testimony or statements; the general rule that excludes hearsay as evidence applies to written, as well as oral statements. There are several exceptions to the hearsay rule under the Rules of Court, among which are entries in official records.[22] Section 44, Rule 130 provides: Entries in official records made in the performance of his duty by a public officer of the Philippines, or by a person in the performance of a duty specially enjoined by law are prima facie evidence of the facts therein stated.

In Alvarez v. PICOP Resources,[23] this Court reiterated the requisites for the admissibility in evidence, as an exception to the hearsay rule of entries in official records, thus: (a) that the entry was made by a public officer or by another person specially enjoined by law to do so; (b) that it was made by the public officer in the performance of his or her duties, or by such other person in the performance of a duty specially enjoined by law; and (c) that the public officer or other person had sufficient knowledge of the facts by him or her stated, which must have been acquired by the public officer or other person personally or through official information. Notably, the presentation of the police report itself is admissible as an exception to the hearsay rule even if the police investigator who prepared it was not presented in court, as long as the above requisites could be adequately proved.[24] Here, there is no dispute that SPO1 Dungga, the on-the-spot investigator, prepared the report, and he did so in the performance of his duty. However, what is not clear is whether SPO1 Dungga had sufficient personal knowledge of the facts contained in his report. Thus, the third requisite is lacking. Respondents failed to make a timely objection to the police reports presentation in evidence; thus, they are deemed to have waived their right to do so.[25] As a result, the police report is still admissible in evidence.

Sufficiency of Evidence Malayan Insurance contends that since Reyes, the driver of the Fuzo Cargo truck, bumped the rear of the Mitsubishi Galant, he is presumed to be negligent unless proved otherwise. It further contends that respondents failed to present any evidence to overturn the presumption of negligence.[26] Contrarily, respondents claim that since Malayan Insurance did not present any witness who shall affirm any negligent act of Reyes in driving the Fuzo Cargo truck before and after the incident, there is no evidence which would show negligence on the part of respondents.[27] We agree with Malayan Insurance. Even if We consider the inadmissibility of the police report in evidence, still, respondents cannot evade liability by virtue of the res ipsa loquitur doctrine. The D.M. Consunji, Inc. case is quite elucidating: Petitioners contention, however, loses relevance in the face of the application of res ipsa loquitur by the CA. The effect of the doctrine is to warrant a presumption or inference that the mere fall of the elevator was a result of the person having charge of the instrumentality was negligent. As a rule of evidence, the doctrine of res ipsa loquitur is peculiar to the law of negligence which recognizes that prima facie negligence may be established without direct proof and furnishes a substitute for specific proof of negligence. The concept of res ipsa loquitur has been explained in this wise: While negligence is not ordinarily inferred or presumed, and while the mere happening of an accident or injury will not generally give rise to an inference or presumption that it was due to negligence on defendants part, under the doctrine of res ipsa loquitur, which means, literally, the thing or transaction speaks for itself, or in one jurisdiction, that the thing or instrumentality speaks for itself, the facts or circumstances accompanying an injury may be such as to raise a presumption, or at least permit an inference of negligence on the part of the defendant, or some other person who is charged with negligence. x x x where it is shown that the thing or instrumentality which caused the injury complained of was under the control or management of the defendant, and that the occurrence resulting in the injury was such as in the ordinary course of things would not happen if those who had its control or management used proper

care, there is sufficient evidence, or, as sometimes stated, reasonable evidence, in the absence of explanation by the defendant, that the injury arose from or was caused by the defendants want of care. One of the theoretical bases for the doctrine is its necessity, i.e., that necessary evidence is absent or not available. The res ipsa loquitur doctrine is based in part upon the theory that the defendant in charge of the instrumentality which causes the injury either knows the cause of the accident or has the best opportunity of ascertaining it and that the plaintiff has no such knowledge, and therefore is compelled to allege negligence in general terms and to rely upon the proof of the happening of the accident in order to establish negligence. The inference which the doctrine permits is grounded upon the fact that the chief evidence of the true cause, whether culpable or innocent, is practically accessible to the defendant but inaccessible to the injured person. It has been said that the doctrine of res ipsa loquitur furnishes a bridge by which a plaintiff, without knowledge of the cause, reaches over to defendant who knows or should know the cause, for any explanation of care exercised by the defendant in respect of the matter of which the plaintiff complains. The res ipsa loquitur doctrine, another court has said, is a rule of necessity, in that it proceeds on the theory that under the peculiar circumstances in which the doctrine is applicable, it is within the power of the defendant to show that there was no negligence on his part, and direct proof of defendants negligence is beyond plaintiffs power. Accordingly, some courts add to the three prerequisites for the application of theres ipsa loquitur doctrine the further requirement that for the res ipsa loquitur doctrine to apply, it must appear that the injured party had no knowledge or means of knowledge as to the cause of the accident, or that the party to be charged with negligence has superior knowledge or opportunity for explanation of the accident. The CA held that all the requisites of res ipsa loquitur are present in the case at bar: There is no dispute that appellees husband fell down from the 14th floor of a building to the basement while he was working with appellants construction project, resulting to his death. The construction site is within the exclusive control and management of appellant. It has a safety engineer, a project superintendent, a carpenter leadman and others who are in complete control of the

situation therein. The circumstances of any accident that would occur therein are peculiarly within the knowledge of the appellant or its employees. On the other hand, the appellee is not in a position to know what caused the accident. Res ipsa loquitur is a rule of necessity and it applies where evidence is absent or not readily available, provided the following requisites are present: (1) the accident was of a kind which does not ordinarily occur unless someone is negligent; (2) the instrumentality or agency which caused the injury was under the exclusive control of the person charged with negligence; and (3) the injury suffered must not have been due to any voluntary action or contribution on the part of the person injured. x x x. No worker is going to fall from the 14th floor of a building to the basement while performing work in a construction site unless someone is negligent[;] thus, the first requisite for the application of the rule of res ipsa loquitur is present. As explained earlier, the construction site with all its paraphernalia and human resources that likely caused the injury is under the exclusive control and management of appellant[;] thus[,] the second requisite is also present. No contributory negligence was attributed to the appellees deceased husband[;] thus[,] the last requisite is also present. All the requisites for the application of the rule of res ipsa loquitur are present, thus a reasonable presumption or inference of appellants negligence arises. x x x. Petitioner does not dispute the existence of the requisites for the application of res ipsa loquitur, but argues that the presumption or inference that it was negligent did not arise since it proved that it exercised due care to avoid the accident which befell respondents husband. Petitioner apparently misapprehends the procedural effect of the doctrine. As stated earlier, the defendants negligence is presumed or inferred when the plaintiff establishes the requisites for the application of res ipsa loquitur. Once the plaintiff makes out a prima facie case of all the elements, the burden then shifts to defendant to explain. The presumption or inference may be rebutted or overcome by other evidence and, under appropriate circumstances a disputable presumption, such as that of due care or innocence, may outweigh the inference. It is not for the defendant to explain or prove its defense to prevent the presumption or inference from arising. Evidence by the defendant of say, due care, comes into play only after the circumstances for the application of the doctrine has been established.[28]

In the case at bar, aside from the statement in the police report, none of the parties disputes the fact that the Fuzo Cargo Truck hit the rear end of the Mitsubishi Galant, which, in turn, hit the rear end of the vehicle in front of it. Respondents, however, point to the reckless driving of the Nissan Bus driver as the proximate cause of the collision, which allegation is totally unsupported by any evidence on record. And assuming that this allegation is, indeed, true, it is astonishing that respondents never even bothered to file a cross-claim against the owner or driver of the Nissan Bus. What is at once evident from the instant case, however, is the presence of all the requisites for the application of the rule of res ipsa loquitur. To reiterate, res ipsa loquitur is a rule of necessity which applies where evidence is absent or not readily available. As explained in D.M. Consunji, Inc., it is partly based upon the theory that the defendant in charge of the instrumentality which causes the injury either knows the cause of the accident or has the best opportunity of ascertaining it and that the plaintiff has no such knowledge, and, therefore, is compelled to allege negligence in general terms and to rely upon the proof of the happening of the accident in order to establish negligence. As mentioned above, the requisites for the application of the res ipsa loquitur rule are the following: (1) the accident was of a kind which does not ordinarily occur unless someone is negligent; (2) the instrumentality or agency which caused the injury was under the exclusive control of the person charged with negligence; and (3) the injury suffered must not have been due to any voluntary action or contribution on the part of the person injured.[29] In the instant case, the Fuzo Cargo Truck would not have had hit the rear end of the Mitsubishi Galant unless someone is negligent. Also, the Fuzo Cargo Truck was under the exclusive control of its driver, Reyes. Even if respondents avert liability by putting the blame on the Nissan Bus driver, still, this allegation was self-serving and totally unfounded. Finally, no contributory negligence was attributed to the driver of the Mitsubishi Galant. Consequently, all the requisites for the application of the doctrine of res ipsa loquitur are present, thereby creating a reasonable presumption of negligence on the part of respondents.

It is worth mentioning that just like any other disputable presumptions or inferences, the presumption of negligence may be rebutted or overcome by other evidence to the contrary. It is unfortunate, however, that respondents failed to present any evidence before the trial court. Thus, the presumption of negligence remains. Consequently, the CA erred in dismissing the complaint for Malayan Insurances adverted failure to prove negligence on the part of respondents. Validity of Subrogation Malayan Insurance contends that there was a valid subrogation in the instant case, as evidenced by the claim check voucher[30] and the Release of Claim and Subrogation Receipt[31] presented by it before the trial court. Respondents, however, claim that the documents presented by Malayan Insurance do not indicate certain important details that would show proper subrogation. As noted by Malayan Insurance, respondents had all the opportunity, but failed to object to the presentation of its evidence. Thus, and as We have mentioned earlier, respondents are deemed to have waived their right to make an objection. As this Court held in Asian Construction and Development Corporation v. COMFAC Corporation:

The rule is that failure to object to the offered evidence renders it admissible, and the court cannot, on its own, disregard such evidence. We note that ASIAKONSTRUCTs counsel of record before the trial court, Atty. Bernard Dy, who actively participated in the initial stages of the case stopped attending the hearings when COMFAC was about to end its presentation. Thus, ASIAKONSTRUCT could not object to COMFACs offer of evidence nor present evidence in its defense; ASIAKONSTRUCT was deemed by the trial court to have waived its chance to do so. Note also that when a party desires the court to reject the evidence offered, it must so state in the form of a timely objection and it cannot raise the objection to the evidence for the first time on appeal. Because of a partys failure to timely object, the evidence becomes part of the evidence in the case. Thereafter, all the parties are considered bound by any outcome arising from the offer of evidence properly presented.[32] (Emphasis supplied.)

Bearing in mind that the claim check voucher and the Release of Claim and Subrogation Receipt presented by Malayan Insurance are already part of the evidence on record, and since it is not disputed that the insurance company, indeed, paid PhP 700,000 to the assured, then there is a valid subrogation in the case at bar. As explained in Keppel Cebu Shipyard, Inc. v. Pioneer Insurance and Surety Corporation :

Subrogation is the substitution of one person by another with reference to a lawful claim or right, so that he who is substituted succeeds to the rights of the other in relation to a debt or claim, including its remedies or securities. The principle covers a situation wherein an insurer has paid a loss under an insurance policy is entitled to all the rights and remedies belonging to the insured against a third party with respect to any loss covered by the policy. It contemplates full substitution such that it places the party subrogated in the shoes of the creditor, and he may use all means that the creditor could employ to enforce payment. We have held that payment by the insurer to the insured operates as an equitable assignment to the insurer of all the remedies that the insured may have against the third party whose negligence or wrongful act caused the loss. The right of subrogation is not dependent upon, nor does it grow out of, any privity of contract. It accrues simply upon payment by the insurance company of the insurance claim. The doctrine of subrogation has its roots in equity. It is designed to promote and to accomplish justice; and is the mode that equity adopts to compel the ultimate payment of a debt by one who, in justice, equity, and good conscience, ought to pay.[33] Considering the above ruling, it is only but proper that Malayan Insurance be subrogated to the rights of the assured. WHEREFORE, the petition is hereby GRANTED. The CAs July 28, 2010 Decision and October 29, 2010 Resolution in CA-G.R. CV No. 93112 are herebyREVERSED and SET ASIDE. The Decision dated February 2, 2009 issued by the trial court in Civil Case No. 99-95885 is hereby REINSTATED. No pronouncement as to cost. SO ORDERED.

PRESBITERO J. VELASCO, JR. Associate Justice WE CONCUR:

DIOSDADO M. PERALTA Associate Justice

JOSE CATRAL MENDOZA Associate Justice

BIENVENIDO L. REYES Associate Justice

ESTELA M. PERLAS-BERNABE Associate Justice

ATTESTATION I attest that the conclusions in the above Decision had been reached in consultation before the case was assigned to the writer of the opinion of the Courts Division.

PRESBITERO J. VELASCO, JR. Associate Justice Chairperson

G.R. No. 173773

November 28, 2012

PARAMOUNT INSURANCE CORPORATION, Petitioner, vs. SPOUSES YVES and MARIA TERESA REMONDEULAZ, Respondents. DECISION PERALTA, J.: Before us is a Petition for Review on Certiorari under Rule 45 of the Rules of Court seeking the reversal and setting aside of the Decision1 dated April 12, 2005 and Resolution2 dated July 20, 2006 of the Court of Appeals in CA-G.R. CV No. 61490. The undisputed facts follow. On May 26, 1994, respondents insured with petitioner their 1994 Toyota Corolla sedan under a comprehensive motor vehicle insurance policy for one year. During the effectivity of said insurance, respondents car was unlawfully taken. Hence, they immediately reported the theft to the Traffic Management Command of the PNP who made them accomplish a complaint sheet. In said complaint sheet, respondents alleged that a certain Ricardo Sales (Sales) took possession of the subject vehicle to add accessories and improvements thereon, however, Sales failed to return the subject vehicle within the agreed three-day period. As a result, respondents notified petitioner to claim for the reimbursement of their lost vehicle. However, petitioner refused to pay. Accordingly, respondents lodged a complaint for a sum of money against petitioner before the Regional Trial Court of Makati City (trial court) praying for the payment of the insured value of their car plus damages on April 21, 1995. After presentation of respondents evidence, petitioner filed a Demurrer to Evidence. Acting thereon, the trial court dismissed the complaint filed by respondents. The full text of said Order3 reads: Before the Court is an action filed by the plaintiffs, spouses Yves and Maria Teresa Remondeulaz against the defendant, Paramount Insurance Corporation, to recover from the defendant the insured value of the motor vehicle. It appears that on 26 May 1994, plaintiffs insured their vehicle, a 1994 Toyota Corolla XL with chassis number EE-100-9524505, with defendant under Private Car Policy No.

PC-37396 for Own Damage, Theft, Third-Party Property Damage and Third-Party Personal Injury, for the period commencing 26 May 1994 to 26 May 1995. Then on 1 December 1994, defendants received from plaintiff a demand letter asking for the payment of the proceeds in the amount of PhP409,000.00 under their policy. They alleged the loss of the vehicle and claimed the same to be covered by the policys provision on "Theft." Defendant disagreed and refused to pay. It appears, however, that plaintiff had successfully prosecuted and had been awarded the amount claimed in this action, in another action (Civil Case No. 95-1524 entitled Sps. Yves and Maria Teresa Remondeulaz versus Standard Insurance Company, Inc.), which involved the loss of the same vehicle under the same circumstances although under a different policy and insurance company. This, considered with the principle that an insured may not recover more than its interest in any property subject of an insurance, leads the court to dismiss this action. SO ORDERED.4 Not in conformity with the trial courts Order, respondents interposed an appeal to the Court of Appeals (appellate court). In its Decision dated April 12, 2005, the appellate court reversed and set aside the Order issued by the trial court, to wit: Indeed, the trial court erred when it dismissed the action on the ground of double recovery since it is clear that the subject car is different from the one insured with another insurance company, the Standard Insurance Company. In this case, defendantappellee herein petitioner denied the reimbursement for the lost vehicle on the ground that the said loss could not fall within the concept of the "theft clause" under the insurance policy x x x xxxx WHEREFORE, the October 7, 1998 Order of the Regional Trial Court of Makati City, Branch 63, is hereby REVERSED and SET ASIDE x x x. SO ORDERED.5 Petitioner, thereafter, filed a motion for reconsideration against said Decision, but the same was denied by the appellate court in a Resolution dated July 20, 2006. Consequently, petitioner filed a petition for review on certiorari before this Court praying that the appellate courts Decision and Resolution be reversed and set aside. In its petition, petitioner raises this issue for our resolution:

Whether or not the Court of Appeals decided the case a quo in a way not in accord with law and/or applicable jurisprudence when it promulgated in favor of the respondents Remondeulaz, making Paramount liable for the alleged "theft" of respondents vehicle. 6 Essentially, the issue is whether or not petitioner is liable under the insurance policy for the loss of respondents vehicle. Petitioner argues that the loss of respondents vehicle is not a peril covered by the policy. It maintains that it is not liable for the loss, since the car cannot be classified as stolen as respondents entrusted the possession thereof to another person. We do not agree. Adverse to petitioners claim, respondents policy clearly undertook to indemnify t he insured against loss of or damage to the scheduled vehicle when caused by theft, to wit: SECTION III LOSS OR DAMAGE 1. The Company will, subject to the Limits of Liability, indemnify the insured against loss of or damage to the Scheduled Vehicle and its accessories and spare parts whilst thereon: (a) by accidental collision or overturning, or collision or overturning consequent upon mechanical breakdown or consequent upon wear and tear; (b) by fire, external explosion, self-ignition or lightning or burglary, housebreaking or theft; (c) by malicious act; (d) whilst in transit (including the process of loading and unloading) incidental to such transit by road, rail, inland waterway, lift or elevator.7 Apropos, we now resolve the issue of whether the loss of respondents vehicle falls within the concept of the "theft clause" under the insurance policy. In People v. Bustinera,8 this Court had the occasion to interpret the "theft clause" of an insurance policy. In this case, the Court explained that when one takes the motor vehicle of another without the latters consent even if the motor vehicle is later returned, there is theft there being intent to gain as the use of the thing unlawfully taken constitutes gain. Also, in Malayan Insurance Co., Inc. v. Court of Appeals,9 this Court held that the taking of a vehicle by another person without the permission or authority from the owner thereof is sufficient to place it within the ambit of the word theft as contemplated in the policy, and is therefore, compensable.

Moreover, the case of Santos v. People10 is worthy of note. Similarly in Santos, the owner of a car entrusted his vehicle to therein petitioner Lauro Santos who owns a repair shop for carburetor repair and repainting. However, when the owner tried to retrieve her car, she was not able to do so since Santos had abandoned his shop. In the said case, the crime that was actually committed was Qualified Theft. However, the Court held that because of the fact that it was not alleged in the information that the object of the crime was a car, which is a qualifying circumstance, the Court found that Santos was only guilty of the crime of Theft and merely considered the qualifying circumstance as an aggravating circumstance in the imposition of the appropriate penalty. The Court therein clarified the distinction between the crime of Estafa and Theft, to wit: x x x The principal distinction between the two crimes is that in theft the thing is taken while in estafa the accused receives the property and converts it to his own use or benefit. However, there may be theft even if the accused has possession of the property. If he was entrusted only with the material or physical (natural) or de facto possession of the thing, his misappropriation of the same constitutes theft, but if he has the juridical possession of the thing his conversion of the same constitutes embezzlement or estafa.11 In the instant case, Sales did not have juridical possession over the vehicle. Hence, it is apparent that the taking of repondents vehicle by Sales is without any c onsent or authority from the former. Records would show that respondents entrusted possession of their vehicle only to the extent that Sales will introduce repairs and improvements thereon, and not to permanently deprive them of possession thereof. Since, Theft can also be committed through misappropriation, the fact that Sales failed to return the subject vehicle to respondents constitutes Qualified Theft. Hence, since repondents car is undeniably covered by a Comprehensive Motor Vehicle Insurance Policy that allows for recovery in cases of theft, petitioner is liable under the policy for the loss of respondents vehicle under the "theft clause." All told, Sales act of depriving respondents of their motor vehicle at, or soon after the transfer of physical possession of the movable property, constitutes theft under the insurance policy, which is compensable.12 WHEREFORE, the instant petition is DENIED. The Decision dated April 12, 2005 and Resolution dated July 20, 2006 of the Court of Appeals are hereby AFFIRMED in toto. SO ORDERED. DIOSDADO M. PERALTA Associate Justice

G.R. No. 147724

June 8, 2004

LORENZO SHIPPING CORP., petitioner, vs. CHUBB and SONS, Inc., GEARBULK, Ltd. and PHILIPPINE TRANSMARINE CARRIERS, INC., respondents. DECISION PUNO, J.: On appeal is the Court of Appeals August 14, 2000 Decision1 in CA-G.R. CV No. 61334 and March 28, 2001 Resolution2 affirming the March 19, 1998 Decision3 of the Regional Trial Court of Manila which found petitioner liable to pay respondent Chubb and Sons, Inc. attorney's fees and costs of suit. Petitioner Lorenzo Shipping Corporation (Lorenzo Shipping, for short), a domestic corporation engaged in coastwise shipping, was the carrier of 581 bundles of black steel pipes, the subject shipment, from Manila to Davao City. From Davao City, respondent Gearbulk, Ltd., a foreign corporation licensed as a common carrier under the laws of Norway and doing business in the Philippines through its agent, respondent Philippine Transmarine Carriers, Inc. (Transmarine Carriers, for short), a domestic corporation, carried the goods on board its vessel M/V San Mateo Victory to the United States, for the account of Sumitomo Corporation. The latter, the consignee, is a foreign corporation organized under the laws of the United States of America. It insured the shipment with respondent Chubb and Sons, Inc., a foreign corporation organized and licensed to engage in insurance business under the laws of the United States of America. The facts are as follows: On November 21, 1987, Mayer Steel Pipe Corporation of Binondo, Manila, loaded 581 bundles of ERW black steel pipes worth US$137,912.844 on board the vessel M/V Lorcon IV, owned by petitioner Lorenzo Shipping, for shipment to Davao City. Petitioner Lorenzo Shipping issued a clean bill of lading designated as Bill of Lading No. T-35 for the account of the consignee, Sumitomo Corporation of San Francisco, California, USA, which in turn, insured the goods with respondent Chubb and Sons, Inc.6 The M/V Lorcon IV arrived at the Sasa Wharf in Davao City on December 2, 1987. Respondent Transmarine Carriers received the subject shipment which was discharged on December 4, 1987, evidenced by Delivery CargoReceipt No. 115090.7 It discovered seawater in the hatch of M/V Lorcon IV, and found the steel pipes submerged in it. The consignee Sumitomo then hired the services of R.J. Del Pan Surveyors to inspect the shipment prior to and subsequent to discharge. Del Pans Survey Report8 dated December 4, 1987 showed that the subject shipment was no longer in good condition, as in fact, the pipes were found with rust formation on top and/or at the sides. Moreover,

the surveyor noted that the cargo hold of the M/V Lorcon IV was flooded with seawater, and the tank top was "rusty, thinning, and with several holes at different places." The rusty condition of the cargo was noted on the mates receipts and the checker of M/V Lorcon IV signed his conforme thereon.9 After the survey, respondent Gearbulk loaded the shipment on board its vessel M/V San Mateo Victory, for carriage to the United States. It issued Bills of Lading Nos. DAV/OAK 1 to 7,10 covering 364 bundles of steel pipes to be discharged at Oakland, U.S.A., and Bills of Lading Nos. DAV/SEA 1 to 6,11 covering 217 bundles of steel pipes to be discharged at Vancouver, Washington, U.S.A. All bills of lading were marked "ALL UNITS HEAVILY RUSTED." While the cargo was in transit from Davao City to the U.S.A., consignee Sumitomo sent a letter12 of intent dated December 7, 1987, to petitioner Lorenzo Shipping, which the latter received on December 9, 1987. Sumitomo informed petitioner Lorenzo Shipping that it will be filing a claim based on the damaged cargo once such damage had been ascertained. The letter reads: Please be advised that the merchandise herein below noted has been landed in bad order ex-Manila voyage No. 87-19 under B/L No. T-3 which arrived at the port of Davao City on December 2, 1987. The extent of the loss and/or damage has not yet been determined but apparently all bundles are corroded. We reserve the right to claim as soon as the amount of claim is determined and the necessary supporting documents are available. Please find herewith a copy of the survey report which we had arranged for after unloading of our cargo from your vessel in Davao. We trust that you shall make everything in order. On January 17, 1988, M/V San Mateo Victory arrived at Oakland, California, U.S.A., where it unloaded 364 bundles of the subject steel pipes. It then sailed to Vancouver, Washington on January 23, 1988 where it unloaded the remaining 217 bundles. Toplis and Harding, Inc. of San Franciso, California, surveyed the steel pipes, and also discovered the latter heavily rusted. When the steel pipes were tested with a silver nitrate solution, Toplis and Harding found that they had come in contact with salt water. The survey report,13 dated January 28, 1988 states: xxx We entered the hold for a close examination of the pipe, which revealed moderate to heavy amounts of patchy and streaked dark red/orange rust on all lifts which were visible. Samples of the shipment were tested with a solution of silver nitrate revealing both positive and occasional negative chloride reactions, indicating pipe had come in contact with salt water. In addition, all tension applied

metal straps were very heavily rusted, and also exhibited chloride reactions on testing with silver nitrate. xxx It should be noted that subject bills of lading bore the following remarks as to conditions of goods: "ALL UNITS HEAVILY RUSTED." Attached herein is a copy of a survey report issued by Del Pan Surveyors of Davao City, Philippines dated, December 4, 1987 at Davao City, Philippines, which describes conditions of the cargo as sighted aboard the vessel "LORCON IV," prior to and subsequent to discharge at Davao City. Evidently, the aforementioned rust damages were apparently sustained while the shipment was in the custody of the vessel "LORCON IV," prior to being laden on board the vessel "SAN MATEO VICTORY" in Davao. Due to its heavily rusted condition, the consignee Sumitomo rejected the damaged steel pipes and declared them unfit for the purpose they were intended.14 It then filed a marine insurance claim with respondent Chubb and Sons, Inc. which the latter settled in the amount of US$104,151.00. 15 On December 2, 1988, respondent Chubb and Sons, Inc. filed a complaint 16 for collection of a sum of money, docketed as Civil Case No. 88-47096, against respondents Lorenzo Shipping, Gearbulk, and Transmarine. Respondent Chubb and Sons, Inc. alleged that it is not doing business in the Philippines, and that it is suing under an isolated transaction. On February 21, 1989, respondents Gearbulk and Transmarine filed their answer17 with counterclaim and cross-claim against petitioner Lorenzo Shipping denying liability on the following grounds: (a) respondent Chubb and Sons, Inc. has no capacity to sue before Philippine courts; (b) the action should be dismissed on the ground offorum non conveniens; (c) damage to the steel pipes was due to the inherent nature of the goods or to the insufficiency of packing thereof; (d) damage to the steel pipes was not due to their fault or negligence; and, (e) the law of the country of destination, U.S.A., governs the contract of carriage. Petitioner Lorenzo Shipping filed its answer with counterclaim on February 28, 1989, and amended it on May 24, 1989. It denied liability, alleging, among others: (a) that rust easily forms on steel by mere exposure to air, moisture and other marine elements; (b) that it made a disclaimer in the bill of lading; (c) that the goods were improperly packed; and, (d) prescription, laches, and extinguishment of obligations and actions had set in. The Regional Trial Court ruled in favor of the respondent Chubb and Sons, Inc., finding that: (1) respondent Chubb and Sons, Inc. has the right to institute this action; and, (2) petitioner Lorenzo Shipping was negligent in the performance of its obligations as a carrier. The dispositive portion of its Decision states:

WHEREFORE, the judgment is hereby rendered ordering Defendant Lorenzo Shipping Corporation to pay the plaintiff the sum of US$104,151.00 or its equivalent in Philippine peso at the current rate of exchange with interest thereon at the legal rate from the date of the institution of this case until fully paid, the attorneys fees in the sum of P50,000.00, plus the costs of the suit, and dismissing the plaintiffs complaint against defendants Gearbulk, Ltd. and Philippine Transmarine Carriers, Inc., for lack of merit, and the two defendants counterclaim, there being no showing that the plaintiff had filed this case against said defendants in bad faith, as well as the two defendants cross -claim against Defendant Lorenzo Shipping Corporation, for lack of factual basis. 18 Petitioner Lorenzo Shipping appealed to the Court of Appeals insisting that: (a) respondent Chubb and Sons does not have capacity to sue before Philippine courts; and, (b) petitioner Lorenzo Shipping was not negligent in the performance of its obligations as carrier of the goods. The appellate court denied the petition and affirmed the decision of the trial court. The Court of Appeals likewise denied petitioner Lorenzo Shippings Motio n for Reconsideration19 dated September 3, 2000, in a Resolution20 promulgated on March 28, 2001. Hence, this petition. Petitioner Lorenzo Shipping submits the following issues for resolution: (1) Whether or not the prohibition provided under Art. 133 of the Corporation Code applies to respondent Chubb, it being a mere subrogee or assignee of the rights of Sumitomo Corporation, likewise a foreign corporation admittedly doing business in the Philippines without a license; (2) Whether or not Sumitomo, Chubbs predecessor-in-interest, validly made a claim for damages against Lorenzo Shipping within the period prescribed by the Code of Commerce; (3) Whether or not a delivery cargo receipt without a notation on it of damages or defects in the shipment, which created a prima facie presumption that the carrier received the shipment in good condition, has been overcome by convincing evidence; (4) Assuming that Lorenzo Shipping was guilty of some lapses in transporting the steel pipes, whether or not Gearbulk and Transmarine, as common carriers, are to share liability for their separate negligence in handling the cargo. 21 In brief, we resolve the following issues: (1) whether respondent Chubb and Sons has capacity to sue before the Philippine courts; and,

(2) whether petitioner Lorenzo Shipping is negligent in carrying the subject cargo. Petitioner argues that respondent Chubb and Sons is a foreign corporation not licensed to do business in the Philippines, and is not suing on an isolated transaction. It contends that because the respondent Chubb and Sons is an insurance company, it was merely subrogated to the rights of its insured, the consignee Sumitomo, after paying the latters policy claim. Sumitomo, however, is a foreign corporation doing business in the Philippines without a license and does not have capacity to sue before Philippine courts. Since Sumitomo does not have capacity to sue, petitioner then concludes that, neither the subrogee-respondent Chubb and Sons could sue before Philippine courts. We disagree with petitioner. In the first place, petitioner failed to raise the defense that Sumitomo is a foreign corporation doing business in the Philippines without a license. It is therefore estopped from litigating the issue on appeal especially because it involves a question of fact which this Court cannot resolve. Secondly, assuming arguendo that Sumitomo cannot sue in the Philippines, it does not follow that respondent, as subrogee, has also no capacity to sue in our jurisdiction. Subrogation is the substitution of one person in the place of another with reference to a lawful claim or right, so that he who is substituted succeeds to the rights of the other in relation to a debt or claim, including its remedies or securities.22 The principle covers the situation under which an insurer that has paid a loss under an insurance policy is entitled to all the rights and remedies belonging to the insured against a third party with respect to any loss covered by the policy.23 It contemplates full substitution such that it places the party subrogated in the shoes of the creditor, and he may use all means which the creditor could employ to enforce payment.24 The rights to which the subrogee succeeds are the same as, but not greater than, those of the person for whom he is substituted he cannot acquire any claim, security, or remedy the subrogor did not have.25 In other words, a subrogee cannot succeed to a right not possessed by the subrogor.26 A subrogee in effect steps into the shoes of the insured and can recover only if insured likewise could have recovered. However, when the insurer succeeds to the rights of the insured, he does so only in relation to the debt. The person substituted (the insurer) will succeed to all the rights of the creditor (the insured), having reference to the debt due the latter.27 In the instant case, the rights inherited by the insurer, respondent Chubb and Sons, pertain only to the payment it made to the insured Sumitomo as stipulated in the insurance contract between them, and which amount it now seeks to recover from petitioner Lorenzo Shipping which caused the loss sustained by the insured Sumitomo. The capacity to sue of respondent Chubb and Sons could not perchance belong to the group of rights, remedies or securities pertaining to the payment respondent insurer made for the loss which was sustained by the insured Sumitomo and covered by the contract of insurance. Capacity to sue is a right personal to its holder. It is conferred by law and not

by the parties. Lack of legal capacity to sue means that the plaintiff is not in the exercise of his civil rights, or does not have the necessary qualification to appear in the case, or does not have the character or representation he claims. It refers to a plaintiffs general disability to sue, such as on account of minority, insanity, incompetence, lack of juridical personality, or any other disqualifications of a party.28Respondent Chubb and Sons who was plaintiff in the trial court does not possess any of these disabilities. On the contrary, respondent Chubb and Sons has satisfactorily proven its capacity to sue, after having shown that it is not doing business in the Philippines, but is suing only under an isolated transaction, i.e., under the one (1) marine insurance policy issued in favor of the consignee Sumitomo covering the damaged steel pipes. The law on corporations is clear in depriving foreign corporations which are doing business in the Philippines without a license from bringing or maintaining actions before, or intervening in Philippine courts. Art. 133 of the Corporation Code states: Doing business without a license. No foreign corporation transacting business in the Philippines without a license, or its successors or assigns, shall be permitted to maintain or intervene in any action, suit or proceeding in any court or administrative agency of the Philippines; but such corporation may be sued or proceeded against before Philippine courts or administrative tribunals on any valid cause of action recognized under Philippine laws. The law does not prohibit foreign corporations from performing single acts of business. A foreign corporation needs no license to sue before Philippine courts on an isolated transaction.29 As held by this Court in the case ofMarshall-Wells Company vs. Elser & Company:30 The object of the statute (Secs. 68 and 69, Corporation Law) was not to prevent the foreign corporation from performing single acts, but to prevent it from acquiring a domicile for the purpose of business without taking the steps necessary to render it amenable to suit in the local courts . . . the implication of the law (being) that it was never the purpose of the legislature to exclude a foreign corporation which happens to obtain an isolated order for business for the Philippines, from seeking redress in the Philippine courts. Likewise, this Court ruled in Universal Shipping Lines, Inc. vs. Intermediate Appellate Court31 that: . . . The private respondent may sue in the Philippine courts upon the marine insurance policies issued by it abroad to cover international-bound cargoes shipped by a Philippine carrier, even if it has no license to do business in this country, for it is not the lack of the prescribed license (to do business in the Philippines) but doing business without such license, which bars a foreign corporation from access to our courts.

We reject the claim of petitioner Lorenzo Shipping that respondent Chubb and Sons is not suing under an isolated transaction because the steel pipes, subject of this case, are covered by two (2) bills of lading; hence, two transactions. The stubborn fact remains that these two (2) bills of lading spawned from the single marine insurance policy that respondent Chubb and Sons issued in favor of the consignee Sumitomo, covering the damaged steel pipes. The execution of the policy is a single act, an isolated transaction. This Court has not construed the term "isolated transaction" to literally mean "one" or a mere single act. In Eriks Pte. Ltd. vs. Court of Appeals, this Court held that:32 . . . What is determinative of "doing business" is not really the number or the quantity of the transactions, but more importantly, the intention of an entity to continue the body of its business in the country. The number and quantity are merely evidence of such intention. The phrase "isolated transaction" has a definite and fixed meaning, i.e. a transaction or series of transactions set apart from the common business of a foreign enterprise in the sense that there is no intention to engage in a progressive pursuit of the purpose and object of the business organization. Whether a foreign corporation is "doing business" does not necessarily depend upon the frequency of its transactions, but more upon the nature and character of the transactions. [Emphasis supplied.] In the case of Gonzales vs. Raquiza, et al.,33 three contracts, hence three transactions were challenged as void on the ground that the three American corporations which are parties to the contracts are not licensed to do business in the Philippines. This Court held that "one single or isolated business transaction does not constitutedoing business within the meaning of the law. Transactions which are occasional, incidental, and casual not of a character to indicate a purpose to engage in business do not constitute the doing or engaging in business as contemplated by law. Where the three transactions indicate no intent by the foreign corporation to engage in a continuity of transactions, they do not constitute doing business in the Philippines." Furthermore, respondent insurer Chubb and Sons, by virtue of the right of subrogation provided for in the policy of insurance,34 is the real party in interest in the action for damages before the court a quo against the carrier Lorenzo Shipping to recover for the loss sustained by its insured. Rule 3, Section 2 of the 1997 Rules of Civil Procedure defines a real party in interest as one who is entitled to the avails of any judgment rendered in a suit, or who stands to be benefited or injured by it. Where an insurance company as subrogee pays the insured of the entire loss it suffered, the insurersubrogee is the only real party in interest and must sue in its own name 35 to enforce its right of subrogation against the third party which caused the loss. This is because the insurer in such case having fully compensated its insured, which payment covers the loss in full, is subrogated to the insureds claims arising from such loss. The subrogated insurer becomes the owner of the claim and, thus entitled to the entire fruits of the action.36 It then, thus possesses the right to enforce the claim and the significant interest in the litigation.37 In the case at bar, it is clear that respondent insurer was suing on its own behalf in order to enforce its right of subrogation.

On the second issue, we affirm the findings of the lower courts that petitioner Lorenzo Shipping was negligent in its care and custody of the consignees goods. The steel pipes, subject of this case, were in good condition when they were loaded at the port of origin (Manila) on board petitioner Lorenzo Shippings M/V Lorcon IV en route to Davao City. Petitioner Lorenzo Shipping issued clean bills of lading covering the subject shipment. A bill of lading, aside from being a contract38 and a receipt,39is also a symbol40 of the goods covered by it. A bill of lading which has no notation of any defect or damage in the goods is called a "clean bill of lading." 41 A clean bill of lading constitutes prima facie evidence of the receipt by the carrier of the goods as therein described.42 The case law teaches us that mere proof of delivery of goods in good order to a carrier and the subsequent arrival in damaged condition at the place of destination raises a prima facie case against the carrier.43 In the case at bar, M/V Lorcon IV of petitioner Lorenzo Shipping received the steel pipes in good order and condition, evidenced by the clean bills of lading it issued. When the cargo was unloaded from petitioner Lorenzo Shippings vessel at the Sasa Wharf in Davao City, the steel pipes were rusted all over. M/V San Mateo Victory of respondent Gearbulk, Ltd, which received the cargo, issued Bills of Lading Nos. DAV/OAK 1 to 7 and Nos. DAV/SEA 1 to 6 covering the entire shipment, all of which were marked "ALL UNITS HEAVILY RUSTED." R.J. Del Pan Surveyors found that the cargo hold of the M/V Lorcon IV was flooded with seawater, and the tank top was rusty, thinning and perforated, thereby exposing the cargo to sea water. There can be no other conclusion than that the cargo was damaged while on board the vessel of petitioner Lorenzo Shipping, and that the damage was due to the latters negligence. In the case at bar, not only did the legal presumption of negligence attach to petitioner Lorenzo Shipping upon the occurrence of damage to the cargo.44 More so, the negligence of petitioner was sufficiently established. Petitioner Lorenzo Shipping failed to keep its vessel in seaworthy condition. R.J. Del Pan Surveyors found the tank top of M/V Lorcon IV to be "rusty, thinning, and with several holes at different places." Witness Captain Pablo Fernan, Operations Manager of respondent Transmarine Carriers, likewise observed the presence of holes at the deck of M/V Lorcon IV.45 The unpatched holes allowed seawater, reaching up to three (3) inches deep, to enter the flooring of the hatch of the vessel where the steel pipes were stowed, submerging the latter in sea water.46 The contact with sea water caused the steel pipes to rust. The silver nitrate test, which Toplis and Harding employed, further verified this conclusion.47 Significantly, petitioner Lorenzo Shipping did not even attempt to present any contrary evidence. Neither did it offer any proof to establish any of the causes that would exempt it from liability for such damage.48 It merely alleged that the: (1) packaging of the goods was defective; and (2) claim for damages has prescribed. To be sure, there is evidence that the goods were packed in a superior condition. John M. Graff, marine surveyor of Toplis and Harding, examined the condition of the cargo on board the vessel San Mateo Victory. He testified that the shipment had superior packing "because the ends were covered with plastic, woven plastic. Whereas typically they

would not go to that bother ... Typically, they come in with no plastic on the ends. They might just be banded, no plastic on the ends ..." 49 On the issue of prescription of respondent Chubb and Sons claim for damages, we rule that it has not yet prescribed at the time it was made. Art. 366 of the Code of Commerce states: Within the twenty-four hours following the receipt of the merchandise, the claim against the carrier for damage or average, which may be found therein upon the opening of the packages, may be made, provided that the indications of the damage or average which gives rise to the claim cannot be ascertained from the outside part of such package, in which case the claim shall be admitted only at the time of the receipt. After the periods mentioned have elapsed, or transportation charges have been paid, no claim shall be admitted against the carrier with regard to the condition in which the goods transported were delivered. A somewhat similar provision is embodied in the Bill of Lading No. T-3 which reads:50 NOTE: No claim for damage or loss shall be honored twenty-four (24) hours after delivery. (Ref. Art. 366 C Com.) The twenty-four-hour period prescribed by Art. 366 of the Code of Commerce within which claims must be presented does not begin to run until the consignee has received such possession of the merchandise that he may exercise over it the ordinary control pertinent to ownership.51 In other words, there must be delivery of the cargo by the carrier to the consignee at the place of destination.52 In the case at bar, consignee Sumitomo has not received possession of the cargo, and has not physically inspected the same at the time the shipment was discharged from M/V Lorcon IV in Davao City. Petitioner Lorenzo Shipping failed to establish that an authorized agent of the consignee Sumitomo received the cargo at Sasa Wharf in Davao City. Respondent Transmarine Carriers as agent of respondent Gearbulk, Ltd., which carried the goods from Davao City to the United States, and the principal, respondent Gearbulk, Ltd. itself, are not the authorized agents as contemplated by law. What is clear from the evidence is that the consignee received and took possession of the entire shipment only when the latter reached the United States shore. Only then was delivery made and completed. And only then did the 24-hour prescriptive period start to run. Finally, we find no merit to the contention of respondents Gearbulk and Transmarine that American law governs the contract of carriage because the U.S.A. is the country of destination. Petitioner Lorenzo Shipping, through its M/V Lorcon IV, carried the goods

from Manila to Davao City. Thus, as against petitioner Lorenzo Shipping, the place of destination is Davao City. Hence, Philippine law applies. IN VIEW THEREOF, the petition is DENIED. The Decision of the Court of Appeals in CA-G.R. CV No. 61334 dated August 14, 2000 and its Resolution dated March 28, 2001 are hereby AFFIRMED. Costs against petitioner. SO ORDERED. Quisumbing, Austria-Martinez, Callejo, Sr., and Tinga, JJ., concur.

G.R. No. L-52756 October 12, 1987 MANILA MAHOGANY MANUFACTURING CORPORATION, petitioner, vs. COURT OF APPEALS AND ZENITH INSURANCE CORPORATION, respondents.

PADILLA, J: Petition to review the decision * of the Court of Appeals, in CA-G.R. No. SP-08642, dated 21 March 1979, ordering petitioner Manila Mahogany Manufacturing Corporation to pay private respondent Zenith Insurance Corporation the sum of Five Thousand Pesos (P5,000.00) with 6% annual interest from 18 January 1973, attorney's fees in the sum of five hundred pesos (P500.00), and costs of suit, and the resolution of the same Court, dated 8 February 1980, denying petitioner's motion for reconsideration of it's decision. From 6 March 1970 to 6 March 1971, petitioner insured its Mercedes Benz 4-door sedan with respondentinsurance company. On 4 May 1970 the insured vehicle was bumped and damaged by a truck owned by San Miguel Corporation. For the damage caused, respondent company paid petitioner five thousand pesos (P5,000.00) in amicable settlement. Petitioner's general manager executed a Release of Claim, subrogating respondent company to all its right to action against San Miguel Corporation. On 11 December 1972, respondent company wrote Insurance Adjusters, Inc. to demand reimbursement from San Miguel Corporation of the amount it had paid petitioner. Insurance Adjusters, Inc. refused reimbursement, alleging that San Miguel Corporation had already paid petitioner P4,500.00 for the damages to petitioner's motor vehicle, as evidenced by a cash voucher and a Release of Claim executed by the General Manager of petitioner discharging San Miguel Corporation from "all actions, claims, demands the rights of action that now exist or hereafter [sic] develop arising out of or as a consequence of the accident." Respondent insurance company thus demanded from petitioner reimbursement of the sum of P4,500.00 paid by San Miguel Corporation. Petitioner refused; hence, respondent company filed suit in the City Court of Manila for the recovery of P4,500.00. The City Court ordered petitioner to pay respondent P4,500.00. On appeal the Court of First Instance of Manila affirmed the City Court's decision in toto, which CFI decision was affirmed by the Court of Appeals, with the modification that petitioner was to pay respondent the total amount of P5,000.00 that it had earlier received from the respondent insurance company. Petitioner now contends it is not bound to pay P4,500.00, and much more, P5,000.00 to respondent company as the subrogation in the Release of Claim it executed in favor of

respondent was conditioned on recovery of the total amount of damages petitioner had sustained. Since total damages were valued by petitioner at P9,486.43 and only P5,000.00 was received by petitioner from respondent, petitioner argues that it was entitled to go after San Miguel Corporation to claim the additional P4,500.00 eventually paid to it by the latter, without having to turn over said amount to respondent. Respondent of course disputes this allegation and states that there was no qualification to its right of subrogation under the Release of Claim executed by petitioner, the contents of said deed having expressed all the intents and purposes of the parties. To support its alleged right not to return the P4,500.00 paid by San Miguel Corporation, petitioner cites Art. 2207 of the Civil Code, which states: If the plaintiff's property has been insured, and he has received indemnity from the insurance company for the injury or loss arising out of the wrong or breach of contract complained of the insurance company shall be subrogated to the rights of the insured against the wrongdoer or the person who has violated the contract. If the amount paid by the insurance company does not fully cover the injury or loss the aggrieved party shall be entitled to recover the deficiency from the person causing the loss or injury. Petitioner also invokes Art. 1304 of the Civil Code, stating. A creditor, to whom partial payment has been made, may exercise his right for the remainder, and he shall be preferred to the person who has been subrogated in his place in virtue of the partial payment of the same credit. We find petitioners arguments to be untenable and without merit. In the absence of any other evidence to support its allegation that a gentlemen's agreement existed between it and respondent, not embodied in the Release of Claim, such ease of Claim must be taken as the best evidence of the intent and purpose of the parties. Thus, the Court of Appeals rightly stated: Petitioner argues that the release claim it executed subrogating Private respondent to any right of action it had against San Miguel Corporation did not preclude Manila Mahogany from filing a deficiency claim against the wrongdoer. Citing Article 2207, New Civil Code, to the effect that if the amount paid by an insurance company does not fully cover the loss, the aggrieved party shall be entitled to recover the deficiency from the person causing the loss, petitioner claims a preferred right to retain the amount coming from San Miguel Corporation, despite the subrogation in favor of Private respondent. Although petitioners right to file a deficiency claim against San Miguel Corporation is with legal basis, without prejudice to the insurer's right of

subrogation, nevertheless when Manila Mahogany executed another release claim (Exhibit K) discharging San Miguel Corporation from "all actions, claims, demands and rights of action that now exist or hereafter arising out of or as a consequence of the accident" after the insurer had paid the proceeds of the policy- the compromise agreement of P5,000.00 being based on the insurance policy-the insurer is entitled to recover from the insured the amount of insurance money paid (Metropolitan Casualty Insurance Company of New York vs. Badler, 229 N.Y.S. 61, 132 Misc. 132 cited in Insurance Code and Insolvency Law with comments and annotations, H.B. Perez 1976, p. 151). Since petitioner by its own acts released San Miguel Corporation, thereby defeating private respondents, the right of subrogation, the right of action of petitioner against the insurer was also nullified. (Sy Keng & Co. vs. Queensland Insurance Co., Ltd., 54 O.G. 391) Otherwise stated: private respondent may recover the sum of P5,000.00 it had earlier paid to petitioner. 1 As held in Phil. Air Lines v. Heald Lumber Co., 2 If a property is insured and the owner receives the indemnity from the insurer, it is provided in [Article 2207 of the New Civil Code] that the insurer is deemed subrogated to the rights of the insured against the wrongdoer and if the amount paid by the insurer does not fully cover the loss, then the aggrieved party is the one entitled to recover the deficiency. ... Under this legal provision, the real party in interest with regard to the portion of the indemnity paid is the insurer and not the insured 3 (Emphasis supplied) The decision of the respondent court ordering petitioner to pay respondent company, not the P4,500.00 as originally asked for, but P5,000.00, the amount respondent company paid petitioner as insurance, is also in accord with law and jurisprudence. In disposing of this issue, the Court of Appeals held: ... petitioner is entitled to keep the sum of P4,500.00 paid by San Miguel Corporation under its clear right to file a deficiency claim for damages incurred, against the wrongdoer, should the insurance company not fully pay for the injury caused (Article 2207, New Civil Code). However, when petitioner released San Miguel Corporation from any liability, petitioner's right to retain the sum of P5,000.00 no longer existed, thereby entitling private respondent to recover the same. (Emphasis supplied) As has been observed: ... The right of subrogation can only exist after the insurer has paid the otherwise the insured will be deprived of his right to full indemnity. If the insurance proceeds are not sufficient to cover the damages suffered by the insured, then he may sue the party responsible for the damage for the

the [sic] remainder. To the extent of the amount he has already received from the insurer enjoy's [sic] the right of subrogation. Since the insurer can be subrogated to only such rights as the insured may have, should the insured, after receiving payment from the insurer, release the wrongdoer who caused the loss, the insurer loses his rights against the latter. But in such a case, the insurer will be entitled to recover from the insured whatever it has paid to the latter, unless the release was made with the consent of the insurer. 4 (Emphasis supplied.) And even if the specific amount asked for in the complaint is P4,500.00 only and not P5,000.00, still, the respondent Court acted well within its discretion in awarding P5,000.00, the total amount paid by the insurer. The Court of Appeals rightly reasoned as follows: It is to be noted that private respondent, in its companies, prays for the recovery, not of P5,000.00 it had paid under the insurance policy but P4,500.00 San Miguel Corporation had paid to petitioner. On this score, We believe the City Court and Court of First Instance erred in not awarding the proper relief. Although private respondent prays for the reimbursement of P4,500.00 paid by San Miguel Corporation, instead of P5,000.00 paid under the insurance policy, the trial court should have awarded the latter, although not prayed for, under the general prayer in the complaint "for such further or other relief as may be deemed just or equitable, (Rule 6, Sec. 3, Revised Rules of Court; Rosales vs. Reyes Ordoveza, 25 Phil. 495 ; Cabigao vs. Lim, 50 Phil. 844; Baguiro vs. Barrios Tupas, 77 Phil 120). WHEREFORE, premises considered, the petition is DENIED. The judgment appealed from is hereby AFFIRMED with costs against petitioner. SO ORDERED.

G.R. No. L-52756 October 12, 1987 MANILA MAHOGANY MANUFACTURING CORPORATION, petitioner, vs. COURT OF APPEALS AND ZENITH INSURANCE CORPORATION, respondents.

PADILLA, J: Petition to review the decision * of the Court of Appeals, in CA-G.R. No. SP-08642, dated 21 March 1979, ordering petitioner Manila Mahogany Manufacturing Corporation to pay private respondent Zenith Insurance Corporation the sum of Five Thousand Pesos (P5,000.00) with 6% annual interest from 18 January 1973, attorney's fees in the sum of five hundred pesos (P500.00), and costs of suit, and the resolution of the same Court, dated 8 February 1980, denying petitioner's motion for reconsideration of it's decision. From 6 March 1970 to 6 March 1971, petitioner insured its Mercedes Benz 4-door sedan with respondentinsurance company. On 4 May 1970 the insured vehicle was bumped and damaged by a truck owned by San Miguel Corporation. For the damage caused, respondent company paid petitioner five thousand pesos (P5,000.00) in amicable settlement. Petitioner's general manager executed a Release of Claim, subrogating respondent company to all its right to action against San Miguel Corporation. On 11 December 1972, respondent company wrote Insurance Adjusters, Inc. to demand reimbursement from San Miguel Corporation of the amount it had paid petitioner. Insurance Adjusters, Inc. refused reimbursement, alleging that San Miguel Corporation had already paid petitioner P4,500.00 for the damages to petitioner's motor vehicle, as evidenced by a cash voucher and a Release of Claim executed by the General Manager of petitioner discharging San Miguel Corporation from "all actions, claims, demands the rights of action that now exist or hereafter [sic] develop arising out of or as a consequence of the accident." Respondent insurance company thus demanded from petitioner reimbursement of the sum of P4,500.00 paid by San Miguel Corporation. Petitioner refused; hence, respondent company filed suit in the City Court of Manila for the recovery of P4,500.00. The City Court ordered petitioner to pay respondent P4,500.00. On appeal the Court of First Instance of Manila affirmed the City Court's decision in toto, which CFI decision was affirmed by the Court of Appeals, with the modification that petitioner was to pay respondent the total amount of P5,000.00 that it had earlier received from the respondent insurance company. Petitioner now contends it is not bound to pay P4,500.00, and much more, P5,000.00 to respondent company as the subrogation in the Release of Claim it executed in favor of

respondent was conditioned on recovery of the total amount of damages petitioner had sustained. Since total damages were valued by petitioner at P9,486.43 and only P5,000.00 was received by petitioner from respondent, petitioner argues that it was entitled to go after San Miguel Corporation to claim the additional P4,500.00 eventually paid to it by the latter, without having to turn over said amount to respondent. Respondent of course disputes this allegation and states that there was no qualification to its right of subrogation under the Release of Claim executed by petitioner, the contents of said deed having expressed all the intents and purposes of the parties. To support its alleged right not to return the P4,500.00 paid by San Miguel Corporation, petitioner cites Art. 2207 of the Civil Code, which states: If the plaintiff's property has been insured, and he has received indemnity from the insurance company for the injury or loss arising out of the wrong or breach of contract complained of the insurance company shall be subrogated to the rights of the insured against the wrongdoer or the person who has violated the contract. If the amount paid by the insurance company does not fully cover the injury or loss the aggrieved party shall be entitled to recover the deficiency from the person causing the loss or injury. Petitioner also invokes Art. 1304 of the Civil Code, stating. A creditor, to whom partial payment has been made, may exercise his right for the remainder, and he shall be preferred to the person who has been subrogated in his place in virtue of the partial payment of the same credit. We find petitioners arguments to be untenable and without merit. In the absence of any other evidence to support its allegation that a gentlemen's agreement existed between it and respondent, not embodied in the Release of Claim, such ease of Claim must be taken as the best evidence of the intent and purpose of the parties. Thus, the Court of Appeals rightly stated: Petitioner argues that the release claim it executed subrogating Private respondent to any right of action it had against San Miguel Corporation did not preclude Manila Mahogany from filing a deficiency claim against the wrongdoer. Citing Article 2207, New Civil Code, to the effect that if the amount paid by an insurance company does not fully cover the loss, the aggrieved party shall be entitled to recover the deficiency from the person causing the loss, petitioner claims a preferred right to retain the amount coming from San Miguel Corporation, despite the subrogation in favor of Private respondent. Although petitioners right to file a deficiency claim against San Miguel Corporation is with legal basis, without prejudice to the insurer's right of

subrogation, nevertheless when Manila Mahogany executed another release claim (Exhibit K) discharging San Miguel Corporation from "all actions, claims, demands and rights of action that now exist or hereafter arising out of or as a consequence of the accident" after the insurer had paid the proceeds of the policy- the compromise agreement of P5,000.00 being based on the insurance policy-the insurer is entitled to recover from the insured the amount of insurance money paid (Metropolitan Casualty Insurance Company of New York vs. Badler, 229 N.Y.S. 61, 132 Misc. 132 cited in Insurance Code and Insolvency Law with comments and annotations, H.B. Perez 1976, p. 151). Since petitioner by its own acts released San Miguel Corporation, thereby defeating private respondents, the right of subrogation, the right of action of petitioner against the insurer was also nullified. (Sy Keng & Co. vs. Queensland Insurance Co., Ltd., 54 O.G. 391) Otherwise stated: private respondent may recover the sum of P5,000.00 it had earlier paid to petitioner. 1 As held in Phil. Air Lines v. Heald Lumber Co., 2 If a property is insured and the owner receives the indemnity from the insurer, it is provided in [Article 2207 of the New Civil Code] that the insurer is deemed subrogated to the rights of the insured against the wrongdoer and if the amount paid by the insurer does not fully cover the loss, then the aggrieved party is the one entitled to recover the deficiency. ... Under this legal provision, the real party in interest with regard to the portion of the indemnity paid is the insurer and not the insured 3 (Emphasis supplied) The decision of the respondent court ordering petitioner to pay respondent company, not the P4,500.00 as originally asked for, but P5,000.00, the amount respondent company paid petitioner as insurance, is also in accord with law and jurisprudence. In disposing of this issue, the Court of Appeals held: ... petitioner is entitled to keep the sum of P4,500.00 paid by San Miguel Corporation under its clear right to file a deficiency claim for damages incurred, against the wrongdoer, should the insurance company not fully pay for the injury caused (Article 2207, New Civil Code). However, when petitioner released San Miguel Corporation from any liability, petitioner's right to retain the sum of P5,000.00 no longer existed, thereby entitling private respondent to recover the same. (Emphasis supplied) As has been observed: ... The right of subrogation can only exist after the insurer has paid the otherwise the insured will be deprived of his right to full indemnity. If the insurance proceeds are not sufficient to cover the damages suffered by the insured, then he may sue the party responsible for the damage for the

the [sic] remainder. To the extent of the amount he has already received from the insurer enjoy's [sic] the right of subrogation. Since the insurer can be subrogated to only such rights as the insured may have, should the insured, after receiving payment from the insurer, release the wrongdoer who caused the loss, the insurer loses his rights against the latter. But in such a case, the insurer will be entitled to recover from the insured whatever it has paid to the latter, unless the release was made with the consent of the insurer. 4 (Emphasis supplied.) And even if the specific amount asked for in the complaint is P4,500.00 only and not P5,000.00, still, the respondent Court acted well within its discretion in awarding P5,000.00, the total amount paid by the insurer. The Court of Appeals rightly reasoned as follows: It is to be noted that private respondent, in its companies, prays for the recovery, not of P5,000.00 it had paid under the insurance policy but P4,500.00 San Miguel Corporation had paid to petitioner. On this score, We believe the City Court and Court of First Instance erred in not awarding the proper relief. Although private respondent prays for the reimbursement of P4,500.00 paid by San Miguel Corporation, instead of P5,000.00 paid under the insurance policy, the trial court should have awarded the latter, although not prayed for, under the general prayer in the complaint "for such further or other relief as may be deemed just or equitable, (Rule 6, Sec. 3, Revised Rules of Court; Rosales vs. Reyes Ordoveza, 25 Phil. 495 ; Cabigao vs. Lim, 50 Phil. 844; Baguiro vs. Barrios Tupas, 77 Phil 120). WHEREFORE, premises considered, the petition is DENIED. The judgment appealed from is hereby AFFIRMED with costs against petitioner. SO ORDERED.

G.R. No. 112360

July 18, 2000

RIZAL SURETY & INSURANCE COMPANY, petitioner, vs. COURT OF APPEALS and TRANSWORLD KNITTING MILLS, INC., respondents. DECISION PURISIMA, J.: At bar is a Petition for Review on Certiorari under Rule 45 of the Rules of Court seeking to annul and set aside the July 15, 1993 Decision1 and October 22, 1993 Resolution2 of the Court of Appeals3 in CA-G.R. CV NO. 28779, which modified the Ruling4 of the Regional Trial Court of Pasig, Branch 161, in Civil Case No. 46106. The antecedent facts that matter are as follows: On March 13, 1980, Rizal Surety & Insurance Company (Rizal Insurance) issued Fire Insurance Policy No. 45727 in favor of Transworld Knitting Mills, Inc. (Transworld), initially for One Million (P1,000,000.00) Pesos and eventually increased to One Million Five Hundred Thousand (P1,500,000.00) Pesos, covering the period from August 14, 1980 to March 13, 1981. Pertinent portions of subject policy on the buildings insured, and location thereof, read: "On stocks of finished and/or unfinished products, raw materials and supplies of every kind and description, the properties of the Insureds and/or held by them in trust, on commission or on joint account with others and/or for which they (sic) responsible in case of loss whilst contained and/or stored during the currency of this Policy in the premises occupied by them forming part of the buildings situate (sic) within own Compound at MAGDALO STREET, BARRIO UGONG, PASIG, METRO MANILA, PHILIPPINES, BLOCK NO. 601. xxx xxx xxx

Said building of four-span lofty one storey in height with mezzanine portions is constructed of reinforced concrete and hollow blocks and/or concrete under galvanized iron roof and occupied as hosiery mills, garment and lingerie factory, transistor-stereo assembly plant, offices, warehouse and caretaker's quarters. 'Bounds in front partly by one-storey concrete building under galvanized iron roof occupied as canteen and guardhouse, partly by building of two and partly one storey constructed of concrete below, timber above undergalvanized iron roof occupied as garage and quarters and partly by open space and/or tracking/ packing, beyond which is the aforementioned Magdalo Street; on its right and left by driveway, thence open spaces, and at the rear by open spaces.'"5

The same pieces of property insured with the petitioner were also insured with New India Assurance Company, Ltd., (New India). On January 12, 1981, fire broke out in the compound of Transworld, razing the middle portion of its four-span building and partly gutting the left and right sections thereof. A two-storey building (behind said four-span building) where fun and amusement machines and spare parts were stored, was also destroyed by the fire. Transworld filed its insurance claims with Rizal Surety & Insurance Company and New India Assurance Company but to no avail. On May 26, 1982, private respondent brought against the said insurance companies an action for collection of sum of money and damages, docketed as Civil Case No. 46106 before Branch 161 of the then Court of First Instance of Rizal; praying for judgment ordering Rizal Insurance and New India to pay the amount of P2,747, 867.00 plus legal interest, P400,000.00 as attorney's fees, exemplary damages, expenses of litigation ofP50,000.00 and costs of suit.6 Petitioner Rizal Insurance countered that its fire insurance policy sued upon covered only the contents of the four-span building, which was partly burned, and not the damage caused by the fire on the two-storey annex building.7 On January 4, 1990, the trial court rendered its decision; disposing as follows: "ACCORDINGLY, judgment is hereby rendered as follows: (1)Dismissing the case as against The New India Assurance Co., Ltd.; (2) Ordering defendant Rizal Surety And Insurance Company to pay Transwrold (sic) Knitting Mills, Inc. the amount of P826, 500.00 representing the actual value of the losses suffered by it; and (3) Cost against defendant Rizal Surety and Insurance Company. SO ORDERED."8 Both the petitioner, Rizal Insurance Company, and private respondent, Transworld Knitting Mills, Inc., went to the Court of Appeals, which came out with its decision of July 15, 1993 under attack, the decretal portion of which reads: "WHEREFORE, and upon all the foregoing, the decision of the court below is MODIFIED in that defendant New India Assurance Company has and is hereby required to pay plaintiff-appellant the amount of P1,818,604.19 while the other Rizal Surety has to pay the plaintiff-appellant P470,328.67, based on the actual losses sustained by plaintiff Transworld in the fire, totalling P2,790,376.00 as against the amounts of fire insurance coverages respectively extended by New India in the amount

of P5,800,000.00 and Rizal Surety and Insurance Company in the amount of P1,500,000.00. No costs. SO ORDERED."9 On August 20, 1993, from the aforesaid judgment of the Court of Appeals New India appealed to this Court theorizing inter alia that the private respondent could not be compensated for the loss of the fun and amusement machines and spare parts stored at the two-storey building because it (Transworld) had no insurable interest in said goods or items. On February 2, 1994, the Court denied the appeal with finality in G.R. No. L-111118 (New India Assurance Company Ltd. vs. Court of Appeals). Petitioner Rizal Insurance and private respondent Transworld, interposed a Motion for Reconsideration before the Court of Appeals, and on October 22, 1993, the Court of Appeals reconsidered its decision of July 15, 1993, as regards the imposition of interest, ruling thus: "WHEREFORE, the Decision of July 15, 1993 is amended but only insofar as the imposition of legal interest is concerned, that, on the assessment against New India Assurance Company on the amount of P1,818,604.19 and that against Rizal Surety & Insurance Company on the amount of P470,328.67, from May 26, 1982 when the complaint was filed until payment is made. The rest of the said decision is retained in all other respects. SO ORDERED."10 Undaunted, petitioner Rizal Surety & Insurance Company found its way to this Court via the present Petition, contending that: I.....SAID DECISION (ANNEX A) ERRED IN ASSUMING THAT THE ANNEX BUILDING WHERE THE BULK OF THE BURNED PROPERTIES WERE STORED, WAS INCLUDED IN THE COVERAGE OF THE INSURANCE POLICY ISSUED BY RIZAL SURETY TO TRANSWORLD. II.....SAID DECISION AND RESOLUTION (ANNEXES A AND B) ERRED IN NOT CONSIDERING THE PICTURES (EXHS. 3 TO 7-C-RIZAL SURETY), TAKEN IMMEDIATELY AFTER THE FIRE, WHICH CLEARLY SHOW THAT THE PREMISES OCCUPIED BY TRANSWORLD, WHERE THE INSURED PROPERTIES WERE LOCATED, SUSTAINED PARTIAL DAMAGE ONLY. III. SAID DECISION (ANNEX A) ERRED IN NOT HOLDING THAT TRANSWORLD HAD ACTED IN PALPABLE BAD FAITH AND WITH MALICE IN

FILING ITS CLEARLY UNFOUNDED CIVIL ACTION, AND IN NOT ORDERING TRANSWORLD TO PAY TO RIZAL SURETY MORAL AND PUNITIVE DAMAGES (ART. 2205, CIVIL CODE), PLUS ATTORNEY'S FEES AND EXPENSES OF LITIGATION (ART. 2208 PARS. 4 and 11, CIVIL CODE).11 The Petition is not impressed with merit. It is petitioner's submission that the fire insurance policy litigated upon protected only the contents of the main building (four-span),12 and did not include those stored in the two-storey annex building. On the other hand, the private respondent theorized that the so called "annex" was not an annex but was actually an integral part of the four-span building13 and therefore, the goods and items stored therein were covered by the same fire insurance policy. Resolution of the issues posited here hinges on the proper interpretation of the stipulation in subject fire insurance policy regarding its coverage, which reads: "xxx contained and/or stored during the currency of this Policy in the premises occupied by them forming part of the buildings situate (sic) within own Compound xxx" Therefrom, it can be gleaned unerringly that the fire insurance policy in question did not limit its coverage to what were stored in the four-span building. As opined by the trial court of origin, two requirements must concur in order that the said fun and amusement machines and spare parts would be deemed protected by the fire insurance policy under scrutiny, to wit: "First, said properties must be contained and/or stored in the areas occupied by Transworld and second, said areas must form part of the building described in the policy xxx"14 'Said building of four-span lofty one storey in height with mezzanine portions is constructed of reinforced concrete and hollow blocks and/or concrete under galvanized iron roof and occupied as hosiery mills, garment and lingerie factory, transistor-stereo assembly plant, offices, ware house and caretaker's quarter.' The Court is mindful of the well-entrenched doctrine that factual findings by the Court of Appeals are conclusive on the parties and not reviewable by this Court, and the same carry even more weight when the Court of Appeals has affirmed the findings of fact arrived at by the lower court.15 In the case under consideration, both the trial court and the Court of Appeals found that the so called "annex " was not an annex building but an integral and inseparable part of the four-span building described in the policy and consequently, the machines and spare parts stored therein were covered by the fire insurance in dispute. The letterreport of the Manila Adjusters and Surveyor's Company, which petitioner itself cited and invoked, describes the "annex" building as follows:

"Two-storey building constructed of partly timber and partly concrete hollow blocks under g.i. roof which is adjoining and intercommunicating with the repair of the first right span of the lofty storey building and thence by property fence wall."16 Verily, the two-storey building involved, a permanent structure which adjoins and intercommunicates with the "first right span of the lofty storey building",17 formed part thereof, and meets the requisites for compensability under the fire insurance policy sued upon. So also, considering that the two-storey building aforementioned was already existing when subject fire insurance policy contract was entered into on January 12, 1981, having been constructed sometime in 1978,18 petitioner should have specifically excluded the said two-storey building from the coverage of the fire insurance if minded to exclude the same but if did not, and instead, went on to provide that such fire insurance policy covers the products, raw materials and supplies stored within the premises of respondent Transworld which was an integral part of the four-span building occupied by Transworld, knowing fully well the existence of such building adjoining and intercommunicating with the right section of the four-span building. After a careful study, the Court does not find any basis for disturbing what the lower courts found and arrived at. Indeed, the stipulation as to the coverage of the fire insurance policy under controversy has created a doubt regarding the portions of the building insured thereby. Article 1377 of the New Civil Code provides: "Art.1377. The interpretation of obscure words or stipulations in a contract shall not favor the party who caused the obscurity" Conformably, it stands to reason that the doubt should be resolved against the petitioner, Rizal Surety Insurance Company, whose lawyer or managers drafted the fire insurance policy contract under scrutiny. Citing the aforecited provision of law in point, the Court in Landicho vs. Government Service Insurance System,19 ruled: "This is particularly true as regards insurance policies, in respect of which it is settled that the 'terms in an insurance policy, which are ambiguous, equivocal, or uncertain x x x are to be construed strictly and most strongly against the insurer, and liberally in favor of the insured so as to effect the dominant purpose of indemnity or payment to the insured, especially where forfeiture is involved' (29 Am. Jur., 181), and the reason for this is that the 'insured usually has no voice in the selection or arrangement of the words employed and that the language of the contract is selected with great care and deliberation by experts and legal advisers employed by, and acting exclusively in the interest of, the insurance company.' (44 C.J.S., p. 1174)."" 20 Equally relevant is the following disquisition of the Court in Fieldmen's Insurance Company, Inc. vs. Vda. De Songco,21 to wit:

"'This rigid application of the rule on ambiguities has become necessary in view of current business practices.1wphi1The courts cannot ignore that nowadays monopolies, cartels and concentration of capital, endowed with overwhelming economic power, manage to impose upon parties dealing with them cunningly prepared 'agreements' that the weaker party may not change one whit, his participation in the 'agreement' being reduced to the alternative to 'take it or leave it' labelled since Raymond Saleilles 'contracts by adherence' (contrats [sic] d'adhesion), in contrast to these entered into by parties bargaining on an equal footing, such contracts (of which policies of insurance and international bills of lading are prime example) obviously call for greater strictness and vigilance on the part of courts of justice with a view to protecting the weaker party from abuses and imposition, and prevent their becoming traps for the unwary (New Civil Code, Article 24; Sent. of Supreme Court of Spain, 13 Dec. 1934, 27 February 1942.)'"22 The issue of whether or not Transworld has an insurable interest in the fun and amusement machines and spare parts, which entitles it to be indemnified for the loss thereof, had been settled in G.R. No. L-111118, entitled New India Assurance Company, Ltd., vs. Court of Appeals, where the appeal of New India from the decision of the Court of Appeals under review, was denied with finality by this Court on February 2, 1994. The rule on conclusiveness of judgment, which obtains under the premises, precludes the relitigation of a particular fact or issue in another action between the same parties based on a different claim or cause of action. "xxx the judgment in the prior action operates as estoppel only as to those matters in issue or points controverted, upon the determination of which the finding or judgment was rendered. In fine, the previous judgment is conclusive in the second case, only as those matters actually and directly controverted and determined and not as to matters merely involved therein." 23 Applying the abovecited pronouncement, the Court, in Smith Bell and Company (Phils.), Inc. vs. Court of Appeals,24 held that the issue of negligence of the shipping line, which issue had already been passed upon in a case filed by one of the insurers, is conclusive and can no longer be relitigated in a similar case filed by another insurer against the same shipping line on the basis of the same factual circumstances. Ratiocinating further, the Court opined: "In the case at bar, the issue of which vessel ('Don Carlos' or 'Yotai Maru') had been negligent, or so negligent as to have proximately caused the collision between them, was an issue that was actually, directly and expressly raised, controverted and litigated in C.A.-G.R. No. 61320-R. Reyes, L.B., J., resolved that issue in his Decision and held the 'Don Carlos' to have been negligent rather than the 'Yotai Maru' and, as already noted, that Decision was affirmed by this Court in G.R. No. L-48839 in a Resolution dated 6 December 1987. The Reyes Decision thus became final and executory approximately two (2) years before the Sison Decision, which is assailed in the case at bar, was promulgated. Applying the rule of conclusiveness of judgment, the question of which vessel had been negligent in the collision between the two (2) vessels, had long

been settled by this Court and could no longer be relitigated in C.A.-G.R. No. 61206-R. Private respondent Go Thong was certainly bound by the ruling or judgment of Reyes, L.B., J. and that of this Court. The Court of Appeals fell into clear and reversible error when it disregarded the Decision of this Court affirming the Reyes Decision." 25 The controversy at bar is on all fours with the aforecited case. Considering that private respondent's insurable interest in, and compensability for the loss of subject fun and amusement machines and spare parts, had been adjudicated, settled and sustained by the Court of Appeals in CA-G.R. CV NO. 28779, and by this Court in G.R. No. L111118, in a Resolution, dated February 2, 1994, the same can no longer be relitigated and passed upon in the present case. Ineluctably, the petitioner, Rizal Surety Insurance Company, is bound by the ruling of the Court of Appeals and of this Court that the private respondent has an insurable interest in the aforesaid fun and amusement machines and spare parts; and should be indemnified for the loss of the same. So also, the Court of Appeals correctly adjudged petitioner liable for the amount of P470,328.67, it being the total loss and damage suffered by Transworld for which petitioner Rizal Insurance is liable.26 All things studiedly considered and viewed in proper perspective, the Court is of the irresistible conclusion, and so finds, that the Court of Appeals erred not in holding the petitioner, Rizal Surety Insurance Company, liable for the destruction and loss of the insured buildings and articles of the private respondent. WHEREFORE, the Decision, dated July 15, 1993, and the Resolution, dated October 22, 1993, of the Court of Appeals in CA-G.R. CV NO. 28779 are AFFIRMED in toto. No pronouncement as to costs. SO ORDERED.

III.

Construction of Insurance Contracts


December 16, 1955

G.R. No. L-8151

VIRGINIA CALANOC, petitioner, vs. COURT OF APPEALS and THE PHILIPPINE AMERICAN LIFE INSURANCE CO., respondents. Lucio Javillonar for petitioner. J. A. Wolfson, Manuel Y. Mecias, Emilio Abello and Anselmo A. Reyes for respondents.

BAUTISTA ANGELO, J.: This suit involves the collection of P2,000 representing the value of a supplemental policy covering accidental death which was secured by one Melencio Basilio from the Philippine American Life Insurance Company. The case originated in the Municipal Court of Manila and judgment being favorable to the plaintiff it was appealed to the court of first instance. The latter court affirmed the judgment but on appeal to the Court of Appeals the judgment was reversed and the case is now before us on a petition for review. Melencio Basilio was a watchman of the Manila Auto Supply located at the corner of Avenida Rizal and Zurbaran. He secured a life insurance policy from the Philippine American Life Insurance Company in the amount of P2,000 to which was attached a supplementary contract covering death by accident. On January 25, 1951, he died of a gunshot wound on the occasion of a robbery committed in the house of Atty. Ojeda at the corner of Oroquieta and Zurbaan streets. Virginia Calanoc, the widow, was paid the sum of P2,000, face value of the policy, but when she demanded the payment of the additional sum of P2,000 representing the value of the supplemental policy, the company refused alleging, as main defense, that the deceased died because he was murdered by a person who took part in the commission of the robbery and while making an arrest as an officer of the law which contingencies were expressly excluded in the contract and have the effect of exempting the company from liability. The pertinent facts which need to be considered for the determination of the questions raised are those reproduced in the decision of the Court of Appeals as follows: The circumstances surrounding the death of Melencio Basilio show that when he was killed at about seven o'clock in the night of January 25, 1951, he was on duty as watchman of the Manila Auto Supply at the corner of Avenida Rizal and Zurbaran; that it turned out that Atty. Antonio Ojeda who had his residence at the corner of Zurbaran and Oroquieta, a block away from Basilio's station, had come home that night and found that his house was well-lighted, but with the windows

closed; that getting suspicious that there were culprits in his house, Atty. Ojeda retreated to look for a policeman and finding Basilio in khaki uniform, asked him to accompany him to the house with the latter refusing on the ground that he was not a policeman, but suggesting that Atty. Ojeda should ask the traffic policeman on duty at the corner of Rizal Avenue and Zurbaran; that Atty. Ojeda went to the traffic policeman at said corner and reported the matter, asking the policeman to come along with him, to which the policeman agreed; that on the way to the Ojeda residence, the policeman and Atty. Ojeda passed by Basilio and somehow or other invited the latter to come along; that as the tree approached the Ojeda residence and stood in front of the main gate which was covered with galvanized iron, the fence itself being partly concrete and partly adobe stone, a shot was fired; that immediately after the shot, Atty. Ojeda and the policeman sought cover; that the policeman, at the request of Atty. Ojeda, left the premises to look for reinforcement; that it turned out afterwards that the special watchman Melencio Basilio was hit in the abdomen, the wound causing his instantaneous death; that the shot must have come from inside the yard of Atty. Ojeda, the bullet passing through a hole waist-high in the galvanized iron gate; that upon inquiry Atty. Ojeda found out that the savings of his children in the amount of P30 in coins kept in his aparador contained in stockings were taken away, the aparador having been ransacked; that a month thereafter the corresponding investigation conducted by the police authorities led to the arrest and prosecution of four persons in Criminal Case No. 15104 of the Court of First Instance of Manila for 'Robbery in an Inhabited House and in Band with Murder'. It is contended in behalf of the company that Basilio was killed which "making an arrest as an officer of the law" or as a result of an "assault or murder" committed in the place and therefore his death was caused by one of the risks excluded by the supplementary contract which exempts the company from liability. This contention was upheld by the Court of Appeals and, in reaching this conclusion, made the following comment: From the foregoing testimonies, we find that the deceased was a watchman of the Manila Auto Supply, and, as such, he was not boud to leave his place and go with Atty. Ojeda and Policeman Magsanoc to see the trouble, or robbery, that occurred in the house of Atty. Ojeda. In fact, according to the finding of the lower court, Atty. Ojeda finding Basilio in uniform asked him to accompany him to his house, but the latter refused on the ground that he was not a policeman and suggested to Atty. Ojeda to ask help from the traffic policeman on duty at the corner of Rizal Avenue and Zurbaran, but after Atty. Ojeda secured the help of the traffic policeman, the deceased went with Ojeda and said traffic policeman to the residence of Ojeda, and while the deceased was standing in front of the main gate of said residence, he was shot and thus died. The death, therefore, of Basilio, although unexpected, was not caused by an accident, being a voluntary and intentional act on the part of the one wh robbed, or one of those who robbed, the house of Atty. Ojeda. Hence, it is out considered opinion that the death of Basilio, though unexpected, cannot be considered accidental, for his death occurred because he left his post and joined policeman Magsanoc and Atty.

Ojeda to repair to the latter's residence to see what happened thereat. Certainly, when Basilio joined Patrolman Magsanoc and Atty. Ojeda, he should have realized the danger to which he was exposing himself, yet, instead of remaining in his place, he went with Atty. Ojeda and Patrolman Magsanoc to see what was the trouble in Atty. Ojeda's house and thus he was fatally shot. We dissent from the above findings of the Court of Appeals. For one thing, Basilio was a watchman of the Manila Auto Supply which was a block away from the house of Atty. Ojeda where something suspicious was happening which caused the latter to ask for help. While at first he declied the invitation of Atty. Ojeda to go with him to his residence to inquire into what was going on because he was not a regular policeman, he later agreed to come along when prompted by the traffic policeman, and upon approaching the gate of the residence he was shot and died. The circumstance that he was a mere watchman and had no duty to heed the call of Atty. Ojeda should not be taken as a capricious desire on his part to expose his life to danger considering the fact that the place he was in duty-bound to guard was only a block away. In volunteering to extend help under the situation, he might have thought, rightly or wrongly, that to know the truth was in the interest of his employer it being a matter that affects the security of the neighborhood. No doubt there was some risk coming to him in pursuing that errand, but that risk always existed it being inherent in the position he was holding. He cannot therefore be blamed solely for doing what he believed was in keeping with his duty as a watchman and as a citizen. And he cannot be considered as making an arrest as an officer of the law, as contended, simply because he went with the traffic policeman, for certainly he did not go there for that purpose nor was he asked to do so by the policeman. Much less can it be pretended that Basilio died in the course of an assault or murder considering the very nature of these crimes. In the first place, there is no proof that the death of Basilio is the result of either crime for the record is barren of any circumstance showing how the fatal shot was fired. Perhaps this may be clarified in the criminal case now pending in court as regards the incident but before that is done anything that might be said on the point would be a mere conjecture. Nor can it be said that the killing was intentional for there is the possibility that the malefactor had fired the shot merely to scare away the people around for his own protection and not necessarily to kill or hit the victim. In any event, while the act may not excempt the triggerman from liability for the damage done, the fact remains that the happening was a pure accident on the part of the victim. The victim could have been either the policeman or Atty. Ojeda for it cannot be pretended that the malefactor aimed at the deceased precisely because he wanted to take his life. We take note that these defenses are included among the risks exluded in the supplementary contract which enumerates the cases which may exempt the company from liability. While as a general rule "the parties may limit the coverage of the policy to certain particular accidents and risks or causes of loss, and may expressly except other risks or causes of loss therefrom" (45 C. J. S. 781-782), however, it is to be desired that the terms and phraseology of the exception clause be clearly expressed so as to be

within the easy grasp and understanding of the insured, for if the terms are doubtful or obscure the same must of necessity be interpreted or resolved aganst the one who has caused the obscurity. (Article 1377, new Civil Code) And so it has bene generally held that the "terms in an insurance policy, which are ambiguous, equivacal, or uncertain . . . are to be construed strictly and most strongly against the insurer, and liberally in favor of the insured so as to effect the dominant purpose of indemnity or payment to the insured, especially where a forfeiture is involved" (29 Am. Jur., 181), and the reason for this rule is that he "insured usually has no voice in the selection or arrangement of the words employed and that the language of the contract is selected with great care and deliberation by experts and legal advisers employed by, and acting exclusively in the interest of, the insurance company." (44 C. J. S., p. 1174.) Insurance is, in its nature, complex and difficult for the layman to understand. Policies are prepared by experts who know and can anticipate the bearings and possible complications of every contingency. So long as insurance companies insist upon the use of ambiguous, intricate and technical provisions, which conceal rather than frankly disclose, their own intentions, the courts must, in fairness to those who purchase insurance, construe every ambiguity in favor of the insured. (Algoe vs. Pacific Mut. L. Ins. Co., 91 Wash. 324, LRA 1917A, 1237.)lawphi1.net An insurer should not be allowed, by the use of obscure phrases and exceptions, to defeat the very purpose for which the policy was procured. (Moore vs. Aetna Life Insurance Co., LRA 1915D, 264.) We are therefore persuaded to conclude that the circumstances unfolded in the present case do not warrant the finding that the death of the unfortunate victim comes within the purview of the exception clause of the supplementary policy and, hence, do not exempt the company from liability. Wherefore, reversing the decision appealed from, we hereby order the company to pay petitioner-appellant the amount of P2,000, with legal interest from January 26, 1951 until fully paid, with costs. Paras, C. J., Bengzon, Padilla, Montemayor, Reyes, A., Jugo, Labrador, Concepcion, and Reyes, J. B. L., JJ., concur.

G.R. No. 114427 February 6, 1995 ARMANDO GEAGONIA, petitioner, vs. COURT OF APPEALS and COUNTRY BANKERS INSURANCE CORPORATION, respondents.

DAVIDE, JR., J.: Four our review under Rule 45 of the Rules of Court is the decision 1 of the Court of Appeals in CA-G.R. SP No. 31916, entitled "Country Bankers Insurance Corporation versus Armando Geagonia," reversing the decision of the Insurance Commission in I.C. Case No. 3340 which awarded the claim of petitioner Armando Geagonia against private respondent Country Bankers Insurance Corporation. The petitioner is the owner of Norman's Mart located in the public market of San Francisco, Agusan del Sur. On 22 December 1989, he obtained from the private respondent fire insurance policy No. F-14622 2 for P100,000.00. The period of the policy was from 22 December 1989 to 22 December 1990 and covered the following: "Stockin-trade consisting principally of dry goods such as RTW's for men and women wear and other usual to assured's business." The petitioner declared in the policy under the subheading entitled CO-INSURANCE that Mercantile Insurance Co., Inc. was the co-insurer for P50,000.00. From 1989 to 1990, the petitioner had in his inventory stocks amounting to P392,130.50, itemized as follows: Zenco Sales, Inc. F. Legaspi Gen. Merchandise Cebu Tesing Textiles P55,698.00 86,432.50 250,000.00 (on credit) P392,130.50

The policy contained the following condition: 3. The insured shall give notice to the Company of any insurance or insurances already affected, or which may subsequently be effected, covering any of the property or properties consisting of stocks in trade, goods in process and/or inventories only hereby insured, and unless such notice be given and the particulars of such insurance or insurances be stated therein or endorsed in this policy pursuant to Section 50 of the Insurance Code, by or on behalf of the Company before the occurrence of any loss or damage, all benefits under this policy shall be deemed

forfeited, provided however, that this condition shall not apply when the total insurance or insurances in force at the time of the loss or damage is not more than P200,000.00. On 27 May 1990, fire of accidental origin broke out at around 7:30 p.m. at the public market of San Francisco, Agusan del Sur. The petitioner's insured stock-in-trade were completely destroyed prompting him to file with the private respondent a claim under the policy. On 28 December 1990, the private respondent denied the claim because it found that at the time of the loss the petitioner's stocks-in-trade were likewise covered by fire insurancepolicies No. GA-28146 and No. GA-28144, for P100,000.00 each, issued by the Cebu Branch of the Philippines First Insurance Co., Inc. (hereinafter PFIC). 3 These policies indicate that the insured was "Messrs. Discount Mart (Mr. Armando Geagonia, Prop.)" with a mortgage clause reading: MORTGAGE: Loss, if any shall be payable to Messrs. Cebu Tesing Textiles, Cebu City as their interest may appear subject to the terms of this policy. CO-INSURANCE DECLARED: P100,000. Phils. First CEB/F 24758. 4 The basis of the private respondent's denial was the petitioner's alleged violation of Condition 3 of the policy. The petitioner then filed a complaint 5 against the private respondent with the Insurance Commission (Case No. 3340) for the recovery of P100,000.00 under fire insurance policy No. F-14622 and for attorney's fees and costs of litigation. He attached as Annex "AM" 6 thereof his letter of 18 January 1991 which asked for the reconsideration of the denial. He admitted in the said letter that at the time he obtained the private respondent's fire insurance policy he knew that the two policies issued by the PFIC were already in existence; however, he had no knowledge of the provision in the private respondent's policy requiring him to inform it of the prior policies; this requirement was not mentioned to him by the private respondent's agent; and had it been mentioned, he would not have withheld such information. He further asserted that the total of the amounts claimed under the three policies was below the actual value of his stocks at the time of loss, which was P1,000,000.00. In its answer, 7 the private respondent specifically denied the allegations in the complaint and set up as its principal defense the violation of Condition 3 of the policy. In its decision of 21 June 1993, 8 the Insurance Commission found that the petitioner did not violate Condition 3 as he had no knowledge of the existence of the two fire insurance policies obtained from the PFIC; that it was Cebu Tesing Textiles which procured the PFIC policies without informing him or securing his consent; and that Cebu Tesing Textile, as his creditor, had insurable interest on the stocks. These findings were based on the petitioner's testimony that he came to know of the PFIC policies only when he filed his claim with the private respondent and that Cebu Tesing Textile obtained

them and paid for their premiums without informing him thereof. The Insurance Commission then decreed: WHEREFORE, judgment is hereby rendered ordering the respondent company to pay complainant the sum of P100,000.00 with legal interest from the time the complaint was filed until fully satisfied plus the amount of P10,000.00 as attorney's fees. With costs. The compulsory counterclaim of respondent is hereby dismissed. Its motion for the reconsideration of the decision 9 having been denied by the Insurance Commission in its resolution of 20 August 1993, 10 the private respondent appealed to the Court of Appeals by way of a petition for review. The petition was docketed as CAG.R. SP No. 31916. In its decision of 29 December 1993, 11 the Court of Appeals reversed the decision of the Insurance Commission because it found that the petitioner knew of the existence of the two other policies issued by the PFIC. It said: It is apparent from the face of Fire Policy GA 28146/Fire Policy No. 28144 that the insurance was taken in the name of private respondent [petitioner herein]. The policy states that "DISCOUNT MART (MR. ARMANDO GEAGONIA, PROP)" was the assured and that "TESING TEXTILES" [was] only the mortgagee of the goods. In addition, the premiums on both policies were paid for by private respondent, not by the Tesing Textiles which is alleged to have taken out the other insurance without the knowledge of private respondent. This is shown by Premium Invoices nos. 46632 and 46630. (Annexes M and N). In both invoices, Tesing Textiles is indicated to be only the mortgagee of the goods insured but the party to which they were issued were the "DISCOUNT MART (MR. ARMANDO GEAGONIA)." In is clear that it was the private respondent [petitioner herein] who took out the policies on the same property subject of the insurance with petitioner. Hence, in failing to disclose the existence of these insurances private respondent violated Condition No. 3 of Fire Policy No. 1462. . . . Indeed private respondent's allegation of lack of knowledge of the provisions insurances is belied by his letter to petitioner [of 18 January 1991. The body of the letter reads as follows;] xxx xxx xxx Please be informed that I have no knowledge of the provision requiring me to inform your office about my prior insurance under FGA-28146 and F-CEB-24758. Your

representative did not mention about said requirement at the time he was convincing me to insure with you. If he only die or even inquired if I had other existing policies covering my establishment, I would have told him so. You will note that at the time he talked to me until I decided to insure with your company the two policies aforementioned were already in effect. Therefore I would have no reason to withhold such information and I would have desisted to part with my hard earned peso to pay the insurance premiums [if] I know I could not recover anything. Sir, I am only an ordinary businessman interested in protecting my investments. The actual value of my stocks damaged by the fire was estimated by the Police Department to be P1,000,000.00 (Please see xerox copy of Police Report Annex "A"). My Income Statement as of December 31, 1989 or five months before the fire, shows my merchandise inventory was already some P595,455.75. . . . These will support my claim that the amount claimed under the three policies are much below the value of my stocks lost. xxx xxx xxx The letter contradicts private respondent's pretension that he did not know that there were other insurances taken on the stock-in-trade and seriously puts in question his credibility. His motion to reconsider the adverse decision having been denied, the petitioner filed the instant petition. He contends therein that the Court of Appeals acted with grave abuse of discretion amounting to lack or excess of jurisdiction: A . . . WHEN IT REVERSED THE FINDINGS OF FACTS OF THE INSURANCE COMMISSION, A QUASI-JUDICIAL BODY CHARGED WITH THE DUTY OF DETERMINING INSURANCE CLAIM AND WHOSE DECISION IS ACCORDED RESPECT AND EVEN FINALITY BY THE COURTS; B . . . WHEN IT CONSIDERED AS EVIDENCE MATTERS WHICH WERE NOT PRESENTED AS EVIDENCE DURING THE HEARING OR TRIAL; AND C . . . WHEN IT DISMISSED THE CLAIM OF THE PETITIONER HEREIN AGAINST THE PRIVATE RESPONDENT.

The chief issues that crop up from the first and third grounds are (a) whether the petitioner had prior knowledge of the two insurance policies issued by the PFIC when he obtained the fire insurance policy from the private respondent, thereby, for not disclosing such fact, violating Condition 3 of the policy, and (b) if he had, whether he is precluded from recovering therefrom. The second ground, which is based on the Court of Appeals' reliance on the petitioner's letter of reconsideration of 18 January 1991, is without merit. The petitioner claims that the said letter was not offered in evidence and thus should not have been considered in deciding the case. However, as correctly pointed out by the Court of Appeals, a copy of this letter was attached to the petitioner's complaint in I.C. Case No. 3440 as Annex "M" thereof and made integral part of the complaint. 12 It has attained the status of a judicial admission and since its due execution and authenticity was not denied by the other party, the petitioner is bound by it even if it were not introduced as an independent evidence. 13 As to the first issue, the Insurance Commission found that the petitioner had no knowledge of the previous two policies. The Court of Appeals disagreed and found otherwise in view of the explicit admission by the petitioner in his letter to the private respondent of 18 January 1991, which was quoted in the challenged decision of the Court of Appeals. These divergent findings of fact constitute an exception to the general rule that in petitions for review under Rule 45, only questions of law are involved and findings of fact by the Court of Appeals are conclusive and binding upon this Court. 14 We agree with the Court of Appeals that the petitioner knew of the prior policies issued by the PFIC. His letter of 18 January 1991 to the private respondent conclusively proves this knowledge. His testimony to the contrary before the Insurance Commissioner and which the latter relied upon cannot prevail over a written admission made ante litem motam. It was, indeed, incredible that he did not know about the prior policies since these policies were not new or original. Policy No. GA-28144 was a renewal of Policy No. F-24758, while Policy No. GA-28146 had been renewed twice, the previous policy being F-24792. Condition 3 of the private respondent's Policy No. F-14622 is a condition which is not proscribed by law. Its incorporation in the policy is allowed by Section 75 of the Insurance Code 15 which provides that "[a] policy may declare that a violation of specified provisions thereof shall avoid it, otherwise the breach of an immaterial provision does not avoid the policy." Such a condition is a provision which invariably appears in fire insurance policies and is intended to prevent an increase in the moral hazard. It is commonly known as the additional or "other insurance" clause and has been upheld as valid and as a warranty that no other insurance exists. Its violation would thus avoid the policy. 16 However, in order to constitute a violation, the other insurance must be upon same subject matter, the same interest therein, and the same risk. 17

As to a mortgaged property, the mortgagor and the mortgagee have each an independent insurable interest therein and both interests may be one policy, or each may take out a separate policy covering his interest, either at the same or at separate times. 18 The mortgagor's insurable interest covers the full value of the mortgaged property, even though the mortgage debt is equivalent to the full value of the property. 19 The mortgagee's insurable interest is to the extent of the debt, since the property is relied upon as security thereof, and in insuring he is not insuring the property but his interest or lien thereon. His insurable interest is prima facie the value mortgaged and extends only to the amount of the debt, not exceeding the value of the mortgaged property. 20Thus, separate insurances covering different insurable interests may be obtained by the mortgagor and the mortgagee. A mortgagor may, however, take out insurance for the benefit of the mortgagee, which is the usual practice. The mortgagee may be made the beneficial payee in several ways. He may become the assignee of the policy with the consent of the insurer; or the mere pledgee without such consent; or the original policy may contain a mortgage clause; or a rider making the policy payable to the mortgagee "as his interest may appear" may be attached; or a "standard mortgage clause," containing a collateral independent contract between the mortgagee and insurer, may be attached; or the policy, though by its terms payable absolutely to the mortgagor, may have been procured by a mortgagor under a contract duty to insure for the mortgagee's benefit, in which case the mortgagee acquires an equitable lien upon the proceeds. 21 In the policy obtained by the mortgagor with loss payable clause in favor of the mortgagee as his interest may appear, the mortgagee is only a beneficiary under the contract, and recognized as such by the insurer but not made a party to the contract himself. Hence, any act of the mortgagor which defeats his right will also defeat the right of the mortgagee. 22 This kind of policy covers only such interest as the mortgagee has at the issuing of the policy. 23 On the other hand, a mortgagee may also procure a policy as a contracting party in accordance with the terms of an agreement by which the mortgagor is to pay the premiums upon such insurance. 24 It has been noted, however, that although the mortgagee is himself the insured, as where he applies for a policy, fully informs the authorized agent of his interest, pays the premiums, and obtains on the assurance that it insures him, the policy is in fact in the form used to insure a mortgagor with loss payable clause. 25 The fire insurance policies issued by the PFIC name the petitioner as the assured and contain a mortgage clause which reads: Loss, if any, shall be payable to MESSRS. TESING TEXTILES, Cebu City as their interest may appear subject to the terms of this policy. This is clearly a simple loss payable clause, not a standard mortgage clause.

It must, however, be underscored that unlike the "other insurance" clauses involved in General Insurance and Surety Corp. vs. Ng Hua 26 or in Pioneer Insurance & Surety Corp. vs. Yap, 27 which read: The insured shall give notice to the company of any insurance or insurances already effected, or which may subsequently be effected covering any of the property hereby insured, and unless such notice be given and the particulars of such insurance or insurances be stated in or endorsed on this Policy by or on behalf of the Company before the occurrence of any loss or damage, all benefits under this Policy shall be forfeited. or in the 1930 case of Santa Ana vs. Commercial Union Assurance Co. 28 which provided "that any outstanding insurance upon the whole or a portion of the objects thereby assured must be declared by the insured in writing and he must cause the company to add or insert it in the policy, without which such policy shall be null and void, and the insured will not be entitled to indemnity in case of loss," Condition 3 in the private respondent's policy No. F-14622 does not absolutely declare void any violation thereof. It expressly provides that the condition "shall not apply when the total insurance or insurances in force at the time of the loss or damage is not more than P200,000.00." It is a cardinal rule on insurance that a policy or insurance contract is to be interpreted liberally in favor of the insured and strictly against the company, the reason being, undoubtedly, to afford the greatest protection which the insured was endeavoring to secure when he applied for insurance. It is also a cardinal principle of law that forfeitures are not favored and that any construction which would result in the forfeiture of the policy benefits for the person claiming thereunder, will be avoided, if it is possible to construe the policy in a manner which would permit recovery, as, for example, by finding a waiver for such forfeiture. 29 Stated differently, provisions, conditions or exceptions in policies which tend to work a forfeiture of insurance policies should be construed most strictly against those for whose benefits they are inserted, and most favorably toward those against whom they are intended to operate. 30 The reason for this is that, except for riders which may later be inserted, the insured sees the contract already in its final form and has had no voice in the selection or arrangement of the words employed therein. On the other hand, the language of the contract was carefully chosen and deliberated upon by experts and legal advisers who had acted exclusively in the interest of the insurers and the technical language employed therein is rarely understood by ordinary laymen. 31 With these principles in mind, we are of the opinion that Condition 3 of the subject policy is not totally free from ambiguity and must, perforce, be meticulously analyzed. Such analysis leads us to conclude that (a) the prohibition applies only to double insurance, and (b) the nullity of the policy shall only be to the extent exceeding P200,000.00 of the total policies obtained.

The first conclusion is supported by the portion of the condition referring to other insurance "covering any of the property or properties consisting of stocks in trade, goods in process and/or inventories only hereby insured," and the portion regarding the insured's declaration on the subheading CO-INSURANCE that the co-insurer is Mercantile Insurance Co., Inc. in the sum of P50,000.00. A double insurance exists where the same person is insured by several insurers separately in respect of the same subject and interest. As earlier stated, the insurable interests of a mortgagor and a mortgagee on the mortgaged property are distinct and separate. Since the two policies of the PFIC do not cover the same interest as that covered by the policy of the private respondent, no double insurance exists. The non-disclosure then of the former policies was not fatal to the petitioner's right to recover on the private respondent's policy. Furthermore, by stating within Condition 3 itself that such condition shall not apply if the total insurance in force at the time of loss does not exceed P200,000.00, the private respondent was amenable to assume a co-insurer's liability up to a loss not exceeding P200,000.00. What it had in mind was to discourage over-insurance. Indeed, the rationale behind the incorporation of "other insurance" clause in fire policies is to prevent over-insurance and thus avert the perpetration of fraud. When a property owner obtains insurance policies from two or more insurers in a total amount that exceeds the property's value, the insured may have an inducement to destroy the property for the purpose of collecting the insurance. The public as well as the insurer is interested in preventing a situation in which a fire would be profitable to the insured. 32 WHEREFORE, the instant petition is hereby GRANTED. The decision of the Court of Appeals in CA-G.R. SP No. 31916 is SET ASIDE and the decision of the Insurance Commission in Case No. 3340 is REINSTATED. Costs against private respondent Country Bankers Insurance Corporation. SO ORDERED.

G.R. No. L-25579 March 29, 1972 EMILIA T. BIAGTAN, JUAN T. BIAGTAN, JR., MIGUEL T. BIAGTAN, GIL T. BIAGTAN and GRACIA T. BIAGTAN,plaintiffs-appellees, vs. THE INSULAR LIFE ASSURANCE COMPANY, LTD., defendant-appellant. Tanopo, Millora, Serafica, and Saez for plaintiff-appellees. Araneta, Mendoza and Papa for defendant-appellant.

MAKALINTAL, J.:p This is an appeal from the decision of the Court of First Instance of Pangasinan in its Civil Case No. D-1700. The facts are stipulated. Juan S. Biagtan was insured with defendant InsularLife Assurance Company under Policy No. 398075 for the sum of P5,000.00 and, under a supplementary contract denominated "Accidental Death Benefit Clause, for an additional sum of P5,000.00 if "the death of the Insured resulted directly from bodily injury effected solely through external and violent means sustained in an accident ... and independently of all other causes." The clause, however,expressly provided that it would not apply where death resulted from an injury"intentionally inflicted by another party." On the night of May 20, 1964, or during the first hours of the following day a band of robbers entered the house of the insured Juan S. Biagtan. What happened then is related in the decision of the trial court as follows: ...; that on the night of May 20, 1964 or the first hours of May 21, 1964, while the said life policy and supplementary contract were in full force and effect, the house of insured Juan S. Biagtan was robbed by a band of robbers who were charged in and convicted by the Court of First Instance of Pangasinan for robbery with homicide; that in committing the robbery, the robbers, on reaching the staircase landing on the second floor, rushed towards the door of the second floor room, where they suddenly met a person near the door of oneof the rooms who turned out to be the insured Juan S. Biagtan who received thrusts from their sharp-pointed instruments, causing wounds on the body of said Juan S. Biagtan resulting in his death at about 7 a.m. on the same day, May 21, 1964; Plaintiffs, as beneficiaries of the insured, filed a claim under the policy. The insurance company paid the basic amount of P5,000.00 but refused to pay the additional sum of P5,000.00 under the accidental death benefit clause, on the ground that the insured's

death resulted from injuries intentionally inflicted by third parties and therefore was not covered. Plaintiffs filed suit to recover, and after due hearing the court a quo rendered judgment in their favor. Hence the present appeal by the insurer. The only issue here is whether under the facts are stipulated and found by the trial court the wounds received by the insured at the hands of the robbers nine in all, five of them mortal and four non-mortal were inflicted intentionally. The court, in ruling negatively on the issue, stated that since the parties presented no evidence and submitted the case upon stipulation, there was no "proof that the act of receiving thrust (sic) from the sharp-pointed instrument of the robbers was intended to inflict injuries upon the person of the insured or any other person or merely to scare away any person so as to ward off any resistance or obstacle that might be offered in the pursuit of their main objective which was robbery." The trial court committed a plain error in drawing the conclusion it did from the admitted facts. Nine wounds were inflicted upon the deceased, all by means of thrusts with sharp-pointed instruments wielded by the robbers. This is a physical fact as to which there is no dispute. So is the fact that five of those wounds caused the death of the insured. Whether the robbers had the intent to kill or merely to scare the victim or to ward off any defense he might offer, it cannot be denied that the act itself of inflicting the injuries was intentional. It should be noted that the exception in the accidental benefit clause invoked by the appellant does not speak of the purpose whether homicidal or not of a third party in causing the injuries, but only of the fact that such injuries have been "intentionally" inflicted this obviously to distinguish them from injuries which, although received at the hands of a third party, are purely accidental. This construction is the basic idea expressed in the coverage of the clause itself, namely, that "the death of the insured resulted directly from bodily injury effected solely through external and violent means sustained in an accident ... and independently of all other causes." A gun which discharges while being cleaned and kills a bystander; a hunter who shoots at his prey and hits a person instead; an athlete in a competitive game involving physical effort who collides with an opponent and fatally injures him as a result: these are instances where the infliction of the injury is unintentional and therefore would be within the coverage of an accidental death benefit clause such as thatin question in this case. But where a gang of robbers enter a house and coming face to face with the owner, even if unexpectedly, stab him repeatedly, it is contrary to all reason and logic to say that his injuries are not intentionally inflicted, regardless of whether they prove fatal or not. As it was, in the present case they did prove fatal, and the robbers have been accused and convicted of the crime of robbery with homicide. The case of Calanoc vs. Court of Appeals, 98 Phil. 79, is relied upon by the trial court in support of its decision. The facts in that case, however, are different from those obtaining here. The insured there was a watchman in a certain company, who happened to be invited by a policeman to come along as the latter was on his way to investigate a reported robbery going on in a private house. As the two of them, together with the owner of the house, approached and stood in front of the main gate, a shot was fired and it turned out afterwards that the watchman was hit in the abdomen, the wound

causing his death. Under those circumstances this Court held that it could not be said that the killing was intentional for there was the possibility that the malefactor had fired the shot to scare people around for his own protection and not necessarrily to kill or hit the victim. A similar possibility is clearly ruled out by the facts in the case now before Us. For while a single shot fired from a distance, and by a person who was not even seen aiming at the victim, could indeed have been fired without intent to kill or injure, nine wounds inflicted with bladed weapons at close range cannot conceivably be considered as innocent insofar as such intent is concerned. The manner of execution of the crime permits no other conclusion. Court decisions in the American jurisdiction, where similar provisions in accidental death benefit clauses in insurance policies have been construed, may shed light on the issue before Us. Thus, it has been held that "intentional" as used in an accident policy excepting intentional injuries inflicted by the insured or any other person, etc., implies the exercise of the reasoning faculties, consciousness and volition. 1 Where a provision of the policy excludes intentional injury, it is the intention of the person inflicting the injury that is controlling. 2 If the injuries suffered by the insured clearly resulted from the intentional act of a third person the insurer is relieved from liability as stipulated. 3 In the case of Hutchcraft's Ex'r v. Travelers' Ins. Co., 87 Ky. 300, 8 S.W. 570, 12 Am. St. Rep. 484, the insured was waylaid and assassinated for the purpose of robbery. Two (2) defenses were interposed to the action to recover indemnity, namely: (1) that the insured having been killed by intentional means, his death was not accidental, and (2) that the proviso in the policy expressly exempted the insurer from liability in case the insured died from injuries intentionally inflicted by another person. In rendering judgment for the insurance company the Court held that while the assassination of the insured was as to him an unforeseen event and therefore accidental, "the clause of the proviso that excludes the (insurer's) liability, in case death or injury is intentionally inflicted by another person, applies to this case." In Butero v. Travelers' Acc. Ins. Co., 96 Wis. 536, 65 Am. St. Rep. 61, 71 S.W. 811, the insured was shot three times by a person unknown late on a dark and stormy night, while working in the coal shed of a railroad company. The policy did not cover death resulting from "intentional injuries inflicted by the insured or any other person." The inquiry was as to the question whether the shooting that caused the insured's death was accidental or intentional; and the Court found that under the facts, showing that the murderer knew his victim and that he fired with intent to kill, there could be no recovery under the policy which excepted death from intentional injuries inflicted by any person. WHEREFORE, the decision appealed from is reversed and the complaint dismissed, without pronouncement as to costs. Zaldivar, Castro, Fernando and Villamor, JJ., concur. Makasiar, J., reserves his vote.

Separate Opinions

BARREDO, J., concurring During the deliberations in this case, I entertained some doubts as to the correctness and validity of the view upheld in the main opinion penned by Justice Makalintal. Further reflection has convinced me, however, that there are good reasons to support it. At first blush, one would feel that every death not suicidal should be considered accidental, for the purposes of an accident insurance policy or a life insurance policy with a double indemnity clause in case death results from accident. Indeed, it is quite logical to think that any event whether caused by fault, negligence, intent of a third party or any unavoidable circumstance, normally unforeseen by the insured and free from any possible connivance on his part, is an accident in the generally accepted sense of the term. And if I were convinced that in including in the policy the provision in question, both the insurer and the insured had in mind to exclude thereby from the coverage of the policy only suicide whether unhelped or helped somehow by a third party, I would disregard the American decisions cited and quoted in the main opinion as not even persuasive authorities. But examining the unequivocal language of the provision in controversy and considering that the insured accepted the policy without asking that it be made clear that the phrase "injury intentionally inflicted by a third party" should be understood to refer only to injuries inflicted by a third party without any wilful intervention on his part (of the insured) or, in other words, without any connivance with him (the insured) in order to augment the proceeds of the policy for his benificiaries, I am inclined to agree that death caused by criminal assault is not covered by the policies of the kind here in question, specially if the assault, as a matter of fact, could have been more or less anticipated, as when the insured happens to have violent enemies or is found in circumstances that would make his life fair game of third parties. As to the rest, I have no doubt that the killing of the insured in this case is as intentional as any intentional act can be, hence this concurrence. TEEHANKEE, J., dissenting: The sole issue at bar is the correctness in law of the lower court's appealed decision adjudging defendant insurance company liable, under its supplementary contract denominated "Accidental Death Benefit Clause" with the deceased insured, to plaintiffsbeneficiaries (excluding plaintiff Emilia T. Biagtan) in an additional amount of P5,000.00

(with corresponding legal interest) and ruling that defendant company had failed to present any evidence to substantiate its defense that the insured's death came within the stipulated exceptions. Defendant's accidental death benefit clause expressly provides: ACCIDENTAL DEATH BENEFIT. (hereinafter called the benefit). Upon receipt and approval of due proof that the death of the Insured resulted directly from bodily injury effected solely through external and violent means sustained in an accident, within ninety days after the date of sustaining such injury, and independently of all other causes, this Company shall pay, in addition to the sum insured specified on the first page of this Policy, a further sum equal to said sum insured payable at the same time and in the same manner as said sum insured, provided, that such death occurred during the continuance of this Clause and of this Policy and before the sixtieth birthday of the Insured." 1 A long list of exceptions and an Automatic Discontinuance clause immediately follow thereafter, thus: EXCEPTIONS. The Benefit shall not apply if the Insured's death shall result, either directly or indirectly, from any one of the following causes: (1) Self-destruction or self-inflicted injuries, whether the Insured be sane or insane; (2) Bodily or mental infirmity or disease of any kind; (3) Poisoning or infection, other than infection occurring simultaneously with and in consequence of a cut or wound sustained in an accident; (4) Injuries of which there is no visible contusions or wound on the exterior of the body, drowning and internal injuries revealed by autopsy excepted; (5) Any injuries received (a) while on police duty in any military, naval or police organization; (b) in anyriot, civil commotion, insurrection or war or any act incident thereto; (c) while travelling as a passenger or otherwise in any form of submarine transportation, or while engaging in submarine operations; (d) in any violation of the law by the Insured or assault provoked by the Insured; (e) that has beeninflicted intentionally by a third party, either with or without provocation on the part of the Insured, and whether or not the attack or the defense by the third party was caused by a violation of the law by the Insured;

(6) Operating or riding in or descending from any kind of aircraft if the Insured is a pilot, officer or member of the crew of the aircraft or is giving or receiving any kind of training or instruction or has any duties aboard the aircraft or requiring descent therefrom; and (7) Atomic energy explosion of any nature whatsoever. The Company, before making any payment under this Clause, shall have the right and opportunity to examine the body and make an autopsy thereof. AUTOMATIC DISCONTINUANCE. This Benefit shall automatically terminate and the additional premium therefor shall cease to be payable when and if: (1) This Policy is surrendered for cash, paid-up insurance or extended term insurance; or (2) The benefit under the Total and Permanent Disability Waiver of Premium Certificate is granted to the insured; or (3) The Insured engages in military, naval or aeronautic service in time of war; or (4) The policy anniversary immediately preceding the sixtieth birthday of the Insured is reached. 2 It is undisputed that, as recited in the lower court's decision, the insured met his death, as follows: "that on the night of May 20, 1964 or the first hours of May 21, 1964, while the said life policy and supplementary contract were in full force and effect, the house of insured Juan S. Biagtan was robbed by a band of robbers who were charged in and convicted by the Court of First Instance of Pangasinan for robbery with homicide; that in committing the robbery, the robbers, on reaching the staircase landing of the second floor, rushed towards the doors of the second floor room, where they suddenly met a person near the door of one of the rooms who turned out to be the insured Juan S. Biagtan who received thrust from their sharp-pointed instruments, causing wounds on the body of said Juan S. Biagtan resulting in his death at about 7 a.m. on the same day, May 21, 1964." 3 Defendant company, while admitting the above-recited circumstances under which the insured met his death, disclaimed liability under its accidental death benefit clause under paragraph 5 of its stipulated "Exceptions" on its theory that the insured's death resulted from injuries "intentionally inflicted by a third party," i.e. the robbers who broke into the insured's house and inflicted fatal injuries on him.

The case was submitted for decision upon the parties' stipulation of facts that (1) insurance companies such as the Lincoln National Life Insurance Co. and Sun Life Assurance Co. of Canada with which the deceased insured Juan S. Biagtan was also insured for much larger sums under similar contracts with accidental death benefit provisions have promptly paid the benefits thereunder to plaintiffs-beneficiaries; (2) the robbers who caused the insured's death were charged in and convicted by the Court of First Instance of Pangasinan for the crime of robbery with homicide; and (3) the injuries inflicted on the insured by the robbers consisted of five mortal and four non-mortal wounds. 4 The lower court thereafter rendered judgment against defendant, as follows: There is no doubt that the insured, Juan S. Biagtan, met his death as a result of the wounds inflicted upon him by the malefactors on the early morning of May 21, 1964 by means of thrusts from sharp-pointed instruments delivered upon his person, and there is likewise no question that the thrusts were made on the occasion of the robbery. However, it is defendants' position that the killing of the insured was intentionally done by the malefactors, who were charged with and convicted of the crime of robbery with homicide by the Court of First Instance of Pangasinan. It must be noted here that no evidence whatsoever was presented by the parties who submitted the case for resolution upon the stipulation of facts presented by them. Thus, the court does not have before it proof that the act of receiving thrust(s) from the sharp-pointed instrument of the robbers wasintended to inflict injuries upon the person of the insured or any other person or merely to scare away any person so as to ward off any resistance or obstacle that might be offered in the pursuit of their main objective which was robbery. It was held that where a provision of the policy excludes intentional injury, it is the intention of the person inflicting the injury that is controlling ... and to come within the exception, the act which causes the injury must be wholly intentional, not merely partly. The case at bar has some similarity with the case of Virginia Calanoc vs. Court of Appeals, et al., L-8151, promulgated December 16, 1965, where the Supreme Court ruled that "the shot (which killed the insured) was merely to scare away the people around for his own protection and not necessarily to kill or hit the victim." In the Calanoc case, one Melencio Basilio, a watchman of a certain company, took out life insurance from the Philippine American Life Insurance Company in the amount of P2,000.00 to which was attached a supplementary contract covering death by accident. Calanoc died of gunshot wounds on the occasion of a robbery committed in the house of a certain Atty. Ojeda in Manila. The insured's widow was paid P2,000.00, the face value of the policy, but when she demanded payment of the

additional sum of P2,000.00 representing the value of the supplemental policy, the company refused alleging, as main defense, that the deceased died because he was murdered by a person who took part in the commission of the robbery and while making an arrest as an officer of the law which contingencies were (as in this case) expressly excluded in the contract and have the effect of exempting the company from liability. The facts in the Calanoc case insofar as pertinent to this case are, as found by the Court of Appeals in its decision which findings of fact were adopted by the Supreme Court, as follows: "...that on the way to the Ojeda residence (which was then being robbed by armed men), the policeman and Atty. Ojeda passed by Basilio (the insured) and somehow or other invited the latter to come along; that as the three approached the Ojeda residence and stood in front of the main gate which was covered by galvanized iron, the fence itself being partly concrete and partly adobe stone, a shot was fired; ... that it turned out afterwards that the special watchman Melencio Basilio was hit in the abdomen, the wound causing his instantaneous death ..." The Court of Appeals arrived at the conclusion that the death of Basilio, although unexpected, was not caused by an accident, being a voluntary and intentional act on the part of the one who robbed, or one of those who robbed, the house of Atty. Ojeda. In reversing this conclusion of the Court of Appeals, the Supreme Court said in part: "... Nor can it be said that the killing was intentional for there is the possibility that the malefactors had fired the shot merely to scare away the people around for his own protection and not necessarily to kill or hit the victim. In any event, while the act may not exempt the triggerman from ability for the damage done, the fact remains that the happening was a pure accidentt on the part of the victim." With this ruling of the Supreme Court, and the utter absence of evidence in this case as to the real intention of the malefactors in making a thrust with their sharp-pointed instrument on any person, the victim in particular, the case falls squarely within the ruling in the Calanoc vs. Court of Appeals case. It is the considered view of this Court that the insured died because of an accident which happened on the occasion of the robbery being committed

in his house. His death was not sought (at least no evidence was presented to show it was), and therefore was fortuitous. "Accident" was defined as that which happens by chance or fortuitously, without intention or design, and which is unexpected, unusual and unforeseen, or that which takes place without one's foresight or expectation an event that proceeds from an unknown cause, or is an unusual effect of a known cause, and therefore not expected. (29 Am. Jur. 706). There is no question that the defense set up by the defendant company is one of those included among the risks excluded in the supplementary contract. However, there is no evidence here that the thrusts with sharppointed instrument (which led to the death of the insured) was "intentional," (sic) so as to exempt the company from liability. It could safely be assumed that it was purely accidental considering that the principal motive of the culprits was robbery, the thrusts being merely intended to scare away persons who might offer resistance or might obstruct them from pursuing their main objective which was robbery. 5 It is respectfully submitted that the lower court committed no error in law in holding defendant insurance company liable to plaintiffs-beneficiaries under its accidental death benefit clause, by virtue of the following considerations: 1. The case of Calanoc cited by the lower court is indeed controlling here. 6 This Court, there construing a similar clause, squarely ruled that fatal injuries inflicted upon an insured by a malefactor(s) during the latter's commission of a crime are deemed accidental and within the coverage of such accidental death benefit clauses and the burden of proving that the killing was intentional so as to have it fall within the stipulated exception of having resulted from injuries "intentionally inflicted by a third party" must be discharged by the insurance company. This Court there clearly held that in such cases where the killing does not amount to murder, it must be held to be a "pure accident" on the part of the victim, compensable with double-indemnity, even though the malefactor is criminally liable for his act. This Court rejected the insurance-company's contrary claim, thus: Much less can it be pretended that Basilio died in the course of an assault or murder considering the very nature of these crimes. In the first place, there is no proof that the death of Basilio is the result of either crime for the record is barren of any circumstance showing how the fatal shot was fired. Perhaps this may be clarified in the criminal case now pending in court a regards the incident but before that is done anything that might be said on the point would be a mere conjecture. Nor can it be said that the killing was intentional for there is the possibility that the malefactor had fired the shot merely to scare away the people around for his own protection and not necessarily to kill or hit the victim. In any event, while the act may not exempt the triggerman from liability for the damage done, the fact remains that the happening was a pure accident on the part of the

victim. The victim could have been either the policeman or Atty. Ojeda for it cannot be pretended that the malefactor aimed at the deceased precisely because he wanted to take his life. 7 2. Defendant company patently failed to discharge its burden of proving that the fatal injuries were inflicted upon the deceased intentionally, i.e. deliberately. The lower court correctly held that since the case was submitted upon the parties' stipulation of facts which did not cover the malefactors' intent at all, there was an "utter absence of evidence in this case as to the real intention of the malefactors in making a thrust with their sharp-pointed instrument(s) on any person, the victim in particular." From the undisputed facts, supra, 8 the robbers had "rushed towards the doors of the second floor room, where they suddenly met a person ... who turned out to be the insured Juan S. Biagtan who received thrusts from their pointed instruments." The thrusts were indeed properly termed "purely accidental" since they seemed to be a reflex action on the robbers' part upon their being surprised by the deceased. To argue, as defendant does, that the robbers' intent to kill must necessarily be deduced from the four mortal wounds inflicted upon the deceased is to beg the question. Defendant must suffer the consequences of its failure to discharge its burden of proving by competent evidence, e.g. the robbers' or eyewitnesses' testimony, that the fatal injuries were intentionally inflicted upon the insured so as to exempt itself from liability. 3. Furthermore, plaintiffs-appellees properly assert in their brief that the sole error assigned by defendant company, to wit, that the fatal injuries were not accidental as held by the lower court but should be held to have been intentionally inflicted, raises a question of fact which defendant is now barred from raising, since it expressly limited its appeal to this Court purely "on questions of law", per its noitice of appeal, 9 Defendant is therefore confined to "raising only questions of law" and "no other questions" under Rule 42, section 2 of the Rules of Court 10 and is deemed to have conceded the findings of fact of the trial court, since he thereby waived all questions of facts. 11 4. It has long been an established rule of construction of so-called contracts of adhesion such as insurance contracts, where the insured is handed a printed insurance policy whose fine-print language has long been selected with great care and deliberation by specialists and legal advisers employed by and acting exclusively in the interest of the insurance company, that the terms and phraseology of the policy, particularly of any exception clauses, must be clearly expressed so as to be easily understood by the insured and any "ambiguous, equivocal or uncertain terms" are to be "construed strictly and most strongly against the insurer and liberally in favor of the insured so as to effect the dominant purpose of indemnity or payment to the insured, especially where a forfeiture is involved. The Court so expressly held in Calanoc that: ... While as a general rule "the parties may limit the coverage of the policy to certain particular accidents and risks or causes of loss, and may

expressly except other risks or causes of loss therefrom" (45 C.J.S. 781782), however, it is to be desired that the terms and phraseology of the exception clause be clearly expressed so as to be within the easy grasp and understanding of the insured, for if the terms are doubtful or obscure the same must of necessity be interpreted or resolved against the one who has caused the obscurity. (Article 1377, new Civil Code) And so it has been generally held that the "terms in an insurance policy, which are ambiguous, equivocal, or uncertain ... are to be construed strictly and most strongly against the insurer, and liberally in favor of the insured so as to effect the dominant purpose of indemnity or payment to the insured, especially where a forfeiture is involved" (29 AM. Jur., 181), and the reason for this rule is that the "insured usually has no voice in the selection or arrangement of the words employed and that the language of the contract is selected with great care and deliberation by experts and legal advisers employed by, and acting exclusively in the interest of, the insurance company." (44 C.J.S., p. 1174) Insurance is, in its nature, complex and difficult for the layman to understand. Policies are prepared by experts who know and can anticipate the bearing and possible complications of every contingency. So long as insurance companies insist upon the use of ambiguous, intricate and technical provisions, which conceal rather than frankly disclose, their own intentions, the courts must, in fairness to those who purchase insurance construe every ambiguity in favor of the insured." (Algoe vs. Pacific Mut. L. Ins. Co., 91 Wash. 324 LRA 1917A, 1237.) "An insurer should not be allowed, by the use of obscure phrases and exceptions, to defeat the very purpose for which the policy was procured ." (Moore vs. Aetna Life Insurance Co., LRA 1915D, 164). 12 The Court has but recently reiterated this doctrine in Landicho vs. GSIS 13 and again applied the provisions of Article 1377 of our Civil Code that "The interpretation of obscure words or stipulations in a contract shall not favor the party who caused the obscurity." 5. The accidental death benefit clause assuring the insured's beneficiaries of double indemnity, upon payment of an extra premium, in the event that the insured meets violent accidental death is contractually stipulated as follows in the policy: "that the death of the insured resulted directly from bodily injury effected solely through external and violent means sustained in an accident," supra. The policy then lists numerous exceptions, which may be classified as follows: Injuries effected through non-external means which are excepted: self-destruction, bodily or mental infirmity or disease, poisoning or infection, injuries with no visible contusions or exterior wounds (exceptions 1 to 4 of policy clause);

Injuries caused by some act of the insured which is proscribed by the policy, and are therefore similarly exepted: injuries received while on police duty, while travelling in any form of submarine transportation, or in any violation of law by the insured or assault provoked by the insured, or in any aircraft if the insured is a pilot or crew member; [exceptions 5 (a), (c) and (d), and 6 of the policy clause]; and Accidents expressly excluded: where death resulted in any riot, civil commotion, insurrection or war or atomic energy explosion. (Exceptions 5[b] and 7 of policy clause). The only exception which is not susceptible of classification is that provided in paragraph 5 (e), the very exception herein involved, which would also except injuries "inflicted intentionally by a third party, either with or without provocation on the part of the insured, and whether or not the attack or the defense by the third party was caused by a violation of the law by the insured." This ambiguous clause conflicts with all the other four exceptions in the same paragraph 5 particularly that immediately preceding it in item (d) which excepts injuries received where the insured has violated the law or provoked the injury, while this clause, construed as the insurance company now claims, would seemingly except also all other injuries, intentionally inflicted by a third party, regardless of any violation of law or provocation by the insured, and defeat the very purpose of the policy of giving the insured double indemnity in case of accidental death by "external and violent means" in the very language of the policy." It is obvious from the very classification of the exceptions and applying the rule of noscitus a sociis that the double-indemnity policy covers the insured against accidental death, whether caused by fault, negligence or intent of a third party which is unforeseen and unexpected by the insured. All the associated words and concepts in the policy plainly exclude the accidental death from the coverage of the policy only where the injuries are self-inflicted or attended by some proscribed act of the insured or are incurred in some expressly excluded calamity such as riot, war or atomic explosion. Finally, the untenability of herein defendant insurer's claim that the insured's death fell within the exception is further heightened by the stipulated fact that two other insurance companies which likewise covered the insured for which larger sums under similar accidental death benefit clauses promptly paid the benefits thereof to plaintiffsbeneficiaries. I vote accordingly for the affirmance in toto of the appealed decision, with costs against defendant-appellant. Concepcion, C.J. and Reyes, J.B.L., J., concur.

Separate Opinions BARREDO, J., concurring During the deliberations in this case, I entertained some doubts as to the correctness and validity of the view upheld in the main opinion penned by Justice Makalintal. Further reflection has convinced me, however, that there are good reasons to support it. At first blush, one would feel that every death not suicidal should be considered accidental, for the purposes of an accident insurance policy or a life insurance policy with a double indemnity clause in case death results from accident. Indeed, it is quite logical to think that any event whether caused by fault, negligence, intent of a third party or any unavoidable circumstance, normally unforeseen by the insured and free from any possible connivance on his part, is an accident in the generally accepted sense of the term. And if I were convinced that in including in the policy the provision in question, both the insurer and the insured had in mind to exclude thereby from the coverage of the policy only suicide whether unhelped or helped somehow by a third party, I would disregard the American decisions cited and quoted in the main opinion as not even persuasive authorities. But examining the unequivocal language of the provision in controversy and considering that the insured accepted the policy without asking that it be made clear that the phrase "injury intentionally inflicted by a third party" should be understood to refer only to injuries inflicted by a third party without any wilful intervention on his part (of the insured) or, in other words, without any connivance with him (the insured) in order to augment the proceeds of the policy for his benificiaries, I am inclined to agree that death caused by criminal assault is not covered by the policies of the kind here in question, specially if the assault, as a matter of fact, could have been more or less anticipated, as when the insured happens to have violent enemies or is found in circumstances that would make his life fair game of third parties. As to the rest, I have no doubt that the killing of the insured in this case is as intentional as any intentional act can be, hence this concurrence. TEEHANKEE, J., dissenting: The sole issue at bar is the correctness in law of the lower court's appealed decision adjudging defendant insurance company liable, under its supplementary contract denominated "Accidental Death Benefit Clause" with the deceased insured, to plaintiffsbeneficiaries (excluding plaintiff Emilia T. Biagtan) in an additional amount of P5,000.00 (with corresponding legal interest) and ruling that defendant company had failed to present any evidence to substantiate its defense that the insured's death came within the stipulated exceptions. Defendant's accidental death benefit clause expressly provides: ACCIDENTAL DEATH BENEFIT. (hereinafter called the benefit). Upon receipt and approval of due proof that the death of the Insured resulted

directly from bodily injury effected solely through external and violent means sustained in an accident, within ninety days after the date of sustaining such injury, and independently of all other causes, this Company shall pay, in addition to the sum insured specified on the first page of this Policy, a further sum equal to said sum insured payable at the same time and in the same manner as said sum insured, provided, that such death occurred during the continuance of this Clause and of this Policy and before the sixtieth birthday of the Insured." 1 A long list of exceptions and an Automatic Discontinuance clause immediately follow thereafter, thus: EXCEPTIONS. The Benefit shall not apply if the Insured's death shall result, either directly or indirectly, from any one of the following causes: (1) Self-destruction or self-inflicted injuries, whether the Insured be sane or insane; (2) Bodily or mental infirmity or disease of any kind; (3) Poisoning or infection, other than infection occurring simultaneously with and in consequence of a cut or wound sustained in an accident; (4) Injuries of which there is no visible contusions or wound on the exterior of the body, drowning and internal injuries revealed by autopsy excepted; (5) Any injuries received (a) while on police duty in any military, naval or police organization; (b) in anyriot, civil commotion, insurrection or war or any act incident thereto; (c) while travelling as a passenger or otherwise in any form of submarine transportation, or while engaging in submarine operations; (d) in any violation of the law by the Insured or assault provoked by the Insured; (e) that has beeninflicted intentionally by a third party, either with or without provocation on the part of the Insured, and whether or not the attack or the defense by the third party was caused by a violation of the law by the Insured; (6) Operating or riding in or descending from any kind of aircraft if the Insured is a pilot, officer or member of the crew of the aircraft or is giving or receiving any kind of training or instruction or has any duties aboard the aircraft or requiring descent therefrom; and (7) Atomic energy explosion of any nature whatsoever.

The Company, before making any payment under this Clause, shall have the right and opportunity to examine the body and make an autopsy thereof. AUTOMATIC DISCONTINUANCE. This Benefit shall automatically terminate and the additional premium therefor shall cease to be payable when and if: (1) This Policy is surrendered for cash, paid-up insurance or extended term insurance; or (2) The benefit under the Total and Permanent Disability Waiver of Premium Certificate is granted to the insured; or (3) The Insured engages in military, naval or aeronautic service in time of war; or (4) The policy anniversary immediately preceding the sixtieth birthday of the Insured is reached. 2 It is undisputed that, as recited in the lower court's decision, the insured met his death, as follows: "that on the night of May 20, 1964 or the first hours of May 21, 1964, while the said life policy and supplementary contract were in full force and effect, the house of insured Juan S. Biagtan was robbed by a band of robbers who were charged in and convicted by the Court of First Instance of Pangasinan for robbery with homicide; that in committing the robbery, the robbers, on reaching the staircase landing of the second floor, rushed towards the doors of the second floor room, where they suddenly met a person near the door of one of the rooms who turned out to be the insured Juan S. Biagtan who received thrust from their sharp-pointed instruments, causing wounds on the body of said Juan S. Biagtan resulting in his death at about 7 a.m. on the same day, May 21, 1964." 3 Defendant company, while admitting the above-recited circumstances under which the insured met his death, disclaimed liability under its accidental death benefit clause under paragraph 5 of its stipulated "Exceptions" on its theory that the insured's death resulted from injuries "intentionally inflicted by a third party," i.e. the robbers who broke into the insured's house and inflicted fatal injuries on him. The case was submitted for decision upon the parties' stipulation of facts that (1) insurance companies such as the Lincoln National Life Insurance Co. and Sun Life Assurance Co. of Canada with which the deceased insured Juan S. Biagtan was also insured for much larger sums under similar contracts with accidental death benefit provisions have promptly paid the benefits thereunder to plaintiffs-beneficiaries; (2) the robbers who caused the insured's death were charged in and convicted by the Court of First Instance of Pangasinan for the crime of robbery with homicide; and (3) the injuries

inflicted on the insured by the robbers consisted of five mortal and four non-mortal wounds. 4 The lower court thereafter rendered judgment against defendant, as follows: There is no doubt that the insured, Juan S. Biagtan, met his death as a result of the wounds inflicted upon him by the malefactors on the early morning of May 21, 1964 by means of thrusts from sharp-pointed instruments delivered upon his person, and there is likewise no question that the thrusts were made on the occasion of the robbery. However, it is defendants' position that the killing of the insured was intentionally done by the malefactors, who were charged with and convicted of the crime of robbery with homicide by the Court of First Instance of Pangasinan. It must be noted here that no evidence whatsoever was presented by the parties who submitted the case for resolution upon the stipulation of facts presented by them. Thus, the court does not have before it proof that the act of receiving thrust(s) from the sharp-pointed instrument of the robbers wasintended to inflict injuries upon the person of the insured or any other person or merely to scare away any person so as to ward off any resistance or obstacle that might be offered in the pursuit of their main objective which was robbery. It was held that where a provision of the policy excludes intentional injury, it is the intention of the person inflicting the injury that is controlling ... and to come within the exception, the act which causes the injury must be wholly intentional, not merely partly. The case at bar has some similarity with the case of Virginia Calanoc vs. Court of Appeals, et al., L-8151, promulgated December 16, 1965, where the Supreme Court ruled that "the shot (which killed the insured) was merely to scare away the people around for his own protection and not necessarily to kill or hit the victim." In the Calanoc case, one Melencio Basilio, a watchman of a certain company, took out life insurance from the Philippine American Life Insurance Company in the amount of P2,000.00 to which was attached a supplementary contract covering death by accident. Calanoc died of gunshot wounds on the occasion of a robbery committed in the house of a certain Atty. Ojeda in Manila. The insured's widow was paid P2,000.00, the face value of the policy, but when she demanded payment of the additional sum of P2,000.00 representing the value of the supplemental policy, the company refused alleging, as main defense, that the deceased died because he was murdered by a person who took part in the commission of the robbery and while making an arrest as an officer of the law which contingencies were (as in this case) expressly excluded in the contract and have the effect of exempting the company from liability.

The facts in the Calanoc case insofar as pertinent to this case are, as found by the Court of Appeals in its decision which findings of fact were adopted by the Supreme Court, as follows: "...that on the way to the Ojeda residence (which was then being robbed by armed men), the policeman and Atty. Ojeda passed by Basilio (the insured) and somehow or other invited the latter to come along; that as the three approached the Ojeda residence and stood in front of the main gate which was covered by galvanized iron, the fence itself being partly concrete and partly adobe stone, a shot was fired; ... that it turned out afterwards that the special watchman Melencio Basilio was hit in the abdomen, the wound causing his instantaneous death ..." The Court of Appeals arrived at the conclusion that the death of Basilio, although unexpected, was not caused by an accident, being a voluntary and intentional act on the part of the one who robbed, or one of those who robbed, the house of Atty. Ojeda. In reversing this conclusion of the Court of Appeals, the Supreme Court said in part: "... Nor can it be said that the killing was intentional for there is the possibility that the malefactors had fired the shot merely to scare away the people around for his own protection and not necessarily to kill or hit the victim. In any event, while the act may not exempt the triggerman from ability for the damage done, the fact remains that the happening was a pure accidentt on the part of the victim." With this ruling of the Supreme Court, and the utter absence of evidence in this case as to the real intention of the malefactors in making a thrust with their sharp-pointed instrument on any person, the victim in particular, the case falls squarely within the ruling in the Calanoc vs. Court of Appeals case. It is the considered view of this Court that the insured died because of an accident which happened on the occasion of the robbery being committed in his house. His death was not sought (at least no evidence was presented to show it was), and therefore was fortuitous. "Accident" was defined as that which happens by chance or fortuitously, without intention or design, and which is unexpected, unusual and unforeseen, or that which takes place without one's foresight or expectation an event that proceeds from an unknown cause, or is an unusual effect of a known cause, and therefore not expected. (29 Am. Jur. 706).

There is no question that the defense set up by the defendant company is one of those included among the risks excluded in the supplementary contract. However, there is no evidence here that the thrusts with sharppointed instrument (which led to the death of the insured) was "intentional," (sic) so as to exempt the company from liability. It could safely be assumed that it was purely accidental considering that the principal motive of the culprits was robbery, the thrusts being merely intended to scare away persons who might offer resistance or might obstruct them from pursuing their main objective which was robbery. 5 It is respectfully submitted that the lower court committed no error in law in holding defendant insurance company liable to plaintiffs-beneficiaries under its accidental death benefit clause, by virtue of the following considerations: 1. The case of Calanoc cited by the lower court is indeed controlling here. 6 This Court, there construing a similar clause, squarely ruled that fatal injuries inflicted upon an insured by a malefactor(s) during the latter's commission of a crime are deemed accidental and within the coverage of such accidental death benefit clauses and the burden of proving that the killing was intentional so as to have it fall within the stipulated exception of having resulted from injuries "intentionally inflicted by a third party" must be discharged by the insurance company. This Court there clearly held that in such cases where the killing does not amount to murder, it must be held to be a "pure accident" on the part of the victim, compensable with double-indemnity, even though the malefactor is criminally liable for his act. This Court rejected the insurance-company's contrary claim, thus: Much less can it be pretended that Basilio died in the course of an assault or murder considering the very nature of these crimes. In the first place, there is no proof that the death of Basilio is the result of either crime for the record is barren of any circumstance showing how the fatal shot was fired. Perhaps this may be clarified in the criminal case now pending in court a regards the incident but before that is done anything that might be said on the point would be a mere conjecture. Nor can it be said that the killing was intentional for there is the possibility that the malefactor had fired the shot merely to scare away the people around for his own protection and not necessarily to kill or hit the victim. In any event, while the act may not exempt the triggerman from liability for the damage done, the fact remains that the happening was a pure accident on the part of the victim. The victim could have been either the policeman or Atty. Ojeda for it cannot be pretended that the malefactor aimed at the deceased precisely because he wanted to take his life. 7 2. Defendant company patently failed to discharge its burden of proving that the fatal injuries were inflicted upon the deceased intentionally, i.e. deliberately. The lower court correctly held that since the case was submitted upon the parties' stipulation of facts which did not cover the malefactors' intent at all, there was an "utter absence of

evidence in this case as to the real intention of the malefactors in making a thrust with their sharp-pointed instrument(s) on any person, the victim in particular." From the undisputed facts, supra, 8 the robbers had "rushed towards the doors of the second floor room, where they suddenly met a person ... who turned out to be the insured Juan S. Biagtan who received thrusts from their pointed instruments." The thrusts were indeed properly termed "purely accidental" since they seemed to be a reflex action on the robbers' part upon their being surprised by the deceased. To argue, as defendant does, that the robbers' intent to kill must necessarily be deduced from the four mortal wounds inflicted upon the deceased is to beg the question. Defendant must suffer the consequences of its failure to discharge its burden of proving by competent evidence, e.g. the robbers' or eyewitnesses' testimony, that the fatal injuries were intentionally inflicted upon the insured so as to exempt itself from liability. 3. Furthermore, plaintiffs-appellees properly assert in their brief that the sole error assigned by defendant company, to wit, that the fatal injuries were not accidental as held by the lower court but should be held to have been intentionally inflicted, raises a question of fact which defendant is now barred from raising, since it expressly limited its appeal to this Court purely "on questions of law", per its noitice of appeal, 9 Defendant is therefore confined to "raising only questions of law" and "no other questions" under Rule 42, section 2 of the Rules of Court 10 and is deemed to have conceded the findings of fact of the trial court, since he thereby waived all questions of facts. 11 4. It has long been an established rule of construction of so-called contracts of adhesion such as insurance contracts, where the insured is handed a printed insurance policy whose fine-print language has long been selected with great care and deliberation by specialists and legal advisers employed by and acting exclusively in the interest of the insurance company, that the terms and phraseology of the policy, particularly of any exception clauses, must be clearly expressed so as to be easily understood by the insured and any "ambiguous, equivocal or uncertain terms" are to be "construed strictly and most strongly against the insurer and liberally in favor of the insured so as to effect the dominant purpose of indemnity or payment to the insured, especially where a forfeiture is involved. The Court so expressly held in Calanoc that: ... While as a general rule "the parties may limit the coverage of the policy to certain particular accidents and risks or causes of loss, and may expressly except other risks or causes of loss therefrom" (45 C.J.S. 781782), however, it is to be desired that the terms and phraseology of the exception clause be clearly expressed so as to be within the easy grasp and understanding of the insured, for if the terms are doubtful or obscure the same must of necessity be interpreted or resolved against the one who has caused the obscurity. (Article 1377, new Civil Code) And so it has been generally held that the "terms in an insurance policy, which are ambiguous, equivocal, or uncertain ... are to be construed strictly and

most strongly against the insurer, and liberally in favor of the insured so as to effect the dominant purpose of indemnity or payment to the insured, especially where a forfeiture is involved" (29 AM. Jur., 181), and the reason for this rule is that the "insured usually has no voice in the selection or arrangement of the words employed and that the language of the contract is selected with great care and deliberation by experts and legal advisers employed by, and acting exclusively in the interest of, the insurance company." (44 C.J.S., p. 1174) Insurance is, in its nature, complex and difficult for the layman to understand. Policies are prepared by experts who know and can anticipate the bearing and possible complications of every contingency. So long as insurance companies insist upon the use of ambiguous, intricate and technical provisions, which conceal rather than frankly disclose, their own intentions, the courts must, in fairness to those who purchase insurance construe every ambiguity in favor of the insured." (Algoe vs. Pacific Mut. L. Ins. Co., 91 Wash. 324 LRA 1917A, 1237.) "An insurer should not be allowed, by the use of obscure phrases and exceptions, to defeat the very purpose for which the policy was procured ." (Moore vs. Aetna Life Insurance Co., LRA 1915D, 164). 12 The Court has but recently reiterated this doctrine in Landicho vs. GSIS 13 and again applied the provisions of Article 1377 of our Civil Code that "The interpretation of obscure words or stipulations in a contract shall not favor the party who caused the obscurity." 5. The accidental death benefit clause assuring the insured's beneficiaries of double indemnity, upon payment of an extra premium, in the event that the insured meets violent accidental death is contractually stipulated as follows in the policy: "that the death of the insured resulted directly from bodily injury effected solely through external and violent means sustained in an accident," supra. The policy then lists numerous exceptions, which may be classified as follows: Injuries effected through non-external means which are excepted: self-destruction, bodily or mental infirmity or disease, poisoning or infection, injuries with no visible contusions or exterior wounds (exceptions 1 to 4 of policy clause); Injuries caused by some act of the insured which is proscribed by the policy, and are therefore similarly exepted: injuries received while on police duty, while travelling in any form of submarine transportation, or in any violation of law by the insured or assault provoked by the insured, or in any aircraft if the insured is a pilot or crew member; [exceptions 5 (a), (c) and (d), and 6 of the policy clause]; and Accidents expressly excluded: where death resulted in any riot, civil commotion, insurrection or war or atomic energy explosion. (Exceptions 5[b] and 7 of policy clause).

The only exception which is not susceptible of classification is that provided in paragraph 5 (e), the very exception herein involved, which would also except injuries "inflicted intentionally by a third party, either with or without provocation on the part of the insured, and whether or not the attack or the defense by the third party was caused by a violation of the law by the insured." This ambiguous clause conflicts with all the other four exceptions in the same paragraph 5 particularly that immediately preceding it in item (d) which excepts injuries received where the insured has violated the law or provoked the injury, while this clause, construed as the insurance company now claims, would seemingly except also all other injuries, intentionally inflicted by a third party, regardless of any violation of law or provocation by the insured, and defeat the very purpose of the policy of giving the insured double indemnity in case of accidental death by "external and violent means" in the very language of the policy." It is obvious from the very classification of the exceptions and applying the rule of noscitus a sociis that the double-indemnity policy covers the insured against accidental death, whether caused by fault, negligence or intent of a third party which is unforeseen and unexpected by the insured. All the associated words and concepts in the policy plainly exclude the accidental death from the coverage of the policy only where the injuries are self-inflicted or attended by some proscribed act of the insured or are incurred in some expressly excluded calamity such as riot, war or atomic explosion. Finally, the untenability of herein defendant insurer's claim that the insured's death fell within the exception is further heightened by the stipulated fact that two other insurance companies which likewise covered the insured for which larger sums under similar accidental death benefit clauses promptly paid the benefits thereof to plaintiffsbeneficiaries. I vote accordingly for the affirmance in toto of the appealed decision, with costs against defendant-appellant. Concepcion, C.J. and Reyes, J.B.L., J., concur.

IV.

Parties to the Contract


May 25, 1951

G.R. No. L-2294

FILIPINAS COMPAIA DE SEGUROS, petitioner, vs. CHRISTERN, HUENEFELD and CO., INC., respondent. Ramirez and Ortigas for petitioner. Ewald Huenefeld for respondent. PARAS, C.J.: On October 1, 1941, the respondent corporation, Christern Huenefeld, & Co., Inc., after payment of corresponding premium, obtained from the petitioner ,Filipinas Cia. de Seguros, fire policy No. 29333 in the sum of P1000,000, covering merchandise contained in a building located at No. 711 Roman Street, Binondo Manila. On February 27, 1942, or during the Japanese military occupation, the building and insured merchandise were burned. In due time the respondent submitted to the petitioner its claim under the policy. The salvage goods were sold at public auction and, after deducting their value, the total loss suffered by the respondent was fixed at P92,650. The petitioner refused to pay the claim on the ground that the policy in favor of the respondent had ceased to be in force on the date the United States declared war against Germany, the respondent Corporation (though organized under and by virtue of the laws of the Philippines) being controlled by the German subjects and the petitioner being a company under American jurisdiction when said policy was issued on October 1, 1941. The petitioner, however, in pursuance of the order of the Director of Bureau of Financing, Philippine Executive Commission, dated April 9, 1943, paid to the respondent the sum of P92,650 on April 19, 1943. The present action was filed on August 6, 1946, in the Court of First Instance of Manila for the purpose of recovering from the respondent the sum of P92,650 above mentioned. The theory of the petitioner is that the insured merchandise were burned up after the policy issued in 1941 in favor of the respondent corporation has ceased to be effective because of the outbreak of the war between the United States and Germany on December 10, 1941, and that the payment made by the petitioner to the respondent corporation during the Japanese military occupation was under pressure. After trial, the Court of First Instance of Manila dismissed the action without pronouncement as to costs. Upon appeal to the Court of Appeals, the judgment of the Court of First Instance of Manila was affirmed, with costs. The case is now before us on appeal by certiorari from the decision of the Court of Appeals. The Court of Appeals overruled the contention of the petitioner that the respondent corporation became an enemy when the United States declared war against Germany, relying on English and American cases which held that a corporation is a citizen of the country or state by and under the laws of which it was created or organized. It rejected

the theory that nationality of private corporation is determine by the character or citizenship of its controlling stockholders. There is no question that majority of the stockholders of the respondent corporation were German subjects. This being so, we have to rule that said respondent became an enemy corporation upon the outbreak of the war between the United States and Germany. The English and American cases relied upon by the Court of Appeals have lost their force in view of the latest decision of the Supreme Court of the United States in Clark vs. Uebersee Finanz Korporation, decided on December 8, 1947, 92 Law. Ed. Advance Opinions, No. 4, pp. 148-153, in which the controls test has been adopted. In "Enemy Corporation" by Martin Domke, a paper presented to the Second International Conference of the Legal Profession held at the Hague (Netherlands) in August. 1948 the following enlightening passages appear: Since World War I, the determination of enemy nationality of corporations has been discussion in many countries, belligerent and neutral. A corporation was subject to enemy legislation when it was controlled by enemies, namely managed under the influence of individuals or corporations, themselves considered as enemies. It was the English courts which first the Daimler case applied this new concept of "piercing the corporate veil," which was adopted by the peace of Treaties of 1919 and the Mixed Arbitral established after the First World War. The United States of America did not adopt the control test during the First World War. Courts refused to recognized the concept whereby American-registered corporations could be considered as enemies and thus subject to domestic legislation and administrative measures regarding enemy property. World War II revived the problem again. It was known that German and other enemy interests were cloaked by domestic corporation structure. It was not only by legal ownership of shares that a material influence could be exercised on the management of the corporation but also by long term loans and other factual situations. For that reason, legislation on enemy property enacted in various countries during World War II adopted by statutory provisions to the control test and determined, to various degrees, the incidents of control. Court decisions were rendered on the basis of such newly enacted statutory provisions in determining enemy character of domestic corporation. The United States did not, in the amendments of the Trading with the Enemy Act during the last war, include as did other legislations the applications of the control test and again, as in World War I, courts refused to apply this concept whereby the enemy character of an American or neutral-registered corporation is determined by the enemy nationality of the controlling stockholders. Measures of blocking foreign funds, the so called freezing regulations, and other administrative practice in the treatment of foreign-owned property in the United

States allowed to large degree the determination of enemy interest in domestic corporations and thus the application of the control test. Court decisions sanctioned such administrative practice enacted under the First War Powers Act of 1941, and more recently, on December 8, 1947, the Supreme Court of the United States definitely approved of the control theory. In Clark vs. Uebersee Finanz Korporation, A. G., dealing with a Swiss corporation allegedly controlled by German interest, the Court: "The property of all foreign interest was placed within the reach of the vesting power (of the Alien Property Custodian) not to appropriate friendly or neutral assets but to reach enemy interest which masqueraded under those innocent fronts. . . . The power of seizure and vesting was extended to all property of any foreign country or national so that no innocent appearing device could become a Trojan horse." It becomes unnecessary, therefore, to dwell at length on the authorities cited in support of the appealed decision. However, we may add that, in Haw Pia vs. China Banking Corporation,* 45 Off Gaz., (Supp. 9) 299, we already held that China Banking Corporation came within the meaning of the word "enemy" as used in the Trading with the Enemy Acts of civilized countries not only because it was incorporated under the laws of an enemy country but because it was controlled by enemies. The Philippine Insurance Law (Act No. 2427, as amended,) in section 8, provides that "anyone except a public enemy may be insured." It stands to reason that an insurance policy ceases to be allowable as soon as an insured becomes a public enemy. Effect of war, generally. All intercourse between citizens of belligerent powers which is inconsistent with a state of war is prohibited by the law of nations. Such prohibition includes all negotiations, commerce, or trading with the enemy; all acts which will increase, or tend to increase, its income or resources; all acts of voluntary submission to it; or receiving its protection; also all acts concerning the transmission of money or goods; and all contracts relating thereto are thereby nullified. It further prohibits insurance upon trade with or by the enemy, upon the life or lives of aliens engaged in service with the enemy; this for the reason that the subjects of one country cannot be permitted to lend their assistance to protect by insurance the commerce or property of belligerent, alien subjects, or to do anything detrimental too their country's interest. The purpose of war is to cripple the power and exhaust the resources of the enemy, and it is inconsistent that one country should destroy its enemy's property and repay in insurance the value of what has been so destroyed, or that it should in such manner increase the resources of the enemy, or render it aid, and the commencement of war determines, for like reasons, all trading intercourse with the enemy, which prior thereto may have been lawful. All individuals therefore, who compose the belligerent powers, exist, as to each other, in a state of utter exclusion, and are public enemies. (6 Couch, Cyc. of Ins. Law, pp. 5352-5353.) In the case of an ordinary fire policy, which grants insurance only from year, or for some other specified term it is plain that when the parties become alien

enemies, the contractual tie is broken and the contractual rights of the parties, so far as not vested. lost. (Vance, the Law on Insurance, Sec. 44, p. 112.) The respondent having become an enemy corporation on December 10, 1941, the insurance policy issued in its favor on October 1, 1941, by the petitioner (a Philippine corporation) had ceased to be valid and enforcible, and since the insured goods were burned after December 10, 1941, and during the war, the respondent was not entitled to any indemnity under said policy from the petitioner. However, elementary rules of justice (in the absence of specific provision in the Insurance Law) require that the premium paid by the respondent for the period covered by its policy from December 11, 1941, should be returned by the petitioner. The Court of Appeals, in deciding the case, stated that the main issue hinges on the question of whether the policy in question became null and void upon the declaration of war between the United States and Germany on December 10, 1941, and its judgment in favor of the respondent corporation was predicated on its conclusion that the policy did not cease to be in force. The Court of Appeals necessarily assumed that, even if the payment by the petitioner to the respondent was involuntary, its action is not tenable in view of the ruling on the validity of the policy. As a matter of fact, the Court of Appeals held that "any intimidation resorted to by the appellee was not unjust but the exercise of its lawful right to claim for and received the payment of the insurance policy," and that the ruling of the Bureau of Financing to the effect that "the appellee was entitled to payment from the appellant was, well founded." Factually, there can be no doubt that the Director of the Bureau of Financing, in ordering the petitioner to pay the claim of the respondent, merely obeyed the instruction of the Japanese Military Administration, as may be seen from the following: "In view of the findings and conclusion of this office contained in its decision on Administrative Case dated February 9, 1943 copy of which was sent to your office and the concurrence therein of the Financial Department of the Japanese Military Administration, and following the instruction of said authority, you are hereby ordered to pay the claim of Messrs. Christern, Huenefeld & Co., Inc. The payment of said claim, however, should be made by means of crossed check." (Emphasis supplied.) It results that the petitioner is entitled to recover what paid to the respondent under the circumstances on this case. However, the petitioner will be entitled to recover only the equivalent, in actual Philippines currency of P92,650 paid on April 19, 1943, in accordance with the rate fixed in the Ballantyne scale. Wherefore, the appealed decision is hereby reversed and the respondent corporation is ordered to pay to the petitioner the sum of P77,208.33, Philippine currency, less the amount of the premium, in Philippine currency, that should be returned by the petitioner for the unexpired term of the policy in question, beginning December 11, 1941. Without costs. So ordered. Feria, Pablo, Bengzon, Tuason, Montemayor, Jugo and Bautista Angelo, JJ., concur

G.R. No. L-14300

January 19, 1920

SAN MIGUEL BREWERY, ETC., plaintiff-appellee, vs. LAW UNION AND ROCK INSURANCE CO., (LTD.) ET AL., defendants-appellees. HENRY HARDING, defendant-appellant. Crossfield and O'Brien for appellant Harding. Lawrence and Ross for appellee Law Union etc. Ins. Co. Sanz and Luzuriaga for appellee "Filipinas, Compaia de Seguros." No appearance for the other appellee. STREET, J.: This action was begun on October 8, 1917, in the Court of First Instance of the city of Manila by the plaintiff, theSan Miguel Brewery, for the purpose of recovering upon two policies of insurance underwritten respectively by Law Union and Rock Insurance Company (Ltd.), and the "Filipinas" Compania de Seguros , for the sum of P7,500 each, insuring certain property which has been destroyed by fire. The plaintiff, the San Miguel Brewery, is named as the party assured in the two policies referred to, but it is alleged in the complaint that said company was in reality interested in the property which was the subject of insurance in the character of a mortgage creditor only, and that the owner of said property upon the date the policies were issued was one D. P. Dunn who was later succeeded as owner by one Henry Harding. Accordingly said Harding was made a defendant, as a person interested in the subject of the litigation. The prayer of the complaint is that judgment be entered in favor of the plaintiff against the two companies named for the sum of P15,000, with interest and costs, and further that upon satisfaction of the balance of P4,505.30 due to the plaintiff upon the mortgage debt, and upon the cancellation of the mortgage, the plaintiff be absolved from liability to the defendants or any of them. The peculiar form of the latter part of the prayer is evidently due to the design of the plaintiff to lay a foundation for Harding to recover the difference between the plaintiff's credit and the amount for which the property was insured. Accordingly, as was to be expected, Harding answered, admitting the material allegations of the complaint and claiming for himself the right to recover the difference between the plaintiff's mortgage credit and the face value of the policies. The two insurance companies also answered, admitting in effect their liability to the San Miguel Brewery to the extent of its mortgage credit, but denying liability to Harding on the ground that under the contracts of insurance the liability of the insurance companies was limited to the insurable interest of the plaintiff therein. Soon after the action was begun the insurance companies effected a settlement with the San Miguel Brewery by paying the full amount of the credit claimed by it, with the result that the litigation as between the original plaintiff and the two insurance companies came to an end, leaving the action to be prosecuted to final judgement by the defendant Harding with respect to the balance claimed to be due to him upon the policies.

Upon hearing the evidence the trial judge came to the conclusion that Harding had no right of action whatever against the companies and absolved them from liability without special finding as to costs. From this decision the said Henry Harding has appealed. The two insurance companies who are named as defendants do not dispute their liability to the San Miguel Brewery, to the extent already stated, and the only question here under discussion is that of the liability of the insurance companies to Harding. It is therefore necessary to take account of such facts only as bear upon this aspect of the case. In this connection it appears that on January 12, 1916, D. P. Dunn, then the owner of the property to which the insurance relates, mortgaged the same to the San Miguel Brewery to secure a debt of P10,000. In the contract of mortgage Dunn agreed to keep the property insured at his expense to the full amount of its value in companies to be selected by the Brewery Company and authorized the latter in case of loss to receive the proceeds of the insurance and to retain such part as might be necessary to cover the mortgage debt. At the same time, in order more conveniently to accomplish the end in view, Dunn authorized and requested the Brewery Company to effect said insurance itself. Accordingly on the same date Antonio Brias, general manager of the Brewery, made a verbal application to the Law Union and Rock Insurance Company for insurance to the extent of P15,000 upon said property. In reply to a question of the company's agent as to whether the Brewery was the owner of the property, he stated that the company was interested only as a mortgagee. No information was asked as to who was the owner of the property, and no information upon this point was given. It seems that the insurance company to whom this application was directed did not want to carry more than one-half the risk. It therefore issued its own policy for P7,500 and procured a policy in a like amount to be issued by the "Filipinas" Compania de Seguros. Both policies were issued in the name of the San Miguel Brewery as the assured, and contained no reference to any other interest in the property. Both policies contain the usual clause requiring assignments to be approved and noted on the policy. The premiums were paid by the Brewery and charged to Dunn. A year later the policies were renewed, without change, the renewal premiums being paid by the Brewery, supposedly for the account of the owner. In the month of March of the year 1917 Dunn sold the insured property to the defendant Henry Harding, but not assignment of the insurance, or of the insurance policies, was at any time made to him. We agree with the trial court that no cause of action in Henry Harding against the insurance companies is show. He is not a party to the contracts of insurance and cannot directly maintain an action thereon. (Uy Tam and Uy Yetvs. Leonard, 30 Phil. Rep., 471.) His claim is merely of an equitable and subsidiary nature and must be made effective, if at all, through the San Miguel Brewery in whose name the contracts are written. Now the Brewery, as mortgagee of the insured property, undoubtedly had an insurable interest therein; but it could not, in any event, recover upon these policies an amount in excess of its mortgage credit. In this connection it will be remembered that Antonio Brias, upon making application for the insurance, informed the company with

which the insurance was placed that the Brewery was interested only as a mortgagee. It would, therefore, be impossible for the Brewery mortgage on the insured property. This conclusion is not only deducible from the principles governing the operation and effect of insurance contracts in general but the point is clearly covered by the express provisions of sections 16 and 50 of the Insurance Act (Act No. 2427). In the first of the sections cited, it is declared that "the measure of an insurable interest in property is the extent to which the insured might be damnified by loss or injury thereof" (sec. 16); while in the other it is stated that "the insurance shall be applied exclusively to the proper interest of the person in whose name it is made unless otherwise specified in the policy" (sec. 50). These provisions would have been fatal to any attempt at recovery even by D. P. Dunn, if the ownership of the property had continued in him up to the time of the loss; and as regards Harding, an additional insuperable obstacle is found in the fact that the ownership of the property had been charged, prior to the loss, without any corresponding change having been effected in the policy of insurance. In section 19 of the Insurance Act we find it stated that "a change of interest in any part of a thing insured unaccompanied by a corresponding change of interest in the insurance, suspends the insurance to an equivalent extent, until the interest in the thing and the interest in the insurance are vested in the same person." Again in section 55 it is declared that "the mere transfer of a thing insured does not transfer the policy, but suspends it until the same person becomes the owner of both the policy and the thing insured." Undoubtedly these policies of insurance might have been so framed as to have been "payable to the Sane Miguel Brewery, mortgagee, as its interest may appear, remainder to whomsoever, during the continuance of the risk, may become the owner of the interest insured." (Sec 54, Act No. 2427.) Such a clause would have proved an intention to insure the entire interest in the property, not merely the insurable interest of the San Miguel Brewery, and would have shown exactly to whom the money, in case of loss, should be paid. But the policies are not so written. It is easy to collect from the facts stated in the decision of the trial judge, no less than from the testimony of Brias, the manager of the San Miguel Brewery, that, as the insurance was written up, the obligation of the insurance companies was different from that contemplated by Dunn, at whose request the insurance was written, and Brias. In the contract of mortgage Dunn had agreed, at his own expense, to insure the mortgaged property for its full value and to indorse the policies in such manner as to authorize the Brewery Company to receive the proceeds in case of loss and to retain such part thereof as might be necessary to satisfy the remainder then due upon the mortgage debt. Instead, however, of effecting the insurance himself Dunn authorized and requested the Brewery Company to procure insurance on the property in the amount of P15,000 at Dunn's expense. The Brewery Company undertook to carry this mandate into effect, and it of course became its duty to procure insurance of the character contemplated, that is, to have the policies so written as to protect not only the

insurable interest of the Brewery, but also the owner. Brias seems to have supposed that the policies as written had this effect, but in this he was mistaken. It was certainly a hardship on the owner to be required to pay the premiums upon P15,000 of insurance when he was receiving no benefit whatever except in protection to the extent of his indebtedness to the Brewery. The blame for the situation thus created rests, however, with the Brewery rather than with the insurance companies, and there is nothing in the record to indicate that the insurance companies were requested to write insurance upon the insurable interest of the owner or intended to make themselves liable to that extent. If during the negotiations which resulted in the writing of this insurance, it had been agreed between the contracting parties that the insurance should be so written as to protect not only the interest of the mortgagee but also the residuary interest of the owner, and the policies had been, by inadvertence, ignorance, or mistake written in the form in which they were issued, a court would have the power to reform the contracts and give effect to them in the sense in which the parties intended to be bound. But in order to justify this, it must be made clearly to appear that the minds of the contracting parties did actually meet in agreement and that they labored under some mutual error or mistake in respect to the expression of their purpose. Thus, in Bailey vs. American Central Insurance Co. (13 Fed., 250), it appeared that a mortgage desiring to insure his own insurable interest only, correctly stated his interest, and asked that the same be insured. The insurance company agreed to accept the risk, but the policy was issued in the name of the owner, because of the mistaken belief of the company's agent that the law required it to be so drawn. It was held that a court of equity had the power, at the suit of the mortgage, to reform the instrument and give judgment in his favor for the loss thereunder, although it had been exactly as it was. Said the court: "If the applicant correctly states his interest and distinctly asks for an insurance thereon, and the agent of the insurer agrees to comply with his request, and assumes to decide upon the form of the policy to be written for that purpose, and by mistake of law adopts the wrong form, a court of equity will reform the instrument so as to make it insurance upon the interest named." (See also Fink vs. Queens Insurance Co., 24 Fed., 318; Esch vs. Home Insurance Co., 78 Iowa, 334; 16 Am. St. Rep., 443; Woodbury Savings etc., Co., vs.Charter Oak Insurance Co., 31 Conn., 517; Balen vs. Hanover Fire Insurance Co., 67 Mich., 179.) Similarly, in cases where the mortgage is by mistake described as owner, the court may grant reformation and permit a recovery by the mortgage in his character as such. (Dalton vs. Milwaukee etc. Insurance Co., 126 Iowa, 377; Spare vs. Home Mutual Insurance Co., 17 Fed., 568.) In Thompson vs. Phoenix Insurance Co. (136 U.S., 287; 34 L. 3d., 408), it appeared that one Kearney made application to an insurance company for insurance on certain property in his hands as receiver and it was understood between him and the company's agent that, in case of loss, the proceeds of the policy should accrue to him and his successors as receiver and to others whom it might concern. However, the policy, as issued, was so worded as to be payable only to him as receiver. In an action brought on the policy by a successor of Kearney, it was alleged that the making of the contract in this form was due to inadvertence, accident, and mistake upon the part of both Kearney and the company.

Said the court: If by inadvertence, accident, or mistake the terms of the contract were not fully set forth in the policy, the plaintiff is entitled to have it reformed. In another case the same court said: We have before us a contract from which by mistake, material stipulations have been omitted, whereby the true intent and meaning of the parties are not fully or accurately expressed. There was a definite concluded agreement as to insurance, which, in point of time, preceded the preparation and delivery of the policy, and this is demonstrated by legal and exact evidence, which removes all doubt as to the sense and undertaking of the parties. In the agreement there has been a mutual mistake, caused chiefly by that contracting party who now seeks to limit the insurance to an interest in the property less than that agreed to be insured. The written agreement did not effect that which the parties intended. That a court of equity can afford relief in such a case, is, we think, well settled by the authorities. (Smell vs. Atlantic, etc., Ins. Co., 98 U.S., 85, 89; 25 L. ed., 52.) But to justify the reformation of a contract, the proof must be of the most satisfactory character, and it must clearly appear that the contract failed to express the real agreement between the parties. (Philippine Sugar Estates Development Company vs. Government of the Philippine Islands, 62 L. ed., 1177, reversing Government of Philippine Island vs. Philippine Sugar Estates Development Co., 30 Phil. Rep., 27.) In the case now before us the proof is entirely insufficient to authorize the application of the doctrine state in the foregoing cases, for it is by means clear from the testimony of Brias and none other was offered that the parties intended for the policy to cover the risk of the owner in addition to that of the mortgagee. It results that the defendant Harding is not entitled to relief in any aspect of the case. The judgment is therefore affirmed, with costs against the appellant. So ordered.

G.R. No. L-15184

May 31, 1963

SAURA IMPORT & EXPORT CO., INC., plaintiff-appellant, vs. PHILIPPINE INTERNATIONAL SURETY CO., INC., and PHILIPPINE NATIONAL BANK, defendants-appellees. Saura, Magno & Associates for plaintiff-appellant. Tolentino, Garcia and D. R. Cruz for defendant-appellee Philippine International Surety Co., Inc. Ramon B. de los Reyes and Antonio P. Cruz for defendant-appellee Philippine National Bank. PAREDES, J.: Instant case was certified by the Court of Appeals to Us, it appearing that the issues involved are purely of law. On December 26, 1952, the Saura Import & Export Co Inc., mortgaged to the Phil. National Bank, a parcel of land covered by T.C.T. No. 40445 of the Registry of Deeds of Davao, issued in its name, to secure the payment ofpromissory note of P27,000.00 (Exhs. P, B-2). On April 30, 1953, the mortgage was amended to guarantee an increased amount, bringing the total mortgaged debt to P37,000.00 (Exhs. P-2, B-3). The provisions of the mortgaged contact, pertinent to the resolution of the present case, provide as follows 2. . . . he shall insure the mortgaged property at all times against fire and earthquake for an amount and with such company satisfactory to the Mortgagee, indorsing to the latter the corresponding policies; he shall keep the mortgaged property in good condition, making repairs and protecting walls that may be necessary; . . . xxx xxx xxx

Erected on the land mortgaged, was a building of strong materials owned by the mortgagor Saura Import & Export Co., Inc., which had always been covered by insurance, many years prior to the mortgage contract. Pursuant to the requirement, Saura insured the building and its contents with the Philippine International Surety, an insurance firm acceptable to mortgagee Bank, for P29,000.00 against fire for the period of one year from October 2, 1954. As required therefor, the insurance policy was endorsed to the mortgagee PNB, in a Memo which states Loss if any, payable to the Philippine National Bank as their interest may appear, subject to the terms, conditions and warranties of this policy (Exh. A).

The policy was delivered to the mortgagee Bank by Saura. On October 15, 1954, barely thirteen (13) days after the issuance of the fire insurance policy (October 2, 1954), the insurer cancelled the same, effective as of the date of issue (Exh. A-2). Notice of the cancellation was given to appellee bank in writing, sent by Registered Mail and personally addressed to Fortunato Domingo, Branch Manager of the appellee Bank's Davao Branch, and was received by the Bank on November 8, 1954. On April 6, 1955, the building and its contents, worth P40,685.69 were burned. On April 11, 1955, Saura filed a claim with the Insurer and mortgagee Bank. Upon the presentation of notice of loss with the PNB, Saura learned for the first time that the policy had previously been cancelled on October 2, 1954, by the insurer, when Saura's folder in the Bank's filed was opened and the notice of cancellation (original and duplicate) sent by the Insurer to the Bank, was found. Upon refusal of the Insurer Philippine International Surety to pay the amount of the insurance, Civil Case No. 26847 was filed with the Manila CFI against the Insurer, and the PNB was later included as party defendant, after it had refused to prosecute the case jointly with Saura Import & Export Co., Inc. At the trial, it was established that neither the Insurer nor the mortgagee Bank informed the plaintiff Saura of the cancellation of the policy. On April 30, 1957, the court a quo rendered the following judgment . . . IN VIEW WHEREOF, complaint dismissed; costs against the plaintiff; but as there is no proof on the counterclaim of the Philippines International Surety, the same is also dismissed. Wherefore, the parties respectfully pray that the foregoing stipulation of facts be admitted and approved by this Honorable Court, without prejudice to the parties adducing other evidence to prove their case not covered by this stipulation of facts. 1wph1.t A motion to reconsider the above judgment, seasonably presented on May 14, 1957, was subsequently denied. The decision rendered and the resolution denying the motion for reconsideration constitute the subject of the instant appeal by plaintiff Saura on the three alleged errors, which converge on the correctness of the ruling, wholly dismissing the complaint absolving both the insurance company and the bank from liability. In the determination of liabilities of the parties herein, let us look into the general principles of insurance, in matters of cancellations of policy by the insurer. Fire insurance policies and other contracts of insurance upon property, in addition to the common provision for cancellation of the policy upon request of the insured, generally provide for cancellation by the insurer by notice to the insured for a prescribed period, which is usually 5 days, and the return of the unearned portion of the premium paid by the insured, such provision for cancellation upon notice being authorized by statutes in some jurisdiction, either specifically or as a provision of an adopted standard form of policy. The purpose of provisions or stipulations for notice to the insured, is to prevent the cancellation of the policy, without allowing the insured ample opportunity to negotiate for other insurance in its stead. The form and sufficiency of a notice of

cancellation is determined by policy provisions. In order to form the basis for the cancellation of a policy, notice to the insured n not be in any particular form, in the absence of a statute or policy provision prescribing such form, and it is sufficient, so long as it positively and unequivocally indicates to the insured, that it is the intention of the company that the policy shall cease to be binding. Where the policy contains no provisions that a certain number of days notice shall be given, a reasonable notice and opportunity to obtain other insurance must be given. Actual personal notice to the insured is essential to a cancellation under a provision for cancellation by notice. The actual receipt by the insured of a notice of cancellation is universally recognized as a condition precedent to a cancellation of the policy by the insurer, and consequently a letter containing notice of cancellation which is mailed by the insurer but not received by the insured, is ineffective as cancellation (29 Am. Jur. pp. 732-741). The policy in question (Exh. A), does not provide for the notice, its form or period. The Insurance Law, Act No. 2427, does not likewise provide for such notice. This being the case, it devolves upon the Court to apply the generally accepted principles of insurance, regarding cancellation of the insurance policy by the insurer. From what has been heretofore stated, actual notice of cancellation in a clear and unequivocal manner, preferably in writing, in view of the importance of an insurance contract, should be given by the insurer to the insured, so that the latter might be given an opportunity to obtain other insurance for his own protection. The notice should be personal to the insured and not to and/or through any unauthorized person by the policy. In the case at bar, the defendant insurance company, must have realized the paramount importance of sending a notice of cancellation, when it sent the notice of cancellation of the policy to the defendant bank (as mortgagee), but not to the insured with which it (insurance company) had direct dealing. It was the primary duty of the defendant-appellee insurance company to notify the insured, but it did not. It should be stated that the house and its contents were burned on April 6, 1955, at the time when the policy was enforced (October 2, 1954 to October 2, 1955); and that under the facts, as found by the trial court, to which We are bound, it is evident that both the insurance company and the appellee bank failed, wittingly or unwittingly, to notify the insured appellant Saura of the cancellation made. Of course, the defendant insurance company contends that it gave notice to the defendant-appellee bank as mortgagee of the property, and that was already a substantial compliance with its duty to notify the insured of the cancellation of the policy. But notice to the bank, as far appellant herein is concerned, is not effective notice. If a mortgage or lien exists against the property insured, and the policy contains a clause stating that loss, if any, shall be payable to such mortgagee or the holder of such lien as interest may appear, notice of cancellation to the mortgagee or lienholder alone is ineffective as a cancellation of the policy to the owner of the property. (Connecticut Ins. Co. v. Caumisar, 218 Ky. 378, 391 SW 776, cited in 29 Am. Jur. p. 743).

Upon authority of the above case, therefore, the liability of the insurance company becomes a fact. It may be argued that in the appeal brief of appellant, no error has been assigned against the insurance company and no prayer is found therein asking that it be made liable. It must be noted, however, that the case was dismissed the lower court and the main object of the appeal is to secure a reversal of the said judgment. This Court is clothed with ample authority to review matters, even if they are not assigned as errors in the appeal, if it finds that their consideration is necessary in arriving at a just decision of the case. Thus it was held: While an assignment of error which is required by law or rule of court has been held essential to appellate review, only those assigned will be considered, there are a number of cases which appear to accord to the appellate court a broad discretionary power to waive the lack of proper assignment of errors and consider errors not assigned. And an unassigned error closely related to an error properly assigned, or upon which the determination of the question raised by the error properly assigned is dependent, will be considered by the appellate court notwithstanding the failure to assign it as error. (Hernandez v. Andal, 78 Phil. 198-199). Although assigned errors apparently appear to be directed against the appellee bank alone, they in essence, seek a reversal of the decision on dismissal, entered by the lower court, which in the main has for its purpose the finding of liability on the policy. In the course of our examination of the records of the case, the decision and the errors assigned, We found that liability attached principally the insurance company, for its failure to give notice of the cancellation of the policy to herein appellant itself. Because of the conclusions reached, We find it unnecessary to discuss the errors assigned against appellee bank. WHEREFORE, the decision appealed from is hereby reversed, and another is entered, condemning the defendant-appellee Philippine International Surety Co., Inc., to pay Saura Import & Export Co., Inc., appellant herein, the sum of P29,000.00, the amount involved in Policy No. 429, subject-matter of the instant case. Without costs. Bengzon, C.J., Padilla, Bautista Angelo, Concepcion, Reyes, J.B.L., Barrera, Dizon, Regala and Makalintal, JJ., concur. Labrador, J., took no part.

G.R. No. L-7667

November 28, 1955

CHERIE PALILEO, plaintiff-appellee, vs. BEATRIZ COSIO, defendant-appellant. Claro M. Recto for appellant. Bengson, Villegas, Jr. and Villar for appellee. BAUTISTA ANGELO, J.: Plaintiff filed a complaint against defendant in the Court of First Instance of Manila praying that (1) the transaction entered into between them on December 18, 1951 be declared as one of loan, and the document executed covering the transaction as one of equitable mortgage to secure the payment of said loan; (2) the defendant be ordered to credit to the plaintiff with the necessary amount from the sum received by the defendant from the Associated Insurance & Surety Co., Inc. and to apply the same to the payment of plaintiff's obligation thus considering it as fully paid; and (3) the defendant be ordered to pay to plaintiff the difference between the alleged indebtedness of plaintiff and the sum received by defendant from the aforementioned insurance company, plus the sum allegedly paid to defendant as interest on the alleged indebtedness. On December 19, 1952, defendant filed her answer setting up as special defense that the transaction entered into between the plaintiff and defendant is one of sale with option to repurchase but that the period for repurchase had expired without plaintiff having returned the price agreed upon as a result of which the ownership of the property had become consolidated in the defendant. Defendant also set up certain counterclaims which involve a total amount of P4,900. On April 7, 1953, the case was set for trial on the merits, but because of several postponements asked by the parties, the same has to be set anew for trial on January 12, 1954. On this date, neither the defendant nor her counsel appeared, even if the latter had been notified of the postponement almost a month earlier, and so the court received the evidence of the plaintiff. On January 18, 1954, the court, having in view the evidence presented, rendered judgment granting the relief prayed for in the complaint. On February 2, 1954, the original counsel for the defendant was substituted and the new counsel immediately moved that the judgment be set aside on the ground that, due to mistake or excusable negligence, defendant was unable to present her evidence and the decision was contrary to law, and this motion having been denied, defendant took the present appeal. The important issue to be determined in this appeal is whether the lower court committed a grave abuse of discretion in not reopening the case to give defendant an opportunity to present her evidence considering that the failure of her original counsel to

appear was due to mistake or execusable negligence which ordinary prudence could not have guarded against. The original counsel of defendant was Atty. Leon Ma. Guerrero. As early as February 11, 1953, said counsel showed interest in the early disposal of this case by moving the court to have it set for trial. The first date set was April 7, 1953, but no hearing was had on that date because plaintiff had moved to postpone it. The case was next set for hearing on April 28, 1953, but on motion again of plaintiff, the hearing was transferred to November 6, 1953. Then, upon petition of defendant, the trial had to be moved to December 15, 1953, and because Atty. Guerrero could not appear on said date because of a case he had in Cebu City, the hearing was postponed to January 18, 1954. And on January 4, 1954, or nineteen days after receiving the notice of hearing, Atty. Guerrero was appointed Undersecretary of Foreign Affairs. It is now contended that the appointment was so sudden and unexpected that Atty. Guerrero, after taking his oath, was unable to wind up his private cases or make any preparation at all. It is averred that "The days that followed his appointment were very busy days for defendant's former counsel. There was an immediate need for clearing the backlog of official business, including the reorganization of the Department of Foreign Affairs and our Foreign Service, and more importantly, he had to assist the Secretary of Foreign Affairs in negotiations of national importance like the Japanese reparations, and the revision of the trade agreement with the United States, that, Atty. Guerrero had to work as much as fourteen hours daily . . . Because of all these unavoidable confusion that followed in the wake of Atty. Guerrero's sudden and unexpected appointment, the trial of this case scheduled for January 18, 1954 escaped his memory, and consequently, Atty. Guerrero and the defendant were unable to appear when the case was called for trial." These reasons, it is intimated, constitute excusable negligence which ordinary prudence could not have guarded against and should have been considered by the trial court as sufficient justification to grant the petition of defendant for a rehearing. It is a well-settled rule that the granting of a motion to set aside a judgment or order on the ground of mistake or excusable negligence is addressed to the sound discretion of the court (see Coombs vs. Santos, 24 Phil., 446; Daipan vs. Sigabu, 25, Phil., 184). And an order issued in the exercise of such discretion is ordinarily not to be disturbed unless it is shown that the court has gravely abused such discretion. (See Tell vs. Tell, 48 Phil., 70; Macke vs. Camps, 5 Phil., 185; Calvo vs. De Gutierrez, 4 Phil., 203; Manzanares vs. Moreta, 38 Phil., 821; Salvavs. Palacio and Leuterio, 90 Phil., 731.) In denying the motion for reopening the trial court said: "After going over the same arguments, this Court is of the opinion, and so holds that the decision of this Court of January 18, 1954 should not be disturbed." Considering the stature, ability and experience of counsel Leon Ma. Guerrero, and the fact that he was given almost one month notice before the date set for trial, we are persuaded to conclude that the trial court did not abuse its discretion in refusing to reconsider its decision.

Coming now to the merits of the case, we note that the lower court made the following findings: On December 18, 1951, plaintiff obtained from defendant a loan in the sum of P12,000 subject to the following conditions: (a) that plaintiff shall pay to defendant an interest in the amount of P250 a month; (b) that defendant shall deduct from the loan certain obligations of plaintiff to third persons amounting to P4,550, plus the sum of P250 as interest for the first month; and (c) that after making the above deductions, defendant shall deliver to plaintiff only the balance of the loan of P12,000. Pursuant to their agreement, plaintiff paid to defendant as interest on the loan a total of P2,250.00 corresponding to nine months from December 18, 1951, on the basis of P250.00 a month, which is more than the maximum interest authorized by law. To secure the payment of the aforesaid loan, defendant required plaintiff to sign a document known as "Conditional Sale of Residential Building", purporting to convey to defendant, with right to repurchase, a two-story building of strong materials belonging to plaintiff. This document did not express the true intention of the parties which was merely to place said property as security for the payment of the loan. After the execution of the aforesaid document, defendant insured the building against fire with the Associated Insurance & Surety Co., Inc. for the sum of P15,000, the insurance policy having been issued in the name of defendant. The building was partly destroyed by fire and, after proper demand, defendant collected from the insurance company an indemnity of P13,107.00. Plaintiff demanded from defendant that she be credited with the necessary amount to pay her obligation out of the insurance proceeds but defendant refused to do so. And on the strength of these facts, the court rendered decision the dispositive part of which reads as follows: Wherefore, judgment is hereby rendered declaring the transaction had between plaintiff and defendant, as shown in Exhibit A, an equitable mortgage to secure the payment of the sum of P12,000 loaned by the defendant to plaintiff; ordering the defendant to credit the sum of P13,107 received by the defendant from the Associated Insurance & surety Co., Inc. to the payment of plaintiff's obligation in the sum of P12,000.00 as stated in the complaint, thus considering the agreement of December 18, 1951 between the herein plaintiff and defendant completely paid and leaving still a balance in the sum of P1,107 from the insurance collected by defendant; that as plaintiff had paid to the defendant the sum of P2,250.00 for nine months as interest on the sum of P12,000 loaned to plaintiff and the legal interest allowed by law in this transaction does not exceed 12 per cent per annum, or the sum of P1,440 for one year, so the herein plaintiff and overpaid the sum of P810 to the defendant, which this Court hereby likewise orders the said defendant to refund to herein plaintiff, plus the balance of P1,107 representing the difference of the sum loan of P12,000 and the collected insurance of P13,107 from the insurance company abovementioned to which the herein plaintiff is entitled to receive, and to pay the costs. The question that now arises is: Is the trial court justified in considering the obligation of plaintiff fully compensated by the insurance amount and in ordering defendant to refund

to plaintiff the sum of P1,107 representing the difference of the loan of P12,000 and the sum of P13,107 collected by said defendant from the insurance company notwithstanding the fact that it was not proven that the insurance was taken for the benefit of the mortgagor? Is is our opinion that on this score the court is in error for its ruling runs counter to the rule governing an insurance taken by a mortgagee independently of the mortgagor. The rule is that "where a mortgagee, independently of the mortgagor, insures the mortgaged property in his own name and for his own interest, he is entitled to the insurance proceeds in case of loss, but in such case, he is not allowed to retain his claim against the mortgagor,but is passed by subrogation to the insurer to the extent of the money paid." (Vance on Insurance, 2d ed., p. 654)Or, stated in another way, "the mortgagee may insure his interest in the property independently of the mortgagor. In that event, upon the destruction of the property the insurance money paid to the mortgagee will not inure to the benefit of the mortgagor, and the amount due under the mortgage debt remains unchanged. The mortgagee, however, is not allowed to retain his claim against the mortgagor, but it passes by subrogation to the insurer, to the extent of the insurance money paid." (Vance on Insurance, 3rd ed., pp. 772-773) This is the same rule upheld by this Court in a case that arose in this jurisdiction. In the case mentioned, an insurance contract was taken out by the mortgagee upon his own interest, it being stipulated that the proceeds would be paid to him only and when the case came up for decision, this Court held that the mortgagee, in case of loss, may only recover upon the policy to the extent of his credit at the time of the loss. It was declared that the mortgaged had no right of action against the mortgagee on the policy. (San Miguel Brewery vs. Law Union, 40 Phil., 674.) It is true that there are authorities which hold that "If a mortgagee procures insurance on his separate interest at his own expense and for his own benefit, without any agreement with the mortgagor with respect thereto, the mortgagor has no interest in the policy, and is not entitled to have the insurance proceeds applied in reduction of the mortgage debt" (19 R.C.L., p. 405), and that, furthermore, the mortgagee "has still a right to recover his whole debt of the mortgagor." (King vs. State Mut. F. Ins. Co., 7 Cush. 1; Suffolk F. Ins. Co. vs. Boyden 9 Allen, 123; See also Loomis vs. Eagle Life & Health Ins. Co., 6 Gray, 396; Washington Mills Emery Mfg. Co. vs. Weymouth & B. Mut. F. Ins. Co., 135 Mass. 506; Foster vs. Equitable Mut. F. Ins. Co., 2 Gray 216.) But these authorities merely represent the minority view (See case note, 3 Lawyers' Report Annotated, new series, p. 79). "The general rule and the weight of authority is, that the insurer is thereupon subrogated to the rights of the mortgagee under the mortgage. This is put upon the analogy of the situation of the insurer to that of a surety." (Jones on Mortgages, Vol. I, pp. 671-672.) Considering the foregoing rules, it would appear that the lower court erred in declaring that the proceeds of the insurance taken out by the defendant on the property mortgaged inured to the benefit of the plaintiff and in ordering said defendant to deliver to the plaintiff the difference between her indebtedness and the amount of insurance received by the defendant, for, in the light of the majority rule we have above

enunciated, the correct solution should be that the proceeds of the insurance should be delivered to the defendant but that her claim against the plaintiff should be considered assigned to the insurance company who is deemed subrogated to the rights of the defendant to the extent of the money paid as indemnity. Consistent with the foregoing pronouncement, we therefore modify the judgment of the lower court as follows:(1) the transaction had between the plaintiff and defendant as shown in Exhibit A is merely an equitable mortgage intended to secure the payment of the loan of P12,000;(2) that the proceeds of the insurance amounting to P13,107.00 was properly collected by defendant who is not required to account for it to the plaintiff; (3) that the collection of said insurance proceeds shall not be deemed to have compensated the obligation of the plaintiff to the defendant, but bars the latter from claiming its payment from the former; and (4) defendant shall pay to the plaintiff the sum of P810.00 representing the overpayment made by plaintiff by way of interest on the loan. No pronouncement as to costs. Bengzon, Montemayor, Reyes, A., Jugo, Labrador , Concepcion, and Reyes, J.B.L., JJ., concur.

G.R. No. 113899 October 13, 1999 GREAT PACIFIC LIFE ASSURANCE CORP., petitioner, vs. COURT OF APPEALS AND MEDARDA V. LEUTERIO, respondents. QUISUMBING, J.: This petition for review, under Rule 45 of the Rules of Court, assails the Decision 1 dated May 17, 1993, of theCourt of Appeals and its Resolution 2 dated January 4, 1994 in CA-G.R. CV No. 18341. The appellate courtaffirmed in toto the judgment of the Misamis Oriental Regional Trial Court, Branch 18, in an insurance claim filed by private respondent against Great Pacific Life Assurance Co. The dispositive portion of the trial court's decision reads: WHEREFORE, judgment is rendered adjudging the defendant GREAT PACIFIC LIFE ASSURANCECORPORATION as insurer under its Group policy No. G-1907, in relation to Certification B-18558 liable and ordered to pay to the DEVELOPMENT BANK OF THE PHILIPPINES as creditor of the insured Dr. Wilfredo Leuterio, the amount of EIGHTY SIX THOUSAND TWO HUNDRED PESOS (P86,200.00); dismissing the claims for damages, attorney's fees and litigation expenses in the complaint and counterclaim, with costs against the defendant and dismissing the complaint in respect to the plaintiffs, other than the widow-beneficiary, for lack of cause of action. 3 The facts, as found by the Court of Appeals, are as follows: A contract of group life insurance was executed between petitioner Great Pacific Life Assurance Corporation (hereinafter Grepalife) and Development Bank of the Philippines (hereinafter DBP). Grepalife agreed to insure the lives of eligible housing loan mortgagors of DBP. On November 11, 1983, Dr. Wilfredo Leuterio, a physician and a housing debtor of DBP applied for membership in the group life insurance plan. In an application form, Dr. Leuterio answered questions concerning his health condition as follows: 7. Have you ever had, or consulted, a physician for a heart condition, high blood pressure, cancer, diabetes, lung; kidney or stomach disorder or any other physical impairment? Answer: No. If so give details _____________. 8. Are you now, to the best of your knowledge, in good health?

Answer: [x] Yes [ ] NO. 4 On November 15, 1983, Grepalife issued Certificate No. B-18558, as insurance coverage of Dr. Leuterio, to the extent of his DBP mortgage indebtedness amounting to eighty-six thousand, two hundred (P86,200.00) pesos.1wphi1.nt On August 6, 1984, Dr. Leuterio died due to "massive cerebral hemorrhage." Consequently, DBP submitted a death claim to Grepalife. Grepalife denied the claim alleging that Dr. Leuterio was not physically healthy when he applied for an insurance coverage on November 15, 1983. Grepalife insisted that Dr. Leuterio did not disclose he had been suffering from hypertension, which caused his death. Allegedly, such nondisclosure constituted concealment that justified the denial of the claim. On October 20, 1986, the widow of the late Dr. Leuterio, respondent Medarda V. Leuterio, filed a complaint with the Regional Trial Court of Misamis Oriental, Branch 18, against Grepalife for "Specific Performance with Damages." 5 During the trial, Dr. Hernando Mejia, who issued the death certificate, was called to testify. Dr. Mejia's findings, based partly from the information given by the respondent widow, stated that Dr. Leuterio complained of headaches presumably due to high blood pressure. The inference was not conclusive because Dr. Leuterio was not autopsied, hence, other causes were not ruled out. On February 22, 1988, the trial court rendered a decision in favor of respondent widow and against Grepalife. On May 17, 1993, the Court of Appeals sustained the trial court's decision. Hence, the present petition. Petitioners interposed the following assigned errors: 1. THE LOWER COURT ERRED IN HOLDING DEFENDANT-APPELLANT LIABLE TO THE DEVELOPMENT BANK OF THE PHILIPPINES (DBP) WHICH IS NOT A PARTY TO THE CASE FOR PAYMENT OF THE PROCEEDS OF A MORTGAGE REDEMPTION INSURANCE ON THE LIFE OF PLAINTIFF'S HUSBAND WILFREDO LEUTERIO ONE OF ITS LOAN BORROWERS, INSTEAD OF DISMISSING THE CASE AGAINST DEFENDANT-APPELLANT [Petitioner Grepalife] FOR LACK OF CAUSE OF ACTION. 2. THE LOWER COURT ERRED IN NOT DISMISSING THE CASE FOR WANT OF JURISDICTION OVER THE SUBJECT OR NATURE OF THE ACTION AND OVER THE PERSON OF THE DEFENDANT. 3. THE LOWER COURT ERRED IN ORDERING DEFENDANT-APPELLANT TO PAY TO DBP THE AMOUNT OF P86,200.00 IN THE ABSENCE OF ANY

EVIDENCE TO SHOW HOW MUCH WAS THE ACTUAL AMOUNT PAYABLE TO DBP IN ACCORDANCE WITH ITS GROUP INSURANCE CONTRACT WITH DEFENDANTAPPELLANT. 4. THE LOWER COURT ERRED IN HOLDING THAT THERE WAS NO CONCEALMENT OF MATERIAL INFORMATION ON THE PART OF WILFREDO LEUTERIO IN HIS APPLICATION FOR MEMBERSHIP IN THE GROUP LIFE INSURANCE PLAN BETWEEN DEFENDANTAPPELLANT OF THE INSURANCE CLAIM ARISING FROM THE DEATH OF WILFREDO LEUTERIO. 6 Synthesized below are the assigned errors for our resolution: 1. Whether the Court of Appeals erred in holding petitioner liable to DBP as beneficiary in a group life insurance contract from a complaint filed by the widow of the decedent/mortgagor? 2. Whether the Court of Appeals erred in not finding that Dr. Leuterio concealed that he had hypertension, which would vitiate the insurance contract? 3. Whether the Court of Appeals erred in holding Grepalife liable in the amount of eighty six thousand, two hundred (P86,200.00) pesos without proof of the actual outstanding mortgage payable by the mortgagor to DBP. Petitioner alleges that the complaint was instituted by the widow of Dr. Leuterio, not the real party in interest, hence the trial court acquired no jurisdiction over the case. It argues that when the Court of Appeals affirmed the trial court's judgment, Grepalife was held liable to pay the proceeds of insurance contract in favor of DBP, the indispensable party who was not joined in the suit. To resolve the issue, we must consider the insurable interest in mortgaged properties and the parties to this type of contract. The rationale of a group insurance policy of mortgagors, otherwise known as the "mortgage redemption insurance," is a device for the protection of both the mortgagee and the mortgagor. On the part of the mortgagee, it has to enter into such form of contract so that in the event of the unexpected demise of the mortgagor during the subsistence of the mortgage contract, the proceeds from such insurance will be applied to the payment of the mortgage debt, thereby relieving the heirs of the mortgagor from paying the obligation. 7 In a similar vein, ample protection is given to the mortgagor under such a concept so that in the event of death; the mortgage obligation will be extinguished by the application of the insurance proceeds to the mortgage indebtedness. 8 Consequently, where the mortgagor pays the

insurance premium under the group insurance policy, making the loss payable to the mortgagee, the insurance is on the mortgagor's interest, and the mortgagor continues to be a party to the contract. In this type of policy insurance, the mortgagee is simply an appointee of the insurance fund, such loss-payable clause does not make the mortgagee a party to the contract. 9 Sec. 8 of the Insurance Code provides: Unless the policy provides, where a mortgagor of property effects insurance in his own name providing that the loss shall be payable to the mortgagee, or assigns a policy of insurance to a mortgagee, the insurance is deemed to be upon the interest of the mortgagor, who does not cease to be a party to the original contract, and any act of his, prior to the loss, which would otherwise avoid the insurance, will have the same effect, although the property is in the hands of the mortgagee, but any act which, under the contract of insurance, is to be performed by the mortgagor, may be performed by the mortgagee therein named, with the same effect as if it had been performed by the mortgagor. The insured private respondent did not cede to the mortgagee all his rights or interests in the insurance, the policy stating that: "In the event of the debtor's death before his indebtedness with the Creditor [DBP] shall have been fully paid, an amount to pay the outstanding indebtedness shall first be paid to the creditor and the balance of sum assured, if there is any, shall then be paid to the beneficiary/ies designated by the debtor." 10 When DBP submitted the insurance claim against petitioner, the latter denied payment thereof, interposing the defense of concealment committed by the insured. Thereafter, DBP collected the debt from the mortgagor and took the necessary action of foreclosure on the residential lot of private respondent. 11 In Gonzales La O vs. Yek Tong Lin Fire & Marine Ins. Co. 12 we held: Insured, being the person with whom the contract was made, is primarily the proper person to bring suit thereon. * * * Subject to some exceptions, insured may thus sue, although the policy is taken wholly or in part for the benefit of another person named or unnamed, and although it is expressly made payable to another as his interest may appear or otherwise. * * * Although a policy issued to a mortgagor is taken out for the benefit of the mortgagee and is made payable to him, yet the mortgagor may sue thereon in his own name, especially where the mortgagee's interest is less than the full amount recoverable under the policy, * * *. And in volume 33, page 82, of the same work, we read the following: Insured may be regarded as the real party in interest, although he has assigned the policy for the purpose of collection, or has assigned as collateral security any judgment he may obtain. 13

And since a policy of insurance upon life or health may pass by transfer, will or succession to any person, whether he has an insurable interest or not, and such person may recover it whatever the insured might have recovered,14 the widow of the decedent Dr. Leuterio may file the suit against the insurer, Grepalife. The second assigned error refers to an alleged concealment that the petitioner interposed as its defense to annul the insurance contract. Petitioner contends that Dr. Leuterio failed to disclose that he had hypertension, which might have caused his death. Concealment exists where the assured had knowledge of a fact material to the risk, and honesty, good faith, and fair dealing requires that he should communicate it to the assured, but he designedly and intentionally withholds the same. 15 Petitioner merely relied on the testimony of the attending physician, Dr. Hernando Mejia, as supported by the information given by the widow of the decedent. Grepalife asserts that Dr. Mejia's technical diagnosis of the cause of death of Dr. Leuterio was a duly documented hospital record, and that the widow's declaration that her husband had "possible hypertension several years ago" should not be considered as hearsay, but as part of res gestae. On the contrary the medical findings were not conclusive because Dr. Mejia did not conduct an autopsy on the body of the decedent. As the attending physician, Dr. Mejia stated that he had no knowledge of Dr. Leuterio's any previous hospital confinement. 16 Dr. Leuterio's death certificate stated that hypertension was only "the possible cause of death." The private respondent's statement, as to the medical history of her husband, was due to her unreliable recollection of events. Hence, the statement of the physician was properly considered by the trial court as hearsay. The question of whether there was concealment was aptly answered by the appellate court, thus: The insured, Dr. Leuterio, had answered in his insurance application that he was in good health and that he had not consulted a doctor or any of the enumerated ailments, including hypertension; when he died the attending physician had certified in the death certificate that the former died of cerebral hemorrhage, probably secondary to hypertension. From this report, the appellant insurance company refused to pay the insurance claim. Appellant alleged that the insured had concealed the fact that he had hypertension. Contrary to appellant's allegations, there was no sufficient proof that the insured had suffered from hypertension. Aside from the statement of the insured's widow who was not even sure if the medicines taken by Dr. Leuterio were for hypertension, the appellant had not proven nor produced any witness who could attest to Dr. Leuterio's medical history . . . xxx xxx xxx

Appellant insurance company had failed to establish that there was concealment made by the insured, hence, it cannot refuse payment of the claim. 17 The fraudulent intent on the part of the insured must be established to entitle the insurer to rescind the contract.18 Misrepresentation as a defense of the insurer to avoid liability is an affirmative defense and the duty to establish such defense by satisfactory and convincing evidence rests upon the insurer. 19 In the case at bar, the petitioner failed to clearly and satisfactorily establish its defense, and is therefore liable to pay the proceeds of the insurance.1wphi1.nt And that brings us to the last point in the review of the case at bar. Petitioner claims that there was no evidence as to the amount of Dr. Leuterio's outstanding indebtedness to DBP at the time of the mortgagor's death. Hence, for private respondent's failure to establish the same, the action for specific performance should be dismissed. Petitioner's claim is without merit. A life insurance policy is a valued policy. 20 Unless the interest of a person insured is susceptible of exact pecuniary measurement, the measure of indemnity under a policy of insurance upon life or health is the sum fixed in the policy. 21 The mortgagor paid the premium according to the coverage of his insurance, which states that: The policy states that upon receipt of due proof of the Debtor's death during the terms of this insurance, a death benefit in the amount of P86,200.00 shall be paid. In the event of the debtor's death before his indebtedness with the creditor shall have been fully paid, an amount to pay the outstanding indebtedness shall first be paid to the Creditor and the balance of the Sum Assured, if there is any shall then be paid to the beneficiary/ies designated by the debtor." 22 (Emphasis omitted) However, we noted that the Court of Appeals' decision was promulgated on May 17, 1993. In private respondent's memorandum, she states that DBP foreclosed in 1995 their residential lot, in satisfaction of mortgagor's outstanding loan. Considering this supervening event, the insurance proceeds shall inure to the benefit of the heirs of the deceased person or his beneficiaries. Equity dictates that DBP should not unjustly enrich itself at the expense of another (Nemo cum alterius detrimenio protest). Hence, it cannot collect the insurance proceeds, after it already foreclosed on the mortgage. The proceeds now rightly belong to Dr. Leuterio's heirs represented by his widow, herein private respondent Medarda Leuterio. WHEREFORE, the petition is hereby DENIED. The Decision and Resolution of the Court of Appeals in CA-G.R. CV 18341 is AFFIRMED with MODIFICATION that the petitioner is ORDERED to pay the insurance proceeds amounting to Eighty-six thousand, two hundred (P86,200.00) pesos to the heirs of the insured, Dr. Wilfredo Leuterio (deceased), upon presentation of proof of prior settlement of mortgagor's

indebtedness to Development Bank of the Philippines. Costs against petitioner.1wphi1.nt SO ORDERED.

V.

Insurable Interest
April 11, 2012

G.R. No. 170290

PHILIPPINE DEPOSIT INSURANCE CORPORATION, Petitioner, vs. CITIBANK, N.A. and BANK OF AMERICA, S.T. & N.A., Respondents. DECISION MENDOZA, J.: This is a petition for review under Rule 45 of the 1997 Revised Rules of Civil Procedure, assailing the October 27, 2005 Decision1 of the Court of Appeals (CA) in CA-G.R. CV No. 61316, entitled "Citibank, N.A. and Bank of America, S.T. & N.A. v. Philippine Deposit Insurance Corporation." The Facts Petitioner Philippine Deposit Insurance Corporation (PDIC) is a government instrumentality created by virtue of Republic Act (R.A.) No. 3591, as amended by R.A. No. 9302.2 Respondent Citibank, N.A. (Citibank) is a banking corporation while respondent Bank of America, S.T. & N.A. (BA)is a national banking association, both of which are duly organized and existing under the laws of the United States of America and duly licensed to do business in the Philippines, with offices in Makati City.3 In 1977, PDIC conducted an examination of the books of account of Citibank. It discovered that Citibank, in the course of its banking business, from September 30, 1974 to June 30, 1977, received from its head office and other foreign branches a total of P11,923,163,908.00 in dollars, covered by Certificates of Dollar Time Deposit that were interest-bearing with corresponding maturity dates.4 These funds, which were lodged in the books of Citibank under the account "Their Account-Head Office/Branches-Foreign Currency," were not reported to PDIC as deposit liabilities that were subject to assessment for insurance.5 As such, in a letter dated March 16, 1978, PDIC assessed Citibank for deficiency in the sum of P1,595,081.96.6 Similarly, sometime in 1979, PDIC examined the books of accounts of BA which revealed that from September 30, 1976 to June 30, 1978, BA received from its head office and its other foreign branches a total of P629,311,869.10 in dollars, covered by Certificates of Dollar Time Deposit that were interest-bearing with corresponding maturity dates and lodged in their books under the account "Due to Head Office/Branches."7 Because BA also excluded these from its deposit liabilities, PDIC wrote to BA on October 9, 1979, seeking the remittance of P109,264.83 representing deficiency premium assessments for dollar deposits.8

Believing that litigation would inevitably arise from this dispute, Citibank and BA each filed a petition for declaratory relief before the Court of First Instance (now the Regional Trial Court) of Rizal on July 19, 1979 and December 11, 1979, respectively. 9 In their petitions, Citibank and BA sought a declaratory judgment stating that the money placements they received from their head office and other foreign branches were not deposits and did not give rise to insurable deposit liabilities under Sections 3 and 4 of R.A. No. 3591 (the PDIC Charter) and, as a consequence, the deficiency assessments made by PDIC were improper and erroneous.10 The cases were then consolidated.11 On June 29, 1998, the Regional Trial Court, Branch 163, Pasig City (RTC) promulgated its Decision12 in favor of Citibank and BA, ruling that the subject money placements were not deposits and did not give rise to insurable deposit liabilities, and that the deficiency assessments issued by PDIC were improper and erroneous. Therefore, Citibank and BA were not liable to pay the same. The RTC reasoned out that the money placements subject of the petitions were not assessable for insurance purposes under the PDIC Charter because said placements were deposits made outside of the Philippines and, under Section 3.05(b) of the PDIC Rules and Regulations,13 such deposits are excluded from the computation of deposit liabilities. Section 3(f) of the PDIC Charter likewise excludes from the definition of the term "deposit" any obligation of a bank payable at the office of the bank located outside the Philippines. The RTC further stated that there was no depositor-depository relationship between the respondents and their head office or other branches. As a result, such deposits were not included as third-party deposits that must be insured. Rather, they were considered inter-branch deposits which were excluded from the assessment base, in accordance with the practice of the United States Federal Deposit Insurance Corporation(FDIC) after which PDIC was patterned. Aggrieved, PDIC appealed to the CA which affirmed the ruling of the RTC in its October 27, 2005 Decision. In so ruling, the CA found that the money placements were received as part of the banks internal dealings by Citibank and BA as agents of their respective head offices. This showed that the head office and the Philippine branch were considered as the same entity. Thus, no bank deposit could have arisen from the transactions between the Philippine branch and the head office because there did not exist two separate contracting parties to act as depositor and depositary. 14 Secondly, the CA called attention to the purpose for the creation of PDIC which was to protect the deposits of depositors in the Philippines and not the deposits of the same bank through its head office or foreign branches.15 Thirdly, because there was no law or jurisprudence on the treatment of inter-branch deposits between the Philippine branch of a foreign bank and its head office and other branches for purposes of insurance, the CA was guided by the procedure observed by the FDIC which considered inter-branch deposits as non-assessable.16 Finally, the CA cited Section 3(f) of R.A. No. 3591, which specifically excludes obligations payable at the office of the bank located outside the Philippines from the definition of a deposit or an insured deposit. Since the subject money placements were made in the respective head offices of Citibank and BA located outside the Philippines, then such placements could not be subject to assessment under the PDIC Charter.17

Hence, this petition. The Issues PDIC raises the issue of whether or not the subject dollar deposits are assessable for insurance purposes under the PDIC Charter with the following assigned errors: A. The appellate court erred in ruling that the subject dollar deposits are money placements, thus, they are not subject to the provisions of Republic Act No. 6426 otherwise known as the "Foreign Currency Deposit Act of the Philippines." B. The appellate court erred in ruling that the subject dollar deposits are not covered by the PDIC insurance.18 Respondents similarly identify only one issue in this case: Whether or not the money placements subject matter of these petitions are assessable for insurance purposes under the PDIC Act.19 The sole question to be resolved in this case is whether the funds placed in the Philippine branch by the head office and foreign branches of Citibank and BA are insurable deposits under the PDIC Charter and, as such, are subject to assessment for insurance premiums. The Courts Ruling The Court rules in the negative. A branch has no separate legal personality; Purpose of the PDIC PDIC argues that the head offices of Citibank and BA and their individual foreign branches are separate and independent entities. It insists that under American jurisprudence, a banks head office and its branches have a principal-agent relationship only if they operate in the same jurisdiction. In the case of foreign branches, however, no such relationship exists because the head office and said foreign branches are deemed to be two distinct entities.20 Under Philippine law, specifically, Section 3(b) of R.A. No. 3591, which defines the terms "bank" and "banking institutions," PDIC contends that the law treats a branch of a foreign bank as a separate and independent banking unit.21

The respondents, on the other hand, initially point out that the factual findings of the RTC and the CA, with regard to the nature of the money placements, the capacity in which the same were received by the respondents and the exclusion of inter-branch deposits from assessment, can no longer be disturbed and should be accorded great weight by this Court.22 They also argue that the money placements are not deposits. They postulate that for a deposit to exist, there must be at least two parties a depositor and a depository each with a legal personality distinct from the other. Because the respondents respective head offices and their branches form only a single legal entity, there is no creditor-debtor relationship and the funds placed in the Philippine branch belong to one and the same bank. A bank cannot have a deposit with itself.23 This Court is of the opinion that the key to the resolution of this controversy is the relationship of the Philippine branches of Citibank and BA to their respective head offices and their other foreign branches. The Court begins by examining the manner by which a foreign corporation can establish its presence in the Philippines. It may choose to incorporate its own subsidiary as a domestic corporation, in which case such subsidiary would have its own separate and independent legal personality to conduct business in the country. In the alternative, it may create a branch in the Philippines, which would not be a legally independent unit, and simply obtain a license to do business in the Philippines.24 In the case of Citibank and BA, it is apparent that they both did not incorporate a separate domestic corporation to represent its business interests in the Philippines. Their Philippine branches are, as the name implies, merely branches, without a separate legal personality from their parent company, Citibank and BA. Thus, being one and the same entity, the funds placed by the respondents in their respective branches in the Philippines should not be treated as deposits made by third parties subject to deposit insurance under the PDIC Charter. For lack of judicial precedents on this issue, the Court seeks guidance from American jurisprudence.1wphi1 In the leading case of Sokoloff v. The National City Bank of New York,25 where the Supreme Court of New York held: Where a bank maintains branches, each branch becomes a separate business entity with separate books of account. A depositor in one branch cannot issue checks or drafts upon another branch or demand payment from such other branch, and in many other respects the branches are considered separate corporate entities and as distinct from one another as any other bank.Nevertheless, when considered with relation to the parent bank they are not independent agencies; they are, what their name imports, merely branches, and are subject to the supervision and control of the parent bank, and are instrumentalities whereby the parent bank carries on its business, and are established for its own particular purposes, and their business conduct and policies are controlled by the parent bank and their property and assets belong to the parent bank, although nominally held in the names of the particular

branches. Ultimate liability for a debt of a branch would rest upon the parent bank. [Emphases supplied] This ruling was later reiterated in the more recent case of United States v. BCCI Holdings Luxembourg26 where the United States Court of Appeals, District of Columbia Circuit, emphasized that "while individual bank branches may be treated as independent of one another, each branch, unless separately incorporated, must be viewed as a part of the parent bank rather than as an independent entity." In addition, Philippine banking laws also support the conclusion that the head office of a foreign bank and its branches are considered as one legal entity. Section 75 of R.A. No. 8791 (The General Banking Law of 2000) and Section 5 of R.A. No. 7221 (An Act Liberalizing the Entry of Foreign Banks) both require the head office of a foreign bank to guarantee the prompt payment of all the liabilities of its Philippine branch, to wit: Republic Act No. 8791: Sec. 75. Head Office Guarantee. In order to provide effective protection of the interests of the depositors and other creditors of Philippine branches of a foreign bank, the head office of such branches shall fully guarantee the prompt payment of all liabilities of its Philippine branch. Residents and citizens of the Philippines who are creditors of a branch in the Philippines of foreign bank shall have preferential rights to the assets of such branch in accordance with the existing laws. Republic Act No. 7721: Sec. 5. Head Office Guarantee. The head office of foreign bank branches shall guarantee prompt payment of all liabilities of its Philippine branches. Moreover, PDIC must be reminded of the purpose for its creation, as espoused in Section 1 of R.A. No. 3591 (The PDIC Charter) which provides: Section 1. There is hereby created a Philippine Deposit Insurance Corporation hereinafter referred to as the "Corporation" which shall insure, as herein provided, the deposits of all banks which are entitled to the benefits of insurance under this Act, and which shall have the powers hereinafter granted. The Corporation shall, as a basic policy, promote and safeguard the interests of the depositing public by way of providing permanent and continuing insurance coverage on all insured deposits. R.A. No. 9576, which amended the PDIC Charter, reaffirmed the rationale for the establishment of the PDIC:

Section 1. Statement of State Policy and Objectives. - It is hereby declared to be the policy of the State to strengthen the mandatory deposit insurance coverage system to generate, preserve, maintain faith and confidence in the country's banking system, and protect it from illegal schemes and machinations. Towards this end, the government must extend all means and mechanisms necessary for the Philippine Deposit Insurance Corporation to effectively fulfill its vital task of promoting and safeguarding the interests of the depositing public by way of providing permanent and continuing insurance coverage on all insured deposits, and in helping develop a sound and stable banking system at all times. The purpose of the PDIC is to protect the depositing public in the event of a bank closure. It has already been sufficiently established by US jurisprudence and Philippine statutes that the head office shall answer for the liabilities of its branch. Now, suppose the Philippine branch of Citibank suddenly closes for some reason. Citibank N.A. would then be required to answer for the deposit liabilities of Citibank Philippines. If the Court were to adopt the posture of PDIC that the head office and the branch are two separate entities and that the funds placed by the head office and its foreign branches with the Philippine branch are considered deposits within the meaning of the PDIC Charter, it would result to the incongruous situation where Citibank, as the head office, would be placed in the ridiculous position of having to reimburse itself, as depositor, for the losses it may incur occasioned by the closure of Citibank Philippines. Surely our law makers could not have envisioned such a preposterous circumstance when they created PDIC. Finally, the Court agrees with the CA ruling that there is nothing in the definition of a "bank" and a "banking institution" in Section 3(b) of the PDIC Charter27 which explicitly states that the head office of a foreign bank and its other branches are separate and distinct from their Philippine branches. There is no need to complicate the matter when it can be solved by simple logic bolstered by law and jurisprudence. Based on the foregoing, it is clear that the head office of a bank and its branches are considered as one under the eyes of the law. While branches are treated as separate business units for commercial and financial reporting purposes, in the end, the head office remains responsible and answerable for the liabilities of its branches which are under its supervision and control. As such, it is unreasonable for PDIC to require the respondents, Citibank and BA, to insure the money placements made by their home office and other branches. Deposit insurance is superfluous and entirely unnecessary when, as in this case, the institution holding the funds and the one which made the placements are one and the same legal entity. Funds not a deposit under the definition of the PDIC Charter; Excluded from assessment PDIC avers that the funds are dollar deposits and not money placements. Citing R.A. No. 6848, it defines money placement as a deposit which is received with authority to

invest. Because there is no evidence to indicate that the respondents were authorized to invest the subject dollar deposits, it argues that the same cannot be considered money placements.28 PDIC then goes on to assert that the funds received by Citibank and BA are deposits, as contemplated by Section 3(f) of R.A. No. 3591, for the following reasons: (1) the dollar deposits were received by Citibank and BA in the course of their banking operations from their respective head office and foreign branches and were recorded in their books as "Account-Head Office/Branches-Time Deposits" pursuant to Central Bank Circular No. 343 which implements R.A. No. 6426; (2) the dollar deposits were credited as dollar time accounts and were covered by Certificates of Dollar Time Deposit which were interest-bearing and payable upon maturity, and (3) the respondents maintain 100% foreign currency cover for their deposit liability arising from the dollar time deposits as required by Section 4 of R.A. No. 6426.29 To refute PDICs allegations, the respondents explain the inter-branch transactions which necessitate the creation of the accounts or placements subject of this case. When the Philippine branch needs to procure foreign currencies, it will coordinate with a branch in another country which handles foreign currency purchases. Both branches have existing accounts with their head office and when a money placement is made in relation to the acquisition of foreign currency from the international market, the amount is credited to the account of the Philippine branch with its head office while the same is debited from the account of the branch which facilitated the purchase. This is further documented by the issuance of a certificate of time deposit with a stated interest rate and maturity date. The interest rate represents the cost of obtaining the funds while the maturity date represents the date on which the placement must be returned. On the maturity date, the amount previously credited to the account of the Philippine branch is debited, together with the cost for obtaining the funds, and credited to the account of the other branch. The respondents insist that the interest rate and maturity date are simply the basis for the debit and credit entries made by the head office in the accounts of its branches to reflect the inter-branch accommodation.30 As regards the maintenance of currency cover over the subject money placements, the respondents point out that they maintain foreign currency cover in excess of what is required by law as a matter of prudent banking practice.31 PDIC attempts to define money placement in order to impugn the responde nts claim that the funds received from their head office and other branches are money placements and not deposits, as defined under the PDIC Charter. In the process, it loses sight of the important issue in this case, which is the determination of whether the funds in question are subject to assessment for deposit insurance as required by the PDIC Charter. In its struggle to find an adequate definition of "money placement," PDIC desperately cites R.A. No. 6848, The Charter of the Al-Amanah Islamic Investment Bank of the Philippines. Reliance on the said law is unfounded because nowhere in the law is the term "money placement" defined. Additionally, R.A. No. 6848 refers to the establishment of an Islamic bank subject to the rulings of Islamic Sharia to assist in the development of the Autonomous Region of Muslim Mindanao (ARMM),32 making it utterly irrelevant to the case at bench. Since Citibank and BA are neither Islamic banks

nor are they located anywhere near the ARMM, then it should be painfully obvious that R.A. No. 6848 cannot aid us in deciding this case. Furthermore, PDIC heavily relies on the fact that the respondents documented the money placements with certificates of time deposit to simply conclude that the funds involved are deposits, as contemplated by the PDIC Charter, and are consequently subject to assessment for deposit insurance. It is this kind of reasoning that creates non-existent obscurities in the law and obstructs the prompt resolution of what is essentially a straightforward issue, thereby causing this case to drag on for more than three decades.1wphi1 Noticeably, PDIC does not dispute the veracity of the internal transactions of the respondents which gave rise to the issuance of the certificates of time deposit for the funds the subject of the present dispute. Neither does it question the findings of the RTC and the CA that the money placements were made, and were payable, outside of the Philippines, thus, making them fall under the exclusions to deposit liabilities. PDIC also fails to impugn the truth of the testimony of John David Shaffer, then a Fiscal Agent and Head of the Assessment Section of the FDIC, that inter-branch deposits were excluded from the assessment base. Therefore, the determination of facts of the lower courts shall be accepted at face value by this Court, following the well-established principle that factual findings of the trial court, when adopted and confirmed by the CA, are binding and conclusive on this Court, and will generally not be reviewed on appeal.33 As explained by the respondents, the transfer of funds, which resulted from the interbranch transactions, took place in the books of account of the respective branches in their head office located in the United States. Hence, because it is payable outside of the Philippines, it is not considered a deposit pursuant to Section 3(f) of the PDIC Charter: Sec. 3(f) The term "deposit" means the unpaid balance of money or its equivalent received by a bank in the usual course of business and for which it has given or is obliged to give credit to a commercial, checking, savings, time or thrift account or which is evidenced by its certificate of deposit, and trust funds held by such bank whether retained or deposited in any department of said bank or deposit in another bank, together with such other obligations of a bank as the Board of Directors shall find and shall prescribe by regulations to be deposit liabilities of the Bank; Provided, that any obligation of a bank which is payable at the office of the bank located outside of the Philippines shall not be a deposit for any of the purposes of this Act or included as part of the total deposits or of the insured deposits; Provided further, that any insured bank which is incorporated under the laws of the Philippines may elect to include for insurance its deposit obligation payable only at such branch. [Emphasis supplied] The testimony of Mr. Shaffer as to the treatment of such inter-branch deposits by the FDIC, after which PDIC was modelled, is also persuasive. Inter-branch deposits refer to

funds of one branch deposited in another branch and both branches are part of the same parent company and it is the practice of the FDIC to exclude such inter-branch deposits from a banks total deposit liabilities subject to assessment.34 All things considered, the Court finds that the funds in question are not deposits within the definition of the PDIC Charter and are, thus, excluded from assessment. WHEREFORE, the petition is DENIED. The October 27, 2005 Decision of the Court of Appeals in CA-G.R. CV No. 61316 is AFFIRMED. JOSE CATRAL MENDOZA Associate Justice WE CONCUR: PRESBITERO J. VELASCO, JR. Associate Justice Chairperson DIOSDADO M. PERALTA Associate Justice ROBERTO A. ABAD Associate Justice

BIENVENIDO L. REYES* Associate Justice ATTESTATION I attest that the conclusions in the above Decision had been reached in consultation before the case was assigned to the writer of the opinion of the Courts Division. PRESBITERO J. VELASCO, JR. Associate Justice Chairperson, Third Division CERTIFICATION Pursuant to Section 13, Article VIII of the Constitution and the Division Chairpersons Attestation, I certify that the conclusions in the above Decision had been reached in consultation before the case was assigned to the writer of the opinion of the Courts Division. RENATO C. CORONA Chief Justice

G.R. No. 23703

September 28, 1925

HILARIO GERCIO, plaintiff-appellee, vs. SUN LIFE ASSURANCE OF CANADA, ET AL., defendants. SUN LIFE ASSURANCE OF CANADA, appellant. Fisher, DeWitt, Perkins and Brady and Jesus Trinidad for appellant. Vicente Romualdez, Feria and La O and P. J. Sevilla for appellee. MALCOLM, J.: The question of first impression in the law of life insurance to be here decided is whether the insured the husband has the power to change the beneficiary the former wife and to name instead his actual wife, where the insured and the beneficiary have been divorced and where the policy of insurance does not expressly reserve to the insured the right to change the beneficiary. Although the authorities have been exhausted, no legal situation exactly like the one before us has been encountered. Hilario Gercio, the insured, is the plaintiff. The Sun Life Assurance Co. of Canada, the insurer, and Andrea Zialcita, the beneficiary, are the defendants. The complaint is in the nature of mandamus. Its purpose is to compel the defendant Sun Life Assurance Co. of Canada to change the beneficiary in the policy issued by the defendant company on the life of the plaintiff Hilario Gercio, with one Andrea Zialcita as beneficiary. A default judgment was taken in the lower court against the defendant Andrea Zialcita. The other defendant, the Sun Life Assurance Co. of Canada, first demurred to the complaint and when the demurrer was overruled, filed an answer in the nature of a general denial. The case was then submitted for decision on an agreed statement of facts. The judgment of the trial court was in favor of the plaintiff without costs, and ordered the defendant company to eliminate from the insurance policy the name of Andrea Zialcita as beneficiary and to substitute therefor such name as the plaintiff might furnish to the defendant for that purpose. The Sun Life Assurance Co. of Canada has appealed and has assigned three errors alleged to have been committed by the lower court. The appellee has countered with a motion which asks the court to dismiss the appeal of the defendant Sun Life Assurance Co. of Canada, with costs. As the motion presented by the appellee and the first two errors assigned by the appellant are preliminary in nature, we will pass upon the first. Appellee argues that the "substantial defendant" was Andrea Zialcita, and that since she was adjudged in default, the Sun Life Assurance Co. of Canada has no interest in the appeal. It will be noticed, however, that the complaint prays for affirmative relief against the insurance company. It will be noticed further that it is stipulated that the insurance company has persistently refused to change the beneficiary as desired by the plaintiff. As the rights of Andrea

Zialcita in the policy are rights which are enforceable by her only against the insurance company, the defendant insurance company will only be fully protected if the question at issue is conclusively determined. Accordingly, we have decided not to accede to the motion of the appellee and not to order the dismissal of the appeal of the appellant. This brings us to the main issue. Before, however, discussing its legal aspects, it is advisable to have before us the essential facts. As they are stipulated, this part of the decision can easily be accomplished. On January 29, 1910, the Sun Life Assurance Co. of Canada issued insurance policy No. 161481 on the life of Hilario Gercio. The policy was what is known as a twenty-year endowment policy. By its terms, the insurance company agreed to insure the life of Hilario Gercio for the sum of P/2,000, to be paid him on February 1, 1930, or if the insured should die before said date, then to his wife, Mrs. Andrea Zialcita, should she survive him; otherwise to the executors, administrators, or assigns of the insured. The policy also contained a schedule of reserves, amounts in cash, paid-up policies, and renewed insurance, guaranteed. The policy did not include any provision reserving to the insured the right to change the beneficiary. On the date the policy was issued, Andrea Zialcita was the lawful wife of Hilario Gercio. Towards the end of the year 1919, she was convicted of the crime of adultery. On September 4, 1920, a decree of divorce was issued in civil case no. 17955, which had the effect of completely dissolving the bonds of matrimony contracted by Hilario Gercio and Andrea Zialcita. On March 4, 1922, Hilario Gercio formally notified the Sun Life Assurance Co. of Canada that he had revoked his donation in favor of Andrea Zialcita, and that he had designated in her stead his present wife, Adela Garcia de Gercio, as the beneficiary of the policy. Gercio requested the insurance company to eliminate Andrea Zialcita as beneficiary. This, the insurance company has refused and still refuses to do. With all of these introductory matters disposed of and with the legal question to the forefront, it becomes our first duty to determine what law should be applied to the facts. In this connection, it should be remembered that the insurance policy was taken out in 1910, that the Insurance Act. No. 2427, became effective in 1914, and that the effort to change the beneficiary was made in 1922. Should the provisions of the Code of Commerce and the Civil Code in force in 1910, or the provisions of the Insurance Act now in force, or the general principles of law, guide the court in its decision? On the supposition, first, that the Code of Commerce is applicable, yet there can be found in it no provision either permitting or prohibiting the insured to change the beneficiary. On the supposition, next, that the Civil Code regulates insurance contracts, it would be most difficult, if indeed it is practicable, to test a life insurance policy by its provisions. Should the insurance contract, whereby the husband names the wife as the beneficiary,

be denominated a donation inter vivos, a donation causa mortis, a contract in favor of a third person, or an aleatory contract? The subject is further complicated by the fact that if an insurance contract should be considered a donation, a husband may then never insure his life in favor of his wife and vice versa, inasmuch as article 1334 prohibits all donations between spouses during marriage. It would seem, therefore, that this court was right when in the case of Del Val vs. Del Val ([1915]), 29 Phil., 534), it declined to consider the proceeds of the insurance policy as a donation or gift, saying "the contract of life insurance is a special contract and the destination of the proceeds thereof is determined by special laws which deal exclusively with that subject. The Civil Code has no provisions which relate directly and specifically to life-insurance contracts or to the destination of life-insurance proceeds. . . ." Some satisfaction is gathered from the perplexities of the Louisiana Supreme Court, a civil law jurisdiction, where the jurists have disagreed as to the classification of the insurance contract, but have agreed in their conclusions as will hereafter see. (Re Succession of Leone Desforges [1914], 52 L.R.A. [N.S.], 689; Lambert vs Penn Mutual Life Insurance Company of Philadelphia and L'Hote & Co. [1898], 50 La. Ann., 1027.) On the further supposition that the Insurance Act applies, it will be found that in this Law, there is likewise no provision either permitting or prohibiting the insured to change the beneficiary. We must perforce conclude that whether the case be considered as of 1910, or 1914, or 1922, and whether the case be considered in the light of the Code of Commerce, the Civil Code, or the Insurance Act, the deficiencies in the law will have to be supplemented by the general principles prevailing on the subject. To that end, we have gathered the rules which follow from the best considered American authorities. In adopting these rules, we do so with the purpose of having the Philippine Law of Insurance conform as nearly as possible to the modern Law of Insurance as found in the United States proper. The wife has an insurable interest in the life of her husband. The beneficiary has an absolute vested interest in the policy from the date of its issuance and delivery. So when a policy of life insurance is taken out by the husband in which the wife is named as beneficiary, she has a subsisting interest in the policy. And this applies to a policy to which there are attached the incidents of a loan value, cash surrender value, an automatic extension by premiums paid, and to an endowment policy, as well as to an ordinary life insurance policy. If the husband wishes to retain to himself the control and ownership of the policy he may so provide in the policy. But if the policy contains no provision authorizing a change of beneficiary without the beneficiary's consent, the insured cannot make such change. Accordingly, it is held that a life insurance policy of a husband made payable to the wife as beneficiary, is the separate property of the beneficiary and beyond the control of the husband. As to the effect produced by the divorce, the Philippine Divorce Law, Act No. 2710, merely provides in section 9 that the decree of divorce shall dissolve the community property as soon as such decree becomes final. Unlike the statutes of a few

jurisdictions, there is no provision in the Philippine Law permitting the beneficiary in a policy for the benefit of the wife of the husband to be changed after a divorce. It must follow, therefore, in the absence of a statute to the contrary, that if a policy is taken out upon a husband's life the wife is named as beneficiary therein, a subsequent divorce does not destroy her rights under the policy. These are some of the pertinent principles of the Law of Insurance. To reinforce them, we would, even at the expense of clogging the decision with unnecessary citation of authority, bring to notice certain decisions which seem to us to have controlling influence. To begin with, it is said that our Insurance Act is mostly taken from the statute of California. It should prove of interest, therefore, to know the stand taken by the Supreme Court of that State. A California decision oft cited in the Cyclopedias is Yore vs. Booth ([1895]), 110 Cal., 238; 52 Am. St. Rep., 81), in which we find the following: . . . It seems to be the settled doctrine, with but slight dissent in the courts of this country, that a person who procures a policy upon his own life, payable to a designated beneficiary, although he pays the premiums himself, and keeps the policy in his exclusive possession, has no power to change the beneficiary, unless the policy itself, or the charter of the insurance company, so provides. In policy, although he has parted with nothing, and is simply the object of another's bounty, has acquired a vested and irrevocable interest in the policy, which he may keep alive for his own benefit by paying the premiums or assessments if the person who effected the insurance fails or refuses to do so. As carrying great weight, there should also be taken into account two decisions coming from the Supreme Court of the United States. The first of these decisions, in point of time, is Connecticut Mutual Life Insurance Company vs Schaefer ([1877]), 94 U.S., 457). There, Mr. Justice Bradley, delivering the opinion of the court, in part said: This was an action on a policy of the court, in part said: July 25, 1868, on the joint lives of George F. and Francisca Schaefer, then husband and wife, payable to the survivor on the death of either. In January, 1870, they were divorced, and alimony was decreed and paid to the wife, and there was never any issue of the marriage. They both subsequently married again, after which, in February, 1871, George F. Schaefer died. This action was brought by Francisca, the survivor. xxx xxx xxx

The other point, relating to the alleged cessation of insurable interest by reason of the divorce of the parties, is entitled to more serious consideration, although we have very little difficulty in disposing of it. It will be proper, in the first place, to ascertain what is an insurable interest. It is generally agreed that mere wager policies, that is, policies in which the insured

party has no interest in its loss or destruction, are void, as against public policy. . . . But precisely what interest is necessary, in order to take a policy out of the category of mere wager, has been the subject of much discussion. In marine and fire insurance the difficulty is not so great, because there insurance is considered as strictly an indemnity. But in life insurance the loss can seldom be measured by pecuniary values. Still, an interest of some sort in the insured life must exist. A man cannot take out insurance on the life of a total stranger, nor on that of one who is not so connected with him as to make the continuance of the life a matter of some real interest to him. It is well settled that a man has an insurable interest in his own life and in that of his wife and children; a woman in the life of her husband; and the creditor in the life of his debtor. Indeed it may be said generally that any reasonable expectation of pecuniary benefit or advantage from the continued life of another creates an insurable interest in such life. And there is no doubt that a man may effect an insurance on his own life for the benefit of a relative or fried; or two or more persons, on their joint lives, for the benefit of the survivor or survivors. The old tontines were based substantially on this principle, and their validity has never been called in question. xxx xxx xxx

The policy in question might, in our opinion, be sustained as a joint insurance, without reference to any other interest, or to the question whether the cessation of interest avoids a policy good at its inception. We do not hesitate to say, however, that a policy taken out in good faith and valid at its inception, is not avoided by the cessation of the insurable interest, unless such be the necessary effect of the provisions of the policy itself. . . . . . . .In our judgment of life policy, originally valid, does not cease to be so by the cessation of the assured party's interest in the life insured. Another controlling decision of the United States Supreme Court is that of the Central National Bank of Washington City vs. Hume ([1888], 128 U.S., 134). Therein, Mr. Chief Justice Fuller, as the organ of the court, announced the following doctrines: We think it cannot be doubted that in the instance of contracts of insurance with a wife or children, or both, upon their insurable interest in the life of the husband or father, the latter, while they are living, can exercise no power of disposition over the same without their consent, nor has he any interest therein of which he can avail himself; nor upon his death have his personal representatives or his creditors any interest in the proceeds of such contracts, which belong to the beneficiaries to whom they are payable. It is indeed the general rule that a policy, and the money to become due under it, belong, the moment it is issued, to the person or persons named in it as the

beneficiary or beneficiaries, and that there is no power in the person procuring the insurance, by any act of his, by deed or by will, to transfer to any other person the interest of the person named. A jurisdiction which found itself in somewhat the same situation as the Philippines, because of having to reconcile the civil law with the more modern principles of insurance, is Louisiana. In a case coming before the Federal Courts, In re Dreuil & Co. ([1915]), 221 Fed., 796), the facts were that an endowment insurance policy provided for payment of the amount thereof at the expiration of twenty years to the insured, or his executors, administrators, or assigns, with the proviso that, if the insured die within such period, payment was to be made to his wife if she survive him. It was held that the wife has a vested interest in the policy, of which she cannot be deprived without her consent. Foster, District Judge, announced: In so far as the law of Louisiana is concerned, it may also be considered settled that where a policy is of the semitontine variety, as in this case, the beneficiary has a vested right in the policy, of which she cannot be deprived without her consent. (Lambert vs Penn Mutual Life Ins. Co., 50 La. Ann., 1027; 24 South., 16.) (See in same connection a leading decision of the Louisiana Supreme Court, Re Succession of Leonce Desforges, [1914], 52 L.R.A. [N.S.], 689.) Some question has arisen as to the power of the insured to destroy the vested interest of the beneficiary in the policy. That point is well covered in the case of Entwistle vs. Travelers Insurance Company ([1902], 202 Pa. St., 141). To quote: . . . The interest of the wife was wholly contingent upon her surviving her husband, and she could convey no greater interest in the policy than she herself had. The interest of the children of the insured, which was created for them by the contract when the policy was issued; vested in them at the same time that the interest of the wife became vested in her. Both interests were contingent. If the wife die before the insured, she will take nothing under the policy. If the insured should die before the wife, then the children take nothing under the policy. We see no reason to discriminate between the wife and the children. They are all payees, under the policy, and together constitute the assured. The contingency which will determine whether the wife, or the children as a class will take the proceeds, has not as yet happened; all the beneficiaries are living, and nothing has occurred by which the rights of the parties are in any way changed. The provision that the policy may be converted into cash at the option of the holder does not change the relative rights of the parties. We agree entirely with the suggestion that "holder" or "holders", as used in this connection, means those who in law are the owners of the policy, and are entitled to the rights and benefits which may accrue under it; in other words, all the beneficiaries; in the present case, not only the wife, by the children of the insured. If for any reason, prudence required the conversion of the policy into cash, a guardian would have no special difficulty in reasonable protecting the interest of his wards. But

however that may be, it is manifest that the option can only be exercised by those having the full legal interest in the policy, or by their assignee. Neither the husband, nor the wife, nor both together had power to destroy the vested interest of the children in the policy. The case most nearly on all fours with the one at bar is that of Wallace vs Mutual Benefit Life Insurance Co.([1906], 97 Minn., 27; 3 L.R.A. [N.S.], 478). The opinion there delivered also invokes added interest when it is noted that it was written by Mr. Justice Elliott, the author of a text on insurance, later a member of this court. In the Minnesota case cited, one Wallace effected a "twenty-year endowment" policy of insurance on his life, payable in the event of his death within twenty years to Emma G. Wallace, his wife, but, if he lived, to himself at the end of twenty years. If Wallace died before the death of his wife, within the twenty years, the policy was payable to the personal representatives of the insured. During the pendency of divorce proceedings, the parties signed a contract by which Wallace agreed that, if a divorce was granted to Mrs. Wallace, the court might award her certain specified property as alimony, and Mrs. Wallace agreed to relinquish all claim to any property arising out of the relation of husband and wife. The divorce was granted. An action was brought by Wallace to compel Mrs. Wallace to relinquish her interest in the insurance policy. Mr. Justice Elliott said: As soon as the policy was issued Mrs. Wallace acquired a vested interest therein, of which she could not be deprived without her consent, except under the terms of the contract with the insurance company. No right to change the beneficiary was reserved. Her interest in the policy was her individual property, subject to be divested only by her death, the lapse of time, or by the failure of the insured to pay the premiums. She could keep the policy alive by paying the premiums, if the insured did not do so. It was contingent upon these events, but it was free from the control of her husband. He had no interest in her property in this policy, contingent or otherwise. Her interest was free from any claim on the part of the insured or his creditors. He could deprive her of her interest absolutely in but one way, by living more than twenty years. We are unable to see how the plaintiff's interest in the policy was primary or superior to that of the husband. Both interests were contingent, but they were entirely separate and distinct, the one from the other. The wife's interest was not affected by the decree of court which dissolved the marriage contract between the parties. It remains her separate property, after the divorce as before. . . . . . . The fact that she was his wife at the time the policy was issued may have been, and undoubtedly was, the reason why she was named as beneficiary in the event of his death. But her property interest in the policy after it was issued did not in any reasonable sense arise out of the marriage relation. Somewhat the same question came before the Supreme Court of Kansas in the leading case of Filley vs. Illinois Life Insurance Company ([1914]), 91 Kansas, 220; L.R.A. [1915 D], 130). It was held, following consideration extending to two motions for rehearing, as follows:

The benefit accruing from a policy of life insurance upon the life of a married man, payable upon his death to his wife, naming her, is payable to the surviving beneficiary named, although she may have years thereafter secured a divorce from her husband, and he was thereafter again married to one who sustained the relation of wife to him at the time of his death. The rights of a beneficiary in an ordinary life insurance policy become vested upon the issuance of the policy, and can thereafter, during the life of the beneficiary, be defeated only as provided by the terms of the policy. If space permitted, the following corroborative authority could also be taken into account: Joyce, The Law of Insurance, second edition, vol. 2, pp. 1649 et seq.; 37 Corpus Juris, pp. 394 et seq.; 14 R.C.L., pp. 1376 et seq.;Green vs. Green ([1912], 147 Ky., 608; 39 L.R.A. [N.S.], 370); Washington Life Insurance Co. vs. Berwald ([1903], 97 Tex., 111); Begley vs. Miller ([1907]), 137 Ill., App., 278); Blum vs. New York L. Ins. Co. ([1906], 197 Mo., 513; 8 L.R.A. [N.S.], 923; Union Central Life Ins. Co. vs. Buxer ([1900], 62 Ohio St., 385; 49 L.R.A., 737); Griffith vs. New York Life Ins. Co. ([1894], 101 Cal., 627; 40 Am. St. Rep., 96); Preston vs. Conn. Mut. L. Ins. Co. of Hartford([1902]); 95 Md., 101); Snyder vs. Supreme Ruler of Fraternal Mystic Circle ([1909], 122 Tenn. 248; 45 L.R.A. [N.S.], 209); Lloyd vs. Royal Union Mut. L. Ins. Co. ([1917], 245 Fed., 162); Phoenix Mut. L. Ins. Co. vs. Dunham([1878], 46 Conn., 79; 33 Am. Rep., 14); McKee vs. Phoenix Ins. Co. ([1859], 28 Mo., 383; 75 Am. Rep., 129);Supreme Council American Legion of Honor vs. Smith and Smith ([1889], 45 N.J. Eq., 466); Overhiser vs. Overhiser ([1900], 63 Ohio St., 77; 81 Am. St. Rep., 612; 50 L.R.A., 552); Condon vs. New York Life Insurance Co. ([1918], 183 Iowa, 658); with which compare Foster vs. Gile ([1880], 50 Wis., 603) and Hatch vs. Hatch ([1904], 35 Tex. Civ. App., 373). On the admitted facts and the authorities supporting the nearly universally accepted principles of insurance, we are irresistibly led to the conclusion that the question at issue must be answered in the negative. The judgment appealed from will be reversed and the complaint ordered dismissed as to the appellant, without special pronouncement as to the costs in either instance. So ordered. Street, Villamor, Ostrand, Johns, and Villa-Real, JJ., concur. Avancea, C.J., concurs in the result. Romualdez, J., took no part.

Separate Opinions JOHNSON, J., concurring in the result.

I agree with the majority of the court, that the judgment of the lower court should be revoked, but for a different reason. In my judgment, the question presented by the plaintiff is purely an academic one. The purpose of the petition is to have declared the rights of certain persons in an insurance policy which is not yet due and payable. It may never become due and payable. The premiums may not be paid, thereby rendering the contract of insurance of non effect, and many other things may occur, before the policy becomes due, which would render it non effective. The plaintiff and the other parties who are claiming an interest in said policy should wait until there is something due them under the same. For the courts to declare now who are the persons entitled to receive the amounts due, if they ever become due and payable, is impossible, for the reason that nothing may ever become payable under the contract of insurance, and for many reasons such persons may never have a right to receive anything when the policy does become due and payable. In my judgment, the action is premature and should have been dismissed

G.R. No. 34774

September 21, 1931

EL ORIENTE FABRICA DE TABACOS, INC., plaintiff-appellant, vs. JUAN POSADAS, Collector of Internal Revenue, defendant-appellee. Gibbs and McDonough and Roman Ozaeta for appellant. Attorney-General Jaranilla for appellee. MALCOLM, J.: The issue in this case is whether the proceeds of insurance taken by a corporation on the life of an important official to indemnify it against loss in case of his death, are taxable as income under the Philippine Income Tax Law. The parties submitted the case to the Court of First Instance of Manila for decision upon the following agreed statement of facts: 1. That the plaintiff is a domestic corporation duly organized and existing under and by virtue of the laws of the Philippine Islands, having its principal office at No. 732 Calle Evangelista, Manila, P.I.; and that the defendant is the duly appointed, qualified and acting Collector of Internal Revenue of the Philippine Islands. 2. That on March 18, 1925, plaintiff, in order to protect itself against the loss that it might suffer by reason of the death of its manager, A. Velhagen, who had had more than thirty-five (35) years of experience in the manufacture of cigars in the Philippine Islands, and whose death would be a serious loss to the plaintiff, procured from the Manufacturers Life Insurance Co., of Toronto, Canada, thru its local agent E.E. Elser, aninsurance policy on the life of the said A. Velhagen for the sum of $50,000, United States currency. 3. That the plaintiff, El Oriente, Fabrica de Tabacos, Inc., designated itself as the sole beneficiary of said policy on the life of its said manager. 4. That during the time the life insurance policy hereinbefore referred to was in force and effect plaintiff paid from its funds all the insurance premiums due thereon. 5. That the plaintiff charged as expenses of its business all the said premiums and deducted the same from its gross incomes as reported in its annual income tax returns, which deductions were allowed by the defendant upon a showing made by the plaintiff that such premiums were legitimate expenses of its (plaintiff's) business.

6. That the said A. Velhagen, the insured, had no interest or participation in the proceeds of said life insurance policy. 7. That upon the death of said A. Velhagen in the year 1929, the plaintiff received all the proceeds of the said life insurance policy, together with the interests and the dividends accruing thereon, aggregating P104,957.88. 8. That over the protest of the plaintiff, which claimed exemption under section 4 of the Income Tax Law, the defendant Collector of Internal Revenue assessed and levied the sum of P3,148.74 as income tax on the proceeds of the insurance policy mentioned in the preceding paragraph, which tax the plaintiff paid under instant protest on July 2, 1930; and that defendant overruled said protest on July 9, 1930. Thereupon, a decision was handed down which absolved the defendant from the complaint, with costs against the plaintiff. From this judgment, the plaintiff appealed, and its counsel now allege that: 1. That trial court erred in holding that section 4 of the Income Tax Law (Act No. 2833) is not applicable to the present case. 2. The trial court erred in reading into the law certain exceptions and distinctions not warranted by its clear and unequivocal provisions. 3. The trial court erred in assuming that the proceeds of the life insurance policy in question represented a net profit to the plaintiff when, as a matter of fact, it merely represented an indemnity, for the loss suffered by it thru the death of its manager, the insured. 4. The trial court erred in refusing to hold that the proceeds of the life insurance policy in question is not taxable income, and in absolving the defendant from the complaint. The Income Tax Law for the Philippines is Act No. 2833, as amended. It is divided into four chapters: Chapter I On Individuals, Chapter II On Corporations, Chapter III General Administrative Provisions, and Chapter IV General Provisions. In chapter I On Individuals, is to be found section 4 which provides that, "The following incomes shall be exempt from the provisions of this law: (a) The proceeds of life insurance policies paid to beneficiaries upon the death of the insured ... ." Section 10, as amended, in Chapter II On Corporations, provides that, There shall be levied, assessed, collected, and paid annually upon the total net income received in the preceding calendar year from all sources by every corporation ... a tax of three per centum upon such income ... ." Section 11 in the same chapter, provides the exemptions under the law, but neither here nor in any other section is reference made to the provisions of section 4 in Chapter I.

Under the view we take of the case, it is sufficient for our purposes to direct attention to the anomalous and vague condition of the law. It is certain that the proceeds of life insurance policies are exempt. It is not so certain that the proceeds of life insurance policies paid to corporate beneficiaries upon the death of the insured are likewise exempt. But at least, it may be said that the law is indefinite in phraseology and does not permit us unequivocally to hold that the proceeds of life insurance policies received by corporations constitute income which is taxable. The situation will be better elucidated by a brief reference to laws on the same subject in the United States. The Income Tax Law of 1916 extended to the Philippine Legislature, when it came to enact Act No. 2833, to copy the American statute. Subsequently, the Congress of the United States enacted its Income Tax Law of 1919, in which certain doubtful subjects were clarified. Thus, as to the point before us, it was made clear, when not only in the part of the law concerning individuals were exemptions provided for beneficiaries, but also in the part concerning corporations, specific reference was made to the exemptions in favor of individuals, thereby making the same applicable to corporations. This was authoritatively pointed out and decided by the United States Supreme Court in the case of United States vs. Supplee-Biddle Hardware Co. ( [1924], 265 U.S., 189), which involved facts quite similar to those before us. We do not think the decision of the higher court in this case is necessarily controlling on account of the divergences noted in the federal statute and the local statute, but we find in the decision certain language of a general nature which appears to furnish the clue to the correct disposition of the instant appeal. Conceding, therefore, without necessarily having to decide, the assignments of error Nos. 1 and 2 are not well taken, we would turn to the third assignment of error. It will be recalled that El Oriente, Fabrica de Tabacos, Inc., took out the insurance on the life of its manager, who had had more than thirty-five years' experience in the manufacture of cigars in the Philippines, to protect itself against the loss it might suffer by reason of the death of its manager. We do not believe that this fact signifies that when the plaintiff received P104,957.88 from the insurance on the life of its manager, it thereby realized a net profit in this amount. It is true that the Income Tax Law, in exempting individual beneficiaries, speaks of the proceeds of life insurance policies as income, but this is a very slight indication of legislative intention. In reality, what the plaintiff received was in the nature of an indemnity for the loss which it actually suffered because of the death of its manager. To quote the exact words in the cited case of Chief Justice Taft delivering the opinion of the court: It is earnestly pressed upon us that proceeds of life insurance paid on the death of the insured are in fact capital, and cannot be taxed as income under the Sixteenth Amendment. Eisner vs. Macomber, 252 U.S., 189, 207; Merchants' Loan & Trust Co. vs. Smietanka, 255 U.S., 509, 518. We are not required to meet this question. It is enough to sustain our construction of the act to say that

proceeds of a life insurance policy paid on the death of the insured are not usually classed as income. . . . Life insurance in such a case is like that of fire and marine insurance, a contract of indemnity. Central Nat. Bank vs. Hume, 128 U.S., 195. The benefit to be gained by death has no periodicity. It is a substitution of money value for something permanently lost, either in a house, a ship, or a life. Assuming, without deciding, that Congress could call the proceeds of such indemnity income, and validly tax it as such, we think that, in view of the popular conception of the life insurance as resulting in a single addition of a total sum to the resources of the beneficiary, and not in a periodical return, such a purpose on its part should be express, as it certainly is not here. Considering, therefore, the purport of the stipulated facts, considering the uncertainty of Philippine law, and considering the lack of express legislative intention to tax the proceeds of life insurance policies paid to corporate beneficiaries, particularly when in the exemption in favor of individual beneficiaries in the chapter on this subject, the clause is inserted "exempt from the provisions of this law," we deem it reasonable to hold the proceeds of the life insurance policy in question as representing an indemnity and not taxable income. The foregoing pronouncement will result in the judgment being reversed and in another judgment being rendered in favor of the plaintiff and against the defendant for the sum of P3,148.74. So ordered, without costs in either instance. Avancea, C.J., Street, Villamor, Ostrand, Romualdez, Villa-Real, and Imperial, JJ., concur.

G.R. No. L-6114

October 30, 1954

SOUTHERN LUZON EMPLOYEES' ASSOCIATION, plaintiff, vs. JUANITA GOLPEO, ET AL., defendants-appellants; AQUILINO MALOLES , ET AL., defendants-appellees; ELSIE HICBAN, ET AL., defendants; MARCELINO CONCEPCION, ET AL., intervenors-appellants. Enrique Al. Capistrano, Pio O. Golfeo, Jose E. Erfe and Hilario Mutuc for appellants. Manuel Alvero and Elden B. Brion for appellees. Juan A. Baes for defendant Elsie Hicban. PARAS, C.J.: The plaintiff, Southern Luzon Employees' Association is composed of laborers and employees of Laguna tayabas Bus Co., and Batangas Transportation Company, and one of its purposes is mutual aid of its members and their defendants in case of death. Roman A. Concepcion was a member until his death on December 13, 1950. The association adopted on September 17, 1949 the following resolution: RESOLVED: That a family record card of each member be printed wherein the members will put down his dependents and/or beneficiaries. BE IT RESOLVED, FURTHER, that a member may, if he chooses, put down his common-law wife as his beneficiary and/or children had with her as the case may be; that in case of a widower, he may put down his legitimate children with the first marriage who are below 21 years of age, single, and may at the same time, also name his common-law wife, if he has any, as dependents and/or beneficiaries; and BE IT RESOLVED: That such person so named by the member will be sole persons to be recognized by the Association regarding claims for condolence contributions. In the form required by the association to be accomplished by its members, with reference to the death benefit, Roman A. Concepcion listed as his beneficiaries Aquilina Maloles, Roman M. Concepcion, Jr., Estela M. Concepcion, Rolando M. Concepcion and Robin M. Concepcion. After the death of Roman A. Concepcion, the association was able to collect voluntary contributions from its members amounting to P2,5055. Three sets of claimants presented themselves, namely, (1) Juanita Golpeo, legal wife of Roman A. Concepcion, and her children, named beneficiaries by the deceased; and (3) Elsie Hicban, another common law wife of Roman A. Concepcion, and her child. The plaintiff association was accordingly constrained to institute in the Court of First Instance of Laguna the present action for interpleading against the three conflicting claimants as defendants. Marcelino and Josefina Concepcion, children of the deceased Roman A.

Concepcion with Juanita Golpeo, intervened in their own rights, aligning themselves with the defendants, Juanita Golpeo and her minor children. After hearing, the court rendered a decision, declaring the defendants Aquilina Maloles and her children the sole beneficiaries of the sum of P2,505.00, and ordering the plaintiff to deliver said amount to them. From this decision only the defendants Juanita Golpeo and her minor children and the intervenors Marcelino and Josefina Concepcion have appealed to this court. The decision is based mainly on the theory that the contract between the plaintiff and the deceased Roman A. Concepcion partook of the nature of an insurance and that, therefore, the amount in question belonged exclusively to the beneficiaries, invoking the following pronouncements of this Court in the case of Del Val vs. Del Val, 29 Phil., 534: With the finding of the trial court that the proceeds of the life-insurance policy belongs exclusively to the defendant as his individual and separate property, we agree. That the proceeds of an insurance policy belong exclusively to the beneficiary and not to the estate of the person whose life was insured, and that such proceeds are the separate and individual property of the beneficiary, and not of the heirs of the person whose life was insured, is the doctrine in America. We believe that the same doctrine obtains in these Islands by virtue of section 428 of the Code of Commerce, which reads: "The amounts which the underwriter must deliver to the person insured, in fulfillment of the contract, shall be the property creditors of any kind whatsoever of the person who effected the insurance in favor of the formers." It is claimed by the attorney for the plaintiffs that the section just quoted in subordinated to the provisions of the civil code as found in article 10035. This article reads: "An heir by force of law surviving with others of the same character to a succession must bring into the hereditary estate the property or securities he may bring into the hereditary estate the property or securities he may have been received from the deceased during the life of the same, by way of dowry, gift, or for any good consideration, in order to compute it in fixing the legal portions and in the amount of the division." Counsel also claims that the proceed of the insurance policy were donation or gift made by the father during his lifetime to the defendant and that, as such, its ultimate destination is determined by those provisions of the Civil Code which relate to donations, especially article 819. This article provides that "gifts made to children which are not betterments shall be considered as part of their legal portion." We cannot agree with these contention. The contract of life insurance is a special contract and the destination of the proceeds thereof is determined by special

laws which deal exclusively with that subject. The Civil Code has no provisions which relate directly and specifically to life-insurance contract or to the destination of life-insurance proceeds. That subject is regulate exclusively by the Code of Commerce which provides for the terms of the contract, the relations of the parties and the destination of the proceeds of the policy. (Supra, pp. 540541.) It is argued for the appellants, however, that the Insurance Law is not applicable because the plaintiff is a mutual benefit association as defined in section 1628 of the Revised Administrative Code. This argument evidently ignore the fact that the trial court has no considered the plaintiff as a regular insurance company but merely ruled that the death benefit in question is analogous to an insurance. Moreover, section 1628 of the Revised Administrative Code defines a mutual benefit association as one, among others, "providing for any method of accident or life insurance among its members out of dues or assessments collected from the membership." The comparison made in the appealed decision is, therefore, well taken. Appellant also contend that the stipulation between the plaintiff and the deceased Roman A. Concepcion regarding the specification of the latter's beneficiaries, and the resolution of September 17, 1949, are void for the being contrary to law, moral or public policy. Specifically, the appellants cite article 2012 of the new Civil Code providing that "Any person who is forbidden from receiving any donation under article 739 cannot be named beneficiary of a life insurance policy and by the person who cannot make any donation to him, according to said article." Inasmuch as, according to article 739 of the new Civil Code, a donation is valid when made "between persons who are guilty or adultery or concubinage at the time of the donation," it is alleged that the defendantappellee Aquilina Maloles, cannot be named a beneficiary, every assuming that the insurance law is applicable. Without considering the intimation in the brief for the defendant appellees that appellant Juanita Golpeo, by her silence and actions, had acquiesced in the illicit relations between her husband and appellee Aquilina Maloles, appellant argument would certainly not apply to the children of Aquilina likewise named beneficiaries by the deceased Roman A. Concepcion. As a matter of a fact the new Civil Code recognized certain successional rights of illegitimate children. (Article 287.) The other contention advanced rather exhaustively by counsel for appellants, and the citations in support there of are either negative or rendered inapplicable by the decisive considerations already stated. In this connection it is noteworthy that the estate of the deceased Roman A. Concepcion was not entirely left without anything legally due it since it is an admitted fact that the sum of P2,500 was paid by Laguna Tayabas Bus Co., employer of the deceased to the appellants under the Workmen's Compensation Act. Wherefore, the appealed decision is affirmed, and it is so ordered without costs. Bengzon, Jugo and Bautista Angelo, JJ., concur. Padilla and Reyes, A., JJ., concur in the result.

Separate Opinions REYES, J.B.L., J., concurring: I concur in the result for the reason that the contract here involved was perfected before the new Civil Code took effect, and hence its provisions cannot be made to apply retroactively. Concepcion and Montemayor, JJ., concur.

G.R. No. L-22796

June 26, 1967

DELFIN NARIO, and ALEJANDRA SANTOS-NARIO, plaintiffs-appellants, vs. THE PHILIPPINE AMERICAN LIFE INSURANCE COMPANY, defendant-appellee. Ricardo T. Bancod and Severino C. Zarasate for plaintiffs-appellants. M. Lim, M. Y. Macias and Associates for defendant-appellee. REYES, J.B.L., J.: Direct appeal, on pure question of law, from a decision of the Court of First Instance of Manila, in its Civil Case No. 54942, dismissing plaintiffs' complaint as well as from a later order of the same court, denying a motion to set aside and/or reconsider said decision of dismissal. The facts of this case may be stated briefly as follows: Mrs. Alejandra Santos-Mario was, upon application, issued, on June 12, 1959, by the Philippine American Life Insurance Co., a life insurance policy (No. 503617) under a 20year endowment plan, with a face value of P5,000.00. She designated thereon her husband, Delfin Nario, and their unemancipated minor son, Ernesto Nario, as her irrevocable beneficiaries. About the middle of June, 1963, Mrs. Nario applied for a loan on the above stated policy with the Insurance Company, which loan she, as policy-holder, has been entitled to avail of under one of the provisions of said policy after the same has been in force for three (3) years, for the purpose of using the proceeds thereof for the school expenses of her minor son, Ernesto Nario. Said application bore the written signature and consent of Delfin Nario in two capacities: first, as one of the irrevocable beneficiaries of the policy; and the other, as the father-guardian of said minor son and irrevocable beneficiary, Ernesto Nario, and as the legal administrator of the minor's properties, pursuant to Article 320 of the Civil Code of the Philippines. The Insurance Company denied said application, manifesting to the policy holder that the written consent for the minor son must not only be given by his father as legal guardian but it must also be authorized by the court in a competent guardianship proceeding. After the denial of said policy loan application, Mrs. Nario signified her decision to surrender her policy to the Insurance Company, which she was also entitled to avail of under one of the provisions of the same policy, and demanded its cash value which then amounted to P520.00. The Insurance Company also denied the surrender of the policy, on the same ground as that given in disapproving the policy loan application; hence, on September 10, 1963,

Mrs. Alejandra Santos-Nario and her husband, Delfin Nario, brought suit against the Philippine American Life Insurance Co. in the above mentioned court of first instance, seeking to compel the latter (defendant) to grant their policy loan application and/or to accept the surrender of said policy in exchange for its cash value.1wph1.t Defendant Insurance Company answered the complaint, virtually admitting its material allegations, but it set up the affirmative defense that inasmuch as the policy loan application and the surrender of the policy involved acts of disposition and alienation of the property rights of the minor, said acts are not within the powers of the legal administrator, under article 320 in relation to article 326 of the Civil Code; hence, mere written consent given by the father-guardian, for and in behalf of the minor son, without any court authority therefor, was not a sufficient compliance of the law, and it (defendant Insurance Company) was, therefore, justified in refusing to grant and in disapproving the proposed transactions in question. There having been no substantial disagreement or dispute as to any material fact, the parties, upon joint motion which the lower court granted, dispensed with the presentation of evidence and submitted their respective memoranda, after which the case was considered submitted for decision. The lower court found and opined that since the parties expressly stipulated in the endorsement attached to the policy and which formed part thereof that It is hereby understood and agreed that, notwithstanding the provisions of this Policy to the contrary, inasmuch as the designation of the beneficiaries have been made by the Insured without reserving the right to change said beneficiaries, the Insured may not designate a new beneficiary or assign, release or surrender this Policy to the Company and exercise any and all other rights and privileges hereunder or agree with the Company to any change in or amendment to this Policy, without the consent of the beneficiaries originally designated; that under the above quoted provision, the minor son, as one of the designated irrevocable beneficiaries, "acquired a vested right to all benefits accruing to the policy, including that of obtaining a policy loan to the extent stated in the schedule of values attached to the policy (Gercio vs. Sun Life Assurance of Canada, 48 Phil. 53, 58)"; that the proposed transactions in question (policy loan and surrender of policy) involved acts of disposition or alienation of the minor's properties for which the consent given by the father-guardian for and in behalf of the minor son, must be with the requisite court authority (U.S.V.A. vs. Bustos, 92 Phil. 327; Visaya vs. Suguitan, G.R. No. L-8300, November 18, 1955; 99 Phil. 1004 [unrep] and in the case at bar, such consent was given by the father-guardian without any judicial authority; said court, agreeing with defendant's contention, sustained defendant's affirmative defense, and rendered, on January 28, 1964, its decision dismissing plaintiffs' complaint. Unable to secure reconsideration of the trial Court's ruling, petitioner appealed directly to this Court, contending that the minor's interest amounted to only one-half of the

policy's cash surrender value of P520.00; that under Rule 96, Section 2 of the Revised Rules of Court, payment of the ward's debts is within the powers of the guardian, where no realty is involved; hence, there is no reason why the father may not validly agree to the proposed transaction on behalf of the minor without need of court authority. The appeal is unmeritorious. We agree with the lower court that the vested interest or right of the beneficiaries in the policy should be measured on its full face value and not on its cash surrender value, for in case of death of the insured, said beneficiaries are paid on the basis of its face value and in case the insured should discontinue paying premiums, the beneficiaries may continue paying it and are entitled to automatic extended term or paid-up insurance options, etc. and that said vested right under the policy cannot be divisible at any given time. We likewise agree with the conclusion of the lower court that the proposed transactions in question (policy loan and surrender of policy) constitute acts of disposition or alienation of property rights and not merely of management or administration because they involve the incurring or termination of contractual obligations. As above noted, the full face value of the policy is P5,000.00 and the minor's vested interest therein, as one of the two (2) irrevocable beneficiaries, consists of one-half () of said amount or P2,500.00. Article 320 of the Civil Code of the Philippines provides The father, or in his absence the mother, is the legal administrator of the property pertaining to the child under parental authority. If the property is worth more than two thousand pesos, the father or mother shall give a bond subject to the approval of the Court of First Instance. and article 326 of the same Code reads When the property of the child is worth more than two thousand pesos, the father or mother shall be considered a guardian of the child's property, subject to the duties and obligations of guardians under the Rules of Court. The above quoted provisions of the Civil Code have already been implemented and clarified in our Revised Rules of Court which provides SEC. 7. Parents as guardians. When the property of the child under parental authority is worth two thousand pesos or less, the father or the mother, without the necessity of court appointment, shall be his legal guardian. When the property of the child is worth more than two thousand pesos, the father or the mother shall be considered guardian of the child's property, with the duties and obligations of guardians under these rules, and shall file the petition required by Section 2 hereof. For good reasons the court may, however, appoint another suitable person. (Rule 93).

It appearing that the minor beneficiary's vested interest or right on the policy exceeds two thousand pesos (P2,000.00); that plaintiffs did not file any guardianship bond to be approved by the court; and as later implemented in the abovequoted Section 7, Rule 93 of the Revised Rules of Court, plaintiffs should have, but, had not, filed a formal application or petition for guardianship, plaintiffs-parents cannot possibly exercise the powers vested on them, as legal administrators of their child's property, under articles 320 and 326 of the Civil Code. As there was no such petition and bond, the consent given by the father-guardian, for and in behalf of the minor son, without prior court authorization, to the policy loan application and the surrender of said policy, was insufficient and ineffective, and defendant-appellee was justified in disapproving the proposed transactions in question. The American cases cited by appellants are not applicable to the case at bar for lack of analogy. In those cases, there were pending guardianship proceedings and the guardians therein were covered by bonds to protect the wards' interests, which circumstances are wanting in this case. The result would be the same even if we regarded the interest of the ward to be worth less than P2,000.00. While the father or mother would in such event be exempt from the duty of filing a bond, and securing judicial appointment, still the parent's authority over the estate of the ward as a legal-guardian would not extend to acts of encumbrance or disposition, as distinguished from acts of management or administration. The distinction between one and the other kind of power is too basic in our law to be ignored. Thus, under Article 1877 of the Civil Code of the Philippines, an agency in general terms does not include power to encumber or dispose of the property of the principal; and the Code explicitly requires a special power or authority for the agent "to loan or borrow money, unless the latter act be urgent or indispensable for the preservation of the thing under administration" (Art. 1878 no. 7). Similarly, special powers are required to required to effect novations, to waive any obligation gratuitously or obligate the principal as a guarantor or surety (Do., nos. 2, 4 and 11). By analogy, since the law merely constitutes the parent as legal administrator of the child's property (which is a general power), the parent requires special authority for the acts above specified, and this authority can be given only by a court. This restricted interpretation of the parent's authority becomes all the more necessary where as in the case before us, there is no bond to guarantee the ward against eventual losses. Appellants seek to bolster their petition by invoking the parental power (p atria potestas) under the Civil Code of 1889, which they claim to have been revived by the Civil Code of the Philippines (Rep. Act 386). The appeal profits them nothing. For the new Civil Code has not effected a restitutio in integrum of the Spanish patria potestas; the revival has been only in part. And, significantly, the Civil Code now in force did not reenact Article 164 of the Civil Code of 1889, that prohibited the alienation by the parents of the real property owned by the child without court authority and led the commentators and interpreters of said Code to infer that the parents could by themselves alienate the child's movable property. The omission of any equivalent precept in the Civil Code now in force proves the absence of any authority in the parents to carry out now acts of

disposition or alienation of the child's goods without court approval, as contended by the appellee and the court below. Wherefore, the decision appealed from is affirmed. Costs against appellants Nario. So ordered. Concepcion, C.J., Dizon, Makalintal, Bengzon, J.P., Zaldivar, Sanchez and Castro, JJ., concur. Regala, J., took no part.

August 31, 1948 G.R. No. L-2227 Intestate estate of the late Esperanza J. Villanueva. MARIANO J. VILLANUEVA, claimant-appellant, vs. PABLO ORO, administrator. Nicolas P. Nonato for claimant and appellant. Rodrigo J. Harder for administrator and appellee. Paras (Edgardo), J.: The West Coast Life Insurance Company issued two policies of insurance on the life of Esperanza J. Villanueva, one for two thousand pesos and maturing on April 1, 1943, and the other for three thousand pesos and maturing on March 31, 1943. In both policies (with corresponding variation in amount and date of maturity) the insurer agreed to pay two thousand pesos, at the home office of the Company, in San Francisco, California, to the insured hereunder, if living, on the 1st day of April 1943, or to the beneficiary Bartolome Villanueva, father of the insured, immediately upon receipt of due proof of the prior death of the insured, Esperanza J. Villanueva, of La Paz, Philippine Islands, during the continuance of this policy, with right on the part of the insured to change the beneficiary. After the death of Bartolome Villanueva in 1940, the latter was duly substituted as beneficiary under the policies by Mariano J. Villanueva, a brother of the insured. Esperanza J. Villanueva survived the insurance period, for she died only on October 15, 1944, without, however, collecting the insurance proceeds. Adverse claims for said proceeds were presented by the estate of Esperanza J. Villanueva on the one hand and by Mariano J. Villanueva on the other, which conflict was squarely submitted in the intestate proceedings of Esperanza J. Villanueva pending in the Court of First Instance of Iloilo. From an order, dated February 26, 1947, holding the estate of the insured is entitled to the insurance proceeds, to the exclusion of the beneficiary, Mariano J. Villanueva, the latter has interposed the present appeal. The lower court committed no error. Under the policies, the insurer obligated itself to pay the insurance proceeds (1) to the insured if the latter lived on the dates of maturity or (2) to the beneficiary if the insured died during the continuance of the policies. The first contingency of course excludes the second, and vice versa. In other words, as the insured Esperanza J. Villanueva was living on April 1, and March 31, 1943, the proceeds are payable exclusively to her estate unless she had before her death otherwise assigned the matured policies. (It is not here pretended and much less proven, that there was such assignment.) The beneficiary, Mariano J. Villanueva, could be entitled to said proceeds only in default of the first contingency. To sustain the beneficiarys claim would be altogether eliminate from the policies the condition that the insurer agrees to pay . . . to the insured hereunder, if living.

There is nothing there in the Insurance Law (Act No. 2427) that militates against the construction placed by the lower court on the disputed condition appearing in the two policies now under advisement. On the contrary, said law provides that an insurance upon life may be made payable on the death of the death of the person, or on his surviving a specified period, or otherwise, contingently on the continuance or cessation of life (section 165), and that a policy of insurance upon life or health mat pass by transfer, will, or succession, to any person, whether he has an insurable interest or not, and such person may recover upon it whatever the insured might have recovered (section 166). Counsel for the beneficiary invokes the decision in Del Val vs. Del Val, 29 Phil. 534, 540, in which it was held that the proceeds of an insurance policy belong exclusively to the beneficiary and not to the estate of the person whose life was insured, and that such proceeds are the separate and individual property of the beneficiary, and not of the heirs of the person whose life was insured. This citation is clearly not controlling, first, because it does not appear therein that the insurance contract contained the stipulation appearing in the policies issued on the life of Esperanza J. Villanueva and on which the appealed order in the case at bar is based; and, secondly, because the Del Val doctrine was made upon the authority of the provisions of the Code of Commerce relating to insurance (particularly section 428) which had been expressly repealed by the present Insurance Act No. 2427. Our pronouncement is not novel, since it tallies with the following typical American authorities: If a policy of insurance provides that the proceeds shall be payable to t he assured, if he lives to a certain date, and, in case of his death before that date, then they shall be payable to the beneficiary designated, the interest of the beneficiary is a contingent one, and the benefit of the policy will only inure to such beneficiary in case the assured dies before the end of the period designated in the policy. (Couch, Cyclopedia of Insurance Law, Vol. 2, sec. 343. p. 1023.) Under endowment of tontine policies payable to the insured at the expiration of a certain period, if alive, but providing for the payment of a stated sum to a designated beneficiary in case of the insured death during the period mentioned, the insured and the beneficiary take contingent interests. The interest of the insured in the proceeds of the insurance depends upon his survival of the expiration of endowment period. Upon the insureds death, within the period, the beneficiary will take, as against the personal representative or the assignee of the insured. Upon the other hand, if the insured survives the endowment period, the benefits are payable to him or to his assignee, notwithstanding a beneficiary is designated in the policy. (29 Am. Jur., section 1277, pp. 952, 953.). The appealed order is, therefore, hereby affirmed, and it is so ordered with costs against the appellant. Feria, Pablo, Perfecto, Bengzon, Briones, Padilla, and Tuason, JJ., concur.

G.R. No. L-54216 July 19, 1989 THE PHILIPPINE AMERICAN INSURANCE COMPANY, petitioner, vs. HONORABLE GREGORIO G. PINEDA in his capacity as Judge of the Court of First Instance of Rizal, and RODOLFO C. DIMAYUGA, respondents.

PARAS, J.: Challenged before Us in this petition for review on certiorari are the Orders of the respondent Judge dated March 19, 1980 and June 10, 1980 granting the prayer in the petition in Sp. Proc. No. 9210 and denying petitioner's Motion for Reconsideration, respectively. The undisputed facts are as follows: On January 15, 1968, private respondent procured an ordinary life insurance policy from the petitioner company and designated his wife and children as irrevocable beneficiaries of said policy. Under date February 22, 1980 private respondent filed a petition which was docketed as Civil Case No. 9210 of the then Court of First Instance of Rizal to amend the designation of the beneficiaries in his life policy from irrevocable to revocable. Petitioner, on March 10, 1980 filed an Urgent Motion to Reset Hearing. Also on the same date, petitioner filed its Comment and/or Opposition to Petition. When the petition was called for hearing on March 19, 1980, the respondent Judge Gregorio G. Pineda, presiding Judge of the then Court of First Instance of Rizal, Pasig Branch XXI, denied petitioner's Urgent Motion, thus allowing the private respondent to adduce evidence, the consequence of which was the issuance of the questioned Order granting the petition. Petitioner promptly filed a Motion for Reconsideration but the same was denied in an Order June 10, 1980. Hence, this petition raising the following issues for resolution: I WHETHER OR NOT THE DESIGNATION OF THE IRREVOCABLE BENEFICIARIES COULD BE CHANGED OR AMENDED WITHOUT THE CONSENT OF ALL THE IRREVOCABLE BENEFICIARIES. II

WHETHER OR NOT THE IRREVOCABLE BENEFICIARIES HEREIN, ONE OF WHOM IS ALREADY DECEASED WHILE THE OTHERS ARE ALL MINORS, COULD VALIDLY GIVE CONSENT TO THE CHANGE OR AMENDMENT IN THE DESIGNATION OF THE IRREVOCABLE BENEFICIARIES. We are of the opinion that his Honor, the respondent Judge, was in error in issuing the questioned Orders. Needless to say, the applicable law in the instant case is the Insurance Act, otherwise known as Act No. 2427 as amended, the policy having been procured in 1968. Under the said law, the beneficiary designated in a life insurance contract cannot be changed without the consent of the beneficiary because he has a vested interest in the policy (Gercio v. Sun Life Ins. Co. of Canada, 48 Phil. 53; Go v. Redfern and the International Assurance Co., Ltd., 72 Phil. 71). In this regard, it is worth noting that the Beneficiary Designation Indorsement in the policy which forms part of Policy Number 0794461 in the name of Rodolfo Cailles Dimayuga states that the designation of the beneficiaries is irrevocable (Annex "A" of Petition in Sp. Proc. No. 9210, Annex "C" of the Petition for Review on Certiorari), to wit: It is hereby understood and agreed that, notwithstanding the provisions of this policy to the contrary, inasmuch as the designation of the primary/contingent beneficiary/beneficiaries in this Policy has been made without reserving the right to change said beneficiary/ beneficiaries, such designation may not be surrendered to the Company, released or assigned; and no right or privilege under the Policy may be exercised, or agreement made with the Company to any change in or amendment to the Policy, without the consent of the said beneficiary/beneficiaries. (Petitioner's Memorandum, p. 72, Rollo) Be it noted that the foregoing is a fact which the private respondent did not bother to disprove. Inevitably therefore, based on the aforequoted provision of the contract, not to mention the law then applicable, it is only with the consent of all the beneficiaries that any change or amendment in the policy concerning the irrevocable beneficiaries may be legally and validly effected. Both the law and the policy do not provide for any other exception, thus, abrogating the contention of the private respondent that said designation can be amended if the Court finds a just, reasonable ground to do so. Similarly, the alleged acquiescence of the six (6) children beneficiaries of the policy (the beneficiary-wife predeceased the insured) cannot be considered an effective ratification to the change of the beneficiaries from irrevocable to revocable. Indubitable is the fact that all the six (6) children named as beneficiaries were minors at the time,** for which

reason, they could not validly give their consent. Neither could they act through their father insured since their interests are quite divergent from one another. In point is an excerpt from the Notes and Cases on Insurance Law by Campos and Campos, 1960, readingThe insured ... can do nothing to divest the beneficiary of his rights without his consent. He cannot assign his policy, nor even take its cash surrender value without the consent of the beneficiary. Neither can the insured's creditors seize the policy or any right thereunder. The insured may not even add another beneficiary because by doing so, he diminishes the amount which the beneficiary may recover and this he cannot do without the beneficiary's consent. Therefore, the parent-insured cannot exercise rights and/or privileges pertaining to the insurance contract, for otherwise, the vested rights of the irrevocable beneficiaries would be rendered inconsequential. Of equal importance is the well-settled rule that the contract between the parties is the law binding on both of them and for so many times, this court has consistently issued pronouncements upholding the validity and effectivity of contracts. Where there is nothing in the contract which is contrary to law, good morals, good customs, public policy or public order the validity of the contract must be sustained. Likewise, contracts which are the private laws of the contracting parties should be fulfilled according to the literal sense of their stipulations, if their terms are clear and leave no room for doubt as to the intention of the contracting parties, for contracts are obligatory, no matter in what form they may be, whenever the essential requisites for their validity are present (Phoenix Assurance Co., Ltd. vs. United States Lines, 22 SCRA 675, Phil. American General Insurance Co., Inc. vs. Mutuc, 61 SCRA 22.) In the recent case of Francisco Herrera vs. Petrophil Corporation, 146 SCRA 385, this Court ruled that: ... it is settled that the parties may establish such stipulations, clauses, terms, and conditions as they may want to include; and as long as such agreements are not contrary to law, good morals, good customs, public policy or public order, they shall have the force of law between them. Undeniably, the contract in the case at bar, contains the indispensable elements for its validity and does not in any way violate the law, morals, customs, orders, etc. leaving no reason for Us to deny sanction thereto. Finally, the fact that the contract of insurance does not contain a contingency when the change in the designation of beneficiaries could be validly effected means that it was never within the contemplation of the parties. The lower court, in gratuitously providing

for such contingency, made a new contract for them, a proceeding which we cannot tolerate. Ergo, We cannot help but conclude that the lower court acted in excess of its authority when it issued the Order dated March 19, 1980 amending the designation of the beneficiaries from "irrevocable" to "revocable" over the disapprobation of the petitioner insurance company. WHEREFORE, premises considered, the questioned Orders of the respondent Judge are hereby nullified and set aside. SO ORDERED. Melencio-Herrera (Chairperson), Sarmiento and Regalado, JJ., concur. Padilla, J., took no part.

G.R. No. L-21642

July 30, 1966

SOCIAL SECURITY SYSTEM, petitioner-appellee, vs. CANDELARIA D. DAVAC, ET AL., respondents; LOURDES Tuplano, respondent-appellant. J. Ma. Francisco and N. G. Bravo for respondent-appellant. Office of the Solicitor General Arturo A. Alafriz, Solicitor Camilo D. Quiason and E. T. Duran for petitioner-appellee. BARRERA, J.: This is an appeal from the resolution of the Social Security Commission declaring respondent Candelaria Davac as the person entitled to receive the death benefits payable for the death of Petronilo Davac. The facts of the case as found by the Social Security Commission, briefly are: The late Petronilo Davac, a former employee of Lianga Bay Logging Co., Inc. became a member of the Social Security System (SSS for short) on September 1, 1957. As such member, he was assigned SS I.D. No. 08-007137. In SSS form E-1 (Member's Record) which he accomplished and filed with the SSS on November 21, 1957, he designated respondent Candelaria Davac as his beneficiary and indicated his relationship to her as that of "wife". He died on April 5, 1959 and, thereupon, each of the respondents (Candelaria Davac and Lourdes Tuplano) filed their claims for death benefit with the SSS. It appears from their respective claims and the documents submitted in support thereof, that the deceased contracted two marriages, the first, with claimant Lourdes Tuplano on August 29, 1946, who bore him a child, Romeo Davac, and the second, with Candelaria Davac on January 18, 1949, with whom he had a minor daughter Elizabeth Davac. Due to their conflicting claims, the processing thereof was held in abeyance, whereupon the SSS filed this petition praying that respondents be required to interpose and litigate between themselves their conflicting claims over the death benefits in question.1wph1.t On February 25, 1963, the Social Security Commission issued the resolution referred to above, Not satisfied with the said resolution, respondent Lourdes Tuplano brought to us the present appeal. The only question to be determined herein is whether or not the Social Security Commission acted correctly in declaring respondent Candelaria Davac as the person entitled to receive the death benefits in question. Section 13, Republic Act No. 1161, as amended by Republic Act No. 1792, in force at the time Petronilo Davac's death on April 5, 1959, provides: 1. SEC. 13. Upon the covered employee's death or total and permanent disability under such conditions as the Commission may define, before becoming eligible

for retirement and if either such death or disability is not compensable under the Workmen's Compensation Act, he or, in case of his death, his beneficiaries, as recorded by his employer shall be entitled to the following benefit: ... . (emphasis supplied.) Under this provision, the beneficiary "as recorded" by the employee's employer is the one entitled to the death benefits. In the case of Tecson vs. Social Security System, (L15798, December 28, 1961), this Court, construing said Section 13, said: It may be true that the purpose of the coverage under the Social Security System is protection of the employee as well as of his family, but this purpose or intention of the law cannot be enforced to the extent of contradicting the very provisions of said law as contained in Section 13, thereof, ... . When the provision of a law are clear and explicit, the courts can do nothing but apply its clear and explicit provisions (Velasco vs. Lopez, 1 Phil, 270; Caminetti vs. U.S., 242 U.S. 470, 61 L. ed. 442). But appellant contends that the designation herein made in the person of the second and, therefore, bigamous wife is null and void, because (1) it contravenes the provisions of the Civil Code, and (2) it deprives the lawful wife of her share in the conjugal property as well as of her own and her child's legitime in the inheritance. As to the first point, appellant argues that a beneficiary under the Social Security System partakes of the nature of a beneficiary in life insurance policy and, therefore, the same qualifications and disqualifications should be applied. Article 2012 of the New Civil Code provides: ART. 2012. Any person who is forbidden from receiving any donation under Article 739 cannot be named beneficiary of a life insurance policy by the person who cannot make any donation to him according to said article. And Article 739 of the same Code prescribes: ART. 739. The following donations shall be void: (1) Those made between persons who were guilty of adultery or concubinage at the time of the donation; xxx xxx xxx

Without deciding whether the naming of a beneficiary of the benefits accruing from membership in the Social Security System is a donation, or that it creates a situation analogous to the relation of an insured and the beneficiary under a life insurance policy, it is enough, for the purpose of the instant case, to state that the disqualification mentioned in Article 739 is not applicable to herein appellee Candelaria Davac because

she was not guilty of concubinage, there being no proof that she had knowledge of the previous marriage of her husband Petronilo.1 Regarding the second point raised by appellant, the benefits accruing from membership in the Social Security System do not form part of the properties of the conjugal partnership of the covered member. They are disbursed from a public special fund created by Congress in pursuance to the declared policy of the Republic "to develop, establish gradually and perfect a social security system which ... shall provide protection against the hazards of disability, sickness, old age and death." 2 The sources of this special fund are the covered employee's contribution (equal to 2- per cent of the employee's monthly compensation);3 the employer's contribution (equivalent to 3- per cent of the monthly compensation of the covered employee); 4 and the Government contribution which consists in yearly appropriation of public funds to assure the maintenance of an adequate working balance of the funds of the System. 5 Additionally, Section 21 of the Social Security Act, as amended by Republic Act 1792, provides: SEC. 21. Government Guarantee. The benefits prescribed in this Act shall not be diminished and to guarantee said benefits the Government of the Republic of the Philippines accepts general responsibility for the solvency of the System. From the foregoing provisions, it appears that the benefit receivable under the Act is in the nature of a special privilege or an arrangement secured by the law, pursuant to the policy of the State to provide social security to the workingmen. The amounts that may thus be received cannot be considered as property earned by the member during his lifetime. His contribution to the fund, it may be noted, constitutes only an insignificant portion thereof. Then, the benefits are specifically declared not transferable, 6 and exempted from tax legal processes, and lien.7 Furthermore, in the settlement of claims thereunder the procedure to be observed is governed not by the general provisions of law, but by rules and regulations promulgated by the Commission. Thus, if the money is payable to the estate of a deceased member, it is the Commission, not the probate or regular court that determines the person or persons to whom it is payable. 8 that the benefits under the Social Security Act are not intended by the lawmaking body to form part of the estate of the covered members may be gathered from the subsequent amendment made to Section 15 thereof, as follows: SEC. 15. Non-transferability of benefit. The system shall pay the benefits provided for in this Act to such persons as may be entitled thereto in accordance with the provisions of this Act. Such benefits are not transferable, and no power of attorney or other document executed by those entitled thereto in favor of any agent, attorney, or any other individual for the collection thereof in their behalf shall be recognized except when they are physically and legally unable to collect personally such benefits: Provided, however, That in the case of death benefits, if no beneficiary has been designated or the designation there of is void, said

benefits shall be paid to the legal heirs in accordance with the laws of succession. (Rep. Act 2658, amending Rep. Act 1161.) In short, if there is a named beneficiary and the designation is not invalid (as it is not so in this case), it is not the heirs of the employee who are entitled to receive the benefits (unless they are the designated beneficiaries themselves). It is only when there is no designated beneficiaries or when the designation is void, that the laws of succession are applicable. And we have already held that the Social Security Act is not a law of succession.9 Wherefore, in view of the foregoing considerations, the resolution of the Social Security Commission appealed from is hereby affirmed, with costs against the appellant. So ordered. Concepcion, C.J., Reyes, J.B.L., Dizon, Makalintal, Bengzon, J.P., Zaldivar and Sanchez, concur.

A.M. No. 190 October 18, 1977 RE: CLAIMS FOR BENEFITS OF THE HEIRS OF THE LATE MARIO V. CHANLIONGCO, FIDELA B. CHANLIONGCO, MARIO B. CHANLIONGCO II, MA. ANGELINA C. BUENAVENTURA and MARIO C. CHANLIONGCO, JR., claimants.+.wph!1 RESOLUTION

MAKASIAR, J.:t.hqw This matter refers to the claims for retirement benefits filed by the heirs of the late ATTY. MARIO V. CHANLIONGCO an attorney in this Court, under the provisions of R.A. No. 1616, as amended by R.A. No. 4986, which was approved by this Court in its resolution of August 19, 1976, effective on July 12, 1976 it a g from the records that at the time of his death on July 12, 1976, Atty. Chanliongco was more than 63 years of age, with more than 38 years of service in the government. He did not have any pending criminal administrative or not case against him, neither did he have any money or property accountability. The highest salary he received was P18,700.00 per annum. The above named flied the appellants for benefits with the accruing and with the Government Service System. Aside from his widow, Dra. Fidel B. Chanliongco and an only Intimate Mario it appears that there are other deceased to namely, Mrs. Angelina C. , Jr., both born out of wedlock to Angelina R Crespo, and duly recognized by the deceased. Except Mario, Jr., who is only 17 years of age, all the claimants are of legal age. According to law, the benefits accruing to the deceased consist of: (1) retirement benefits; (2) money value of terminal leave; (3) life insurance and (4) refund of retirement premium. From the records now before US, it appears that the GSIS had already the release the life insurance proceeds; and the refund of rent to the claimants. What, therefore, to be settled are the retirement benefits and the money value of leave, both of which are to be paid by this court as the deceased's last employer. The record also shows that the late Atty. Chanliongco died ab intestato and that he filed or over to state in his application for membership with the GSIS the beneficiary or benefits of his retirement benefits, should he die before retirement. Hence, the retirement benefits shall accrue to his estate and will be distributed among his Legal

heirs in with the benefits on intestate s , as in the caw of a fife if no benefit is named in the policy (Vda. de vs. GSIS, L-28093, Jan. 30, 1971, 37 SCRA 315, 325). Insofar therefore as the retirement benefits are WE adopt in toto, for being in accordance with law, the GSIS determination of the amount of the retirement the kill heirs and their e shares as indicated in its letter to US, dated March 15, 1977, to wit: +.wph!1 (a) Amount of retirement grautity:
1

. Total creditable service

37.57169 years Pl,558.33333/mo. 50.14338 months

2. Highest rate of salary 3. Gratuity in terms of months 4. Amount of gratuity (highest salary) x (No. of grautity months) (b) Legal heirs:
1

P78,140,10

. Fidela B. Chanliongco.

widow legitimate son illegitimate child

2. Mario B. Chanliongco II. 3. Ma. Angelina C. Buenaventura 4. Mario Chanliongco Jr. (c) Distribution (1) 8/16 share to Mario II (2) 4/16 share to the widow,

illegitimate child

P39,070.050 19,535.025

Fidela B. Chanliongco (3) 2/16 share, or P9,767.5125 each to the two illegitimate children Ma. Angelina C. Buenaventura and Mario Chanliongco, Jr. TOTAL 19 535 25

P78.140.100

Coming now to the money value of the terminal leave, unpaid salary and 10% adjustment pursuant to Budget Circular No. 240, dated July 22, 1974, this Court's Finance Officer, in a memorandum dated March 23, 1977, indicated the breakdown of these items as follows: Unpaid salary for July 8-12, 1976 @ P1,416.66/mo. 10% salary adj. for July 1-12, 1976 Money value of terminal leave for the period from July 13, 1976 to September 14,1977 @ P1,558.33 Sub-Total Less: Withholding Tax Supreme Court P1,400.00 21,962.54 P22,9245.87 P228.49 54.84

Savings & Loan Association NET PROCEEDS 7,340.42 8.740.42 P13,505.45

It further appears that at the time of his death the late Atty. Chanliongco had an outstanding account with the Supreme Court Savings & Loans Association in the sum of P7,340.42. Deduction this amount plus another sum of P1,400.00, representing withhold tax due from him, or a total of P8,740.42, from above sub-total sum of P22,245.87. WE have at the net sum P13,505.45, available for distribute to the claimants as follows:
1

. Fidela B. Chanliongco a. As her conjugal share b. As a legal heir P 6,752.72

P 1,688.18 P 3,376.36 844.10

2. Mario Chanliongco II 3. Ma. Angelina C. Buenaventura 4. Mario Jr. TOTAL

844.09 P13,505.45

It will be seen from the f distribution that the money value of the unused vacation and sick leave, unpaid will and 10% adjustment due to the has been treated as conjugal property. Accordingly, one-half (l/2) goes to the widow as her share in the conjugal hip and the other half P6,752.725 is to be distributed to the deceased's kill him, using the same one WE used in distributing the retirement benefits. This is so because "Vacation with pay is not a gratuity but is compensation for services rendered." (Ramey vs. State, 296 NW 323, 296 Mich. 449).

WHEREFORE, THE CLAIMS ARE HEREBY APPROVED. THE FINANCE AND/OR DISBURSING OFFICER OF THIS COURT IS ORDERED To pay IMMEDIATELY TO EACH AND EVERY CLAIMANT HE VARIOUS SUMS HEREUNDER INDICATED OPPOSITE THEIR NAMES, AS FOLLOWS:
1

. FIDELA B. CHANLIONGCO P19,535.025

A. HER 4/16 SHARE OF RETIREMENT GRATUITY B. HER SHARE FROM MONEY VALUE OF TEAL LEAVE, UNPAID SALARY AND 10% ADJUSTMENT: (1) AS HER CONJUGAL SHARE (2) AS A LEGAL HEIR TOTAL AMOUNT DUE HER 2. MARIO CHANLIONGCO II A. HIS 8/16 SHARE OF RETIREMENT GRATUITY B. HIS SHARE FROM MONEY VALUE OF TERMINAL LEAVE, UNPAID SALARY AND 10% ADJUSTMENT TOTAL AMOUNT DUE HIM 3. MA. ANGELINA C. BUENAVENTURA: A. HER 2/16 SHARE OF RETIREMENT GRATUITY

6,752.72 P1,688.18 P27,975.93

P39,070.05

3,376.36

P42,446.41

P9,767.51

B. HER SHARE FROM MONEY VALUE OF TERMINAL LEAVE, UNPAID SALARY AND 10% ADJUSTMENT TOTAL AMOUNT DUE HER 4. MARIO CHANLIONGCO JR. TO BE PAID THROUGH HIS MOTHER AND NATURAL GUARDIAN, ANGELINA CRESPO): A. HIS 2/16 SHARE OF RETIREMENT GRATUITY B. HIS SHARE FROM MONEY VALUE OF TERMINAL LEAVE, UNPAID SALARY AND 10% ADJUSTMENT TOTAL AMOUNT DUE HIM SO ORDERED.

844.10

P10,611.61

P9,767.51

844.10

P10,611.61

Castro, C.J., Barredo, Antonio, Mu;oz Palma, Concepcion, Jr., Martin, Santos, Fernandez and Guerrero, JJ., concur.1wph1.t Fernando, J., is on leave.

Separate Opinions

AQUINO, J., concurring: I concur. The provisions on legitime are found under the rubric of testamentary succession. That does not mean that the legitime is taken into account only in testamentary succession. The legitime must also be taken into consideration in legal succession.

There may be instances, like the instant case, where in legal succession the estate is distributed according to the rules on legitime without applying the rules on intestate ion. The reason is that sometimes the estate is not even sufficient to satisfy the legitimes. The legitimes of the primary compulsory heirs, like a child or descendant, should first be satisfied. In this case the decedent's legal heirs are his legitimate child, his widow and two intimate children. His estate is partitioned among those heirs by giving them their respective time. The legitimate child gets one-half of the estate as his legitime which is regarded as his share as a legal heir Art 888, Civil Code). The widow's legitime is one-fourth of the estate. That represents also her share as a legal heir (Art. 892, 1st sentence, Civil Code). The remaining one-fourth of the estate, which is the free portion, goes to the illegitimate children in equal shares, as their legitime, Pursuant to the provision that 'the legitimate of the illegitimate children shall be taken from the portion of the estate at the free disposal of the testator, provoked that in no case shall the total legitime of such illegitimate children exceed that free portion, and that the legitime of the surviving spouse must first be fully satisfied par., art. 895, Civil Code). The rule in Santillon vs. Miranda, L-19281, June 30, 1965, 14 SCRA 563, that when the surviving spouse concurs with only one legitimate child, the spouse is entitled to onehalf of the estate and the gets the other half, t to article 996 of the Civil Code, does not apply to the case because here intimate children concur with the surviving spouse and the intimate child. In this case, to divide the estate between the surviving spouse and the ligitemate child that deprive the illegitimate children of their legitime. So, the decendent's estate is distributed in the proportion of 1/2 for the legitimate child, 1/4 for the widow and 1/8 each for the two illegitimate children. Also not of possible application to this case is the rule that the legal of an acknowledge natural child is 1/2 of the legitime of the legitimate child of that the of the spurious child is 2/5 of that of the of the intimate child or 4/5 of that of that of the acknowledged natural child. The rule be applied because the estate is not sufficient to cover legitimes of all compulsory heirs. That is one of the flaws of the law of succession.

A situation as in the instant case may arise where the illegitimate children get less than their legitime. With respect to the decendant's unpaid salary and the money value of his leave, the same are conjugal properties because of the rule that property "obtained by the or work, or as salary of the spouses, or either of them", is conjugal in character (Art. 153[2], Civil Code).

Separate Opinions AQUINO, J., concurring: I concur. The provisions on legitime are found under the rubric of testamentary succession. That does not mean that the legitime is taken into account only in testamentary succession. The legitime must also be taken into consideration in legal succession. There may be instances, like the instant case, where in legal succession the estate is distributed according to the rules on legitime without applying the rules on intestate ion. The reason is that sometimes the estate is not even sufficient to satisfy the legitimes. The legitimes of the primary compulsory heirs, like a child or descendant, should first be satisfied. In this case the decedent's legal heirs are his legitimate child, his widow and two intimate children. His estate is partitioned among those heirs by giving them their respective time. The legitimate child gets one-half of the estate as his legitime which is regarded as his share as a legal heir Art 888, Civil Code). The widow's legitime is one-fourth of the estate. That represents also her share as a legal heir (Art. 892, 1st sentence, Civil Code). The remaining one-fourth of the estate, which is the free portion, goes to the illegitimate children in equal shares, as their legitime, Pursuant to the provision that 'the legitimate of the illegitimate children shall be taken from the portion of the estate at the free disposal of the testator, provoked that in no case shall the total legitime of such illegitimate children exceed that free portion, and that the legitime of the surviving spouse must first be fully satisfied par., art. 895, Civil Code). The rule in Santillon vs. Miranda, L-19281, June 30, 1965, 14 SCRA 563, that when the surviving spouse concurs with only one legitimate child, the spouse is entitled to onehalf of the estate and the gets the other half, t to article 996 of the Civil Code, does not

apply to the case because here intimate children concur with the surviving spouse and the intimate child. In this case, to divide the estate between the surviving spouse and the ligitemate child that deprive the illegitimate children of their legitime. So, the decendent's estate is distributed in the proportion of 1/2 for the legitimate child, 1/4 for the widow and 1/8 each for the two illegitimate children. Also not of possible application to this case is the rule that the legal of an acknowledge natural child is 1/2 of the legitime of the legitimate child of that the of the spurious child is 2/5 of that of the of the intimate child or 4/5 of that of that of the acknowledged natural child. The rule be applied because the estate is not sufficient to cover legitimes of all compulsory heirs. That is one of the flaws of the law of succession. A situation as in the instant case may arise where the illegitimate children get less than their legitime. With respect to the decendant's unpaid salary and the money value of his leave, the same are conjugal properties because of the rule that property "obtained by the or work, or as salary of the spouses, or either of them", is conjugal in character (Art. 153[2], Civil Code).

G.R. No. L-28093 January 30, 1971 BASILIA BERDIN VDA. DE CONSUEGRA; JULIANA, PACITA, MARIA LOURDES, JOSE, JR., RODRIGO, LINEDA and LUIS, all surnamed CONSUEGRA, petitionersappellants, vs. GOVERNMENT SERVICE INSURANCE SYSTEM, COMMISSIONER OF PUBLIC HIGHWAYS, HIGHWAY DISTRICT ENGINEER OF SURIGAO DEL NORTE, COMMISSIONER OF CIVIL SERVICE, and ROSARIO DIAZ, respondents-appellees. Bernardino O. Almeda for petitioners-appellants. Binag and Arevalo, Jr. for respondent-appellee Government Service Insurance System. Office of the Solicitor General for other respondents-appellees.

ZALDIVAR, J.: Appeal on purely questions of law from the decision of the Court of First Instance of Surigao del Norte, dated March 7, 1967, in its Special Proceeding No. 1720. The pertinent facts, culled from the stipulation of facts submitted by the parties, are the following: The late Jose Consuegra, at the time of his death, was employed as a shop foreman of the office of the District Engineer in the province of Surigao del Norte. In his lifetime, Consuegra contracted two marriages, the first with herein respondent Rosario Diaz, solemnized in the parish church of San Nicolas de Tolentino, Surigao, Surigao, on July 15, 1937, out of which marriage were born two children, namely, Jose Consuegra, Jr. and Pedro Consuegra, but both predeceased their father; and the second, which was contracted in good faith while the first marriage was subsisting, with herein petitioner Basilia Berdin, on May 1, 1957 in the same parish and municipality, out of which marriage were born seven children, namely, Juliana, Pacita, Maria Lourdes, Jose, Rodrigo, Lenida and Luz, all surnamed Consuegra. Being a member of the Government Service Insurance System (GSIS, for short) when Consuegra died on September 26, 1965, the proceeds of his life insurance under policy No. 601801 were paid by the GSIS to petitioner Basilia Berdin and her children who were the beneficiaries named in the policy. Having been in the service of the government for 22.5028 years, Consuegra was entitled to retirement insurance benefits in the sum of P6,304.47 pursuant to Section 12(c) of Commonwealth Act 186 as amended by Republic Acts 1616 and 3836. Consuegra did not designate any

beneficiary who would receive the retirement insurance benefits due to him. Respondent Rosario Diaz, the widow by the first marriage, filed a claim with the GSIS asking that the retirement insurance benefits be paid to her as the only legal heir of Consuegra, considering that the deceased did not designate any beneficiary with respect to his retirement insurance benefits. Petitioner Basilia Berdin and her children, likewise, filed a similar claim with the GSIS, asserting that being the beneficiaries named in the life insurance policy of Consuegra, they are the only ones entitled to receive the retirement insurance benefits due the deceased Consuegra. Resolving the conflicting claims, the GSIS ruled that the legal heirs of the late Jose Consuegra were Rosario Diaz, his widow by his first marriage who is entitled to one-half, or 8/16, of the retirement insurance benefits, on the one hand; and Basilia Berdin, his widow by the second marriage and their seven children, on the other hand, who are entitled to the remaining one-half, or 8/16, each of them to receive an equal share of 1/16. Dissatisfied with the foregoing ruling and apportionment made by the GSIS, Basilia Berdin and her children1 filed on October 10, 1966 a petition for mandamus with preliminary injunction in the Court of First Instance of Surigao, naming as respondents the GSIS, the Commissioner of Public Highways, the Highway District Engineer of Surigao del Norte, the Commissioner of Civil Service, and Rosario Diaz, praying that they (petitioners therein) be declared the legal heirs and exclusive beneficiaries of the retirement insurance of the late Jose Consuegra, and that a writ of preliminary injunction be issued restraining the implementation of the adjudication made by the GSIS. On October 26, 1966, the trial court issued an order requiring therein respondents to file their respective answers, but refrained from issuing the writ of preliminary injunction prayed for. On February 11, 1967, the parties submitted a stipulation of facts, prayed that the same be admitted and approved and that judgment be rendered on the basis of the stipulation of facts. On March 7, 1967, the court below rendered judgment, the pertinent portions of which are quoted hereunder: This Court, in conformity with the foregoing stipulation of facts, likewise is in full accord with the parties with respect to the authority cited by them in support of said stipulation and which is herein-below cited for purposes of this judgment, to wit: "When two women innocently and in good faith are legally united in holy matrimony to the same man, they and their children, born of said wedlock, will be regarded as legitimate children and each family be entitled to one half of the estate. Lao & Lao vs. Dee Tim, 45 Phil. 739; Estrella vs. Laong Masa, Inc., (CA) 39 OG 79; Pisalbon vs. Bejec, 74 Phil. 88. WHEREFORE, in view of the above premises, this Court is of the opinion that the foregoing stipulation of facts is in order and in accordance with law and the same is hereby approved. Judgment, therefore, is hereby rendered declaring the petitioner

Basilia Berdin Vda. de Consuegra and her co-petitioners Juliana, Pacita, Maria Lourdes, Jose, Jr., Rodrigo, Lenida and Luis, all surnamed Consuegra, beneficiary and entitled to one-half (1/2) of the retirement benefit in the amount of Six Thousand Three Hundred Four Pesos and Fourty-Seven Centavos (P6,304.47) due to the deceased Jose Consuegra from the Government Service Insurance System or the amount of P3,152.235 to be divided equally among them in the proportional amount of 1/16 each. Likewise, the respondent Rosario Diaz Vda. de Consuegra is hereby declared beneficiary and entitled to the other half of the retirement benefit of the late Jose Consuegra or the amount of P3,152.235. The case with respect to the Highway District Engineer of Surigao del Norte is hereby ordered dismissed. Hence the present appeal by herein petitioners-appellants, Basilia Berdin and her children. It is the contention of appellants that the lower court erred in not holding that the designated beneficiaries in the life insurance of the late Jose Consuegra are also the exclusive beneficiaries in the retirement insurance of said deceased. In other words, it is the submission of appellants that because the deceased Jose Consuegra failed to designate the beneficiaries in his retirement insurance, the appellants who were the beneficiaries named in the life insurance should automatically be considered the beneficiaries to receive the retirement insurance benefits, to the exclusion of respondent Rosario Diaz. From the arguments adduced by appellants in their brief We gather that it is their stand that the system of life insurance and the system of retirement insurance, that are provided for in Commonwealth Act 186 as amended, are simply complementary to each other, or that one is a part or an extension of the other, such that whoever is named the beneficiary in the life insurance is also the beneficiary in the retirement insurance when no such beneficiary is named in the retirement insurance. The contention of appellants is untenable. It should be noted that the law creating the Government Service Insurance System is Commonwealth Act 186 which was enacted by the National Assembly on November 14, 1936. As originally approved, Commonwealth Act 186 provided for the compulsory membership in the Government Service Insurance System of all regularly and permanently appointed officials and employees of the government, considering as automatically insured on life all such officials and employees, and issuing to them the corresponding membership policy under the terms and conditions as provided in the Act.2 Originally, Commonwealth Act 186 provided for life insurance only. Commonwealth Act 186 was amended by Republic Act 660 which was enacted by the Congress of the Philippines on June 16, 1951, and, among others, the amendatory Act provided that

aside from the system of life insurance under the Government Service Insurance System there was also established the system of retirement insurance. Thus, We will note in Republic Act 660 that there is a chapter on life insurance and another chapter on retirement insurance. 3 Under the chapter on life insurance are sections 8, 9 and 10 of Commonwealth Act 186, as amended; and under the chapter on retirement insurance are sections 11, 12, 13 and 13-A. On May 31, 1957, Republic Act 1616 was enacted by Congress, amending section 12 of Commonwealth Act 186 as amended by Republic Act 660, by adding thereto two new subsections, designated as subsections (b) and (c). This subsection (c) of section 12 of Commonwealth Act 186, as amended by Republic Acts 660, 1616 and 3096, was again amended by Republic Act 3836 which was enacted on June 22, 1963.lwph1.t The pertinent provisions of subsection (c) of Section 12 of Commonwealth Act 186, as thus amended and reamended, read as follows: (c) Retirement is likewise allowed to a member, regardless of age, who has rendered at least twenty years of service. The benefit shall, in addition to the return of his personal contributions plus interest and the payment of the corresponding employer's premiums described in subsection (a) of Section 5 hereof, without interest, be only a gratuity equivalent to one month's salary for every year of service, based on the highest rate received, but not to exceed twenty-four months; Provided, That the retiring officer or employee has been in the service of the said employer or office for at least four years, immediately preceding his retirement. xxx xxx xxx The gratuity is payable by the employer or office concerned which is hereby authorized to provide the necessary appropriation to pay the same from any unexpended items of appropriations. Elective or appointive officials and employees paid gratuity under this subsection shall be entitled to the commutation of the unused vacation and sick leave, based on the highest rate received, which they may have to their credit at the time of retirement. Jose Consuegra died on September 26, 1965, and so at the time of his death he had acquired rights under the above-quoted provisions of subsection (c) of Section 12 of Com. Act 186, as finally amended by Rep. Act 3836 on June 22, 1963. When Consuegra died on September 26, 1965, he had to his credit 22.5028 years of service in the government, and pursuant to the above-quoted provisions of subsection (c) of Section 12 of Com. Act 186, as amended, on the basis of the highest rate of salary received by him which was P282.83 per month, he was entitled to receive retirement insurance benefits in the amount of P6,304.47. This is the retirement benefits that are the subject of dispute between the appellants, on the one hand, and the appellee

Rosario Diaz, on the other, in the present case. The question posed is: to whom should this retirement insurance benefits of Jose Consuegra be paid, because he did not, or failed to, designate the beneficiary of his retirement insurance? If Consuegra had 22.5028 years of service in the government when he died on September 26, 1965, it follows that he started in the government service sometime during the early part of 1943, or before 1943. In 1943 Com. Act 186 was not yet amended, and the only benefits then provided for in said Com. Act 186 were those that proceed from a life insurance. Upon entering the government service Consuegra became a compulsory member of the GSIS, being automatically insured on his life, pursuant to the provisions of Com. Act 186 which was in force at the time. During 1943 the operation of the Government Service Insurance System was suspended because of the war, and the operation was resumed sometime in 1946. When Consuegra designated his beneficiaries in his life insurance he could not have intended those beneficiaries of his life insurance as also the beneficiaries of his retirement insurance because the provisions on retirement insurance under the GSIS came about only when Com. Act 186 was amended by Rep. Act 660 on June 16, 1951. Hence, it cannot be said that because herein appellants were designated beneficiaries in Consuegra's life insurance they automatically became the beneficiaries also of his retirement insurance. Rep. Act 660 added to Com. Act 186 provisions regarding retirement insurance, which are Sections 11, 12, and 13 of Com. Act 186, as amended. Subsection (b) of Section 11 of Com. Act 186, as amended by Rep. Act 660, provides as follows: (b) Survivors benefit. Upon death before he becomes eligible for retirement, his beneficiaries as recorded in the application for retirement annuity filed with the System shall be paid his own premiums with interest of three per centum per annum, compounded monthly. If on his death he is eligible for retirement, then the automatic retirement annuity or the annuity chosen by him previously shall be paid accordingly. The above-quoted provisions of subsection (b) of Section 11 of Commonwealth Act 186, as amended by Rep. Act 660, clearly indicate that there is need for the employee to file an application for retirement insurance benefits when he becomes a member of the GSIS, and he should state in his application the beneficiary of his retirement insurance. Hence, the beneficiary named in the life insurance does not automatically become the beneficiary in the retirement insurance unless the same beneficiary in the life insurance is so designated in the application for retirement insurance. Section 24 of Commonwealth Act 186, as amended by Rep. Act 660, provides for a life insurance fund and for a retirement insurance fund. There was no such provision in Com. Act 186 before it was amended by Rep. Act 660. Thus, subsections (a) and (b) of Section 24 of Commonwealth Act 186, as amended by Rep. Act 660, partly read as follows:

(a) Life insurance fund. This shall consist of all premiums for life insurance benefit and/or earnings and savings therefrom. It shall meet death claims as they may arise or such equities as any member may be entitled to, under the conditions of his policy, and shall maintain the required reserves to the end of guaranteeing the fulfillment of the life insurance contracts issued by the System ... (b) Retirement insurance fund. This shall consist of all contributions for retirement insurance benefit and of earnings and savings therefrom. It shall meet annuity payments and establish the required reserves to the end of guaranteeing the fulfillment of the contracts issued by the System. ... Thus, We see that the GSIS offers two separate and distinct systems of benefits to its members one is the life insurance and the other is the retirement insurance. These two distinct systems of benefits are paid out from two distinct and separate funds that are maintained by the GSIS. In the case of the proceeds of a life insurance, the same are paid to whoever is named the beneficiary in the life insurance policy. As in the case of a life insurance provided for in the Insurance Act (Act 2427, as amended), the beneficiary in a life insurance under the GSIS may not necessarily be a heir of the insured. The insured in a life insurance may designate any person as beneficiary unless disqualified to be so under the provisions of the Civil Code.4 And in the absence of any beneficiary named in the life insurance policy, the proceeds of the insurance will go to the estate of the insured. Retirement insurance is primarily intended for the benefit of the employee to provide for his old age, or incapacity, after rendering service in the government for a required number of years. If the employee reaches the age of retirement, he gets the retirement benefits even to the exclusion of the beneficiary or beneficiaries named in his application for retirement insurance. The beneficiary of the retirement insurance can only claim the proceeds of the retirement insurance if the employee dies before retirement. If the employee failed or overlooked to state the beneficiary of his retirement insurance, the retirement benefits will accrue to his estate and will be given to his legal heirs in accordance with law, as in the case of a life insurance if no beneficiary is named in the insurance policy. It is Our view, therefore, that the respondent GSIS had correctly acted when it ruled that the proceeds of the retirement insurance of the late Jose Consuegra should be divided equally between his first living wife Rosario Diaz, on the one hand, and his second wife Basilia Berdin and his children by her, on the other; and the lower court did not commit error when it confirmed the action of the GSIS, it being accepted as a fact that the second marriage of Jose Consuegra to Basilia Berdin was contracted in good faith. The lower court has correctly applied the ruling of this Court in the case of Lao, et al. vs. Dee

Tim, et al., 45 Phil. 739 as cited in the stipulation of facts and in the decision appealed from.5 In the recent case of Gomez vs. Lipana, L-23214, June 30, 1970, 6 this Court, in construing the rights of two women who were married to the same man a situation more or less similar to the case of appellant Basilia Berdin and appellee Rosario Diaz held "that since the defendant's first marriage has not been dissolved or declared void the conjugal partnership established by that marriage has not ceased. Nor has the first wife lost or relinquished her status as putative heir of her husband under the new Civil Code, entitled to share in his estate upon his death should she survive him. Consequently, whether as conjugal partner in a still subsisting marriage or as such putative heir she has an interest in the husband's share in the property here in dispute.... " And with respect to the right of the second wife, this Court observed that although the second marriage can be presumed to be void ab initio as it was celebrated while the first marriage was still subsisting, still there is need for judicial declaration of such nullity. And inasmuch as the conjugal partnership formed by the second marriage was dissolved before judicial declaration of its nullity, "[t]he only lust and equitable solution in this case would be to recognize the right of the second wife to her share of one-half in the property acquired by her and her husband and consider the other half as pertaining to the conjugal partnership of the first marriage." WHEREFORE, the decision appealed from is affirmed, with costs against petitionersappellants. It is so ordered. Concepcion, C.J., Reyes, J.B.L., Dizon, Makalintal, Castro, Fernando, Teehankee, Barredo, Villamor and Makasiar, JJ., concur.

G.R. No. 147839

June 8, 2006

GAISANO CAGAYAN, INC. Petitioner, vs. INSURANCE COMPANY OF NORTH AMERICA, Respondent. DECISION AUSTRIA-MARTINEZ, J.: Before the Court is a petition for review on certiorari of the Decision1 dated October 11, 2000 of the Court of Appeals (CA) in CA-G.R. CV No. 61848 which set aside the Decision dated August 31, 1998 of the Regional Trial Court, Branch 138, Makati (RTC) in Civil Case No. 92-322 and upheld the causes of action for damages of Insurance Company of North America (respondent) against Gaisano Cagayan, Inc. (petitioner); and the CA Resolution dated April 11, 2001 which denied petitioner's motion for reconsideration. The factual background of the case is as follows: Intercapitol Marketing Corporation (IMC) is the maker of Wrangler Blue Jeans. Levi Strauss (Phils.) Inc. (LSPI) is the local distributor of products bearing trademarks owned by Levi Strauss & Co.. IMC and LSPI separately obtained from respondent fire insurance policies with book debt endorsements. The insurance policies provide for coverage on "book debts in connection with ready-made clothing materials which have been sold or delivered to various customers and dealers of the Insured anywhere in the Philippines."2 The policies defined book debts as the "unpaid account still appearing in the Book of Account of the Insured 45 days after the time of the loss covered under this Policy."3 The policies also provide for the following conditions: 1. Warranted that the Company shall not be liable for any unpaid account in respect of the merchandise sold and delivered by the Insured which are outstanding at the date of loss for a period in excess of six (6) months from the date of the covering invoice or actual delivery of the merchandise whichever shall first occur. 2. Warranted that the Insured shall submit to the Company within twelve (12) days after the close of every calendar month all amount shown in their books of accounts as unpaid and thus become receivable item from their customers and dealers. x x x4 xxxx Petitioner is a customer and dealer of the products of IMC and LSPI. On February 25, 1991, the Gaisano Superstore Complex in Cagayan de Oro City, owned by petitioner,

was consumed by fire. Included in the items lost or destroyed in the fire were stocks of ready-made clothing materials sold and delivered by IMC and LSPI. On February 4, 1992, respondent filed a complaint for damages against petitioner. It alleges that IMC and LSPI filed with respondent their claims under their respective fire insurance policies with book debt endorsements; that as of February 25, 1991, the unpaid accounts of petitioner on the sale and delivery of ready-made clothing materials with IMC was P2,119,205.00 while with LSPI it was P535,613.00; that respondent paid the claims of IMC and LSPI and, by virtue thereof, respondent was subrogated to their rights against petitioner; that respondent made several demands for payment upon petitioner but these went unheeded.5 In its Answer with Counter Claim dated July 4, 1995, petitioner contends that it could not be held liable because the property covered by the insurance policies were destroyed due to fortuities event or force majeure; that respondent's right of subrogation has no basis inasmuch as there was no breach of contract committed by it since the loss was due to fire which it could not prevent or foresee; that IMC and LSPI never communicated to it that they insured their properties; that it never consented to paying the claim of the insured.6 At the pre-trial conference the parties failed to arrive at an amicable settlement. 7 Thus, trial on the merits ensued. On August 31, 1998, the RTC rendered its decision dismissing respondent's complaint. 8 It held that the fire was purely accidental; that the cause of the fire was not attributable to the negligence of the petitioner; that it has not been established that petitioner is the debtor of IMC and LSPI; that since the sales invoices state that "it is further agreed that merely for purpose of securing the payment of purchase price, the above-described merchandise remains the property of the vendor until the purchase price is fully paid", IMC and LSPI retained ownership of the delivered goods and must bear the loss. Dissatisfied, petitioner appealed to the CA.9 On October 11, 2000, the CA rendered its decision setting aside the decision of the RTC. The dispositive portion of the decision reads: WHEREFORE, in view of the foregoing, the appealed decision is REVERSED and SET ASIDE and a new one is entered ordering defendant-appellee Gaisano Cagayan, Inc. to pay: 1. the amount of P2,119,205.60 representing the amount paid by the plaintiff-appellant to the insured Inter Capitol Marketing Corporation, plus legal interest from the time of demand until fully paid;

2. the amount of P535,613.00 representing the amount paid by the plaintiff-appellant to the insured Levi Strauss Phil., Inc., plus legal interest from the time of demand until fully paid. With costs against the defendant-appellee. SO ORDERED.10 The CA held that the sales invoices are proofs of sale, being detailed statements of the nature, quantity and cost of the thing sold; that loss of the goods in the fire must be borne by petitioner since the proviso contained in the sales invoices is an exception under Article 1504 (1) of the Civil Code, to the general rule that if the thing is lost by a fortuitous event, the risk is borne by the owner of the thing at the time the loss under the principle of res perit domino; that petitioner's obligation to IMC and LSPI is not the delivery of the lost goods but the payment of its unpaid account and as such the obligation to pay is not extinguished, even if the fire is considered a fortuitous event; that by subrogation, the insurer has the right to go against petitioner; that, being a fire insurance with book debt endorsements, what was insured was the vendor's interest as a creditor.11 Petitioner filed a motion for reconsideration12 but it was denied by the CA in its Resolution dated April 11, 2001.13 Hence, the present petition for review on certiorari anchored on the following Assignment of Errors: THE COURT OF APPEALS ERRED IN HOLDING THAT THE INSURANCE IN THE INSTANT CASE WAS ONE OVER CREDIT. THE COURT OF APPEALS ERRED IN HOLDING THAT ALL RISK OVER THE SUBJECT GOODS IN THE INSTANT CASE HAD TRANSFERRED TO PETITIONER UPON DELIVERY THEREOF. THE COURT OF APPEALS ERRED IN HOLDING THAT THERE WAS AUTOMATIC SUBROGATION UNDER ART. 2207 OF THE CIVIL CODE IN FAVOR OF RESPONDENT.14 Anent the first error, petitioner contends that the insurance in the present case cannot be deemed to be over credit since an insurance "on credit" belies not only the nature of fire insurance but the express terms of the policies; that it was not credit that was insured since respondent paid on the occasion of the loss of the insured goods to fire and not because of the non-payment by petitioner of any obligation; that, even if the insurance is deemed as one over credit, there was no loss as the accounts were not yet due since no prior demands were made by IMC and LSPI against petitioner for payment

of the debt and such demands came from respondent only after it had already paid IMC and LSPI under the fire insurance policies.15 As to the second error, petitioner avers that despite delivery of the goods, petitionerbuyer IMC and LSPI assumed the risk of loss when they secured fire insurance policies over the goods. Concerning the third ground, petitioner submits that there is no subrogation in favor of respondent as no valid insurance could be maintained thereon by IMC and LSPI since all risk had transferred to petitioner upon delivery of the goods; that petitioner was not privy to the insurance contract or the payment between respondent and its insured nor was its consent or approval ever secured; that this lack of privity forecloses any real interest on the part of respondent in the obligation to pay, limiting its interest to keeping the insured goods safe from fire. For its part, respondent counters that while ownership over the ready- made clothing materials was transferred upon delivery to petitioner, IMC and LSPI have insurable interest over said goods as creditors who stand to suffer direct pecuniary loss from its destruction by fire; that petitioner is liable for loss of the ready-made clothing materials since it failed to overcome the presumption of liability under Article 1265 16 of the Civil Code; that the fire was caused through petitioner's negligence in failing to provide stringent measures of caution, care and maintenance on its property because electric wires do not usually short circuit unless there are defects in their installation or when there is lack of proper maintenance and supervision of the property; that petitioner is guilty of gross and evident bad faith in refusing to pay respondent's valid claim and should be liable to respondent for contracted lawyer's fees, litigation expenses and cost of suit.17 As a general rule, in petitions for review, the jurisdiction of this Court in cases brought before it from the CA is limited to reviewing questions of law which involves no examination of the probative value of the evidence presented by the litigants or any of them.18 The Supreme Court is not a trier of facts; it is not its function to analyze or weigh evidence all over again.19 Accordingly, findings of fact of the appellate court are generally conclusive on the Supreme Court.20 Nevertheless, jurisprudence has recognized several exceptions in which factual issues may be resolved by this Court, such as: (1) when the findings are grounded entirely on speculation, surmises or conjectures; (2) when the inference made is manifestly mistaken, absurd or impossible; (3) when there is grave abuse of discretion; (4) when the judgment is based on a misapprehension of facts; (5) when the findings of facts are conflicting; (6) when in making its findings the CA went beyond the issues of the case, or its findings are contrary to the admissions of both the appellant and the appellee; (7)

when the findings are contrary to the trial court; (8) when the findings are conclusions without citation of specific evidence on which they are based; (9) when the facts set forth in the petition as well as in the petitioner's main and reply briefs are not disputed by the respondent; (10) when the findings of fact are premised on the supposed absence of evidence and contradicted by the evidence on record; and (11) when the CA manifestly overlooked certain relevant facts not disputed by the parties, which, if properly considered, would justify a different conclusion.21 Exceptions (4), (5), (7), and (11) apply to the present petition. At issue is the proper interpretation of the questioned insurance policy. Petitioner claims that the CA erred in construing a fire insurance policy on book debts as one covering the unpaid accounts of IMC and LSPI since such insurance applies to loss of the readymade clothing materials sold and delivered to petitioner. The Court disagrees with petitioner's stand. It is well-settled that when the words of a contract are plain and readily understood, there is no room for construction.22 In this case, the questioned insurance policies provide coverage for "book debts in connection with ready-made clothing materials which have been sold or delivered to various customers and dealers of the Insured anywhere in the Philippines."23 ; and defined book debts as the "unpaid account still appearing in the Book of Account of the Insured 45 days after the time of the loss covered under this Policy."24 Nowhere is it provided in the questioned insurance policies that the subject of the insurance is the goods sold and delivered to the customers and dealers of the insured. Indeed, when the terms of the agreement are clear and explicit that they do not justify an attempt to read into it any alleged intention of the parties, the terms are to be understood literally just as they appear on the face of the contract.25 Thus, what were insured against were the accounts of IMC and LSPI with petitioner which remained unpaid 45 days after the loss through fire, and not the loss or destruction of the goods delivered. Petitioner argues that IMC bears the risk of loss because it expressly reserved ownership of the goods by stipulating in the sales invoices that "[i]t is further agreed that merely for purpose of securing the payment of the purchase price the above described merchandise remains the property of the vendor until the purchase price thereof is fully paid."26 The Court is not persuaded. The present case clearly falls under paragraph (1), Article 1504 of the Civil Code:

ART. 1504. Unless otherwise agreed, the goods remain at the seller's risk until the ownership therein is transferred to the buyer, but when the ownership therein is transferred to the buyer the goods are at the buyer's risk whether actual delivery has been made or not, except that: (1) Where delivery of the goods has been made to the buyer or to a bailee for the buyer, in pursuance of the contract and the ownership in the goods has been retained by the seller merely to secure performance by the buyer of his obligations under the contract, the goods are at the buyer's risk from the time of such delivery; (Emphasis supplied) xxxx Thus, when the seller retains ownership only to insure that the buyer will pay its debt, the risk of loss is borne by the buyer.27 Accordingly, petitioner bears the risk of loss of the goods delivered. IMC and LSPI did not lose complete interest over the goods. They have an insurable interest until full payment of the value of the delivered goods. Unlike the civil law concept of res perit domino, where ownership is the basis for consideration of who bears the risk of loss, in property insurance, one's interest is not determined by concept of title, but whether insured has substantial economic interest in the property. 28 Section 13 of our Insurance Code defines insurable interest as "every interest in property, whether real or personal, or any relation thereto, or liability in respect thereof, of such nature that a contemplated peril might directly damnify the insured." Parenthetically, under Section 14 of the same Code, an insurable interest in property may consist in: (a) an existing interest; (b) an inchoate interest founded on existing interest; or (c) an expectancy, coupled with an existing interest in that out of which the expectancy arises. Therefore, an insurable interest in property does not necessarily imply a property interest in, or a lien upon, or possession of, the subject matter of the insurance, and neither the title nor a beneficial interest is requisite to the existence of such an interest, it is sufficient that the insured is so situated with reference to the property that he would be liable to loss should it be injured or destroyed by the peril against which it is insured.29 Anyone has an insurable interest in property who derives a benefit from its existence or would suffer loss from its destruction.30 Indeed, a vendor or seller retains an insurable interest in the property sold so long as he has any interest therein, in other words, so long as he would suffer by its destruction, as where he has a vendor's lien. 31 In this case, the insurable interest of IMC and LSPI pertain to the unpaid accounts appearing in their Books of Account 45 days after the time of the loss covered by the policies.

The next question is: Is petitioner liable for the unpaid accounts? Petitioner's argument that it is not liable because the fire is a fortuitous event under Article 117432 of the Civil Code is misplaced. As held earlier, petitioner bears the loss under Article 1504 (1) of the Civil Code. Moreover, it must be stressed that the insurance in this case is not for loss of goods by fire but for petitioner's accounts with IMC and LSPI that remained unpaid 45 days after the fire. Accordingly, petitioner's obligation is for the payment of money. As correctly stated by the CA, where the obligation consists in the payment of money, the failure of the debtor to make the payment even by reason of a fortuitous event shall not relieve him of his liability.33 The rationale for this is that the rule that an obligor should be held exempt from liability when the loss occurs thru a fortuitous event only holds true when the obligation consists in the delivery of a determinate thing and there is no stipulation holding him liable even in case of fortuitous event. It does not apply when the obligation is pecuniary in nature.34 Under Article 1263 of the Civil Code, "[i]n an obligation to deliver a generic thing, the loss or destruction of anything of the same kind does not extinguish the obligation." If the obligation is generic in the sense that the object thereof is designated merely by its class or genus without any particular designation or physical segregation from all others of the same class, the loss or destruction of anything of the same kind even without the debtor's fault and before he has incurred in delay will not have the effect of extinguishing the obligation.35 This rule is based on the principle that the genus of a thing can never perish. Genus nunquan perit.36 An obligation to pay money is generic; therefore, it is not excused by fortuitous loss of any specific property of the debtor. 37 Thus, whether fire is a fortuitous event or petitioner was negligent are matters immaterial to this case. What is relevant here is whether it has been established that petitioner has outstanding accounts with IMC and LSPI. With respect to IMC, the respondent has adequately established its claim. Exhibits "C" to "C-22"38 show that petitioner has an outstanding account with IMC in the amount of P2,119,205.00. Exhibit "E"39 is the check voucher evidencing payment to IMC. Exhibit "F"40 is the subrogation receipt executed by IMC in favor of respondent upon receipt of the insurance proceeds. All these documents have been properly identified, presented and marked as exhibits in court. The subrogation receipt, by itself, is sufficient to establish not only the relationship of respondent as insurer and IMC as the insured, but also the amount paid to settle the insurance claim. The right of subrogation accrues simply upon payment by the insurance company of the insurance claim. 41 Respondent's action against petitioner is squarely sanctioned by Article 2207 of the Civil Code which provides:

Art. 2207. If the plaintiff's property has been insured, and he has received indemnity from the insurance company for the injury or loss arising out of the wrong or breach of contract complained of, the insurance company shall be subrogated to the rights of the insured against the wrongdoer or the person who has violated the contract. x x x Petitioner failed to refute respondent's evidence. As to LSPI, respondent failed to present sufficient evidence to prove its cause of action. No evidentiary weight can be given to Exhibit "F Levi Strauss",42 a letter dated April 23, 1991 from petitioner's General Manager, Stephen S. Gaisano, Jr., since it is not an admission of petitioner's unpaid account with LSPI. It only confirms the loss of Levi's products in the amount of P535,613.00 in the fire that razed petitioner's building on February 25, 1991. Moreover, there is no proof of full settlement of the insurance claim of LSPI; no subrogation receipt was offered in evidence. Thus, there is no evidence that respondent has been subrogated to any right which LSPI may have against petitioner. Failure to substantiate the claim of subrogation is fatal to petitioner's case for recovery of the amount of P535,613.00. WHEREFORE, the petition is partly GRANTED. The assailed Decision dated October 11, 2000 and Resolution dated April 11, 2001 of the Court of Appeals in CA-G.R. CV No. 61848 are AFFIRMED with the MODIFICATION that the order to pay the amount of P535,613.00 to respondent is DELETED for lack of factual basis. No pronouncement as to costs. SO ORDERED. MA. ALICIA AUSTRIA-MARTINEZ Associate Justice WE CONCUR: ARTEMIO V. PANGANIBAN Chief Justice Chairperson (On Leave) CONSUELO YNARES-SANTIAGO Associate Justice MINITA V. CHICO-NAZARIO Associate Justice

ROMEO J. CALLEJO, SR. Asscociate Justice

CERTIFICATION Pursuant to Section 13, Article VIII of the Constitution, it is hereby certified that the conclusions in the above Decision were reached in consultation before the case was assigned to the writer of the opinion of the Court's Division. ARTEMIO V. PANGANIBAN Chief Justice

TRADERS INSURANCE & SURETY CO., RECURRENTE, CONTRA JUAN GOLANGCO Y OTRA, RECURRIDOS. DECISION PABLO, M.: Se trata del cobro de una poliza de seguro contra incendio por valor de P10,000. El demandante tenia derecho a recibir Pl,100 mensuales de Melitona Estrella en concepto de alquileres del edificio No. 34 Plaza Sta. Cruz, Manila, por espacio de cinco arios. El demandante aseguro este derecho, pago la prima y la demandada expidio la poliza correspondiente. Dos meses despues de haberse incendiado el edificio, el demandante requirio a la Traders Insurance & Surety Co. el pago del importe del seguro. Como la demandada rehusaba pagarlo, el demandante acudio al Juzgado de Primera Instancia de Manila, el cual dicto sentencia a favor de este. El Tribunal de Apelacion confirmo la sentencia. En recurso de certiorari, la Traders Insurance & Surety Co. acude a este Tribunal, alegando que el Tribunal de Apelacion cometio dos errores: 1 (a) al dictar sentencia sin una completa conclusion de hechos de todas las cuestiones suscitadas, como requiere el articulo 33 de la Ley No. 296, y 1 (b) al no hacer conclusiones de hecho en cuanto a la aplicacion de la regla de prueba oral; y (2) al dictar sentencia sin conclusiones de hecho en cuanto a los Exhibits 10-H y 10-I. Cuanto al error 1 (a). Cuales son las cuestiones que habian sido propiamente suscitadas ante el Tribunal de Apelacion? La mejor contestacion es la primera pagina del alegato de la apelante (hoy recurrente) que dice asi: BRIEF FOR THE APPELLANT "ASSIGNMENT OF ERRORS I "THE LOWER COURT ERRED IN HOLDING THAT THE FIRE INSURANCE POLICY EXHIBIT 'A' COVERS ALL APPELLEE'S INTERESTS IN THE PREMISES NO. 34 PLAZA STA. CRUZ, MANILA, ESPECIALLY HIS RIGHT TO COLLECT RENTALS THEREFROM. II "THE LOWER COURT ERRED IN HOLDING THAT THE APPELLEE HAD INSURABLE INTEREST CONSISTING OP A RIGHT TO RECEIVE RENTALS BOTH AT THE TIME WHEN THE INSURANCE TOOK EFFECT AND WHEN THE LOSS OCCURRED."

Al resolver estas cuestiones el Tribunal de Apelacion, despues de transcribir toda la decision del Juzgado de Primera Instancia, dijo lo siguiente en su decision: "As stated by the lower court, the basic facts on which both parties base their respective contentions are not disputed, and we have quoted the decision appealed from in full because we find that the facts established in the case cannot lead to other conclusions than those arrived at by the trial judge. There is no doubt in our mind that both at the time of the execution of the fire policy (Exhibit A) on April 7, 1949, and on June 5, 1949, when the destruction by fire of the property for which the said policy was issued took place, plaintiff Juan Golangco had an insurable interest on the property insured which included the rents of premises No. 34 Plaza Sta. Cruz, Manila, Philippines, District 4, Block No. 47; and it is particularly so because the policy prepared and issued by the very defendant specifically states that all insurance covered under said policy, includes the 'rent or other subject matter of insurance in respect of or in connection with any building or any property contained in any building'. Under the evidence on record We cannot alter in the least the decision aforequoted which is hereby adopted by this Court." Las conclusiones de hecho del Juzgado de Primera Instancia (sin incluir la relacion de hechos) que fueron adoptadas por el Tribunal de Apelacion son las siguientes: "After considering the manner of testifying of these witnesses, the evasiveness of the witness Limpe, the improbability of his testimony, and the failure of defendant to present Antonio Paredes, the clerk who admittedly investigated the premises in question, the Court finds that plaintiff's version is more credible; that, before the policy (Exhibit A) was issued, plaintiff made full and clear exposal of hia interests in the premises; and that the said fire policy, (Exhibit A) covers all of plaintiff's interests in the premises No. 34 Plaza Sta. Cruz, Manila, especially his right to collect rentals therefrom under the decision of this Court in Civil Case No. 6306 (Exhibit C). This finding is further strengthened by the fact that paragraph 4 of the said fire policy (Exhibit A), above quoted, includes insurance 'on rent'; and accords with rule that a policy is to be interpreted in favor of the assured. "The argument of the defendant that, under section 49 of the Insurance Law, a policy of insurance must specify the interest of the insured in the property insured, if he is not the absolute owner thereof, is not meritorious because it was the defendant, not plaintiff, who prepared that policy, and it cannot take advantage of its own acts to plaintiff's detriment; and, in any case, this provision was substantially complied with by plaintiff when he made a full and clear statement of his interests to defendant's manager. "Having found that the policy covered all of plaintiff's interests in the premises described therein, including his right to receive rentals, we must next determine whether he had any insurable interest therein when the policy was issued and when the fire occurred.

We find that he did so have. By virtue of the contract between Tomas B. Lianco and the Archbishop, Lianco erected the building of which the premises in question form part and became owner thereof (Exhibit 4-D). He transferred the ownership of the premises in question to Kaw Eng Si (Exhibit D), who in turn transferred it to plaintiff Juan Golangco (Exhibit E). Lianco and the actual occupant of the premises acknowledged plaintiff's right to collect rentals thereon in a compromise agreement which was incorporated in a judicial judgment (Exhibit C). Both at the time of the issuance of the policy and at the time of the fire, plaintiff Golangco was in legal possession of the premises, collecting rentals from its occupant (tr., Nov. 7, 1950, pp. 8, 10). It seems plain that if the premises were destroyedas they wereby fire, Golangco would be, as he was, directly damnified thereby; and hence he had an insurable interest therein (section 12, Insurance Law). "Defendant's contrary contentions are without merit. The contract between Lianco and the Archbishop only forbade Lianco from transferring 'his rights as lessee' (Exhibit. 4-D); but the contracts Lianco made in favor of Kaw Eng Si (Exhibit D) and plaintiff Golangco (Exhibit C) did not transfer such rights; and hence no written consent thereto was necessary. At worst, the contract would be voidable, but not a void contract, at the option of the Archbishop; but this would not deprive Golangco of his insurable interest until such option were exercised; and it does not appear that it was ever exercised. "The ejectment case filed by the Archbishop against Lianco did not remove nor destroy plaintiff's insurable interest: first, because plaintiff was not a party thereto and cannot be bound thereby; and second, because the judgment of the Municipal Court, at least as late as February 14, 1950, had not been executed so far as possession of the premises were concerned (Exhibit G-10). In fact, not even garnishments were issued against Melitona Estrella, So Eng Si (her husband) or plaintiff Golangco, the actual and legal possessors of the premises (Exhibit F); so that, as far as plaintiff Golangco was concerned, his right to the premises and to the rentals thereon continued to exist on June 5, 1949 when the fire took place." Las conclusiones de hecho adoptadas por el Tribunal de Apelacion establecen que el demandante aseguro su interes en el edificio No. 34 Plaza Sta. Cruz, consistente en el derecho de cobrar alquileres y que dicho interes asegurable existia al tiempo del seguro y al ocurrir el incendio. Carece de base, por tanto, la contencion de la recurrente de que la decision del Tribunal de Apelacion no contiene conclusiones de hecho de las cuestiones debidamente suscitadas. Error 1 (b). La contencion de la recurrente de que el Tribunal de Apelacion debio de haber hecho constar en su decision las conclusiones de hecho relativas a la aplicacion

de la regla sobre prueba oral tampoco tiene fundamento. Si la apelante queria suscitar la indebida admision del testimonio oral del demandante, o si queria pedir el descarte de dicho testimonio, debio de haberlo senalado en la relation de errores, diciendo que el juez erro al admitir el testimonio oral del demandante, o que el juez erro al no descartar el testimonio oral del demandante a pesar de la petition debidamente presentada. En el parrafo 3 de los argumentos en apoyo del primer error, bajo el titulo de Argument, (pag. 13 del alegato presentado en el Tribunal de Apelacion), es cuando tal cuestion se planteo por primera vez, en vez de suscitarla en la relation de errores: no se planteo, pues, en su debido lugar y en el tiempo oportuno. (Regla 48, art. 17). El Tribunal de Apelacion no es un buzo que tiene que buscar en los argumentos del alegato cuales son los errores cometidos. En cuanto al segundo error, o sea, que el Tribunal de Apelacion no ha establecido conclusions de hecho sobre los Exhibits 10-H y 10-I, opinamos que esta despojado de merito, por dos razones: l.a porque no se suscito en la relation de errores, y 2.a porque en la decision existen tales conclusiones, aunque no son del agrado de la recurrente. Ella dice en su alegato, pagina SI: "The Court of Appeals adopted the finding of the trial court that 'not even garnishment was issued against Melitona Estrella, So Eng Si (her husband), or plaintiff Golangco'. The decision of the Court of Appeals should therefore contain the following finding of fact: (a) That Exhibits '10-H' and '10-I' clearly prove that notice of garnishment was served on No. 34 Plaza Sta. Cruz (the property in question) and the occupants of No. 34 Plaza Sta. Cruz made return to the said garnishment." Por lo visto, la recurrente desea que este Tribunal enmiende las conclusiones de hecho del Tribunal de Apelacion sobre los Exhibits 10-H y 10-I, cosa que no podemos hacer. No revisamos las pruebas. De si ha errado o no el Tribunal de Apelacion en dichas conclusiones, no esta en nosotros el enderezarlas. No debemos inmiscuirnos en las funciones que, por disposition de la ley, corresponden a dicho tribunal. La ley citada por la recurrente dice asi: "Every decision of the Court of Appeals shall contain complete findings of fact on all issues properly raised before it." Pero en espanol dice asi: "Toda decision del Tribunal de Apelaciones contendra una relacion completa de los hechos de todas las cuestiones que se susciten ante el mismo."

La traduccion al castellano no eg exacta. Si nos atuviesemos a la traduccion al espanol, la decision del Tribunal de Apelacion no deberia contener mas que una "relation completa de los hechos de todas las cuestiones" que se hubieren suscitado en el mismo. Faltaria alga, a saber: las conclusiones de hecho del tribunal. La "relation completa de los hechos" no es equivalents a "la relation completa de las conclusiones de hecho". La relation de hechos "statement of facts" viene a ser la narration de los hechos del asunto para que se sepa en que consiste. La ley en ingles habla de complete findings of fact o conclusiones completas de hecho. Las conclusiones de hecho (findings of fact) son las que el tribunal adopta despues de estudiar, discutir y considerar las pruebas contradictorias presentadas en juicio. Como la Ley No. 296 se discutio y se aprobo en ingles, tenemos que adoptar el texto ingles y no la deficiente traduccion al castellano. El articulo 33 de dicha ley, al hablar de cuestiones que se susciten debidamente ante el Tribunal de Apelacion, no quiere decir que este tiene que resolver todas las cuestiones que se susciten en cualquier estado de la causa o en el curso de los argumentos. No es ese el lugar apropiado: hay que suscitarlo en la relation de errores. Notese que emplea las palabras "properly raised" y no raised solamente. No basta suscitar la cuestion: es necesario suscitarla en el lugar y tiempo oportunos. Es una bien establecida practica forense la de que no se considerara ninguna cuestion que no se haya suscitado debidamente en la relation de errores, a menos que se trate de falta de jurisdiction, que se puede suscitar en cualquier estado del asunto. (Enriquez y otros contra Enriquez y otra, 8 Jur. Fil., 574; Capellania de Tambobong contra Antonio, 8 Jur. Fil., 693; Paterno contra Ciudad de Manila, 17 Jur. Fil., 26; Santiago contra Felix, 24 Jur. Fil., 391; Tan Me Nio contra Administrador de Aduanas, 34 Jur. Fil., 992; Granados y Granados contra Bandelaria, 45 Jur. Fil., 530; Gemora contra Concejo Municipal de Hog, 58 Jur. Fil., 377; Sanchez contra Director de Terrenos, 63 Jur. Fil., 403; Tan Si Kiok, et al. vs. Macario Tiacho, 45 Off. Gaz., 2466, 79 Phil., 696; y Villareal vs. The People of the Philippines, 47 Off. Gaz., 191, 84 Phil., 264.) "Where an appeal is taken to this court from any court, the appellant shall file with the clerk of the court below, with his petition for appeal, an assignment of errors, which shall set out separately and particularly each error asserted. No appeal shall be allowed unless such an assignment of errors shall accompany the petition." (Rule 9, Revised Rules of the Supreme Court of the United States, 11 U. S. Supreme Court Reports Digest.) "The Supreme Court of the United States will not consider a question not raised below, not discussed by the lower court, and not included in the assignment, of errors." (Pacific States Box & Basket Co. vs. S. T. White, et al., 80 L ed., 138.) Podriamos enumerar una larga lista de decisiones de los Tribunales Supremos de los estados de la Union Americana que tienen reglamento similar al nuestro; pero los casos citados bastan.

"Error no apuntado en la relacion de errores en una causa civil se conaidera error consentido por la parte interesada." (Vitug vs. Montemayor, 49 Off. Gaz., 5350.) Declaramos que una cuestion no especificada en la relacion de errores no esta debidamente planteada, y el Tribunal de Apelacion no esta obligado a resolverla. Se deniega la petition con costas contra la recurrente. Paras, C. J., Bengzon, Padilla, Montemayor, Reyes, A., Jugo, Baubista Angelo, Concepcion, y Reyes, J. B. L., MM., estan comformes.

[GR No. L-6442, September 21, 1954] TRADERS INSURANCE & SURETY CO., APPELLANT, AND ANOTHER AGAINST JUAN GOLANGCO, Appellees. DECISION PAUL, M.: This is the collection of a fire insurance policy worth P10, 000. The plaintiff was entitled to receive Pl, 100 monthly Melitona Star in rent Building No. 34 Plaza Sta Cruz, Manila, for five Aryans. The applicant assure this right, pay the premium and the corresponding defendant issued your policy. Two months after the building have been burned, he required the applicant to Traders Insurance & Surety Co. to pay the amount of insurance. As the defendant refused to pay the complainant went to the Court of First Instance of Manila, which issued a ruling in favor of this. The Court of Appeals confirmed the judgment. On certiorari, the Traders Insurance & Surety Co. comes to this Court, alleging that the Court of Appeals made two mistakes: 1 (a) sentencing without a full finding of facts of all the issues raised, as required by Article 33 of Law No. 296, and 1 (b) by failing to make findings of fact regarding the application of the rule of oral evidence, and (2) to pass judgment without findings of fact as to the 10-H Exhibits and 10-I. The error 1 (a). What are the issues that had been properly raised before the Court of Appeals? The best answer is the first page of the allegation of the appellant (now appellant) that reads: BRIEF FOR THE Appellant "ASSIGNMENT OF ERRORS I "THE LOWER COURT IN HOLDING THAT THE erred FIRE INSURANCE POLICY EXHIBIT 'A' appellee COVERS ALL'S INTERESTS IN THE PREMISES NO. PLAZA STA 34. CRUZ, MANILA, ESPECIALLY HIS RIGHT TO COLLECT RENTALS therefrom. II "THE LOWER COURT IN HOLDING THAT THE erred appellee insurable INTEREST HAD A RIGHT TO CONSISTING OP RENTALS RECEIVE BOTH AT THE TIME WHEN THE INSURANCE AND TOOK EFFECT WHEN THE LOSS OCCURRED."

In resolving these issues, the Court of Appeals, after transcribing the entire decision of the Court of First Instance, said the following in its decision: "As stated by the lower court, the basic facts on Which Both Their respective contentions based parties are not disputed, and info we have quoted the decision appealed from in full Because We Find That the facts established in the case can not lead to conclusions other than Those arrived at by the trial judge. There is no doubt in our mind That Both at the time of the execution of the fire policy (Exhibit A) on April 7, 1949, and on June 5, 1949 When the destruction by fire of the Which property for the said policy was issued took place, plaintiff had an insurable Juan Golangco interest on the property insured Which included the rents of premises No. 34 Plaza Sta Cruz, Manila, Philippines, District 4, Block No. 47, and it Because Particularly so is the policy prepared and issued by the defendant very especficamente That All states covered under said insurance policy, includes the 'rent or other subject matter of insurance in respect of or in Connection with any building or any property in any building Contained' . Under the evidence on record We can not alter the decision in the Least aforequoted Which is hereby ADOPTED by This Court. " The findings of fact of the Court of First Instance (not including the relation of facts) which were adopted by the Court of Appeals are as follows: "After Considering the manner of Testifying of These witnesses, the evasiveness of the witness Limpe, the improbability of his testimony, and the failure of defendant to present Antonio Paredes, the clerk who admittedly Investigated the premises in question, the Court finds That plaintiff's version is more credible; That, before the policy (Exhibit A) was issued, plaintiff made full and clear exposal of hia interests in the premises, and That the said fire policy (Exhibit A) covers all of plaintiff's interests in the premises No. 34 Plaza Sta Cruz, Manila, Especially his right to collect rentals therefrom under the decision of this Court in Civil Case No. 6306 (Exhibit C). This finding is further Strengthened by the fact That paragraph 4 of the said fire policy (Exhibit A), quoted above, includes insurance 'on rent', and accords with rule That a policy is to be interpreted in behalf of the assured. "The argument of the defendant That, under section 49 of the Insurance Law, a policy of insurance must specify the interest of the insured in the property insured, if he is not the absolute owner thereof, is not meritorious porque was the defendant, not plaintiff, who prepared That policy, and it can not take advantage of its own acts to plaintiff's detriment, and, in any case, This provision was Substantially complied with by plaintiff When He made a full and clear statement of his interests to defendant's manager. "Having found That the policy covered all of plaintiff's interests in the premises Described Therein, including his right to receive rentals, we must next determine

Whether I had any insurable interest Therein When the policy was issued and when to the fire occurred. We Find That I so did have. By virtue of the Contract Between Tomas B. Lianco and the Archbishop, the building erected Lianco Which of the premises in question form and Became part owner thereof (Exhibit 4-D).'ve transferred is the ownership of the premises in question to Kaw Eng Si (Exhibit D), who in turn transferred is it to plaintiff Juan Golangco (Exhibit E). Lianco and the current occupant of the premises Acknowledged plaintiff's right to collect rentals thereon in a compromise agreement Which was incorporated in a court judgment ( Exhibit C). Both at the time of the issuance of the policy and at the time of the fire, Golangco plaintiff was in possession of the legal premises, collecting rentals from its occupant (tr., Nov. 7, 1950, pp. 8. ., 10) It Seems That if the premises plain Were Peak Gust destroyed-were-by fire, Golangco would be, as I was, Directly damnified thereby, and HENCE I had an insurable interest Therein (section 12 Insurance Law). . "Defendant's contentions are without merit Contrary The Contract Between Lianco and the Archbishop forbade only Lianco Transferring from 'his rights as lessee' (Exhibit. 4D), but the contracts made in behalf of Lianco Kaw Eng Si (Exhibit D) and Golangco plaintiff (Exhibit C) Such rights did not transfer,. HENCE and written consent thereto was not Necessary At worst, the contract would be voidable, but not void a contract, at the option of the Archbishop, But This would not deprive Golangco of until his insurable interest Such Were Exercised option, and it does not Appear That it was ever Exercised. "The ejectment case filed by the Archbishop against Lianco did not remove nor destroy plaintiff's insurable interest: first, Because plaintiff was not a party thereto and can not be bound thereby, and second, Because the judgment of the Municipal Court, At least as late as February 14, 1950, had not Been executed so far as possession of the premises concerned Were (Exhibit G-10.) In fact, not even garnishments Were issued against Melitona Star, So If Eng (her husband) or plaintiff Golangco, the current and legal possessors of the premises (Exhibit F) That so, as far as plaintiff was concerned Golangco, his right to the premises and to the rentals thereon continued to exist on June 5, 1949 When the fire took place. " The findings of fact made by the Court of Appeals established that the applicant assure your interest in the building No. 34 Plaza Sta Cruz, consisting of the right to collect rents and said while there was insurable interest insurance and fire occur . No basis, therefore, the contention of the appellant that the decision of the Court of Appeals does not contain findings of fact properly raised issues. Error 1 (b). The contention of the appellant that the Court of Appeals should have been recorded in its decision findings of fact regarding the application of the rule on oral test

also unfounded. If the appellant wanted to raise the improper admission of oral testimony of the plaintiff, or if I wanted to order the dismissal of that testimony, I must've noted in relation error, saying the errors in admitting oral testimony of the complainant judge, or the errors by not discarding the oral testimony of the applicant despite the judge properly submitted petition. In paragraph 3 of the arguments in support of the first error, under the title of Argument, (page 13 of the brief filed in the Court of Appeals), is when this question was raised for the first time, instead of arousing it in relation Error: can not therefore wont in its proper place at the right time. (Rule 48, art. 17). The Court of Appeals is not a diver needs to look at the arguments which are alleged mistakes. As for the second error, that is, that the Court of Appeals has not established conclusions of fact on Exhibits 10-H and 10-I, we believe that this stripped merit for two reasons: because they are not aroused in relation errors, and because the decision 2D exist such conclusions, although they are not liked by the appellant. She says in her brief, page SI: "The Court of Appeals ADOPTED the finding of the trial court that 'not even garnishment was issued against Melitona Star, So If Eng (her husband), or Golangco plaintiff." The decision of the Court of Appeals Should Contain the Following THEREFORE finding of fact: (A) That-H Exhibits '10 'and '10-I' Clearly prove That notice of garnishment was served on No. 34 Plaza Sta Cruz (the property in question) and the occupants of No. 34 Plaza Sta Cruz made return to the said garnishment. " Apparently, the appellant wants the Court to amend the findings of fact of the Court of Appeals on Exhibits 10-H and 10-I, which we can not do. We do not review the evidence. Of if you have not missed or the Court of Appeals in those conclusions, not on us straighten. We should not interfere in the functions, by disposition of law, correspond to that court. The law cited by the appellant reads: "Every decision of the Court of Appeals Shall Contain complete findings of fact on all issues properly raised before it." But in Spanish reads: "Any decision of the Court of Appeals will contain a full statement of the facts of all questions arising before it." The translation into Castilian eg not accurate. If we atuviesemos the translations to Spanish, the decision of the Court of Appeals should not contain more than a "full relation of the events of all issues" that may have been raised in it. Would fail alga,

namely the factual findings of the court. The "full relation of the facts" is not equivalents to "the complete relation of the findings of fact." The relation of facts "statement of facts" becomes the narration of the facts of the case so you know where it is. The English law speaks of complete findings of fact or conclusions of fact complete. Findings of fact (findings of fact) is that the court adopts after study, discuss and consider the conflicting evidence presented at trial. As Law No. 296 was discussed and was approved in English, we have to take the English text and not the poor translation into Castilian. Article 33 of the Act, to discuss issues that arise due to the Court of Appeals, does not mean that it has to resolve all issues that may arise at any stage of the proceedings or in the course of the arguments. This is not the appropriate place: we must stir it in relation error. Note that uses the words "properly raised" not only raised. Not enough to raise the question: is necessary arousing it in place and appropriate time. It is a wellestablished forensic practices that any question that has not been properly raised in relation error, unless in the case of lack of jurisdiction, which may raise at any stage of the case was not considered. (Enriquez and others v. Enriquez and another 8 Phil, 574,. Chaplaincy of Tambobong against Antonio, 8 Phil, 693,. Paterno v. City of Manila, 17 Jur Fil, 26;. Against Felix Santiago, 24 Jur Fil, 391,. against Tan Me Nio Collector of Customs, 34 Jur Fil, 992,. Granados Granados and against Bandelaria, 45 Jur Fil, 530,. Gemora against Hog City Council, 58 Jur Fil. 377; Sanchez v. Director of Lands, 63 Jur Fil, 403;. Kiok Tan Si, et al vs Tiacho Macario, 45 Off Gaz, 2466, 79 Phil, 696,......, and Villareal vs The People of the Philippines, 47 Off. Gaz., 191, 84 Phil., 264.) "Where an appeal is taken to this court from any court, the appellant Shall file With the clerk of the court below, With his petition for appeal, an assignment of errors, que Shall set out separately and each mistake Particularly Asserted. Shall not appeal Unless be allowed Such an assignment of errors Shall Accompany the petition. " (Rule 9 Revised Rules of the Supreme Court of the United States, 11 U.S. Supreme Court Reports Digest.) "The Supreme Court of the United States will not Consider a question not raised below, not by the lower court discussed This, and not included in the assignment, of errors. " (Pacific States Box & Basket Co. v.. ST White, et al., 80 L ed., 138.) We could enumerate a long list of decisions of the Supreme Courts of the States of the American Union that are similar to our rules, but the cases cited enough. "Error not shared in the ratio of errors in a civil case by mistake consent conaidera interested party." (Vitug vs. Montemayor, 49 Off. Gaz., 5350.) Declare that a matter not specified in the ratio of errors not properly raised, and the Court of Appeals is not required to resolve it. The petition with costs against the appellant is denied.

Paras, CJ, Bengzon, Padilla, Montemayor, Reyes, A., Jugo, Baubista Angelo, Concepcion and Reyes, JBL, MM., Are comformes.

G.R. No. 85141 November 28, 1989 FILIPINO MERCHANTS INSURANCE CO., INC., petitioner, vs. COURT OF APPEALS and CHOA TIEK SENG, respondents. Balgos & Perez Law Offices for petitioner. Lapuz Law office for private respondent.

REGALADO, J.: This is a review of the decision of the Court of Appeals, promulgated on July 19,1988, the dispositive part of which reads: WHEREFORE, the judgment appealed from is affirmed insofar as it orders defendant Filipino Merchants Insurance Company to pay the plaintiff the sum of P51,568.62 with interest at legal rate from the date of filing of the complaint, and is modified with respect to the third party complaint in that (1) third party defendant E. Razon, Inc. is ordered to reimburse third party plaintiff the sum of P25,471.80 with legal interest from the date of payment until the date of reimbursement, and (2) the third-party complaint against third party defendant Compagnie Maritime Des Chargeurs Reunis is dismissed. 1 The facts as found by the trial court and adopted by the Court of Appeals are as follows: This is an action brought by the consignee of the shipment of fishmeal loaded on board the vessel SS Bougainville and unloaded at the Port of Manila on or about December 11, 1976 and seeks to recover from the defendant insurance company the amount of P51,568.62 representing damages to said shipment which has been insured by the defendant insurance company under Policy No. M-2678. The defendant brought a third party complaint against third party defendants Compagnie Maritime Des Chargeurs Reunis and/or E. Razon, Inc. seeking judgment against the third (sic) defendants in case Judgment is rendered against the third party plaintiff. It appears from the evidence presented that in December 1976, plaintiff insured said shipment with defendant insurance company under said cargo Policy No. M-2678 for the sum of P267,653.59 for the goods described as 600 metric tons of fishmeal in new gunny bags of 90 kilos each from Bangkok, Thailand to Manila against all risks under warehouse to warehouse terms. Actually, what was imported was 59.940 metric tons not 600 tons at $395.42 a ton CNF Manila. The fishmeal in 666 new gunny bags were unloaded from the ship on December 11, 1976 at Manila unto the arrastre contractor E. Razon, Inc. and defendant's surveyor ascertained and certified that in such discharge 105 bags were in bad order condition as jointly surveyed by the ship's agent and the arrastre contractor.

The condition of the bad order was reflected in the turn over survey report of Bad Order cargoes Nos. 120320 to 120322, as Exhibit C-4 consisting of three (3) pages which are also Exhibits 4, 5 and 6- Razon. The cargo was also surveyed by the arrastre contractor before delivery of the cargo to the consignee and the condition of the cargo on such delivery was reflected in E. Razon's Bad Order Certificate No. 14859, 14863 and 14869 covering a total of 227 bags in bad order condition. Defendant's surveyor has conducted a final and detailed survey of the cargo in the warehouse for which he prepared a survey report Exhibit F with the findings on the extent of shortage or loss on the bad order bags totalling 227 bags amounting to 12,148 kilos, Exhibit F-1. Based on said computation the plaintiff made a formal claim against the defendant Filipino Merchants Insurance Company for P51,568.62 (Exhibit C) the computation of which claim is contained therein. A formal claim statement was also presented by the plaintiff against the vessel dated December 21, 1976, Exhibit B, but the defendant Filipino Merchants Insurance Company refused to pay the claim. Consequently, the plaintiff brought an action against said defendant as adverted to above and defendant presented a third party complaint against the vessel and the arrastre contractor. 2 The court below, after trial on the merits, rendered judgment in favor of private respondent, the decretal portion whereof reads: WHEREFORE, on the main complaint, judgment is hereby rendered in favor of the plaintiff and against the defendant Filipino Merchant's (sic) Insurance Co., ordering the defendants to pay the plaintiff the following amount: The sum of P51,568.62 with interest at legal rate from the date of the filing of the complaint; On the third party complaint, the third party defendant Compagnie Maritime Des Chargeurs Reunis and third party defendant E. Razon, Inc. are ordered to pay to the third party plaintiff jointly and severally reimbursement of the amounts paid by the third party plaintiff with legal interest from the date of such payment until the date of such reimbursement. Without pronouncement as to costs. 3 On appeal, the respondent court affirmed the decision of the lower court insofar as the award on the complaint is concerned and modified the same with regard to the adjudication of the third-party complaint. A motion for reconsideration of the aforesaid decision was denied, hence this petition with the following assignment of errors: 1. The Court of Appeals erred in its interpretation and application of the "all risks" clause of the marine insurance policy when it held the petitioner liable to the private respondent for the partial loss of the cargo, notwithstanding the clear absence of proof of some

fortuitous event, casualty, or accidental cause to which the loss is attributable, thereby contradicting the very precedents cited by it in its decision as well as a prior decision of the same Division of the said court (then composed of Justices Cacdac, CastroBartolome, and Pronove); 2. The Court of Appeals erred in not holding that the private respondent had no insurable interest in the subject cargo, hence, the marine insurance policy taken out by private respondent is null and void; 3. The Court of Appeals erred in not holding that the private respondent was guilty of fraud in not disclosing the fact, it being bound out of utmost good faith to do so, that it had no insurable interest in the subject cargo, which bars its recovery on the policy. 4 On the first assignment of error, petitioner contends that an "all risks" marine policy has a technical meaning in insurance in that before a claim can be compensable it is essential that there must be "some fortuity, " "casualty" or "accidental cause" to which the alleged loss is attributable and the failure of herein private respondent, upon whom lay the burden, to adduce evidence showing that the alleged loss to the cargo in question was due to a fortuitous event precludes his right to recover from the insurance policy. We find said contention untenable. The "all risks clause" of the Institute Cargo Clauses read as follows: 5. This insurance is against all risks of loss or damage to the subject-matter insured but shall in no case be deemed to extend to cover loss, damage, or expense proximately caused by delay or inherent vice or nature of the subject-matter insured. Claims recoverable hereunder shall be payable irrespective of percentage. 5 An "all risks policy" should be read literally as meaning all risks whatsoever and covering all losses by an accidental cause of any kind. The terms "accident" and "accidental", as used in insurance contracts, have not acquired any technical meaning. They are construed by the courts in their ordinary and common acceptance. Thus, the terms have been taken to mean that which happens by chance or fortuitously, without intention and design, and which is unexpected, unusual and unforeseen. An accident is an event that takes place without one's foresight or expectation; an event that proceeds from an unknown cause, or is an unusual effect of a known cause and, therefore, not expected. 6 The very nature of the term "all risks" must be given a broad and comprehensive meaning as covering any loss other than a willful and fraudulent act of the insured. 7 This is pursuant to the very purpose of an "all risks" insurance to give protection to the insured in those cases where difficulties of logical explanation or some mystery surround the loss or damage to property. 8 An "all asks" policy has been evolved to

grant greater protection than that afforded by the "perils clause," in order to assure that no loss can happen through the incidence of a cause neither insured against nor creating liability in the ship; it is written against all losses, that is, attributable to external causes. 9 The term "all risks" cannot be given a strained technical meaning, the language of the clause under the Institute Cargo Clauses being unequivocal and clear, to the effect that it extends to all damages/losses suffered by the insured cargo except (a) loss or damage or expense proximately caused by delay, and (b) loss or damage or expense proximately caused by the inherent vice or nature of the subject matter insured. Generally, the burden of proof is upon the insured to show that a loss arose from a covered peril, but under an "all risks" policy the burden is not on the insured to prove the precise cause of loss or damage for which it seeks compensation. The insured under an "all risks insurance policy" has the initial burden of proving that the cargo was in good condition when the policy attached and that the cargo was damaged when unloaded from the vessel; thereafter, the burden then shifts to the insurer to show the exception to the coverage. 10 As we held in Paris-Manila Perfumery Co. vs. Phoenix Assurance Co., Ltd. 11 the basic rule is that the insurance company has the burden of proving that the loss is caused by the risk excepted and for want of such proof, the company is liable. Coverage under an "all risks" provision of a marine insurance policy creates a special type of insurance which extends coverage to risks not usually contemplated and avoids putting upon the insured the burden of establishing that the loss was due to the peril falling within the policy's coverage; the insurer can avoid coverage upon demonstrating that a specific provision expressly excludes the loss from coverage. 12 A marine insurance policy providing that the insurance was to be "against all risks" must be construed as creating a special insurance and extending to other risks than are usually contemplated, and covers all losses except such as arise from the fraud of the insured. 13 The burden of the insured, therefore, is to prove merely that the goods he transported have been lost, destroyed or deteriorated. Thereafter, the burden is shifted to the insurer to prove that the loss was due to excepted perils. To impose on the insured the burden of proving the precise cause of the loss or damage would be inconsistent with the broad protective purpose of "all risks" insurance. In the present case, there being no showing that the loss was caused by any of the excepted perils, the insurer is liable under the policy. As aptly stated by the respondent Court of Appeals, upon due consideration of the authorities and jurisprudence it discussed ... it is believed that in the absence of any showing that the losses/damages were caused by an excepted peril, i.e. delay or the inherent vice or nature of the subject

matter insured, and there is no such showing, the lower court did not err in holding that the loss was covered by the policy. There is no evidence presented to show that the condition of the gunny bags in which the fishmeal was packed was such that they could not hold their contents in the course of the necessary transit, much less any evidence that the bags of cargo had burst as the result of the weakness of the bags themselves. Had there been such a showing that spillage would have been a certainty, there may have been good reason to plead that there was no risk covered by the policy (See Berk vs. Style [1956] cited in Marine Insurance Claims, Ibid, p. 125). Under an 'all risks' policy, it was sufficient to show that there was damage occasioned by some accidental cause of any kind, and there is no necessity to point to any particular cause. 14 Contracts of insurance are contracts of indemnity upon the terms and conditions specified in the policy. The agreement has the force of law between the parties. The terms of the policy constitute the measure of the insurer's liability. If such terms are clear and unambiguous, they must be taken and understood in their plain, ordinary and popular sense. 15 Anent the issue of insurable interest, we uphold the ruling of the respondent court that private respondent, as consignee of the goods in transit under an invoice containing the terms under "C & F Manila," has insurable interest in said goods. Section 13 of the Insurance Code defines insurable interest in property as every interest in property, whether real or personal, or any relation thereto, or liability in respect thereof, of such nature that a contemplated peril might directly damnify the insured. In principle, anyone has an insurable interest in property who derives a benefit from its existence or would suffer loss from its destruction whether he has or has not any title in, or lien upon or possession of the property y. 16 Insurable interest in property may consist in (a) an existing interest; (b) an inchoate interest founded on an existing interest; or (c) an expectancy, coupled with an existing interest in that out of which the expectancy arises. 17 Herein private respondent, as vendee/consignee of the goods in transit has such existing interest therein as may be the subject of a valid contract of insurance. His interest over the goods is based on the perfected contract of sale. 18 The perfected contract of sale between him and the shipper of the goods operates to vest in him an equitable title even before delivery or before be performed the conditions of the sale. 19 The contract of shipment, whether under F.O.B., C.I.F., or C. & F. as in this case, is immaterial in the determination of whether the vendee has an insurable interest or not in the goods in transit. The perfected contract of sale even without delivery vests in the

vendee an equitable title, an existing interest over the goods sufficient to be the subject of insurance. Further, Article 1523 of the Civil Code provides that where, in pursuance of a contract of sale, the seller is authorized or required to send the goods to the buyer, delivery of the goods to a carrier, whether named by the buyer or not, for, the purpose of transmission to the buyer is deemed to be a delivery of the goods to the buyer, the exceptions to said rule not obtaining in the present case. The Court has heretofore ruled that the delivery of the goods on board the carrying vessels partake of the nature of actual delivery since, from that time, the foreign buyers assumed the risks of loss of the goods and paid the insurance premium covering them. 20 C & F contracts are shipment contracts. The term means that the price fixed includes in a lump sum the cost of the goods and freight to the named destination. 21 It simply means that the seller must pay the costs and freight necessary to bring the goods to the named destination but the risk of loss or damage to the goods is transferred from the seller to the buyer when the goods pass the ship's rail in the port of shipment. 22 Moreover, the issue of lack of insurable interest was not among the defenses averred in petitioners answer. It was neither an issue agreed upon by the parties at the pre-trial conference nor was it raised during the trial in the court below. It is a settled rule that an issue which has not been raised in the court a quo cannot be raised for the first time on appeal as it would be offensive to the basic rules of fair play, justice and due process. 23 This is but a permuted restatement of the long settled rule that when a party deliberately adopts a certain theory, and the case is tried and decided upon that theory in the court below, he will not be permitted to change his theory on appeal because, to permit him to do so, would be unfair to the adverse party. 24 If despite the fundamental doctrines just stated, we nevertheless decided to indite a disquisition on the issue of insurable interest raised by petitioner, it was to put at rest all doubts on the matter under the facts in this case and also to dispose of petitioner's third assignment of error which consequently needs no further discussion. WHEREFORE, the instant petition is DENIED and the assailed decision of the respondent Court of Appeals is AFFIRMED in toto. SO ORDERED. Paras, Padilla and Sarmiento, JJ., concur. Melencio-Herrera (Chairperson), J., is on leave.

FROILAN LOPEZ, PLAINTIFF AND APPELLANT, VS. SALVADOR V. DEL ROSARIO AND BENITA QUIOGUE DE V. DEL ROSARIO, DEFENDANTS AND APPELLANTS. DECISION MALCOLM, J.: Both parties to this action appeal from the judgment of Judge Simplicio del Rosario of the Court of First Instance of Manila awarding the plaintiff tire sum of P88,495.21 with legal interest from May 13, 1921, without special finding as to costs. The many points pressed by contending counsel can be best disposed of by, first, making a statement of the facts; next, considering plaintiff's appeal; next, considering defendant's appeal; and, lastly, rendering judgment. STATEMENT OF THE FACTS On and prior to June 6, 1920, Benita Quiogue de V. del Rosario, whom we will hereafter call Mrs. Del Rosario, was the owner of a bonded warehouse situated in the City of Manila. She was engaged in the business of a warehouse keeper, and stored copra and other merchandise in the said building. Among the persons who had copra deposited in the Del Rosario warehouse was Froilan Lopez, the holder of fourteen warehouse receipts in his own name, and the name of Elias T. Zamora. (Exhibits C, D, and R.) The warehouse receipts, or negotiable warrants, or quedans (as they are variously termed) of Lopez named a declared value of P107,990.40 (Exhibits L-1 to L-13). The warehouse receipts provided: (1) For insurance at the rate of 1 per cent per month on the declared value; (2) the company reserves to itself the right to raise and /or lower the rates of storage and /or of insurance on giving one calendar month's notice in writing; (3) this warrant carries no insurance unless so noted on the face hereof, cost of which is in addition to storage; (4) the time for which storage and /or insurance is charged is thirty (30) days; (5) payment for storage and /or insurance, etc., shall be made in advance, and /or within five (5) days after presentation of bill. It is admitted that insurance was paid by Lopez to May 18, 1920, but not thereafter. Mrs. Del Rosario secured insurance on the warehouse and its contents with the National Insurance Co., Inc., the Commercial Union Insurance Company, the Alliance Insurance Company, the South British Insurance Co., Ltd., and the British Traders Insurance Co., Ltd., in the amount of P404,800. All the policies were in the name of Sra. Benita Quiogue de V. del Rosario, with the exception of one of the National Insurance Company, Inc., for P40,000, in favor of the Compaia Coprera de Tayabas. (Exhibits N, O, P, R-1 to R-4.)

The warehouse of Mrs. Del Rosario and its contents were destroyed by fire on June 6, 1920. The warehouse was a total loss, while of the copra stored therein, only an amount equal to P49,985 was salvaged. Following an unsuccessful attempt by Henry Hunter Bayne, Fire Loss Adjuster, to effect a settlement between the insurance companies and Mrs. Del Rosario, the latter, on August 24, 1920, authorized Attorney F. C. Fisher to negotiate with the various insurance companies. (Exhibit A.) As a result, an agreement between Mrs. Del Rosario and the insurance companies to submit the matter to arbitration was executed in September, 1920. (Exhibit B.) Mrs. Del Rosario laid claim before the arbitrators, Messrs. Muir and Campbell, to P419,683.95, and the proceeds of the salvage sale. The arbitrators in their report allowed Mrs. Del Rosario P363,610, which, with the addition of the money received from the salvaged copra amounting to P49,985, and interest, made a total of P414,258, collected by her from the companies. (Exhibits E, F, G, H, and Q.) Mrs. Del Rosario seems to have satisfied all of the persons who had copra stored in her warehouse, including the stockholders in the Compaia Coprera de Tayabas (whose stock she took over), with the exception of Froilan Lopez, the plaintiff. Ineffectual attempts by Mrs. Del Rosario to effect a compromise with Lopez first for P71,994, later raised to P72,724, and finally reduced to P17,000, were made. (Exhibits Y, 1, 3, 4, 6, 7, 8, 12.) But Lopez stubbornly contended, or, at least, his attorney contended for him, that he should receive not a centavo less than P88,595.43. (Exhibits 4, 5.) PLAINTIFF'S APPEAL Plaintiff, by means of his &m assignment of error, lays claim to P88,595.43 in lieu of P88,495.21 allowed by the trial court. The slight difference of P100.22 is asked for so that plaintiff can participate in the interest money which accrued on the amount received for the salvaged copra. (Exhibits EE and FF.) Defendant makes no specific denial of this claim. We think the additional sum should accrue to the plaintiff. Plaintiff's second and third assignments of error present the point that the defendant has fraudulently-and even criminally-refrained from paying the plaintiff, and that the plaintiff should recover interest at the rate of 12 per cent per annum. We fail to grasp plaintiff's point of view. "The defendant has not sought to elude her moral and legal obligations. The controversy is merely one which unfortunately all too often arises between litigious persons. Plaintiff has exactly the rights of any litigant, equally situated, and no more. It has been the constant practice of the court to make article 1108 of the Civil Code the basis for the calculation of interest. Damages in the form of interest at the rate of 12 per cent, as claimed by the plaintiff, are too remote and speculative to be allowed. The deprivation of an opportunity for making money which might have proved beneficial or might have been ruinous is of too uncertain character to be weighed in the even

balances of the law. (Civil Code, art. 1108; Gonzales Quiros vs. Palanca Tan-Guinlay [1906], 5 Phil., 675; Tin Fian vs. Tan [1909], 14 Phil., 126; Sun Life Insurance Co. of Canada vs. Rueda Hermanos & Co. and Delgado [1918], 37 Phil., 844; Scvola, Codigo Civil, vol. 19, p. 576; 8 R. C. L., 463; 17 C. J., 864.) DEFENDANT'S APPEAL Counsel for defendant have adroitly and ingeniously attempted to avoid all liability. However, we remain unimpressed by many of these arguments. Much time has been spent by counsel for both parties in discussing the question, of whether the defendant acted as the agent of the plaintiff, in taking out insurance on the contents of the bodega, or whether the defendant acted as a reinsurer of the copra. Giving a natural expression to the terms of the warehouse receipts, the first hypothesis is the correct one. The agency can be deduced from the warehouse receipts, the insurance policies, and the circumstances surrounding the transaction. After all, however, this is not so vitally important, for it might well bealthough we do not have to decidethat under any aspect of the case, the defendant would be liable. The law is that a policy effected by a bailee and covering by its terms his own property and property held in trust, inures, in the event of a loss, equally and proportionately to the benefit of all the owners of the property insured. Even if one secured insurance covering his own goods and goods stored with him, and even if the owner of the stored goods did not request or know of the insurance, and did not ratify it before the payment of the loss, yet it has been held by a reputable court that the warehouseman is liable to the owner of such stored goods for his share. (Snow vs. Carr [1878], 61 Ala., 363; 32 Am. Rep., 3; Broussard vs. South Texas Rice Co. [1910], 103 Tex., 535; Ann, Cas., 1913-A, 142, and note; Home Insurance Co. of New York vs. Baltimore Warehouse Co. [1876], 93 U. S., 527.) Moreover, it has next escaped our notice that in two documents, one the agreement for arbitration, and the other the statement of claim of Mrs. Del Rosario, against the insurance companies, she acknowledged her responsibility to the owners of the stored merchandise, against risk of loss by fire. (Exhibits B and C-3.) The award of the arbitrators covered not alone Mrs. Del Rosario's warehouse but the products stored in the warehouse by Lopez and others. Plaintiff's rights to the insurance money have not been forfeited by failure to pay the insurance provided for in the warehouse receipts. A preponderance of the proof does not demonstrate that the plaintiff ever ordered the cancellation of his insurance with the defendant. Nor is it shown that the plaintiff ever refused to pay the insurance when the bills were presented to him, and that notice of an intention to cancel the insurance was ever given the plaintiff.

The record of the proceedings before the board of arbitrators, and its report and findings, were properly taken into consideration by the trial court as a basis for the determination of the amount due from the defendant to the plaintiff. In a case of contributing policies, adjustments of loss made by an expert or by a board of arbitrators may be submitted to the court not as evidence of the facts stated therein, or as obligatory, but for the purpose of assisting the court in calculating the amount of liability. (Home Insurance Co. vs. Baltimore Warehouse Co., supra.) Counsel for the defendant have dwelt at length on the phraseology of the policies of the National Insurance Company, Inc. Special emphasis has been laid upon one policy (Exhibit 9) in the name of the Compaia Coprera de Tayabas. In this connection it may be said that three members of the court, including the writer of this opinion, have been favorably impressed by this argument, and would have preferred at least to eliminate the policy for which premiums were paid, not by Mrs. Del Rosario on behalf of Lopez and others, but by the Compaia Coprera de Tayabas. A majority, of the court, however, believe that all the assets should be marshalled and that the plaintiff should receive the benefit accruing from the gross amount realized from all the policies. Consequently, no deduction for this claim can be made. The remaining contention of the defendant that the plaintiff cannot claim the benefits of the agency without sharing in the expenses, is well taken. Although the plaintiff did not expressly authorize the agreement to submit the matter to arbitration, yet on his own theory of the case, Mrs. Del Rosario was acting as his agent in securing insurance, while he benefits from the amicable adjustment of the insurance claims. As no intimation is made that the expenses were exorbitant, we necessarily accept the statement of the same appearing in Exhibits Q and 8. Of the insurance money, totalling P414,258, P382,558 was for copra and the remainder for buildings, corn, etc. The expenses for collecting the P414,258 totalled P33,600. 382,558/414,258 of P33,600 equals P31,028.85, the proportionate part of the expenses with reference to the copra. Of the expenses amounting, as we have said, to P31,028.85, plaintiff would be liable for his proportionate share or 88,595.43/382,558.00 of P31,028.85 orP7,185.875. The parties finally agree that the plaintiff at the time of the fire was indebted to the defendant for storage and insurance in the sum of P315.90. JUDGMENT In resume, the result is to sustain plaintiff's first assignment of error and to overrule his second and third assignments of error, to overrule defendant's assignments of error 1, 2, 3, and 4 in toto and to accede to defendant's assignments of error 5, 6, and 7 in part. If our mathematics are correct, and the amounts can be figured in several different

ways, plaintiff is entitled to P88,595.43 minus P7,185.88, his share of the expenses, minus P315.90, due for insurance and storage, or approximately a net amount of P81,093.65, with legal interest. This sum the defendant must disgorge. Wherefore, judgment is modified and the plaintiff shall have and recover from the defendants the sum of P81,093.65, with interest at 6 per cent per annum from May 13, 1921, until paid. Without special finding as to costs in either instance, it is so ordered. Araullo, C. J., Street, Avancea, Villamor, Ostrand, Johns, and Romualdez, JJ., concur. Johnson, J., did not take part.

SPOUSES NILO CHA AND STELLA UY CHA, AND UNITED INSURANCE CO., INC., PETITIONERS, VS. COURT OF APPEALS AND CKS DEVELOPMENT CORPORATION, RESPONDENTS. DECISION PADILLA, J.: This petition for review on certiorari under Rule 45 of the Rules of Court seeks to set aside a decision of respondent Court of Appeals. The undisputed facts of the case are as follows: 1. Petitioner-spouses Nilo Cha and Stella Uy-Cha, as lessees, entered into a lease contract with private respondent CKS Development Corporation (hereinafter CKS), as lessor, on 5 October 1988. 2. One of the stipulations of the one (1) year lease contract states:

18. x x x. The LESSEE shall not insure against fire the chattels, merchandise, textiles, goods and effects placed at any stall or store or space in the leased premises without first obtaining the written consent and approval of the LESSOR. If the LESSEE obtain(s) the insurance thereof without the consent of the LESSOR then the policy is deemed assigned and transferred to the LESSOR for its own benefit; x x x[1] 3. Notwithstanding the above stipulation in the lease contract, the Cha spouses insured against loss by fire their merchandise inside the leased premises for Five Hundred Thousand (P500,000.00) with the United Insurance Co., Inc. (hereinafter United) without the written consent of private respondents CKS. 4. On the day that the lease contract was to expire, fire broke out inside the leased premises. 5. When CKS learned of the insurance earlier procured by the Cha spouses (without its consent), it wrote the insurer (United) a demand letter asking that the proceeds of the insurance contract (between the Cha spouses and United) be paid directly to CKS, based on its lease contract with Cha spouses. 6. United refused to pay CKS. Hence, the latter filed a complaint against the Cha spouses and United. 7. On 2 June 1992, the Regional Trial Court, Branch 6, Manila, rendered a decision*

ordering therein defendant United to pay CKS the amount of P335,063.11 and defendant Cha spouses to pay P50,000.00 as exemplary damages, P20,000.00 as attorneys fees and costs of suit. 8. On appeal, respondent Court of Appeals in CA GR CV No. 39328 rendered a decision** dated 11 January 1996, affirming the trial court decision, deleting however the awards for exemplary damages and attorneys fees. A motion for reconsideration by United was denied on 29 March 1996. In the present petition, the following errors are assigned by petitioners to the Court of Appeals: I

THE HONORABLE COURT OF APPEALS ERRED IN FAILING TO DECLARE THAT THE STIPULATION IN THE CONTRACT OF LEASE TRANSFERRING THE PROCEEDS OF THE INSURANCE TO RESPONDENT IS NULL AND VOID FOR BEING CONTRARY TO LAW, MORALS AND PUBLIC POLICY II

THE HONORABLE COURT OF APPEALS ERRED IN FAILING TO DECLARE THE CONTRACT OF LEASE ENTERED INTO AS A CONTRACT OF ADHESION AND THEREFORE THE QUESTIONABLE PROVISION THEREIN TRANSFERRING THE PROCEEDS OF THE INSURANCE TO RESPONDENT MUST BE RULED OUT IN FAVOR OF PETITIONER III

THE HONORABLE COURT OF APPEALS ERRED IN AWARDING PROCEEDS OF AN INSURANCE POLICY TO APPELLEE WHICH IS NOT PRIVY TO THE SAID POLICY IN CONTRAVENTION OF THE INSURANCE LAW IV

THE HONORABLE COURT OF APPEALS ERRED IN AWARDING PROCEEDS OF AN INSURANCE POLICY ON THE BASIS OF A STIPULATION WHICH IS VOID FOR BEING WITHOUT CONSIDERATION AND FOR BEING TOTALLY DEPENDENT ON THE WILL OF THE RESPONDENT CORPORATION.[2]

The core issue to be resolved in this case is whether or not the aforequoted paragraph 18 of the lease contract entered into between CKS and the Cha spouses is valid insofar as it provides that any fire insurance policy obtained by the lessee (Cha spouses) over their merchandise inside the leased premises is deemed assigned or transferred to the lessor (CKS) if said policy is obtained without the prior written of the latter. It is, of course, basic in the law on contracts that the stipulations contained in a contract cannot be contrary to law, morals, good customs, public order or public policy. [3] Sec. 18 of the Insurance Code provides: Sec. 18. No contract or policy of insurance on property shall be enforceable except for the benefit of some person having an insurable interest in the property insured. A non-life insurance policy such as the fire insurance policy taken by petitioner-spouses over their merchandise is primarily a contract of indemnity. Insurable interest in the property insured must exist at the time the insurance takes effect and at the time the loss occurs.[4] The basis of such requirement of insurable interest in property insured is based on sound public policy: to prevent a person from taking out an insurance policy on property upon which he has no insurable interest and collecting the proceeds of said policy in case of loss of the property. In such a case, the contract of insurance is a mere wager which is void under Section 25 of the Insurance Code, which provides: SECTION 25. Every stipulation in a policy of Insurance for the payment of loss, whether the person insured has or has not any interest in the property insured, or that the policy shall be received as proof of such interest, and every policy executed by way of gaming or wagering, is void. In the present case, it cannot be denied that CKS has no insurable interest in the goods and merchandise inside the leased premises under the provisions of Section 17 of the Insurance Code which provide. Section 17. The measure of an insurable interest in property is the extent to which the insured might be damnified by loss of injury thereof." Therefore, respondent CKS cannot, under the Insurance Code a special law be validly a beneficiary of the fire insurance policy taken by the petitioner-spouses over their merchandise. This insurable interest over said merchandise remains with the insured, the Cha spouses. The automatic assignment of the policy to CKS under the provision of the lease contract previously quoted is void for being contrary to law and/or public policy. The proceeds of the fire insurance policy thus rightfully belong to the

spouses Nilo Cha and Stella Uy-Cha (herein co-petitioners). The insurer (United) cannot be compelled to pay the proceeds of the fire insurance policy to a person (CKS) who has no insurable interest in the property insured. The liability of the Cha spouses to CKS for violating their lease contract in that Cha spouses obtained a fire insurance policy over their own merchandise, without the consent of CKS, is a separate and distinct issue which we do not resolve in this case. WHEREFORE, the decision of the Court of Appeals in CA-G.R. CV No. 39328 is SET ASIDE and a new decision is hereby entered, awarding the proceeds of the fire insurance policy to petitioners Nilo Cha and Stella Uy-Cha. SO ORDERED. Bellosillo, Vitug, Kapunan, and Hermosisima, Jr., JJ., concur.

G.R. No. L-55397 February 29, 1988 TAI TONG CHUACHE & CO., petitioner, vs. THE INSURANCE COMMISSION and TRAVELLERS MULTI-INDEMNITY CORPORATION, respondents.

GANCAYCO, J.: This petition for review on certiorari seeks the reversal of the decision of the Insurance Commission in IC Case #367 1 dismissing the complaint 2 for recovery of the alleged unpaid balance of the proceeds of the Fire Insurance Policies issued by herein respondent insurance company in favor of petitioner-intervenor. The facts of the case as found by respondent Insurance Commission are as follows: Complainants acquired from a certain Rolando Gonzales a parcel of land and a building located at San Rafael Village, Davao City. Complainants assumed the mortgage of the building in favor of S.S.S., which building was insured with respondent S.S.S. Accredited Group of Insurers for P25,000.00. On April 19, 1975, Azucena Palomo obtained a loan from Tai Tong Chuache Inc. in the amount of P100,000.00. To secure the payment of the loan, a mortgage was executed over the land and the building in favor of Tai Tong Chuache & Co. (Exhibit "1" and "1A"). On April 25, 1975, Arsenio Chua, representative of Thai Tong Chuache & Co. insured the latter's interest with Travellers Multi-Indemnity Corporation for P100,000.00 (P70,000.00 for the building and P30,000.00 for the contents thereof) (Exhibit "A-a," contents thereof) (Exhibit "A-a"). On June 11, 1975, Pedro Palomo secured a Fire Insurance Policy No. F- 02500 (Exhibit "A"), covering the building for P50,000.00 with respondent Zenith Insurance Corporation. On July 16, 1975, another Fire Insurance Policy No. 8459 (Exhibit "B") was procured from respondent Philippine British Assurance Company, covering the same building for P50,000.00 and the contents thereof for P70,000.00. On July 31, 1975, the building and the contents were totally razed by fire. Adjustment Standard Corporation submitted a report as follow xxx xxx xxx ... Thus the apportioned share of each company is as follows:

Policy No.. MIRO F-02500

Company Zenith Insurance Corp.

Risk Building

Insures P50,000

Pays P17,610.93

F-84590

Phil. British Assco. Co. Inc.

Household

70,000

24,655.31

FFF & F5 Risk

50,000 Insures

39,186.10 Pays

Policy No. FIC-15381

Company SSSAccre dited Group of Insurers

Building Totals

P25,000 P195,000

P8,805.47 P90,257.81

We are showing hereunder another apportionment of the loss which includes the Travellers Multi-Indemnity policy for reference purposes. Policy No. MIRO/ F-02500 Company Zenith Insurance Risk Injures Pays

Corp. F-84590 Phil. British Assco. Co.

Building

P50,000

P11,877.14

I-Building

70,000 II-Building

16,628.00

FFF & PE PVC-15181 SSS Group of Insurers F-599 DV Insurers Multi Building I-Ref II-Building Totals Accredited

50,000

24,918.79

25,000 30,000 70,000 P295.000

5,938.50 14,467.31 16,628.00 P90,257.81

Based on the computation of the loss, including the Travellers Multi- Indemnity, respondents, Zenith Insurance, Phil. British Assurance and S.S.S. Accredited Group of Insurers, paid their corresponding shares of the loss. Complainants were paid the following: P41,546.79 by Philippine British Assurance Co., P11,877.14 by Zenith Insurance Corporation, and P5,936.57 by S.S.S. Group of Accredited Insurers (Par. 6. Amended Complaint). Demand was made from respondent Travellers Multi-Indemnity for its share in the loss but the same was refused. Hence, complainants demanded from the other three (3) respondents the balance of each share in the loss based on the computation of the Adjustment Standards Report excluding Travellers Multi-Indemnity in the amount of P30,894.31 (P5,732.79-Zenith Insurance: P22,294.62, Phil. British: and P2,866.90, SSS Accredited) but the same was refused, hence, this action.

In their answers, Philippine British Assurance and Zenith Insurance Corporation admitted the material allegations in the complaint, but denied liability on the ground that the claim of the complainants had already been waived, extinguished or paid. Both companies set up counterclaim in the total amount of P 91,546.79. Instead of filing an answer, SSS Accredited Group of Insurers informed the Commission in its letter of July 22, 1977 that the herein claim of complainants for the balance had been paid in the amount of P 5,938.57 in full, based on the Adjustment Standards Corporation Report of September 22, 1975. Travellers Insurance, on its part, admitted the issuance of the Policy No. 599 DV and alleged as its special and affirmative defenses the following, to wit: that Fire Policy No. 599 DV, covering the furniture and building of complainants was secured by a certain Arsenio Chua, mortgage creditor, for the purpose of protecting his mortgage credit against the complainants; that the said policy was issued in the name of Azucena Palomo, only to indicate that she owns the insured premises; that the policy contains an endorsement in favor of Arsenio Chua as his mortgage interest may appear to indicate that insured was Arsenio Chua and the complainants; that the premium due on said fire policy was paid by Arsenio Chua; that respondent Travellers is not liable to pay complainants. On May 31, 1977, Tai Tong Chuache & Co. filed a complaint in intervention claiming the proceeds of the fire Insurance Policy No. F-559 DV, issued by respondent Travellers Multi-Indemnity. Travellers Insurance, in answer to the complaint in intervention, alleged that the Intervenor is not entitled to indemnity under its Fire Insurance Policy for lack of insurable interest before the loss of the insured premises and that the complainants, spouses Pedro and Azucena Palomo, had already paid in full their mortgage indebtedness to the intervenor. 3 As adverted to above respondent Insurance Commission dismissed spouses Palomos' complaint on the ground that the insurance policy subject of the complaint was taken out by Tai Tong Chuache & Company, petitioner herein, for its own interest only as mortgagee of the insured property and thus complainant as mortgagors of the insured property have no right of action against herein respondent. It likewise dismissed petitioner's complaint in intervention in the following words: We move on the issue of liability of respondent Travellers Multi-Indemnity to the Intervenor-mortgagee. The complainant testified that she was still indebted to Intervenor in the amount of P100,000.00. Such allegation has not however, been sufficiently proven by documentary evidence. The certification (Exhibit 'E-e') issued by the Court of

First Instance of Davao, Branch 11, indicate that the complainant was Antonio Lopez Chua and not Tai Tong Chuache & Company. 4 From the above decision, only intervenor Tai Tong Chuache filed a motion for reconsideration but it was likewise denied hence, the present petition. It is the contention of the petitioner that respondent Insurance Commission decided an issue not raised in the pleadings of the parties in that it ruled that a certain Arsenio Lopez Chua is the one entitled to the insurance proceeds and not Tai Tong Chuache & Company. This Court cannot fault petitioner for the above erroneous interpretation of the decision appealed from considering the manner it was written. 5 As correctly pointed out by respondent insurance commission in their comment, the decision did not pronounce that it was Arsenio Lopez Chua who has insurable interest over the insured property. Perusal of the decision reveals however that it readily absolved respondent insurance company from liability on the basis of the commissioner's conclusion that at the time of the occurrence of the peril insured against petitioner as mortgagee had no more insurable interest over the insured property. It was based on the inference that the credit secured by the mortgaged property was already paid by the Palomos before the said property was gutted down by fire. The foregoing conclusion was arrived at on the basis of the certification issued by the then Court of First Instance of Davao, Branch II that in a certain civil action against the Palomos, Antonio Lopez Chua stands as the complainant and not petitioner Tai Tong Chuache & Company. We find the petition to be impressed with merit. It is a well known postulate that the case of a party is constituted by his own affirmative allegations. Under Section 1, Rule 131 6 each party must prove his own affirmative allegations by the amount of evidence required by law which in civil cases as in the present case is preponderance of evidence. The party, whether plaintiff or defendant, who asserts the affirmative of the issue has the burden of presenting at the trial such amount of evidence as required by law to obtain favorable judgment. 7 Thus, petitioner who is claiming a right over the insurance must prove its case. Likewise, respondent insurance company to avoid liability under the policy by setting up an affirmative defense of lack of insurable interest on the part of the petitioner must prove its own affirmative allegations. It will be recalled that respondent insurance company did not assail the validity of the insurance policy taken out by petitioner over the mortgaged property. Neither did it deny that the said property was totally razed by fire within the period covered by the insurance. Respondent, as mentioned earlier advanced an affirmative defense of lack of insurable interest on the part of the petitioner that before the occurrence of the peril insured against the Palomos had already paid their credit due the petitioner.

Respondent having admitted the material allegations in the complaint, has the burden of proof to show that petitioner has no insurable interest over the insured property at the time the contingency took place. Upon that point, there is a failure of proof. Respondent, it will be noted, exerted no effort to present any evidence to substantiate its claim, while petitioner did. For said respondent's failure, the decision must be adverse to it. However, as adverted to earlier, respondent Insurance Commission absolved respondent insurance company from liability on the basis of the certification issued by the then Court of First Instance of Davao, Branch II, that in a certain civil action against the Palomos, Arsenio Lopez Chua stands as the complainant and not Tai Tong Chuache. From said evidence respondent commission inferred that the credit extended by herein petitioner to the Palomos secured by the insured property must have been paid. Such is a glaring error which this Court cannot sanction. Respondent Commission's findings are based upon a mere inference. The record of the case shows that the petitioner to support its claim for the insurance proceeds offered as evidence the contract of mortgage (Exh. 1) which has not been cancelled nor released. It has been held in a long line of cases that when the creditor is in possession of the document of credit, he need not prove non-payment for it is presumed. 8 The validity of the insurance policy taken b petitioner was not assailed by private respondent. Moreover, petitioner's claim that the loan extended to the Palomos has not yet been paid was corroborated by Azucena Palomo who testified that they are still indebted to herein petitioner. 9 Public respondent argues however, that if the civil case really stemmed from the loan granted to Azucena Palomo by petitioner the same should have been brought by Tai Tong Chuache or by its representative in its own behalf. From the above premise respondent concluded that the obligation secured by the insured property must have been paid. The premise is correct but the conclusion is wrong. Citing Rule 3, Sec. 2 10 respondent pointed out that the action must be brought in the name of the real party in interest. We agree. However, it should be borne in mind that petitioner being a partnership may sue and be sued in its name or by its duly authorized representative. The fact that Arsenio Lopez Chua is the representative of petitioner is not questioned. Petitioner's declaration that Arsenio Lopez Chua acts as the managing partner of the partnership was corroborated by respondent insurance company. 11 Thus Chua as the managing partner of the partnership may execute all acts of administration 12 including the right to sue debtors of the partnership in case of their failure to pay their obligations when it became due and demandable. Or at the very least, Chua being a partner of petitioner Tai Tong Chuache & Company is an agent of the partnership. Being an agent, it is understood that he acted for and in behalf of the firm. 13 Public respondent's allegation that the civil

case flied by Arsenio Chua was in his capacity as personal creditor of spouses Palomo has no basis. The respondent insurance company having issued a policy in favor of herein petitioner which policy was of legal force and effect at the time of the fire, it is bound by its terms and conditions. Upon its failure to prove the allegation of lack of insurable interest on the part of the petitioner, respondent insurance company is and must be held liable. IN VIEW OF THE FOREGOING, the decision appealed from is hereby SET ASIDE and ANOTHER judgment is rendered order private respondent Travellers Multi-Indemnity Corporation to pay petitioner the face value of Insurance Policy No. 599-DV in the amount of P100,000.00. Costs against said private respondent. SO ORDERED. Teehankee, C.J., Narvasa, Cruz and Grio-Aquino, JJ., concur.

G.R. No. L-5715 December 20, 1910 E. M. BACHRACH, plaintiff-appellee, vs. BRITISH AMERICAN ASSURANCE COMPANY, a corporation, defendant-appellant. Haussermann, Ortigas, Cohn and Fisher, for appellant Kincaid & Hurd and Thomas L. Hartigan, for appellee.

JOHNSON, J.: On the 13th of July, 1908, the plaintiff commenced an action against the defendant to recover the sum of P9,841.50, the amount due, deducting the salvage, upon the following fire insurance policy issued by the defendant to the plaintiff: [Fire policy No. 3007499.] This policy of insurance witnesseth, that E. M. Bachrach, esq., Manila (hereinafter called the insured), having paid to the undersigned, as authorized agent of the British American Assurance Company (hereinafter called the company), the sum of two thousand pesos Philippine currency, for insuring against loss or damage by fire, as hereinafter mentioned, the property hereinafter described, in the sum of several sums following, viz: Ten thousand pesos Philippine currency, on goods, belonging to a general furniture store, such as iron and brass bedsteads, toilet tables, chairs, ice boxes, bureaus, washstands, mirrors, and sea-grass furniture (in accordance with warranty "D" of the tariff attached hereto) the property of the assured, in trust, on commission or for which he is responsible, whilst stored in the ground floor and first story of house and dwelling No. 16 Calle Martinez, district 3, block 70, Manila, built, ground floor of stone and or brick, first story of hard wood and roofed with galvanized iron bounded in the front by the said calle, on one side by Calle David and on the other two sides by buildings of similar construction and occupation. Co-insurance allowed, particulars of which to be declared in the event of loss or claim. The company hereby agrees with the insured (but subject to the conditions on the back hereof, which are to be taken as a part of this policy) that if the property above described, or any part thereof, shall be destroyed or damaged by fire, at any time between the 21st day of February, 1908, and 4 o'clock in the afternoon of the 21st day of February, 1909, or (in case of the renewal of this policy) at any time afterwards, so long as, and during the period in respect of which the insured shall have paid to the

company, and they shall have accepted, the sum required for the renewal of this policy, the company will, out of their capital stock, and funds, pay or make good to the insured the value of the property so destroyed, or the amount of such damage thereto, to any amount not exceeding, in respect of each or any of the several matters above specified, the sum set opposite thereto, respectively, and not exceeding in the whole the sum of ten thousand pesos, and also not exceeding, in any case, the amount of the insurable interest therein of the insured at the time of the happening of such fire. In witness whereof, the British American Assurance Company has accused these presents to be signed this 21st day of February, in the year of our Lord 1908. For the company. W. F. STEVENSON & Co. LTD., "By..............................................., "Manager Agents." And indorsed on the back the following: The within policy and includes a "Calalac" automobile to the extent of (P1,250) twelve hundred and fifty pesos Philippine currency. Memo: Permission is hereby granted for the use of gasoline not to exceed 10 gallons for the above automobile, but only whilst contained in the reservoir of the car. It is further warranted that the car be neither filled nor emptied in the within-described building or this policy be null and void. Manila, 27th February, 1908. "W. F. STEVENSON & Co. LTD., "By......................................................., "Manager Agents." The defendant answered the complaint, admitting some of the facts alleged by the plaintiff and denying others. The defendant also alleged certain facts under which it claimed that it was released from all obligations whatever under said policy. These special facts are as follows: First. That the plaintiff maintained a paint and varnish shop in the said building where the goods which were insured were stored. Second. That the plaintiff transferred his interest in and to the property covered by the policy to H. W. Peabody & Co. to secure certain indebtedness due and owing to said

company, and also that the plaintiff had transferred his interest in certain of the goods covered by the said policy to one Macke, to secure certain obligations assumed by the said Macke for and on behalf of the insured. That the sanction of the said defendant had not been obtained by the plaintiff, as required by the said policy. Third. That the plaintiff, on the 18th of April, 1908, and immediately preceding the outbreak of the alleged fire, willfully placed a gasoline can containing 10 gallons of gasoline in the upper story of said building in close proximity to a portion of said goods, wares, and merchandise, which can was so placed by the plaintiff as to permit the gasoline to run on the floor of said second story, and after so placing said gasoline, he, the plaintiff, placed in close proximity to said escaping gasoline a lighted lamp containing alcohol, thereby greatly increasing the risk of fire. Fourth. That the plaintiff made no proof of the loss within the time required by condition five of said policy, nor did the insured file a statement with he municipal or any other judge or court of the goods alleged to have been in said building at the time of the alleged fire, nor of the goods saved, nor the loss suffered. The plaintiff, after denying nearly all of the facts set out in the special answer of the defendant, alleged: First. That he had been acquitted in a criminal action against him, after a trial duly and regularly had, upon a charge of arson, based upon the same alleged facts set out in the answer of the defendant. Second. That her had made no proof of the loss set up in his complaint for the reason that immediately after he had, on the 20th of April, 1908, given the defendant due notice in writing of said loss, the defendant, on the 21st of April, 1908, and thereafter on other occasions, had waived all right to require proof of said loss by denying all liability under the policy and by declaring said policy to be null and void. After hearing the evidence adduced during the trial of the cause, the lower court found that the defendant was liable to the plaintiff and rendered a judgment against the defendant for the sum of P9,841.50, with interest for a period of one year at 6 per cent, making a total of P10,431.99, with costs. From that decision the defendant appealed and made the following assignments of error: 1. The court erred in failing to hold that the use of the building, No. 16 Calle Martinez, as a paint and varnish shop annulled the policy of insurance. 2. The court erred in failing to hold the execution of the chattel mortgages without the knowledge and consent of the insurance company annulled the policy of insurance.

3. The court erred in holding that the keeping of gasoline and alcohol not in bottles in the building No. 16 Calle Martinez was not such a violation of the conditions of the policy as to render the same null and void. 4. The court erred in failing to find as a fact that E. M. Bachrach, the insured, willfully placed a gasoline can containing about 10 gallons of gasoline in the upper story of said building, No. 16 Calle Martinez, in close proximity to a portion of the goods, wares, and merchandise stored therein, and that said can was so placed by said Bachrach as to permit the gasoline to run on the floor of said second story. 5. The court erred in failing to find as a fact that E. M. Bachrach, after placing said gasoline can in close proximity to the goods, wares, and merchandise covered by the policy of insurance, the he (Bachrach) placed in close proximity to said escaping gasoline a lighted lamp containing alcohol, thereby greatly increasing the risk of fire. 6. The court erred in holding that the policy of insurance was in force at the time of said fire, and that the acts or omissions on the part of the insured which cause, or tended to cause, the forfeiture of the policy, were waived by the defendant. 7. The court erred in holding the defendant liable for the loss under the policy.lawphil.net 8. The court erred in refusing to deduct from the loss sustained by Bachrach the value of the automobile, which was saved without damage. 9. The court erred in refusing to grant the motion for a new trial. 10. The court erred in refusing to enter judgment in favor of the defendant and against the plaintiff. With reference to the first above assignment of error, the lower court in its decision said: It is claimed that either gasoline or alcohol was kept in violation of the policy in the bodega containing the insured property. The testimony on this point is somewhat conflicting, but conceding all of the defendant's claims, the construction given to this claim by American courts would not justify the forfeiture of the policy on that ground. The property insured consisted mainly of household furniture kept for the purpose of sale. The preservation of the furniture in a salable condition by retouching or otherwise was incidental to the business. The evidence offered by the plaintiff is to the effect that alcohol was used in preparing varnish for the purpose of retouching, though he also says that the alcohol was kept in store and not in the bodega where the furniture was. It is well settled that the keeping of inflammable oils on the premises, though prohibited by the policy, does not void it if such keeping is incidental to the business. Thus, where a furniture factory keeps benzine for the purposes of operation (Davis vs. Pioneer

Furniture Company, 78 N. W. Rep., 596; Faust vs. American Fire Insurance Company, 91 Wis., 158), or where it is used for the cleaning machinery (Mears vs. Humboldt Insurance Company, 92 Pa. St., 15; 37 Am. Rep., 647), the insurer can not on that ground avoid payment of loss, though the keeping of the benzine on the premises is expressly prohibited. These authorities also appear sufficient to answer the objection that the insured automobile contained gasoline and that the plaintiff on one occasion was seen in the bodega with a lighted lamp. The first was incidental to the use of the insured article and the second being a single instance falls within the doctrine of the case last cited. It may be added that there was no provision in the policy prohibiting the keeping of paints and varnishes upon the premises where the insured property was stored. If the company intended to rely upon a condition of that character, it ought to have been plainly expressed in the policy. With reference to the second above assignment of error, the defendant and appellant contends that the lower court erred in failing to hold that the execution of the said chattel mortgage, without the knowledge and consent of the insurance company and without receiving the sanction of said company, annulled the said policy of insurance. With reference to this assignment of error, upon reading the policy of insurance issued by the defendant to the plaintiff, it will be noted that there is no provision in said policy prohibiting the plaintiff from placing a mortgage upon the property insured, but, admitting that such a provision was intended, we think the lower court has completely answered this contention of the defendant. He said, in passing upon this question as it was presented: It is claimed that the execution of a chattel mortgage on the insured property violated what is known as the "alienation clause," which is now found in most policies, and which is expressed in the policies involved in cases 6496 and 6497 by a purchase imposing forfeiture if the interest in the property pass from the insured. (Cases 6496 and 6497, in which are involved other action against other insurance companies for the same loss as in the present action.) This clause has been the subject of a vast number of judicial decisions (13 Am. & Eng. Encyc. of Law, 2d ed., pp. 239 et seq.), and it is held by the great weight of authority that the interest in property insured does not pass by the mere execution of a chattel mortgage and that while a chattel mortgage is a conditional sale, there is no alienation within the meaning of the insurance law until the mortgage acquires a right to take possession by default under the terms of the mortgage. No such right is claimed to have accrued in the case at bar, and the alienation clause is therefore inapplicable.

With reference to the third assignment of error above noted, upon a reading of the decision of the lower court it will be found that there is nothing in the decision of the lower court relating to the facts stated in this assignment of error, neither is there any provision in the policy relating to the facts alleged in said assignment of error. Assignment of error numbers 4 and 5 above noted may be considered together. The record discloses that some time prior to the commencement of this present action, a criminal action was commenced against the plaintiff herein in the Court of First Instance of the city of Manila, in which he was charged with willfully and maliciously burning the property covered by the policy in the present case. At the conclusion of the criminal action and after hearing the evidence adduced during the trial, the lower court, with the assistance of two assessors, found that the evidence was insufficient to show beyond peradventure of doubt that the defendant was guilty of the crime. The evidence adduced during the trial of the criminal cause was introduced as evidence in the present cause. While the evidence shows some very peculiar and suspicious circumstances concerning the burning of the goods covered by the said policy, yet, nevertheless, in view of the findings of the lower court and in view of the apparent conflict in the testimony, we can not find that there is a preponderance of evidence showing that the plaintiff did actually set fire or cause fire to be set to the goods in question. The lower court, in discussing this question, said: As to the claim that the loss occurred through the voluntary act of the insured, we consider it unnecessary to review the evidence in detail. That was done by another branch of this court in disposing of the criminal prosecution brought against the insured, on the same ground, based mainly on the same evidence. And regardless of whether or not the judgment in that proceeding is res adjudicata as to anything here, we are at least of the opinion that the evidence to establish this defense should not be materially less convincing than that required in order to convict the insured of the crime of arson. (Turtell vs. Beamount, 25 Rev. Rep., 644.) In order to find that the defense of incendiarism was established here, we would be obliged, therefore, in effect to set aside the findings of the judge and assessors in the criminal cause, and this we would be loath to do even though the evidence now produced were much stronger than it is. With reference to the sixth assignment of error above noted, to wit:itc@alf That the court erred in holding that the policy of insurance was in force at the time of said fire and that the acts or omissions on the part of the insured which caused or tended to cause a forfeiture of the policy were waived by the defendant, the lower court, in discussing this question, said: Regardless of the question whether the plaintiff's letter of April 20 (Exhibit B) was a sufficient compliance with the requirement that he furnish notice of loss, the fact

remains that on the following day the insurers replied by a letter (Exhibit C) declaring that the "policies were null and void," and in effect denying liability. It is well settled by a preponderance of authorities that such a denial is a waiver of notice of loss, because if the "policies are null and void," the furnishing of such notice would be vain and useless. (13 Am. & Eng. Encyc. of Law, 347, 348, 349.) Besides, "immediate notice" is construed to mean only within a reasonable time. Much the same may be said as to the objection that the insured failed to furnish to the insurers his books and papers or to present a detailed statement to the "juez municipal," in accordance with article 404 of the Code of Commerce. The last-named provision is similar to one appearing in many American policies requiring a certificate from a magistrate nearest the loss regarding the circumstance thereof. A denial of liability on other grounds waives this requirement (O'Niel vs. Buffalo Fire Insurance Company, 3 N. Y., 122; Peoria Marine Ins. Co. vs. Whitehill, 25 Ill., 382), as well as that relating to the production of books and papers (Ga. Home Ins. Co. vs. Goode & Co., 95 Va., 751; 66 Jur. Civ., 16). Besides, the insured might have had difficulty in attempting to comply with this clause, for there is no longer an official here with the title of "juez municipal." Besides the foregoing reasons, it may be added that there was no requirement in the policy in question that such notice be given. With reference to the assignments of error numbers 7, 9, and 10, they are too general in their character to merit consideration. With reference to the eight assignment of error above noted, the defendant and appellant contends that he was entitled to have the amount of his responsibility reduced by the full value (P1,250) of the said automobile. It does not positively appear of record that the automobile in question was not included in the other policies. It does appear that the automobile was saved and was considered as a part of the salvaged. It is alleged that the salvage amounted to P4,000, including the automobile. This amount (P4,000) was distributed among the different insurers and the amount of their responsibility was proportionately reduced. The defendant and appellant in the present case made no objection at any time in the lower court to that distribution of the salvage. The claim is now made for the first time. No reason is given why the objection was not made at the time of the distribution of the salvage, including the automobile, among all of the insurers. The lower court had no opportunity to pass upon the question now presented for the first time. The defendant stood by and allowed the other insurers to share in the salvage, which he claims now wholly belonged to him. We think it is now too late to raise the question. For all the foregoing reasons, we are of the opinion that the judgment of the lower court should be affirmed, and it is hereby ordered that judgment be entered against the

defendant and in favor of the plaintiff for the sum of P9,841.50, with interest at the rate of 6 per cent from the 13th of July, 1908, with costs. So ordered. Arellano, C. J., and Torres, J., concur. Trent, J., concurs in the result. Moreland, J., dissents.

WEEK 5 VI. Concealment


March 19, 1928

G.R. No. L-24899

BERNARDO ARGENTE, plaintiff-appellant, vs. WEST COAST LIFE INSURANCE CO., defendant-appellee. Abad Santos, Camus, Delgado & Recto for appellant. Gibbs & McDonough and Roman Ozaeta for appellee. MALCOLM, J.: This is an action upon a joint life insurance policy for P15,000 issued by the defendant, the West Coast Life Insurance Co., on May 15, 1925, in favor of the plaintiff, Bernardo Argente, and his wife, Vicenta de Ocampo, the latter having died on November 18, 1925. Fraud in obtaining the policy was pleaded by way of special defense. On the issue thus suggested, the court adopted the theory of the defendant, and held the insurance policy null and void, with the result that the complaint was dismissed, with costs. On February 9, 1925, Bernardo Argente signed an application for joint insurance with his wife in the sum of P2,000. The wife, Vicenta de Ocampo, signed a like application for the same policy. Both applications, with the exception of the names and the signatures of the applicants, were written by Jose Geronimo del Rosario, an agent for the West Coast Life Insurance Co. But all the information contained in the applications was furnished the agent by Bernardo Argente. Pursuant to his application, Bernardo Argente was examined by Dr. Cesareo Sta. Ana, a medical examiner for the West Coast Life Insurance Co., on February 10, 1925, in the office of the Customs House. The result of such examination was recorded in the Medical Examiner's Report, and with the exception of the signature of Bernardo Argente, was in the hand-writing of Doctor Sta. Ana. But the information or answers to the questions contained on the face of the Medical Examiner's Report were furnished the doctor by the applicant, Bernardo Argente. Pursuant to her application, Vicenta de Ocampo, wife of the plaintiff, was examined by Dr. Cesareo Sta. Ana on February 10, 1925, at her residence in Manila. The result of the medical examination, including among other things, the answers given by Vicenta

de Ocampo to the questions propounded to her by the physician, appears in the Medical Examiner's Report. On May 9, 1925, Bernardo Argente and his wife submitted to the West Coast Life Insurance Co. an amended application for insurance, increasing the amount thereof to P15,000, and asked that the policy be dated May 15, 1925. The amended application was accompanied by the documents entitled "Short Form Medical Report." In both of these documents appear certain questions and answers. A temporary policy for P15,000 was issued to Bernardo Argente and his wife as of May 15, but it was not delivered to Bernardo Argente until July 2, 1925, when the first quarterly premium on the policy was paid. In view of the fact that more than thirty days had elapsed since the applicants were examined by the company's physician, each of them was required to file a certificate of health before the policy was delivered to them. On November 18, 1925, Vicenta de Ocampo died of cerebral apoplexy. Thereafter Bernardo Argente presented a claim in due form to the West Coast Life Insurance Co. for the payment of the sum of P15,000 the amount of the joint life Insurance policy. Following investigation conducted by the Manager of the Manila office of the insurance company, it was apparently disclosed that the answers given by the insured in their medical examinations with regard to their health and previous illness and medical attendance were untrue. For that reason, the West Coast Life Insurance Co. refused to pay the claim of Bernardo Argente, and on May 25, 1926, wrote him to the effect that the claim was rejected because the insurance was obtained through fraud and misrepresentation. It is admitted that it appears in the Medical Examiner's Report that Bernardo Argente, in response to the question asked by the medical examiner, "Have you ever consulted a physician for, or have you ever suffered from any ailment or disease of, the brain or nervous system?" answered "No." To the question, "Have you consulted a physician for any ailment or disease not included in your above answer," answered "Yes. Nature of Ailment, Disease or Injury. Scabies, Number of attacks 1, Date 1911. Duration 1 month, Severity Fair, results and, if within five years, name and address of every physician consulted. Dr. P. Guazon. Cured. Dr. Guazon is dead now." And to the question, "What physician or physicians, if any, not named above, have you consulted or been treated by, within the last five years and for what illness or ailment? (If none, so state)" answered "No." It is, however, not disputed that on January 10, 11, and 13, 1923, Bernardo Argente was confined in the Philippine General Hospital where he was treated by Dr. Agerico B. M. Sison for cerebral congestion and Bell's Palsy. It is further admitted that it appears in the Medical Examiner's Report that Vicenta de Ocampo, in response to the question asked by the medical examiner, "How frequently,

if at all, and in what quantity do you use beer, wine, spirits or other intoxicants?" answered "Beer only in small quantities occasionally." To the question, "Have you ever consulted a physician for or have you ever suffered from any ailment or disease of the brain or nervous system?" answered "No." To the question, "What physician or physicians, if any, not named above, have you consulted or been treated by, within the last five years and for what illness or ailment? (If none, so state)" answered "None." And to the question, "Are you in good health as far as you know and believe?" answered "Yes." It is, however, not disputed that Vicenta de Ocampo was taken by a patrolman, at the request of her husband, Bernardo Argente, on May 19, 1924, to the Meisic police station, and from there was transferred to the San Lazaro Hospital. In San Lazaro Hospital, her case was diagnosed by the admitting physician as "alcoholism," but later Doctor Domingo made a diagnosis of probable "manic-depressive psychosis," and still, later in Mary Chiles Hospital, made a final diagnosis of "phycho-neurosis." The plaintiff, Bernardo Argente, while readily conceding most of the facts herein narrated, yet alleges that both he and his wife revealed to the company's physician. Doctor Sta. Ana, all the facts concerning the previous illnesses and medical attendance, but that Doctor Sta. Ana, presumably acting in collusion, with the insurance agent, Jose Geronimo del Rosario, failed to record them in the medical reports. The evidence on these points consists of the testimony of the plaintiff and his subordinate clerk, Apolonio Espiritu, on the one hand, and of the testimony of Doctor Sta. Ana and Jose Geronimo del Rosario on the other. On the question of fact thus raised, the trial judge found with the insurance company. In so doing, we believe that His Honor gave proper inclination to the weight of the proof. There appears no motive whatever on the part of Doctor Sta. Ana to falsify the Medical Examiner's Reports and thereby not only jeopardize his career as a physician, but also gravely implicate himself criminally. What has heretofore been stated in this decision is gleaned to a great extent the carefully prepared decision of the trial judge, the Honorable George R. Harvey. The court found from the evidence that the representations made by Bernardo Argente and his wife in their applications to the defendant for life insurance were false with respect to their estate of health during the period of five years preceding the date of such applications, and that they knew the representations made by them in their applications were false. The court further found from the evidence that the answers given by Bernardo Argente and his wife at the time of the medical examination by Doctor Sta. Ana were false with respect to the condition of their health at that time and for a period of several years prior thereto. Based on these findings which must here be accepted since the stenographic transcript is incomplete, the question arises as to the estate of the law in relation thereto. One ground for the rescission of a contract of insurance under the Insurance Act is "a concealment," which in section 25 is defined as "A neglect to communicate that which a

party knows and ought to communicate." Appellant argues that the alleged concealment was immaterial and insufficient to avoid the policy. We cannot agree. In an action on a life insurance policy where the evidence conclusively shows that the answers to questions concerning diseases were untrue, the truth of falsity of the answers become the determining factor. In the policy was procured by fraudulent representations, the contract of insurance apparently set forth therein was never legally existent. It can fairly be assumed that had the true facts been disclosed by the assured, the insurance would never have been granted. In Joyce, The Law of Insurance, second edition, volume 3, Chapter LV, is found the following: Concealment exists where the assured has knowledge of a fact material to the risk, and honesty, good faith, and fair dealing requires that he should communicate it to the assured, but he designated and intentionally with holds the same. Another rule is that if the assured undertakes to state all the circumstances affecting the risk, a full and fair statement of all is required. It is also held that the concealment must, in the absence of inquiries, be not only material, but fraudulent, or the fact must have been intentionally withheld; so it is held under English law that if no inquiries are made and no fraud or design to conceal enters into the concealment the contract is not avoided. And it is determined that even though silence may constitute misrepresentation or concealment it is not itself necessarily so as it is a question of fact. Nor is there a concealment justifying a forfeiture where the fact of insanity is not disclosed no questions being asked concerning the same. . . . But it would seem that if a material fact is actually known to the assured, its concealment must of itself necessarily be a fraud, and if the fact is one which the assured ought to know, or is presumed to know, the presumption of knowledge ought to place the assured in the same position as in the former case with relation to material facts; and if the jury in such cases find the fact material, and one tending to increase the risk, it is difficult to see how the inference of a fraudulent intent or intentional concealment can be avoided. And it is declared that if a material fact concealed by assured it is equivalent to a false representation that it does not exist and that the essentials are the truth of the representations whether they were intended to mislead and did insurer accept them as true and act upon them to his prejudice. So it is decided that under a stipulation voiding the policy for concealment or misrepresentation of any material fact or if his interest is not truly stated or is either than the sole and unconditional ownership the facts are unimportant that insured did not intend to deceive or withhold information as to encumbrances even though no questions were asked. And if insured while being examined for life insurance and knowing that she had heart

disease, falsely stated that she was in good health, and though she could not read the application, it was explained to her and the questions asked through an interpreter, and the application like the policy contained and provision that no liability should be incurred unless the policy was delivered while the insured was in good health, the court properly directed a verdict for the insurer, though a witness who was present at the examination testified that the insured was not asked whether she had heart disease. xxx xxx xxx

The basis of the rule vitiating the contract in case of concealment is that it misleads or deceives the insurer into accepting the risk, or accepting it at the rate of premium agreed upon. The insurer, relying upon the belief that the assured will disclose every material within his actual or presumed knowledge, is misled into a belief that the circumstance withheld does not exist, and he is thereby induced to estimate the risk upon a false basis that it does not exist. The principal question, therefore, must be, Was the assurer misled or deceived into entering a contract obligation or in fixing the premium of insurance by a withholding of material information of facts within the assured's knowledge or presumed knowledge? It therefore follows that the assurer in assuming a risk is entitled to know every material fact of which the assured has exclusive or peculiar knowledge, as well as all material facts which directly tend to increase the hazard or risk which are known by the assured, or which ought to be or are presumed to be known by him. And a concealment of such facts vitiates the policy. "It does not seem to be necessary . . . that the . . . suppression of the truth should have been willful." If it were but an inadvertent omission, yet if it were material to the risk and such as the plaintiff should have known to be so, it would render the policy void. But it is held that if untrue or false answers are given in response to inquiries and they relate to material facts the policy is avoided without regard to the knowledge or fraud of assured, although under the statute statements are representations which must be fraudulent to avoid the policy. So under certain codes the important inquiries are whether the concealment was willful and related to a matter material to the risk. xxx xxx xxx

If the assured has exclusive knowledge of material facts, he should fully and fairly disclose the same, whether he believes them material or not. But notwithstanding this general rule it will not infrequently happen, especially in life risks, that the assured may have a knowledge actual or presumed of material facts, and yet entertain an honest belief that they are not material. . . . The determination of the point whether there has or has not been a material concealment must rest largely in all cases upon the form of the questions propounded and the exact terms of the contract. Thus, where in addition to

specifically named diseases the insured was asked whether he had any sickness within ten years, to which he answered "No," and it was proven that within that period he had a slight of pharyngitis, it was held a question properly for the jury whether such an inflammation of the throat was a "sickness" within the intent of the inquiry, and the court remarked on the appeal decision that if it could be held as a matter of law that the policy was thereby avoided, then it was a mere device on the part of insurance companies to obtain money without rendering themselves liable under the policy. . . . . . . The question should be left to the jury whether the assured truly represented the state of his health so as not mislead or deceive the insurer; and if he did not deal a good faith with insurer in that matter, that the inquiry should be made, Did he know the state of his health so as to be able to furnish a proper answer to such questions as are propounded? A Massachusetts case, if construed as it is frequently cited, would be opposed to the above conclusion; but, on the contrary, it sustains it, for the reason that symptoms of consumption had so far developed themselves within a few months prior to effecting the insurance as to induce a reasonable belief that the applicant had that fatal disease, and we should further construe this case as establishing the rule that such a matter cannot rest alone upon the assured's belief irrespective of what is a reasonable belief, but that it ought to be judged by the criterion whether the belief is one fairly warranted by the circumstances. A case in Indiana, however, holds that if the assured has some affection or ailment of one or more of the organs inquired about so welldefined and marked as to materially derange for a time the functions of such organ, as in the case of Bright's disease, the policy will be avoided by a nondisclosure, irrespective of the fact whether the assured knew of such ailment or not. . . . Lastly, appellant contends that even if the insurance company had a right to rescind the contract, such right cannot now be enforced in view of the provisions of section 47 of the Insurance Act providing "Whenever a right to rescind a contract of insurance is given to their insurer by provision of this chapter, such right must be exercised previous to the commencement of an action on the contract." This section was derived from section 2583 of the California Civil Code, but in contrast thereto, makes use of the imperative "must" instead of the permissive "may." Nevertheless, there are two answers to the problem as propounded. The first is that the California law as construed by the code examiners, at whose recommendation it was adopted, conceded that "A failure to exercise the right (of rescission), cannot, of course, prejudice any defense to the action which the concealment may furnish." (Codes of California annotated; Tan Chay Heng vs. West Coast Life Insurance Company [1927], p. 80, ante.) The second answer is that the insurance company more than one month previous to the commencement of the present action wrote the plaintiff and informed him that the insurance contract was void because it had been procured through fraudulent representations, and offered to refund to the plaintiff the premium which the latter had paid upon the return of the policy for

cancellation. As held in California as to a fire insurance policy, where any of the material representations are false, the insurer's tender of the premium and notice that the policy is canceled, before the commencement of suit thereon, operate to rescind the contract of insurance. (Rankin vs. Amazon Insurance Co. [1891], 89 Cal., 203.) We are content to rest our judgment on the findings of the trial court, and on the law governing those facts, with the result that the various assignments of error are found to be without persuasive merit. Judgment affirmed, with the costs of this instance against the appellant. Villamor, Ostrand, Johns, Romualdez and Villa-Real, JJ., concur.

May 29, 1959 G.R. No. L-12465 YU PANG CHENG alias YU PANG CHING, petitioner, vs. THE COURT OF APPEALS, ET AL., respondents. M. de la Rosa and Yuseco, Abdon, Yuseco and Narvasa for petitioner. Perkin and Ponce Enrile for respondents. , J.: Plaintiff brought this action to collect from defendant the sum of P10,000.00, value of an insurance policy taken upon the life of one Yu Pang Eng, plus interest thereon at the legal rate, the sum of P10,000.00 as moral damages the further sum of P3,000.00 as attorneys fees, and the costs of action. Defendant, in its answer, set up the defense that the insured was guilty of misrepresentation and concealment of material facts in that he gave false and untruthful answers to certain questions asked him in his application for insurance which were material to the risk insured against and have the effect of avoiding the insurance policy. After trial, the court rendered judgment ordering defendant to pay plaintiff the sum of P10,000.00, with legal interest thereon from the filing of the complaint, plus the sum of P2,000.00 as attorneys fees, and the costs of suit. On appeal, the Court of Appeals reversed the decision of the trial court, holding that the insured was guilty of concealment of material facts which relieves defendant from liability. Hence the present petition for review. On September 5, 1950, Yu Pang Eng submitted parts II and III of his application for insurance consisting of the medical declaration made by him to the medical examiner of defendant and the medical examiners report. On September 7, he submitted part I of his application which is the declaration made by him to an agent of defendant, and on September 8, based on said application, and upon payment of the first premium in the sum of P591.70, defendant issued to the insured Policy No. 812858. On December 27, 1950, the insured entered St. Lukes Hospital for medical treatment but he died on February 27, 1951. According to the death certificate, he died of infiltrating medullary carcinoma, Grade 4, advanced cardiac and of lesser curvature, stomach metastases spleen. Plaintiff, brother and beneficiary of the insured, demanded from the defendant the payment of the proceeds of the insurance policy and when the demand was refused, he brought the present action.

The issue to be determined is whether the insured is guilty of concealment of some facts material to the risk insured against which has the effect of avoiding the policy as found by respondent court. The insured, in his application for insurance, particularly in his declarations to the examining physician, stated the following in answering the questions propounded to him: 14. Have you ever had any of the following diseases or symptoms? Each question must be read and answered Yes or No. xxxxxxxxx Gastritis, Ulcer of the Stomach or any disease of that organ? No. Vertigo, Dizziness, Fainting-spells or Unconscious? No. Cancer, Tumors or Ulcers of any kind? No. 15. Have you ever consulted any physician not included in any of the above answers? Give names and address or physicians list ailments or accidents and date. No. It appears that the insured entered the Chinese General Hospital for medical treatment on January 29, 1950 having stayed there up to February 11, 1950. Upon entering the hospital, he complained of dizziness, anemia, abdominal pains and tarry stools, and in the evening of his admission he had several abdominal pains and his discharges were with black tarry stools and felt dizzy and weak. The history of his illness shows that the same started a year ago as frequent dizziness. An X-Ray picture of his stomach was taken and the diagnosis made of him by his doctors showed that his illness was peptic ulcer, bleeding. It should be noted that the insureds confinement in the Chinese General Hospital took place from January 29, 1950 to February 11, 1950, whereas his application for insurance wherein he stated his answer to the questions propounded to him by the examining physician of defendant was submitted to defendant on September 5, 1950. It is apparent that when the insured gave his answers regarding his previous ailment, particularly with regard to Gastritis, Ulcer of the Stomach or any disease of that organ and Vertigo, Dizziness, Fainting-spells or Unconsciousness, he concealed the ailment of which he was treated in the Chinese General, Hospital which precisely has direct connection with the subject of the questions propounded. The negative answers given by the insured regarding his previous ailment, or his concealment of the fact that he was hospitalized and treated for sometime of peptic ulcer and had suffered from dizziness, anemia, abdominal pains and tarry stools, deprived defendant of the opportunity to make the necessary inquiry as to the nature of his past illness so that as it may form its

estimate relative to the approval of his application. Had defendant been given such opportunity, considering the previous illness of the insured as disclosed by the record of the Chinese General Hospital, defendant would probably had never consented to the issuance of the policy in question. In fact, according to the death certificate, the insured died of infiltrating medullary carcinoma, Grade 4, advanced cardiac and of lesser curvature, stomach metastases spleen, which may have direct connection with his previous illness. Our Insurance law provides that A neglect to communicate that which a party knows and ought to communicate, is called concealment (Section 25, Act No. 2427). Whether intentional or unintentional, the concealment entitles the insurer to rescind the contract of insurance (Section 26). Our law even requires the insured to communicate to the insurer all facts within his knowledge which are material to the contract and which the other party has not the means of ascertaining (Section 27), and the materiality is to be determined not by the event but solely by the probable and reasonable influence of the facts upon the party to whom the communication is due (Section 30). In the case of Argente vs. West Coast Life Insurance Co., 51 Phil. 725 this Court said: One ground for the rescission of a contract of insurance under the insurance Act is a concealment, which in section 25 is defined A neglect to communicate that which a party knows and ought to communicate. Appellant argues that the concealment was immaterial and insufficient to avoid the policy. We cannot agree. In an action on a life insurance policy where the evidence conclusively shows that the answers to questions concerning diseases were untrue, the truth or falsity of the answers become the determining factor. If the policy was procured by fraudulent representations, the contract of insurance apparently set forth therein was never legally existent. It can fairly be assumed that had the true facts been disclosed by the assured, the insurance would never have been granted. Upon the foregoing reasons, we are persuaded to conclude that respondent court did err in declaring the policy ineffective on the ground of concealment and in relieving appellee from liability thereunder. Wherefore, the decision appealed from is affirmed, with costs against petitionerappellant. Paras, C.J., Bengzon, Padilla, Montemayor, Reyes, A., Labrador, Concepcion and Endencia, JJ., concur.

G.R. No. L-16163

February 28, 1963

IGNACIO SATURNINO, in his own behalf and as the JUDICIAL GUARDIAN OF CARLOS SATURNINO, minor, plaintiffs-appellants, vs. THE PHILIPPINE AMERICAN LIFE INSURANCE COMPANY, defendant-appellee. Eleazaro A. Samson for plaintiffs-appellants. Abello & Macias for defendant-appellee. MAKALINTAL, J.: Plaintiffs, now appellants, filed this action in the Court of First Instance of Manila to recover the sum of P5,000.00, corresponding to the face value of an insurance policy issued by defendant on the life of Estefania A. Saturnino, and the sum of P1,500.00 as attorney's fees. Defendant, now appellee, set up special defenses in its answer, with a counterclaim for damages allegedly sustained as a result of the unwarranted presentation of this case. Both the complaint and the counterclaim were dismissed by the trial court; but appellants were declared entitled to the return of the premium already paid; plus interest at 6% up to January 8, 1959, when a check for the corresponding amount P359.65 was sent to them by appellee. The policy sued upon is one for 20-year endowment non-medical insurance. This kind of policy dispenses with the medical examination of the applicant usually required in ordinary life policies. However, detailed information is called for in the application concerning the applicant's health and medical history. The written application in this case was submitted by Saturnino to appellee on November 16, 1957, witnessed by appellee's agent Edward A. Santos. The policy was issued on the same day, upon payment of the first year's premium of P339.25. On September 19, 1958 Saturnino died of pneumonia, secondary to influenza. Appellants here, who are her surviving husband and minor child, respectively, demanded payment of the face value of the policy. The claim was rejected and this suit was subsequently instituted. It appears that two months prior to the issuance of the policy or on September 9, 1957, Saturnino was operated on for cancer, involving complete removal of the right breast, including the pectoral muscles and the glands found in the right armpit. She stayed in the hospital for a period of eight days, after which she was discharged, although according to the surgeon who operated on her she could not be considered definitely cured, her ailment being of the malignant type. Notwithstanding the fact of her operation Estefania A. Saturnino did not make a disclosure thereof in her application for insurance. On the contrary, she stated therein that she did not have, nor had she ever had, among other ailments listed in the

application, cancer or other tumors; that she had not consulted any physician, undergone any operation or suffered any injury within the preceding five years; and that she had never been treated for nor did she ever have any illness or disease peculiar to her sex, particularly of the breast, ovaries, uterus, and menstrual disorders. The application also recites that the foregoing declarations constituted "a further basis for the issuance of the policy." The question at issue is whether or not the insured made such false representations of material facts as to avoid the policy. There can be no dispute that the information given by her in her application for insurance was false, namely, that she had never had cancer or tumors, or consulted any physician or undergone any operation within the preceding period of five years. Are the facts then falsely represented material? The Insurance Law (Section 30) provides that "materiality is to be determined not by the event, but solely by the probable and reasonable influence of the facts upon the party to whom the communication is due, in forming his estimate of the proposed contract, or in making his inquiries." It seems to be the contention of appellants that the facts subject of the representation were not material in view of the "non-medical" nature of the insurance applied for, which does away with the usual requirement of medical examination before the policy is issued. The contention is without merit. If anything, the waiver of medical examination renders even more material the information required of the applicant concerning previous condition of health and diseases suffered, for such information necessarily constitutes an important factor which the insurer takes into consideration in deciding whether to issue the policy or not. It is logical to assume that if appellee had been properly apprised of the insured's medical history she would at least have been made to undergo medical examination in order to determine her insurability. Appellants argue that due information concerning the insured's previous illness and operation had been given to appellees agent Edward A. Santos, who filled the application form after it was signed in blank by Estefania A. Saturnino. This was denied by Santos in his testimony, and the trial court found such testimony to be true. This is a finding of fact which is binding upon us, this appeal having been taken upon questions of law alone. We do not deem it necessary, therefore, to consider appellee's additional argument, which was upheld by the trial court, that in signing the application form in blank and leaving it to Edward A. Santos to fill (assuming that to be the truth) the insured in effect made Santos her agent for that purpose and consequently was responsible for the errors in the entries made by him in that capacity. In the application for insurance signed by the insured in this case, she agreed to submit to a medical examination by a duly appointed examiner of appellee if in the latter's opinion such examination was necessary as further evidence of insurability. In not asking her to submit to a medical examination, appellants maintain, appellee was guilty of negligence, which precluded it from finding about her actual state of health. No such

negligence can be imputed to appellee. It was precisely because the insured had given herself a clean bill of health that appellee no longer considered an actual medical checkup necessary. Appellants also contend there was no fraudulent concealment of the truth inasmuch as the insured herself did not know, since her doctor never told her, that the disease for which she had been operated on was cancer. In the first place the concealment of the fact of the operation itself was fraudulent, as there could not have been any mistake about it, no matter what the ailment. Secondly, in order to avoid a policy it is not necessary to show actual fraud on the part of the insured. In the case of Kasprzyk v. Metropolitan Insurance Co., 140 N.Y.S. 211, 214, it was held: Moreover, if it were the law that an insurance company could not depend a policy on the ground of misrepresentation, unless it could show actual knowledge on the part of the applicant that the statements were false, then it is plain that it would be impossible for it to protect itself and its honest policyholders against fraudulent and improper claims. It would be wholly at the mercy of any one who wished to apply for insurance, as it would be impossible to show actual fraud except in the extremest cases. It could not rely on an application as containing information on which it could act. There would be no incentive to an applicant to tell the truth. Wherefore, the parties respectfully pray that the foregoing stipulation of facts be admitted and approved by this Honorable Court, without prejudice to the parties adducing other evidence to prove their case not covered by this stipulation of facts. 1wph1.t In this jurisdiction a concealment, whether intentional or unintentional, entitles the insurer to rescind the contract of insurance, concealment being defined as "negligence to communicate that which a party knows and ought to communicate" (Sections 24 & 26, Act No. 2427). In the case of Argente v. West Coast Life Insurance Co., 51 Phil. 725, 732, this Court said, quoting from Joyce, The Law of Insurance, 2nd ed., Vol. 3: "The basis of the rule vitiating the contract in cases of concealment is that it misleads or deceives the insurer into accepting the risk, or accepting it at the rate of premium agreed upon. The insurer, relying upon the belief that the assured will disclose every material fact within his actual or presumed knowledge, is misled into a belief that the circumstance withheld does not exist, and he is thereby induced to estimate the risk upon a false basis that it does not exist." The judgment appealed from, dismissing the complaint and awarding the return to appellants of the premium already paid, with interest at 6% up to January 29, 1959, affirmed, with costs against appellants.

Bengzon, C.J., Padilla, Bautista Angelo, Labrador, Concepcion, Reyes, J.B.L., Barrera, Paredes, Dizon and Regala, JJ., concur.

G.R. No. L-14373

January 30, 1960

GENERAL INSURANCE AND SURETY CORPORATION, petitioner, vs. NG HUA, respondent. Jose P. Bengzon, Guido Advincula and Potenciano Villegas, Jr., petitioner. Crispin D. Baizas for respondent. BENGZON, J.: Suit to recover on a fire insurance policy. The insurer presented several defenses in the Manila court of first instance. After trial, it was required to pay. On appeal to the Courts of Appeal, the judgment was affirmed. This is now a revision on certiorari, upon the insurer's insistence on two of its main defenses: prescription and breach of warranty. The principal of facts on which adjudication may rest are these: On April 15, 1952, the defendant General Insurance and Surety Corporation issued its insurance Policy No. 471, insuring against fire, for one year, the stock in trade of the Central Pomade Factory owned by Ng Hua, the court insured. The next day, the Pomade factory building burned, resulting in destruction by fire of the insured properties. Ng Hua claimed indemnity from the insurer. The policy covered damages up to P10,000.00; but after some negotiations and upon suggestion of the Manila Adjustment Company, he reduced the claim of P5,000.00. Nevertheless, the defendant insurer refused to pay for various reasons, namely (a) action was not filed in time; (b) violation of warranty; (c) submission of fraudulent claim; and (f) failure to pay the premium. The aforesaid Policy No. 471 contains this stipulation on the back thereof;. 3. The insured shall give notice to the company of any insurance or insurances already affected, or which may subsequently be effected, covering any of the property hereby insured, and unless such notice be given and the particulars of such insurance or insurances be stated in or endorsed on this Policy by or on behalf of the Company before the occurrence of any loss or damage, all benefits under the policy shall be forfeited. (Emphasis ours.) The face of the policy bore the annotation: "Co-Insurance Declared NIL"

It is undenied that Ng Hua had obtained fire insurance on the same goods, for the same period of time, in the amount of P20,000.00 from General Indemnity Co. However, the Court of Appeals referring to the annotation and overruling the defense, held that there was no violation of the above clause, inasmuch as "co-insurance exists when a condition of the policy requires the insured to bear ratable proportion of the loss when the value of the insured property exceeds the face value of the policy," hence there is no co-insurance here. Discussion Undoubtedly, co-insurance exists under the condition described by the appellate court. But that is one kind of co-insurance. It is not the only situation where coinsurance exists. Other insurers of the same property against the same hazard are sometimes referred as co-insurers and the ensuing combination as co-insurance.1 And considering the terms of the policy which required the insured to declare other insurances, the statement in question must be deemed to be a statement (warranty) binding on both insurer and insured, that there were no other insurance on the property. Remember it runs "Co-Insurance declared"; emphasis on the last word. If "CoInsurance" means that the Court of Appeals says, the annotation served no purpose. It would even be contrary to the policy itself, which in its clause No. 17 made the insured a co-insurer for the excess of the value of the property over the amount of the policy. The annotation then, must be deemed to be a warranty that the property was not insured by any other policy. Violation thereof entitles the insurer to rescind. (Sec. 69. Insurance Act) Such misrepresentation is fatal in the light of our views in Santa Ana vs. Commercial Union Assurance Company, Ltd., 55 Phil., 329. The materiality of nondisclosure of other insurance policies is not open to doubt. Furthermore, even if the annotations were overlooked, the defendant insurer would still be free from liability because there is no question that the policy issued by General Indemnity had not been stated in nor endorsed on Policy No. 471 of defendant. And as stipulated in the above-quoted provisions of such policy "all benefit under this policy shall be forfeited."2 To avoid the dissastrous effect of the misrepresentation or concealment of the other insurance policy, Ng Hua alleges "actual knowledge" on the part of General insurance of the fact that he had taken out additional insurance with General Indemnity. He does not say when such knowledge was acquired or imparted. If General Insurance know before issuing its policy or before the fire, such knowledge might overcome the insurer's defense.3 However, the Court of Appeals found no evidence of such knowledge. We have read the pages of the stenographic notes cited by Ng Hua and we all gather is evidence of the existence of the Insurance General Indemnity Company. As to knowledge of General Insurance before issuance of its policy or the fire, there was none.

Indeed, this concealment and violation was expressly set up as a special defense in the answer. Yet plaintiff did not, in avoidance, reply nor assert such knowledge. And it is doubtful whether the evidence on the point would be admissible under the pleadings. (See Rule 11, sec. 1.) All the above considerations lead to the conclusion that the defendant insurer successfully established its defense of warranty breach or concealment of the other insurance and/or violation of the provision of the policy above-mentioned. Having reached the conclusion, we deem it unnecessary to discuss the other defenses. Wherefore, the judgment under review will be revoked, and the defendant insurer (herein petitioner) acquitted from all the liability under the policy. Costs against respondent. So ordered. Paras, C.J., Padilla, Montemayor, Bautista Angelo, Labrador, Concepcion Reyes, J.B.L., Endencia, and Barrera, JJ., concur.

G.R. No. 92492 June 17, 1993 THELMA VDA. DE CANILANG, petitioner, vs. HON. COURT OF APPEALS and GREAT PACIFIC LIFE ASSURANCE CORPORATION, respondents. Simeon C. Sato for petitioner. FELICIANO, J.: On 18 June 1982, Jaime Canilang consulted Dr. Wilfredo B. Claudio and was diagnosed as suffering from "sinus tachycardia." The doctor prescribed the following fro him: Trazepam, a tranquilizer; and Aptin, a beta-blocker drug. Mr. Canilang consulted the same doctor again on 3 August 1982 and this time was found to have "acute bronchitis." On next day, 4 August 1982, Jaime Canilang applied for a "non-medical" insurance policy with respondent Great Pacific Life Assurance Company ("Great Pacific") naming his wife, Thelma Canilang, as his beneficiary. 1 Jaime Canilang was issued ordinary life insurance Policy No. 345163, with the face value of P19,700, effective as of 9 August 1982. On 5 August 1983, Jaime Canilang died of "congestive heart failure," "anemia," and "chronic anemia." 2 Petitioner, widow and beneficiary of the insured, filed a claim with Great Pacific which the insurer denied on 5 December 1983 upon the ground that the insured had concealed material information from it. Petitioner then filed a complaint against Great Pacific with the Insurance Commission for recovery of the insurance proceeds. During the hearing called by the Insurance Commissioner, petitioner testified that she was not aware of any serious illness suffered by her late husband 3 and that, as far as she knew, her husband had died because of a kidney disorder. 4 A deposition given by Dr. Wilfredo Claudio was presented by petitioner. There Dr. Claudio stated that he was the family physician of the deceased Jaime Canilang 5 and that he had previously treated him for "sinus tachycardia" and "acute bronchitis." 6 Great Pacific for its part presented Dr. Esperanza Quismorio, a physician and a medical underwriter working for Great Pacific. 7 She testified that the deceased's insurance application had been approved on the basis of his medical declaration. 8 She explained that as a rule, medical examinations are required only in cases where the applicant has indicated in his application for insurance coverage that he has previously undergone medical consultation and hospitalization. 9

In a decision dated 5 November 1985, Insurance Commissioner Armando Ansaldo ordered Great Pacific to pay P19,700 plus legal interest and P2,000.00 as attorney's fees after holding that: 1. the ailment of Jaime Canilang was not so serious that, even if it had been disclosed, it would not have affected Great Pacific's decision to insure him; 2. Great Pacific had waived its right to inquire into the health condition of the applicant by the issuance of the policy despite the lack of answers to "some of the pertinent questions" in the insurance application; 3. there was no intentional concealment on the part of the insured Jaime Canilang as he had thought that he was merely suffering from a minor ailment and simple cold; 10 and 4. Batas Pambansa Blg. 847 which voids an insurance contract, whether or not concealment was intentionally made, was not applicable to Canilang's case as that law became effective only on 1 June 1985. On appeal by Great Pacific, the Court of Appeals reversed and set aside the decision of the Insurance Commissioner and dismissed Thelma Canilang's complaint and Great Pacific's counterclaim. The Court of Appealed found that the use of the word "intentionally" by the Insurance Commissioner in defining and resolving the issue agreed upon by the parties at pre-trial before the Insurance Commissioner was not supported by the evidence; that the issue agreed upon by the parties had been whether the deceased insured, Jaime Canilang, made a material concealment as the state of his health at the time of the filing of insurance application, justifying respondent's denial of the claim. The Court of Appeals also found that the failure of Jaime Canilang to disclose previous medical consultation and treatment constituted material information which should have been communicated to Great Pacific to enable the latter to make proper inquiries. The Court of Appeals finally held that the Ng Gan Zee case which had involved misrepresentation was not applicable in respect of the case at bar which involves concealment. Petitioner Thelma Canilang is now before this Court on a Petition for Review on Certiorari alleging that: 1. . . . the Honorable Court of Appeals, speaking with due respect, erred in not holding that the issue in the case agreed upon between the parties before the Insurance Commission is whether or not Jaime Canilang "intentionally" made material concealment in stating his state of health;

2. . . . at any rate, the non-disclosure of certain facts about his previous health conditions does not amount to fraud and private respondent is deemed to have waived inquiry thereto. 11 The medical declaration which was set out in the application for insurance executed by Jaime Canilang read as follows: MEDICAL DECLARATION I hereby declare that: (1) I have not been confined in any hospital, sanitarium or infirmary, nor receive any medical or surgical advice/attention within the last five (5) years. (2) I have never been treated nor consulted a physician for a heart condition, high blood pressure, cancer, diabetes, lung, kidney, stomach disorder, or any other physical impairment. (3) I am, to the best of my knowledge, in good health. EXCEPTIONS: ______________________________________________________________________ __________ GENERAL DECLARATION I hereby declare that all the foregoing answers and statements are complete, true and correct. I hereby agree that if there be any fraud or misrepresentation in the above statements material to the risk, the INSURANCE COMPANY upon discovery within two (2) years from the effective date of insurance shall have the right to declare such insurance null and void. That the liabilities of the Company under the said Policy/TA/Certificate shall accrue and begin only from the date of commencement of risk stated in the Policy/TA/Certificate, provided that the first premium is paid and the Policy/TA/Certificate is delivered to, and accepted by me in person, when I am in actual good health. Signed at Manila his 4th day of August, 1992. Illegible Signature of Applicant. 12 We note that in addition to the negative statements made by Mr. Canilang in paragraph 1 and 2 of the medical declaration, he failed to disclose in the appropriate space, under

the caption "Exceptions," that he had twice consulted Dr. Wilfredo B. Claudio who had found him to be suffering from "sinus tachycardia" and "acute bronchitis." The relevant statutory provisions as they stood at the time Great Pacific issued the contract of insurance and at the time Jaime Canilang died, are set out in P.D. No. 1460, also known as the Insurance Code of 1978, which went into effect on 11 June 1978. These provisions read as follows: Sec. 26. A neglect to communicate that which a party knows and ought to communicate, is called a concealment. xxx xxx xxx Sec. 28. Each party to a contract of insurance must communicate to the other, in good faith, all factors within his knowledge which are material to the contract and as to which he makes no warranty, and which the other has not the means of ascertaining. (Emphasis supplied) Under the foregoing provisions, the information concealed must be information which the concealing party knew and "ought to [have] communicate[d]," that is to say, information which was "material to the contract." The test of materiality is contained in Section 31 of the Insurance Code of 1978 which reads: Sec. 31. Materially is to be determined not by the event, but solely by the probable and reasonable influence of the facts upon the party to whom the communication is due, in forming his estimate of the disadvantages of the proposed contract, or in making his inquiries. (Emphasis supplied) "Sinus tachycardia" is considered present "when the heart rate exceeds 100 beats per minute." 13 The symptoms of this condition include pounding in the chest and sometimes faintness and weakness of the person affected. The following elaboration was offered by Great Pacific and set out by the Court of Appeals in its Decision: Sinus tachycardia is defined as sinus-initiated; heart rate faster than 100 beats per minute. (Harrison' s Principles of Internal Medicine, 8th ed. [1978], p. 1193.) It is, among others, a common reaction to heart disease, including myocardial infarction, and heart failure per se. (Henry J.L. Marriot, M.D., Electrocardiography, 6th ed., [1977], p. 127.) The medication prescribed by Dr. Claudio for treatment of Canilang's ailment on June 18, 1982, indicates the condition that said physician was trying to manage. Thus, he prescribed Trazepam, (Philippine Index of Medical Specialties (PIMS), Vol. 14, No. 3, Dec. 1985, p. 112) which is anti-anxiety, anti-convulsant, muscle-relaxant; and Aptin, (Idem, p. 36) a cardiac drug, for palpitations and nervous heart. Such treatment could

have been a very material information to the insurer in determining the action to be take on Canilang's application for life insurance coverage. 14 We agree with the Court of Appeals that the information which Jaime Canilang failed to disclose was material to the ability of Great Pacific to estimate the probable risk he presented as a subject of life insurance. Had Canilang disclosed his visits to his doctor, the diagnosis made and medicines prescribed by such doctor, in the insurance application, it may be reasonably assumed that Great Pacific would have made further inquiries and would have probably refused to issue a non-medical insurance policy or, at the very least, required a higher premium for the same coverage. 15 The materiality of the information withheld by Great Pacific did not depend upon the state of mind of Jaime Canilang. A man's state of mind or subjective belief is not capable of proof in our judicial process, except through proof of external acts or failure to act from which inferences as to his subjective belief may be reasonably drawn. Neither does materiality depend upon the actual or physical events which ensue. Materiality relates rather to the "probable and reasonable influence of the facts" upon the party to whom the communication should have been made, in assessing the risk involved in making or omitting to make further inquiries and in accepting the application for insurance; that "probable and reasonable influence of the facts" concealed must, of course, be determined objectively, by the judge ultimately. The insurance Great Pacific applied for was a "non-medical" insurance policy. In Saturnino v. Philippine-American Life Insurance Company, 16 this Court held that: . . . if anything, the waiver of medical examination [in a non-medical insurance contract] renders even more material the information required of the applicant concerning previous condition of health and diseases suffered, for such information necessarily constitutes an important factor which the insurer takes into consideration in deciding whether to issue the policy or not . . . . 17 (Emphasis supplied) The Insurance Commissioner had also ruled that the failure of Great Pacific to convey certain information to the insurer was not "intentional" in nature, for the reason that Jaime Canilang believed that he was suffering from minor ailment like a common cold. Section 27 of the Insurance Code of 1978 as it existed from 1974 up to 1985, that is, throughout the time range material for present purposes, provided that: Sec. 27. A concealment entitles the injured party to rescind a contract of insurance. The preceding statute, Act No. 2427, as it stood from 1914 up to 1974, had provided: Sec. 26. A concealment, whether intentional or unintentional, entitles the injured party to rescind a contract of insurance. (Emphasis supplied)

Upon the other hand, in 1985, the Insurance Code of 1978 was amended by B.P. Blg. 874. This subsequent statute modified Section 27 of the Insurance Code of 1978 so as to read as follows: Sec. 27. A concealment whether intentional or unintentional entitles the injured party to rescind a contract of insurance. (Emphasis supplied) The unspoken theory of the Insurance Commissioner appears to have been that by deleting the phrase "intentional or unintentional," the Insurance Code of 1978 (prior to its amendment by B.P. Blg. 874) intended to limit the kinds of concealment which generate a right to rescind on the part of the injured party to "intentional concealments." This argument is not persuasive. As a simple matter of grammar, it may be noted that "intentional" and "unintentional" cancel each other out. The net result therefore of the phrase "whether intentional or unitentional" is precisely to leave unqualified the term "concealment." Thus, Section 27 of the Insurance Code of 1978 is properly read as referring to "any concealment" without regard to whether such concealment is intentional or unintentional. The phrase "whether intentional or unintentional" was in fact superfluous. The deletion of the phrase "whether intentional or unintentional" could not have had the effect of imposing an affirmative requirement that a concealment must be intentional if it is to entitle the injured party to rescind a contract of insurance. The restoration in 1985 by B.P. Blg. 874 of the phrase "whether intentional or unintentional" merely underscored the fact that all throughout (from 1914 to 1985), the statute did not require proof that concealment must be "intentional" in order to authorize rescission by the injured party. In any case, in the case at bar, the nature of the facts not conveyed to the insurer was such that the failure to communicate must have been intentional rather than merely inadvertent. For Jaime Canilang could not have been unaware that his heart beat would at times rise to high and alarming levels and that he had consulted a doctor twice in the two (2) months before applying for non-medical insurance. Indeed, the last medical consultation took place just the day before the insurance application was filed. In all probability, Jaime Canilang went to visit his doctor precisely because of the discomfort and concern brought about by his experiencing "sinus tachycardia." We find it difficult to take seriously the argument that Great Pacific had waived inquiry into the concealment by issuing the insurance policy notwithstanding Canilang's failure to set out answers to some of the questions in the insurance application. Such failure precisely constituted concealment on the part of Canilang. Petitioner's argument, if accepted, would obviously erase Section 27 from the Insurance Code of 1978. It remains only to note that the Court of Appeals finding that the parties had not agreed in the pretrial before the Insurance Commission that the relevant issue was whether or

not Jaime Canilang had intentionally concealed material information from the insurer, was supported by the evidence of record, i.e., the Pre-trial Order itself dated 17 October 1984 and the Minutes of the Pre-trial Conference dated 15 October 1984, which "readily shows that the word "intentional" does not appear in the statement or definition of the issue in the said Order and Minutes." 18 WHEREFORE, the Petition for Review is DENIED for lack of merit and the Decision of the Court of Appeals dated 16 October 1989 in C.A.-G.R. SP No. 08696 is hereby AFFIRMED. No pronouncement as to the costs. SO ORDERED. Bidin, Davide, Jr., Romero and Melo, JJ., concur.

G.R. No. 105135 June 22, 1995 SUNLIFE ASSURANCE COMPANY OF CANADA, petitioner, vs. The Hon. COURT OF APPEALS and Spouses ROLANDO and BERNARDA BACANI, respondents.

QUIASON, J.: This is a petition for review for certiorari under Rule 45 of the Revised Rules of Court to reverse and set aside the Decision dated February 21, 1992 of the Court of Appeals in CA-G.R. CV No. 29068, and its Resolution dated April 22, 1992, denying reconsideration thereof. We grant the petition. I On April 15, 1986, Robert John B. Bacani procured a life insurance contract for himself from petitioner. He was issued Policy No. 3-903-766-X valued at P100,000.00, with double indemnity in case of accidental death. The designated beneficiary was his mother, respondent Bernarda Bacani. On June 26, 1987, the insured died in a plane crash. Respondent Bernarda Bacani filed a claim with petitioner, seeking the benefits of the insurance policy taken by her son. Petitioner conducted an investigation and its findings prompted it to reject the claim. In its letter, petitioner informed respondent Bernarda Bacani, that the insured did not disclose material facts relevant to the issuance of the policy, thus rendering the contract of insurance voidable. A check representing the total premiums paid in the amount of P10,172.00 was attached to said letter. Petitioner claimed that the insured gave false statements in his application when he answered the following questions: 5. Within the past 5 years have you: a) consulted any doctor or other health practitioner? b) submitted to: EGG? X-rays?

blood tests? other tests? c) attended or been admitted to any hospital or other medical facility? 6. Have you ever had or sought advice for: xxx xxx xxx b) urine, kidney or bladder disorder? (Rollo, p. 53) The deceased answered question No. 5(a) in the affirmative but limited his answer to a consultation with a certain Dr. Reinaldo D. Raymundo of the Chinese General Hospital on February 1986, for cough and flu complications. The other questions were answered in the negative (Rollo, p. 53). Petitioner discovered that two weeks prior to his application for insurance, the insured was examined and confined at the Lung Center of the Philippines, where he was diagnosed for renal failure. During his confinement, the deceased was subjected to urinalysis, ultra-sonography and hematology tests. On November 17, 1988, respondent Bernarda Bacani and her husband, respondent Rolando Bacani, filed an action for specific performance against petitioner with the Regional Trial Court, Branch 191, Valenzuela, Metro Manila. Petitioner filed its answer with counterclaim and a list of exhibits consisting of medical records furnished by the Lung Center of the Philippines. On January 14, 1990, private respondents filed a "Proposed Stipulation with Prayer for Summary Judgment" where they manifested that they "have no evidence to refute the documentary evidence of concealment/misrepresentation by the decedent of his health condition (Rollo, p. 62). Petitioner filed its Request for Admissions relative to the authenticity and due execution of several documents as well as allegations regarding the health of the insured. Private respondents failed to oppose said request or reply thereto, thereby rendering an admission of the matters alleged. Petitioner then moved for a summary judgment and the trial court decided in favor of private respondents. The dispositive portion of the decision is reproduced as follows: WHEREFORE, judgment is hereby rendered in favor of the plaintiffs and against the defendant, condemning the latter to pay the former the amount of One Hundred Thousand Pesos (P100,000.00) the face value of insured's Insurance Policy No. 3903766, and the Accidental Death Benefit in the amount of One Hundred Thousand

Pesos (P100,000.00) and further sum of P5,000.00 in the concept of reasonable attorney's fees and costs of suit. Defendant's counterclaim is hereby Dismissed (Rollo, pp. 43-44). In ruling for private respondents, the trial court concluded that the facts concealed by the insured were made in good faith and under a belief that they need not be disclosed. Moreover, it held that the health history of the insured was immaterial since the insurance policy was "non-medical". Petitioner appealed to the Court of Appeals, which affirmed the decision of the trial court. The appellate court ruled that petitioner cannot avoid its obligation by claiming concealment because the cause of death was unrelated to the facts concealed by the insured. It also sustained the finding of the trial court that matters relating to the health history of the insured were irrelevant since petitioner waived the medical examination prior to the approval and issuance of the insurance policy. Moreover, the appellate court agreed with the trial court that the policy was "non-medical" (Rollo, pp. 4-5). Petitioner's motion for reconsideration was denied; hence, this petition. II We reverse the decision of the Court of Appeals. The rule that factual findings of the lower court and the appellate court are binding on this Court is not absolute and admits of exceptions, such as when the judgment is based on a misappreciation of the facts (Geronimo v. Court of Appeals, 224 SCRA 494 [1993]). In weighing the evidence presented, the trial court concluded that indeed there was concealment and misrepresentation, however, the same was made in "good faith" and the facts concealed or misrepresented were irrelevant since the policy was "nonmedical". We disagree. Section 26 of The Insurance Code is explicit in requiring a party to a contract of insurance to communicate to the other, in good faith, all facts within his knowledge which are material to the contract and as to which he makes no warranty, and which the other has no means of ascertaining. Said Section provides: A neglect to communicate that which a party knows and ought to communicate, is called concealment. Materiality is to be determined not by the event, but solely by the probable and reasonable influence of the facts upon the party to whom communication is due, in

forming his estimate of the disadvantages of the proposed contract or in making his inquiries (The Insurance Code, Sec. 31). The terms of the contract are clear. The insured is specifically required to disclose to the insurer matters relating to his health. The information which the insured failed to disclose were material and relevant to the approval and issuance of the insurance policy. The matters concealed would have definitely affected petitioner's action on his application, either by approving it with the corresponding adjustment for a higher premium or rejecting the same. Moreover, a disclosure may have warranted a medical examination of the insured by petitioner in order for it to reasonably assess the risk involved in accepting the application. In Vda. de Canilang v. Court of Appeals, 223 SCRA 443 (1993), we held that materiality of the information withheld does not depend on the state of mind of the insured. Neither does it depend on the actual or physical events which ensue. Thus, "goad faith" is no defense in concealment. The insured's failure to disclose the fact that he was hospitalized for two weeks prior to filing his application for insurance, raises grave doubts about his bonafides. It appears that such concealment was deliberate on his part. The argument, that petitioner's waiver of the medical examination of the insured debunks the materiality of the facts concealed, is untenable. We reiterate our ruling in Saturnino v. Philippine American Life Insurance Company, 7 SCRA 316 (1963), that " . . . the waiver of a medical examination [in a non-medical insurance contract] renders even more material the information required of the applicant concerning previous condition of health and diseases suffered, for such information necessarily constitutes an important factor which the insurer takes into consideration in deciding whether to issue the policy or not . . . " Moreover, such argument of private respondents would make Section 27 of the Insurance Code, which allows the injured party to rescind a contract of insurance where there is concealment, ineffective (See Vda. de Canilang v. Court of Appeals, supra). Anent the finding that the facts concealed had no bearing to the cause of death of the insured, it is well settled that the insured need not die of the disease he had failed to disclose to the insurer. It is sufficient that his non-disclosure misled the insurer in forming his estimates of the risks of the proposed insurance policy or in making inquiries (Henson v. The Philippine American Life Insurance Co., 56 O.G. No. 48 [1960]). We, therefore, rule that petitioner properly exercised its right to rescind the contract of insurance by reason of the concealment employed by the insured. It must be

emphasized that rescission was exercised within the two-year contestability period as recognized in Section 48 of The Insurance Code. WHEREFORE, the petition is GRANTED and the Decision of the Court of Appeals is REVERSED and SET ASIDE. SO ORDERED. Padilla, Davide, Jr., Bellosillo and Kapunan, JJ., concur.

G.R. No. L-30685 May 30, 1983 NG GAN ZEE, plaintiff-appellee, vs. ASIAN CRUSADER LIFE ASSURANCE CORPORATION, defendant-appellant. Alberto Q. Ubay for plaintiff-appellee. Santiago F. A lidio for defendant-appellant.

ESCOLIN, J.: This is an appeal from the judgment of the Court of First Instance of Manila, ordering the appellant Asian-Crusader Life Assurance Corporation to pay the face value of an insurance policy issued on the life of Kwong Nam the deceased husband of appellee Ng Gan Zee. Misrepresentation and concealment of material facts in obtaining the policy were pleaded to avoid the policy. The lower court rejected the appellant's theory and ordered the latter to pay appellee "the amount of P 20,000.00, with interest at the legal rate from July 24, 1964, the date of the filing of the complaint, until paid, and the costs. " The Court of Appeals certified this appeal to Us, as the same involves solely a question of law. On May 12, 1962, Kwong Nam applied for a 20-year endowment insurance on his life for the sum of P20,000.00, with his wife, appellee Ng Gan Zee as beneficiary. On the same date, appellant, upon receipt of the required premium from the insured, approved the application and issued the corresponding policy. On December 6, 1963, Kwong Nam died of cancer of the liver with metastasis. All premiums had been religiously paid at the time of his death. On January 10, 1964, his widow Ng Gan Zee presented a claim in due form to appellant for payment of the face value of the policy. On the same date, she submitted the required proof of death of the insured. Appellant denied the claim on the ground that the answers given by the insured to the questions appealing in his application for life insurance were untrue. Appellee brought the matter to the attention of the Insurance Commissioner, the Hon. Francisco Y. Mandamus, and the latter, after conducting an investigation, wrote the appellant that he had found no material concealment on the part of the insured and that, therefore, appellee should be paid the full face value of the policy. This opinion of the Insurance Commissioner notwithstanding, appellant refused to settle its obligation.

Appellant alleged that the insured was guilty of misrepresentation when he answered "No" to the following question appearing in the application for life insuranceHas any life insurance company ever refused your application for insurance or for reinstatement of a lapsed policy or offered you a policy different from that applied for? If, so, name company and date. In its brief, appellant rationalized its thesis thus: ... As pointed out in the foregoing summary of the essential facts in this case, the insured had in January, 1962, applied for reinstatement of his lapsed life insurance policy with the Insular Life Insurance Co., Ltd, but this was declined by the insurance company, although later on approved for reinstatement with a very high premium as a result of his medical examination. Thus notwithstanding the said insured answered 'No' to the [above] question propounded to him. ... 1 The lower court found the argument bereft of factual basis; and We quote with approval its disquisition on the matterOn the first question there is no evidence that the Insular Life Assurance Co., Ltd. ever refused any application of Kwong Nam for insurance. Neither is there any evidence that any other insurance company has refused any application of Kwong Nam for insurance. ... The evidence shows that the Insular Life Assurance Co., Ltd. approved Kwong Nam's request for reinstatement and amendment of his lapsed insurance policy on April 24, 1962 [Exh. L-2 Stipulation of Facts, Sept. 22, 1965). The Court notes from said application for reinstatement and amendment, Exh. 'L', that the amount applied for was P20,000.00 only and not for P50,000.00 as it was in the lapsed policy. The amount of the reinstated and amended policy was also for P20,000.00. It results, therefore, that when on May 12, 1962 Kwong Nam answered 'No' to the question whether any life insurance company ever refused his application for reinstatement of a lapsed policy he did not misrepresent any fact. ... the evidence shows that the application of Kwong Nam with the Insular Life Assurance Co., Ltd. was for the reinstatement and amendment of his lapsed insurance policy-Policy No. 369531 -not an application for a 'new insurance policy. The Insular Life Assurance Co., Ltd. approved the said application on April 24, 1962. Policy No. 369531 was reinstated for the amount of P20,000.00 as applied for by Kwong Nam [Exhs. 'L', 'Ll' and 'L-2']. No new policy was issued by the Insular Life Assurance Co., Ltd. to Kwong Nam in connection with said application for reinstatement and amendment. Such being the case, the Court finds that there is no misrepresentation on this matter. 2

Appellant further maintains that when the insured was examined in connection with his application for life insurance, he gave the appellant's medical examiner false and misleading information as to his ailment and previous operation. The alleged false statements given by Kwong Nam are as follows: Operated on for a Tumor [mayoma] of the stomach. Claims that Tumor has been associated with ulcer of stomach. Tumor taken out was hard and of a hen's egg size. Operation was two [2] years ago in Chinese General Hospital by Dr. Yap. Now, claims he is completely recovered. To demonstrate the insured's misrepresentation, appellant directs Our attention to: [1] The report of Dr. Fu Sun Yuan the physician who treated Kwong Nam at the Chinese General Hospital on May 22, 1960, i.e., about 2 years before he applied for an insurance policy on May 12, 1962. According to said report, Dr. Fu Sun Yuan had diagnosed the patient's ailment as 'peptic ulcer' for which, an operation, known as a 'sub-total gastric resection was performed on the patient by Dr. Pacifico Yap; and [2] The Surgical Pathology Report of Dr. Elias Pantangco showing that the specimen removed from the patient's body was 'a portion of the stomach measuring 12 cm. and 19 cm. along the lesser curvature with a diameter of 15 cm. along the greatest dimension. On the bases of the above undisputed medical data showing that the insured was operated on for peptic ulcer", involving the excision of a portion of the stomach, appellant argues that the insured's statement in his application that a tumor, "hard and of a hen's egg size," was removed during said operation, constituted material concealment. The question to be resolved may be propounded thus: Was appellant, because of insured's aforesaid representation, misled or deceived into entering the contract or in accepting the risk at the rate of premium agreed upon? The lower court answered this question in the negative, and We agree. Section 27 of the Insurance Law [Act 2427] provides: Sec. 27. Such party a contract of insurance must communicate to the other, in good faith, all facts within his knowledge which are material to the contract, and which the other has not the means of ascertaining, and as to which he makes no warranty. 3 Thus, "concealment exists where the assured had knowledge of a fact material to the risk, and honesty, good faith, and fair dealing requires that he should communicate it to the assurer, but he designedly and intentionally withholds the same." 4

It has also been held "that the concealment must, in the absence of inquiries, be not only material, but fraudulent, or the fact must have been intentionally withheld." 5 Assuming that the aforesaid answer given by the insured is false, as claimed by the appellant. Sec. 27 of the Insurance Law, above-quoted, nevertheless requires that fraudulent intent on the part of the insured be established to entitle the insurer to rescind the contract. And as correctly observed by the lower court, "misrepresentation as a defense of the insurer to avoid liability is an 'affirmative' defense. The duty to establish such a defense by satisfactory and convincing evidence rests upon the defendant. The evidence before the Court does not clearly and satisfactorily establish that defense." It bears emphasis that Kwong Nam had informed the appellant's medical examiner that the tumor for which he was operated on was "associated with ulcer of the stomach." In the absence of evidence that the insured had sufficient medical knowledge as to enable him to distinguish between "peptic ulcer" and "a tumor", his statement that said tumor was "associated with ulcer of the stomach, " should be construed as an expression made in good faith of his belief as to the nature of his ailment and operation. Indeed, such statement must be presumed to have been made by him without knowledge of its incorrectness and without any deliberate intent on his part to mislead the appellant. While it may be conceded that, from the viewpoint of a medical expert, the information communicated was imperfect, the same was nevertheless sufficient to have induced appellant to make further inquiries about the ailment and operation of the insured. Section 32 of Insurance Law [Act No. 24271 provides as follows: Section 32. The right to information of material facts maybe waived either by the terms of insurance or by neglect to make inquiries as to such facts where they are distinctly implied in other facts of which information is communicated. It has been held that where, upon the face of the application, a question appears to be not answered at all or to be imperfectly answered, and the insurers issue a policy without any further inquiry, they waive the imperfection of the answer and render the omission to answer more fully immaterial. 6 As aptly noted by the lower court, "if the ailment and operation of Kwong Nam had such an important bearing on the question of whether the defendant would undertake the insurance or not, the court cannot understand why the defendant or its medical examiner did not make any further inquiries on such matters from the Chinese General Hospital or require copies of the hospital records from the appellant before acting on the application for insurance. The fact of the matter is that the defendant was too eager to accept the application and receive the insured's premium. It would be inequitable now to allow the defendant to avoid liability under the circumstances."

Finding no reversible error committed by the trial court, the judgment appealed from is hereby affirmed, with costs against appellant Asian-Crusader life Assurance Corporation. SO ORDERED. Makasiar (Chairman), Aquino, Concepcion, Jr., Guerrero and De Castro), JJ., concur. Abad Santos, J., I reserve my vote.

G.R. No. L-47593 December 29, 1943 THE INSULAR LIFE ASSURANCE CO., LTD., petitioner, vs. SERAFIN D. FELICIANO ET AL., respondents. Manuel Roxas and Araneta, Zaragoza, Araneta and Bautista for petitioner. Deflfin Joven and Pablo Lorenzo for respondents. Ramirez and Ortigas as amici curiae.

OZAETA, J.: In a four-to-three decision promulgated on September 13, 1941, 1 this Court affirmed the judgment of the Court of Appeals in favor of the respondents and against the petitioner for the sum of P25,000, representing the value of two insurance policies issued by the petitioner on the life of Evaristo Feliciano. A motion to reconsider and set aside said decision has been filed by the petitioner, and both parties have submitted exhaustive and luminous written arguments in support of their respective contentions. The facts of the case are set forth in the majority and dissenting opinions heretofore handed down by this Court, the salient points of which may be briefly restated as follows: Evaristo Feliciano, who died on September 29, 1935, was suffering with advanced pulmonary tuberculosis when he signed his applications for insurance with the petitioner on October 12, 1934. On that same date Doctor Trepp, who had taken X-ray pictures of his lungs, informed the respondent Dr. Serafin D. Feliciano, brother of Evaristo, that the latter "was already in a very serious ad practically hopeless condition." Nevertheless the question contained in the application "Have you ever suffered from any ailment or disease of the lungs, pleurisy, pneumonia or asthma?" appears to have been answered , "No" And above the signature of the applicant, following the answers to the various questions propounded to him, is the following printed statement:1awphil.net I declare on behalf of myself and of any person who shall have or claim any interest in any policy issued hereunder, that each of the above answers is full, complete and true, and that to the best of my knowledge and belief I am a proper subject for life insurance. (Exhibit K.) The false answer above referred to, as well as the others, was written by the Company's soliciting agent Romulo M. David, in collusion with the medical examiner Dr. Gregorio Valdez, for the purpose of securing the Company's approval of the application so that the policy to be issued thereon might be credited to said agent in connection with the

inter-provincial contest which the Company was then holding among its soliciting agents to boost the sales of its policies. Agent David bribed Medical Examiner Valdez with money which the former borrowed from the applicant's mother by way of advanced payment on the premium, according to the finding of the Court of Appeals. Said court also found that before the insured signed the application he, as well as the members of his family, told the agent and the medical examiner that he had been sick and coughing for some time and that he had gone three times to the Santol Sanatorium and had X-ray pictures of his lungs taken; but that in spite of such information the agent and the medical examiner told them that the applicant was a fit subject for insurance. Each of the policies sued upon contains the following stipulations: This policy and the application herefor constitute the entire contract between the parties hereto. . . . Only the President, or the Manager, acting jointly with the Secretary or Assistant Secretary (and then only in writing signed by them) have power in behalf of the Company to issue permits, or to modify this or any contract, or to extend the same time for making any premium payment, and the Company shall not be bound by any promise or representation heretofore or hereafter given by any person other than the above-named officials, and by them only in writing and signed conjointly as stated. The application contains, among others, the following statements: 18. I [the applicant] hereby declare that all the above statements and answers as well as all those that I may make to the Company's Medical Examiner in continuation of this application, to be complete, true and correct to the best of my knowledge and belief, and I hereby agree as follows: 1. That his declaration, with the answers to be given by me to the Medical Examiner, shall be the basis of the policy and form part of same.

3. That the said policy shall not take effect until the first premium has been paid and the policy has been delivered to and accepted by me, while I am in good health. 4. That the agent taking this application has no authority to make, modify or discharge contracts, or to waive any of the Company's rights or requirements. 5. My acceptance of any policy issued on this application will constitute a ratification by me of any corrections in or additions to this application made by the Company in the space provided "For Home Office Corrections or Additions Only." I agree that photographic copy of this applications as corrected or added to shall constitute sufficient notice to me of the changes made. (Emphasis added.)

The petitioner insists that upon the facts of the case the policies in question are null and void ab initio and that all that the respondents are entitled to is the refund of the premiums paid thereon. After a careful re-examination of the facts and the law, we are persuaded that petitioner's contention is correct. To the reasons adduced in the dissenting opinion heretofore published, we only desire to add the following considerations: When Evaristo Feliciano, the applicant for insurance, signed the application in blank and authorized the soliciting agent and/or medical examiner of the Company to write the answers for him, he made them his own agents for that purpose, and he was responsible for their acts in that connection. If they falsified the answers for him, he could not evade the responsibility for he falsification. He was not supposed to sign the application in blank. He knew that the answers to the questions therein contained would be "the basis of the policy," and for that every reason he was required with his signature to vouch for truth thereof. Moreover, from the facts of the case we cannot escape the conclusion that the insured acted in connivance with the soliciting agent and the medical examiner of the Company in accepting the policies in question. Above the signature of the applicant is the printed statement or representation: " . . . I am a proper subject for life insurance." In another sheet of the same application and above another signature of the applicant was also printed this statement: "That the said policy shall not take effect until he first premium has been paid and the policy as been delivered to and accepted by me, while I am in good health." When the applicant signed the application he was "having difficulty in breathing, . . . with a very high fever." He had gone three times to the Santol Sanatorium and had X-ray pictures taken of his lungs. He therefore knew that he was not "a proper subject for life insurance." When he accepted the policy, he knew that he was not in good health. Nevertheless, he not only accepted the first policy of P20,000 but then and there applied for and later accepted another policy of P5,000. We cannot bring ourselves to believe that the insured did not take the trouble to read the answers contained in the photostatic copy of the application attached to and made a part of the policy before he accepted it and paid the premium thereon. He must have notice that the answers to the questions therein asked concerning his clinical history were false, and yet he accepted the first policy and applied for another. In any event, he obligated himself to read the policy when he subscribed to this statement: "My acceptance of any policy issued on this application will constitute a ratification by me of any corrections in or additions to this application made by the Company . . ." By accepting the policy he became charged with knowledge of its contents, whether he actually read it or not. He could not ostrich-like hide his head from it in order to avoid his part of the bargain and at the same time claim the benefit thereof. He knew, or was chargeable with knowledge, from the very terms of the two policies sued upon (one of

which is printed in English and the other in Spanish) that the soliciting agent and the medical examiner had no power to bind the Company by any verbal promise or oral representation. The insured, therefore, had no right to rely and we cannot believe he relied in good faith upon the oral representation. The insured, therefore, had no right to rely and we cannot believe he relied in good faith upon the oral representation of said agent and medical examiner that he (the applicant) was a fit subject for insurance notwithstanding that he had been and was still suffering with advanced pulmonary tuberculosis. From all the facts and circumstances of this case, we are constrained to conclude that the insured was a coparticipant, and coresponsible with Agent David and Medical Examiner Valdez, in the fraudulent procurement of the policies in question and that by reason thereof said policies are void ab initio. Wheretofore, the motion for reconsideration is sustained and the judgment of the Court of Appeals is hereby reversed. Let another judgment be entered in favor of the respondents and against the petitioner for the refund of the premiums amounting to P1,389, with legal interest thereon from the date of the complaint, and without any finding as to costs. Moran, Paras and Bocobo, JJ., concur.

Separate Opinions

YULO, C.J., concurring: I can find no quarrel with the legal considerations and conclusions set forth in the original decision promulgated by this Court. As general rules of law they find full support not only in reason and in logic, but also in simple human sense of justice. More so, modern and complicated practices attendant to the ever growing trade in life insurance demand the strictest accountability by insurance companies for acts of their authorized agents. In this way only may the State afford reasonable protection to the unwary public from abuse by such organizations as may be found to be of questionable moral standards. But a careful consideration of the evidentiary facts as set forth in the decision of the Court of Appeals leads me to conclude that the ends of justice would not be serve by

the application to the present case of the rules so enunciated. Rather, to serve the ends of justice the case of the respondents should be removed from the protection of such rules. The subject of the insurance policies under consideration is the life of the assured. It is contended by his beneficiaries that they took these policies on the basis of a life expectancy of a person gravely stricken with tuberculosis. They have consistently made protestations that they had so informed the agents of the insurance company. But the policies were issued upon the life of the assured, as a perfectly normal and healthy person. The error is vital and goes to the very existence of the contract itself. Who is responsible for the error? The direct cause, of course, is the false recitals in the application for insurance. While it is true that it was the agents of the insurance company who filled out such application, yet it was the assured who, by signing the application in blank, made it possible for the said agents to procure the issuance of the policies on the basis of false information, in order to suit their own purposes. Upon the admitted facts, I am of the opinion that in justice and in equity, the responsibility for the falsifications made by the insurance agents in the preparation of the insurance application should be laid at the door of the assured and his beneficiaries. I vote with the majority in granting the motion for reconsideration and in reversing the decision under review. HONTIVEROS, J., dissenting: The reasons given in the dissenting opinion in this case, as published in the Official Gazette of October 4, 1941 (pp. 2847 to 2855), supplemented by those in the resolution of the majority on the motion for reconsideration, do not seem to me sufficient to overthrow the decision rendered by the Court of First Instance, confirmed by the Court of Appeals, and sustained by this Supreme court in its decision of September 18, 1941. The alleged connivance between the insured Evaristo Feliciano, the agent Romulo M. David, and the medical examiner Dr. Gregorio Valdez not only does not clearly appear of record, but on the contrary is denied in the finding of facts of the court a quo and of the Court of Appeals which cannot be reviewed or altered by this Court. The mere fact that the insured signed at the bottom of the application for insurance when some of its lines intended for answers to certain questions were still in blank, answers which according to the evidence and to the findings of the two inferior courts he had grounds to believe will be made in accordance with the information which he and his family had given to agent David and to Dr. Valdez, does not convert these two persons into agents of the insured in a way as to make the latter responsible for the acts of the former. That the photostatic copies of said forms which are attached to the

policies object of this case are almost illegible, is a fact which should be taken into account, together with the other fact that Evaristo Feliciano does not know English, the language in which those documents are written. In support of this dissenting opinion, the following authorities may be cited: The mere failure of the insured to inform himself of the insertion of false answers in the application which has been filled out by the agent of the insurer does not convict him of lack of good faith. (Vol. 5, Cooley's Briefs on Insurance, 2nd Ed., p. 4136, and many cases cited.) The insured is not chargeable with such negligence as will render him liable for false answers inserted by the agent merely because he signed the application in blank and trusted the agent to fill out by the agent, without reading it. (Id., p. 4136, and many cases cited.) An illiterate person or one who does not understand the English language (as is the case with Evaristo Feliciano) is not guilty of inexcusable negligence in failing to read the application or having it read to him, nor can it be said that such person deliberately made a false statement because he did not read over the application. (81 ALR 865, 866, W. 117 ALR 796.) Nor can it be said that the assured, who has fully, frankly, truthfully, and in good faith answered all the required questions, is guilty of negligence in signing, without reading, the application which is thereupon prepared by the agent. He is justified in assuming that the agent, has, with equal good faith, truthfully recorded the answers give. He may well say to the Company: 'You accredited this man to me as your representative, and I signed the application thus prepared by him, relying upon the character which you gave him, when you commissioned him to come to me as your agent. If he acted dishonestly in the matter, you, and not I, must suffer the consequences . . .! (Germania Life Ins. Co. vs. Lunkeheimer [1931] Ind., 538; 26 N. E., 1052) In such case the acceptance of the policy, with this application attached, does not require the insured to institute an investigation into its provisions, or the conditions upon which is was issued, to ascertain whether the agent has acted in good faith, since, under such circumstances, the insured may rely upon the presumption that he has been honestly dealt with the insurer. (Otto vs. Hartford Ins. Co., 38 Minn., 423). Besides, the principles that the insured is not bound to know the contents of the application, and may rely on the agent's assurances that his answers have been correctly written will, of course, apply with special force where the insured is illiterate and unable to read, or is ignorant of the language. (Vol. 5, Cooley's Briefs on Insurance, 2nd Ed. p. 4138, cases cited.)

And also where the photostatic copies of the application embodied in the policy are practically illegible, the insured is not bound to know the contents of the application. (New York Ins. Co. vs. Holpem D.C. 57 Fed. 2nd, 200). According to the great weight of authority, if an agent of the insurer, after obtaining from an applicant for insurance a correct and truthful answer to interrogations contained in the application for insurance, without knowledge of the applicant fills in false answers, either fraudulently or otherwise, the insurer cannot assert the falsity of such answers as a defense to the liability on the policy and this is generally without regard to the subject matter of the answers or the nature of the agent's duties or limitations on his authority, at least if not brought to the attention of the applicant. It is equally well settled that if a correct representation is made in a written application, or the insurance agent issuing the policy is appraised of the true facts concerning the matter in question, as for instance the title to the insured premises, but the agent inserts an incorrect statement in the policy, the insurer cannot rely upon the error in avoidance of its liability". Home Ins. Co. vs. Mendenhall, 154 Ill., 452, 45 NE., 1078, 36 LRA., 374; Phoenix Ins. Co. vs. Tucker, 92 Ill., 64, 34 Am Rep., 106; Commercial Ins. Co. vs. Spanknoble, 52 Ill., 53, 4 Am. Report, 582; Young vs. Hartford F. Ins. Co. 45 Iowa, 377, 24 Am. Rep., 754; Welsh vs. London Assur. 151 Pa., 607, 25 A, 142, 21 Am St. Rep., 726 (Taken from Am Juris. on Insurance Vol. 29, par. 843). An insured may be justified in signing an application in blank at the request of the insurer's agent, who agrees to fill it in from data furnished by the insured or from an old application. In fact, an insurer cannot urge the falsity of representations contained in the policy issued, or in the application, where such representations were inserted therein, either by the company or its agent, after the application was signed, without the knowledge or consent of the insured, who has made no such representations. (Couch on Insurance, Vol. 4, par. 842 b.) I believe that the motion for reconsideration presented in this case should be denied, not only because of the weighty reasons relied upon in the decision which it attacks, but also because a dangerous precedent would otherwise be established, for, with the destruction of the confidence which the public has hitherto reposed in the duly accredited agents of insurance companies and in their examining physicians, this branch of the economic life of the people will have to be unfavorably affected. Imperial, J., dissents.

G.R. No. L-24833

September 23, 1968

FIELDMEN'S INSURANCE CO., INC., petitioner, vs. MERCEDES VARGAS VDA. DE SONGCO, ET AL. and COURT OF APPEALS, respondents. Jose S. Suarez for petitioner. Eligio G. Lagman for respondents.

FERNANDO, J.: An insurance firm, petitioner Fieldmen's Insurance Co., Inc., was not allowed to escape liability under a common carrier insurance policy on the pretext that what was insured, not once but twice, was a private vehicle and not a common carrier, the policy being issued upon the insistence of its agent who discounted fears of the insured that his privately owned vehicle might not fall within its terms, the insured moreover being "a man of scant education," finishing only the first grade. So it was held in a decision of the lower court thereafter affirmed by respondent Court of Appeals. Petitioner in seeking the review of the above decision of respondent Court of Appeals cannot be so sanguine as to entertain the belief that a different outcome could be expected. To be more explicit, we sustain the Court of Appeals. The facts as found by respondent Court of Appeals, binding upon us, follow: "This is a peculiar case. Federico Songco of Floridablanca, Pampanga, a man of scant education being only a first grader ..., owned a private jeepney with Plate No. 41-289 for the year 1960. On September 15, 1960, as such private vehicle owner, he was induced by Fieldmen's Insurance Company Pampanga agent Benjamin Sambat to apply for a Common Carrier's Liability Insurance Policy covering his motor vehicle ... Upon paying an annual premium of P16.50, defendant Fieldmen's Insurance Company, Inc. issued on September 19, 1960, Common Carriers Accident Insurance Policy No. 45-HO- 4254 ... the duration of which will be for one (1) year, effective September 15, 1960 to September 15, 1961. On September 22, 1961, the defendant company, upon payment of the corresponding premium, renewed the policy by extending the coverage from October 15, 1961 to October 15, 1962. This time Federico Songco's private jeepney carried Plate No. J-68136-Pampanga-1961. ... On October 29, 1961, during the effectivity of the renewed policy, the insured vehicle while being driven by Rodolfo Songco, a duly licensed driver and son of Federico (the vehicle owner) collided with a car in the municipality of Calumpit, province of Bulacan, as a result of which mishap Federico Songco (father) and Rodolfo Songco (son) died, Carlos Songco (another son),

the latter's wife, Angelita Songco, and a family friend by the name of Jose Manuel sustained physical injuries of varying degree." 1 It was further shown according to the decision of respondent Court of Appeals: "Amor Songco, 42-year-old son of deceased Federico Songco, testifying as witness, declared that when insurance agent Benjamin Sambat was inducing his father to insure his vehicle, he butted in saying: 'That cannot be, Mr. Sambat, because our vehicle is an "owner" private vehicle and not for passengers,' to which agent Sambat replied: 'whether our vehicle was an "owner" type or for passengers it could be insured because their company is not owned by the Government and the Government has nothing to do with their company. So they could do what they please whenever they believe a vehicle is insurable' ... In spite of the fact that the present case was filed and tried in the CFI of Pampanga, the defendant company did not even care to rebut Amor Songco's testimony by calling on the witness-stand agent Benjamin Sambat, its Pampanga Field Representative." 2 The plaintiffs in the lower court, likewise respondents here, were the surviving widow and children of the deceased Federico Songco as well as the injured passenger Jose Manuel. On the above facts they prevailed, as had been mentioned, in the lower court and in the respondent Court of Appeals.1awphl.nt The basis for the favorable judgment is the doctrine announced in Qua Chee Gan v. Law Union and Rock Insurance Co., Ltd., 3 with Justice J. B. L. Reyes speaking for the Court. It is now beyond question that where inequitable conduct is shown by an insurance firm, it is "estopped from enforcing forfeitures in its favor, in order to forestall fraud or imposition on the insured." 4 As much, if not much more so than the Qua Chee Gan decision, this is a case where the doctrine of estoppel undeniably calls for application. After petitioner Fieldmen's Insurance Co., Inc. had led the insured Federico Songco to believe that he could qualify under the common carrier liability insurance policy, and to enter into contract of insurance paying the premiums due, it could not, thereafter, in any litigation arising out of such representation, be permitted to change its stand to the detriment of the heirs of the insured. As estoppel is primarily based on the doctrine of good faith and the avoidance of harm that will befall the innocent party due to its injurious reliance, the failure to apply it in this case would result in a gross travesty of justice. That is all that needs be said insofar as the first alleged error of respondent Court of Appeals is concerned, petitioner being adamant in its far-from-reasonable plea that estoppel could not be invoked by the heirs of the insured as a bar to the alleged breach of warranty and condition in the policy. lt would now rely on the fact that the insured owned a private vehicle, not a common carrier, something which it knew all along when

not once but twice its agent, no doubt without any objection in its part, exerted the utmost pressure on the insured, a man of scant education, to enter into such a contract. Nor is there any merit to the second alleged error of respondent Court that no legal liability was incurred under the policy by petitioner. Why liability under the terms of the policy 5 was inescapable was set forth in the decision of respondent Court of Appeals. Thus: "Since some of the conditions contained in the policy issued by the defendant-appellant were impossible to comply with under the existing conditions at the time and 'inconsistent with the known facts,' the insurer 'is estopped from asserting breach of such conditions.' From this jurisprudence, we find no valid reason to deviate and consequently hold that the decision appealed from should be affirmed. The injured parties, to wit, Carlos Songco, Angelito Songco and Jose Manuel, for whose hospital and medical expenses the defendant company was being made liable, were passengers of the jeepney at the time of the occurrence, and Rodolfo Songco, for whose burial expenses the defendant company was also being made liable was the driver of the vehicle in question. Except for the fact, that they were not fare paying passengers, their status as beneficiaries under the policy is recognized therein." 6 Even if it be assumed that there was an ambiguity, an excerpt from the Qua Chee Gan decision would reveal anew the weakness of petitioner's contention. Thus: "Moreover, taking into account the well known rule that ambiguities or obscurities must be strictly interpreted against the party that caused them, the 'memo of warranty' invoked by appellant bars the latter from questioning the existence of the appliances called for in the insured premises, since its initial expression, 'the undernoted appliances for the extinction of fire being kept on the premises insured hereby, ... it is hereby warranted ...,' admits of interpretation as an admission of the existence of such appliances which appellant cannot now contradict, should the parol evidence rule apply." 7 To the same effect is the following citation from the same leading case: "This rigid application of the rule on ambiguities has become necessary in view of current business practices. The courts cannot ignore that nowadays monopolies, cartels and concentration of capital, endowed with overwhelming economic power, manage to impose upon parties dealing with them cunningly prepared 'agreements' that the weaker party may not change one whit, his participation in the 'agreement' being reduced to the alternative to 'take it or leave it' labelled since Raymond Saleilles 'contracts by adherence' (contrats d'adhesion), in contrast to those entered into by parties bargaining on an equal footing, such contracts (of which policies of insurance and international bills of lading are prime examples) obviously call for greater strictness and vigilance on the part of courts of justice with a view to protecting the weaker party from abuses and imposition, and prevent their becoming traps for the unwary (New Civil Code. Article 24; Sent. of Supreme Court of Spain, 13 Dec. 1934, 27 February 1942)." 8

The last error assigned which would find fault with the decision of respondent Court of Appeals insofar as it affirmed the lower court award for exemplary damages as well as attorney's fees is, on its face, of no persuasive force at all. The conclusion that inescapably emerges from the above is the correctness of the decision of respondent Court of Appeals sought to be reviewed. For, to borrow once again from the language of the Qua Chee Gan opinion: "The contract of insurance is one of perfect good faith (uberima fides) not for the insured alone,but equally so for the insurer; in fact, it is more so for the latter, since its dominant bargaining position carries with it stricter responsibility." 9 This is merely to stress that while the morality of the business world is not the morality of institutions of rectitude like the pulpit and the academe, it cannot descend so low as to be another name for guile or deception. Moreover, should it happen thus, no court of justice should allow itself to lend its approval and support.1awphl.nt We have no choice but to recognize the monetary responsibility of petitioner Fieldmen's Insurance Co., Inc. It did not succeed in its persistent effort to avoid complying with its obligation in the lower court and the Court of Appeals. Much less should it find any receptivity from us for its unwarranted and unjustified plea to escape from its liability. WHEREFORE, the decision of respondent Court of Appeals of July 20, 1965, is affirmed in its entirety. Costs against petitioner Fieldmen's Insurance Co., Inc. Concepcion, C.J., Reyes, J.B.L., Dizon, Makalintal, Zaldivar, Sanchez, Castro and Angeles, JJ., concur.

WEEK 6 VII. Representations

G.R. No. L-12707

August 10, 1918

MRS. HENRY E. HARDING, and her husband, plaintiffs-appellees, vs. COMMERCIAL UNION ASSURANCE COMPANY, defendant-appellant. Lawrence & Ross for appellant. Gibbs, McDonough & Johnson for appellees. FISHER, J.: This was an action by plaintiffs to recover from defendant the sum of P3,000 and interest, alleged to be due under the terms of a policy of insurance. The trial court gave plaintiffs judgment for the amount demanded, with interest and costs, and from that decision the defendant appeals. The court below stated the issues made by the pleadings in this case, and its finding of fact, as follows: It is alleged by plaintiffs and admitted by defendant that plaintiffs are husband and wife and residents of the city of Manila; that the defendant is a foreign corporation organized and existing under and by virtue of the laws of Great Britain and duly registered in the Philippine Islands, and Smith, Bell & Co. (limited), a corporation organized and existing under the laws of the Philippine Islands, with its principal domicile in the city of Manila, is the agent in the Philippine Islands of said defendant. The plaintiffs alleged that on February 16, 1916, the plaintiff Mrs. Henry E. Harding was the owner of a Studebaker automobile, registered number 2063, in the city of Manila; that on said date; in consideration of the payment to the defendant of the premium of P150, by said plaintiff, Mrs. Henry E. Harding, with the consent of her husband, the defendant by its duly authorized agent, Smith, Bell & Company (limited), made its policy of insurance in writing upon said automobile was set forth in said policy to be P3,000 that the value of said automobile was set forth in said policy (Exhibit A) to be P3,000; that on March 24, 1916, said automobile was totally destroyed by fire; that the loss thereby to plaintiffs was the sum of P3,000; that thereafter, within the period mentioned in the said policy of insurance, the plaintiff, Mrs. Henry E. Harding, furnished the defendant the proofs of her said loss and interest, and otherwise performed all the

conditions of said policy on her part, and that the defendant has not paid said loss nor any part thereof, although due demand was made upon defendant therefor. The defendant, by its answer, admitted the allegations of the residence and status of the parties and denied all the other allegation of the said complaint, and for a separate and affirmative defense alleged (1) that on February 17, 1916, at the city of Manila, P.I. the defendant upon request of plaintiff, Mrs. Henry E. Harding, issued to the said plaintiff the policy of insurance on an automobile alleged by the said plaintiff to be her property; that the said request for the issuance of said policy of insurance was made by means of a proposal in writing signed and delivered by said plaintiff to the defendant, guaranteeing the truth of the statements contained therein which said proposal is referred to in the said policy of insurance made a part thereof; (2) that certain of the statements and representations contained in said proposal and warranted by said plaintiff to be true, to wit: (a) the price paid by the proposer for the said automobile; (b) the value of said automobile at the time of the execution and delivery of the said proposal and (c) the ownership of said automobile, were false and known to be false by the said plaintiff at the time of signing and delivering the said proposal and were made for the purpose of misleading and deceiving the defendant, and inducing the defendant, relying upon the warranties, statements, and representations contained in the said proposal and believing the same to be true, issued the said policy of insurance. The defendant prays that judgment be entered declaring the said policy of insurance to be null and void, and that plaintiffs take nothing by this action; and for such further relief as to the court may seem just and equitable. The evidence in this case shows that some time in the year 1913 Levy Hermanos, the Manila agents for the Studebaker automobile, sold the automobile No. 2063 to John Canson for P3,200 (testimony of Mr. Diehl); that under date of October 14, 1914, John Canson sold the said automobile to Henry Harding for the sum of P1,500 (Exhibit 2); that under date of November 19, 1914, the said Henry Harding sold the said automobile No. 2063 to J. Brannigan, of Los Baos, Province of Laguna, P.I., for the sum of P2,000 (Exhibit 3); that under date of December 20, 1915, J. C. Graham of Los Baos, Province of Laguna, P.I., sold the said automobile No. 2063 to Henry Harding of the city of Manila for the sum of P2,800 (Exhibit 4 and testimony of J. C. Graham); that on or about January 1, 1916, the said Henry Harding gave the said automobile to his wife; Mrs. Henry E. Harding, one of the plaintiffs, as a present; that said automobile was repaired and repainted at the Luneta Garage at a cost of some P900 (testimony of Mr. Server); that while the said automobile was at the Luneta Garage; the said Luneta Garage, acting as agent for Smith, Bell & Company, (limited), solicited of the plaintiff Mrs. Harding the insurance of said automobile by the defendant Company (testimony of Mrs. Henry Harding and Mr. Server); that a proposal was filled out by the said agent and signed by the plaintiff Mrs. Henry E. Harding, and in said proposal under the heading

"Price paid by proposer," is the amount of "3,500" and under another heading "Present value" is the amount of "3,000" (Exhibit 1). The evidence tends to show that after the said proposal was made a representative of the Manila agent of defendant went to the Luneta Garage and examined said automobile No. 2063 and Mr. Server, the General Manager of the Luneta Garage, an experienced automobile mechanic, testified that at the time this automobile was insured it was worth about P3,000, and the defendant, by and through its said agent Smith, Bell & Company (limited), thereafter issued a policy of insurance upon proposal in which policy the said automobile was described as of the "present value" of P3,000 and the said defendant charged the said plaintiff Mrs. Henry E. Harding as premium on said policy the sum of P150, or 5 per cent of the then estimated value of P3,000. (Exhibit A.) The "Schedule" in said policy of insurance describes the automobile here in question, and provides in part of follows: "Now it is hereby agreed as follows: "That during the period above set forth and during any period for which the company may agree to renew this policy the company will subject to the exception and conditions contained herein or endorsed hereon indemnify the insured against loss of or damage to any motor car described in the schedule hereto (including accessories) by whatever cause such loss or damage may be occasioned and will further indemnify the insured up to the value of the car or P3,000 whichever is the greater against any claim at common law made by any person (not being a person in the said motor car nor in the insured's service) for loss of life or for accidental bodily injury or damage to property caused by the said motor car including law costs payable in connection with such claim when incurred with the consent of the company." The evidence further shows that on March 24, 1916, the said automobile was totally destroyed by fire, and that the iron and steel portions of said automobile which did not burn were taken into the possession of the defendant by and through its agent Smith, Bell & Company (limited), and sold by it for a small sum, which had never been tendered to the plaintiff prior to the trial of this case, but in open court during the trial the sum of P10 as the proceeds of such sale was tendered to plaintiff and refused. Upon the facts so found, which we hold are supported by the evidence, the trial judge decided that there was no proof of fraud on the part of plaintiff in her statement of the value of the automobile, or with respect to its ownership; that she had an insurable interest therein; and that defendant, having agreed to the estimated value, P3,000, and having insured the automobile for that amount, upon the basis of which the premium was paid, is bound by it and must pay the loss in accordance with the stipulated insured value. The assignments of error made on behalf of appellant put in issue the

correctness of those conclusions of law, and some others of minor importance relating to the exclusion of evidence. Disposing of the minor objections first, as we have reached the conclusion that the trial court was right in holding that the defendant is bound by the estimated value of the automobile upon which policy was issued, and that the plaintiff was not guilty of fraud in regard thereto, the exclusion of the testimony of the witness Diehl is without importance. It merely tended to show the alleged actual value of the automobile, and in the view we take of the case such evidence was irrelevant. Appellant contends that Mrs. Harding was not the owner of the automobile at the time of the issuance of the policy, and, therefore, had no insurable interest in it. The court below found that the automobile was given to plaintiff by her husband shortly after the issuance of the policy here in question. Appellant does not dispute the correctness of this finding, but contends that the gift was void, citing article 1334 of the Civil Code which provides that "All gifts between spouses during the marriage shall be void. Moderate gifts which the spouses bestow on each other on festive days of the family are not included in this rule." We are of the opinion that this contention is without merit. In the case of Cook vs. McMicking 27 Phil. Rep., 10), this court said: It is claimed by the appellants that the so-called transfer from plaintiff's husband to her was completely void under article 1458 of the Civil Code and that, therefore, the property still remains the property of Edward Cook and subject to levy under execution against him. In our opinion the position taken by appellants is untenable. They are not in a position to challenge the validity of the transfer, if it may be called such. They bore absolutely no relation to the parties to the transfer at the time it occurred and had no rights or interests inchoate, present, remote, or otherwise, in the property in question at the time the transfer occurred. Although certain transfers from husband to wife or from wife to husband are prohibited in the article referred to, such prohibition can be taken advantage of only by persons who bear such a relation to the parties making the transfer or to the property itself that such transfer interferes with their rights or interests. Unless such a relationship appears the transfer cannot be attacked. Even assuming that defendant might have invoked article 1334 as a defense, the burden would be upon it to show that the gift in question does not fall within the exception therein established. We cannot say, as a matter of law, that the gift of an automobile by a husband to his wife is not a moderate one. Whether it is or is not would depend upon the circumstances of the parties, as to which nothing is disclosed by the record.

Defendant contends that the statement regarding the cost of the automobile was a warranty, that the statement was false, and that, therefore, the policy never attached to the risk. We are of the opinion that it has not been shown by the evidence that the statement was false on the contrary we believe that it shows that the automobile had in fact cost more than the amount mentioned. The court below found, and the evidence shows, that the automobile was bought by plaintiff's husband a few weeks before the issuance of the policy in question for the sum of P2,800, and that between that time and the issuance of the policy some P900 was spent upon it in repairs and repainting. The witness Server, an expert automobile mechanic, testified that the automobile was practically as good as new at the time the insurance was effected. The form of proposal upon which the policy was issued does not call for a statement regarding the value of the automobile at the time of its acquisition by the applicant for the insurance, but merely a statement of its cost. The amount stated was less than the actual outlay which the automobile represented to Mr. Harding, including repairs, when the insurance policy was issued. It is true that the printed form calls for a statement of the "price paid by the proposer," but we are of the opinion that it would be unfair to hold the policy void simply because the outlay represented by the automobile was made by the plaintiff's husband and not by his wife, to whom he had given the automobile. It cannot be assumed that defendant should not have issued the policy unless it were strictly true that the price representing the cost of the machine had been paid by the insured and by no other person that it would no event insure an automobile acquired by gift, inheritance, exchange, or any other title not requiring the owner to make a specific cash outlay for its acquisition. Furthermore, the court below found and the evidence shows, without dispute, that the proposal upon which the policy in question was issued was made out by defendant's agent by whom the insurance was solicited, and that appellee simply signed the same. It also appears that an examiner employed by the defendant made an inspection of the automobile before the acceptance of the risk, and that the sum after this examination. The trial court found that Mrs. Harding, in fixing the value of the automobile at P3,000, acted upon information given her by her husband and by Mr. Server, the manager of the Luneta Garage. The Luneta Garage, it will be remembered, was the agent of the defendant corporation in the solicitation of the insurance. Mrs. Harding did not state of her own knowledge that the automobile originally cost P3,000, or that its value at the time of the insurance was P3,000. She merely repeated the information which had been given her by her husband, and at the same time disclosed to defendant's agent the source of her information. There is no evidence to sustain the contention that this communication was made in bad faith. It appears that the statements in the proposal as to the price paid for the automobile and as to its value were written by Mr. Quimby who solicited the insurance on behalf of defendant, in his capacity as an employee of the Luneta Garage, and wrote out the proposal for Mrs. Harding to sign. Under these

circumstances, we do not think that the facts stated in the proposal can be held as a warranty of the insured, even if it should have been shown that they were incorrect in the absence of proof of willful misstatement. Under such circumstance, the proposal is to be regarded as the act of the insurer and not of the insured. This question was considered in the case of the Union Insurance Company vs. Wilkinson (13 Wall., 222; 20 L. ed., 617), in which the Supreme Court of the United States said: This question has been decided differently by courts of the highest respectability in cases precisely analogous to the present. It is not to be denied that the application logically considered, is the work of the assured, and if left to himself or to such assistance as he might select, the person so selected would be his agent, and he alone would be responsible. On the other hand, it is well-known, so well that no court would be justified in shutting its eyes to it, that insurance companies organized under the laws of one State, and having in that State their principal business office, send these agents all over the land, with directions to solicit and procure applications for policies furnishing them with printed arguments in favor of the value and necessity of life insurance, and of the special advantages of the corporation which the agent represents. They pay these agents large commissions on the premiums thus obtained, and the policies are delivered at their hands to the assured. The agents are stimulated by letters and instructions to activity in procuring contracts, and the party who is in this manner induced to take out a policy, rarely sees or knows anything about the company or its officers by whom it is issued, but looks to and relies upon the agent who has persuaded him to effect insurance as the full and complete representative of the company, in all that is said or done in making the contract. Has he not a right to so regard him? It is quite true that the reports of judicial decisions are filled with the efforts of these companies, by their counsel, to establish the doctrine for the acts of these agents to the simple receipt of the premium and delivery of the policy, the argument being that, as to all other acts of the agent, he is the agent of the assured. This proposition is not without support in some of the earlier decision on the subject; and, at a time when insurance companies waited for parties to come to them to seek assurance, or to forward applications on their own motion, the doctrine had a reasonable foundation to rest upon. But to apply such a doctrine, in its full force, to the system of selling policies through agents, which we have described, would be a snare and a delusion, leading, as it has done in numerous instances, to the grossest frauds, of which the insurance corporations receive the benefits, and the parties supposing themselves insured are the victims. The tendency of the modern decisions in this country is steadily in the opposite direction. The powers of the agent are, prima facie, co-extensive with the business intrusted to his care, and will not be narrowed by limitations not communicated to the person with whom he deals. (Bebee vs. Ins. Co., 25 Conn., 51; Lycoming Ins. Co. vs. Schoolenberger, 44 Pa., 259; Beal vs. Ins. Co., 16 Wis., 241; Davenport vs. Ins. Co., 17 Iowa, 276.) An insurance company, establishing a local agency, must be held responsible to the parties

with whom they transact business, for the acts and declarations of the agent, within the scope of his employment, as if they proceeded from the principal. (Sav. Bk. vs. Ins. Co., 31 Conn., 517; Hortwitz vs. Ins. Co., 40 Mo., 557; Ayres vs. Ins. Co., 17 Iowa, 176; Howard Ins. Co. vs. Bruner, 23 Pa., 50.) In the fifth edition of American Leading Cases, 917, after a full consideration of the authorities, it is said: "By the interested or officious zeal of the agents employed by the insurance companies in the wish to outbid each other and procure customers, they not unfrequently mislead the insured, by a false or erroneous statement of what the application should contain; or, taking the preparation of it into their own hands, procure his signature by an assurance that it is properly drawn, and will meet the requirements of the policy. The better opinion seems to be that, when this course is pursued, the description of the risk should, though nominally proceeding from the insured, be regarded as the act of the insurers." (Rowley vs. Empire Ins. Co., 36 N.Y., 550.) The modern decisions fully sustain this proposition, and they seem to us founded on reason and justice, and meet our entire approval. This principle does not admit oral testimony to vary or contradict that which is in writing, but it goes upon the idea that the writing offered in evidence was not the instrument of the party whose name is signed to it; that it was procured under such circumstances by the other side as estops that side from using it or relying on its contents; not that it may be contradicted by oral testimony, but that it may be shown by such testimony that it cannot be lawfully used against the party whose name is signed to it. (See also Am. Life Ins. Co. vs. Mahone, 21 Wallace, 152.) The defendant, upon the information given by plaintiff, and after an inspection of the automobile by its examiner, having agreed that it was worth P3,000, is bound by this valuation in the absence of fraud on the part of the insured. All statements of value are, of necessity, to a large extent matters of opinion, and it would be outrageous to hold that the validity of all valued policies must depend upon the absolute correctness of such estimated value. As was said by the Supreme Court of the United States in the case of the First National Bank vs. Hartford Fire Insurance Co. (5 Otto, 673; 24 L. ed., 563), at. p. 565 of the Lawyers Edition: The ordinary test of the value of property is the price it will commend in the market if offered for sale. But that test cannot, in the very nature of the case, be applied at the time application is made for insurance. Men may honestly differ about the value of property, or as to what it will bring in the market; and such differences are often very marked among those whose special business it is to buy and sell property of all kinds. The assured could do no more than estimate such value; and that, it seems, was all that

he was required to do in this case. His duty was to deal fairly with the Company in making such estimate. The special finding shows that he discharged that duty and observed good faith. We shall not presume that the Company, after requiring the assured in his application to give the "estimated value," and then to covenant that he had stated all material facts in regard to such value, so far as known to him, and after carrying that covenant, by express words, into the written contract, intended to abandon the theory upon which it sought the contract, and make the absolute correctness of such estimated value a condition precedent to any insurance whatever. The application, with its covenant and stipulations, having been made a part of the policy, that presumption cannot be indulged without imputing to the Company a purpose, by studied intricacy or an ingenious framing of the policy, to entrap the assured into incurring obligations which, perhaps, he had no thought of assuming. Section 163 of the Insurance Law (Act No. 2427) provides that "the effect of a valuation in a policy of fire insurance is the same as in a policy of marine insurance." By the terms of section 149 of the Act cited, the valuation in a policy of marine insurance is conclusive if the insured had an insurable interest and was not guilty of fraud. We are, therefore, of the opinion and hold that plaintiff was the owner of the automobile in question and had an insurable interest therein; that there was no fraud on her part in procuring the insurance; that the valuation of the automobile, for the purposes of the insurance, is binding upon the defendant corporation, and that the judgment of the court below is, therefore, correct and must be affirmed, with interest, the costs of this appeal to be paid by the appellant. So ordered. Arellano, C.J., Torres, Street, Malcolm and Avancea, JJ., concur.

G.R. No. L-41794

August 30, 1935

SEGUNDINA MUSGI, ET AL., plaintiffs-appellees, vs. WEST COAST LIFE INSURANCE CO., defendant-appellant. Courtney Whitney for appellants. Laurel, Del Rosario and Sabido for appellees. IMPERIAL, J.: The plaintiffs, as beneficiaries, brought suit against the defendant to recover the value of two life insurance policies. The defendant appealed from a judgment sentencing it to pay the plaintiffs the amount of said policies, and the costs. The principal facts of the case are embodied in the following written stipulation entered into by the parties: 1. That Arsenio T. Garcia was insured by the defendant company in the sum of P5,000 as evidenced by Policy No. 129454 effective as of July 25, 1931, hereby attached and marked as Exhibit A; 2. That the said Arsenio T. Garcia was again insured by the defendant company in the sum of P10,000 effective as of October 20, 1931, as evidenced by Policy No. 130381 hereby attached and marked as Exhibit B; 3. That the two policies aforementioned were valid and subsisting at the time of the death of the insured on December 30, 1932; the fact of said death is evidenced by the accompanying death certificate issued by the Civil Register of Pasay, Rizal, which is marked as Exhibit C; 4. That the plaintiffs herein are the beneficiaries in said policies, Segundina Musgi of Policy No. 129454, and Buenaventura Garcia of Policy No. 130381; 5. That demand was made upon the defendant company for the payment of the two policies above referred to, but the defendant company refused to pay on the grounds stated in the answer. The two policies were issued upon applications filed by the insured on July 20, 1931 and October 15, of the same year, respectively. In both applications, the insured had to answer inquiries as to his state of health and that of his family, which he did voluntarily. In each of the said applications the following question was asked: "1. What physician or practitioner or any other person not named above have you consulted or been treated by, and for what illness, or ailment? (If none, so state.)" In the first application, the insured answered "None", and in the second, "No". These answers of the insured as well as his other statements contained in his applications were one of the causes or

considerations for the issuance of the policies, and they so positively appear therein. After the death of the insured and as a result of the demand made by the beneficiaries upon the defendant to pay the value of the policies, the latter discovered that the aforementioned answers were false and fraudulent, because the truth was that the insured, before answering and signing the applications and before the issuance of the policies, had been treated in the General Hospital by a lady physician for different ailments. It indisputably appears that between May 13 and 19, 1929, the insured had entered the General Hospital in Manila, and was treated by Doctor Pilar V. Cruz for peptic ulcer and chronic catarrhal nasopharyngitis; on August 5, 1930, he entered the same hospital and was treated by the same physician for chronic pyelocystitis and for incipient pulmonary tuberculosis; on the 13th of the same month he returned to the hospital and was treated by the same physician for chronic suppurative pyelocystitis and for chronic bronchitis; on the 20th of the same month he again entered the hospital and was treated by the same doctor for acute tracheo-bronchitis and chronic suppurative pyelocystitis; on the 27th of the same month he again entered the same hospital and was treated for the same ailments; on December 11, 1930, he again entered the hospital and was treated for the same ailments; on the 18th of the same month, he again entered the hospital and was treated for the same ailments; on the 28th of the same month he again entered the hospital and was treated for the same ailments, and, finally, on January 11, 1931, he again entered the hospital and was treated by the same doctor for the same ailments. The defendant contended at the outset that the two policies did not create any valid obligation because they were fraudulently obtained by the insured. The appealed decision holds that the health of the insured before the acceptance of his applications and the issuance of the policies could neither be discussed nor questioned by the defendant, because the insured was examined by three physicians of the company and all of them unanimously certified that he was in good health and that he could be properly insured. The question here is not whether the physicians' reports or the answers which the insured gave to them relative to his health were correct or not. It is admitted that such information was substantially correct, in the sense that the physicians of the defendant who examined the insured, for failure to make a detailed examination, did not discover the ailments suffered by the insured. However, the question raised for our determination is whether the two answers given by the insured in his applications are false, and if they were the cause, or one of the causes, which induced the defendant to issue the policies. On the first point, the facts above set out leave no room for doubt. The insured knew that he had suffered from a number of ailments, including incipient pulmonary tuberculosis, before subscribing the applications, yet he concealed them and omitted the hospital where he was confined as well as the name of the lady physician who treated him. That this concealment and the false statements constituted fraud, is likewise clear, because the defendant by reason thereof accepted the risk which it would otherwise have flatly refused. When not otherwise specially provided for by the Insurance Law, the contract of life insurance is governed by the general rules of the civil law regarding contracts. Article 1261 of the Civil Code provides that there is no contract unless there should be, in addition to consent and a definite object, a consideration for the obligation established. And article 1276 provides

that the statement of a false consideration shall render the contract void. The two answers being one of the considerations of the policies, and it appearing that they are false and fraudulent, it is evident that the insurance contracts were null and void and did not give rise to any right to recover their value or amount. A similar case was already decided by this court in Argente vs. West Coast Life Insurance Co. (51 Phil., 725). In that case the insured concealed from the physician who examined her that she had consulted and had been treated by another physician for cerebral congestion and Bell's Palsy, and that she was addicted to alcohol, so much so that on one occasion she was confined in the San Lazaro Hospital suffering from "alcoholism"; this court held that such concealments and false and fraudulent statements rendered the policy null and void. In discussing the legal phase of the case, this court said: One ground for the rescission of a contract of insurance under the Insurance Act is a "concealment", which in section 25 is defined as "A neglect to communicate that which a party knows and ought to communicate". Appellant argues that the alleged concealment was immaterial and insufficient to avoid the policy. We cannot agree. In an action on a life insurance policy where the evidence conclusively shows that the answers to questions concerning diseases were untrue, the truth or falsity of the answers become the determining factor. If the policy was procured by fraudulent representations, the contract of insurance apparently set forth therein was never legally existent. It can fairly be assumed that had the true facts been disclosed by the assured, the insurance would never have been granted. In Joyce, The Law of Insurance, second edition, volume 3, Chapter LV, is found the following: "Concealment exists where the assured has knowledge of a fact material to the risk, and honesty, good faith and fair dealing requires that he should communicate it to the assured, but he designedly and intentionally withholds the same. "Another rule is that if the assured undertakes to state all the circumstances affecting the risk, a full and fair statement of all is required. "It is also held that the concealment must, in the absence of inquiries, be not only material, but fraudulent, or the fact must have been intentionally withheld; so it is held under English law that if no inquiries are made and no fraud or design to conceal enters into the concealment the contract is not avoided. And it is determined that even though silence may constitute misrepresentation or concealment it is not of itself necessarily so as it is a question of fact. Nor is there a concealment justifying a forfeiture where the fact of insanity is not disclosed no questions being asked concerning the same. . . . "But it would seem that if a material fact is actually known to the assured, its concealment must of itself necessarily be a fraud, and if the fact is one which the

assured ought to know, or is presumed to know, the presumption of knowledge ought to place the assured in the same position as in the former case with relation to material facts; and if the jury in such cases find the fact material, and one tending to increase the risk, it is difficult to see how the inference of a fraudulent intent or intentional concealment can be avoided. And it is declared that if a material fact is concealed by assured it is equivalent to a false representation that it does not exist and that the essentials are the truth of the representations whether they were intended to mislead and did insurer accept them as true and act upon them to his prejudice. So it is decided that under a stipulation voiding the policy for concealment or misrepresentation of any material fact or if his interest is not truly stated or is other than the sole and unconditional ownership the facts are unimportant that insured did not intend to deceive or withhold information as to encumbrances even though no questions were asked. And if insured while being examined for life insurance, and knowing that she had heart disease, falsely stated that she was in good health, and though she could not read the application, it was explained to her and the questions asked through an interpreter, and the application like the policy contained a provision that no liability should be incurred unless the policy was delivered while the insured was in good health, the court properly directed a verdict for the insurer, though a witness who was present at the examination testified that the insured was not asked whether she had heart disease. xxx xxx xxx

"The basis of the rule vitiating the contract in cases of concealment is that it misleads or deceives the insurer into accepting the risk, or accepting it at the rate of premium agreed upon. The insurer, relying upon the belief that the assured will disclose every material fact within his actual or presumed knowledge, is misled into a belief that the circumstance withheld does not exist, and he is thereby induced to estimate the risk upon a false basis that it does not exist. The principal question, therefore, must be, Was the assurer misled or deceived into entering a contract obligation or in fixing the premium of insurance by a withholding of material information or facts within the assured's knowledge or presumed knowledge? "It therefore follows that the assurer in assuming a risk is entitled to know every material fact of which the assured has exclusive or peculiar knowledge, as well as all material facts which directly tend to increase the hazard or risk which are known by the assured, or which ought to be or are presumed to be known by him. And a concealment of such facts vitiates the policy. "It does not seem to be necessary ... that the ... suppression of the truth should have been willful." If it were but an inadvertent omission, yet if it were material to the risk and such as the plaintiff should have known to be so, it would render the policy void. But it is held that if untrue or false answers are given in response to inquiries and they relate to material facts the policy is avoided without regard to the knowledge or fraud of assured, although under the statute statements are representations

which must be fraudulent to avoid the policy. So under certain codes the important inquiries are whether the concealment was willful and related to a matter material to the risk. xxx xxx xxx

"If the assured has exclusive knowledge of material facts, he should fully and fairly disclose the same, whether he believes them material or not. But notwithstanding this general rule it will not infrequently happen, especially in life risks, that the assured may have a knowledge actual or presumed of material facts, and yet entertain an honest belief that they are not material. ... The determination of the point whether there has or has not been a material concealment must rest largely in all cases upon the form of the questions propounded and the exact terms of the contract. Thus, where in addition to specifically named diseases the insured was asked whether he had had any sickness within ten years, to which he answered "No", and it was proven that within that period he had had a slight attack of pharyngitis, it was held a question properly for the jury whether such an inflammation of the throat was a "sickness" within the intent of the inquiry, and the court remarked on the appealed decision that if it could be held as a matter of law that the policy was thereby avoided, then it was a mere devise on the part of insurance companies to obtain money without rendering themselves liable under the policy. . . . ". . . The question should be left to the jury whether the assured truly represented the state of his health so as not to mislead or deceive the insurer; and if he did not deal in good faith with the insurer in that matter, then the inquiry should be made, Did he know the state of his health so as to be able to furnish a proper answer to such questions as are propounded. A Massachusetts case, if construed as it is frequently cited, would be opposed to the above conclusion; but, on the contrary, it sustains it, for the reason that symptoms of consumption had so far developed themselves within a few months prior to effecting the insurance as to induce a reasonable belief that the applicant had that fatal disease, and we should further construe this case as establishing the rule that such a matter cannot rest alone upon the assured's belief irrespective of what is a reasonable belief, but that it ought to be judged by the criterion whether the belief is one fairly warranted by the circumstances. A case in Indiana, however, holds that if the assured has some affection or ailment of one or more of the organs inquired about so well-defined and marked as to materially derange for a time the functions of such organ, as in the case of Bright's disease, the policy will be avoided by a nondisclosure, irrespective of the fact whether the assured knew of such ailment or not. . . ." In view of the foregoing, we are of the opinion that the appellant's first two assignments of error are well founded, wherefore, the appealed judgment is reversed and the defendant absolved from the complaint, with the costs of both instances to the plaintiffs. So ordered.

G.R. No. L-34200 September 30, 1982 REGINA L. EDILLON, as assisted by her husband, MARCIAL EDILLON, petitionersappellants, vs. MANILA BANKERS LIFE INSURANCE CORPORATION and the COURT OF FIRST INSTANCE OF RIZAL, BRANCH V, QUEZON CITY, respondents-appellees. K.V. Faylona for petitioners-appellants. L. L. Reyes for respondents-appellees.

VASQUEZ, J.: The question of law raised in this case that justified a direct appeal from a decision of the Court of First Instance Rizal, Branch V, Quezon City, to be taken directly to the Supreme Court is whether or not the acceptance by the private respondent insurance corporation of the premium and the issuance of the corresponding certificate of insurance should be deemed a waiver of the exclusionary condition of overage stated in the said certificate of insurance. The material facts are not in dispute. Sometime in April 1969, Carmen O, Lapuz applied with respondent insurance corporation for insurance coverage against accident and injuries. She filled up the blank application form given to her and filed the same with the respondent insurance corporation. In the said application form which was dated April 15, 1969, she gave the date of her birth as July 11, 1904. On the same date, she paid the sum of P20.00 representing the premium for which she was issued the corresponding receipt signed by an authorized agent of the respondent insurance corporation. (Rollo, p. 27.) Upon the filing of said application and the payment of the premium on the policy applied for, the respondent insurance corporation issued to Carmen O. Lapuz its Certificate of Insurance No. 128866. (Rollo, p. 28.) The policy was to be effective for a period of 90 days. On May 31, 1969 or during the effectivity of Certificate of Insurance No. 12886, Carmen O. Lapuz died in a vehicular accident in the North Diversion Road. On June 7, 1969, petitioner Regina L. Edillon, a sister of the insured and who was the named beneficiary in the policy, filed her claim for the proceeds of the insurance, submitting all the necessary papers and other requisites with the private respondent. Her claim having been denied, Regina L. Edillon instituted this action in the Court of First Instance of Rizal on August 27, 1969. In resisting the claim of the petitioner, the respondent insurance corporation relies on a provision contained in the Certificate of Insurance, excluding its liability to pay claims

under the policy in behalf of "persons who are under the age of sixteen (16) years of age or over the age of sixty (60) years ..." It is pointed out that the insured being over sixty (60) years of age when she applied for the insurance coverage, the policy was null and void, and no risk on the part of the respondent insurance corporation had arisen therefrom. The trial court sustained the contention of the private respondent and dismissed the complaint; ordered the petitioner to pay attorney's fees in the sum of ONE THOUSAND (P1,000.00) PESOS in favor of the private respondent; and ordered the private respondent to return the sum of TWENTY (P20.00) PESOS received by way of premium on the insurancy policy. It was reasoned out that a policy of insurance being a contract of adhesion, it was the duty of the insured to know the terms of the contract he or she is entering into; the insured in this case, upon learning from its terms that she could not have been qualified under the conditions stated in said contract, what she should have done is simply to ask for a refund of the premium that she paid. It was further argued by the trial court that the ruling calling for a liberal interpretation of an insurance contract in favor of the insured and strictly against the insurer may not be applied in the present case in view of the peculiar facts and circumstances obtaining therein. We REVERSE the judgment of the trial court. The age of the insured Carmen 0. Lapuz was not concealed to the insurance company. Her application for insurance coverage which was on a printed form furnished by private respondent and which contained very few items of information clearly indicated her age of the time of filing the same to be almost 65 years of age. Despite such information which could hardly be overlooked in the application form, considering its prominence thereon and its materiality to the coverage applied for, the respondent insurance corporation received her payment of premium and issued the corresponding certificate of insurance without question. The accident which resulted in the death of the insured, a risk covered by the policy, occurred on May 31, 1969 or FORTY-FIVE (45) DAYS after the insurance coverage was applied for. There was sufficient time for the private respondent to process the application and to notice that the applicant was over 60 years of age and thereby cancel the policy on that ground if it was minded to do so. If the private respondent failed to act, it is either because it was willing to waive such disqualification; or, through the negligence or incompetence of its employees for which it has only itself to blame, it simply overlooked such fact. Under the circumstances, the insurance corporation is already deemed in estoppel. It inaction to revoke the policy despite a departure from the exclusionary condition contained in the said policy constituted a waiver of such condition, as was held in the case of "Que Chee Gan vs. Law Union Insurance Co., Ltd.,", 98 Phil. 85. This case involved a claim on an insurance policy which contained a provision as to the installation of fire hydrants the number of which depended on the height of the external wan perimeter of the bodega that was insured. When it was determined that the bodega should have eleven (11) fire hydrants in the compound as required by the terms of the policy, instead of only two (2) that it had, the claim under the policy was resisted on that ground. In ruling that the said deviation from the terms of the policy did not prevent the claim under the same, this Court stated the following:

We are in agreement with the trial Court that the appellant is barred by waiver (or rather estoppel) to claim violation of the so-called fire hydrants warranty, for the reason that knowing fully an that the number of hydrants demanded therein never existed from the very beginning, the appellant nevertheless issued the policies in question subject to such warranty, and received the corresponding premiums. It would be perilously close to conniving at fraud upon the insured to allow appellant to claim now as void ab initio the policies that it had issued to the plaintiff without warning of their fatal defect, of which it was informed, and after it had misled the defendant into believing that the policies were effective. The insurance company was aware, even before the policies were issued, that in the premises insured there were only two fire hydrants installed by Que Chee Gan and two others nearby, owned by the municipality of Tabaco, contrary to the requirements of the warranty in question. Such fact appears from positive testimony for the insured that appellant's agents inspected the premises; and the simple denials of appellant's representative (Jamiczon) can not overcome that proof. That such inspection was made it moreover rendered probable by its being a prerequisite for the fixing of the discount on the premium to which the insured was entitled, since the discount depended on the number of hydrants, and the fire fighting equipment available (See"'Scale of Allowances" to which the policies were expressly made subject). The law, supported by a long line of cases, is expressed by American Jurisprudence (Vol. 29, pp. 611-612) to be as follows: It is usually held that where the insurer, at the time of the issuance of a policy of insurance, has knowledge of existing facts which, if insisted on, would invalidate the contract from its very inception, such knowledge constitutes a waiver of conditions in the contract inconsistent with the known facts, and the insurer is stopped thereafter from asserting the breach of such conditions. The law is charitable enough to assume, in the absence of any showing to the contrary, that an insurance company intends to execute a valid contract in return for the premium received; and when the policy contains a condition which renders it voidable at its inception, and this result is known to the insurer, it will be presumed to have intended to waive the conditions and to execute a binding contract, rather than to have deceived the insured into thinking he is insured when in fact he is not, and to have taken is money without consideration.' (29 Am. Jur., Insurance, section 807, at pp. 611-612.) The reason for the rule is not difficult to find. The plain, human justice of this doctrine is perfectly apparent. To allow a company to accept one's money for a policy of insurance which it then knows to be void and of no effect, though it knows as it must, that the assured believes it to be valid and binding, is so contrary to the dictates of honesty and fair dealing, and so closely related to positive fraud, as to be abhorent to fairminded men. It would be to allow the company to treat the policy as valid long enough to get the premium on it, and leave it at liberty to repudiate it the next moment. This cannot be deemed to be the real intention of the parties. To hold that a literal construction of the policy expressed the true intention of

the company would be to indict it, for fraudulent purposes and designs which we cannot believe it to be guilty of (Wilson vs. Commercial Union Assurance Co., 96 Atl. 540, 543544). A similar view was upheld in the case of Capital Insurance & Surety Co., Inc. vs. Plastic Era Co., Inc., 65 SCRA 134, which involved a violation of the provision of the policy requiring the payment of premiums before the insurance shall become effective. The company issued the policy upon the execution of a promissory note for the payment of the premium. A check given subsequent by the insured as partial payment of the premium was dishonored for lack of funds. Despite such deviation from the terms of the policy, the insurer was held liable. Significantly, in the case before Us the Capital Insurance accepted the promise of Plastic Era to pay the insurance premium within thirty (30) days from the effective date of policy. By so doing, it has impliedly agreed to modify the tenor of the insurance policy and in effect, waived the provision therein that it would only pay for the loss or damage in case the same occurs after the payment of the premium. Considering that the insurance policy is silent as to the mode of payment, Capital Insurance is deemed to have accepted the promissory note in payment of the premium. This rendered the policy immediately operative on the date it was delivered. The view taken in most cases in the United States: ... is that although one of conditions of an insurance policy is that "it shall not be valid or binding until the first premium is paid", if it is silent as to the mode of payment, promissory notes received by the company must be deemed to have been accepted in payment of the premium. In other words, a requirement for the payment of the first or initial premium in advance or actual cash may be waived by acceptance of a promissory note... WHEREFORE, the judgment appealed from is hereby REVERSED and SET ASIDE. In lieu thereof, the private respondent insurance corporation is hereby ordered to pay to the petitioner the sum of TEN THOUSAND (P10,000.00) PESOS as proceeds of Insurance Certificate No. 128866 with interest at the legal rate from May 31, 1969 until fully paid, the further sum of TWO THOUSAND (P2,000.00) PESOS as and for attorney's fees, and the costs of suit. SO ORDERED. Teehankee (Chairman), Makasiar, Plana, Relova and Gutierrez, Jr., JJ., concur. Melencio-Herrera, J., took no part.

G.R. No. L-27541

November 21, 1927

TAN CHAY HENG, plaintiff-appellee, vs. THE WEST COAST LIFE INSURANCE COMPANY, defendant-appellant. Gibbs and McDonough and Roman Ozaeta for appellant. Hilado and Hilado for appellee. STATEMENT Plaintiff alleges that he is of age and a resident of Bacolod, Occidental Negros; that the defendant is a foreign insurance corporation duly organized by the laws of the Philippines to engage in the insurance business, its main office of which is in the City of Manila; that in the month of April, 1925, on his application the defendant accepted and approved a life insurance policy of for the sum of P10,000 in which the plaintiff was the sole beneficiary; that the policy was issued upon the payment by the said Tan Ceang of the first year's premium amounting to P936; that in and by its terms, the defendant agreed to pay the plaintiff as beneficiary the amount of the policy upon the receipt of the proofs of the death of the insured while the policy was in force; that without any premium being due or unpaid, Tan Ceang died on May 10, 1925; that in June, 1925, plaintiff submitted the proofs of the death of Tan Ceang with a claim for the payment of the policy which the defendant refused to pay, for which he prays for a corresponding judgment, with legal interest from the date of the policy, and costs. In February, 1926, the defendant filed an answer to the complaint in which it made a general and specific denial, and then announced its intention to file an amended answer, alleging special defense, and on August 31, 1926, it filed the following: AMENDED ANSWER Comes now the defendant, by its undersigned attorneys, and with leave of court amends its answer to plaintiff's complaint herein by making it reads as follows: I That it admits paragraph 1 of said complaint. II That it denies each and every other allegation contained in each and every other paragraph of said complaint. SPECIAL DEFENSE

By way of special defense, defendant alleges: I That the insurance policy on the life of Tan Ceang, upon which plaintiff's action is based, was obtained by the plaintiff in confabulation with one Go Chulian, of Bacolod, Negros Occidental; Francisco Sanchez of the same place; and Dr. V. S. Locsin, of La Carlota, Negros Occidental, thru fraud and deceit perpetrated against this defendant in the following manner, to wit: 1. That on or about the 22d day of February, 1925, in the municipality of Pulupandan, Occidental Negros, the present plaintiff and the said Go Chulian, Francisco Sanchez and Dr., V. S. Locsin, conspiring and confederating together for the purpose of defrauding and cheating the defendant in the sum of P10,000, caused one Tan Caeng to sign an application for insurance with the defendant in the sum of P10,000, in which application it was falsely represented to the defendant that the said Tan Ceang was single and was a merchant, and that the plaintiff Tan Chai Heng, the beneficiary, was his nephew, whereas in truth and in fact and as the plaintiff and his said coconspirators well knew, the said Tan Ceang was not single but was legally married to Marcelina Patalita with whom he had several children; and that he was not a merchant but was a mere employee of another Chinaman by the name of Tan Quina from whom he received only a meager salary, and that the present plaintiff was not a nephew of the said Tan Ceang. 2. That on said date, February 22, 1925, the said Tan Ceang was seriously ill, suffering from pulmonary tuberculosis of about three years' duration, which illness was incurable and was well known to the plaintiff and his said coconspirators. 3. That on or about the same date, February 22, 1925, the said Dr. V. S. Locsin, in his capacity as medical examiner for the defendant insurance company, pursuant to the conspiracy above mentioned, prepared and falsified the necessary medical certificate, in which it was made to appear, among other things, that the said Tan Ceang had never used morphine, cocaine or any other drug; that he was then in good health and had never consulted any physician; that he had never spit blood; and that there was no sign of either present or past disease of his lungs; whereas in truth and in fact, as the plaintiff and his said coconspirators well knew, the said Tan Ceang was addicted to morphine, cocaine, and opium and had been convicted and imprisoned therefor, and was then, and for about three year prior thereto had been suffering from pulmonary tuberculosis. 4. That on or about the same date, to wit, February 22, 1925, the plaintiff and his said coconspirators, pursuant to the conspiracy above mentioned, cause a confidential report to the defendant insurance company to be signed by one V.

Sy Yock Kian, who was an employee of Go Chulian, in which confidential report, among other things, it was falsely represented to the defendant insurance company that the said Tan Ceang was worth about P40,000, had an annual income of from eight to ten thousand pesos net, had the appearance of good health, and never had tuberculosis; that the plaintiff and his said coconspirators well knew that said representations were false; and that they were made for the purpose of deceiving the defendant and inducing it to accept the said application for insurance. 5. That after the said application for insurance, medical certificate and confidential report had been prepared and falsified, as aforesaid, the plaintiff and his said coconspirators caused the same to be forwarded to the defendant at its office in Manila, the medical certificate thru the said Dr. V. S. Locsin as medical examiner, and said application for insurance and confidential report thru the said Francisco Sanchez in his capacity as one of the agents of the defendant insurance company in the Province of Occidental Negros; that the defendant, believing that the representations made in said document were true, and relying thereon, provisionally accepted the said application for insurance on the life of Tan Ceang in the sum of P10,000 and issued a temporary policy pending the final approval or disapproval of said application by defendant's home-office in San Francisco, California, where in case of approval a permanent policy was to be issued; that such permanent policy was never delivered to the plaintiff because defendant discovered the fraud before its delivery. 6. That the first agreed annual premium on the insurance in question of P936.50 not having been paid within sixty (60) days after the date of the supposed medical examination of the applicant as required by the regulations of the defendant insurance company, of which regulations the said Francisco Sanchez as agent of the defendant had knowledge, the plaintiff and his said coconspirators in order to secure the delivery to them of said temporary policy, and in accordance with said regulations of the defendant company, caused the said Tan Ceang on April 10, 1925 to sign the following document: lawphil.net WEST COAST LIFE INSURANCE COMPANY SAN FRANCISCO, CALIFORNIA HEALTH CERTIFICATE FOR RE-INSTATEMENT I herewith request the West Coast Life Insurance Company to re-instate Policy No. ............................. issued by it upon my life, the first unpaid premium on which became due .............................., 19................ I certify and state that I am now in good and sound health, that since the date of my examination under the application on which said policy was written, I have had no injury, sickness, impairment of health or symptom thereof, and that since said date I have neither consulted a physician nor made any application for

life insurance that has not been granted in exact kind and amount applied for, except: NADA (State fully all exceptions to all above statements. If no exceptions insert "NONE.") I agree that, if said policy re-instated, it shall be only on condition of the truth of the above statements and such re-instatement shall not operate as a waiver on the part of said Company of its right to refuse to accept any future overdue premiums or installments thereof. Witness: (Sgd.) TAN CHAI HENG TAN CAENG Signature of Applicant.

"Dated at Palupandan on this 10 day of April, 1925." that the statements and representations contained in the application for reinstatement above set forth with regard to the health and physical condition of the said Tan Ceang were false and known to the plaintiff and his said coconspirators to be false; that the said temporary policy was delivered by defendant to the insured on April 10, 1925, in the belief that said statements and representations were true and in reliance thereon. 7. That on May 10, 1925, that is to say, two months and a half after the supposed medical examination above referred to, and exactly one month after the date of the health certificate for reinstatement above set forth, the said Tan Ceang died in Valladolid, Occidental Negros, of pulmonary tuberculosis, the same illnes from which suffering at the time it is supposed he was examined by Dr. V. S. Locsin, but that the plaintiff and his said coconspirators, pursuant to their conspiracy, caused the said Dr. V. S. Locsin to state falsely in the certificate of death that the said Tan Ceang had died of cerebral hemorrhage. II That the plaintiff Tan Chai Heng, on the dates herein-above mentioned, was, liked V. Sy Yock Kian who signed the confidential report above mentioned, an employee of the said Go Chulian; that the latter was the ringleader of a gang of malefactors, who, during, and for some years previous to the dates above mentioned, were engaged in the illicit enterprise of procuring fraudulent life insurances from the present defendant, similar to the one in question, and which enterprise was capitalized by him by furnishing the funds with which to pay the premium on said fraudulent insurance; that the said Go Chulian was the one who furnished the money with which to pay the first and only annual premium on the insurance here in question, amounting to P936.50; that the said Go Chulian, on August 28, 1926, was convicted by the Court of First Instance of the City of

Manila, in criminal case No. 31425 of that court, of the crime of falsification of private documents in connection with an fraudulent insurance, similar to the present, committed against this defendant in the month of September, 1924; that in the same case the said Francisco Sanchez was one of the coaccused of the said Go Chulian but was discharged from the complaint, because he offered himself and was utilized as a state's witness; that there is another civil action now pending against Go Chulian and Sanchez in the Court of First Instance of Manila (civil case No. 28680), in which the present defendant is the plaintiff, for the recovery of the amounts of two insurance policies aggregating P19,000, fraudulently obtained by the said Go Chulian and Sanchez upon the lives of one Tan Deco, who was also suffering from and died of tuberculosis, and one Tan Anso, who was suffering from and died of beriberi. III That by reason of all the facts above set forth, the temporary policy of insurance on the life of Tan Caeng for the sum of P10,000 upon which the present action is base is null and void. Wherefore, defendant prays that it be absolved from plaintiff's complaint, with costs against the plaintiff. To this special defense, the plaintiff, claiming that it was a cross-complaint, filed a general demurrer upon the ground that it does not state facts sufficient to constitute a cause of defense. After exhaustive arguments and on September 16, 1926, the court rendered the following decision: After considering the demurrer filed by the plaintiff to the special defense contained in the amended answer of the defendant, dated August 31, 1926, without prejudice to writing a more extensive decision, said demurrer is sustained, and the defendant is given a period of five days within which to amend its aforesaid answer. So ordered. To which the defendant duly excepted. As a result of the trial the general issues, the lower court rendered judgment for the plaintiff for P10,000, with legal interest from January 4, 1926, and costs, to which the defendant duly excepted and filed a motion for a new trial, which was overruled. On appeal the defendant assigns the following errors: The trial court erred

1. In sustaining plaintiff's demurrer to the special defense contained in defendant's amended answer. 2. In holding, in effect, that an insurer cannot avoid a policy which had been procured by fraud unless he brings an action to rescind it before he is sued thereon. 3. In rejecting all proofs offered by the defendant during the trial for the purpose of defeating plaintiff's fraudulent claim. 4. In not absolving the defendant from plaintiff's complaint.

JOHNS, J.: It will thus be noted that the premium was paid on April 10, 1925, at which time the temporary policy was issued; that the plaintiff's action was commenced on January 4, 1926; that the original answer of the defendant, consisting of a general and specific denial, was filed on February 27, 1926; and that its amended answer was filed on August 31, 1926. Based upon those facts the plaintiff vigorously contended in the lower court and now contends in the court, that section 47 of the Insurance Act should be applied, and that when so applied, defendant is barred and estopped to plead and set forth the matters alleged in its special defense. That section is as follows: Whenever a right to rescind a contract of insurance is given to the insurer by any provision of this chapter, such right must be exercised previous to the commencement of an action on the contract. The defendant contended in the lower court and now contends in this court, that section 47 does not apply to the new matters alleged in the special defense. If in legal effect defendant's special defense is in the nature of an act to rescind "a contract of insurance," then such right must be exercised prior to an action enforce the contract. That is the real question involved in this appeal. Defendant's original answer was a general and specific denial. In other words, it specifically denied that if ever issued the policy in question, or that it ever agreed with Tan Ceang in the even of his death to pay P10,000 to the plaintiff or any one else. In its amended answer the defendant again makes a general and specific denial, and alleges the reasons, the specific facts, and the reasons why it never made or entered into the contract alleged in the complaint, and based upon those alleged facts, defendant contends that it never did enter into any contract of insurance on the life of Tan Caeng.

The word "rescind" has a well defined legal meaning, and as applied to contracts, it presupposes the existence of a contract to rescind. Word & Phrases, volume 7, page 6139, says: To rescind is to abrogate, annual, avoid, or cancel a contract. The word "rescind," as used in a statement by a party to a contrary as follows. "I hereby terminate and rescind my said written contract," is synonymous with the word `terminate,' and the rescission therefore relates only to the unfulfilled part, and not to the entire agreement, making the party rescinding liable on notes executed pursuant to the contract which matured before the rescission. The rescission is the unmaking of a contract, requiring the same concurrence of wills as that which made it, and nothing short of this will suffice. There is a wide difference between the rescission of a contract and its mere termination or cancellation. After a contract has been broken, whether by an inability to perform it, or by rescinding against right or otherwise, the party not in fault may sue the other for the damages suffered, or, if the parties can be placed in status quo, he may, should he prefer, return what he has received and recover in a suit value of what he has paid or done. The latter remedy is termed "rescission." In the instant case, it will be noted that even in its prayer, the defendant does not seek to have the alleged insurance contract rescinded. It denies that it ever made any contract of insurance on the life of Tan Ceang or that any such a contract ever existed, and that is the question which it seeks to have litigated by its special defense. In the very nature of things, if the defendant never made or entered into the contract in question, there is no contract to rescind, and, hence, section 47 upon which the lower based its decision in sustaining the demurrer does not apply. As stated, an action to rescind a contract is founded upon and presupposes the existence of the contract which is sought to be rescinded. If all of the material matters set forth and alleged in the defendant's special plea are true, there was no valid contract of insurance, for the simple reason that the minds of the parties never met and never agreed upon the terms and conditions of the contract. We are clearly of the opinion that, if such matters are known to exist by a preponderance of the evidence, they would constitute a valid defense to plaintiff's cause of action. Upon the question as to whether or not they or are not true, we do not at this time have or express any opinion, but we are clear that section 47 does not apply to the allegations made in the answer, and that the trial court erred in sustaining the demurrer. The judgment of the lower court is reversed and the case is remanded for such other and further proceedings as are not inconsistent with this opinion, with costs against the plaintiff. So ordered.

G.R. No. 48049 June 29, 1989

EMILIO TAN, JUANITO TAN, ALBERTO TAN and ARTURO TAN, petitioners, vs. THE COURT OF APPEALS and THE PHILIPPINE AMERICAN LIFE INSURANCE COMPANY, respondents.

O.F. Santos & P.C. Nolasco for petitioners.

Ferry, De la Rosa and Associates for private respondent.

GUTIERREZ, JR., J.:

This is a petition for review on certiorari of the Court of Appeals' decision affirming the decision of the Insurance Commissioner which dismissed the petitioners' complaint against respondent Philippine American Life Insurance Company for the recovery of the proceeds from their late father's policy. The facts of the case as found by the Court of Appeals are:

Petitioners appeal from the Decision of the Insurance Commissioner dismissing herein petitioners' complaint against respondent Philippine American Life Insurance Company for the recovery of the proceeds of Policy No. 1082467 in the amount of P 80,000.00.

On September 23,1973, Tan Lee Siong, father of herein petitioners, applied for life insurance in the amount of P 80,000.00 with respondent company. Said application was

approved and Policy No. 1082467 was issued effective November 6,1973, with petitioners the beneficiaries thereof (Exhibit A).

On April 26,1975, Tan Lee Siong died of hepatoma (Exhibit B). Petitioners then filed with respondent company their claim for the proceeds of the life insurance policy. However, in a letter dated September 11, 1975, respondent company denied petitioners' claim and rescinded the policy by reason of the alleged misrepresentation and concealment of material facts made by the deceased Tan Lee Siong in his application for insurance (Exhibit 3). The premiums paid on the policy were thereupon refunded .

Alleging that respondent company's refusal to pay them the proceeds of the policy was unjustified and unreasonable, petitioners filed on November 27, 1975, a complaint against the former with the Office of the Insurance Commissioner, docketed as I.C. Case No. 218.

After hearing the evidence of both parties, the Insurance Commissioner rendered judgment on August 9, 1977, dismissing petitioners' complaint. (Rollo, pp. 91-92)

The Court of Appeals dismissed ' the petitioners' appeal from the Insurance Commissioner's decision for lack of merit

Hence, this petition.

The petitioners raise the following issues in their assignment of errors, to wit:

A. The conclusion in law of respondent Court that respondent insurer has the right to rescind the policy contract when insured is already dead is not in accordance with existing law and applicable jurisprudence.

B. The conclusion in law of respondent Court that respondent insurer may be allowed to avoid the policy on grounds of concealment by the deceased assured, is contrary to the provisions of the policy contract itself, as well as, of applicable legal provisions and established jurisprudence.

C. The inference of respondent Court that respondent insurer was misled in issuing the policy are manifestly mistaken and contrary to admitted evidence. (Rollo, p. 7)

The petitioners contend that the respondent company no longer had the right to rescind the contract of insurance as rescission must allegedly be done during the lifetime of the insured within two years and prior to the commencement of action.

The contention is without merit.

The pertinent section in the Insurance Code provides:

Section 48. Whenever a right to rescind a contract of insurance is given to the insurer by any provision of this chapter, such right must be exercised previous to the commencement of an action on the contract.

After a policy of life insurance made payable on the death of the insured shall have been in force during the lifetime of the insured for a period of two years from the date of its issue or of its last reinstatement, the insurer cannot prove that the policy is void ab initio or is rescindable by reason of the fraudulent concealment or misrepresentation of the insured or his agent.

According to the petitioners, the Insurance Law was amended and the second paragraph of Section 48 added to prevent the insurance company from exercising a right to rescind after the death of the insured.

The so-called "incontestability clause" precludes the insurer from raising the defenses of false representations or concealment of material facts insofar as health and previous diseases are concerned if the insurance has been in force for at least two years during the insured's lifetime. The phrase "during the lifetime" found in Section 48 simply means that the policy is no longer considered in force after the insured has died. The key phrase in the second paragraph of Section 48 is "for a period of two years."

As noted by the Court of Appeals, to wit:

The policy was issued on November 6,1973 and the insured died on April 26,1975. The policy was thus in force for a period of only one year and five months. Considering that the insured died before the two-year period had lapsed, respondent company is not, therefore, barred from proving that the policy is void ab initio by reason of the insured's fraudulent concealment or misrepresentation. Moreover, respondent company rescinded the contract of insurance and refunded the premiums paid on September 11, 1975, previous to the commencement of this action on November 27,1975. (Rollo, pp. 99-100)

xxx xxx xxx

The petitioners contend that there could have been no concealment or misrepresentation by their late father because Tan Lee Siong did not have to buy insurance. He was only pressured by insistent salesmen to do so. The petitioners state:

Here then is a case of an assured whose application was submitted because of repeated visits and solicitations by the insurer's agent. Assured did not knock at the door of the insurer to buy insurance. He was the object of solicitations and visits.

Assured was a man of means. He could have obtained a bigger insurance, not just P 80,000.00. If his purpose were to misrepresent and to conceal his ailments in anticipation of death during the two-year period, he certainly could have gotten a bigger insurance. He did not.

Insurer Philamlife could have presented as witness its Medical Examiner Dr. Urbano Guinto. It was he who accomplished the application, Part II, medical. Philamlife did not.

Philamlife could have put to the witness stand its Agent Bienvenido S. Guinto, a relative to Dr. Guinto, Again Philamlife did not. (pp. 138139, Rollo)

xxx xxx xxx

This Honorable Supreme Court has had occasion to denounce the pressure and practice indulged in by agents in selling insurance. At one time or another most of us have been subjected to that pressure, that practice. This court took judicial cognizance of the whirlwind pressure of insurance selling-especially of the agent's practice of 'supplying the information, preparing and answering the application, submitting the application to their companies, concluding the transactions and otherwise smoothing out all difficulties.

We call attention to what this Honorable Court said in Insular Life v. Feliciano, et al., 73 Phil. 201; at page 205:

It is of common knowledge that the selling of insurance today is subjected to the whirlwind pressure of modern salesmanship.

Insurance companies send detailed instructions to their agents to solicit and procure applications.

These agents are to be found all over the length and breadth of the land. They are stimulated to more active efforts by contests and by the keen competition offered by the other rival insurance companies.

They supply all the information, prepare and answer the applications, submit the applications to their companies, conclude the transactions, and otherwise smooth out all difficulties.

The agents in short do what the company set them out to do.

The Insular Life case was decided some forty years ago when the pressure of insurance salesmanship was not overwhelming as it is now; when the population of this country was less than one-fourth of what it is now; when the insurance companies competing with one another could be counted by the fingers. (pp. 140-142, Rollo)

xxx xxx xxx

In the face of all the above, it would be unjust if, having been subjected to the whirlwind pressure of insurance salesmanship this Court itself has long denounced, the assured who dies within the two-year period, should stand charged of fraudulent concealment and misrepresentation." (p. 142, Rollo)

The legislative answer to the arguments posed by the petitioners is the "incontestability clause" added by the second paragraph of Section 48.

The insurer has two years from the date of issuance of the insurance contract or of its last reinstatement within which to contest the policy, whether or not, the insured still lives within such period. After two years, the defenses of concealment or misrepresentation, no matter how patent or well founded, no longer lie. Congress felt this was a sufficient answer to the various tactics employed by insurance companies to avoid liability. The petitioners' interpretation would give rise to the incongruous situation where the beneficiaries of an insured who dies right after taking out and paying for a life insurance policy, would be allowed to collect on the policy even if the insured fraudulently concealed material facts.

The petitioners argue that no evidence was presented to show that the medical terms were explained in a layman's language to the insured. They state that the insurer should have presented its two medical field examiners as witnesses. Moreover, the petitioners allege that the policy intends that the medical examination must be conducted before its issuance otherwise the insurer "waives whatever imperfection by ratification."

We agree with the Court of Appeals which ruled:

On the other hand, petitioners argue that no evidence was presented by respondent company to show that the questions appearing in Part II of the application for insurance were asked, explained to and understood by the deceased so as to prove concealment on his part. The same is not well taken. The deceased, by affixing his signature on the application form, affirmed the correctness of all the entries and answers appearing therein. It is but to be expected that he, a businessman, would not have affixed his signature on the application form unless he clearly understood its significance. For, the presumption is that a person intends the ordinary consequence of his voluntary act and takes ordinary care of his concerns. [Sec. 5(c) and (d), Rule 131, Rules of Court].

The evidence for respondent company shows that on September 19,1972, the deceased was examined by Dr. Victoriano Lim and was found to be diabetic and hypertensive; that by January, 1973, the deceased was complaining of progressive weight loss and abdominal pain and was diagnosed to be suffering from hepatoma, (t.s.n. August 23, 1976, pp. 8-10; Exhibit 2). Another physician, Dr. Wenceslao Vitug, testified that the deceased came to see him on December 14, 1973 for consolation and claimed to have been diabetic for five years. (t.s.n., Aug. 23,1976, p. 5; Exhibit 6) Because of the concealment made by the deceased of his consultations and treatments for hypertension, diabetes and liver disorders, respondent company was thus misled into accepting the risk and approving his application as medically standard (Exhibit 5- C) and dispensing with further medical investigation and examination (Exhibit 5-A). For as long as no adverse medical history is revealed in the application form, an applicant for insurance is presumed to be healthy and physically fit and no further medical investigation or examination is conducted by respondent company. (t.s.n., April 8,1976, pp. 6-8). (Rollo, pp. 96-98)

There is no strong showing that we should apply the "fine print" or "contract of adhesion" rule in this case. (Sweet Lines, Inc. v. Teves, 83 SCRA 361 [1978]). The petitioners cite:

It is a matter of common knowledge that large amounts of money are collected from ignorant persons by companies and associations which adopt high sounding titles and print the amount of benefits they agree to pay in large black-faced type, following such undertakings by fine print conditions which destroy the substance of the promise. All provisions, conditions, or exceptions which in any way tend to work a forfeiture of the policy should be construed most strongly against those for whose benefit they are inserted, and most favorably toward those against whom they are meant to operate. (Trinidad v. Orient Protective Assurance Assn., 67 Phil. 184)

There is no showing that the questions in the application form for insurance regarding the insured's medical history are in smaller print than the rest of the printed form or that they are designed in such a way as to conceal from the applicant their importance. If a warning in bold red letters or a boxed warning similar to that required for cigarette advertisements by the Surgeon General of the United States is necessary, that is for Congress or the Insurance Commission to provide as protection against high pressure insurance salesmanship. We are limited in this petition to ascertaining whether or not the respondent Court of Appeals committed reversible error. It is the petitioners' burden to show that the factual findings of the respondent court are not based on substantial evidence or that its conclusions are contrary to applicable law and jurisprudence. They have failed to discharge that burden.

WHEREFORE, the petition is hereby DENIED for lack of merit. The questioned decision of the Court of Appeals is AFFIRMED.

SO ORDERED.

Fernan, (C.J., Chairman), Bidin and Cortes, JJ., concur.

Feliciano, took no part.

G.R. No. 125678

March 18, 2002

PHILAMCARE HEALTH SYSTEMS, INC., petitioner, vs. COURT OF APPEALS and JULITA TRINOS, respondents. YNARES-SANTIAGO, J.: Ernani Trinos, deceased husband of respondent Julita Trinos, applied for a health care coverage with petitioner Philamcare Health Systems, Inc. In the standard application form, he answered no to the following question: Have you or any of your family members ever consulted or been treated for high blood pressure, heart trouble, diabetes, cancer, liver disease, asthma or peptic ulcer? (If Yes, give details).1 The application was approved for a period of one year from March 1, 1988 to March 1, 1989. Accordingly, he was issued Health Care Agreement No. P010194. Under the agreement, respondents husband was entitled to avail of hospitalization benefits, whether ordinary or emergency, listed therein. He was also entitled to avail of "outpatient benefits" such as annual physical examinations, preventive health care and other out-patient services. Upon the termination of the agreement, the same was extended for another year from March 1, 1989 to March 1, 1990, then from March 1, 1990 to June 1, 1990. The amount of coverage was increased to a maximum sum of P75,000.00 per disability.2 During the period of his coverage, Ernani suffered a heart attack and was confined at the Manila Medical Center (MMC) for one month beginning March 9, 1990. While her husband was in the hospital, respondent tried to claim the benefits under the health care agreement. However, petitioner denied her claim saying that the Health Care Agreement was void. According to petitioner, there was a concealment regarding Ernanis medical history. Doctors at the MMC allegedly discovered at the time of Ernanis confinement that he was hypertensive, diabetic and asthmatic, contrary to his answer in the application form. Thus, respondent paid the hospitalization expenses herself, amounting to about P76,000.00. After her husband was discharged from the MMC, he was attended by a physical therapist at home. Later, he was admitted at the Chinese General Hospital. Due to financial difficulties, however, respondent brought her husband home again. In the morning of April 13, 1990, Ernani had fever and was feeling very weak. Respondent was constrained to bring him back to the Chinese General Hospital where he died on the same day. On July 24, 1990, respondent instituted with the Regional Trial Court of Manila, Branch 44, an action for damages against petitioner and its president, Dr. Benito Reverente,

which was docketed as Civil Case No. 90-53795. She asked for reimbursement of her expenses plus moral damages and attorneys fees. After trial, the lower court ruled against petitioners, viz: WHEREFORE, in view of the forgoing, the Court renders judgment in favor of the plaintiff Julita Trinos, ordering: 1. Defendants to pay and reimburse the medical and hospital coverage of the late Ernani Trinos in the amount of P76,000.00 plus interest, until the amount is fully paid to plaintiff who paid the same; 2. Defendants to pay the reduced amount of moral damages of P10,000.00 to plaintiff; 3. Defendants to pay the reduced amount of P10,000.00 as exemplary damages to plaintiff; 4. Defendants to pay attorneys fees of P20,000.00, plus costs of suit. SO ORDERED.3 On appeal, the Court of Appeals affirmed the decision of the trial court but deleted all awards for damages and absolved petitioner Reverente.4 Petitioners motion for reconsideration was denied.5 Hence, petitioner brought the instant petition for review, raising the primary argument that a health care agreement is not an insurance contract; hence the "incontestability clause" under the Insurance Code 6 does not apply.1wphi1.nt Petitioner argues that the agreement grants "living benefits," such as medical check-ups and hospitalization which a member may immediately enjoy so long as he is alive upon effectivity of the agreement until its expiration one-year thereafter. Petitioner also points out that only medical and hospitalization benefits are given under the agreement without any indemnification, unlike in an insurance contract where the insured is indemnified for his loss. Moreover, since Health Care Agreements are only for a period of one year, as compared to insurance contracts which last longer,7 petitioner argues that the incontestability clause does not apply, as the same requires an effectivity period of at least two years. Petitioner further argues that it is not an insurance company, which is governed by the Insurance Commission, but a Health Maintenance Organization under the authority of the Department of Health. Section 2 (1) of the Insurance Code defines a contract of insurance as an agreement whereby one undertakes for a consideration to indemnify another against loss, damage or liability arising from an unknown or contingent event. An insurance contract exists where the following elements concur: 1. The insured has an insurable interest;

2. The insured is subject to a risk of loss by the happening of the designated peril; 3. The insurer assumes the risk; 4. Such assumption of risk is part of a general scheme to distribute actual losses among a large group of persons bearing a similar risk; and 5. In consideration of the insurers promise, the insured pays a premium. 8 Section 3 of the Insurance Code states that any contingent or unknown event, whether past or future, which may damnify a person having an insurable interest against him, may be insured against. Every person has an insurable interest in the life and health of himself. Section 10 provides: Every person has an insurable interest in the life and health: (1) of himself, of his spouse and of his children; (2) of any person on whom he depends wholly or in part for education or support, or in whom he has a pecuniary interest; (3) of any person under a legal obligation to him for the payment of money, respecting property or service, of which death or illness might delay or prevent the performance; and (4) of any person upon whose life any estate or interest vested in him depends. In the case at bar, the insurable interest of respondents husband in obtaining the health care agreement was his own health. The health care agreement was in the nature of non-life insurance, which is primarily a contract of indemnity.9 Once the member incurs hospital, medical or any other expense arising from sickness, injury or other stipulated contingent, the health care provider must pay for the same to the extent agreed upon under the contract. Petitioner argues that respondents husband concealed a material fact in his application. It appears that in the application for health coverage, petitioners required respondents husband to sign an express authorization for any person, organization or entity that has any record or knowledge of his health to furnish any and all information relative to any hospitalization, consultation, treatment or any other medical advice or examination. 10 Specifically, the Health Care Agreement signed by respondents husband states: We hereby declare and agree that all statement and answers contained herein and in any addendum annexed to this application are full, complete and true and bind all parties in interest under the Agreement herein applied for, that there shall be no contract of health care coverage unless and until an Agreement is issued

on this application and the full Membership Fee according to the mode of payment applied for is actually paid during the lifetime and good health of proposed Members; that no information acquired by any Representative of PhilamCare shall be binding upon PhilamCare unless set out in writing in the application; that any physician is, by these presents, expressly authorized to disclose or give testimony at anytime relative to any information acquired by him in his professional capacity upon any question affecting the eligibility for health care coverage of the Proposed Members and that the acceptance of any Agreement issued on this application shall be a ratification of any correction in or addition to this application as stated in the space for Home Office Endorsement.11 (Underscoring ours) In addition to the above condition, petitioner additionally required the applicant for authorization to inquire about the applicants medical history, thus: I hereby authorize any person, organization, or entity that has any record or knowledge of my health and/or that of __________ to give to the PhilamCare Health Systems, Inc. any and all information relative to any hospitalization, consultation, treatment or any other medical advice or examination. This authorization is in connection with the application for health care coverage only. A photographic copy of this authorization shall be as valid as the original. 12 (Underscoring ours) Petitioner cannot rely on the stipulation regarding "Invalidation of agreement" which reads: Failure to disclose or misrepresentation of any material information by the member in the application or medical examination, whether intentional or unintentional, shall automatically invalidate the Agreement from the very beginning and liability of Philamcare shall be limited to return of all Membership Fees paid. An undisclosed or misrepresented information is deemed material if its revelation would have resulted in the declination of the applicant by Philamcare or the assessment of a higher Membership Fee for the benefit or benefits applied for.13 The answer assailed by petitioner was in response to the question relating to the medical history of the applicant. This largely depends on opinion rather than fact, especially coming from respondents husband who was not a medical doctor. Where matters of opinion or judgment are called for, answers made in good faith and without intent to deceive will not avoid a policy even though they are untrue.14 Thus, (A)lthough false, a representation of the expectation, intention, belief, opinion, or judgment of the insured will not avoid the policy if there is no actual fraud in inducing the acceptance of the risk, or its acceptance at a lower rate of premium, and this is likewise the rule although the statement is material to the risk, if the statement is obviously of the foregoing character, since in such case the insurer

is not justified in relying upon such statement, but is obligated to make further inquiry. There is a clear distinction between such a case and one in which the insured is fraudulently and intentionally states to be true, as a matter of expectation or belief, that which he then knows, to be actually untrue, or the impossibility of which is shown by the facts within his knowledge, since in such case the intent to deceive the insurer is obvious and amounts to actual fraud. 15 (Underscoring ours) The fraudulent intent on the part of the insured must be established to warrant rescission of the insurance contract.16 Concealment as a defense for the health care provider or insurer to avoid liability is an affirmative defense and the duty to establish such defense by satisfactory and convincing evidence rests upon the provider or insurer. In any case, with or without the authority to investigate, petitioner is liable for claims made under the contract. Having assumed a responsibility under the agreement, petitioner is bound to answer the same to the extent agreed upon. In the end, the liability of the health care provider attaches once the member is hospitalized for the disease or injury covered by the agreement or whenever he avails of the covered benefits which he has prepaid. Under Section 27 of the Insurance Code, "a concealment entitles the injured party to rescind a contract of insurance." The right to rescind should be exercised previous to the commencement of an action on the contract.17 In this case, no rescission was made. Besides, the cancellation of health care agreements as in insurance policies require the concurrence of the following conditions: 1. Prior notice of cancellation to insured; 2. Notice must be based on the occurrence after effective date of the policy of one or more of the grounds mentioned; 3. Must be in writing, mailed or delivered to the insured at the address shown in the policy; 4. Must state the grounds relied upon provided in Section 64 of the Insurance Code and upon request of insured, to furnish facts on which cancellation is based. 18 None of the above pre-conditions was fulfilled in this case. When the terms of insurance contract contain limitations on liability, courts should construe them in such a way as to preclude the insurer from non-compliance with his obligation.19 Being a contract of adhesion, the terms of an insurance contract are to be construed strictly against the party which prepared the contract the insurer.20 By reason of the exclusive control of the insurance company over the terms and phraseology of the insurance contract, ambiguity must be strictly interpreted against the insurer and liberally in favor of the insured, especially to avoid forfeiture.21 This is equally applicable to Health Care Agreements. The phraseology used in medical or hospital service contracts, such as the one at bar, must be liberally construed in favor of the subscriber, and if doubtful or

reasonably susceptible of two interpretations the construction conferring coverage is to be adopted, and exclusionary clauses of doubtful import should be strictly construed against the provider.22 Anent the incontestability of the membership of respondents husband, we quote with approval the following findings of the trial court: (U)nder the title Claim procedures of expenses, the defendant Philamcare Health Systems Inc. had twelve months from the date of issuance of the Agreement within which to contest the membership of the patient if he had previous ailment of asthma, and six months from the issuance of the agreement if the patient was sick of diabetes or hypertension. The periods having expired, the defense of concealment or misrepresentation no longer lie.23 Finally, petitioner alleges that respondent was not the legal wife of the deceased member considering that at the time of their marriage, the deceased was previously married to another woman who was still alive. The health care agreement is in the nature of a contract of indemnity. Hence, payment should be made to the party who incurred the expenses. It is not controverted that respondent paid all the hospital and medical expenses. She is therefore entitled to reimbursement. The records adequately prove the expenses incurred by respondent for the deceaseds hospitalization, medication and the professional fees of the attending physicians. 24 WHEREFORE, in view of the foregoing, the petition is DENIED. The assailed decision of the Court of Appeals dated December 14, 1995 is AFFIRMED. SO ORDERED. Davide, Jr., C.J., Puno, and Kapunan, JJ., concur.

G.R. No. L-4611

December 17, 1955

QUA CHEE GAN, plaintiff-appellee, vs. LAW UNION AND ROCK INSURANCE CO., LTD., represented by its agent, WARNER, BARNES AND CO., LTD., defendant-appellant. Delgado, Flores & Macapagal for appellant. Andres Aguilar, Zacarias Gutierrez Lora, Gregorio Sabater and Perkins, Ponce Enrile & Contreras for appellee.

REYES, J. B. L., J.: Qua Chee Gan, a merchant of Albay, instituted this action in 1940, in the Court of First Instance of said province, seeking to recover the proceeds of certain fire insurance policies totalling P370,000, issued by the Law Union & Rock Insurance Co., Ltd., upon certain bodegas and merchandise of the insured that were burned on June 21, 1940. The records of the original case were destroyed during the liberation of the region, and were reconstituted in 1946. After a trial that lasted several years, the Court of First Instance rendered a decision in favor of the plaintiff, the dispositive part whereof reads as follows: Wherefore, judgment is rendered for the plaintiff and against the defendant condemning the latter to pay the former (a) Under the first cause of action, the sum of P146,394.48; (b) Under the second cause of action, the sum of P150,000; (c) Under the third cause of action, the sum of P5,000; (d) Under the fourth cause of action, the sum of P15,000; and (e) Under the fifth cause of action, the sum of P40,000; all of which shall bear interest at the rate of 8% per annum in accordance with Section 91 (b) of the Insurance Act from September 26, 1940, until each is paid, with costs against the defendant. The complaint in intervention of the Philippine National Bank is dismissed without costs. (Record on Appeal, 166-167.) From the decision, the defendant Insurance Company appealed directly to this Court.

The record shows that before the last war, plaintiff-appellee owned four warehouses or bodegas (designated as Bodegas Nos. 1 to 4) in the municipality of Tabaco, Albay, used for the storage of stocks of copra and of hemp, baled and loose, in which the appellee dealth extensively. They had been, with their contents, insured with the defendant Company since 1937, and the lose made payable to the Philippine National Bank as mortgage of the hemp and crops, to the extent of its interest. On June, 1940, the insurance stood as follows: Policy No. 2637164 (Exhibit "LL") Property Insured Bodega No. 1 (Building) Bodega No. 2 (Building) 2637165 (Exhibit "JJ") Bodega No. 3 (Building) Bodega No. 4 (Building) Hemp Press moved by steam engine 2637345 (Exhibit "X") 2637346 (Exhibit "Y") 2637067 (Exhibit "GG") Total Merchandise contents (copra and empty sacks of Bodega No. 1) Merchandise contents (hemp) of Bodega No. 3 Merchandise contents (loose hemp) of Bodega No. 4 Amount P15,000.00 10,000.00 25,000.00 10,000.00 5,000.00 150,000.00 150,000.00 5,000.00

P370,000.00

Fire of undetermined origin that broke out in the early morning of July 21, 1940, and lasted almost one week, gutted and completely destroyed Bodegas Nos. 1, 2 and 4, with the merchandise stored theren. Plaintiff-appellee informed the insurer by telegram on the same date; and on the next day, the fire adjusters engaged by appellant insurance company arrived and proceeded to examine and photograph the premises, pored over the books of the insured and conducted an extensive investigation. The plaintiff having submitted the corresponding fire claims, totalling P398,562.81 (but reduced to the full amount of the insurance, P370,000), the Insurance Company resisted payment, claiming violation of warranties and conditions, filing of fraudulent claims, and that the fire had been deliberately caused by the insured or by other persons in connivance with him. With counsel for the insurance company acting as private prosecutor, Que Chee Gan, with his brother, Qua Chee Pao, and some employees of his, were indicted and tried in 1940 for the crime of arson, it being claimed that they had set fire to the destroyed warehouses to collect the insurance. They were, however, acquitted by the trial court in

a final decision dated July 9, 1941 (Exhibit WW). Thereafter, the civil suit to collect the insurance money proceeded to its trial and termination in the Court below, with the result noted at the start of this opinion. The Philippine National Bank's complaint in intervention was dismissed because the appellee had managed to pay his indebtedness to the Bank during the pendecy of the suit, and despite the fire losses. In its first assignment of error, the insurance company alleges that the trial Court should have held that the policies were avoided for breach of warranty, specifically the one appearing on a rider pasted (with other similar riders) on the face of the policies (Exhibits X, Y, JJ and LL). These riders were attached for the first time in 1939, and the pertinent portions read as follows: Memo. of Warranty. The undernoted Appliances for the extinction of fire being kept on the premises insured hereby, and it being declared and understood that there is an ample and constant water supply with sufficient pressure available at all seasons for the same, it is hereby warranted that the said appliances shall be maintained in efficient working order during the currency of this policy, by reason whereof a discount of 2 1/2 per cent is allowed on the premium chargeable under this policy. Hydrants in the compound, not less in number than one for each 150 feet of external wall measurement of building, protected, with not less than 100 feet of hose piping and nozzles for every two hydrants kept under cover in convenient places, the hydrants being supplied with water pressure by a pumping engine, or from some other source, capable of discharging at the rate of not less than 200 gallons of water per minute into the upper story of the highest building protected, and a trained brigade of not less than 20 men to work the same.' It is argued that since the bodegas insured had an external wall perimeter of 500 meters or 1,640 feet, the appellee should have eleven (11) fire hydrants in the compound, and that he actually had only two (2), with a further pair nearby, belonging to the municipality of Tabaco. We are in agreement with the trial Court that the appellant is barred by waiver (or rather estoppel) to claim violation of the so-called fire hydrants warranty, for the reason that knowing fully all that the number of hydrants demanded therein never existed from the very beginning, the appellant neverthless issued the policies in question subject to such warranty, and received the corresponding premiums. It would be perilously close to conniving at fraud upon the insured to allow appellant to claims now as void ab initio the policies that it had issued to the plaintiff without warning of their fatal defect, of which it was informed, and after it had misled the defendant into believing that the policies were effective. The insurance company was aware, even before the policies were issued, that in the premises insured there were only two fire hydrants installed by Qua Chee Gan and two others nearby, owned by the municipality of TAbaco, contrary to the requirements of the

warranty in question. Such fact appears from positive testimony for the insured that appellant's agents inspected the premises; and the simple denials of appellant's representative (Jamiczon) can not overcome that proof. That such inspection was made is moreover rendered probable by its being a prerequisite for the fixing of the discount on the premium to which the insured was entitled, since the discount depended on the number of hydrants, and the fire fighting equipment available (See "Scale of Allowances" to which the policies were expressly made subject). The law, supported by a long line of cases, is expressed by American Jurisprudence (Vol. 29, pp. 611-612) to be as follows: It is usually held that where the insurer, at the time of the issuance of a policy of insurance, has knowledge of existing facts which, if insisted on, would invalidate the contract from its very inception, such knowledge constitutes a waiver of conditions in the contract inconsistent with the facts, and the insurer is stopped thereafter from asserting the breach of such conditions. The law is charitable enough to assume, in the absence of any showing to the contrary, that an insurance company intends to executed a valid contract in return for the premium received; and when the policy contains a condition which renders it voidable at its inception, and this result is known to the insurer, it will be presumed to have intended to waive the conditions and to execute a binding contract, rather than to have deceived the insured into thinking he is insured when in fact he is not, and to have taken his money without consideration. (29 Am. Jur., Insurance, section 807, at pp. 611-612.) The reason for the rule is not difficult to find. The plain, human justice of this doctrine is perfectly apparent. To allow a company to accept one's money for a policy of insurance which it then knows to be void and of no effect, though it knows as it must, that the assured believes it to be valid and binding, is so contrary to the dictates of honesty and fair dealing, and so closely related to positive fraud, as to the abhorent to fairminded men. It would be to allow the company to treat the policy as valid long enough to get the preium on it, and leave it at liberty to repudiate it the next moment. This cannot be deemed to be the real intention of the parties. To hold that a literal construction of the policy expressed the true intention of the company would be to indict it, for fraudulent purposes and designs which we cannot believe it to be guilty of (Wilson vs. Commercial Union Assurance Co., 96 Atl. 540, 543-544). The inequitableness of the conduct observed by the insurance company in this case is heightened by the fact that after the insured had incurred the expense of installing the two hydrants, the company collected the premiums and issued him a policy so worded that it gave the insured a discount much smaller than that he was normaly entitledto. According to the "Scale of Allowances," a policy subject to a warranty of the existence of one fire hydrant for every 150 feet of external wall entitled the insured to a discount of 7 1/2 per cent of the premium; while the existence of "hydrants, in compund" (regardless of number) reduced the allowance on the premium to a mere 2 1/2 per cent.

This schedule was logical, since a greater number of hydrants and fire fighting appliances reduced the risk of loss. But the appellant company, in the particular case now before us, so worded the policies that while exacting the greater number of fire hydrants and appliances, it kept the premium discount at the minimum of 2 1/2 per cent, thereby giving the insurance company a double benefit. No reason is shown why appellant's premises, that had been insured with appellant for several years past, suddenly should be regarded in 1939 as so hazardous as to be accorded a treatment beyond the limits of appellant's own scale of allowances. Such abnormal treatment of the insured strongly points at an abuse of the insurance company's selection of the words and terms of the contract, over which it had absolute control. These considerations lead us to regard the parol evidence rule, invoked by the appellant as not applicable to the present case. It is not a question here whether or not the parties may vary a written contract by oral evidence; but whether testimony is receivable so that a party may be, by reason of inequitable conduct shown, estopped from enforcing forfeitures in its favor, in order to forestall fraud or imposition on the insured. Receipt of Premiums or Assessments afte Cause for Forfeiture Other than Nonpayment. It is a well settled rule of law that an insurer which with knowledge of facts entitling it to treat a policy as no longer in force, receives and accepts a preium on the policy, estopped to take advantage of the forfeiture. It cannot treat the policy as void for the purpose of defense to an action to recover for a loss thereafter occurring and at the same time treat it as valid for the purpose of earning and collecting further premiums." (29 Am. Jur., 653, p. 657.) It would be unconscionable to permit a company to issue a policy under circumstances which it knew rendered the policy void and then to accept and retain premiums under such a void policy. Neither law nor good morals would justify such conduct and the doctrine of equitable estoppel is peculiarly applicable to the situation. (McGuire vs. Home Life Ins. Co. 94 Pa. Super Ct. 457.) Moreover, taking into account the well known rule that ambiguities or obscurities must be strictly interpreted aganst the prty that caused them, 1the "memo of warranty" invoked by appellant bars the latter from questioning the existence of the appliances called for in the insured premises, since its initial expression, "the undernoted appliances for the extinction of fire being kept on the premises insured hereby, . . . it is hereby warranted . . .", admists of interpretation as an admission of the existence of such appliances which appellant cannot now contradict, should the parol evidence rule apply. The alleged violation of the warranty of 100 feet of fire hose for every two hydrants, must be equally rejected, since the appellant's argument thereon is based on the assumption that the insured was bound to maintain no less than eleven hydrants (one per 150 feet of wall), which requirement appellant is estopped from enforcing. The supposed breach of the wter pressure condition is made to rest on the testimony of

witness Serra, that the water supply could fill a 5-gallon can in 3 seconds; appellant thereupon inferring that the maximum quantity obtainable from the hydrants was 100 gallons a minute, when the warranty called for 200 gallons a minute. The transcript shows, however, that Serra repeatedly refused and professed inability to estimate the rate of discharge of the water, and only gave the "5-gallon per 3-second" rate because the insistence of appellant's counsel forced the witness to hazard a guess. Obviously, the testimony is worthless and insufficient to establish the violation claimed, specially since the burden of its proof lay on appellant. As to maintenance of a trained fire brigade of 20 men, the record is preponderant that the same was organized, and drilled, from time to give, altho not maintained as a permanently separate unit, which the warranty did not require. Anyway, it would be unreasonable to expect the insured to maintain for his compound alone a fire fighting force that many municipalities in the Islands do not even possess. There is no merit in appellant's claim that subordinate membership of the business manager (Co Cuan) in the fire brigade, while its direction was entrusted to a minor employee unders the testimony improbable. A business manager is not necessarily adept at fire fighting, the qualities required being different for both activities. Under the second assignment of error, appellant insurance company avers, that the insured violated the "Hemp Warranty" provisions of Policy No. 2637165 (Exhibit JJ), against the storage of gasoline, since appellee admitted that there were 36 cans (latas) of gasoline in the building designed as "Bodega No. 2" that was a separate structure not affected by the fire. It is well to note that gasoline is not specifically mentioned among the prohibited articles listed in the so-called "hemp warranty." The cause relied upon by the insurer speaks of "oils (animal and/or vegetable and/or mineral and/or their liquid products having a flash point below 300o Fahrenheit", and is decidedly ambiguous and uncertain; for in ordinary parlance, "Oils" mean "lubricants" and not gasoline or kerosene. And how many insured, it may well be wondered, are in a position to understand or determine "flash point below 003o Fahrenheit. Here, again, by reason of the exclusive control of the insurance company over the terms and phraseology of the contract, the ambiguity must be held strictly against the insurer and liberraly in favor of the insured, specially to avoid a forfeiture (44 C. J. S., pp. 1166-1175; 29 Am. Jur. 180). Insurance is, in its nature, complex and difficult for the layman to understand. Policies are prepared by experts who know and can anticipate the hearing and possible complications of every contingency. So long as insurance companies insist upon the use of ambiguous, intricate and technical provisions, which conceal rather than frankly disclose, their own intentions, the courts must, in fairness to those who purchase insurance, construe every ambiguity in favor of the insured. (Algoe vs. Pacific Mut. L. Ins. Co., 91 Wash. 324, LRA 1917A, 1237.) An insurer should not be allowed, by the use of obscure phrases and exceptions, to defeat the very purpose for which the policy was procured (Moore vs. Aetna Life Insurance Co., LRA 1915D, 264).

We see no reason why the prohibition of keeping gasoline in the premises could not be expressed clearly and unmistakably, in the language and terms that the general public can readily understand, without resort to obscure esoteric expression (now derisively termed "gobbledygook"). We reiterate the rule stated in Bachrach vs. British American Assurance Co. (17 Phil. 555, 561): If the company intended to rely upon a condition of that character, it ought to have been plainly expressed in the policy. This rigid application of the rule on ambiguities has become necessary in view of current business practices. The courts cannot ignore that nowadays monopolies, cartels and concentrations of capital, endowed with overwhelming economic power, manage to impose upon parties dealing with them cunningly prepared "agreements" that the weaker party may not change one whit, his participation in the "agreement" being reduced to the alternative to take it or leave it" labelled since Raymond Baloilles" contracts by adherence" (con tracts d'adhesion), in contrast to these entered into by parties bargaining on an equal footing, such contracts (of which policies of insurance and international bills of lading are prime examples) obviously call for greater strictness and vigilance on the part of courts of justice with a view to protecting the weaker party from abuses and imposition, and prevent their becoming traps for the unwarry (New Civil Coee, Article 24; Sent. of Supreme Court of Spain, 13 Dec. 1934, 27 February 1942). Si pudiera estimarse que la condicion 18 de la poliza de seguro envolvia alguna oscuridad, habra de ser tenido en cuenta que al seguro es, practicamente un contrato de los llamados de adhesion y por consiguiente en caso de duda sobre la significacion de las clausulas generales de una poliza redactada por las compafijas sin la intervencion alguna de sus clientes se ha de adoptar de acuerdo con el articulo 1268 del Codigo Civil, la interpretacion mas favorable al asegurado, ya que la obscuridad es imputable a la empresa aseguradora, que debia haberse explicado mas claramante. (Dec. Trib. Sup. of Spain 13 Dec. 1934) The contract of insurance is one of perfect good faith (uferrimal fidei) not for the insured alone, but equally so for the insurer; in fact, it is mere so for the latter, since its dominant bargaining position carries with it stricter responsibility. Another point that is in favor of the insured is that the gasoline kept in Bodega No. 2 was only incidental to his business, being no more than a customary 2 day's supply for the five or six motor vehicles used for transporting of the stored merchandise (t. s. n., pp. 1447-1448). "It is well settled that the keeping of inflammable oils on the premises though prohibited by the policy does not void it if such keeping is incidental to the business." Bachrach vs. British American Ass. Co., 17 Phil. 555, 560); and "according to the weight of authority, even though there are printed prohibitions against keeping certain articles on the insured premises the policy will not be avoided by a violation of these prohibitions, if the prohibited articles are necessary or in customary use in

carrying on the trade or business conducted on the premises." (45 C. J. S., p. 311; also 4 Couch on Insurance, section 966b). It should also be noted that the "Hemp Warranty" forbade storage only "in the building to which this insurance applies and/or in any building communicating therewith", and it is undisputed that no gasoline was stored in the burned bodegas, and that "Bodega No. 2" which was not burned and where the gasoline was found, stood isolated from the other insured bodegas. The charge that the insured failed or refused to submit to the examiners of the insurer the books, vouchers, etc. demanded by them was found unsubstantiated by the trial Court, and no reason has been shown to alter this finding. The insured gave the insurance examiner all the date he asked for (Exhibits AA, BB, CCC and Z), and the examiner even kept and photographed some of the examined books in his possession. What does appear to have been rejected by the insured was the demand that he should submit "a list of all books, vouchers, receipts and other records" (Age 4, Exhibit 9-c); but the refusal of the insured in this instance was well justified, since the demand for a list of all the vouchers (which were not in use by the insured) and receipts was positively unreasonable, considering that such listing was superfluous because the insurer was not denied access to the records, that the volume of Qua Chee Gan's business ran into millions, and that the demand was made just after the fire when everything was in turmoil. That the representatives of the insurance company were able to secure all the date they needed is proved by the fact that the adjuster Alexander Stewart was able to prepare his own balance sheet (Exhibit L of the criminal case) that did not differ from that submitted by the insured (Exhibit J) except for the valuation of the merchandise, as expressly found by the Court in the criminal case for arson. (Decision, Exhibit WW). How valuations may differ honestly, without fraud being involved, was strikingly illustrated in the decision of the arson case (Exhibit WW) acquiting Qua Choc Gan, appellee in the present proceedings. The decision states (Exhibit WW, p. 11): Alexander D. Stewart declaro que ha examinado los libros de Qua Choc Gan en Tabaco asi como su existencia de copra y abaca en las bodega al tiempo del incendio durante el periodo comprendido desde el 1.o de enero al 21 de junio de 1940 y ha encontrado que Qua Choc Gan ha sufrico una perdida de P1,750.76 en su negocio en Tabaco. Segun Steward al llegar a este conclusion el ha tenidoen cuenta el balance de comprobacion Exhibit 'J' que le ha entregado el mismo acusado Que Choc Gan en relacion con sus libros y lo ha encontrado correcto a excepcion de los precios de abaca y copra que alli aparecen que no estan de acuerdo con los precios en el mercado. Esta comprobacion aparece en el balance mercado exhibit J que fue preparado por el mismo testigo. In view of the discrepancy in the valuations between the insured and the adjuster Stewart for the insurer, the Court referred the controversy to a government auditor, Apolonio Ramos; but the latter reached a different result from the other two. Not only that, but Ramos reported two different valuations that could be reached according to the methods employed (Exhibit WW, p. 35):

La ciencia de la contabilidad es buena, pues ha tenido sus muchos usos buenos para promovar el comercio y la finanza, pero en el caso presente ha resultado un tanto cumplicada y acomodaticia, como lo prueba el resultado del examen hecho por los contadores Stewart y Ramos, pues el juzgado no alcanza a ver como habiendo examinado las mismas partidas y los mismos libros dichos contadores hayan de llegara dos conclusiones que difieron sustancialmente entre si. En otras palabras, no solamente la comprobacion hecha por Stewart difiere de la comprobacion hecha por Ramos sino que, segun este ultimo, su comprobacion ha dado lugar a dos resultados diferentes dependiendo del metodo que se emplea. Clearly then, the charge of fraudulent overvaluation cannot be seriously entertained. The insurer attempted to bolster its case with alleged photographs of certain pages of the insurance book (destroyed by the war) of insured Qua Chee Gan (Exhibits 26-A and 26-B) and allegedly showing abnormal purchases of hemp and copra from June 11 to June 20, 1940. The Court below remained unconvinced of the authenticity of those photographs, and rejected them, because they were not mentioned not introduced in the criminal case; and considering the evident importance of said exhibits in establishing the motive of the insured in committing the arson charged, and the absence of adequate explanation for their omission in the criminal case, we cannot say that their rejection in the civil case constituted reversible error. The next two defenses pleaded by the insurer, that the insured connived at the loss and that the fraudulently inflated the quantity of the insured stock in the burnt bodegas, are closely related to each other. Both defenses are predicted on the assumption that the insured was in financial difficulties and set the fire to defraud the insurance company, presumably in order to pay off the Philippine National Bank, to which most of the insured hemp and copra was pledged. Both defenses are fatally undermined by the established fact that, notwithstanding the insurer's refusal to pay the value of the policies the extensive resources of the insured (Exhibit WW) enabled him to pay off the National Bank in a short time; and if he was able to do so, no motive appears for attempt to defraud the insurer. While the acquittal of the insured in the arson case is not res judicata on the present civil action, the insurer's evidence, to judge from the decision in the criminal case, is practically identical in both cases and must lead to the same result, since the proof to establish the defense of connivance at the fire in order to defraud the insurer "cannot be materially less convincing than that required in order to convict the insured of the crime of arson"(Bachrach vs. British American Assurance Co., 17 Phil. 536). As to the defense that the burned bodegas could not possibly have contained the quantities of copra and hemp stated in the fire claims, the insurer's case rests almost exclusively on the estimates, inferences and conclusionsAs to the defense that the burned bodegas could not possibly have contained the quantities of copra and hemp stated in the fire claims, the insurer's case rests almost exclusively on the estimates, inferences and conclusions of its adjuster investigator, Alexander D. Stewart, who examined the premises during and after the fire. His testimony, however, was based on

inferences from the photographs and traces found after the fire, and must yield to the contradictory testimony of engineer Andres Bolinas, and specially of the then Chief of the Loan Department of the National Bank's Legaspi branch, Porfirio Barrios, and of Bank Appraiser Loreto Samson, who actually saw the contents of the bodegas shortly before the fire, while inspecting them for the mortgagee Bank. The lower Court was satisfied of the veracity and accuracy of these witnesses, and the appellant insurer has failed to substantiate its charges aganst their character. In fact, the insurer's repeated accusations that these witnesses were later "suspended for fraudulent transactions" without giving any details, is a plain attempt to create prejudice against them, without the least support in fact. Stewart himself, in testifying that it is impossible to determine from the remains the quantity of hemp burned (t. s. n., pp. 1468, 1470), rebutted appellant's attacks on the refusal of the Court below to accept its inferences from the remains shown in the photographs of the burned premises. It appears, likewise, that the adjuster's calculations of the maximum contents of the destroyed warehouses rested on the assumption that all the copra and hemp were in sacks, and on the result of his experiments to determine the space occupied by definite amounts of sacked copra. The error in the estimates thus arrived at proceeds from the fact that a large amount of the insured's stock were in loose form, occupying less space than when kept in sacks; and from Stewart's obvious failure to give due allowance for the compression of the material at the bottom of the piles (t. s. n., pp. 1964, 1967) due to the weight of the overlying stock, as shown by engineer Bolinas. It is probable that the errors were due to inexperience (Stewart himself admitted that this was the first copra fire he had investigated); but it is clear that such errors render valueles Stewart's computations. These were in fact twice passed upon and twice rejected by different judges (in the criminal and civil cases) and their concordant opinion is practically conclusive. The adjusters' reports, Exhibits 9-A and 9-B, were correctly disregarded by the Court below, since the opinions stated therein were based on ex parte investigations made at the back of the insured; and the appellant did not present at the trial the original testimony and documents from which the conclusions in the report were drawn.lawphi1.net Appellant insurance company also contends that the claims filed by the insured contained false and fraudulent statements that avoided the insurance policy. But the trial Court found that the discrepancies were a result of the insured's erroneous interpretation of the provisions of the insurance policies and claim forms, caused by his imperfect knowledge of English, and that the misstatements were innocently made and without intent to defraud. Our review of the lengthy record fails to disclose reasons for rejecting these conclusions of the Court below. For example, the occurrence of previous fires in the premises insured in 1939, altho omitted in the claims, Exhibits EE and FF, were nevertheless revealed by the insured in his claims Exhibits Q (filed simultaneously with them), KK and WW. Considering that all these claims were submitted to the smae agent, and that this same agent had paid the loss caused by the 1939 fire, we find no error in the trial Court's acceptance of the insured's explanation that the omission in

Exhibits EE and FF was due to inadvertance, for the insured could hardly expect under such circumstances, that the 1939 would pass unnoticed by the insurance agents. Similarly, the 20 per cent overclaim on 70 per cent of the hemo stock, was explained by the insured as caused by his belief that he was entitled to include in the claim his expected profit on the 70 per cent of the hemp, because the same was already contracted for and sold to other parties before the fire occurred. Compared with other cases of over-valuation recorded in our judicial annals, the 20 per cent excess in the case of the insured is not by itself sufficient to establish fraudulent intent. Thus, in Yu Cua vs. South British Ins. Co., 41 Phil. 134, the claim was fourteen (14) times (1,400 per cent) bigger than the actual loss; in Go Lu vs. Yorkshire Insurance Co., 43 Phil., 633, eight (8) times (800 per cent); in Tuason vs. North China Ins. Co., 47 Phil. 14, six (6) times (600 per cent); in Tan It vs. Sun Insurance, 51 Phil. 212, the claim totalled P31,860.85 while the goods insured were inventoried at O13,113. Certainly, the insured's overclaim of 20 per cent in the case at bar, duly explained by him to the Court a quo, appears puny by comparison, and can not be regarded as "more than misstatement, more than inadvertence of mistake, more than a mere error in opinion, more than a slight exaggeration" (Tan It vs. Sun Insurance Office, ante) that would entitle the insurer to avoid the policy. It is well to note that the overchange of 20 per cent was claimed only on a part (70 per cent) of the hemp stock; had the insured acted with fraudulent intent, nothing prevented him from increasing the value of all of his copra, hemp and buildings in the same proportion. This also applies to the alleged fraudulent claim for burned empty sacks, that was likewise explained to our satisfaction and that of the trial Court. The rule is that to avoid a policy, the false swearing must be wilful and with intent to defraud (29 Am. Jur., pp. 849-851) which was not the cause. Of course, the lack of fraudulent intent would not authorize the collection of the expected profit under the terms of the polices, and the trial Court correctly deducte the same from its award. We find no reversible error in the judgment appealed from, wherefore the smae is hereby affirmed. Costs against the appellant. So ordered. Paras, C. J., Padilla, Montemayor, Reyes, A., Jugo, Labrador, and Concepcion, JJ., concur.

WEEK 7 VIII. Warranties and Incontestability Clause

G.R. No. 151890

June 20, 2006

PRUDENTIAL GUARANTEE and ASSURANCE INC., petitioner, vs. TRANS-ASIA SHIPPING LINES, INC., Respondent. x- - - - - - - - - - - - - - - - - - - - - - - - - x G.R. No. 151991 June 20, 2006

TRANS-ASIA SHIPPING LINES, INC., petitioner, vs. PRUDENTIAL GUARANTEE and ASSURANCE INC., Respondent. DECISION CHICO-NAZARIO, J: This is a consolidation of two separate Petitions for Review on Certiorari filed by petitioner Prudential Guarantee and Assurance, Inc. (PRUDENTIAL) in G.R. No. 151890 and Trans-Asia Shipping Lines, Inc. (TRANS-ASIA) in G.R. No. 151991, assailing the Decision1 dated 6 November 2001 of the Court of Appeals in CA G.R. CV No. 68278, which reversed the Judgment2 dated 6 June 2000 of the Regional Trial Court (RTC), Branch 13, Cebu City in Civil Case No. CEB-20709. The 29 January 2002 Resolution3 of the Court of Appeals, denying PRUDENTIALs Motion for Reconsideration and TRANS-ASIAs Partial Motion for Reconsideration of the 6 November 2001 Decision, is likewise sought to be annulled and set aside. The Facts The material antecedents as found by the court a quo and adopted by the appellate court are as follows: Plaintiff [TRANS-ASIA] is the owner of the vessel M/V Asia Korea. In consideration of payment of premiums, defendant [PRUDENTIAL] insured M/V Asia Korea for loss/damage of the hull and machinery arising from perils, inter alia, of fire and explosion for the sum of P40 Million, beginning [from] the period [of] July 1, 1993 up to July 1, 1994. This is evidenced by Marine Policy No. MH93/1363 (Exhibits "A" to "A11"). On October 25, 1993, while the policy was in force, a fire broke out while [M/V Asia Korea was] undergoing repairs at the port of Cebu. On October 26, 1993 plaintiff

[TRANS-ASIA] filed its notice of claim for damage sustained by the vessel. This is evidenced by a letter/formal claim of even date (Exhibit "B"). Plaintiff [TRANS-ASIA] reserved its right to subsequently notify defendant [PRUDENTIAL] as to the full amount of the claim upon final survey and determination by average adjuster Richard Hogg International (Phil.) of the damage sustained by reason of fire. An adjusters report on the fire in question was submitted by Richard Hogg International together with the UMarine Surveyor Report (Exhibits "4" to "4-115"). On May 29, 1995[,] plaintiff [TRANS-ASIA] executed a document denominated "Loan and Trust receipt", a portion of which read (sic): "Received from Prudential Guarantee and Assurance, Inc., the sum of PESOS THREE MILLION ONLY (P3,000,000.00) as a loan without interest under Policy No. MH 93/1353 [sic], repayable only in the event and to the extent that any net recovery is made by Trans-Asia Shipping Corporation, from any person or persons, corporation or corporations, or other parties, on account of loss by any casualty for which they may be liable occasioned by the 25 October 1993: Fire on Board." (Exhibit "4") In a letter dated 21 April 1997 defendant [PRUDENTIAL] denied plaintiffs claim (Exhibit "5"). The letter reads: "After a careful review and evaluation of your claim arising from the above-captioned incident, it has been ascertained that you are in breach of policy conditions, among them "WARRANTED VESSEL CLASSED AND CLASS MAINTAINED". Accordingly, we regret to advise that your claim is not compensable and hereby DENIED." This was followed by defendants letter dated 21 July 1997 requesting the return or payment of the P3,000,000.00 within a period of ten (10) days from receipt of the letter (Exhibit "6").4 Following this development, on 13 August 1997, TRANS-ASIA filed a Complaint5 for Sum of Money against PRUDENTIAL with the RTC of Cebu City, docketed as Civil Case No. CEB-20709, wherein TRANS-ASIA sought the amount of P8,395,072.26 from PRUDENTIAL, alleging that the same represents the balance of the indemnity due upon the insurance policy in the total amount of P11,395,072.26. TRANS-ASIA similarly sought interest at 42% per annum citing Section 2436 of Presidential Decreee No. 1460, otherwise known as the "Insurance Code," as amended. In its Answer,7 PRUDENTIAL denied the material allegations of the Complaint and interposed the defense that TRANS-ASIA breached insurance policy conditions, in particular: "WARRANTED VESSEL CLASSED AND CLASS MAINTAINED." PRUDENTIAL further alleged that it acted as facts and law require and incurred no liability to TRANS-ASIA; that TRANS-ASIA has no cause of action; and, that its claim has been effectively waived and/or abandoned, or it is estopped from pursuing the same. By way of a counterclaim, PRUDENTIAL sought a refund of P3,000,000.00, which it allegedly advanced to TRANS-ASIA by way of a loan without interest and

without prejudice to the final evaluation of the claim, including the amounts of P500,000.00, for survey fees and P200,000.00, representing attorneys fees. The Ruling of the Trial Court On 6 June 2000, the court a quo rendered Judgment8 finding for (therein defendant) PRUDENTIAL. It ruled that a determination of the parties liabilities hinged on whether TRANS-ASIA violated and breached the policy conditions on WARRANTED VESSEL CLASSED AND CLASS MAINTAINED. It interpreted the provision to mean that TRANS-ASIA is required to maintain the vessel at a certain class at all times pertinent during the life of the policy. According to the court a quo, TRANS-ASIA failed to prove compliance of the terms of the warranty, the violation thereof entitled PRUDENTIAL, the insured party, to rescind the contract.9 Further, citing Section 10710 of the Insurance Code, the court a quo ratiocinated that the concealment made by TRANS-ASIA that the vessel was not adequately maintained to preserve its class was a material concealment sufficient to avoid the policy and, thus, entitled the injured party to rescind the contract. The court a quo found merit in PRUDENTIALs contention that there was nothing in the adjustment of the particular average submitted by the adjuster that would show that TRANS-ASIA was not in breach of the policy. Ruling on the denominated loan and trust receipt, the court a quo said that in substance and in form, the same is a receipt for a loan. It held that if TRANS-ASIA intended to receive the amount of P3,000,000.00 as advance payment, it should have so clearly stated as such. The court a quo did not award PRUDENTIALs claim for P500,000.00, representing expert survey fees on the ground of lack of sufficient basis in support thereof. Neither did it award attorneys fees on the rationalization that the instant case does not fall under the exceptions stated in Article 220811 of the Civil Code. However, the court a quo granted PRUDENTIALs counterclaim stating that there is factual and legal basis for TRANS-ASIA to return the amount of P3,000,000.00 by way of loan without interest. The decretal portion of the Judgment of the RTC reads: WHEREFORE, judgment is hereby rendered DISMISSING the complaint for its failure to prove a cause of action. On defendants counterclaim, plaintiff is directed to return the sum of P3, 000,000.00 representing the loan extended to it by the defendant, within a period of ten (10) days from and after this judgment shall have become final and executory. 12 The Ruling of the Court of Appeals On appeal by TRANS-ASIA, the Court of Appeals, in its assailed Decision of 6 November 2001, reversed the 6 June 2000 Judgment of the RTC.

On the issue of TRANS-ASIAs alleged breach of warranty of the policy condition CLASSED AND CLASS MAINTAINED, the Court of Appeals ruled that PRUDENTIAL, as the party asserting the non-compensability of the loss had the burden of proof to show that TRANS-ASIA breached the warranty, which burden it failed to discharge. PRUDENTIAL cannot rely on the lack of certification to the effect that TRANS-ASIA was CLASSED AND CLASS MAINTAINED as its sole basis for reaching the conclusion that the warranty was breached. The Court of Appeals opined that the lack of a certification does not necessarily mean that the warranty was breached by TRANS-ASIA. Instead, the Court of Appeals considered PRUDENTIALs admission that at the time the insurance contract was entered into between the parties, the vessel was properly classed by Bureau Veritas, a classification society recognized by the industry. The Court of Appeals similarly gave weight to the fact that it was the responsibility of Richards Hogg International (Phils.) Inc., the average adjuster hired by PRUDENTIAL, to secure a copy of such certification to support its conclusion that mere absence of a certification does not warrant denial of TRANS-ASIAs claim under the insurance policy. In the same token, the Court of Appeals found the subject warranty allegedly breached by TRANS-ASIA to be a rider which, while contained in the policy, was inserted by PRUDENTIAL without the intervention of TRANS-ASIA. As such, it partakes of a nature of a contract dadhesion which should be construed against PRUDENTIAL, the party which drafted the contract. Likewise, according to the Court of Appeals, PRUDENTIALs renewal of the insurance policy from noon of 1 July 1994 to noon of 1 July 1995, and then again, until noon of 1 July 1996 must be deemed a waiver by PRUDENTIAL of any breach of warranty committed by TRANS-ASIA. Further, the Court of Appeals, contrary to the ruling of the court a quo, interpreted the transaction between PRUDENTIAL and TRANS-ASIA as one of subrogation, instead of a loan. The Court of Appeals concluded that TRANS-ASIA has no obligation to pay back the amount of P3,000.000.00 to PRUDENTIAL based on its finding that the aforesaid amount was PRUDENTIALs partial payment to TRANS-ASIAs claim under the policy. Finally, the Court of Appeals denied TRANS-ASIAs prayer for attorneys fees, but held TRANS-ASIA entitled to double interest on the policy for the duration of the delay of payment of the unpaid balance, citing Section 244 13 of the Insurance Code. Finding for therein appellant TRANS-ASIA, the Court of Appeals ruled in this wise: WHEREFORE, the foregoing consideration, We find for Appellant. The instant appeal is ALLOWED and the Judgment appealed from REVERSED. The P3,000,000.00 initially paid by appellee Prudential Guarantee Assurance Incorporated to appellant Trans-Asia and covered by a "Loan and Trust Receipt" dated 29 May 1995 is HELD to be in partial settlement of the loss suffered by appellant and covered by Marine Policy No. MH93/1363 issued by appellee. Further, appellee is hereby ORDERED to pay appellant the additional amount of P8,395,072.26 representing the balance of the loss suffered by the latter as recommended by the average adjuster Richard Hogg International (Philippines) in its Report, with double interest starting from the time Richard Hoggs Survey Report was completed, or on 13 August 1996, until the same is fully paid.

All other claims and counterclaims are hereby DISMISSED. All costs against appellee.14 Not satisfied with the judgment, PRUDENTIAL and TRANS-ASIA filed a Motion for Reconsideration and Partial Motion for Reconsideration thereon, respectively, which motions were denied by the Court of Appeals in the Resolution dated 29 January 2002. The Issues Aggrieved, PRUDENTIAL filed before this Court a Petition for Review, docketed as G.R. No. 151890, relying on the following grounds, viz: I. THE AWARD IS GROSSLY UNCONSCIONABLE. II. THE COURT OF APPEALS ERRED IN HOLDING THAT THERE WAS NO VIOLATION BY TRANS-ASIA OF A MATERIAL WARRANTY, NAMELY, WARRANTY CLAUSE NO. 5, OF THE INSURANCE POLICY. III. THE COURT OF APPEALS ERRED IN HOLDING THAT PRUDENTIAL, AS INSURER HAD THE BURDEN OF PROVING THAT THE ASSURED, TRANS-ASIA, VIOLATED A MATERIAL WARRANTY. IV. THE COURT OF APPEALS ERRED IN HOLDING THAT THE WARRANTY CLAUSE EMBODIED IN THE INSURANCE POLICY CONTRACT WAS A MERE RIDER. V. THE COURT OF APPEALS ERRED IN HOLDING THAT THE ALLEGED RENEWALS OF THE POLICY CONSTITUTED A WAIVER ON THE PART OF PRUDENTIAL OF THE BREACH OF THE WARRANTY BY TRANS-ASIA. VI. THE COURT OF APPEALS ERRED IN HOLDING THAT THE "LOAN AND TRUST RECEIPT" EXECUTED BY TRANS-ASIA IS AN ADVANCE ON THE POLICY, THUS CONSTITUTING PARTIAL PAYMENT THEREOF.

VII. THE COURT OF APPEALS ERRED IN HOLDING THAT THE ACCEPTANCE BY PRUDENTIAL OF THE FINDINGS OF RICHARDS HOGG IS INDICATIVE OF A WAIVER ON THE PART OF PRUDENTIAL OF ANY VIOLATION BY TRANS-ASIA OF THE WARRANTY. VIII. THE COURT OF APPEALS ERRRED (sic) IN REVERSING THE TRIAL COURT, IN FINDING THAT PRUDENTIAL "UNJUSTIFIABLY REFUSED" TO PAY THE CLAIM AND IN ORDERING PRUDENTIAL TO PAY TRANS-ASIA P8,395,072.26 PLUS DOUBLE INTEREST FROM 13 AUGUST 1996, UNTIL [THE] SAME IS FULLY PAID. 15 Similarly, TRANS-ASIA, disagreeing in the ruling of the Court of Appeals filed a Petition for Review docketed as G.R. No. 151991, raising the following grounds for the allowance of the petition, to wit: I. THE HONORABLE COURT OF APPEALS ERRED IN NOT AWARDING ATTORNEYS FEES TO PETITIONER TRANS-ASIA ON THE GROUND THAT SUCH CAN ONLY BE AWARDED IN THE CASES ENUMERATED IN ARTICLE 2208 OF THE CIVIL CODE, AND THERE BEING NO BAD FAITH ON THE PART OF RESPONDENT PRUDENTIAL IN DENYING HEREIN PETITIONER TRANS-ASIAS INSURANCE CLAIM. II. THE "DOUBLE INTEREST" REFERRED TO IN THE DECISION DATED 06 NOVEMBER 2001 SHOULD BE CONSTRUED TO MEAN DOUBLE INTEREST BASED ON THE LEGAL INTEREST OF 12%, OR INTEREST AT THE RATE OF 24% PER ANNUM.16 In our Resolution of 2 December 2002, we granted TRANS-ASIAs Motion for Consolidation17 of G.R. Nos. 151890 and 151991;18 hence, the instant consolidated petitions. In sum, for our main resolution are: (1) the liability, if any, of PRUDENTIAL to TRANSASIA arising from the subject insurance contract; (2) the liability, if any, of TRANS-ASIA to PRUDENTIAL arising from the transaction between the parties as evidenced by a document denominated as "Loan and Trust Receipt," dated 29 May 1995; and (3) the amount of interest to be imposed on the liability, if any, of either or both parties. Ruling of the Court

Prefatorily, it must be emphasized that in a petition for review, only questions of law, and not questions of fact, may be raised.19 This rule may be disregarded only when the findings of fact of the Court of Appeals are contrary to the findings and conclusions of the trial court, or are not supported by the evidence on record. 20 In the case at bar, we find an incongruence between the findings of fact of the Court of Appeals and the court a quo, thus, in our determination of the issues, we are constrained to assess the evidence adduced by the parties to make appropriate findings of facts as are necessary. I. A. PRUDENTIAL failed to establish that TRANS-ASIA violated and breached the policy condition on WARRANTED VESSEL CLASSED AND CLASS MAINTAINED, as contained in the subject insurance contract. In resisting the claim of TRANS-ASIA, PRUDENTIAL posits that TRANS-ASIA violated an express and material warranty in the subject insurance contract, i.e., Marine Insurance Policy No. MH93/1363, specifically Warranty Clause No. 5 thereof, which stipulates that the insured vessel, "M/V ASIA KOREA" is required to be CLASSED AND CLASS MAINTAINED. According to PRUDENTIAL, on 25 October 1993, or at the time of the occurrence of the fire, "M/V ASIA KOREA" was in violation of the warranty as it was not CLASSED AND CLASS MAINTAINED. PRUDENTIAL submits that Warranty Clause No. 5 was a condition precedent to the recovery of TRANS-ASIA under the policy, the violation of which entitled PRUDENTIAL to rescind the contract under Sec. 7421 of the Insurance Code. The warranty condition CLASSED AND CLASS MAINTAINED was explained by PRUDENTIALs Senior Manager of the Marine and Aviation Division, Lucio Fernandez. The pertinent portions of his testimony on direct examination is reproduced hereunder, viz: ATTY. LIM Q Please tell the court, Mr. Witness, the result of the evaluation of this claim, what final action was taken? A It was eventually determined that there was a breach of the policy condition, and basically there is a breach of policy warranty condition and on that basis the claim was denied. Q To refer you (sic) the "policy warranty condition," I am showing to you a policy here marked as Exhibits "1", "1-A" series, please point to the warranty in the policy which you said was breached or violated by the plaintiff which constituted your basis for denying the claim as you testified. A Warranted Vessel Classed and Class Maintained.

ATTY. LIM Witness pointing, Your Honor, to that portion in Exhibit "1-A" which is the second page of the policy below the printed words: "Clauses, Endorsements, Special Conditions and Warranties," below this are several typewritten clauses and the witness pointed out in particular the clause reading: "Warranted Vessel Classed and Class Maintained." COURT Q Will you explain that particular phrase? A Yes, a warranty is a condition that has to be complied with by the insured. When we say a class warranty, it must be entered in the classification society. COURT Slowly. WITNESS (continued) A A classification society is an organization which sets certain standards for a vessel to maintain in order to maintain their membership in the classification society. So, if they failed to meet that standard, they are considered not members of that class, and thus breaching the warranty, that requires them to maintain membership or to maintain their class on that classification society. And it is not sufficient that the member of this classification society at the time of a loss, their membership must be continuous for the whole length of the policy such that during the effectivity of the policy, their classification is suspended, and then thereafter, they get reinstated, that again still a breach of the warranty that they maintained their class (sic). Our maintaining team membership in the classification society thereby maintaining the standards of the vessel (sic). ATTY. LIM Q Can you mention some classification societies that you know? A Well we have the Bureau Veritas, American Bureau of Shipping, D&V Local Classification Society, The Philippine Registration of Ships Society, China Classification, NKK and Company Classification Society, and many others, we have among others, there are over 20 worldwide. 22 At the outset, it must be emphasized that the party which alleges a fact as a matter of defense has the burden of proving it. PRUDENTIAL, as the party which asserted the claim that TRANS-ASIA breached the warranty in the policy, has the burden of evidence to establish the same. Hence, on the part of PRUDENTIAL lies the initiative to show

proof in support of its defense; otherwise, failing to establish the same, it remains selfserving. Clearly, if no evidence on the alleged breach of TRANS-ASIA of the subject warranty is shown, a fortiori, TRANS-ASIA would be successful in claiming on the policy. It follows that PRUDENTIAL bears the burden of evidence to establish the fact of breach. In our rule on evidence, TRANS-ASIA, as the plaintiff below, necessarily has the burden of proof to show proof of loss, and the coverage thereof, in the subject insurance policy. However, in the course of trial in a civil case, once plaintiff makes out a prima facie case in his favor, the duty or the burden of evidence shifts to defendant to controvert plaintiffs prima facie case, otherwise, a verdict must be returned in favor of plaintiff.23 TRANS-ASIA was able to establish proof of loss and the coverage of the loss, i.e., 25 October 1993: Fire on Board. Thereafter, the burden of evidence shifted to PRUDENTIAL to counter TRANS-ASIAs case, and to prove its special and affirmative defense that TRANS-ASIA was in violation of the particular condition on CLASSED AND CLASS MAINTAINED. We sustain the findings of the Court of Appeals that PRUDENTIAL was not successful in discharging the burden of evidence that TRANS-ASIA breached the subject policy condition on CLASSED AND CLASS MAINTAINED. Foremost, PRUDENTIAL, through the Senior Manager of its Marine and Aviation Division, Lucio Fernandez, made a categorical admission that at the time of the procurement of the insurance contract in July 1993, TRANS-ASIAs vessel, "M/V Asia Korea" was properly classed by Bureau Veritas, thus: Q Kindly examine the records particularly the policy, please tell us if you know whether M/V Asia Korea was classed at the time (sic) policy was procured perthe (sic) insurance was procured that Exhibit "1" on 1st July 1993 (sic). WITNESS A I recall that they were classed. ATTY. LIM Q With what classification society? A I believe with Bureau Veritas.24 As found by the Court of Appeals and as supported by the records, Bureau Veritas is a classification society recognized in the marine industry. As it is undisputed that TRANSASIA was properly classed at the time the contract of insurance was entered into, thus, it becomes incumbent upon PRUDENTIAL to show evidence that the status of TRANSASIA as being properly CLASSED by Bureau Veritas had shifted in violation of the warranty. Unfortunately, PRUDENTIAL failed to support the allegation.

We are in accord with the ruling of the Court of Appeals that the lack of a certification in PRUDENTIALs records to the effect that TRANS-ASIAs "M/V Asia Korea" was CLASSED AND CLASS MAINTAINED at the time of the occurrence of the fire cannot be tantamount to the conclusion that TRANS-ASIA in fact breached the warranty contained in the policy. With more reason must we sustain the findings of the Court of Appeals on the ground that as admitted by PRUDENTIAL, it was likewise the responsibility of the average adjuster, Richards Hogg International (Phils.), Inc., to secure a copy of such certification, and the alleged breach of TRANS-ASIA cannot be gleaned from the average adjusters survey report, or adjustment of particular average per "M/V Asia Korea" of the 25 October 1993 fire on board. We are not unmindful of the clear language of Sec. 74 of the Insurance Code which provides that, "the violation of a material warranty, or other material provision of a policy on the part of either party thereto, entitles the other to rescind." It is generally accepted that "[a] warranty is a statement or promise set forth in the policy, or by reference incorporated therein, the untruth or non-fulfillment of which in any respect, and without reference to whether the insurer was in fact prejudiced by such untruth or nonfulfillment, renders the policy voidable by the insurer."25However, it is similarly indubitable that for the breach of a warranty to avoid a policy, the same must be duly shown by the party alleging the same. We cannot sustain an allegation that is unfounded. Consequently, PRUDENTIAL, not having shown that TRANS-ASIA breached the warranty condition, CLASSED AND CLASS MAINTAINED, it remains that TRANS-ASIA must be allowed to recover its rightful claims on the policy. B. Assuming arguendo that TRANS-ASIA violated the policy condition on WARRANTED VESSEL CLASSED AND CLASS MAINTAINED, PRUDENTIAL made a valid waiver of the same. The Court of Appeals, in reversing the Judgment of the RTC which held that TRANSASIA breached the warranty provision on CLASSED AND CLASS MAINTAINED, underscored that PRUDENTIAL can be deemed to have made a valid waiver of TRANS-ASIAs breach of warranty as alleged, ratiocinating, thus: Third, after the loss, Prudential renewed the insurance policy of Trans-Asia for two (2) consecutive years, from noon of 01 July 1994 to noon of 01 July 1995, and then again until noon of 01 July 1996. This renewal is deemed a waiver of any breach of warranty.26 PRUDENTIAL finds fault with the ruling of the appellate court when it ruled that the renewal policies are deemed a waiver of TRANS-ASIAs alleged breach, averring herein that the subsequent policies, designated as MH94/1595 and MH95/1788 show that they were issued only on 1 July 1994 and 3 July 1995, respectively, prior to the time it made a request to TRANS-ASIA that it be furnished a copy of the certification specifying that the insured vessel "M/V Asia Korea" was CLASSED AND CLASS MAINTAINED. PRUDENTIAL posits that it came to know of the breach by TRANS-ASIA of the subject warranty clause only on 21 April 1997. On even date, PRUDENTIAL sent TRANS-ASIA

a letter of denial, advising the latter that their claim is not compensable. In fine, PRUDENTIAL would have this Court believe that the issuance of the renewal policies cannot be a waiver because they were issued without knowledge of the alleged breach of warranty committed by TRANS-ASIA.27 We are not impressed. We do not find that the Court of Appeals was in error when it held that PRUDENTIAL, in renewing TRANS-ASIAs insurance policy for two consecutive years after the loss covered by Policy No. MH93/1363, was considered to have waived TRANS-ASIAs breach of the subject warranty, if any. Breach of a warranty or of a condition renders the contract defeasible at the option of the insurer; but if he so elects, he may waive his privilege and power to rescind by the mere expression of an intention so to do. In that event his liability under the policy continues as before. 28 There can be no clearer intention of the waiver of the alleged breach than the renewal of the policy insurance granted by PRUDENTIAL to TRANS-ASIA in MH94/1595 and MH95/1788, issued in the years 1994 and 1995, respectively. To our mind, the argument is made even more credulous by PRUDENTIALs lack of proof to support its allegation that the renewals of the policies were taken only after a request was made to TRANS-ASIA to furnish them a copy of the certificate attesting that "M/V Asia Korea" was CLASSED AND CLASS MAINTAINED. Notwithstanding PRUDENTIALs claim that no certification was issued to that effect, it renewed the policy, thereby, evidencing an intention to waive TRANS-ASIAs alleged breach. Clearly, by granting the renewal policies twice and successively after the loss, the intent was to benefit the insured, TRANS-ASIA, as well as to waive compliance of the warranty. The foregoing finding renders a determination of whether the subject warranty is a rider, moot, as raised by the PRUDENTIAL in its assignment of errors. Whether it is a rider will not effectively alter the result for the reasons that: (1) PRUDENTIAL was not able to discharge the burden of evidence to show that TRANS-ASIA committed a breach, thereof; and (2) assuming arguendo the commission of a breach by TRANS-ASIA, the same was shown to have been waived by PRUDENTIAL. II. A. The amount of P3,000,000.00 granted by PRUDENTIAL to TRANS- ASIA via a transaction between the parties evidenced by a document denominated as "Loan and Trust Receipt," dated 29 May 1995 constituted partial payment on the policy. It is undisputed that TRANS-ASIA received from PRUDENTIAL the amount of P3,000,000.00. The same was evidenced by a transaction receipt denominated as a "Loan and Trust Receipt," dated 29 May 1995, reproduced hereunder: LOAN AND TRUST RECEIPT

Claim File No. MH-93-025 P3,000,000.00 Check No. PCIB066755

May 29, 1995

Received FROM PRUDENTIAL GUARANTEE AND ASSURANCE INC., the sum of PESOS THREE MILLION ONLY (P3,000,000.00) as a loan without interest, under Policy No. MH93/1353, repayable only in the event and to the extent that any net recovery is made by TRANS ASIA SHIPPING CORP., from any person or persons, corporation or corporations, or other parties, on account of loss by any casualty for which they may be liable, occasioned by the 25 October 1993: Fire on Board. As security for such repayment, we hereby pledge to PRUDENTIAL GUARANTEE AND ASSURANCE INC. whatever recovery we may make and deliver to it all documents necessary to prove our interest in said property. We also hereby agree to promptly prosecute suit against such persons, corporation or corporations through whose negligence the aforesaid loss was caused or who may otherwise be responsible therefore, with all due diligence, in our own name, but at the expense of and under the exclusive direction and control of PRUDENTIAL GUARANTEE AND ASSURANCE INC. TRANS-ASIA SHIPPING CORPORATION29 PRUDENTIAL largely contends that the "Loan and Trust Receipt" executed by the parties evidenced a loan of P3,000,000.00 which it granted to TRANS-ASIA, and not an advance payment on the policy or a partial payment for the loss. It further submits that it is a customary practice for insurance companies in this country to extend loans gratuitously as part of good business dealing with their assured, in order to afford their assured the chance to continue business without embarrassment while awaiting outcome of the settlement of their claims.30 According to PRUDENTIAL, the "Trust and Loan Agreement" did not subrogate to it whatever rights and/or actions TRANS-ASIA may have against third persons, and it cannot by no means be taken that by virtue thereof, PRUDENTIAL was granted irrevocable power of attorney by TRANS-ASIA, as the sole power to prosecute lies solely with the latter. The Court of Appeals held that the real character of the transaction between the parties as evidenced by the "Loan and Trust Receipt" is that of an advance payment by PRUDENTIAL of TRANS-ASIAs claim on the insurance, thus: The Philippine Insurance Code (PD 1460 as amended) was derived from the old Insurance Law Act No. 2427 of the Philippine Legislature during the American Regime. The Insurance Act was lifted verbatim from the law of California, except Chapter V thereof, which was taken largely from the insurance law of New York. Therefore, ruling case law in that jurisdiction is to Us persuasive in interpreting provisions of our own Insurance Code. In addition, the application of the adopted statute should correspond in fundamental points with the application in its country of origin x x x. xxxx

Likewise, it is settled in that jurisdiction that the (sic) notwithstanding recitals in the Loan Receipt that the money was intended as a loan does not detract from its real character as payment of claim, thus: "The receipt of money by the insured employers from a surety company for losses on account of forgery of drafts by an employee where no provision or repayment of the money was made except upon condition that it be recovered from other parties and neither interest nor security for the asserted debts was provided for, the money constituted the payment of a liability and not a mere loan, notwithstanding recitals in the written receipt that the money was intended as a mere loan." What is clear from the wordings of the so-called "Loan and Trust Receipt Agreement" is that appellant is obligated to hand over to appellee "whatever recovery (Trans Asia) may make and deliver to (Prudential) all documents necessary to prove its interest in the said property." For all intents and purposes therefore, the money receipted is payment under the policy, with Prudential having the right of subrogation to whatever net recovery Trans-Asia may obtain from third parties resulting from the fire. In the law on insurance, subrogation is an equitable assignment to the insurer of all remedies which the insured may have against third person whose negligence or wrongful act caused the loss covered by the insurance policy, which is created as the legal effect of payment by the insurer as an assignee in equity. The loss in the first instance is that of the insured but after reimbursement or compensation, it becomes the loss of the insurer. It has been referred to as the doctrine of substitution and rests on the principle that substantial justice should be attained regardless of form, that is, its basis is the doing of complete, essential, and perfect justice between all the parties without regard to form.31 We agree. Notwithstanding its designation, the tenor of the "Loan and Trust Receipt" evidences that the real nature of the transaction between the parties was that the amount of P3,000,000.00 was not intended as a loan whereby TRANS-ASIA is obligated to pay PRUDENTIAL, but rather, the same was a partial payment or an advance on the policy of the claims due to TRANS-ASIA. First, the amount of P3,000,000.00 constitutes an advance payment to TRANS-ASIA by PRUDENTIAL, subrogating the former to the extent of "any net recovery made by TRANS ASIA SHIPPING CORP., from any person or persons, corporation or corporations, or other parties, on account of loss by any casualty for which they may be liable, occasioned by the 25 October 1993: Fire on Board." 32 Second, we find that per the "Loan and Trust Receipt," even as TRANS-ASIA agreed to "promptly prosecute suit against such persons, corporation or corporations through whose negligence the aforesaid loss was caused or who may otherwise be responsible therefore, with all due diligence" in its name, the prosecution of the claims against such third persons are to be carried on "at the expense of and under the exclusive direction and control of PRUDENTIAL GUARANTEE AND ASSURANCE INC."33 The clear import of the phrase "at the expense of and under the exclusive direction and control" as used

in the "Loan and Trust Receipt" grants solely to PRUDENTIAL the power to prosecute, even as the same is carried in the name of TRANS-ASIA, thereby making TRANS-ASIA merely an agent of PRUDENTIAL, the principal, in the prosecution of the suit against parties who may have occasioned the loss. Third, per the subject "Loan and Trust Receipt," the obligation of TRANS-ASIA to repay PRUDENTIAL is highly speculative and contingent, i.e., only in the event and to the extent that any net recovery is made by TRANS-ASIA from any person on account of loss occasioned by the fire of 25 October 1993. The transaction, therefore, was made to benefit TRANS-ASIA, such that, if no recovery from third parties is made, PRUDENTIAL cannot be repaid the amount. Verily, we do not think that this is constitutive of a loan.34 The liberality in the tenor of the "Loan and Trust Receipt" in favor of TRANSASIA leads to the conclusion that the amount of P3,000,000.00 was a form of an advance payment on TRANS-ASIAs claim on MH93/1353. III. A. PRUDENTIAL is directed to pay TRANS-ASIA the amount of P8,395,072.26, representing the balance of the loss suffered by TRANS-ASIA and covered by Marine Policy No. MH93/1363. Our foregoing discussion supports the conclusion that TRANS-ASIA is entitled to the unpaid claims covered by Marine Policy No. MH93/1363, or a total amount of P8,395,072.26. B. Likewise, PRUDENTIAL is directed to pay TRANS-ASIA, damages in the form of attorneys fees equivalent to 10% of P8,395,072.26. The Court of Appeals denied the grant of attorneys fees. It held that attorneys fees cannot be awarded absent a showing of bad faith on the part of PRUDENTIAL in rejecting TRANS-ASIAs claim, notwithstanding that the rejection was erroneous. According to the Court of Appeals, attorneys fees can be awarded only in the cases enumerated in Article 2208 of the Civil Code which finds no application in the instant case. We disagree. Sec. 244 of the Insurance Code grants damages consisting of attorneys fees and other expenses incurred by the insured after a finding by the Insurance Commissioner or the Court, as the case may be, of an unreasonable denial or withholding of the payment of the claims due. Moreover, the law imposes an interest of twice the ceiling prescribed by the Monetary Board on the amount of the claim due the insured from the date following the time prescribed in Section 24235 or in Section 243,36 as the case may be, until the claim is fully satisfied. Finally, Section 244 considers the failure to pay the claims within the time prescribed in Sections 242 or 243, when applicable, as prima facie evidence of unreasonable delay in payment.

To the mind of this Court, Section 244 does not require a showing of bad faith in order that attorneys fees be granted. As earlier stated, under Section 244, a prima facie evidence of unreasonable delay in payment of the claim is created by failure of the insurer to pay the claim within the time fixed in both Sections 242 and 243 of the Insurance Code. As established in Section 244, by reason of the delay and the consequent filing of the suit by the insured, the insurers shall be adjudged to pay damages which shall consist of attorneys fees and other expenses incurred by the insured.37 Section 244 reads: In case of any litigation for the enforcement of any policy or contract of insurance, it shall be the duty of the Commissioner or the Court, as the case may be, to make a finding as to whether the payment of the claim of the insured has been unreasonably denied or withheld; and in the affirmative case, the insurance company shall be adjudged to pay damages which shall consist of attorneys fees and other expenses incurred by the insured person by reason of such unreasonable denial or withholding of payment plus interest of twice the ceiling prescribed by the Monetary Board of the amount of the claim due the insured, from the date following the time prescribed in section two hundred forty-two or in section two hundred forty-three, as the case may be, until the claim is fully satisfied; Provided, That the failure to pay any such claim within the time prescribed in said sections shall be considered prima facie evidence of unreasonable delay in payment. Sections 243 and 244 of the Insurance Code apply when the court finds an unreasonable delay or refusal in the payment of the insurance claims. In the case at bar, the facts as found by the Court of Appeals, and confirmed by the records show that there was an unreasonable delay by PRUDENTIAL in the payment of the unpaid balance of P8,395,072.26 to TRANS-ASIA. On 26 October 1993, a day after the occurrence of the fire in "M/V Asia Korea", TRANS-ASIA filed its notice of claim. On 13 August 1996, the adjuster, Richards Hogg International (Phils.), Inc., completed its survey report recommending the amount of P11,395,072.26 as the total indemnity due to TRANS-ASIA.38 On 21 April 1997, PRUDENTIAL, in a letter39 addressed to TRANSASIA denied the latters claim for the amount of P8,395,072.26 representing the balance of the total indemnity. On 21 July 1997, PRUDENTIAL sent a second letter40 to TRANSASIA seeking a return of the amount of P3,000,000.00. On 13 August 1997, TRANSASIA was constrained to file a complaint for sum of money against PRUDENTIAL praying, inter alia, for the sum of P8,395,072.26 representing the balance of the proceeds of the insurance claim. As can be gleaned from the foregoing, there was an unreasonable delay on the part of PRUDENTIAL to pay TRANS-ASIA, as in fact, it refuted the latters right to the insurance claims, from the time proof of loss was shown and the ascertainment of the loss was made by the insurance adjuster. Evidently, PRUDENTIALs unreasonable

delay in satisfying TRANS-ASIAs unpaid claims compelled the latter to file a suit for collection. Succinctly, an award equivalent to ten percent (10%) of the unpaid proceeds of the policy as attorneys fees to TRANS-ASIA is reasonable under the circumstances, or otherwise stated, ten percent (10%) of P8,395,072.26. In the case of Cathay Insurance, Co., Inc. v. Court of Appeals,41 where a finding of an unreasonable delay under Section 244 of the Insurance Code was made by this Court, we grant an award of attorneys fees equivalent to ten percent (10%) of the total proceeds. We find no reason to deviate from this judicial precedent in the case at bar. C. Further, the aggregate amount (P8,395,072.26 plus 10% thereof as attorneys fees) shall be imposed double interest in accordance with Section 244 of the Insurance Code. Section 244 of the Insurance Code is categorical in imposing an interest twice the ceiling prescribed by the Monetary Board due the insured, from the date following the time prescribed in Section 242 or in Section 243, as the case may be, until the claim is fully satisfied. In the case at bar, we find Section 243 to be applicable as what is involved herein is a marine insurance, clearly, a policy other than life insurance. Section 243 is hereunder reproduced: SEC. 243. The amount of any loss or damage for which an insurer may be liable, under any policy other than life insurance policy, shall be paid within thirty days after proof of loss is received by the insurer and ascertainment of the loss or damage is made either by agreement between the insured and the insurer or by arbitration; but if such ascertainment is not had or made within sixty days after such receipt by the insurer of the proof of loss, then the loss or damage shall be paid within ninety days after such receipt. Refusal or failure to pay the loss or damage within the time prescribed herein will entitle the assured to collect interest on the proceeds of the policy for the duration of the delay at the rate of twice the ceiling prescribed by the Monetary Board, unless such failure or refusal to pay is based on the ground that the claim is fraudulent. As specified, the assured is entitled to interest on the proceeds for the duration of the delay at the rate of twice the ceiling prescribed by the Monetary Board except when the failure or refusal of the insurer to pay was founded on the ground that the claim is fraudulent. D. The term "double interest" as used in the Decision of the Court of Appeals must be interpreted to mean 24% per annum. PRUDENTIAL assails the award of interest, granted by the Court of Appeals, in favor of TRANS-ASIA in the assailed Decision of 6 November 2001. It is PRUDENTIALs stance that the award is extortionate and grossly unsconscionable. In support thereto, PRUDENTIAL makes a reference to TRANS-ASIAs prayer in the Complaint filed with the court a quo wherein the latter sought, "interest double the prevailing rate of interest

of 21% per annum now obtaining in the banking business or plus 42% per annum pursuant to Article 243 of the Insurance Code x x x."42 The contention fails to persuade. It is settled that an award of double interest is lawful and justified under Sections 243 and 244 of the Insurance Code. 43 In Finman General Assurance Corporation v. Court of Appeals,44 this Court held that the payment of 24% interest per annum is authorized by the Insurance Code.45 There is no gainsaying that the term "double interest" as used in Sections 243 and 244 can only be interpreted to mean twice 12% per annum or 24% per annum interest, thus: The term "ceiling prescribed by the Monetary Board" means the legal rate of interest of twelve per centum per annum (12%) as prescribed by the Monetary Board in C.B. Circular No. 416, pursuant to P.D. No. 116, amending the Usury Law; so that when Sections 242, 243 and 244 of the Insurance Code provide that the insurer shall be liable to pay interest "twice the ceiling prescribed by the Monetary Board", it means twice 12% per annum or 24% per annum interest on the proceeds of the insurance. 46 E. The payment of double interest should be counted from 13 September 1996. The Court of Appeals, in imposing double interest for the duration of the delay of the payment of the unpaid balance due TRANS-ASIA, computed the same from 13 August 1996 until such time when the amount is fully paid. Although not raised by the parties, we find the computation of the duration of the delay made by the appellate court to be patently erroneous. To be sure, Section 243 imposes interest on the proceeds of the policy for the duration of the delay at the rate of twice the ceiling prescribed by the Monetary Board. Significantly, Section 243 mandates the payment of any loss or damage for which an insurer may be liable, under any policy other than life insurance policy, within thirty days after proof of loss is received by the insurer and ascertainment of the loss or damage is made either by agreement between the insured and the insurer or by arbitration. It is clear that under Section 243, the insurer has until the 30th day after proof of loss and ascertainment of the loss or damage to pay its liability under the insurance, and only after such time can the insurer be held to be in delay, thereby necessitating the imposition of double interest. In the case at bar, it was not disputed that the survey report on the ascertainment of the loss was completed by the adjuster, Richard Hoggs International (Phils.), Inc. on 13 August 1996. PRUDENTIAL had thirty days from 13 August 1996 within which to pay its liability to TRANS-ASIA under the insurance policy, or until 13 September 1996. Therefore, the double interest can begin to run from 13 September 1996 only. IV. A. An interest of 12% per annum is similarly imposed on the TOTAL amount of liability adjudged in section III herein, computed from the time of finality of judgment until the full

satisfaction thereof in conformity with this Courts ruling in Eastern Shipping Lines, Inc. v. Court of Appeals. This Court in Eastern Shipping Lines, Inc. v. Court of Appeals,47 inscribed the rule of thumb48 in the application of interest to be imposed on obligations, regardless of their source. Eastern emphasized beyond cavil that when the judgment of the court awarding a sum of money becomes final and executory, the rate of legal interest, regardless of whether the obligation involves a loan or forbearance of money, shall be 12% per annum from such finality until its satisfaction, this interim period being deemed to be by then an equivalent to a forbearance49 of credit. We find application of the rule in the case at bar proper, thus, a rate of 12% per annum from the finality of judgment until the full satisfaction thereof must be imposed on the total amount of liability adjudged to PRUDENTIAL. It is clear that the interim period from the finality of judgment until the satisfaction of the same is deemed equivalent to a forbearance of credit, hence, the imposition of the aforesaid interest. Fallo WHEREFORE, the Petition in G.R. No. 151890 is DENIED. However, the Petition in G.R. No. 151991 is GRANTED, thus, we award the grant of attorneys fees and make a clarification that the term "double interest" as used in the 6 November 2001 Decision of the Court of Appeals in CA GR CV No. 68278 should be construed to mean interest at the rate of 24% per annum, with a further clarification, that the same should be computed from 13 September 1996 until fully paid. The Decision and Resolution of the Court of Appeals, in CA-G.R. CV No. 68278, dated 6 November 2001 and 29 January 2002, respectively, are, thus, MODIFIED in the following manner, to wit: 1. PRUDENTIAL is DIRECTED to PAY TRANS-ASIA the amount of P8,395,072.26, representing the balance of the loss suffered by TRANS-ASIA and covered by Marine Policy No. MH93/1363; 2. PRUDENTIAL is DIRECTED further to PAY TRANS-ASIA damages in the form of attorneys fees equivalent to 10% of the amount of P8,395,072.26; 3. The aggregate amount (P8,395,072.26 plus 10% thereof as attorneys fees) shall be imposed double interest at the rate of 24% per annum to be computed from 13 September 1996 until fully paid; and 4. An interest of 12% per annum is similarly imposed on the TOTAL amount of liability adjudged as abovestated in paragraphs (1), (2), and (3) herein, computed from the time of finality of judgment until the full satisfaction thereof. No costs. SO ORDERED.

MINITA V. CHICO-NAZARIO Associate Justice WE CONCUR: ARTEMIO V. PANGANIBAN Chief Justice Chairperson CONSUELO YNARES-SANTIAGO Associate Justice MA. ALICIA AUSTRIA-MARTINEZ Asscociate Justice

ROMEO J. CALLEJO, SR. Associate Justice CERTIFICATION Pursuant to Article VIII, Section 13 of the Constitution, it is hereby certified that the conclusions in the above Decision were reached in consultation before the case was assigned to the writer of the opinion of the Courts Division. ARTEMIO V. PANGANIBAN Chief Justice

G.R. NO. 147039

January 27, 2006

DBP POOL OF ACCREDITED INSURANCE COMPANIES, Petitioner, vs. RADIO MINDANAO NETWORK, INC., Respondent. DECISION AUSTRIA-MARTINEZ, J.: This refers to the petition for certiorari under Rule 45 of the Rules of Court seeking the review of the Decision1dated November 16, 2000 of the Court of Appeals (CA) in CAG.R. CV No. 56351, the dispositive portion of which reads: Wherefore, premises considered, the appealed Decision of the Regional Trial Court of Makati City, Branch 138 in Civil Case No. 90-602 is hereby AFFIRMED with MODIFICATION in that the interest rate is hereby reduced to 6% per annum. Costs against the defendants-appellants. SO ORDERED.2 The assailed decision originated from Civil Case No. 90-602 filed by Radio Mindanao Network, Inc. (respondent) against DBP Pool of Accredited Insurance Companies (petitioner) and Provident Insurance Corporation (Provident) for recovery of insurance benefits. Respondent owns several broadcasting stations all over the country. Provident covered respondents transmitter equipment and generating set for the amount ofP13,550,000.00 under Fire Insurance Policy No. 30354, while petitioner covered respondents transmitter, furniture, fixture and other transmitter facilities for the amount of P5,883,650.00 under Fire Insurance Policy No. F-66860. In the evening of July 27, 1988, respondents radio station located in SSS Building, Bacolod City, was razed by fire causing damage in the amount of P1,044,040.00. Respondent sought recovery under the two insurance policies but the claims were

denied on the ground that the cause of loss was an excepted risk excluded under condition no. 6 (c) and (d), to wit: 6. This insurance does not cover any loss or damage occasioned by or through or in consequence, directly or indirectly, of any of the following consequences, namely: (c) War, invasion, act of foreign enemy, hostilities, or warlike operations (whether war be declared or not), civil war. (d) Mutiny, riot, military or popular rising, insurrection, rebellion, revolution, military or usurped power.3 The insurance companies maintained that the evidence showed that the fire was caused by members of the Communist Party of the Philippines/New Peoples Army (CPP/NPA); and consequently, denied the claims. Hence, respondent was constrained to file Civil Case No. 90-602 against petitioner and Provident. After trial on the merits, the Regional Trial Court of Makati, Branch 138, rendered a decision in favor of respondent. The dispositive portion of the decision reads: IN VIEW THEREOF, judgment is rendered in favor of plaintiff. Defendant Provident Insurance Corporation is directed to pay plaintiff the amount of P450,000.00 representing the value of the destroyed property insured under its Fire Insurance Policy plus 12% legal interest from March 2, 1990 the date of the filing of the Complaint. Defendant DBP Pool Accredited Insurance Companies is likewise ordered to pay plaintiff the sum of P602,600.00 representing the value of the destroyed property under its Fire Insurance Policy plus 12% legal interest from March 2, 1990. SO ORDERED.4 Both insurance companies appealed from the trial courts decision but the CA affirmed the decision, with the modification that the applicable interest rate was reduced to 6% per annum. A motion for reconsideration was filed by petitioner DBP which was denied by the CA per its Resolution dated January 30, 2001.5 Hence, herein petition by DBP Pool of Accredited Insurance Companies, 6 with the following assignment of errors: Assignment of Errors THE HONORABLE COURT OF APPEALS ERRED WHEN IT HELD THAT THERE WERE NO SUFFICIENT EVIDENCE SHOWING THAT THE APPROXIMATELY TENTY [sic] (20) ARMED MEN WHO CUSED [sic] THE FIRE AT RESPONDENTS RMN PROPERTY AT BACOLOD CITY WERE MEMBERS OF THE CPP-NPA.

THE HONORABLE COURT OF APPEALS ERRED WHEN IT ADJUDGED THAT RESPONDENT RMN CANNOT BEHELD [sic] FOR DAMAGES AND ATTORNEYS FEES FOR INSTITUTING THE PRESENT ACTION AGAINST THE PETITIONER UNDER ARTICLES 21, 2208, 2229 AND 2232 OF THE CIVIL CODE OF THE PHILIPPINES.7 Petitioner assails the factual finding of both the trial court and the CA that its evidence failed to support its allegation that the loss was caused by an excepted risk, i.e., members of the CPP/NPA caused the fire. In upholding respondents claim for indemnity, the trial court found that: The only evidence which the Court can consider to determine if the fire was due to the intentional act committed by the members of the New Peoples Army (NPA), are the testimony [sic] of witnesses Lt. Col. Nicolas Torres and SPO3 Leonardo Rochar who were admittedly not present when the fire occurred. Their testimony [sic] was [sic] limited to the fact that an investigation was conducted and in the course of the investigation they were informed by bystanders that "heavily armed men entered the transmitter house, poured gasoline in (sic) it and then lighted it. After that, they went out shouting "Mabuhay ang NPA" (TSN, p. 12., August 2, 1995). The persons whom they investigated and actually saw the burning of the station were not presented as witnesses. The documentary evidence particularly Exhibits "5" and "5-C" do not satisfactorily prove that the author of the burning were members of the NPA. Exhibit "5B" which is a letter released by the NPA merely mentions some dissatisfaction with the activities of some people in the media in Bacolod. There was no mention there of any threat on media facilities.8 The CA went over the evidence on record and sustained the findings of the trial court, to wit: To recapitulate, defendants-appellants presented the following to support its claim, to wit: police blotter of the burning of DYHB, certification of the Negros Occidental Integrated National Police, Bacolod City regarding the incident, letter of alleged NPA members Celso Magsilang claiming responsibility for the burning of DYHB, fire investigation report dated July 29, 1988, and the testimonies of Lt. Col. Nicolas Torres and SFO III Leonardo Rochas. We examined carefully the report on the police blotter of the burning of DYHB, the certification issued by the Integrated National Police of Bacolod City and the fire investigation report prepared by SFO III Rochas and there We found that none of them categorically stated that the twenty (20) armed men which burned DYHB were members of the CPP/NPA. The said documents simply stated that the said armed men were believed to be or suspected of being members of the said group. Even SFO III Rochas admitted that he was not sure that the said armed men were members of the CPP-NPA, thus:

In fact the only person who seems to be so sure that that the CPP-NPA had a hand in the burning of DYHB was Lt. Col. Nicolas Torres. However, though We found him to be persuasive in his testimony regarding how he came to arrive at his opinion, We cannot nevertheless admit his testimony as conclusive proof that the CPP-NPA was really involved in the incident considering that he admitted that he did not personally see the armed men even as he tried to pursue them. Note that when Lt. Col. Torres was presented as witness, he was presented as an ordinary witness only and not an expert witness. Hence, his opinion on the identity or membership of the armed men with the CPP-NPA is not admissible in evidence. Anent the letter of a certain Celso Magsilang, who claims to be a member of NPANIROC, being an admission of person which is not a party to the present action, is likewise inadmissible in evidence under Section 22, Rule 130 of the Rules of Court. The reason being that an admission is competent only when the declarant, or someone identified in legal interest with him, is a party to the action.9 The Court will not disturb these factual findings absent compelling or exceptional reasons. It should be stressed that a review by certiorari under Rule 45 is a matter of discretion. Under this mode of review, the jurisdiction of the Court is limited to reviewing only errors of law, not of fact.10 Moreover, when supported by substantial evidence, findings of fact of the trial court as affirmed by the CA are conclusive and binding on the parties,11 which this Court will not review unless there are exceptional circumstances. There are no exceptional circumstances in this case that would have impelled the Court to depart from the factual findings of both the trial court and the CA. Both the trial court and the CA were correct in ruling that petitioner failed to prove that the loss was caused by an excepted risk. Petitioner argues that private respondent is responsible for proving that the cause of the damage/loss is covered by the insurance policy, as stipulated in the insurance policy, to wit: Any loss or damage happening during the existence of abnormal conditions (whether physical or otherwise) which are occasioned by or through in consequence directly or indirectly, of any of the said occurrences shall be deemed to be loss or damage which is not covered by the insurance, except to the extent that the Insured shall prove that such loss or damage happened independently of the existence of such abnormal conditions. In any action, suit or other proceeding where the Companies allege that by reason of the provisions of this condition any loss or damage is not covered by this insurance, the burden of proving that such loss or damage is covered shall be upon the Insured.12

An insurance contract, being a contract of adhesion, should be so interpreted as to carry out the purpose for which the parties entered into the contract which is to insure against risks of loss or damage to the goods. Limitations of liability should be regarded with extreme jealousy and must be construed in such a way as to preclude the insurer from noncompliance with its obligations.13 The "burden of proof" contemplated by the aforesaid provision actually refers to the "burden of evidence" (burden of going forward).14 As applied in this case, it refers to the duty of the insured to show that the loss or damage is covered by the policy. The foregoing clause notwithstanding, the burden of proof still rests upon petitioner to prove that the damage or loss was caused by an excepted risk in order to escape any liability under the contract. Burden of proof is the duty of any party to present evidence to establish his claim or defense by the amount of evidence required by law, which is preponderance of evidence in civil cases. The party, whether plaintiff or defendant, who asserts the affirmative of the issue has the burden of proof to obtain a favorable judgment. For the plaintiff, the burden of proof never parts.15 For the defendant, an affirmative defense is one which is not a denial of an essential ingredient in the plaintiffs cau se of action, but one which, if established, will be a good defense i.e. an "avoidance" of the claim.16 Particularly, in insurance cases, where a risk is excepted by the terms of a policy which insures against other perils or hazards, loss from such a risk constitutes a defense which the insurer may urge, since it has not assumed that risk, and from this it follows that an insurer seeking to defeat a claim because of an exception or limitation in the policy has the burden of proving that the loss comes within the purview of the exception or limitation set up. If a proof is made of a loss apparently within a contract of insurance, the burden is upon the insurer to prove that the loss arose from a cause of loss which is excepted or for which it is not liable, or from a cause which limits its liability.17 Consequently, it is sufficient for private respondent to prove the fact of damage or loss. Once respondent makes out a prima facie case in its favor, the duty or the burden of evidence shifts to petitioner to controvert respondents prima facie case. 18 In this case, since petitioner alleged an excepted risk, then the burden of evidence shifted to petitioner to prove such exception. It is only when petitioner has sufficiently proven that the damage or loss was caused by an excepted risk does the burden of evidence shift back to respondent who is then under a duty of producing evidence to show why such excepted risk does not release petitioner from any liability. Unfortunately for petitioner, it failed to discharge its primordial burden of proving that the damage or loss was caused by an excepted risk. Petitioner however, insists that the evidence on record established the identity of the author of the damage. It argues that the trial court and the CA erred in not appreciating the reports of witnesses Lt. Col Torres and SFO II Rochar that the bystanders they

interviewed claimed that the perpetrators were members of the CPP/NPA as an exception to the hearsay rule as part of res gestae. A witness can testify only to those facts which he knows of his personal knowledge, which means those facts which are derived from his perception. 19 A witness may not testify as to what he merely learned from others either because he was told or read or heard the same. Such testimony is considered hearsay and may not be received as proof of the truth of what he has learned. The hearsay rule is based upon serious concerns about the trustworthiness and reliability of hearsay evidence inasmuch as such evidence are not given under oath or solemn affirmation and, more importantly, have not been subjected to cross-examination by opposing counsel to test the perception, memory, veracity and articulateness of the out-of-court declarant or actor upon whose reliability on which the worth of the out-of-court statement depends.20 Res gestae, as an exception to the hearsay rule, refers to those exclamations and statements made by either the participants, victims, or spectators to a crime immediately before, during, or after the commission of the crime, when the circumstances are such that the statements were made as a spontaneous reaction or utterance inspired by the excitement of the occasion and there was no opportunity for the declarant to deliberate and to fabricate a false statement. The rule in res gestae applies when the declarant himself did not testify and provided that the testimony of the witness who heard the declarant complies with the following requisites: (1) that the principal act, the res gestae, be a startling occurrence; (2) the statements were made before the declarant had the time to contrive or devise a falsehood; and (3) that the statements must concern the occurrence in question and its immediate attending circumstances.21 The Court is not convinced to accept the declarations as part of res gestae. While it may concede that these statements were made by the bystanders during a startling occurrence, it cannot be said however, that these utterances were made spontaneously by the bystanders and before they had the time to contrive or devise a falsehood. Both SFO III Rochar and Lt. Col. Torres received the bystanders statements while they were making their investigations during and after the fire. It is reasonable to assume that when these statements were noted down, the bystanders already had enough time and opportunity to mill around, talk to one another and exchange information, not to mention theories and speculations, as is the usual experience in disquieting situations where hysteria is likely to take place. It cannot therefore be ascertained whether these utterances were the products of truth. That the utterances may be mere idle talk is not remote. At best, the testimonies of SFO III Rochar and Lt. Col. Torres that these statements were made may be considered as independently relevant statements gathered in the course of their investigation, and are admissible not as to the veracity thereof but to the fact that they had been thus uttered.22

Furthermore, admissibility of evidence should not be equated with its weight and sufficiency.23 Admissibility of evidence depends on its relevance and competence, while the weight of evidence pertains to evidence already admitted and its tendency to convince and persuade.24 Even assuming that the declaration of the bystanders that it was the members of the CPP/NPA who caused the fire may be admitted as evidence, it does not follow that such declarations are sufficient proof. These declarations should be calibrated vis--vis the other evidence on record. And the trial court aptly noted that there is a need for additional convincing proof, viz.: The Court finds the foregoing to be insufficient to establish that the cause of the fire was the intentional burning of the radio facilities by the rebels or an act of insurrection, rebellion or usurped power. Evidence that persons who burned the radio facilities shouted "Mabuhay ang NPA" does not furnish logical conclusion that they are member [sic] of the NPA or that their act was an act of rebellion or insurrection. Additional convincing proof need be submitted. Defendants failed to discharge their responsibility to present adequate proof that the loss was due to a risk excluded.25 While the documentary evidence presented by petitioner, i.e., (1) the police blotter; (2) the certification from the Bacolod Police Station; and (3) the Fire Investigation Report may be considered exceptions to the hearsay rule, being entries in official records, nevertheless, as noted by the CA, none of these documents categorically stated that the perpetrators were members of the CPP/NPA.26 Rather, it was stated in the police blotter that: "a group of persons accompanied by one (1) woman all believed to be CPP/NPA more or less 20 persons suspected to be CPP/NPA,"27 while the certification from the Bacolod Police station stated that " some 20 or more armed men believed to be members of the New Peoples Army NPA,"28 and the fire investigation report concluded that "(I)t is therefore believed by this Investigating Team that the cause of the fire is intentional, and the armed mensuspected to be members of the CPP/NPA where (sic) the ones responsible "29 All these documents show that indeed, the "suspected" executor of the fire were believed to be members of the CPP/NPA. But suspicion alone is not sufficient, preponderance of evidence being the quantum of proof. All told, the Court finds no reason to grant the present petition. WHEREFORE, the petition is DISMISSED. The Court of Appeals Decision dated November 16, 2000 and Resolution dated January 30, 2001 rendered in CA-G.R. CV No. 56351 are AFFIRMED in toto. SO ORDERED. MA. ALICIA AUSTRIA-MARTINEZ Associate Justice WE CONCUR:

ARTEMIO V. PANGANIBAN Chief Justice Chairman CONSUELO YNARES-SANTIAGO Associate Justice ROMEO J. CALLEJO, SR. Asscociate Justice

MINITA V. CHICO-NAZARIO Associate Justice CERTIFICATION Pursuant to Section 13, Article VIII of the Constitution, it is hereby certified that the conclusions in the above Decision were reached in consultation before the case was assigned to the writer of the opinion of the Courts Division. ARTEMIO V. PANGANIBAN Chief Justice

G.R. No. 48049 June 29, 1989 EMILIO TAN, JUANITO TAN, ALBERTO TAN and ARTURO TAN, petitioners, vs. THE COURT OF APPEALS and THE PHILIPPINE AMERICAN LIFE INSURANCE COMPANY, respondents. O.F. Santos & P.C. Nolasco for petitioners. Ferry, De la Rosa and Associates for private respondent. GUTIERREZ, JR., J.: This is a petition for review on certiorari of the Court of Appeals' decision affirming the decision of the Insurance Commissioner which dismissed the petitioners' complaint against respondent Philippine American Life Insurance Company for the recovery of the proceeds from their late father's policy. The facts of the case as found by the Court of Appeals are: Petitioners appeal from the Decision of the Insurance Commissioner dismissing herein petitioners' complaint against respondent Philippine American Life Insurance Company for the recovery of the proceeds of Policy No. 1082467 in the amount of P 80,000.00. On September 23,1973, Tan Lee Siong, father of herein petitioners, applied for life insurance in the amount of P 80,000.00 with respondent company. Said application was approved and Policy No. 1082467 was issued effective November 6,1973, with petitioners the beneficiaries thereof (Exhibit A). On April 26,1975, Tan Lee Siong died of hepatoma (Exhibit B). Petitioners then filed with respondent company their claim for the proceeds of the life insurance policy. However, in a letter dated September 11, 1975, respondent company denied petitioners' claim and rescinded the policy by reason of the alleged misrepresentation and concealment of material facts made by the deceased Tan Lee Siong in his application for insurance (Exhibit 3). The premiums paid on the policy were thereupon refunded .

Alleging that respondent company's refusal to pay them the proceeds of the policy was unjustified and unreasonable, petitioners filed on November 27, 1975, a complaint against the former with the Office of the Insurance Commissioner, docketed as I.C. Case No. 218. After hearing the evidence of both parties, the Insurance Commissioner rendered judgment on August 9, 1977, dismissing petitioners' complaint. (Rollo, pp. 91-92) The Court of Appeals dismissed ' the petitioners' appeal from the Insurance Commissioner's decision for lack of merit Hence, this petition. The petitioners raise the following issues in their assignment of errors, to wit: A. The conclusion in law of respondent Court that respondent insurer has the right to rescind the policy contract when insured is already dead is not in accordance with existing law and applicable jurisprudence. B. The conclusion in law of respondent Court that respondent insurer may be allowed to avoid the policy on grounds of concealment by the deceased assured, is contrary to the provisions of the policy contract itself, as well as, of applicable legal provisions and established jurisprudence. C. The inference of respondent Court that respondent insurer was misled in issuing the policy are manifestly mistaken and contrary to admitted evidence. (Rollo, p. 7) The petitioners contend that the respondent company no longer had the right to rescind the contract of insurance as rescission must allegedly be done during the lifetime of the insured within two years and prior to the commencement of action. The contention is without merit. The pertinent section in the Insurance Code provides: Section 48. Whenever a right to rescind a contract of insurance is given to the insurer by any provision of this chapter, such right must be exercised previous to the commencement of an action on the contract. After a policy of life insurance made payable on the death of the insured shall have been in force during the lifetime of the insured for a period of two years from the date of its issue or of its last reinstatement, the insurer cannot prove that the policy is void ab initio or is rescindable by reason of

the fraudulent concealment or misrepresentation of the insured or his agent. According to the petitioners, the Insurance Law was amended and the second paragraph of Section 48 added to prevent the insurance company from exercising a right to rescind after the death of the insured. The so-called "incontestability clause" precludes the insurer from raising the defenses of false representations or concealment of material facts insofar as health and previous diseases are concerned if the insurance has been in force for at least two years during the insured's lifetime. The phrase "during the lifetime" found in Section 48 simply means that the policy is no longer considered in force after the insured has died. The key phrase in the second paragraph of Section 48 is "for a period of two years." As noted by the Court of Appeals, to wit: The policy was issued on November 6,1973 and the insured died on April 26,1975. The policy was thus in force for a period of only one year and five months. Considering that the insured died before the two-year period had lapsed, respondent company is not, therefore, barred from proving that the policy is void ab initio by reason of the insured's fraudulent concealment or misrepresentation. Moreover, respondent company rescinded the contract of insurance and refunded the premiums paid on September 11, 1975, previous to the commencement of this action on November 27,1975. (Rollo, pp. 99-100) xxx xxx xxx The petitioners contend that there could have been no concealment or misrepresentation by their late father because Tan Lee Siong did not have to buy insurance. He was only pressured by insistent salesmen to do so. The petitioners state: Here then is a case of an assured whose application was submitted because of repeated visits and solicitations by the insurer's agent. Assured did not knock at the door of the insurer to buy insurance. He was the object of solicitations and visits. Assured was a man of means. He could have obtained a bigger insurance, not just P 80,000.00. If his purpose were to misrepresent and to conceal his ailments in anticipation of death during the two-year period, he certainly could have gotten a bigger insurance. He did not. Insurer Philamlife could have presented as witness its Medical Examiner Dr. Urbano Guinto. It was he who accomplished the application, Part II, medical. Philamlife did not.

Philamlife could have put to the witness stand its Agent Bienvenido S. Guinto, a relative to Dr. Guinto, Again Philamlife did not. (pp. 138139, Rollo) xxx xxx xxx This Honorable Supreme Court has had occasion to denounce the pressure and practice indulged in by agents in selling insurance. At one time or another most of us have been subjected to that pressure, that practice. This court took judicial cognizance of the whirlwind pressure of insurance selling-especially of the agent's practice of 'supplying the information, preparing and answering the application, submitting the application to their companies, concluding the transactions and otherwisesmoothing out all difficulties. We call attention to what this Honorable Court said in Insular Life v. Feliciano, et al., 73 Phil. 201; at page 205: It is of common knowledge that the selling of insurance today is subjected to the whirlwind pressure of modern salesmanship. Insurance companies send detailed instructions to their agents to solicit and procure applications. These agents are to be found all over the length and breadth of the land. They are stimulated to more active efforts by contests and by the keen competition offered by the other rival insurance companies. They supply all the information, prepare and answer the applications, submit the applications to their companies, conclude the transactions, and otherwise smooth out all difficulties. The agents in short do what the company set them out to do. The Insular Life case was decided some forty years ago when the pressure of insurance salesmanship was not overwhelming as it is now; when the population of this country was less than one-fourth of what it is now; when the insurance companies competing with one another could be counted by the fingers. (pp. 140-142, Rollo) xxx xxx xxx In the face of all the above, it would be unjust if, having been subjected to the whirlwind pressure of insurance salesmanship this Court itself has long denounced, the assured who dies within the two-year period, should

stand charged of fraudulent concealment and misrepresentation." (p. 142, Rollo) The legislative answer to the arguments posed by the petitioners is the "incontestability clause" added by the second paragraph of Section 48. The insurer has two years from the date of issuance of the insurance contract or of its last reinstatement within which to contest the policy, whether or not, the insured still lives within such period. After two years, the defenses of concealment or misrepresentation, no matter how patent or well founded, no longer lie. Congress felt this was a sufficient answer to the various tactics employed by insurance companies to avoid liability. The petitioners' interpretation would give rise to the incongruous situation where the beneficiaries of an insured who dies right after taking out and paying for a life insurance policy, would be allowed to collect on the policy even if the insured fraudulently concealed material facts. The petitioners argue that no evidence was presented to show that the medical terms were explained in a layman's language to the insured. They state that the insurer should have presented its two medical field examiners as witnesses. Moreover, the petitioners allege that the policy intends that the medical examination must be conducted before its issuance otherwise the insurer "waives whatever imperfection by ratification." We agree with the Court of Appeals which ruled: On the other hand, petitioners argue that no evidence was presented by respondent company to show that the questions appearing in Part II of the application for insurance were asked, explained to and understood by the deceased so as to prove concealment on his part. The same is not well taken. The deceased, by affixing his signature on the application form, affirmed the correctness of all the entries and answers appearing therein. It is but to be expected that he, a businessman, would not have affixed his signature on the application form unless he clearly understood its significance. For, the presumption is that a person intends the ordinary consequence of his voluntary act and takes ordinary care of his concerns. [Sec. 5(c) and (d), Rule 131, Rules of Court]. The evidence for respondent company shows that on September 19,1972, the deceased was examined by Dr. Victoriano Lim and was found to be diabetic and hypertensive; that by January, 1973, the deceased was complaining of progressive weight loss and abdominal pain and was diagnosed to be suffering from hepatoma, (t.s.n. August 23, 1976, pp. 810; Exhibit 2). Another physician, Dr. Wenceslao Vitug, testified that the deceased came to see him on December 14, 1973 for consolation and claimed to have been diabetic for five years. (t.s.n., Aug. 23,1976, p. 5; Exhibit 6) Because of the concealment made by the deceased of his consultations and treatments for hypertension, diabetes and liver

disorders, respondent company was thus misled into accepting the risk and approving his application as medically standard (Exhibit 5- C) and dispensing with further medical investigation and examination (Exhibit 5A). For as long as no adverse medical history is revealed in the application form, an applicant for insurance is presumed to be healthy and physically fit and no further medical investigation or examination is conducted by respondent company. (t.s.n., April 8,1976, pp. 6-8). (Rollo, pp. 96-98) There is no strong showing that we should apply the "fine print" or "contract of adhesion" rule in this case. (Sweet Lines, Inc. v. Teves, 83 SCRA 361 [1978]). The petitioners cite: It is a matter of common knowledge that large amounts of money are collected from ignorant persons by companies and associations which adopt high sounding titles and print the amount of benefits they agree to pay in large black-faced type, following such undertakings by fine print conditions which destroy the substance of the promise. All provisions, conditions, or exceptions which in any way tend to work a forfeiture of the policy should be construed most strongly against those for whose benefit they are inserted, and most favorably toward those against whom they are meant to operate. (Trinidad v. Orient Protective Assurance Assn., 67 Phil. 184) There is no showing that the questions in the application form for insurance regarding the insured's medical history are in smaller print than the rest of the printed form or that they are designed in such a way as to conceal from the applicant their importance. If a warning in bold red letters or a boxed warning similar to that required for cigarette advertisements by the Surgeon General of the United States is necessary, that is for Congress or the Insurance Commission to provide as protection against high pressure insurance salesmanship. We are limited in this petition to ascertaining whether or not the respondent Court of Appeals committed reversible error. It is the petitioners' burden to show that the factual findings of the respondent court are not based on substantial evidence or that its conclusions are contrary to applicable law and jurisprudence. They have failed to discharge that burden. WHEREFORE, the petition is hereby DENIED for lack of merit. The questioned decision of the Court of Appeals is AFFIRMED. SO ORDERED. Fernan, (C.J., Chairman), Bidin and Cortes, JJ., concur. Feliciano, took no part. G.R. No. 186983 February 22, 2012

MA. LOURDES S. FLORENDO, Petitioner, vs. PHILAM PLANS, INC., PERLA ABCEDE MA. CELESTE ABCEDE, Respondents. DECISION ABAD, J.: This case is about an insureds alleged concealment in his pension plan application of his true state of health and its effect on the life insurance portion of that plan in case of death. The Facts and the Case On October 23, 1997 Manuel Florendo filed an application for comprehensive pension plan with respondent Philam Plans, Inc. (Philam Plans) after some convincing by respondent Perla Abcede. The plan had a pre-need price of P997,050.00, payable in 10 years, and had a maturity value of P2,890,000.00 after 20 years.1 Manuel signed the application and left to Perla the task of supplying the information needed in the application.2Respondent Ma. Celeste Abcede, Perlas daughter, signed th e application as sales counselor.3 Aside from pension benefits, the comprehensive pension plan also provided life insurance coverage to Florendo.4This was covered by a Group Master Policy that Philippine American Life Insurance Company (Philam Life) issued to Philam Plans.5 Under the master policy, Philam Life was to automatically provide life insurance coverage, including accidental death, to all who signed up for Philam Plans comprehensive pension plan.6 If the plan holder died before the maturity of the plan, his beneficiary was to instead receive the proceeds of the life insurance, equivalent to the pre-need price. Further, the life insurance was to take care of any unpaid premium until the pension plan matured, entitling the beneficiary to the maturity value of the pension plan.7 On October 30, 1997 Philam Plans issued Pension Plan Agreement PP43005584 8 to Manuel, with petitioner Ma. Lourdes S. Florendo, his wife, as beneficiary. In time, Manuel paid his quarterly premiums.9 Eleven months later or on September 15, 1998, Manuel died of blood poisoning. Subsequently, Lourdes filed a claim with Philam Plans for the payment of the benefits under her husbands plan.10 Because Manuel died before his pension plan matured and his wife was to get only the benefits of his life insurance, Philam Plans forwarded her claim to Philam Life.11 On May 3, 1999 Philam Plans wrote Lourdes a letter,12 declining her claim. Philam Life found that Manuel was on maintenance medicine for his heart and had an implanted pacemaker. Further, he suffered from diabetes mellitus and was taking insulin. Lourdes

renewed her demand for payment under the plan13 but Philam Plans rejected it,14prompting her to file the present action against the pension plan company before the Regional Trial Court (RTC) of Quezon City.15 On March 30, 2006 the RTC rendered judgment,16 ordering Philam Plans, Perla and Ma. Celeste, solidarily, to pay Lourdes all the benefits from her husbands pension plan, namely: P997,050.00, the proceeds of his term insurance, and P2,890,000.00 lump sum pension benefit upon maturity of his plan; P100,000.00 as moral damages; and to pay the costs of the suit. The RTC ruled that Manuel was not guilty of concealing the state of his health from his pension plan application. On December 18, 2007 the Court of Appeals (CA) reversed the RTC decision, 17 holding that insurance policies are traditionally contracts uberrimae fidae or contracts of utmost good faith. As such, it required Manuel to disclose to Philam Plans conditions affecting the risk of which he was aware or material facts that he knew or ought to know. 18 Issues Presented The issues presented in this case are: 1. Whether or not the CA erred in finding Manuel guilty of concealing his illness when he kept blank and did not answer questions in his pension plan application regarding the ailments he suffered from; 2. Whether or not the CA erred in holding that Manuel was bound by the failure of respondents Perla and Ma. Celeste to declare the condition of Manuels health in the pension plan application; and 3. Whether or not the CA erred in finding that Philam Plans approval of Manuels pension plan application and acceptance of his premium payments precluded it from denying Lourdes claim. Rulings of the Court One. Lourdes points out that, seeing the unfilled spaces in Manuels pension plan application relating to his medical history, Philam Plans should have returned it to him for completion. Since Philam Plans chose to approve the application just as it was, it cannot cry concealment on Manuels part. Further, Lourdes adds that Philam Plans never queried Manuel directly regarding the state of his health. Consequently, it could not blame him for not mentioning it.19 But Lourdes is shifting to Philam Plans the burden of putting on the pension plan application the true state of Manuels health. She forgets that since Philam Plans waived medical examination for Manuel, it had to rely largely on his stating the truth regarding his health in his application. For, after all, he knew more than anyone that he had been under treatment for heart condition and diabetes for more than five years

preceding his submission of that application. But he kept those crucial facts from Philam Plans. Besides, when Manuel signed the pension plan application, he adopted as his own the written representations and declarations embodied in it. It is clear from these representations that he concealed his chronic heart ailment and diabetes from Philam Plans. The pertinent portion of his representations and declarations read as follows: I hereby represent and declare to the best of my knowledge that: xxxx (c) I have never been treated for heart condition, high blood pressure, cancer, diabetes, lung, kidney or stomach disorder or any other physical impairment in the last five years. (d) I am in good health and physical condition. If your answer to any of the statements above reveal otherwise, please give details in the space provided for: Date of confinement : ____________________________ Name of Hospital or Clinic : ____________________________ Name of Attending Physician : ____________________________ Findings : ____________________________ Others: (Please specify) : ____________________________ x x x x.20 (Emphasis supplied) Since Manuel signed the application without filling in the details regarding his continuing treatments for heart condition and diabetes, the assumption is that he has never been treated for the said illnesses in the last five years preceding his application. This is implicit from the phrase "If your answer to any of the statements above (specifically, the statement: I have never been treated for heart condition or diabetes) reveal otherwise, please give details in the space provided for." But this is untrue since he had been on "Coumadin," a treatment for venous thrombosis,21 and insulin, a drug used in the treatment of diabetes mellitus, at that time.22 Lourdes insists that Manuel had concealed nothing since Perla, the soliciting agent, knew that Manuel had a pacemaker implanted on his chest in the 70s or about 20 years before he signed up for the pension plan.23 But by its tenor, the responsibility for preparing the application belonged to Manuel. Nothing in it implies that someone else

may provide the information that Philam Plans needed. Manuel cannot sign the application and disown the responsibility for having it filled up. If he furnished Perla the needed information and delegated to her the filling up of the application, then she acted on his instruction, not on Philam Plans instruction. Lourdes next points out that it made no difference if Manuel failed to reveal the fact that he had a pacemaker implant in the early 70s since this did not fall within the five-year timeframe that the disclosure contemplated.24 But a pacemaker is an electronic device implanted into the body and connected to the wall of the heart, designed to provide regular, mild, electric shock that stimulates the contraction of the heart muscles and restores normalcy to the heartbeat.25 That Manuel still had his pacemaker when he applied for a pension plan in October 1997 is an admission that he remained under treatment for irregular heartbeat within five years preceding that application. Besides, as already stated, Manuel had been taking medicine for his heart condition and diabetes when he submitted his pension plan application. These clearly fell within the five-year period. More, even if Perlas knowledge of Manuels pacemaker may be applied to Philam Plans under the theory of imputed knowledge, 26 it is not claimed that Perla was aware of his two other afflictions that needed medical treatments. Pursuant to Section 2727 of the Insurance Code, Manuels concealment entitles Philam Plans to rescind its contract of insurance with him. Two. Lourdes contends that the mere fact that Manuel signed the application in blank and let Perla fill in the required details did not make her his agent and bind him to her concealment of his true state of health. Since there is no evidence of collusion between them, Perlas fault must be considered solely her own and cannot prejudice Manuel. 28 But Manuel forgot that in signing the pension plan application, he certified that he wrote all the information stated in it or had someone do it under his direction. Thus: APPLICATION FOR PENSION PLAN (Comprehensive) I hereby apply to purchase from PHILAM PLANS, INC. a Pension Plan Program described herein in accordance with the General Provisions set forth in this application and hereby certify that the date and other information stated herein are written by me or under my direction. x x x.29 (Emphasis supplied) Assuming that it was Perla who filled up the application form, Manuel is still bound by what it contains since he certified that he authorized her action. Philam Plans had every right to act on the faith of that certification. Lourdes could not seek comfort from her claim that Perla had assured Manuel that the state of his health would not hinder the approval of his application and that what is written on his application made no difference to the insurance company. But, indubitably, Manuel was made aware when he signed the pension plan application that,

in granting the same, Philam Plans and Philam Life were acting on the truth of the representations contained in that application. Thus: DECLARATIONS AND REPRESENTATIONS xxxx I agree that the insurance coverage of this application is based on the truth of the foregoing representations and is subject to the provisions of the Group Life Insurance Policy issued by THE PHILIPPINE AMERICAN LIFE INSURANCE CO. to PHILAM PLANS, INC.30 (Emphasis supplied) As the Court said in New Life Enterprises v. Court of Appeals:31 It may be true that x x x insured persons may accept policies without reading them, and that this is not negligence per se. But, this is not without any exception. It is and was incumbent upon petitioner Sy to read the insurance contracts, and this can be reasonably expected of him considering that he has been a businessman since 1965 and the contract concerns indemnity in case of loss in his money-making trade of which important consideration he could not have been unaware as it was precisely the reason for his procuring the same.32 The same may be said of Manuel, a civil engineer and manager of a construction company.33 He could be expected to know that one must read every document, especially if it creates rights and obligations affecting him, before signing the same. Manuel is not unschooled that the Court must come to his succor. It could reasonably be expected that he would not trifle with something that would provide additional financial security to him and to his wife in his twilight years. Three. In a final attempt to defend her claim for benefits under Manuels pension plan, Lourdes points out that any defect or insufficiency in the information provided by his pension plan application should be deemed waived after the same has been approved, the policy has been issued, and the premiums have been collected. 34 The Court cannot agree. The comprehensive pension plan that Philam Plans issued contains a one-year incontestability period. It states: VIII. INCONTESTABILITY After this Agreement has remained in force for one (1) year, we can no longer contest for health reasons any claim for insurance under this Agreement, except for the reason that installment has not been paid (lapsed), or that you are not insurable at the time you bought this pension program by reason of age. If this Agreement lapses but is reinstated afterwards, the one (1) year contestability period shall start again on the date of approval of your request for reinstatement.351wphi1

The above incontestability clause precludes the insurer from disowning liability under the policy it issued on the ground of concealment or misrepresentation regarding the health of the insured after a year of its issuance. Since Manuel died on the eleventh month following the issuance of his plan, 36 the one year incontestability period has not yet set in. Consequently, Philam Plans was not barred from questioning Lourdes entitlement to the benefits of her husbands pension plan. WHEREFORE, the Court AFFIRMS in its entirety the decision of the Court of Appeals in CA-G.R. CV 87085 dated December 18, 2007. SO ORDERED. ROBERTO A. ABAD Associate Justice WE CONCUR: PRESBITERO J. VELASCO, JR. Associate Justice Chairperson DIOSDADO M. PERALTA Associate Justice JOSE CATRAL MENDOZA Associate Justice

ESTELA M. PERLAS-BERNABE Associate Justice ATTESTATION I attest that the conclusions in the above Decision had been reached in consultation before the case was assigned to the writer of the opinion of the Courts Division. PRESBITERO J. VELASCO, JR. Associate Justice Chairperson, Third Division CERTIFICATION Pursuant to Section 13, Article VIII of the Constitution and the Division Chairpersons Attestation, I certify that the conclusions in the above Decision had been reached in consultation before the case was assigned to the writer of the opinion of the Courts Division.

RENATO C. CORONA Chief Justice

G.R. No. 180440

December 5, 2012

DR. GENEVIEVE L. HUANG, Petitioner, vs. PHILIPPINE HOTELIERS, INC., DUSIT THANI PUBLIC CO., LTD. And FIRST LEPANTO TAISHO INSURANCE CORPORATION, Respondents. DECISION PEREZ, J.: For this Courts resolution is a Petition for Review on Certiorari under Rule 45 of the Rules of Court, assailing the Decision1 of the Court of Appeals in CA-G.R. CV No. 87065 dated 9 August 2007, affirming the Decision2 of Branch 56 of the Regional Trial Court (RTC) of Makati City in Civil Case No. 96-1367 dated 21 February 2006,

dismissing for lack of merit herein petitioner Dr. Genevieve L. Huangs Complaint for Damages. Assailed as well is the Court of Appeals Resolution3 dated 5 November 2007 denying for lack of merit petitioners Motion for Reconsideration. This case stemmed from a Complaint for Damages filed on 28 August 1996 by petitioner Dr. Genevieve L. Huang4against herein respondents Philippine Hoteliers, Inc. (PHI)5 and Dusit Thani Public Co., Ltd. (DTPCI),6 as owners of Dusit Thani Hotel Manila (Dusit Hotel);7 and co-respondent First Lepanto Taisho Insurance Corporation (First Lepanto),8 as insurer of the aforesaid hotel. The said Complaint was premised on the alleged negligence of respondents PHI and DTPCIs staff, in the untimely putting off all the lights within the hotels swimming pool area, as well as the locking of the main entrance door of the area, prompting petitioner to grope for a way out. While doing so, a folding wooden counter top fell on her head causing her serious brain injury. The negligence was allegedly compounded by respondents PHI and DTPCIs failure to render prompt and adequate medical assistance. Petitioners version of the antecedents of this case is as follows: On 11 June 1995, Delia Goldberg (Delia), a registered guest of Dusit Hotel, invited her friend, petitioner Dr. Genevieve L. Huang, for a swim at the hotels swimming pool facility. They started bathing at around 5:00 p.m. At around 7:00 p.m., the hotels swimming pool attendant informed them that the swimming pool area was about to be closed. The two subsequently proceeded to the shower room adjacent to the swimming pool to take a shower and dress up. However, when they came out of the bathroom, the entire swimming pool area was already pitch black and there was no longer any person around but the two of them. They carefully walked towards the main door leading to the hotel but, to their surprise, the door was locked.9 Petitioner and Delia waited for 10 more minutes near the door hoping someone would come to their rescue but they waited in vain. Delia became anxious about their situation so petitioner began to walk around to look for a house phone. Delia followed petitioner. After some time, petitioner saw a phone behind the lifeguards counter. While slowly walking towards the phone, a hard and heavy object, which later turned out to be the folding wooden counter top, fell on petitioners head that knocked her down almost unconscious.10 Delia immediately got hold of the house phone and notified the hotel telephone operator of the incident. Not long after, the hotel staff arrived at the main entrance door of the swimming pool area but it took them at least 20 to 30 minutes to get inside. When the door was finally opened, three hotel chambermaids assisted petitioner by placing an ice pack and applying some ointment on her head. After petitioner had slightly recovered, she requested to be assisted to the hotels coffee shop to have some rest. Petitioner demanded the services of the hotel physician.11 Dr. Violeta Dalumpines (Dr. Dalumpines) arrived. She approached petitioner and introduced herself as the hotel physician. However, instead of immediately providing the

needed medical assistance, Dr. Dalumpines presented a "Waiver" and demanded that it be signed by petitioner, otherwise, the hotel management will not render her any assistance. Petitioner refused to do so.12 After eating her dinner and having rested for a while, petitioner left the hotels coffee shop and went home. Thereupon, petitioner started to feel extraordinary dizziness accompanied by an uncomfortable feeling in her stomach, which lasted until the following day. Petitioner was constrained to stay at home, thus, missing all her important appointments with her patients. She also began experiencing "on" and "off" severe headaches that caused her three (3) sleepless nights.13 Petitioner, thus, decided to consult a certain Dr. Perry Noble (Dr. Noble), a neurologist from Makati Medical Center, who required her to have an X-ray and a Magnetic Resonance Imaging (MRI) tests.14 The MRI Report15dated 23 August 1995 revealed the following findings: CONSULTATION REPORT: MRI examination of the brain shows scattered areas of intraparenchymal contusions and involving mainly the left middle and posterior temporal and slightly the right anterior temporal lobe. Other small areas of contusions with suggestive pertechiae are seen in the left frontoparietal, left parieto-occipital and with deep frontal periventricular subcortical and cortical regions. There is no mass effect nor signs of localized hemorrhagic extravasation. The ventricles are not enlarged, quite symmetrical without shifts or deformities; the peripheral sulci are within normal limits. The C-P angles, petromastoids, sella, extrasellar and retro orbital areas appear normal. The brainstem is unremarkable. IMPRESSION: Scattered small intraparenchymal contusions mainly involving the left middle-posterior temporal lobe and also right medial anterior temporal, both deep frontal subcortical, left parieto-occipital subcortical and cortical regions. Ischemic etiology not ruled out. No localized intra - or extracerebral hemorrhage.16 Petitioner claimed that the aforesaid MRI result clearly showed that her head was bruised. Based also on the same MRI result, Dr. Noble told her that she has a very serious brain injury. In view thereof, Dr. Noble prescribed the necessary medicine for her condition.17 Petitioner likewise consulted a certain Dr. Ofelia Adapon, also a neurologist from Makati Medical Center, who required her to undergo an Electroencephalogram examination

(EEG) to measure the electrostatic in her brain.18Based on its result,19 Dr. Ofelia Adapon informed her that she has a serious condition a permanent one. Dr. Ofelia Adapon similarly prescribed medicines for her brain injury.20 Petitioners condition did not get better. Hence, sometime in September 1995, she consulted another neuro-surgeon by the name of Dr. Renato Sibayan (Dr. Sibayan), who required her to have an X-ray test.21 According to petitioner, Dr. Sibayans finding was the same as those of the previous doctors that she had consultedshe has a serious brain injury.22 By reason of the unfortunate 11 June 1995 incident inside the ho tels swimming pool area, petitioner also started to feel losing her memory, which greatly affected and disrupted the practice of her chosen profession.23 Thus, on 25 October 1995, petitioner, through counsel, sent a demand letter24 to respondents PHI and DTPCI seeking payment of an amount not less than P100,000,000.00 representing loss of earnings on her remaining life span. But, petitioners demand was unheeded. In November 1995, petitioner went to the United States of America (USA) for further medical treatment. She consulted a certain Dr. Gerald Steinberg and a certain Dr. Joel Dokson25 from Mount Sinai Hospital who both found that she has "post traumatic-post concussion/contusion cephalgias-vascular and neuralgia."26 She was then prescribed to take some medications for severe pain and to undergo physical therapy. Her condition did not improve so she returned to the Philippines.27 Petitioner, once again, consulted Dr. Sibayan, who simply told her to just relax and to continue taking her medicines. Petitioner also consulted other neurologists, who all advised her to just continue her medications and to undergo physical therapy for her neck pain.28 Sometime in 1996, petitioner consulted as well a certain Dr. Victor Lopez (Dr. Lopez), an ophthalmologist from the Makati Medical Center, because of her poor vision, which she has experienced for several months.29 Petitioners Eye Report dated 5 March 199630 issued by Dr. Lopez stated: "IMPRESSION: Posterior vitreous detachment, right eye of floaters." Dr. Lopez told petitioner that her detached eye is permanent and very serious. Dr. Lopez then prescribed an eye drop to petitioner.31 For petitioners frustration to dissipate and to regain her former strength and physical well-being, she consulted another neuro-surgeon from Makati Medical Center by the name of Dr. Leopoldo P. Pardo, Jr. (Dr. Pardo, Jr.).32She disclosed to Dr. Pardo, Jr. that at the age of 18 she suffered a stroke due to mitral valve disease and that she was given treatments, which also resulted in thrombocytopenia. In Dr. Pardo, Jr.s medical evaluation of petitioner dated 15 May 1996,33 he made the following diagnosis and opinion: DIAGNOSIS AND OPINION:

This patient sustained a severe head injury in (sic) 11 June 1995 and as a result of which she developed the following injuries: 1. Cerebral Concussion and Contusion 2. Post-traumatic Epilepsy 3. Post-concussional Syndrome 4. Minimal Brain Dysfunction 5. Cervical Sprain, chronic recurrent It is my opinion that the symptoms she complained of in the foregoing history are all related to and a result of the injury sustained on 11 June 1995. It is further my opinion that the above diagnosis and complaints do materially affect her duties and functions as a practicing physician and dermatologist, and that she will require treatment for an undetermined period of time. The percentage of disability is not calculated at this time and will require further evaluation and observation.34 Dr. Pardo, Jr. then advised petitioner to continue her medications. 35 Petitioner likewise consulted a certain Dr. Tenchavez36 for her follow-up EEG.37 He similarly prescribed medicine for petitioners deep brain injury. He also gave her pain killer for her headache and advised her to undergo physical therapy. Her symptoms, however, persisted all the more.38 In 1999, petitioner consulted another neurologist at the Makati Medical Center by the name of Dr. Martesio Perez (Dr. Perez) because of severe fleeting pains in her head, arms and legs; difficulty in concentration; and warm sensation of the legs, which symptoms also occurred after the 11 June 1995 incident. Upon examination, Dr. Perez observed that petitioner has been experiencing severe pains and she has a slight difficulty in concentration. He likewise noted that there was a slight spasm of petitioners neck muscle but, otherwise, there was no objective neurologic finding. The rest of petitioners neurologic examination was essentially normal.39 Dr. Perezs neurologic evaluation40 of petitioner reflected, among others: (1) petitioners past medical history, which includes, among others, mitral valve stenosis; (2) an interpretation of petitioners EEG results in October 1995 and in January 1999, i.e., the first EEG showed sharp waves seen bilaterally more on the left while the second one was normal; and (3) interpretation of petitioners second MRI result, i.e., petitioner has a permanent damage in the brain, which can happen either after a head injury or after a

stroke. Dr. Perez concluded that petitioner has post-traumatic or post concussion syndrome.41 Respondents, on the other hand, denied all the material allegations of petitioner and, in turn, countered the latters statement of facts, thus: According to respondents PHI and DTPCI, a sufficient notice had been posted on the glass door of the hotel leading to the swimming pool area to apprise the people, especially the hotel guests, that the swimming pool area is open only from 7:00 a.m. to 7:00 p.m.42 Though the hotels swimming pool area is open only between the aforestated time, the lights thereon are kept on until 10:00 p.m. for, (1) security reasons; (2) housekeeping personnel to do the cleaning of the swimming pool surroundings; and (3) people doing their exercise routine at the Slimmers World Gym adjacent to the swimming pool area, which was then open until 10:00 p.m., to have a good view of the hotels swimming pool. Even granting that the lights in the hotels swimming pool area were turned off, it would not render the area completely dark as the Slimmers World Gym near it was well-illuminated.43 Further, on 11 June 1995, at round 7:00 p.m., the hotels swimming pool attendant advised petitioner and Delia to take their showers as it was already closing time. Afterwards, at around 7:40 p.m., Pearlie Benedicto-Lipana (Ms. Pearlie), the hotel staff nurse, who was at the hotel clinic located at the mezzanine floor, received a call from the hotel telephone operator informing her that there was a guest requiring medical assistance at the hotels swimming pool area located one floor above the clinic. 44 Immediately, Ms. Pearlie got hold of her medical kit and hurriedly went to the hotels swimming pool area. There she saw Delia and petitioner, who told her that she was hit on the head by a folding wooden counter top. Although petitioner looked normal as there was no indication of any blood or bruise on her head, Ms. Pearlie still asked her if she needed any medical attention to which petitioner replied that she is a doctor, she was fine and she did not need any medical attention. Petitioner, instead, requested for a hirudoid cream to which Ms. Pearlie acceded.45 At about 8:00 p.m., after attending to petitioner, Ms. Pearlie went back to the hotel clinic to inform Dr. Dalumpines of the incident at the hotels swimming pool area. But before she could do that, Dr. Dalumpines had already chanced upon Delia and petitioner at the hotels coffee shop and the latter reported to Dr. Dalumpines that her head was hit by a folding wooden counter top while she was inside the hotels swimming pool area. When asked by Dr. Dalumpines how she was, petitioner responded she is a doctor, she was fine and she was already attended to by the hotel nurse, who went at the hotels swimming pool area right after the accident. Dr. Dalumpines then called Ms. Pearlie to verify the same, which the latter confirmed.46 Afterwards, Dr. Dalumpines went back to petitioner and checked the latters condition. Petitioner insisted that she was fine and that the hirudoid cream was enough. Having been assured that everything was fine, Dr. Dalumpines requested petitioner to execute

a handwritten certification47 regarding the incident that occurred that night. Dr. Dalumpines then suggested to petitioner to have an X-ray test. Petitioner replied that it was not necessary. Petitioner also refused further medical attention. 48 On 13 June 1995, petitioner called up Dr. Dalumpines. The call, however, had nothing to do with the 11 June 1995 incident. Instead, petitioner merely engaged in small talk with Dr. Dalumpines while having her daily massage. The two talked about petitioners personal matters, i.e., past medical history, differences with siblings and family over inheritance and difficulty in practice. Petitioner even disclosed to Dr. Dalumpines that she once fell from a horse; that she had a stroke; had hysterectomy and is incapable of having children for her uterus had already been removed; that she had blood disorder, particularly lack of platelets, that can cause bleeding; and she had an "on" and "off" headaches. Petitioner oftentimes called Dr. Dalumpines at the hotel clinic to discuss topics similar to those discussed during their 13 June 1995 conversation.49 Also, during one of their telephone conversations, petitioner requested for a certification regarding the 11 June 1995 incident inside the hotels swimming pool area. Dr. Dalumpines accordingly issued Certification dated 7 September 1995, which states that:50 CERTIFICATION This is to certify that as per Clinic records, duty nurse Pearlie was called to attend to an accident at the poolside at 7:45PM on 11 June 1995. Same records show that there, she saw petitioner who claimed the folding countertop fell on her head when she lifted it to enter the lifeguards counter to use the phone. She asked for Hirudoid. The same evening petitioner met Dr. Dalumpines at the Coffee Shop. After narrating the poolside incident and declining Dr. Dalumpines offer of assistance, she reiterated that the Hirudoid cream was enough and that petitioner being a doctor herself, knew her condition and she was all right. This certification is given upon the request of petitioner for whatever purpose it may serve, 7 September 1995 at Makati City.51 (Emphasis supplied). Petitioner personally picked up the afore-quoted Certification at the hotel clinic without any objection as to its contents.52 From 11 June 1995 until 7 September 1995, the hotel clinic never received any complaint from petitioner regarding the latters condition. The hotel itself neither received any written complaint from petitioner.53 After trial, the court a quo in its Decision dated 21 February 2006 dismissed petitioners Complaint for lack of merit.

The trial court found petitioners testimony self-serving, thus, devoid of credibility. Petitioner failed to present any evidence to substantiate her allegation that the lights in the hotels swimming pool area were shut off at the time of the incident. She did not even present her friend, Delia, to corroborate her testimony. More so, petitioners testimony was contradicted by one of the witnesses presented by the respondents who positively declared that it has been a normal practice of the hotel management not to put off the lights until 10:00 p.m. to allow the housekeepers to do the cleaning of the swimming pool surroundings, including the toilets and counters. Also, the lights were kept on for security reasons and for the people in the nearby gym to have a good view of the swimming pool while doing their exercise routine. Besides, there was a remote possibility that the hotels swimming pool area was in complete darkness as the aforesaid gym was then open until 10:00 p.m., and the lights radiate to the hotels swimming pool area. As such, petitioner would not have met the accident had she only acted with care and caution.54 The trial court further struck down petitioners contention that the hotel management did not extend medical assistance to her in the aftermath of the accident. Records showed that the hotel management immediately responded after being notified of the accident. The hotel nurse and the two chambermaids placed an ice pack on petitioners head. They were willing to extend further emergency assistance but petitioner refused and merely asked for a hirudoid cream. Petitioner even told them she is a doctor and she was fine. Even the medical services offered by the hotel physician were turned down by petitioner. Emphatically, petitioner cannot fault the hotel for the injury she sustained as she herself did not heed the warning that the swimming pool area is open only from 7:00 a.m. to 7:00 p.m. As such, since petitioners own negligence was the immediate and proximate cause of her injury, she cannot recover damages.55 The trial court similarly observed that the records revealed no indication that the head injury complained of by petitioner was the result of the alleged 11 June 1995 accident. Firstly, petitioner had a past medical history which might have been the cause of her recurring brain injury. Secondly, the findings of Dr. Perez did not prove a causal relation between the 11 June 1995 accident and the brain damage suffered by petitioner. Even Dr. Perez himself testified that the symptoms being experienced by petitioner might have been due to factors other than the head trauma she allegedly suffered. It bears stressing that petitioner had been suffering from different kinds of brain problems since she was 18 years old, which may have been the cause of the recurring symptoms of head injury she is experiencing at present. Absent, therefore, of any proof establishing the causal relation between the injury she allegedly suffered on 11 June 1995 and the head pains she now suffers, her claim must fail. Thirdly, Dr. Teresita Sanchezs (Dr. Sanchez) testimony cannot be relied upon since she testified on the findings and conclusions of persons who were never presented in court. Ergo, her testimony thereon was hearsay. Fourthly, the medical reports/evaluations/certifications issued by myriads of doctors whom petitioner sought for examination or treatment were neither identified nor testified to by those who issued them. Being deemed as hearsay, they cannot be given probative value. Even assuming that petitioner suffered head injury as a consequence of the 11 June 1995 accident, she cannot blame anyone but herself for

staying at the hotels swimming pool area beyond its closing hours and for lifting the folding wooden counter top that eventually hit her head.56 For petitioners failure to prove that her serious and permanent injury was the result of the 11 June 1995 accident, thus, her claim for actual or compensatory damages, loss of income, moral damages, exemplary damages and attorneys fees, must all fail.57 With regard to respondent First Lepantos liability, the trial court ruled that under the contract of insurance, suffice it to state that absent any cause for any liability against respondents PHI and DTPCI, respondent First Lepanto cannot be made liable thereon. Dissatisfied, petitioner elevated the matter to the Court of Appeals with the following assignment of errors: (1) the trial court erred in finding that the testimony of petitioner is self-serving and thus void of credibility; (2) the trial court erred in applying the doctrine of proximate cause in cases of breach of contract and even assuming arguendo that the doctrine is applicable, petitioner was able to prove by sufficient evidence the causal connection between her injuries and respondents PHI and DTPCIs negligent act; and (3) the trial court erred in holding that petitioner is not entitled to damages. 58 On 9 August 2007, the Court of Appeals rendered a Decision affirming the findings and conclusions of the trial court. The Court of Appeals ratiocinated in this wise: At the outset, it is necessary for our purpose to determine whether to decide this case on the theory that herein respondents PHI and DTPCI are liable for breach of contract or on the theory of quasi-delict. xxxx It cannot be gainsaid that herein petitioners use of the hotels poo l was only upon the invitation of Delia, the hotels registered guest. As such, she cannot claim contractual relationship between her and the hotel. Since the circumstances of the present case do not evince a contractual relation between petitioner and respondents, the rules on quasi-delict , thus, govern. The pertinent provision of Art. 2176 of the Civil Code which states: "Whoever by act or omission causes damage to another, there being fault or negligence, is obliged to pay for the damage done. Such fault or negligence, if there is no pre-existing contractual relation between the parties, is called quasi-delict." A perusal of Article 2176 shows that obligations arising from quasi-delict or tort, also known as extra-contractual obligations, arise only between parties not otherwise bound by contract, whether express or implied. Thus, to sustain a claim liability under quasidelict, the following requisites must concur: (a) damages suffered by the plaintiff; (b) fault or negligence of the defendant, or some other person for whose acts he must

respond; and (c) the connection of cause and effect between the fault or negligence of the defendant and the damages incurred by the plaintiff. Viewed from the foregoing, the question now is whether respondents PHI and DTPCI and its employees were negligent? We do not think so. Several factors militate against petitioners contention. One. Petitioner recognized the fact that the pool areas closing time is 7:00 p.m.. She, herself, admitted during her testimony that she was well aware of the sign when she and Delia entered the pool area. Hence, upon knowing, at the outset, of the pools closing time, she took the risk of overstaying when she decided to take shower and leave the area beyond the closing hour. In fact, it was only upon the advise of the pool attendants that she thereafter took her shower. Two. She admitted, through her certification that she lifted the wooden bar countertop, which then fell onto her head. The admission in her certificate proves the circumstances surrounding the occurrence that transpired on the night of 11 June 1995. This is contrary to her assertion in the complaint and testimony that, while she was passing through the counter door, she was suddenly knocked out by a hard and heavy object. In view of the fact that she admitted having lifted the counter top, it was her own doing, therefore, that made the counter top fell on to her head. Three. We cannot likewise subscribe to petitioners assertion that the pool area was totally dark in that she herself admitted that she saw a telephone at the counter after searching for one. It must be noted that petitioner and Delia had walked around the pool area with ease since they were able to proceed to the glass entrance door from shower room, and back to the counter area where the telephone was located without encountering any untoward incident. Otherwise, she could have easily stumbled over, or slid, or bumped into something while searching for the telephone. This negates her assertion that the pool area was completely dark, thereby, totally impairing her vision. xxxx The aforementioned circumstances lead us to no other conclusion than that the proximate and immediate cause of the injury of petitioner was due to her own negligence. Moreover, petitioner failed to sufficiently substantiate that the medical symptoms she is currently experiencing are the direct result of the head injury she sustained on 11 June 1995 as was aptly discussed in the lower courts findings. xxxx

It bears stressing that in civil cases, the law requires that the party who alleges a fact and substantially asserts the affirmative of the issue has the burden of proving it. Hence, for petitioner to be entitled to damages, she must show that she had suffered an actionable injury. Regrettably, petitioner failed in this regard.59 (Emphasis supplied). Petitioners Motion for Reconsideration was denied for lack of merit in a Resolution dated 5 November 2007. Hence, this Petition raising the following issues: (1) Whether or not the findings of fact of the trial court and of the Court of Appeals are conclusive in this case. (2) Whether or not herein respondents PHI and DTPCI are responsible by implied contract to exercise due care for the safety and welfare of the petitioner. (3) Whether or not the cause of action of the petitioner can be based on both breach of contract and tort. (4) Whether or not it is respondents PHI and DTPCI and its employees who are liable to the petitioner for negligence, applying the well-established doctrines of res ipsa loquitur and respondeat superior. (5) Whether the petitioners debilitating and permanent injuries were a result of the accident she suffered at the hotel on 11 June 1995. (6) Whether or not the petitioner is entitled to the payment of damages, attorneys fees, interest, and the costs of suit. (7) Whether or not the respondent insurance company is liable, even directly, to the petitioner. (8) Whether or not petitioners motion for reconsideration of the decision of the Court of Appeals is pro forma.60 Petitioner argues that the rule that "findings of fact of the lower courts are conclusive and must be respected on appeal" finds no application herein because this case falls under the jurisprudentially established exceptions. Moreover, since the rationale behind the afore-mentioned rule is that "the trial judge is in a vantage point to appreciate the conduct and behavior of the witnesses and has the unexcelled opportunity to evaluate their testimony," one logical exception to the rule that can be deduced therefrom is when the judge who decided the case is not the same judge who heard and tried the case. Petitioner further faults the Court of Appeals in ruling that no contractual relationship existed between her and respondents PHI and DTPCI since her use of the hotels

swimming pool facility was only upon the invitation of the hotels registered guest. On the contrary, petitioner maintains that an implied contract existed between them in view of the fact that the hotel guest status extends to all those who avail of its services its patrons and invitees. It follows then that all those who patronize the hotel and its facilities, including those who are invited to partake of those facilities, like petitioner, are generally regarded as guests of the hotel. As such, respondents PHI and DTPCI are responsible by implied contract for the safety and welfare of petitioner while the latter was inside their premises by exercising due care, which they failed to do. Petitioner even asserts that the existence of a contract between the parties does not bar any liability for tort since the act that breaks a contract may also be a tort. Hence, the concept of change of theory of cause of action pointed to by respondents is irrelevant. Petitioner similarly avows that the doctrines of res ipsa loquitur and respondeat superior are applicable in this case. She argues that a person who goes in a hotel without a "bukol" or hematoma and comes out of it with a "bukol" or hematoma is a clear case of res ipsa loquitur. It was an accident caused by the fact that the hotel staff was not present to lift the heavy counter top for petitioner as is normally expected of them because they negligently locked the main entrance door of the hotels swimming pool area. Following the doctrine of res ipsa loquitur, respondents PHI and DTPCIs negligence is presumed and it is incumbent upon them to prove otherwise but they failed to do so. Further, respondents PHI and DTPCI failed to observe all the diligence of a good father of a family in the selection and supervision of their employees, hence, following the doctrine of respondeat superior, they were liable for the negligent acts of their staff in not verifying if there were still people inside the swimming pool area before turning off the lights and locking the door. Had respondents PHI and DTPCIs employees done so, petitioner would not have been injured. Since respondents PHI and DTPCIs negligence need not be proved, the lower courts erred in shifting the burden to petitioner and, thereafter, holding the hotel and its employees not negligent for petitioners failure to prove their negligence. Moreover, petitioner alleges that there was no contributory negligence on her part for she did not do anything that could have contributed to her injury. And, even if there was, the same does not bar recovery. Petitioner equally declares that the evidence on record, including the objective medical findings, had firmly established that her permanent debilitating injuries were the direct result of the 11 June 1995 accident inside the hotels swimming pool area. This fact has not been totally disputed by the respondents. Further, the medical experts who had been consulted by petitioner were in unison in their diagnoses of her condition. Petitioner was also able to prove that the falling of the folding wooden counter top on her head while she was at the hotels swimming pool area was the cause of her head, eye and neck injuries. Petitioner reiterates her claim for an award of damages, to wit: actual, including loss of income; moral, exemplary; as well as attorneys fees, interest and costs of suit. She states that respondents PHI and DTPCI are liable for quasi-delict under Articles 19, 2176 and 2180 of the New Civil Code. At the same time, they are liable under an

implied contract for they have a public duty to give due courtesy, to exercise reasonable care and to provide safety to hotel guests, patrons and invitees. Respondent First Lepanto, on the other hand, is directly liable under the express contract of insurance. Lastly, petitioner contends that her Motion for Reconsideration before the Court of Appeals was not pro forma for it specifically pointed out the alleged errors in the Court of Appeals Decision. The instant Petition is devoid of merit. Primarily, only errors of law and not of facts are reviewable by this Court in a Petition for Review on Certiorari under Rule 45 of the Rules of Court.61 This Court is not a trier of facts and it is beyond its function to re-examine and weigh anew the respective evidence of the parties.62 Besides, this Court adheres to the long standing doctrine that the factual findings of the trial court, especially when affirmed by the Court of Appeals, are conclusive on the parties and this Court.63 Nonetheless, this Court has, at times, allowed exceptions thereto, to wit: (a) When the findings are grounded entirely on speculation, surmises, or conjectures; (b) When the inference made is manifestly mistaken, absurd, or impossible; (c) When there is grave abuse of discretion; (d) When the judgment is based on a misapprehension of facts; (e) When the findings of facts are conflicting; (f) When in making its findings the Court of Appeals went beyond the issues of the case, or its findings are contrary to the admissions of both the appellant and the appellee; (g) When the Court of Appeals findings are contrary to those by the trial court; (h) When the findings are conclusions without citation of specific evidence on which they are based; (i) When the facts set forth in the petition as well as in the petitioners main and reply briefs are not disputed by the respondent; (j) When the findings of fact are premised on the supposed absence of evidence and contradicted by the evidence on record; or

(k) When the Court of Appeals manifestly overlooked certain relevant facts not disputed by the parties, which, if properly considered, would justify a different conclusion.64 Upon meticulous perusal of the records, however, this Court finds that none of these exceptions is obtaining in this case. No such justifiable or compelling reasons exist for this Court to depart from the general rule. This Court will not disturb the factual findings of the trial court as affirmed by the Court of Appeals and adequately supported by the evidence on record. Also, this Court will not review the factual findings of the trial court simply because the judge who heard and tried the case was not the same judge who penned the decision. This fact alone does not diminish the veracity and correctness of the factual findings of the trial court.65 Indeed, "the efficacy of a decision is not necessarily impaired by the fact that its writer only took over from a colleague who had earlier presided at the trial, unless there is showing of grave abuse of discretion in the factual findings reached by him."66 In this case, there was none. It bears stressing that in this jurisdiction there is a disputable presumption that the trial courts decision is rendered by the judge in the regular performance of his official duties. While the said presumption is only disputable, it is satisfactory unless contradicted or overcame by other evidence. Encompassed in this presumption of regularity is the presumption that the trial court judge, in resolving the case and drafting the decision, reviewed, evaluated, and weighed all the evidence on record. That the said trial court judge is not the same judge who heard the case and received the evidence is of little consequence when the records and transcripts of stenographic notes (TSNs) are complete and available for consideration by the former,67 just like in the present case. Irrefragably, the fact that the judge who penned the trial courts decision was not the same judge who heard the case and received the evidence therein does not render the findings in the said decision erroneous and unreliable. While the conduct and demeanor of witnesses may sway a trial court judge in deciding a case, it is not, and should not be, his only consideration. Even more vital for the trial court judges decision are the contents and substance of the witnesses testimonies, as borne out by the TSNs, as well as the object and documentary evidence submitted and made part of the records of the case.68 This Court examined the records, including the TSNs, and found no reason to disturb the factual findings of both lower courts. This Court, thus, upholds their conclusiveness. In resolving the second and third issues, a determination of the cause of action on which petitioners Complaint for Damages was anchored upon is called for. Initially, petitioner was suing respondents PHI and DTPCI mainly on account of their negligence but not on any breach of contract. Surprisingly, when the case was elevated on appeal to the Court of Appeals, petitioner had a change of heart and later claimed

that an implied contract existed between her and respondents PHI and DTPCI and that the latter were liable for breach of their obligation to keep her safe and out of harm. This allegation was never an issue before the trial court. It was not the cause of action relied upon by the petitioner not until the case was before the Court of Appeals. Presently, petitioner claims that her cause of action can be based both on quasi-delict and breach of contract. A perusal of petitioners Complaint evidently shows that her cause of action was based solely on quasi-delict. Telling are the following allegations in petitioners Complaint: 6. THAT, in the evening of 11 June 1995, between the hours from 7:00 to 8:00 oclock, after herein petitioner and her friend from New York, Delia, the latter being then a Hotel guest, were taking their shower after having a dip in the hotels swimming pool, without any notice or warning, the Hotels staff put off all the lights within the pool area including the lights on the hallway and also locked the main entrance door of the pool area, x x x; 7. THAT, Hotel guest Delia started to panic while petitioner pacified her by telling her not to worry as they would both find their way out. Petitioner knowing that within the area there is a house phone, started to look around while Delia was following her, eventually petitioner saw a phone behind the counter x x x, that while slowly moving on towards the phone on a stooping manner due to the darkness CAUSED BY UNTIMELY AND NEGLIGENTLY PUTTING OFF WITH THE LIGHTS BY THE HEREIN RESPONDENTS PHI AND DTPCIS EMPLOYEE while passing through the open counter door with its Folding Counter Top also opened, x x x, a hard and heavy object fell onto the head of the petitioner that knocked her down almost unconscious which hard and heavy object turned out to be the Folding Counter Top; 8. THAT, Delia immediately got hold of the house phone and notified the Hotel Telephone Operator about the incident, immediately the hotel staffs (sic) arrived but they were stranded behind the main door of the pool entrance and it too (sic) them more than twenty (20) minutes to locate the hotel maintenance employee who holds the key of the said main entrance door; 9. THAT, when the door was opened, two Hotel Chamber Maids assisted the petitioner to get out of the counter door. Petitioner being a Physician tried to control her feelings although groggy and requested for a HURIDOID, a medicine for HEMATOMA, as a huge lump developed on her head while the two Chamber Maids assisted petitioner by holding the bag of ice on her head and applying the medicine on the huge lump; 10. THAT, petitioner after having recovered slightly from her nightmare, though still feeling weak, asked to be assisted to the Hotel Coffee Shop to take a rest but requested for the hotels Physician. Despite her insistent requests, the Dus it Hotel refused to lift a finger to assists petitioner who was then in distress until a

lady approached and introduced herself as the Hotels house Doctor. Instead however of assisting petitioner by asking her what kind of assistance the Hotel could render, in a DISCOURTEOUS MANNER presented instead a paper and demanding petitioner to affix her signature telling her that the Hotel Management would only assists and answer for all expenses incurred if petitioner signs the paper presented, but she refused and petitioner instead wrote a marginal note on the said paper stating her reason therefore, said paper later on turned out to be a WAIVER OF RIGHT or QUIT CLAIM; xxxx 14. THAT, due to the unfortunate incident caused by respondents PHI and DTPCIs gross negligence despite medical assistance, petitioner started to feel losing her memory that greatly affected and disrupted the practice of her chosen profession x x x. xxxx 19. THAT, due to respondents PHI and DTPCIs gross negligence as being narrated which caused petitioner to suffer sleepless nights, depression, mental anguish, serious anxiety, wounded feelings, and embarrassment with her Diplomate friends in the profession and industry, her social standing in the community was greatly affected and hence, respondents PHI and DTPCI must be imposed the hereunder damages, prayed for x x x and Artile (sic) 2176 and 2199 of the New Civil Code of the Philippines x x x. xxxx 22. THAT, as to Moral, Exemplary and Actual Damages, as well as petitioners Loss of Income, the amounts are stated in its prayer hereunder.69 It is clear from petitioners allegations that her Complaint for Damages was predicated on the alleged negligence of respondents PHI and DTPCIs staff in the untimely putting off of all the lights within the hotels swimming pool area, as well as the locking of its main door, prompting her to look for a way out leading to the fall of the folding wooden counter top on her head causing her serious brain injury. The said negligence was allegedly compounded by respondents PHI and DTPCIs failure to render prompt and adequate medical assistance. These allegations in petitioners Complaint constitute a cause of action for quasi-delict, which under the New Civil Code is defined as an act, or omission which causes damage to another, there being fault or negligence. 70 It is evident from petitioners Complaint and from her open court testimony that the reliance was on the alleged tortious acts committed against her by respondents PHI and DTPCI, through their management and staff. It is now too late in the day to raise the said argument for the first time before this Court.71

Petitioners belated reliance on breach of contract as her cause of action cannot be sanctioned by this Court. Well-settled is the rule that a party is not allowed to change the theory of the case or the cause of action on appeal. Matters, theories or arguments not submitted before the trial court cannot be considered for the first time on appeal or certiorari.72 When a party adopts a certain theory in the court below, he will not be permitted to change his theory on appeal for to permit him to do so would not only be unfair to the other party but it would also be offensive to the basic rules of fair play, justice and due process.73 Hence, a party is bound by the theory he adopts and by the cause of action he stands on and cannot be permitted after having lost thereon to repudiate his theory and cause of action and adopt another and seek to re-litigate the matter anew either in the same forum or on appeal.74 In that regard, this Court finds it significant to take note of the following differences between quasi-delict (culpa aquilina) and breach of contract (culpa contractual). In quasi-delict, negligence is direct, substantive and independent, while in breach of contract, negligence is merely incidental to the performance of the contractual obligation; there is a pre-existing contract or obligation.75 In quasi-delict, the defense of "good father of a family" is a complete and proper defense insofar as parents, guardians and employers are concerned, while in breach of contract, such is not a complete and proper defense in the selection and supervision of employees.76 In quasi- delict , there is no presumption of negligence and it is incumbent upon the injured party to prove the negligence of the defendant, otherwise, the formers complaint will be dismissed, while in breach of contract, negligence is presumed so long as it can be proved that there was breach of the contract and the burden is on the defendant to prove that there was no negligence in the carrying out of the terms of the contract; the rule of respondeat superior is followed.77 Viewed from the foregoing, petitioners change of theory or cause of action from quasidelict to breach of contract only on appeal would necessarily cause injustice to respondents PHI and DTPCI. First, the latter will have no more opportunity to present evidence to contradict petitioners new argument. Second, the burden of proof will be shifted from petitioner to respondents PHI and DTPCI. Petitioners change of theory from quasi-delict to breach ofcontract must be repudiated. As petitioners cause of action is based on quasi-delict, it is incumbent upon her to prove the presence of the following requisites before respondents PHI and DTPCI can be held liable, to wit: (a) damages suffered by the plaintiff; (b) fault or negligence of the defendant, or some other person for whose acts he must respond; and (c) the connection of cause and effect between the fault or negligence of the defendant and the damages incurred by the plaintiff.78 Further, since petitioners case is for quasi-delict , the negligence or fault should be clearly established as it is the basis of her action.79 The burden of proof is upon petitioner. Section 1, Rule 131 of the Rules of Court provides that "burden of proof is the duty of a party to present evidence on the facts in issue necessary to establish his claim or defense by the amount of evidence required by law." It is then up for the plaintiff to establish his cause of action or the defendant to establish his defense. Therefore, if the plaintiff alleged in his complaint that

he was damaged because of the negligent acts of the defendant, he has the burden of proving such negligence. It is even presumed that a person takes ordinary care of his concerns. The quantum of proof required is preponderance of evidence. 80 In this case, as found by the trial court and affirmed by the Court of Appeals, petitioner utterly failed to prove the alleged negligence of respondents PHI and DTPCI. Other than petitioners self-serving testimony that all the lights in the hotels swimming pool area were shut off and the door was locked, which allegedly prompted her to find a way out and in doing so a folding wooden counter top fell on her head causing her injury, no other evidence was presented to substantiate the same. Even her own companion during the night of the accident inside the hotels swimming pool area was never presented to corroborate her allegations. Moreover, petitioners aforesaid allegations were successfully rebutted by respondents PHI and DTPCI. Here, we quote with conformity the observation of the trial court, thus: x x x Besides not being backed up by other supporting evidence, said statement is being contradicted by the testimony of Engineer Dante L. Costas,81 who positively declared that it has been a normal practice of the Hotel management not to put off the lights until 10:00P.M. in order to allow the housekeepers to do the cleaning of the pools surrounding, the toilets and the counters. It was also confirmed that the lights were kept on for security reasons and so that the people exercising in the nearby gym may be able to have a good view of the swimming pool. This Court also takes note that the nearby gymnasium was normally open until 10:00 P.M. so that there was a remote possibility the pool area was in complete darkness as was alleged by herein petitioner, considering that the illumination which reflected from the gym. Ergo, considering that the area were sufficient (sic) illuminated when the alleged incident occurred, there could have been no reason for the petitioner to have met said accident, much less to have been injured as a consequence thereof, if she only acted with care and caution, which every ordinary person is expected to do.82 More telling is the ratiocination of the Court of Appeals, to wit: Viewed from the foregoing, the question now is whether respondents PHI and DTPCI and its employees were negligent? We do not think so. Several factors militate against petitioners contention. One. Petitioner recognized the fact that the pool areas closing time is 7:00 p.m.. She, herself, admitted during her testimony that she was well aware of the sign when she and Delia entered the pool area. Hence, upon knowing, at the outset, of the pools closing time, she took the risk of overstaying when she decided to take shower and leave the area beyond the closing hour. In fact, it was only upon the advise of the pool attendants that she thereafter took her shower. Two. She admitted, through her certification, that she lifted the wooden bar countertop, which then fell on to her head. The admission in her certificate proves the circumstances surrounding the occurrence that transpired on the night of 11 June 1995.

This is contrary to her assertion in the complaint and testimony that, while she was passing through the counter door, she was suddenly knocked out by a hard and heavy object. In view of the fact that she admitted having lifted the countertop, it was her own doing, therefore, that made the counter top fell on to her head. Three. We cannot likewise subscribe to petitioners assertion that the pool area was totally dark in that she herself admitted that she saw a telephone at the counter after searching for one. It must be noted that petitioner and Delia had walked around the pool area with ease since they were able to proceed to the glass entrance door from the shower room, and back to the counter area where the telephone was located without encountering any untoward incident. Otherwise, she could have easily stumbled over, or slid, or bumped into something while searching for the telephone. This negates her assertion that the pool area was completely dark, thereby, totally impairing her vision. xxxx The aforementioned circumstances lead us to no other conclusion than that the proximate and immediate cause of the injury of petitioner was due to her own negligence.83 (Emphasis supplied). Even petitioners assertion of negligence on the part of respondents PHI and DTPCI in not rendering medical assistance to her is preposterous. Her own Complaint affirmed that respondents PHI and DTPCI afforded medical assistance to her after she met the unfortunate accident inside the hotels swimming pool facility. Below is the portion of petitioners Complaint that would contradict her very own statement, th us: 14. THAT, due to the unfortunate incident caused by respondents PHI and DTPCIs gross negligence despite medical assistance, petitioner started to feel losing her memory that greatly affected and disrupted the practice of her chosen profession. x x x.84 (Emphasis supplied). Also, as observed by the trial court, respondents PHI and DTPCI, indeed, extended medical assistance to petitioner but it was petitioner who refused the same. The trial court stated, thus: Further, herein petitioners asseverations that the Hotel Management did not extend medical assistance to her in the aftermath of the alleged accident is not true. Again, this statement was not supported by any evidence other that the sole and self-serving testimony of petitioner. Thus, this Court cannot take petitioners statement as a gospel truth. It bears stressing that the Hotel Management immediately responded after it received notice of the incident. As a matter of fact, Ms. Pearlie, the Hotel nurse, with two chambermaids holding an ice bag placed on petitioners head came to the petitioner to extend emergency assistance when she was notified of the incident, but petitioner merely asked for Hirudoid, saying she was fine, and that she was a doctor and know how to take care of herself. Also, the Hotel, through its in-house physician, Dr. Dalumpines offered its medical services to petitioner when they met at the Hotels

coffee shop, but again petitioner declined the offer. Moreover, the Hotel as a show of concern for the petitioners welfare, shouldered the expenses for the MRI services performed on petitioner at the Makati Medical Center. Emphatically, petitioner herself cannot fault the Hotel for the injury she allegedly suffered because she herself did not heed the warning at the pool to the effect that it was only open from 7:00 to 7:00 P.M. Thus, when the petitioners own negligence was the immediate and proximate cause of his injury, shecannot recover damages x x x.85 With the foregoing, the following were clearly established, to wit: (1) petitioner stayed in the hotels swimming pool facility beyond its closing hours; (2) she lifted the folding wooden counter top that eventually hit her head; and (3) respondents PHI and DTPCI extended medical assistance to her. As such, no negligence can be attributed either to respondents PHI and DTPCI or to their staff and/or management. Since the question of negligence is one of fact, this Court is bound by the said factual findings made by the lower courts. It has been repeatedly held that the trial court's factual findings, when affirmed by the Court of Appeals, are conclusive and binding upon this Court, if they are not tainted with arbitrariness or oversight of some fact or circumstance of significance and influence. Petitioner has not presented sufficient ground to warrant a deviation from this rule.86 With regard to petitioners contention that the principles of res ipsa loquitur and respondeat superior are applicable in this case, this Court holds otherwise. Res ipsa loquitur is a Latin phrase which literally means "the thing or the transaction speaks for itself." It relates to the fact of an injury that sets out an inference to the cause thereof or establishes the plaintiffs prima facie case. The doctrine rests on inferen ce and not on presumption. The facts of the occurrence warrant the supposition of negligence and they furnish circumstantial evidence of negligence when direct evidence is lacking.87 Simply stated, this doctrine finds no application if there is direct proof of absence or presence of negligence. If there is sufficient proof showing the conditions and circumstances under which the injury occurred, then the creative reason for the said doctrine disappears.88 Further, the doctrine of res ipsa loquitur applies where, (1) the accident was of such character as to warrant an inference that it would not have happened except for the defendants negligence; (2) the accident must have been caused by an agency or instrumentality within the exclusive management or control of the person charged with the negligence complained of; and (3) the accident must not have been due to any voluntary action or contribution on the part of the person injured.89 In the case at bench, even granting that respondents PHI and DTPCIs staff negligently turned off the lights and locked the door, the folding wooden counter top would still not fall on petitioners head had she not lifted the same. Although the folding woo den counter top is within the exclusive management or control of respondents PHI and DTPCI, the falling of the same and hitting the head of petitioner was not due to the negligence of the former. As found by both lower courts, the folding wooden counter top

did not fall on petitioners head without any human intervention. Records showed that petitioner lifted the said folding wooden counter top that eventually fell and hit her head. The same was evidenced by the, (1) 11 June 1995 handwritten certification of petitioner herself; (2) her Letter dated 30 August 1995 addressed to Mr. Yoshikazu Masuda (Mr. Masuda), General Manager of Dusit Hotel; and, (3) Certification dated 7 September 1995 issued to her by Dr. Dalumpines upon her request, which contents she never questioned. Here, we, respectively, quote the 11 June 1995 handwritten certification of petitioner; her letter to Mr. Masuda dated 30 August 1995; and Dr. Dalumpines Certification dated 7 September 1995, to wit: Petitioners 11 June 1995 Handwritten Certification: I was requested by Dr. Dalumpines to write that I was assured of assistance should it be necessary with regard an accident at the pool. x x x The phone was in an enclosed area on a chair I lifted the wooden bar counter top which then fell on my head producing a large hematoma x x x.90 Petitioners Letter addressed to Mr. Masuda dated 30 August 1995: Dear Mr. Masuda, xxxx x x x We searched and saw a phone on a chair behind a towel counter. However, in order to get behind the counter I had to lift a hinged massive wooden section of the counter which subsequently fell and knocked me on my head x x x. 91 Dr. Dalumpines Certification dated 7 September 1995: CERTIFICATION This is to certify that as per Clinic records, duty nurse Pearlie was called to attend to an accident at the poolside at 7:45PM on 11 June 1995. Same records show that there, she saw petitioner who claimed the folding countertop fell on her head when she lifted it to enter the lifeguards counter to use the phone. She asked for Hirudoid. The same evening petitioner met Dr. Dalumpnes at the Coffee Shop. After narrating the poolside incident and declining Dr. Dalumpines offer of assistance, she reiterated that the Hirudoid cream was enough and that petitioner]being a doctor herself, knew her condition and she was all right.

This certification is given upon the request of petitioner for whatever purpose it may serve, 7 September 1995 at Makati City.92 (Emphasis supplied). This Court is not unaware that in petitioners Complaint and in her open court testimony, her assertion was, "while she was passing through the counter door, she was suddenly knocked out by a hard and heavy object, which turned out to be the folding wooden counter top." However, in her open court testimony, particularly during crossexamination, petitioner confirmed that she made such statement that "she lifted the hinge massive wooden section of the counter near the swimming pool." 93 In view thereof, this Court cannot acquiesce petitioners theory that her case is one of res ipsa loquitur as it was sufficiently established how petitioner obtained that "bukol" or "hematoma." The doctrine of respondeat superior finds no application in the absence of any showing that the employees of respondents PHI and DTPCI were negligent. Since in this case, the trial court and the appellate court found no negligence on the part of the employees of respondents PHI and DTPCI, thus, the latter cannot also be held liable for negligence and be made to pay the millions of pesos damages prayed for by petitioner. The issue on whether petitioners debilitating and permanent injuries were the result of the accident she suffered at the hotels swimming pool area on 11 June 1995 is another question of fact, which is beyond the function of this Court to resolve. More so, this issue has already been properly passed upon by the trial court and the Court of Appeals. To repeat, this Court is bound by the factual findings of the lower courts and there is no cogent reason to depart from the said rule. The following observations of the trial court are controlling on this matter: Firstly, petitioner had a past medical history which might have been the cause of her recurring brain injury. Secondly, the findings of Dr. Perez did not prove a causal relation between the 11 June 1995 accident and the brain damage suffered by petitioner. Dr. Perez himself testified that the symptoms being experienced by petitioner might have been due to factors other than the head trauma she allegedly suffered. Emphasis must be given to the fact that petitioner had been suffering from different kinds of brain problems since she was 18 years old, which may have been the cause of the recurring symptoms of head injury she is experiencing at present. Thirdly, Dr. Sanchezs testimony cannot be relied upon since she testified on the findings and conclusions of persons who were never presented in court. Ergo, her testimony thereon was hearsay. A witness can testify only with regard to facts of which they have personal knowledge. Testimonial or documentary evidence is hearsay if it is based, not on the personal knowledge of the witness, but on the knowledge of some other person not on the witness stand. Consequently, hearsay evidence -- whether objected to or not -- has no probative value.94

Fourthly, the medical reports/evaluations/certifications issued by myriads of doctors whom petitioner sought for examination or treatment were neither identified nor testified to by those who issued them. Being deemed as hearsay, they cannot be given probative value.1wphi1 The aforesaid medical reports/evaluations/certifications of different doctors in favor of petitioner cannot be given probative value and their contents cannot be deemed to constitute proof of the facts stated therein. It must be stressed that a document or writing which is admitted not as independent evidence but merely as part of the testimony of a witness does not constitute proof of the facts related therein. 95 In the same vein, the medical certificate which was identified and interpreted in court by another doctor was not accorded probative value because the doctor who prepared it was not presented for its identification. Similarly, in this case, since the doctors who examined petitioner were not presented to testify on their findings, the medical certificates issued on their behalf and identified by another doctor cannot be admitted as evidence. Since a medical certificate involves an opinion of one who must first be established as an expert witness, it cannot be given weight or credit unless the doctor who issued it is presented in court to show his qualifications.96 Thus, an unverified and unidentified private document cannot be accorded probative value. It is precluded because the party against whom it is presented is deprived of the right and opportunity to cross-examine the person to whom the statements or writings are attributed. Its executor or author should be presented as a witness to provide the other party to the litigation the opportunity to question its contents. Being mere hearsay evidence, failure to present the author of the letter renders its contents suspect and of no probative value.97 All told, in the absence of negligence on the part of respondents PHI and DTPCI, as well as their management and staff, they cannot be made Iiable to pay for the millions of damages prayed for by the petitioner. Since respondents PHI and DTPCI arc not liable, it necessarily follows that respondent First Lepanto cannot also be made liable under the contract or Insurance. WHEREFORE, premises considered, the Decision and Resolution or the Court of Appeals in CA-G.R. CV No. 87065 dated 9 August 2007 and 5 November 2007, respectively, are hereby AFFIRMED. Costs against petitioner. SO ORDERED. JOSE PORTUGAL PEREZ Associate Justice WE CONCUR: ARTURO D. BRION Associate Justice Acting Chairperson

PRESBITERO J. VELASCO, JR.* Associate Justice

MARTIN S. VILLARAMA, JR.** Associate Justice

ESTELA M. PERLAS-BERNABE Associate Justice ATTESTATION I attest that the conclusions in the above Decision were reached in consultation before the case was assigned to the writer of the opinion of the Court's Division. ARTURO D. BRION Associate Justice Acting Chairperson, Second Division CERTIFICATION Pursuant to Section 13, Article VIII of the Constitution and the Division Chairperson's Attestation, f certify that the conclusions in the above Decision had been reached in consultation before the case was assigned to the writer of the opinion of the Court's Division. ANTONIO T. CARPIO Acting Chief Justice

G.R. No. 41702

September 4, 1935

FORTUNATA LUCERO VIUDA DE SINDAYEN, plaintiff-appellant, vs. THE INSULAR LIFE ASSURANCE CO., LTD., defendant-appellee. Jos. N. Wolfson for appellant. Araneta, Zaragoza and Araneta for appellee. BUTTE, J.: This if, an appeal from a judgment of the Court of First Instance of Manila in an action brought by the plaintiff-appellant as beneficiary to recover P1,000 upon a life insurance policy issued by the defendant on the life of her deceased husband, Arturo Sindayen. The essential facts upon which this case turns are not in dispute and may be stated as follows:

Arturo Sindayen, up to the time of his death on January 19, 1933, was employed as a linotype operator in the Bureau of Printing at Manila and had been such for eleven years prior thereto. He and his wife went to Camiling, Tarlac, to spend the Christmas vacation with his aunt, Felicidad Estrada. While there he made a written application on December 26, 1932, to the defendant Insular Life Assurance Co., Ltd., through its agent, Cristobal Mendoza, for a policy of insurance on his life in the sum of P1,000 and he paid to the agent P15 cash as part of the first premium. It was agreed with the agent that the policy, when and if issued, should be delivered to his aunt. Felicidad Estrada, with whom Sindayen left the sum of P26.06 to complete the payment of the first annual premium of P40.06. On January 1, 1933, Sindayen, who was then twenty-nine years of age, was examined by the company's doctor who made a favorable report, to the company. On January 2, 1933, Sindayen returned to Manila and resumed his work a linotype operator in the Bureau of Printing. On January 11, 1933, The company accepted the risk and issued policy No. 47710 dated back to December 1, 1932, and mailed the same to its agent, Cristobal Mendoza, in Camiling, Tarlac, for delivery to the insured. On January 11, 1933, Sindayen was at work in the Bureau of Printing. On January 12, he complained of a severe headache and remained at home. On January 15, he called a physician who found that he was suffering from acute nephritis and uremia. His illness did not yield to treatment and on January 19, 1933, he died. The policy which the company issued and mailed in Manila on January 11, 1933, was received by its agent in Camiling, Tarlac, on January 16, 1933. On January 18, 1933, the agent, in accordance with his agreement with the insured, delivered the policy to Felicidad Estrada upon her payment of the balance of the first year's annual premium. The agent asked Felicidad Estrada if her nephew was in good health and she replied that she believed so because she had no information that he was sick and he thereupon delivered to her the policy. On January 20, 1933, the agent learned of the death of Arturo Sindayen and called on Felicidad Estrada and asked her to return the policy. He testified: "pedia a ella que me devolviera a poliza para traerla a Manila para esperar la de decision de la compaia" (t. s. n. p. 19). But he did not return or offer to return the premium paid. Felicidad Estrada on his aforesaid statement gave him the policy. On February 4, 1933, under circumstances which it is not necessary to relate here, the company obtained from the beneficiary, the widow of Arturo Sindayen, her signature to a legal document entitled "ACCORD, SATISFACTION AND RELEASE" whereby in consideration of the sum of P40.06 paid to her by a check of the company, she "assigns, releases and forever discharges said Isular Life Assurance Co., Ltd., its successors and assigns, of all claims, obligation in or indebtedness which she, as such beneficiary ever had or now has, hereafter ca, shall, or may have, for, upon, or by reason of said policy of life insurance numbered 47710 upon the life of said Arturo Sindayen, the latter now deceased, or arising therefrom or connected therewith in any manner", which appears in the record as Exhibit A, attached to the deposition of the notary who executed th fraudulent acknowledgment to Exhibit A. The said check for

P40.06 was never cashed but returned to the company and appears in the record of this case as Exhibit D. Thereupon this action was brought to enforce payment of the policy. By the terms of the policy, an annual premium of P40.06 is due on the first day of December of each year, the first premium already paid by the insured covering the period from December 1, 1932. It is to December 1, 1933. It is to be noted that the policy was not issued and the company assumed no actual risk prior to January 11, 1933. The policy contains the following paragraph: THE CONTRACT. This Policy and the application herefor constitute the entire contract between the parties hereto. All statements made by the Insured shall, in the absence of fraud, be deemed representations and not warranties, and no such statement shall void the Policy unless it is contained in the written application, a copy of which is attached to this Policy. Only the President, or the Manager, acting jointly with the Secretary or Assistant Secretary (and then only in writing signed by them) have power in behalf of the Company to issue permits, or to modify this or any contract, or to extend the time for making any premium payment, and the Company shall t bound by any promise or representation heretofore hereafter given by any person other than the above-named officials, and by them only in writing and signed conjointly as stated.". The application which the insured signed in Camiling, Tarlac, on December 26, 1932, contained among others the following provisions: 2. That if this application is accepted and a policy issued in my favor, I bind myself to accept the same and to pay at least the first year's premium thereon in the City of Manila. 3. That the said policy shall not take effect until the first premium has been paid and the policy has been delivered to and accepted by me, while I am in good health. 4. That the agent taking this application has no authority to make, modify or discharge contracts, or to waive any of the Company's right or requirements.". The insurance company does not set up any defense of fraud, misconduct or omission of duty of the insured or his agent, Felicidad Estrada or of the beneficiary. In its answer it pleads the "ACCORD, SATISFACTION AND RELEASE" (Exhibit A) signed by the widow of Arturo Sindayen, the plaintiff-appellant. With respect to Exhibit A, it suffices to say that this release is so inequitable, not to say fraudulent, that we are pleased to note that counsel for the defendant company, on page 51 of their brief, state: "si resultara que la poliza aqui en cuestion es valida la apelada seria la primera en no dar validez alguno al documento Exhibit A aunque la apelante hubiera afirmado que lo otorgo con conocimiento de causa."

It is suggested in appellee's brief that fhere was no delivery of the policy in this case because the policy was not delivered to and accepted by the insured in person. Delivery to the insured in person is not necessary. Delivery may be made by mail or to a duly constituted agent. Appellee cites no authorities to support its proposition and none need be cited to refute it. We come now to the main defense of the company in this case, namely, that the said policy never took effect because of paragraph 3 of the application above quoted, for at the time of its delivery by the agent as aforesaid the insured was not in good health. We have not heretofore been called upon to interpret and apply this clause in life insurance application, but identical or substantially identical clauses have been construed and applied in a number of cases in the United States and the decisions thereon are far from uniform or harmonious. We do not find it practicable to attempt to determine where the weight of the authority lies and propose to resolve this case on its own facts. There is one line of cases which holds that the stipulation contained in paragraph 3 is in the nature of a condition precedent, that is to say, that there can be no valid delivery to the insured unless he is in good health at the time; that this condition precedent goes to the very essence of the contract and cannot be waived by the agent making delivery of the policy, (Rathbun is. New York Life Insurance Co., 30 Idaho, 34; 165 Pac., 997; American Bankers Insurance Co. vs. Thomas, 53 Okla., 11; 154 Pac., 44; Gordon vs. Prudential Insurance Co., 231 Pa., 404; Reliance Life Insurance Co. vs. Hightower, 148 Ga., 843; 98 S.E., 469.) On the other hand, a number of American decisions hold that an agent to whom a life insurance policy similar to the one here involved was sent with instructions to deliver it to the insured has authority to bind the company by making such delivery, although the insured was not in good health at the time of delivery, on the theory that the delivery of the policy being the final act to the consummation of the contract, the condition as to the insurer's good health was waived by the company. (Kansas City Life Insurance Co. vs. Ridout, 147 Ark., 563; 228 S.W., 55; Metropolitan Life Insurance Co. vs. Willis, 37 Ind. App., 48; 76 N.E., 560; Grier vs. Mutual Life Insurance Co. of New York, 132 N.C., 543; 44 S.E., 38; Bell vs. Missouri State Life Insurance Co., 166 Mo. App., 390; 149 S.W., 33.) A number of these cases go to the of holding that the delivery of the policy by the agent to the insured consummates the contract even though the agent knew that the insured was not in good health at the time, the theory being that his knowledge is the company's knowledge and his delivery of the policy is the company's delivery; that when the delivery is made notwithstanding this knowledge of the defect, the company is deemed to have waived the defect. Although that appears to be the prevailing view in the American decisions (14 R.C.L., 900) and leads to the same conclusion, namely, that the act of delivery of the policy in the absence of fraud or other ground for recission consummates the insurance, we are inclined to the view that it is more consonant with the well known practice of life insurance companies and the evidence in the present case to rest our decision on the proposition that Mendoza was authorized by the

company to make the delivery of the policy when he received the payment of the first premium and he was satisfied that the insured was in good health. As was well said in the case of MeLaurin vs. Mutual Life Insurance Co. (115 S.C., 59; 104 S.E., 327): So much comes from the necessity of the case; the president, the vice-president, and the secretary cannot solicit, or collect, or deliver; they must commit that to others, and along with it the discretions we have adverted to. . . . The power in the local agent to withhold the policy involves the power to deliver it; there is no escape from that conclusion. But the appellant says, even though the local agent should have concluded that the applicant was in good health, yet, if the fact be the contrary, then the policy never operated. The parties intended to make a contract, and that involved the doing of everything necessary to carry it into operation, to wit, the acceptance of the applicant as a person in good health. They never intended to leave open that one essential element of the contract, when the parties dealth fairly one with the other. It is plain, therefore, that upon the facts it is not necessarily a case of waiver or of estoppel, but a case where the local agents, in the exercise of the powers lodged in them, accepted the premium and delivered the policy. That act binds their principal, the defendant. Mendoza was duly licensed by the Insurance Commissioner to act as the agent of the defendant insurance company. The well known custom of the insurance business and the evidence in this case prove that Mendoza was not regarded by the company as a mere conduit or automaton for the performance of the physical act of placing the policy in the hands of the insured. If Mendoza were only an automaton then the legally effective delivery of the policy and the consummation of the contract occurred when the company expressed its will to release the policy by mailing it to its agent, namely, on January 11, 1933. In such a case the agent would perform a purely ministerial act and have no discretion. He could do nothing but make unconditional delivery. The legal result would be the same as if the company had mailed the policy on January 11, 1933, to the insured directly using the post-office as its conduit for delivery. On January 11, 1933, the insured was in good health performing his regular duties in the Bureau of Printing. But we are not inclined to take such a restrictive view of the agent's authority because the evidence in the record shows that Mendoza had the authority, given him by the company, to withhold the delivery of the policy to the insured "until the first premium has been paid and the policy has been delivered to and accepted by me (the insured) while I am in good health". Whether that condition had been met or not plainly calls for the exercise of discretion. Granted that Mendoza's decision that the condition had been met by the insured and that it was proper to make a delivery of the policy to him is just as binding on the company as if the decision had been made by its board of directors. Granted that Mendoza made a mistake of judgement because he acted on insufficient evidence as to the state of health of the insured. But it is not charged that the mistake was induced by any misconduct or omission of duty of the insured.

It is the interest not only the applicant but of all insurance companies as well that there should be some act which gives the applicant the definite assurance that the contract has been consummated. This sense of security and of peace of mind that one's defendants are provided for without risk either of loss or of litigation is the bedrock of life insurance. A cloud will be thrown over the entire insurance business if the condition of health of the the insured at the time of delivery of the policy may be required into years afterwards with the view to avoiding the policy on the ground that it never took effect because of an alleged lack of good health, at the time of delivery. Suppose in the present instance that Sindayen had recovered his health, but was killed in an automobile accident six months after the delivery of the policy; and that when called on to pay the loss, the company learns of Sindayen's grave illness on January 18, 1933, and alleges that the policy had never taken effect. It is difficult to imagine that the insurance company would take such a position in the face of the common belief of the insuring public that when the policy is delivered, in the absence of fraud or other grounds for rescission, the contract of insurance is consummated. The insured rests and acts on that faith. So does the insurance company, for that matter, for from the date of delivery of the policy it appropriates to its own use the premium paid by the insured. When the policy is issued and delivered, in the absence of fraud or other grounds for rescission, it is plainly not within the intention of the parties that there should be any questions held in abeyance or reserved for future determination that leave the very existence of the contract in suspense and doubt. If this were not so, the entire business world which deals so voluminously in insurance would be affected by this uncertainly. Policies that have been delivered to the insured are constantly being assigned for credit and other purposes. Although such policies are not negotiable instruments and are subject to defenses for fraud, it would be a most serious handicap to business if the very existence of the contract remains in doubt even though the policy has been issued and delivered with all the formalities required by the law. It is therefore in the public interest, for the public is profoundly and generally interested in life insurance, as well as in the interest of the insurance companies themselves by giving certainly and security to their policies, that we are constrained to hold, as we, do, that the delivery of the policy to the insured by an agent of the company who is authorized to make delivery or without delivery is the final act which binds the company (and the insured as well) in the absence of fraud or other legal ground for rescission. The fact that the agent to whom it has entrusted this duty (and corporation can only act through agents) is derelict or negligent or even dishonest in the performance of the duty which has been entrusted to him would create a liability of the agent to the company but does not resolve the company's obligation based upon the authorized acts of the agent toward a third party who was not in collusion with the agent. Paragraph 4 of the application to the effect "that the agent taking this application has no authority to make, modify or discharge contracts or to waive any of the company's rights or requirements" is not in point. Mendoza neither waived nor pretended to waive any right or requirement of the company. In fact, his inquiry as to the state of health of the insured discloses that he was endeavoring to assure himself that this requirement of the company had been satisfied. In doing so, he acted within the authority conferred on him by his agency and his acts within that authority bind the company. The company

therefore having decided that all the conditions precedent to the taking effect of the policy had been complied with and having accepted the premium and delivered the policy thereafter to the insured, the company is now estopped to assert that it never intended that the policy should take effect. (Cf. Northwestern Life Association vs. Findley, 29 Tex. Civ. App., 494; 68 S.W, 695; McLaurin vs. Mutual Life Insurance Co., 115 S.C., 59; 104 S.E., 327; 14 Aal. Jur., par. 12, pages 425-427.) In view of the premises, we hold that the defendant company assumed the risk covered by policy No. 47710 on the life of Arturo Sindayen on January 18, 1933, the date when the policy was delivered to the insured. The judgment appealed from is therefore reversed with directions to enter judgment against the appellee in the sum of P1,000 together with interest at the legal rate from and after May 4, 1933, with costs in both instances against the appellee. Malcolm. Villa-Real, Abad Santos, Hull, Vickers, Goddard, and Recto, JJ., concur.

Separate Opinions AVANCEA, C.J., concurring: I concur in the result of this decision. I agree with the conclusion arrived in the majority opinion in the sense that the contract in question was consummated. I am of the opinion, however, that this contract was consummated by the defendant due to an error regarding an essential condition, to wit: the the good health of the insured. There is no doubt but that the defendant would not have consummated the contract had it known that the insured was hopelessly ill, inasmuch as this consideration is essential in this kind of contracts. It is not true that the defendant or its agent had waived this condition inasmuch as it consummated the contract in the belief that this condition had been compiled with, in view of the information given to it in good faith by the agent of the insured to the effect that the latter might continue to be in good health for the reason that she had not received any information from him to the contrary. This being so, the defendant's consent is vitiated by error, and, inasmuch as it affects an essential condition of the contract, it may give rise to the nullity thereof. However, inasmuch as the nullity of the contract has not been set up as a a defense in this case, I concur with the majority in the result.

IMPERIAL, J., dissenting:

The plaintiff, as beneficiary brought this action recover from the defendant, an insurance Company, the sum of P1,000, the value of a life insurance policy issued the name of Arturo Sindayen, the plaintiff's husband. The plaintiff appealed from the judgment dismissing the complaint, without special pronouncement as to costs. On December 26, 1932, Arturo Sindayen signed Exhibit 6 wherein he applied for life insurance in the sum of P1,000 under certain conditions, among others, the following: 3. That the said policy shall not take effect until the first premium has been paid and the policy has been delivered to and accepted by me, while I am in good health. 4. That the agent taking this application has no authority to make, modify or discharge contracts, or to waive any of the company's right or requirements. On the back of the policy said conditions were endorsed as follows: THE CONTRACT. This Policy and the application herefor constitute the entire contract between the parties hereto. All statements made by the Insured shall, in the absence of fraud, be deemed representations and not warranties, and no such statement shall void the Policy unless it is contained in the written application, a copy of which is attached to this Policy. Only the President, or the Manager, acting jointly with the Secretary or Assistant Secretary (and then only in writing signed by them) have power in behalf of the Company to issue permits, or to modify this or any contract, or to extend the time for making any premium payment, and the Company shall not be bound by any promise or representation heretofore or hereafter given by any person other than the above-named officials, and by them only in writing and signed conjointly as stated. The insurance was secured by the defendant's agent Cristobal Mendoza in Camiling, Tarlac. The first premium to be paid by the insured amounted to P40.06 and on account of this sum he paid the agent P15 after he signed the application, with the understanding between them that the balance of P25.06 would be paid in the same town on the date the policy would be delivered. The insured designated his aunt Felicidad Estrada to act as his representative and to receive in his name the policy and to pay the balance of the premium. On January 11, 1933, the defendant issued insurance policy No. 47710, dated December 1, 1932 and sent it by registered mail to its agent in Camiling, Tarlac. On January 16th the agent got the policy from the post office and on the 18th he looked for the insured, but Felicidad Estrada informed him that the insured had returned to Manila. The agent asked her whether the insured continued to be sound and in good health, to which she replied that she believed that he was in good health inasmuch as she received no information that he was sick, whereupon the agent delivered the policy to Felicidad Estrada with instruction to hand it to the insured and, after receiving the sum of P25.06, he issued the receipt for the payment of the

premium of P40.06, signing it as defendant's agent. On January 19th Felicidad Estrada came to Manila, to the home of the insured at No. 14 Teresa Street, to deliver the policy, but she found that he died a few hours before her arrival and there she saw his lifeless body. Felicidad Estrada delivered the policy to the plaintiff as beneficiary. On January 20th of the same year the agent had knowledge of the death of the insured and went to see Felicidad Estrada whom be requested to return the policy so that the defendant would decide what was to be done. On that occasion the agent conveyed to Felicidad Estrada his belief that the insured was not in good health when he delivered the policy to her. Felicidad Estrada returned the policy to the agent on the afternoon of said date. The agent gave notice to the defendant of the death of the insured and of the circumstances under which, he had delivered the policy, and the defendant on February 4th of the same year returned to the plaintiff by check all the premium theretofore received, and furthermore secured from her Exhibit A (Accord, Satisfaction and Release), by virtue of which said plaintiff acknowledged having received the aforesaid premium and that in further consideration thereof she formally waived whatever right she might have, as beneficiary, in the insurance policy issued in the name of her deceased husband. With respect to the sickness of the deceased, it appears that on January 1, 1933 he was examined by the physician of the defendant company. On the 12th of the same month he felt ill and consulted Dr. Alfredo L. Guerrero who, after an examination, found him suffering from nephritis. On the 15th he was treated for the second time by the physician, who found him seriously ill and with fever. In the afternoon of January 19, 1933, he died from nephritis and uremia in his home in Manila. In its answer the defendant set up two special defenses: (1) That the plaintiff bas lost any and an right to collect the value of the policy because at the time the first premium was paid and the policy was delivered to the insured, the latter was not in good health, thus violating clause 3 of the application which he signed and was made an integral part of the policy as one of the conditions thereof; and (2) that the plaintiff by means of the document known as "Accord, Satisfaction and Release" has waived whatever right she might derive from the insurance policy. A stipulation or contract between the company and the applicant in the sense that the insurance policy will produce no effect or will not be binding on the company unless the first premium shall have been paid while the applicant is alive and in good health, is valid will will be enforced in accordance with the terms thereof; it is a condition precedent to the liability of the company, and compliance therewith or its waiver are necessary for the enforcement and fulfillment of the insurance contract, unless the case should come under the provisions of an uncontestable clause. ([Perry vs. Security L., etc., Co., 150 N.C., 143; 63 S.E., 679; Rathbun vs. New York L. Ins. Co, 30 Ida., 34; 165 P., 997; Hawley vs. Michigan Mut. L. Ins. Co., 92 Iowa, 593; 61 N.W., 201; Whiting vs.Massachusetts Mut. L. Ins. Co., 129 Mass., 240; 37 Am. Rep., 317; Missouri State L. Ins. Co. vs. Salisbury, 279 Mo., 40; 213 S.W., 786; Ormond vs. Fidelity Life Assoc., 96 N.C., 158; 1 S.E., 796; Bowen vs. New York Mut. L. Ins. Co., 20 S.D., 103;

104 N.W., 1040; Rositer vs. Aetna L. Ins. Co., 91 Wis., 121; 64 N.W., 876; Anders vs. Life Ins. Clearing Co., 62 Neb., 585; 87 N.W., 331; Reliance L. Ins. Co. vs. Hightower, 148 Ga., 843; 98 S. E., 469; Clark vs.Mutual L. Ins. Co., 129 Ga., 571; 59 S.E., 283; Reese vs. Fidelity Mut. Life Assoc., 111 Ga., 482; 36 S.E., 637 [foll. Williams vs. Empire L. Ins. Co., 146 Ga., 246; 91 S.E., 44); Oliver vs. New York Mut. L. Ins. Co., 97 Va., 134; 33 S.E., 526; Reese vs. Fidelity Mut. Life Assoc., 111 Ga., 482; 36 S.E., 637; Anders vs. Life Ins. Clearing Co., 62 Neb., 585; 87 N.W., 331; Perry vs. Security L. etc., Co., 150 N.C., 143; 63 S.E., 679; Strigham vs. Mutual Ins. Co., 44 Ore., 447; 75 Pac., 822; Dibble vs. Reliance L. Ins. Co., 170 Cal., 199; 149 Pac., 171.] Ann. Cas. 1917E, 34.) In the case of Reliance Life Ins. Co. vs. Hightower, supra, the Supreme Court of Georgia, in a similar case, said the following: . . . An application for life insurance, signed by the applicant, contained a provision as follows: "I hereby declare and agree that all statements and answers written in this application . . . are true, full, and complete, and are offered to the company as a consideration for the contract of insurance, which I hereby agree to accept, and which shall not take effect until the first premium shall have been actually paid while I am in good health and the policy shall have been signed by the duly authorized officers of the company and issued." The policy itself contained, among others, the following provisions: "Agents are not authorized to modify this policy or to extend the time for paying a premium . . .. All insurance provided by this policy is based upon the application therefore, a copy of which is hereto attached and made a part of this policy." xxx xxx xxx

Applying to the facts above stated the principles recognized in Reese vs. Fidelity Mutual Life Association (111 Ga., 482; 36 S. E., 637), it must be ruled: (1) It was within the power of the insurance company, as between itself and its agent, to define and limit the powers of the latter. Limitations upon the power of the agent affect all third persons dealing with him, who have knowledge or notice thereof; and any notice of limitations upon the agent's power which a prudent man is bound to regard, is the equivalent of knowledge to the insured; (2) the stipulation in the signed application, that the insurance "shall not take effect until the first premium shall have been actually paid while I am in good health," coupled with the words in the policy, "Agents are not authorized to modify this policy or to extend the time for paying a premium," were sufficient to charge the applicant with notice that he was dealing with a special agent with limited powers; (3) the actual payment of the first premium during the good health of the applicant was a condition precedent to liability under the policy, and the agent of the company could not waive such condition.

In the case of Missouri State Life Ins. Co. vs. Salisbury, supra, the Supreme Court of Missouri, in another similar case, said: The application has this clause: "6. That the insurance hereby applied for shall not take effect unless the first premium is paid and the policy delivered to and accepted by me during and lifetime and good health." Another reason why the contract was never completed was because the first premium was na paid nor tendered during the good health of Mrs. Salisbury, as required by the stipulation in the application quoted above. A stipulation of that character, requiring the payment of a first premium in advance as a condition upon which the policy was to take effect, is is always recognized and enforced by the courts. The policy, in such case, is not effective until that condition is complied with. (Kilcullen vs. Life Ins. Co., 108 Mo. App., 61; 82 S.W., 966; Wallingford vs. Home Mut. Fire & Marine Ins. Co., 30 Mo., 46; Ormond vs. Insurance Co., 96 N.C., 158; 1 S.E., 796; Bowen vs. Mutual Life Ins. Co., 20 S.D., 103; 104 N.W., 1040.) In the case of Rathbun vs. New York Life Ins. Co., supra, the Supreme Court of Idaho said: In its answer and on the trial of the case, the main contention of the insurance company were: First, that under, the terms of the contract the first premium was to be paid in cash; and, second, the policy was not to take effect until the insured was in good health at the time it was delivered to him. Said contentions are partly based upon the stipulations above quoted from the application for said insurance. The court in its findings of fact, among other things, found as follows. "The court further finds that Ernest C. Rathbun, the applied in writing for insurance on his life, that the insurance thereby applied for effect unless the first premium was paid and the policy was delivered to and received by him during his lifetime and good health. After applying for the policy and before its delivery, the applicant was taken with appendicitis, from which he died. While he was in the hospital, the soliciting agent at Spoken, in total ignorance of the changed condition of the applicant's health, mailed him the policy. The applicant's friends thereafter paid the first premium, which the company promptly returned when it discovered facts." The evidence is clearly sufficient to sustain this finding of fact. Then if the parties understood and agreed that the policy should not become effective unless the first premium was paid and the policy was delivered to and

received by the applicant during his lifetime and while he was in good health, and both of those conditions failed, the contract of insurance was never completed, and the policy was of no force and effect. It is a well-recognized rule that life insurance results from contract, and that the true rule is that no other or different rule is to be applied to a contract of insurance than is applied to other contracts. (Quinlan vs. Providence-Washington Ins. Co., 133 N.Y., 356; 28 Am. St. Rep., 645; 31 N.E., 31.) In life insurance contracts, the assent of both parties is required as in any other contract. (Stephens vs. Capital Ins. Co., 87 Iowa, 283; 54 N.W., 136; Weidenaar vs. N.Y. Life Ins. Co., 36 Mont., 592; 122 Am. St., 330; 94 Pac., 1.) In the determination of this case, the application and the policy itself must be examined and considered in order to ascertain the true situation of the parties under the negotiations and agreements between them. (Iowa Life Ins. Co. vs. Lewis, 187 U.S., 335; 23 Sup. Ct., 126; 47 Law. ed. 204; Behling vs. N.W. Nat. Life Ins. Co., 117 Wis., 24; 93 N.W., 80O.) If we concede in this case that the premium was paid by the payment of the $5 and the delivery of the insured's promissory note to the agent of the company for the balance, the plaintiffs would not be entitled to recover, for the reason that the policy was not delivered to and received by the applicant while he was in good health, but hen he was fatally ill. He became ill with appendicitis on the 28th of April, 1913, was operated on that day and thereafter died on the 10th day of May, 1918, five days after receiving the policy. In the case of Gordon vs. Prudential Insurance Company (231 Pa., 404), the Supreme Court of Pennsylvania said: . . . In the case at bar, the policy was issued and handed to the agent, who delivered it to the insured before payment of the premium, and upon the insured giving a receipt, in which it was stated that the policy was "received for the purpose of inspection only and upon the understanding that it is not to be in force until the first premium payable thereunder has been paid by me and the official receipt of the company delivered to me during my lifetime and in good health, as provided in my application upon which the above numbered policy was issued." This, therefore, was a conditional delivery of the policy and the contract could not be consummated except upon performance of that condition, namely, payment of the premium, thereafter, while the insured was alive and in good health, as provided in both the application and receipt for the policy. xxx xxx xxx

It is therefore undisputed that on the day of the payment of the premium, Mr. Gordon was ill of the disease which caused his death within sixty-four hours after such payment. There was no dispute, nor contradictory testimony as to the

condition of Mr. Gordon's health on the day of payment, and, therefore, nothing for the jury to pass upon in this respect. xxx xxx xxx

In the case at bar, there was no question of the condition of the health of the insured on the day of the payment of the premium, and and no conflicting testimony as to the serious nature of his illness on that day, nor as to any other material fact in the cause. No person testified that Mr. Gordon was in "good health" on Saturday, May 16, the day the premium was paid, but on the witness who had knowledge of his condition and who was asked the question, including the, plaintiff herself, said that he was not in "good health" on that day. How, then, can a jury be permitted to find that he was in "good heath" at the time of the payment of the premium in the absence of any evidence to warrant or support such finding? xxx xxx xxx

In this case it is impossible to find from the evidence that on Saturday, May 16, the day of the payment of the premium, and at the time of such payment, the applicant had no grave, important or serious disease, or that he was free from any ailment that seriously affected the general soundness and healthfulness of his system, or that he suffered a mere temporary indisposition which did not tend to weaken or undermine his constitution at the time of paying the premium. Nor is it possible to find that he enjoyed such health and strength as to justify a reasonable belief that he was free from derangement of organic functions, or free from symptoms calculated to cause a reasonable apprehension of such derangement, and that to ordinary observation and outward appearance his health was reasonably such that he might, with ordinary safety, be insured and upon ordinary terms which only would satisfy the requirement of "good health". But on the contrary, the testimony conclusively shows that on Saturday May 16,1908, at the time of the payment of the premium, the condition of Mr. Gordon's health was both a serious and a dangerous one, and such as would preclude the possibility of any life insurance company, with knowledge of his condition, issuing its policy upon his life for anything like the ordinary premium; in other words, his condition at that time was such as to render him a hazardous and dangerous risk, which would not be assumed by any insurance company upon receipt of the of the ordinary premium for insurance upon the life of an ordinary risk. With the question of good faith on the part of the insured at the time of paying the premium, we have nothing to do. The fact is that his physical condition was not disclosed to the company or its agent at the time of the payment of the premium; and that his condition was not at that time such as, in his application for insurance, he stated it to be. This being true, it is no leader hardship upon the

beneficiary in the policy to say that the premium paid under such conditions does not entitle her to recover the amount of insurance from the defendant company. In the case of Powell vs. Prudential Insurance Co. of America (153 Ala., 611), the Supreme Court of Alabama, in a similar cause, said: On June 22, 1904, Claude D. Powell applied to the defendant company for insurance on his life for $1,000. In his application for insurance, he stated: "I am in good health, . . . and all the statements and answers to the above questions are complete and true, and that the foregoing, together with this declaration, shall constitute the application, and become a part of the contract for insurance hereby applied for. And it is agreed that the policy herein applied for shall be accepted subject to the privileges and provisions therein contained, and said policy shall not take effect until the same shall be issued and delivered by the said company, and the first premium paid thereon in full, while my health is in the same condition as described in this application." xxx xxx xxx

Here we find that two absolute conditions precedent of the contract of insurance, were set aside or annulled, in what the friends of the deceased attempted to do, in that, the the firsts premium was never paid by the assured one any one for him, and if, by any possible construction, it could be held that it was not totally sick at the time, of which fact the company was ignorant; and further, it is not denied that the policy was never delivered if was done could possibly amount to delivery until after the death of the assured. To hold that the policy was good under such circumstances, would be to abrogate and set aside the contract of insurance, and hold the company liable for a payment of the policy against the very terms of its contract. The same principle controls and applies when, as in the instant case, it is stipulated that the policy shall be of no effect if at the time of its delivery to the insured he is not in good health. The condition is valid and binding when its refers only to the payment of the first premium as well as to the delivery of the policy, or to both. In the case of Nyman vs. Manufactures' & Merchants' Life Ass'n. (104 N.E., 653), the Supreme Court of IIlinois said: . . . The proof is direct and positive that on the last-named date she was not in good health, and that two months and three months day later she died from the disease the proof showed she was suffering from on that day. If there had been no proof of the condition of Mrs. Nyman's health on the day the certificate was delivered, then there would be some force in plaintiff's contention that the

inference might be indulged that, if she was in good health on April 11th, she so continued until the 19th. But no such inference can be indulged, when the uncontradicted proof shows she was in bad health the day the certificate was delivered, and so continued until her death. Defendant proved its third special plea, and, in our opinion, plaintiff offered no evidence that legitimately tended to rebut defendant's evidence. The trial court therefore erred in refusing to direct a verdict in favor of defendant under the issue made by the third special plea. (Libby, McNeill & Libby vs. Cook, 222 Ill., 206; 78 N.E., 599.) In the case of American Bankers' Ins. Co. vs. Themas (53 Okla. Rep., 11), the Supreme Court of Oklahoma said: That part of the policy which provides that the same shall not take effect until it is delivered by the company while the insured is in good health prescribes a condition precedent to the attachment of the risk under the policy. (1 Cooley's Briefs on the Law of Insurance, p. 451.) Recognizing it to be such, plaintiff properly pleaded a waiver thereof by setting up the facts as stated. (Western, etc., Ins. Co. vs. Coon, 38 Okla., 453; 134 Pac., 22; Anders vs. Life Ins. Clearing Co., 62 Neb., 585; 87 N.W., 33 1.) In the case of Steinsultz vs. Illinois Bankers Life Association (229 Ill. App. Rep., 199), the third district of the Appellate Courts, in a similar cause, said: The policy of insurance contains the following clause: "I agree to accept the Policy issued hereon and that the same shall not take effect until the first payment shall have been made and the Policy issued and actually delivered to me during my continuance in good health." The main question in this case, in the opinion of this court, is the question as to whether a valid and legal policy ever was issued and actually delivered to the insured, Myrtle May Steinsultz. It is argued that the clause in question is a condition precedent and requires that the insured shall be in good health at the time of the payment of the first premium and the actual delivery of the policy to her, otherwise that the policy never became operative and for the purposes of this suit is void. It will be noticed that plaintiff in representing his main case made no effort to submit or show anything as to the health of the insured prior to the claimed delivery of the policy. If the clause in question is a condition precedent to recovery, which we shall discuss later, the general issue filed by the defendant denied the existence of a valid policy and raised this question and required proof on the part of the plaintiff to show that the insured was in good health at the time of the claimed delivery of the policy. No much proof was shown and the defendant, appellant, at the close of plaintiff's case, moved the court to instruct the jury, under the pleadings and evidence in the case, to find verdict for the defendant and form a verdict was submitted with the motion. This motion the court overruled, to which ruling appellant duly excepted and this issue is

therefore squarely raised by the proceedings as the existence of legal and binding policy in the case under the terms of said contract. In Ellis vs. State Mut. Life Assur. Co. of Worcester (206 III. App., 226), the appellant insurance company filed a plea of the general issue with notice of special matter of defense, the special matter being that the policy was not to be in effect until actually delivered and the first premium paid during the lifetime of the assured, and while he was in the same condition of health as when his application was signed, and that the policy was not so delivered. There was a trial, verdict and judgment in favor of appellee, being the amount of the policy and interest. To reverse said judgment the appellant prosecuted appeal. In this case the application, signed by Ellis, contained, among other things, the following provision: "That the contract or policy applied for shall not take effect until the first premium thereon shall have been actually paid and the policy delivered to me during my lifetime and the present condition of health." The policy issued thereon contained this provision: "This policy shall not take effect until actually delivered and the first premium paid thereon during the lifetime of the insured." Said policy contained the further provisions: "This policy and the application therefor shall constitute the entire contract between the parties hereto." In this case, likewise, the appellant at the close of appellee's evidence and then again at the close of all the evidence, moved the court to direct a verdict in its favor. Appellant objected to the admission of the policy sued upon, in evidence. In this case on December 14, 1914, the insured was injured and was carried to his home and died between 4:30 and 5 p.m. on that day, and it appears that the policy of insurance had been returned to the office of the agent of the insurance company the evening before but had not been delivered personally to the insured at the time of his death. In this case the contention was made by the holders of the policy and that the delivery to the agent was a delivery to the insured. The court goes into the question in the Ellis case very exhaustively, quoting from a great many cases and qouting from Devine vs. Federal Life Ins. Co. (250 III., 203), in which the Supreme Court in discussing the question of the delivery of an insurance policy, at page 206, says: "The application may or not provide that the insurance shall effect only upon the delivery of the policy to the insured. Unless expressly made so by the contract itself, an actual delivery of a policy of insurance to the insured is not essential to the validity of the contract, and the rule under such circumstances is that a policy becomes binding upon the insurer when signed and that forwarded to the insurance broker to whom the application as made, to be delivered to the insured."

And quoting 25 Cyc 718, 719, it is stated with reference to the delivery of insurance policies that: "The placing of the completed policy on hands of the agent for the delivery, without condition, to the insured completes the contract, though the actual delivery by the agent to the insured is not made before the death of the insured. But if the delivery to the agent of the company is with the understanding that it is to be delivered by the agent to the insured only after the performance of some condition, then until the condition is performed and it becomes the duty of the agent to deliver the policy to the insured, the contract is not complete. . . . It is usual condition of a life insurance policy that the delivery shall not be effectual to create a binding contract unless the insured is alive in good health when the policy is delivered and the first premium paid, and under such conditions the death of the insured before the delivery of the policy will prevent its becoming effectual. It was held in the Ellis case that in view of foregoing authorities, numerous of which we have not cited here, that the policy sued on was never delivered and that the court erred in not directing a verdict in favor of appellant and reversed the judgment with a finding of fact. The language in the policy in question, "I agree to accept the Policy issued hereon and that the same shall not take effect until the first payment shall have been made and the Policy issued and actually delivered to me during may continuance in good health," is a condition precedent to the existence of any binding legal contract of insurance upon the appellant. It means just what its says and it was entered into signed by the insured. The statement was a warranty that the insured was in good health at the time she signed said application and further was a binding obligation that she should continue in good health at the time the policy was delivered to her, otherwise the policy never should become binding and obligatory. It is condition that goes to the very existence of the policy and its validity, and under the facts in this case it is insisted strenuously that no binding policy was ever issued and delivered by the appellant. And in the case of Federal Life Ins. Co. vs. Wright (230 S.W., 795), the Civil Appellate Court of Texas said: . . . The application and the policy contain the entire contract between the parties, and it is not only agreed in the application that all of the statements therein "are full, true, and complete," but it is stipulated therein, as above shown, that the policy of insurance applied for shall not take effect until the policy shall have been actually delivered to the insured and the premium paid during his life and while he was in good health. The purpose and meaning of this provision, standing alone or taken in connection with any or all other provisions of the contract, is clear, without ambiguity, and not to open to construction. It unquestionably means that the policy should not take effect as a contract of insurance unless actually delivered to the applicant therefor while he was in good health. This being the meaning of the provision, and the appellee having

admitted in her pleadings and in open court at the trial that the applicant or insured was afflicted with tuberculosis of the lungs at the time the policy was delivered to him, and that such disease caused his death, the policy by its terms never became an obligation of the appellant. Applications for policies of life insurance frequently provide, as in the present instance, that the policy shall not take effect unless it is delivered to the insured and the premium paid while he is in good health, and the great weight of authority is to the effect that such provision is valid, and that if the insured was not in fact in good health on the date the policy was delivered the company is not liable. (Gallant vs. Metropolitan L. Ins. Co., 167 Mass., 79; 44 N.E. 1073; Murphy vs. Metropolitan Life Ins. Co., 106 Minn., 112; 118 N. W., 365; Logan vs. New York L. Insurance Co., 107 Wash., 253; 181 Pac., 906; Metropolitan L. Insurance Co. vs.Willis, 37 Ind. App., 48; 76 N.E., 560; Gallop vs. Royal Neighbors of America, 167 Mo. App., 85; 150 S.W., 1118; Metropolitan L. Insurance Co. vs. Betz, 44 Tex. Civ. App., 557; 99 S.W., 1140; American Nat. Insurance Co. vs. Anderson, 179 S.W, 66; Security Mut. L. Ins. Co. vs. Calvert, 39 Tex. Civ. App., 382; 87 S.W., 889; Seaback vs. Metropolitan L. Ins. Co., 274 Ill., 516; 113 N.E., 862; Mutual L. Insurance Co. vs.Willey, 133 Md., 665; 106 Atl., 163.) It is also held that it is immaterial that the condition of the insurer's health has changed since his application was made, or that he was ignorant of his condition. (Carmichael vs.Hancock Mut. Ins. Co., 116 App. Div., 291; 101 N. Y. Supp., 602; Metropolitan L. Ins. Co. vs. Howle, 62 Ohio, 204; 56 N.E. 908, Id., 68 Ohio, 614; 68 N.E., 4; Oliver vs. Matual L. Ins. Co., 97 Va., 134; 33 S.E., 536; Packard vs. Metropolitan L. Ins. Co., 72 N.H., 1; 54 Atl., 287.) This defense, as we now view it, is separate and distinct from the defense that misrepresentations were made in the application for the policy, and our conclusion is that the failure of the appellant to give notice to the insured or beneficiary, within a reasonable time after discovering that the insured had tuberculosis of the lungs, that it would not be bound by the contract of insurance did not render unavailing the provision that unless the policy was delivered while the insured was in good health the contract should not take effect. Under article 4948 of the statute, it was necessary for the appellant, in order to avail itself of the defense based upon misrepresentations made in the application to secure the policy, to show that it gave the insured or beneficiary notice within a reasonable time after discovering the falsity of such representations that it would not be bound by the contract of insurance; but in order to sustain the firstmentioned defense, the same having been asserted within the contestable period, it was necessary only to show that the insured was not in good health when the policy was delivered. We do not agree with the contention to the effect that by pleading and proving that the first premium was paid and received when the application for the policy was made, which was a few days prior to the delivery of the policy, the appellee showed an express waiver of the provision in the application making the assumption of any liability on the part of appellant

dependent upon the good health of the insured at the time the policy was delivered. The provision, as before stated, is clear and unambiguous and susceptible of but one construction. By its plain and unmistakable terms the insured agrees that all the statements and answers contained in the application are full, true, and complete in every respect, and are offered to the insurance company as a consideration a contract of insurance, which shall not take effect unless the policy shall have been actually delivered to him while he was in good health. Nor shall it take effect unless the first premium shall have been actually during his life and paid while he was in good health. In other words, if the insure was not in good health at the time the policy was delivered to him, or if he was dead or in bad health when the first premium was paid, then, in either event, no obligation on the part of the insurance company was assumed, and, of course, there was no contract of insurance. It was as much a condition precedent to the taking effect of the contract that the first premium be paid during the life of the insured and while he was in good health, as that the policy be delivered while he was in good health, and the fact that the premium was paid when the application was made, and a few days in advance of the delivery of the policy, can furnish no basis for the holding that thereby the other condition was abrogated or waived. We can see no good reason for saying that the provision relative to good health at the time of the payment of the first premium of the policy was inserted to cover cases "when the first premium was collected at a time subsequent to the issuance of the policy, either at or prior to the delivery thereof." The provision under consideration is not one which the insurance company may avail itself of to avoid an executed contract, or one which in the ordinary sense constitutes a warranty of the good health of the insured, but its effect was to prevent the taking effect of the contemplated contract, unless there was a compliance with the conditions precedent named therein. Differently stated, with such a provision in the application for the policy the contract is not a completed one, is not absolute but conditional, and in this case it is the fact of sound health, etc., in the insured on the date of the delivery of the policy that determines the liability of the appellant. In her motion for a rehearing the appellee asserts that our holding on the appellant's motion for rehearing, to the effect that since the application for the policy sued on, which as a part of the contract of insurance, stipulated that the policy should not take effect until the same was actually delivered to the insured and the first premium paid during his life and while he was in good health, and since it was admitted by the appellee and conclusively shown that the insured had tuberculosis of the lungs at the time the policy was delivered to him the first premium paid, the policy its terms never became an obligation of the and the appellant, is different from or in conflict with the decision in the cases of American National Life Insurance Co. vs. Rowell (175 S.W., 170); American National Insurance Co. vs. Burnside (175 S. W., 169) ; American National Life

Insurance Co. vs. Fawcett (162 S.W. 169); National Fire Ins. Co. vs. Carter (199 S.W., 507); and Mecca Fire Insurance Co. vs. Stricker (136 S.W., 599) The first three of the cases mentioned were decided by this court, the fourth by the Court of Civil Appeals for the First District, and the fifth by the Court of Civil Appeals for the Third District. Our conclusion is that neither of these cases is in conflict with the decision in the first case referred to and the present case, but it seems manifest, from a careful examination and analysis of the opinion in that case, that the court did not have in mind the precise question here involved, and did not there expressly pass on it. There it was urged that the trial court erred in over ruling the insurance company's demurrers to Rowell's petition, because it was not alleged that the insured was in sound health at the time the policy sued on was issued, and this court held that there was no error in overruling the demurrers, since, if the insured was not, in fact, in sound health at that time, such fact was a matter of defense to be pleaded by the company. It was further there held that while the defendant averred that the insured was not in sound health when the policy was issued, such defense was not sufficiently pleaded to justify the isffitc of testimony to establish it. The opinion also indicates that the insurance company in its pleadings and assignments of error treated the provision in the policy, that no obligation was assumed by it unless on the date of issuance the insured was in good health, as a representation or warranty, and that this court, discussing the matter as presented, after stating in substance the provisions of article 4948 of the statute said that the failure to give the notice prescribed in that statute absolutely barred the insurance company from defending in action on the policy because of alleged misrepresentations. We also declared that said statute applied to covenants of warranty as well as to statements in the application not made warranties by the contract, citing Mecca Fire Ins. Co. vs. Stricker,supra. Moreover, the stipulation that the insurance contract shall produce no effect unless the payment of the first premium and the delivery of the policy be made when the insured is in good health, is not in conflict with any provision of the Insurance Law now in force, nor with any other law of a general character; neither is said stipulation contrary to morals or public order, and therefore the same is valid and binding upon the parties. (Articles 1255, 1257 and 1258, Civil Code.) The majority opinion states that the delivery of the policy by the agent after he has made use of the discretion conferred upon him by the defendant to deliver or to withhold said policy, is binding upon the defendant and the latter cannot evade the consequences thereof. This same legal question has been raised before various appellate courts of several states of the Union, which made a distinction between agents whose only power consisted in soliciting insurance and in delivering policies and those who, in addition to such power, were authorized to issue policies and accept risks on behalf of insurance companies. In the first case the doctrine is uniform that the acts of agents with limited powers are not binding upon the insurance companies, whereas in the second case the acts of the agents bind and prejudice the insurance companies

represented by them. This legal question has been extensively considered and squarely decided in the case American Bankers' Ins. Co. vs. Thomas, supra, as follows: Favoring liability, she contends that the knowledge of Martin of the ill health of the insured at the time the policies were delivered was the knowledge of the company and waiver of the condition. Not so Assuming that Martin, was the agent of the company at that time, with authority to deliver the policies, it failing to appear that he had anything to do with the execution thereof or the acceptance of the risk, his knowledge was not that of the company. In Merchants' & Planters' Ins. Co. vs. Marsh (34 Okla., 453; 125 Pac., 1100), we held that the knowledge of the agent was the knowledge of the company only where the authority of such agent, derived from the company, was to solicit applications and execute and deliver contracts of insurance as an alter ego of the company, and that it was only in such case that he had power to waive the conditions of the policy. In that case the agent was, as here, a local or soliciting agent, and there the policy sued on was, as here, a 'home office policy", or one issued direct by the president and secretary of the company as distinguished from one issued by the local agent. There, in the syllabus, we said: "A local agent of an insurance company, whose only power is to solicit applications for insurance, and forward them to the company for approval, when, if approved to the insured, has no power to waive any of the provision of the policy so delivered.". . . Also in keeping with this rule is Des Moines Ins. Co. vs. Moon (33 Okla., 437; 126 Pac., 753). There we said: ". . . Where the local agent has the power to accept a risk and deliver a policy of insurance, and is advised and has full knowledge, at the time of the delivery of the policy, that certain conditions of the policy, which may be waived, are violated, such policy is binding upon the company, notwithstanding the fact that it contains a provision that none of the company's officers or agents can waive any of its provisions, except in writing, in upon the policy. This case (referring to Western National Ins. Co, Marsh, 34 Okla., 414; 125 Pac., 1049), unanimously concurred in by the members of the courts, settles the rule in this jurisdiction as to contracts of insurance written after the administration of the state: . . ." Of course, if the local agent had not power, as here, to accept the risk, he had no power to waive the condition precedent in the policy. Cases relied on by plaintiff which hold the contrary practically under the same state of facts fail to draw this distinction, and seem to hold that the knowledge of a mere soliciting agent of the company of the ill health of the insured at the time of the delivery of the policy is the knowledge of the company, and hence a delivery with such knowledge constitutes a waiver of the condition under consideration. They are Roe vs. National Life, etc. Co. (137 Iowa, 696,: 115 N.W., 500: 17 L.R.A. [N.S.], 1144); Connecticut, etc. Ins. Co. vs. Grogan ([Ky.] 52 S.W., 959); N.W. Life Ins.

Co. vs. Findley (29 Tex. Civ. App., 494; 68 S. W., 695) ; National Life Ins. Co. vs. Twiddel ([Ky.), 58 S.W,, 699) ; Home Forum Ben. Ordervs. Varnado ([Tex. Civ. App.], 55 S.W., 364), and others. But the distinction is referred to in Bell vs. Ins. Co. (166 Mo. App., 390; 149 S.W. 33). In that case the insured, who was plaintiff's brother, died at Nogales, Ariz., as a result of injuries received while working as a telegraph lineman. On July 17, 1909, he made application to defendant for policy of life insurance, payable in event of his death to plaintiff. He made it to defendants' soliciting agents at that place, and paid the first annual premium cash in hand. The application was forwarded to defendant by mail, and duly received in St. Louis, Mo., on July 23, 1909. The policy was conditioned the same as here. On July 27, the application was duly accepted, and the policy issued and was mailed August 4, 1909, to the soliciting agents for delivery to the insured. Upon its arrival on August 8, 1909, pursuant to instructions, the policy was deposited for him in the safe of the soliciting agents, along with other private papers of the insured kept there by him. Two days before that died on the night of August 11th. On August 6th, one of the soliciting agents visited the insured and knew of his injury. The court said: "There can be no doubt that it is competent for the parties to stipulate in the application for insurance, as here, that the policy shall not be affective or binding until delivered to, and accepted by the insured while in good health and the payment of the first premium is made. It is said that a contract of life insurance is not complete until the last act necessary to the done by the insured, under the conditions of the contract after acceptance of the application by the company, has been done by him, and the courts, therefore, in proper cases, sustain such agreements which operate to postpone the taking effect of the policy until the delivery and premium payment while the insured is in good health. (See I Bacon, Life Ins. [3d ed.], see. 272; Kilcullenvs. Met. Life Ins. Co., 108 Mo. App., 61; 82 S.W. 966; Misselhorn vs. Mutual Reserve, etc., Life Ins. Co., 30 Mo. App., 589; McGregor vs. Met. Life Ins. Co. [143 Ky., 488], 136 S. W., 889.) But though such be true, the provision for thus suspending the policy, as an effective contract, until the premium is paid and its delivery, while the insured is in good health, is for the benefit of the insurer, and obviously may be waived by it or by it or by its agent possessing authority with respect to that matter. (See Rhodus vs. Kansas City, etc., Ins. Co., 156 Mo. App. 281; 137 S.W., 907.) . . . But it is insisted that a mere soliciting agent, such as Cummings, is without authority to waive the condition in the policy here relied upon, and, for the purpose of the case, the proposition may be conceded as true. Whereupon the court proceeded to consider whether the company, under the facts in that case, had waived the condition in the policy relied upon. We are therefore of opinion that Martin was without authority to waive the condition relied on, and that plaintiff cannot recover unless defendant is stopped to deny that liability attached by in the petition. Joining issue on these allegations, defendant by answer in effect admitted accepting the premiums back to representative of the assured and demanded a return of the policies, which was refused, and the

for the reason, it is urged, defendant is not estopped to assert that no liability attached under the policies. It is clear, therefore, that the delivery of the policy by Mendoza does not bind the defendant, nor is the defendant estopped from alleging its defense, for the simple reason that Mendoza was not an agent with authority to issue policies or to accept risks in the name of his principle. There is another ground upon which the majority opinion is based, namely, that the defendant waived the defense it now invokes, by reason of the delivery of the policy by its invokes, by reason of the delivery of the policy by its agent. It is admitted that if the delivery of the policy was due to fraud, legally there could have been no waiver. In view of the facts established and admitted, there is no doubt, as to the existence of the fraud. A restatement of the facts will show such existence. It will be remembered that before the delivery of the policy Mendoza asked Estrada whether the insured continued enjoying good health, to which she answered that she thought he was in good health because she had had no information that he was sick. It will likewise be noted that the information, far from being correct or truthful, was incorrect and misleading because, it reality, on that occasion the insured was seriously ill from nephritis and uremia, almost in a moribund state. Estrada, as a representative of the insured was not only bound to give a truthful information on the state of health of the insured, but it was her duty to find out it his true state of health in order to give true and correct information. When she gave Mendoza as incorrect information tending to create the impression that the insured was well when in fact he was seriously ill, there is no doubt that she committed fraud and imparted a deceitful information to the defendant agent. It matters not that the fraud was involuntary and not chargeable to Estrada ; the truth is that it existed and that by reason of such fraud the policy was delivered, and both the agent and the defendant were misled into believing that the insured was enjoying good health. In case of Cable vs. United States Life ins. co. (111 Fed. Rep., 19), the seventh circuit of the United States Circuit Courts of Appeals, in deciding the same question of waiver, said: It is, however, urged that sufficient information was disclosed by Lord to McCabe to put the company upon inquiry, and that, with such notice, McCabe delivered the policy and received the premium; that McCabe was the agent of the company, and notice to him was notice to the company, and the delivery of the policy constituted a waiver of the condition and warrant. Upon the assumption that McCabe was such agent of the company, and that his action must be treated as the action of the company, and that his question which we do not determine, it becomes us to inquire of the sufficiency of the notice given, and whether the act of the delivery of the policy involved a waiver of the warranty. . . . The holder of the policy cannot be permitted to conceal from the company an important fact like that of the assured being in extremes, and then to claim a waiver of the forfeiture created by the act which brought the insured to that condition to permit such concealment and yet to give to the action of the company the same effect as though no concealment were made, would tend to

sanction fraud on the part of the policy holder, instead of protecting him against the commission of one by the company. (Insurance Co. vs. Wolff, 95 U.S., 326, 333; 24 Law. ed., 387, 390.) It cannot here be doubted that if the insurance company, or McCabe as its agent, had been informed of the fact, within the personal knowledge of Lord, that Cable was seriously ill with acute pneumonia, the policy would not have been delivered. It is difficult for us to believe that Lord, with that knowledge, could think he had a right to accept this policy; but, whether so or not, the concealment of the fact was a fraud upon the company. The statement made was deceptive and misleading, whatever were the intentions of Lord, and a court of equity ought not to permit the completion of the wrong. Courts of equity cannot sustain an insurance upon the life of a dying man when the nature of his malady and the seriousness of his illness are concealed from the insurer. The same doctrine has been applied when there is an attempt to show that the waiver or estoppel arises from the payment of the premium. In the case of Nyman vs. Manufacturers' & Merchants' Life Assn., supra, the court said: It is further insisted by plaintiff that defendant, by accepting and retaining premiums or assessments from the insured, is estopped from denying the validity of the certificate. The first premium was paid on the day the policy was delivered, and the last one two days before the insured's death. There is no proof whatever that defendant or its agent knew, before the the death of Mrs. Nyman, that, at the time the policy was delivered and the first premium paid, she was not in good health. Receiving premiums subsequently, with knowledge that she was them ill, could have no significance, if defendant was ignorant of the fact that the insured was in bad health when the policy was delivered and the first premium paid. If Mrs. Nyman had been in good health when she received the policy and paid the the first premium, defendant would not have been justified in refusing to accept premium if she afterwards from denying liability in this case must be knowledge that the insured was not in good health when the policy was delivered. The case presents another aspect, namely, the waiver made by the plaintiff of any and all benefits accruing from the policy, which waiver expressly appears in document Exhibit A, known as "Accord, Satisfaction and Release". The pertinent clauses of the document read as follows: Whereas, the. Insular Life Assce. Co., Ltd., claims that the delivery of the said policy No. 47710 was not valid because said delivery was made while the said Arturo Sindayen was not in good health; Whereas, the undersigned, Fortunata Lucero Sindayen, widow of the said Arturo Sindayen, is named as beneficiary in the said policy of life insurance; and

Whereas, it is the desire of the Insular Life Assce. Co., Ltd., and of the beneficiary, Fortunata Lucero Sindayen that all differences, controversies and disputes that may grow out of the insurance of the said policy of life insurance and out of the claims that the said beneficiary may make under the said policy of life insurance the settled and compromised; and Whereas, the said Insular Life Assce. Co., Ltd. has at the date hereof paid Fortunato Lucero Sinadyen, the beneficiary named in said policy of life insurance, the sum of Forty Pesos and Sixty Centavos (40.06), lawful money of the Philippine Islands, the receipt whereof is hereby acknowledge; Now, thereof, in consideration of the promises and the sum of Forty Pesos and Sixty Centavos (P40.06), said Fortunata Lucero Sindayen, for herself, her heirs, executors, administrators and assigns, release and forever discharge said Insular Life Assurance Co., Ltd., its successors, and assigns, of all claims, obligation or indebtedness which she, as such beneficiary over had or now has, hereafter can, shall, or may have, for, upon, or by reason of said policy of life insurance numbered 47710 upon the life of said Arturo Sindayen, the latter now deceased, or arising therefrom or connected therewith in any matter. There is no dispute that the aforesaid document was signed by the plaintiff. There was irregularity in its execution because it was authenticated by the notary public in the absence of plaintiff. It is admitted that due to this irregularity the document is not a public instrument, but there is no doubt that it is an authentic private instrument whose evidentiary value cannot be disregarded. Its terms are binding upon the plaintiff, who understood the same notwithstanding her denial. However, it it said that the defendant likewise waived the defense which gas hereinbefore been extensively considered, because it failed to return the first premium collected, and this alleged failure is predicated upon the statement contained in the penultimate paragraph of the instrument stating that the check for P40.06 was returned to the plaintiff in consideration of her waiver of any claim whatsoever. A careful reading of the instrument will convince the mind that what was really meant is that the delivery of the check was another consideration of the plaintiff's waiver, it being self-evident that said check constituted, in effect, a refund of the first premium paid by insured and received by the insurer. It is ridiculous to think that such a negligible amount has been the only consideration of the plaintiff's waiver of any right or benefit accurring to her from the policy. A careful perusal of the instrument will show that the real consideration of the plaintiff's waiver was the unenforceability of the policy due to her husband's illness and the mutual desire of the plaintiff of the insurer to settle amicably the cases instead of resorting to courts. In conclusion it is my opinion: (1) That the policy has not produced any effect from which the plaintiff may derive any right, and (2) that she has expressly waived any all rights accurring from the policy; and for these reasons I dissent from the majority opinion.

G.R. No. L-15895

November 29, 1920

RAFAEL ENRIQUEZ, as administrator of the estate of the late Joaquin Ma. Herrer, plaintiff-appellant, vs. SUN LIFE ASSURANCE COMPANY OF CANADA, defendant-appellee. Jose A. Espiritu for appellant. Cohn, Fisher and DeWitt for appellee.

MALCOLM, J.: This is an action brought by the plaintiff ad administrator of the estate of the late Joaquin Ma. Herrer to recover from the defendant life insurance company the sum of pesos 6,000 paid by the deceased for a life annuity. The trial court gave judgment for the defendant. Plaintiff appeals. The undisputed facts are these: On September 24, 1917, Joaquin Herrer made application to the Sun Life Assurance Company of Canada through its office in Manila for a life annuity. Two days later he paid the sum of P6,000 to the manager of the company's Manila office and was given a receipt reading as follows: MANILA, I. F., 26 de septiembre, 1917. PROVISIONAL RECEIPT Pesos 6,000 Recibi la suma de seis mil pesos de Don Joaquin Herrer de Manila como prima dela Renta Vitalicia solicitada por dicho Don Joaquin Herrer hoy, sujeta al examen medico y aprobacion de la Oficina Central de la Compaia. The application was immediately forwarded to the head office of the company at Montreal, Canada. On November 26, 1917, the head office gave notice of acceptance by cable to Manila. (Whether on the same day the cable was received notice was sent by the Manila office of Herrer that the application had been accepted, is a disputed point, which will be discussed later.) On December 4, 1917, the policy was issued at Montreal. On December 18, 1917, attorney Aurelio A. Torres wrote to the Manila office of the company stating that Herrer desired to withdraw his application. The following day the local office replied to Mr. Torres, stating that the policy had been issued, and called

attention to the notification of November 26, 1917. This letter was received by Mr. Torres on the morning of December 21, 1917. Mr. Herrer died on December 20, 1917. As above suggested, the issue of fact raised by the evidence is whether Herrer received notice of acceptance of his application. To resolve this question, we propose to go directly to the evidence of record. The chief clerk of the Manila office of the Sun Life Assurance Company of Canada at the time of the trial testified that he prepared the letter introduced in evidence as Exhibit 3, of date November 26, 1917, and handed it to the local manager, Mr. E. E. White, for signature. The witness admitted on cross-examination that after preparing the letter and giving it to he manager, he new nothing of what became of it. The local manager, Mr. White, testified to having received the cablegram accepting the application of Mr. Herrer from the home office on November 26, 1917. He said that on the same day he signed a letter notifying Mr. Herrer of this acceptance. The witness further said that letters, after being signed, were sent to the chief clerk and placed on the mailing desk for transmission. The witness could not tell if the letter had every actually been placed in the mails. Mr. Tuason, who was the chief clerk, on November 26, 1917, was not called as a witness. For the defense, attorney Manuel Torres testified to having prepared the will of Joaquin Ma. Herrer, that on this occasion, Mr. Herrer mentioned his application for a life annuity, and that he said that the only document relating to the transaction in his possession was the provisional receipt. Rafael Enriquez, the administrator of the estate, testified that he had gone through the effects of the deceased and had found no letter of notification from the insurance company to Mr. Herrer. Our deduction from the evidence on this issue must be that the letter of November 26, 1917, notifying Mr. Herrer that his application had been accepted, was prepared and signed in the local office of the insurance company, was placed in the ordinary channels for transmission, but as far as we know, was never actually mailed and thus was never received by the applicant. Not forgetting our conclusion of fact, it next becomes necessary to determine the law which should be applied to the facts. In order to reach our legal goal, the obvious signposts along the way must be noticed. Until quite recently, all of the provisions concerning life insurance in the Philippines were found in the Code of Commerce and the Civil Code. In the Code of the Commerce, there formerly existed Title VIII of Book III and Section III of Title III of Book III, which dealt with insurance contracts. In the Civil Code there formerly existed and presumably still exist, Chapters II and IV, entitled insurance contracts and life annuities, respectively, of Title XII of Book IV. On the after July 1, 1915, there was, however, in force the Insurance Act. No. 2427. Chapter IV of this Act concerns life and health insurance. The Act expressly repealed Title VIII of Book II and Section III of Title III of Book III of the code of Commerce. The law of insurance is consequently now found in the Insurance Act and the Civil Code.

While, as just noticed, the Insurance Act deals with life insurance, it is silent as to the methods to be followed in order that there may be a contract of insurance. On the other hand, the Civil Code, in article 1802, not only describes a contact of life annuity markedly similar to the one we are considering, but in two other articles, gives strong clues as to the proper disposition of the case. For instance, article 16 of the Civil Code provides that "In matters which are governed by special laws, any deficiency of the latter shall be supplied by the provisions of this Code." On the supposition, therefore, which is incontestable, that the special law on the subject of insurance is deficient in enunciating the principles governing acceptance, the subject-matter of the Civil code, if there be any, would be controlling. In the Civil Code is found article 1262 providing that "Consent is shown by the concurrence of offer and acceptance with respect to the thing and the consideration which are to constitute the contract. An acceptance made by letter shall not bind the person making the offer except from the time it came to his knowledge. The contract, in such case, is presumed to have been entered into at the place where the offer was made." This latter article is in opposition to the provisions of article 54 of the Code of Commerce. If no mistake has been made in announcing the successive steps by which we reach a conclusion, then the only duty remaining is for the court to apply the law as it is found. The legislature in its wisdom having enacted a new law on insurance, and expressly repealed the provisions in the Code of Commerce on the same subject, and having thus left a void in the commercial law, it would seem logical to make use of the only pertinent provision of law found in the Civil code, closely related to the chapter concerning life annuities. The Civil Code rule, that an acceptance made by letter shall bind the person making the offer only from the date it came to his knowledge, may not be the best expression of modern commercial usage. Still it must be admitted that its enforcement avoids uncertainty and tends to security. Not only this, but in order that the principle may not be taken too lightly, let it be noticed that it is identical with the principles announced by a considerable number of respectable courts in the United States. The courts who take this view have expressly held that an acceptance of an offer of insurance not actually or constructively communicated to the proposer does not make a contract. Only the mailing of acceptance, it has been said, completes the contract of insurance, as the locus poenitentiae is ended when the acceptance has passed beyond the control of the party. (I Joyce, The Law of Insurance, pp. 235, 244.) In resume, therefore, the law applicable to the case is found to be the second paragraph of article 1262 of the Civil Code providing that an acceptance made by letter shall not bind the person making the offer except from the time it came to his knowledge. The pertinent fact is, that according to the provisional receipt, three things had to be accomplished by the insurance company before there was a contract: (1) There had to be a medical examination of the applicant; (2) there had to be approval of the application by the head office of the company; and (3) this approval had in some way to be communicated by the company to the applicant. The further admitted facts are that the head office in Montreal did accept the application, did cable the Manila

office to that effect, did actually issue the policy and did, through its agent in Manila, actually write the letter of notification and place it in the usual channels for transmission to the addressee. The fact as to the letter of notification thus fails to concur with the essential elements of the general rule pertaining to the mailing and delivery of mail matter as announced by the American courts, namely, when a letter or other mail matter is addressed and mailed with postage prepaid there is a rebuttable presumption of fact that it was received by the addressee as soon as it could have been transmitted to him in the ordinary course of the mails. But if any one of these elemental facts fails to appear, it is fatal to the presumption. For instance, a letter will not be presumed to have been received by the addressee unless it is shown that it was deposited in the postoffice, properly addressed and stamped. (See 22 C.J., 96, and 49 L. R. A. [N. S.], pp. 458, et seq., notes.) We hold that the contract for a life annuity in the case at bar was not perfected because it has not been proved satisfactorily that the acceptance of the application ever came to the knowledge of the applicant.lawph!l.net Judgment is reversed, and the plaintiff shall have and recover from the defendant the sum of P6,000 with legal interest from November 20, 1918, until paid, without special finding as to costs in either instance. So ordered. Mapa, C.J., Araullo, Avancea and Villamor, JJ., concur. Johnson, J., dissents.

G.R. No. L-48563 May 25, 1979 VICENTE E. TANG, petitioner, vs. HON. COURT OF APPEALS and PHILIPPINE AMERICAN LIFE INSURANCE COMPANY, respondents. Ambrosio D. Go for petitioner. Ferry, De la Rosa, Deligero Salonga & Associates for private respondent.

ABAD SANTOS, J.: This is a petition to review on certiorari of the decision of the Court of Appeals (CA-G.R. No. 55407-R, June 8, 1978) which affirmed the decision of the Court of First Instance of Manila in Civil Case No. 90062 wherein the petitioner herein was the plaintiff and Philippine American Life Insurance Co. the herein respondent was the defendant. The

action was for the enforcement of two insurance policies that had been issued by the defendant company under the following circumstances. On September 25, 1965, Lee See Guat, a widow, 61 years old, and an illiterate who spoke only Chinese, applied for an insurance on her life for P60,000 with the respondent Company. The application consisted of two parts, both in the English language. The second part of her application dealt with her state of health and because her answers indicated that she was healthy, the Company issued her Policy No. 0690397, effective October 23, 1965, with her nephew Vicente E. Tang, herein Petitioner, as her beneficiary, On November 15, 1965, Lee See Guat again applied with the respondent Company for an additional insurance on her life for P40,000. Considering that her first application had just been approved, no further medical examination was made but she was required to accomplish and submit Part I of the application which reads: "I/WE HEREBY DECLARE AND AGREE that all questions, statements answers contained herein, as well as those made to or to be made to the Medical Examiner in Part II are full, complete and true and bind all parties in interest under the policy herein applied for; that there shall be no contract of insurance unless a policy is issued on this application and the fun first premium thereon, according to the mode of payment specified in answer to question 4D above, actually paid during the lifetime and good health of the Proposed Insured." Moreover, her answers in Part II of her previous application were used in appraising her insurability for the second insurance. On November 28, 1965, Policy No. 695632 was issued to Lee See Guat with the same Vicente E. Tang as her beneficiary. On April 20, 1966, Lee See Guat died of lung cancer. Thereafter, the beneficiary of the two policies, Vicente E. Tang claimed for their face value in the amount of P100,000 which the insurance company refused to pay on the ground that the insured was guilty of concealment and misrepresentation at the time she applied for the two policies. Hence, the filing of Civil Case No. 90062 in the Court of First Instance of Manila which dismissed the claim because of the concealment practised by the insured in violation of the Insurance Law. On appeal, the Court of Appeals, affirmed the decision. In its decision, the Court of Appeals stated, inter alia: "There is no doubt that she deliberately concealed material facts about her physical condition and history and/or conspired with whoever assisted her in relaying false information to the medical examiner, assuming that the examiner could not communicate directly with her." The issue in this appeal is the application of Art. 1332 of the Civil Code which stipulates: Art. 1332. When one of the parties is unable to read, or if the contract is in a language not understood by him, and mistake or fraud is alleged, the person enforcing the contract must show that the terms thereof have been fully explained to the former.

According to the Code Commission: "This rule is especially necessary in the Philippines where unfortunately there is still a fairly large number of illiterates, and where documents are usually drawn up in English or Spanish." (Report of the Code Commission, p. 136.) Art. 1332 supplements Art. 24 of the Civil Code which provides that " In all contractual, property or other relations, when one of the parties is at a disadvantage on account of his moral dependence, ignorance, indigence, mental weakness, tender age or other handicap, the court must be vigilant for his protection. It is the position of the petitioner that because Lee See Guat was illiterate and spoke only Chinese, she could not be held guilty of concealment of her health history because the applications for insurance were in English and the insurer has not proved that the terms thereof had been fully explained to her. It should be noted that under Art. 1332 above quoted, the obligation to show that the terms of the contract had been fully explained to the party who is unable to read or understand the language of the contract, when fraud or mistake is alleged, devolves on the party seeking to enforce it. Here the insurance company is not seeking to enforce the contracts; on the contrary, it is seeking to avoid their performance. It is petitioner who is seeking to enforce them even as fraud or mistake is not alleged. Accordingly, respondent company was under no obligation to prove that the terms of the insurance contracts were fully explained to the other party. Even if we were to say that the insurer is the one seeking the performance of the contracts by avoiding paying the claim, it has to be noted as above stated that there has been no imputation of mistake or fraud by the illiterate insured whose personality is represented by her beneficiary the petitioner herein. In sum, Art. 1332 is inapplicable to the case at bar. Considering the findings of both the CFI and Court of Appeals that the insured was guilty of concealment as to her state of health, we have to affirm. WHEREFORE, the decision of the Court of Appeals is hereby affirmed. No special pronouncement as to costs. SO ORDERED. Concepcion, Jr., and Santos, JJ., concur. Aquino, J., concurs in the result.

Separate Opinions

ANTONIO, J., concurring: I concur.

In a contract of insurance each party "must communicate to the other, in good faith, all facts within his knowledgewhich are material to the contract, and which the other has not the means of ascertaining ... (section 27, Act 2427, as amended. Emphasis supplied). As a general rule, a failure by the insured to disclose conditions affecting the risk, of which he is aware makes the contract voidable at the option of the insurer (45 C.J.S. 153). The reason for this rule is that insurance policies are traditionally contracts "ubemae fidei" which means most abundant good faith absolute and perfect candor or openness and honesty; the absence of any concealment or deception however slight. Here, the Court of Appeals found that the insured "deliberately concealed material facts about her physical condition and history and/or concealed with whoever assisted her in relaying false information to the medical examiner ... " Certainly, petitioner cannot assume inconsistent positions by attempting to enforce the contract of insurance for the purpose of collecting the proceeds of the policy and at the same time nullify the contract by claiming that he executed the same thru fraud or mistake.

# Separate Opinions ANTONIO, J., concurring: I concur. In a contract of insurance each party "must communicate to the other, in good faith, all facts within his knowledgewhich are material to the contract, and which the other has not the means of ascertaining ... (section 27, Act 2427, as amended. Emphasis supplied). As a general rule, a failure by the insured to disclose conditions affecting the risk, of which he is aware makes the contract voidable at the option of the insurer (45 C.J.S. 153). The reason for this rule is that insurance policies are traditionally contracts "ubemae fidei" which means most abundant good faith absolute and perfect candor or openness and honesty; the absence of any concealment or deception however slight. Here, the Court of Appeals found that the insured "deliberately concealed material facts about her physical condition and history and/or concealed with whoever assisted her in relaying false information to the medical examiner ... " Certainly, petitioner cannot assume inconsistent positions by attempting to enforce the contract of insurance for the purpose of collecting the proceeds of the policy and at the same time nullify the contract by claiming that he executed the same thru fraud or mistake.

G.R. No. 112329

January 28, 2000

VIRGINIA A. PEREZ, petitioner, vs. COURT OF APPEALS and BF LIFEMAN INSURANCE CORPORATION, respondents. YNARES-SANTIAGO, J.: A contract of insurance, like all other contracts, must be assented to by both parties, either in person or through their agents and so long as an application for insurance has not been either accepted or rejected, it is merely a proposal or an offer to make a contract. Petitioner Virginia A. Perez assails the decision of respondent Court of Appeals dated July 9, 1993 in CA-G.R. CV 35529 entitled, "BF Lifeman Insurance Corporations; Plaintiff-Appellant versus Virginia A. Perez. Defendant-Appellee," which declared Insurance Policy 056300 for P50,000.00 issued by private respondent corporation in favor of the deceased Primitivo B. Perez, null and void and rescinded, thereby reversing the decision rendered by the Regional Trial Court of Manila, Branch XVI. The facts of the case as summarized by respondent Court of Appeals are not in dispute. Primitivo B. Perez had been insured with the BF Lifeman Insurance Corporation since 1980 for P20,000.00. Sometime in October 1987, an agent of the insurance corporation, Rodolfo Lalog, visited Perez in Guinayangan, Quezon and convinced him to apply for additional insurance coverage of P50,000.00, to avail of the ongoing promotional discount of P400.00 if the premium were paid annually.1wphi1.nt On October 20, 1987, Primitivo B. Perez accomplished an application form for the additional insurance coverage of P50,000.00. On the same day, petitioner Virginia A. Perez, Primitivo's wife, paid P2,075.00 to Lalog. The receipt issued by Lalog indicated the amount received was a "deposit."1 Unfortunately, Lalog lost the application form accomplished by Perez and so on October 28, 1987, he asked the latter to fill up another application form.2 On November 1, 1987, Perez was made to undergo the required medical examination, which he passed.3 Pursuant to the established procedure of the company, Lalog forwarded the application for additional insurance of Perez, together with all its supporting papers, to the office of BF Lifeman Insurance Corporation at Gumaca, Quezon which office was supposed to forward the papers to the Manila office. On November 25, 1987, Perez died in an accident. He was riding in a banca which capsized during a storm. At the time of his death, his application papers for the additional insurance of P50,000.00 were still with the Gumaca office. Lalog testified that when he went to follow up the papers, he found them still in the Gumaca office and so he personally brought the papers to the Manila office of BF Lifeman Insurance

Corporation. It was only on November 27, 1987 that said papers were received in Manila. Without knowing that Perez died on November 25, 1987, BF Lifeman Insurance Corporation approved the application and issued the corresponding policy for the P50,000.00 on December 2, 1987.4 Petitioner Virginia Perez went to Manila to claim the benefits under the insurance policies of the deceased. She was paid P40,000.00 under the first insurance policy for P20,000.00 (double indemnity in case of accident) but the insurance company refused to pay the claim under the additional policy coverage of P50,000.00, the proceeds of which amount to P150,000.00 in view of a triple indemnity rider on the insurance policy. In its letter' of January 29, 1988 to Virginia A. Perez, the insurance company maintained that the insurance for P50,000.00 had not been perfected at the time of the death of Primitivo Perez. Consequently, the insurance company refunded the amount of P2,075.00 which Virginia Perez had paid. On September 21, 1990, private respondent BF Lifeman Insurance Corporation filed a complaint against Virginia A. Perez seeking the rescission and declaration of nullity of the insurance contract in question. Petitioner Virginia A. Perez, on the other hand, averred that the deceased had fulfilled all his prestations under the contract and all the elements of a valid contract are present. She then filed a counterclaim against private respondent for the collection of P150,000.00 as actual damages, P100,000.00 as exemplary damages, P30,000.00 as attorney's fees and P10,000.00 as expenses for litigation. On October 25, 1991, the trial court rendered a decision in favor of petitioner, the dispositive portion of which reads as follows: WHEREFORE PREMISES CONSIDERED, judgment is hereby rendered in favor of defendant Virginia A. Perez, ordering the plaintiff BF Lifeman Insurance Corporation to pay to her the face value of BF Lifeman Insurance Policy No. 056300, plus double indemnity under the SARDI or in the total amount of P150,000.00 (any refund made and/or premium deficiency to be deducted therefrom). SO ORDERED.5 The trial court, in ruling for petitioner, held that the premium for the additional insurance of P50,000.00 had been fully paid and even if the sum of P2,075.00 were to be considered merely as partial payment, the same does not affect the validity of the policy. The trial court further stated that the deceased had fully complied with the requirements of the insurance company. He paid, signed the application form and passed the medical examination. He should not be made to suffer the subsequent delay

in the transmittal of his application form to private respondent's head office since these were no longer within his control. The Court of Appeals, however, reversed the decision of the trial court saying that the insurance contract for P50,000.00 could not have been perfected since at the time that the policy was issued, Primitivo was already dead.6 Citing the provision in the application form signed by Primitivo which states that: . . . there shall be no contract of insurance unless and until a policy is issued on this application and that the policy shall not take effect until the first premium has been paid and the policy has been delivered to and accepted by me/us in person while I/we, am/are in good health the Court of Appeals held that the contract of insurance had to be assented to by both parties and so long as the application for insurance has not been either accepted or rejected, it is merely an offer or proposal to make a contract. Petitioner's motion for reconsideration having been denied by respondent court, the instant petition for certiorariwas filed on the ground that there was a consummated contract of insurance between the deceased and BF Lifeman Insurance Corporation and that the condition that the policy issued by the corporation be delivered and received by the applicant in good health, is potestative, being dependent upon the will of the insurance company, and is therefore null and void. The petition is bereft of merit. Insurance is a contract whereby, for a stipulated consideration, one party undertakes to compensate the other for loss on a specified subject by specified perils.7 A contract, on the other hand, is a meeting of the minds between two persons whereby one binds himself, with respect to the other to give something or to render some service.8Under Article 1318 of the Civil Code, there is no contract unless the following requisites concur: (1) Consent of the contracting parties; (2) Object certain which is the subject matter of the contract; (3) Cause of the obligation which is established. Consent must be manifested by the meeting of the offer and the acceptance upon the thing and the cause which are to constitute the contract. The offer must be certain and the acceptance absolute. When Primitivo filed an application for insurance, paid P2,075.00 and submitted the results of his medical examination, his application was subject to the acceptance of private respondent BF Lifeman Insurance Corporation. The perfection of the contract of

insurance between the deceased and respondent corporation was further conditioned upon compliance with the following requisites stated in the application form: there shall be no contract of insurance unless and until a policy is issued on this application and that the said policy shall not take effect until the premium has been paid and the policy delivered to and accepted by me/us in person while I/We, am/are in good health.9 The assent of private respondent BF Lifeman Insurance Corporation therefore was not given when it merely received the application form and all the requisite supporting papers of the applicant. Its assent was given when it issues a corresponding policy to the applicant. Under the abovementioned provision, it is only when the applicant pays the premium and receives and accepts the policy while he is in good health that the contract of insurance is deemed to have been perfected. It is not disputed, however, that when Primitivo died on November 25, 1987, his application papers for additional insurance coverage were still with the branch office of respondent corporation in Gumaca and it was only two days later, or on November 27, 1987, when Lalog personally delivered the application papers to the head office in Manila. Consequently, there was absolutely no way the acceptance of the application could have been communicated to the applicant for the latter to accept inasmuch as the applicant at the time was already dead. In the case of Enriquez vs. Sun Life Assurance Co. of Canada,10 recovery on the life insurance of the deceased was disallowed on the ground that the contract for annuity was not perfected since it had not been proved satisfactorily that the acceptance of the application ever reached the knowledge of the applicant. Petitioner insists that the condition imposed by respondent corporation that a policy must have been delivered to and accepted by the proposed insured in good health is potestative being dependent upon the will of the corporation and is therefore null and void. We do not agree. A potestative condition depends upon the exclusive will of one of the parties. For this reason, it is considered void. Article 1182 of the New Civil Code states: When the fulfillment of the condition depends upon the sole will the debtor, the conditional obligation shall be void. In the case at bar, the following conditions were imposed by the respondent company for the perfection of the contract of insurance: (a) a policy must have been issued; (b) the premiums paid; and

(c) the policy must have been delivered to and accepted by the applicant while he is in good health. The condition imposed by the corporation that the policy must have been delivered to and accepted by the applicant while he is in good health can hardly be considered as a potestative or facultative condition. On the contrary, the health of the applicant at the time of the delivery of the policy is beyond the control or will of the insurance company. Rather, the condition is a suspensive one whereby the acquisition of rights depends upon the happening of an event which constitutes the condition. In this case, the suspensive condition was the policy must have been delivered and accepted by the applicant while he is in good health. There was non-fulfillment of the condition, however, inasmuch as the applicant was already dead at the time the policy was issued. Hence, the non-fulfillment of the condition resulted in the non-perfection of the contract. As stated above, a contract of insurance, like other contracts, must be assented to by both parties either in person or by their agents. So long as an application for insurance has not been either accepted or rejected, it is merely an offer or proposal to make a contract. The contract, to be binding from the date of application, must have been a completed contract, one that leaves nothing to be done, nothing to be completed, nothing to be passed upon, or determined, before it shall take effect. There can be no contract of insurance unless the minds of the parties have met in agreement.11 Prescinding from the foregoing, respondent corporation cannot be held liable for gross negligence. It should be noted that an application is a mere offer which requires the overt act of the insurer for it to ripen into a contract. Delay in acting on the application does not constitute acceptance even though the insured has forwarded his first premium with his application. The corporation may not be penalized for the delay in the processing of the application papers. Moreover, while it may have taken some time for the application papers to reach the main office, in the case at bar, the same was acted upon less than a week after it was received. The processing of applications by respondent corporation normally takes two to three weeks, the longest being a month.12 In this case, however, the requisite medical examination was undergone by the deceased on November 1, 1987; the application papers were forwarded to the head office on November 27, 1987; and the policy was issued on December 2, 1987. Under these circumstances, we hold that the delay could not be deemed unreasonable so as to constitute gross negligence. A final note. It has not escaped our notice that the Court of Appeals declared Insurance Policy 056300 for P50,000.00 null and void and rescinded. The Court of Appeals corrected this in its Resolution of the motion for reconsideration filed by petitioner, thus: Anent the appearance of the word "rescinded" in the dispositive portion of the decision, to which defendant-appellee attaches undue significance and makes capital of, it is clear that the use of the words "and rescinded" is, as it is hereby declared, a superfluity. It is apparent from the context of the decision that the

insurance policy in question was found null and void, and did not have to be "rescinded".13 True, rescission presupposes the existence of a valid contract. A contract which is null and void is no contract at all and hence could not be the subject of rescission. WHEREFORE, the decision rendered by the Court of Appeals in CA-G.R. CV No. 35529 is AFFIRMED insofar as it declared Insurance Policy No. 056300 for P50,000.00 issued by BF Lifeman Insurance Corporation of no force and effect and hence null and void. No costs.1wphi1.nt SO ORDERED. Davide, Jr., C.J., Puno, Kapunan and Pardo, JJ., concur.

G.R. No. L-15774

November 29, 1920

PILAR C. DE LIM, plaintiff-appellant, vs. SUN LIFE ASSURANCE COMPANY OF CANADA, defendant-appellee. Sanz and Luzuriaga for appellant. Cohn and Fisher for appellee.

MALCOLM, J.:

This is an appeal by plaintiff from an order of the Court of First Instance of Zamboanga sustaining a demurrer to plaintiff's complaint upon the ground that it fails to state a cause of action. As the demurrer had the effect of admitting the material facts set forth in the complaint, the facts are those alleged by the plaintiff. On July 6, 1917, Luis Lim y Garcia of Zamboanga made application to the Sun Life Assurance Company of Canada for a policy of insurance on his life in the sum of P5,000. In his application Lim designated his wife, Pilar C. de Lim, the plaintiff herein, as the beneficiary. The first premium of P433 was paid by Lim, and upon such payment the company issued what was called a "provisional policy." Luis Lim y Garcia died on August 23, 1917, after the issuance of the provisional policy but before approval of the application by the home office of the insurance company. The instant action is brought by the beneficiary, Pilar C. de Lim, to recover from the Sun Life Assurance Company of Canada the sum of P5,000, the amount named in the provisional policy. The "provisional policy" upon which this action rests reads as follows: Received (subject to the following stipulations and agreements) the sum of four hundred and thirty-three pesos, being the amount of the first year's premium for a Life Assurance Policy on the life of Mr. Luis D. Lim y Garcia of Zamboanga for P5,000, for which an application dated the 6th day of July, 1917, has been made to the Sun Life Assurance Company of Canada. The above-mentioned life is to be assured in accordance with the terms and conditions contained or inserted by the Company in the policy which may be granted by it in this particular case for four months only from the date of the application, provided that the Company shall confirm this agreement by issuing a policy on said application when the same shall be submitted to the Head Office in Montreal. Should the Company not issue such a policy, then this agreement shall be null and void ab initio, and the Company shall be held not to have been on the risk at all, but in such case the amount herein acknowledged shall be returned. [SEAL.] (Sgd.) T. B. MACAULAY, President. (Sgd.) A. F. Peters, Agent.

Our duty in this case is to ascertain the correct meaning of the document above quoted. A perusal of the same many times by the writer and by other members of the court leaves a decided impression of vagueness in the mind. Apparently it is to be a provisional policy "for four months only from the date of this application." We use the term "apparently" advisedly, because immediately following the words fixing the four months period comes the word "provided" which has the meaning of "if." Otherwise stated, the policy for four months is expressly made subjected to the affirmative condition that "the company shall confirm this agreement by issuing a policy on said application when the same shall be submitted to the head office in Montreal." To reenforce the same there follows the negative condition

Should the company not issue such a policy, then this agreement shall be null and void ab initio, and the company shall be held not to have been on the risk." Certainly, language could hardly be used which would more clearly stipulate that the agreement should not go into effect until the home office of the company should confirm it by issuing a policy. As we read and understand the so-called provisional policy it amounts to nothing but an acknowledgment on behalf of the company, that it has received from the person named therein the sum of money agreed upon as the first year's premium upon a policy to be issued upon the application, if the application is accepted by the company. It is of course a primary rule that a contract of insurance, like other contracts, must be assented to by both parties either in person or by their agents. So long as an application for insurance has not been either accepted or rejected, it is merely an offer or proposal to make a contract. The contract, to be binding from the date of the application, must have been a completed contract, one that leaves nothing to be done, nothing to be completed, nothing to be passed upon, or determined, before it shall take effect. There can be no contract of insurance unless the minds of the parties have met in agreement. Our view is, that a contract of insurance was not here consummated by the parties.lawph!l.net Appellant relies on Joyce on Insurance. Beginning at page 253, of Volume I, Joyce states the general rule concerning the agent's receipt pending approval or issuance of policy. The first rule which Joyce lays down is this: If the act of acceptance of the risk by the agent and the giving by him of a receipt, is within the scope of the agent's authority, and nothing remains but to issue a policy, then the receipt will bind the company. This rule does not apply, for while here nothing remained but to issue the policy, this was made an express condition to the contract. The second rule laid down by Joyce is this: Where an agreement is made between the applicant and the agent whether by signing an application containing such condition, or otherwise, that no liability shall attach until the principal approves the risk and a receipt is given buy the agent, such acceptance is merely conditional, and it subordinated to the act of the company in approving or rejecting; so in life insurance a "binding slip" or "binding receipt" does not insure of itself. This is the rule which we believe applies to the instant case. The third rule announced by Joyce is this: Where the acceptance by the agent is within the scope of his authority a receipt containing a contract for insurance for a specific time which is not absolute but conditional, upon acceptance or rejection by the principal, covers the specified period unless the risk is declined within that period. The case cited by Joyce to substantiate the last principle is that a Goodfellow vs. Times & Beacon Assurance Com. (17 U. C. Q. B., 411), not available. The two cases most nearly in point come from the federal courts and the Supreme Court of Arkansas. In the case of Steinle vs. New York Life Insurance Co. ([1897], 81 Fed., 489} the facts were that the amount of the first premium had been paid to an insurance agent and a receipt given therefor. The receipt, however, expressly declared that if the

application was accepted by the company, the insurance shall take effect from the date of the application but that if the application was not accepted, the money shall be returned. The trite decision of the circuit court of appeal was, "On the conceded facts of this case, there was no contract to life insurance perfected and the judgment of the circuit court must be affirmed." In the case of Cooksey vs. Mutual Life Insurance Co. ([1904], 73 Ark., 117) the person applying for the life insurance paid and amount equal to the first premium, but the application and the receipt for the money paid, stipulated that the insurance was to become effective only when the application was approved and the policy issued. The court held that the transaction did not amount to an agreement for preliminary or temporary insurance. It was said: It is not an unfamiliar custom among life insurance companies in the operation of the business, upon receipt of an application for insurance, to enter into a contract with the applicant in the shape of a so-called "binding receipt" for temporary insurance pending the consideration of the application, to last until the policy be issued or the application rejected, and such contracts are upheld and enforced when the applicant dies before the issuance of a policy or final rejection of the application. It is held, too, that such contracts may rest in parol. Counsel for appellant insists that such a preliminary contract for temporary insurance was entered into in this instance, but we do not think so. On the contrary, the clause in the application and the receipt given by the solicitor, which are to be read together, stipulate expressly that the insurance shall become effective only when the "application shall be approved and the policy duly signed by the secretary at the head office of the company and issued." It constituted no agreement at all for preliminary or temporary insurance; Mohrstadt vs. Mutual Life Ins. Co., 115 Fed., 81, 52 C. C. A., 675; Steinle vs. New York Life Ins. Co., 81 Fed., 489, 26 C. C. A., 491." (See further Weinfeld vs. Mutual Reserve Fund Life Ass'n. [1892], 53 Fed, 208' Mohrstadt vs. Mutual Life Insurance Co. [1902], 115 Fed., 81; Insurance co. vs. Young's Administrator [1875], 90 U. S., 85; Chamberlain vs. Prudential Insurance Company of America [1901], 109 Wis., 4; Shawnee Mut. Fire Ins. Co. vs. McClure [1913], 39 Okla., 509; Dorman vs. Connecticut Fire Ins. Co. [1914], 51 contra, Starr vs. Mutual Life Ins. Co. [1905], 41 Wash., 228.) We are of the opinion that the trial court committed no error in sustaining the demurrer and dismissing the case. It is to be noted, however, that counsel for appellee admits the liability of the company for the return of the first premium to the estate of the deceased. It is not to be doubted but that the Sun Life Assurance Company of Canada will immediately, on the promulgation of this decision, pay to the estate of the late Luis Lim y Garcia the of P433. The order appealed from, in the nature of a final judgment is affirmed, without special finding as to costs in this instance. So ordered. Mapa, C.J., Johnson, Araullo, Avancea and Villamor, JJ., concur.

G.R. No. L-38613 February 25, 1982 PACIFIC TIMBER EXPORT CORPORATION, petitioner, vs. THE HONORABLE COURT OF APPEALS and WORKMEN'S INSURANCE COMPANY, INC., respondents.

DE CASTRO, ** J.: This petition seeks the review of the decision of the Court of Appeals reversing the decision of the Court of First Instance of Manila in favor of petitioner and against private respondent which ordered the latter to pay the sum of Pll,042.04 with interest at the rate of 12% interest from receipt of notice of loss on April 15, 1963 up to the complete payment, the sum of P3,000.00 as attorney's fees and the costs 1 thereby dismissing petitioner s complaint with costs. 2 The findings of the of fact of the Court of Appeals, which are generally binding upon this Court, Except as shall be indicated in the discussion of the opinion of this Court the substantial correctness of still particular finding having been disputed, thereby raising a question of law reviewable by this Court 3 are as follows: March 19, l963, the plaintiff secured temporary insurance from the defendant for its exportation of 1,250,000 board feet of Philippine Lauan and Apitong logs to be shipped from the Diapitan. Bay, Quezon Province to Okinawa and Tokyo, Japan. The defendant issued on said date Cover Note No. 1010, insuring the said cargo of the plaintiff "Subject to the Terms and Conditions of the WORKMEN'S INSURANCE COMPANY, INC. printed Marine Policy form as filed with and approved by the Office of the Insurance Commissioner (Exhibit A). The regular marine cargo policies were issued by the defendant in favor of the plaintiff on April 2, 1963. The two marine policies bore the numbers 53 HO 1032 and 53 HO 1033 (Exhibits B and C, respectively). Policy No. 53 H0 1033 (Exhibit B) was for 542 pieces of logs equivalent to 499,950

board feet. Policy No. 53 H0 1033 was for 853 pieces of logs equivalent to 695,548 board feet (Exhibit C). The total cargo insured under the two marine policies accordingly consisted of 1,395 logs, or the equivalent of 1,195.498 bd. ft. After the issuance of Cover Note No. 1010 (Exhibit A), but before the issuance of the two marine policies Nos. 53 HO 1032 and 53 HO 1033, some of the logs intended to be exported were lost during loading operations in the Diapitan Bay. The logs were to be loaded on the 'SS Woodlock' which docked about 500 meters from the shoreline of the Diapitan Bay. The logs were taken from the log pond of the plaintiff and from which they were towed in rafts to the vessel. At about 10:00 o'clock a. m. on March 29, 1963, while the logs were alongside the vessel, bad weather developed resulting in 75 pieces of logs which were rafted together co break loose from each other. 45 pieces of logs were salvaged, but 30 pieces were verified to have been lost or washed away as a result of the accident. In a letter dated April 4, 1963, the plaintiff informed the defendant about the loss of 'appropriately 32 pieces of log's during loading of the 'SS Woodlock'. The said letter (Exhibit F) reads as follows: April 4, 1963 Workmen's Insurance Company, Inc. Manila, Philippines Gentlemen: This has reference to Insurance Cover Note No. 1010 for shipment of 1,250,000 bd. ft. Philippine Lauan and Apitong Logs. We would like to inform you that we have received advance preliminary report from our Office in Diapitan, Quezon that we have lost approximately 32 pieces of logs during loading of the SS Woodlock. We will send you an accurate report all the details including values as soon as same will be reported to us. Thank you for your attention, we wish to remain. Very respectfully yours, PACIFIC TIMBER EXPORT CORPORATION (Sgd.) EMMANUEL S. ATILANO Asst. General Manager.

Although dated April 4, 1963, the letter was received in the office of the defendant only on April 15, 1963, as shown by the stamp impression appearing on the left bottom corner of said letter. The plaintiff subsequently submitted a 'Claim Statement demanding payment of the loss under Policies Nos. 53 HO 1032 and 53 HO 1033, in the total amount of P19,286.79 (Exhibit G). On July 17, 1963, the defendant requested the First Philippine Adjustment Corporation to inspect the loss and assess the damage. The adjustment company submitted its 'Report on August 23, 1963 (Exhibit H). In said report, the adjuster found that 'the loss of 30 pieces of logs is not covered by Policies Nos. 53 HO 1032 and 1033 inasmuch as said policies covered the actual number of logs loaded on board the 'SS Woodlock' However, the loss of 30 pieces of logs is within the 1,250,000 bd. ft. covered by Cover Note 1010 insured for $70,000.00. On September 14, 1963, the adjustment company submitted a computation of the defendant's probable liability on the loss sustained by the shipment, in the total amount of Pl1,042.04 (Exhibit 4). On January 13, 1964, the defendant wrote the plaintiff denying the latter's claim, on the ground they defendant's investigation revealed that the entire shipment of logs covered by the two marines policies No. 53 110 1032 and 713 HO 1033 were received in good order at their point of destination. It was further stated that the said loss may be considered as covered under Cover Note No. 1010 because the said Note had become 'null and void by virtue of the issuance of Marine Policy Nos. 53 HO 1032 and 1033'(Exhibit J-1). The denial of the claim by the defendant was brought by the plaintiff to the attention of the Insurance Commissioner by means of a letter dated March 21, 1964 (Exhibit K). In a reply letter dated March 30, 1964, Insurance Commissioner Francisco Y. Mandanas observed that 'it is only fair and equitable to indemnify the insured under Cover Note No. 1010', and advised early settlement of the said marine loss and salvage claim (Exhibit L). On June 26, 1964, the defendant informed the Insurance Commissioner that, on advice of their attorneys, the claim of the plaintiff is being denied on the ground that the cover note is null and void for lack of valuable consideration (Exhibit M). 4 Petitioner assigned as errors of the Court of Appeals, the following: I

THE COURT OF APPEALS ERRED IN HOLDING THAT THE COVER NOTE WAS NULL AND VOID FOR LACK OF VALUABLE CONSIDERATION BECAUSE THE COURT DISREGARDED THE PROVEN FACTS THAT PREMIUMS FOR THE COMPREHENSIVE INSURANCE COVERAGE THAT INCLUDED THE COVER NOTE WAS PAID BY PETITIONER AND THAT INCLUDED THE COVER NOTE WAS PAID BY PETITIONER AND THAT NO SEPARATE PREMIUMS ARE COLLECTED BY PRIVATE RESPONDENT ON ALL ITS COVER NOTES. II THE COURT OF APPEALS ERRED IN HOLDING THAT PRIVATE RESPONDENT WAS RELEASED FROM LIABILITY UNDER THE COVER NOTE DUE TO UNREASONABLE DELAY IN GIVING NOTICE OF LOSS BECAUSE THE COURT DISREGARDED THE PROVEN FACT THAT PRIVATE RESPONDENT DID NOT PROMPTLY AND SPECIFICALLY OBJECT TO THE CLAIM ON THE GROUND OF DELAY IN GIVING NOTICE OF LOSS AND, CONSEQUENTLY, OBJECTIONS ON THAT GROUND ARE WAIVED UNDER SECTION 84 OF THE INSURANCE ACT. 5 1. Petitioner contends that the Cover Note was issued with a consideration when, by express stipulation, the cover note is made subject to the terms and conditions of the marine policies, and the payment of premiums is one of the terms of the policies. From this undisputed fact, We uphold petitioner's submission that the Cover Note was not without consideration for which the respondent court held the Cover Note as null and void, and denied recovery therefrom. The fact that no separate premium was paid on the Cover Note before the loss insured against occurred, does not militate against the validity of petitioner's contention, for no such premium could have been paid, since by the nature of the Cover Note, it did not contain, as all Cover Notes do not contain particulars of the shipment that would serve as basis for the computation of the premiums. As a logical consequence, no separate premiums are intended or required to be paid on a Cover Note. This is a fact admitted by an official of respondent company, Juan Jose Camacho, in charge of issuing cover notes of the respondent company (p. 33, tsn, September 24, 1965). At any rate, it is not disputed that petitioner paid in full all the premiums as called for by the statement issued by private respondent after the issuance of the two regular marine insurance policies, thereby leaving no account unpaid by petitioner due on the insurance coverage, which must be deemed to include the Cover Note. If the Note is to be treated as a separate policy instead of integrating it to the regular policies subsequently issued, the purpose and function of the Cover Note would be set at naught or rendered meaningless, for it is in a real sense a contract, not a mere application for insurance which is a mere offer. 6

It may be true that the marine insurance policies issued were for logs no longer including those which had been lost during loading operations. This had to be so because the risk insured against is not for loss during operations anymore, but for loss during transit, the logs having already been safely placed aboard. This would make no difference, however, insofar as the liability on the cover note is concerned, for the number or volume of logs lost can be determined independently as in fact it had been so ascertained at the instance of private respondent itself when it sent its own adjuster to investigate and assess the loss, after the issuance of the marine insurance policies. The adjuster went as far as submitting his report to respondent, as well as its computation of respondent's liability on the insurance coverage. This coverage could not have been no other than what was stipulated in the Cover Note, for no loss or damage had to be assessed on the coverage arising from the marine insurance policies. For obvious reasons, it was not necessary to ask petitioner to pay premium on the Cover Note, for the loss insured against having already occurred, the more practical procedure is simply to deduct the premium from the amount due the petitioner on the Cover Note. The non-payment of premium on the Cover Note is, therefore, no cause for the petitioner to lose what is due it as if there had been payment of premium, for nonpayment by it was not chargeable against its fault. Had all the logs been lost during the loading operations, but after the issuance of the Cover Note, liability on the note would have already arisen even before payment of premium. This is how the cover note as a "binder" should legally operate otherwise, it would serve no practical purpose in the realm of commerce, and is supported by the doctrine that where a policy is delivered without requiring payment of the premium, the presumption is that a credit was intended and policy is valid. 7 2. The defense of delay as raised by private respondent in resisting the claim cannot be sustained. The law requires this ground of delay to be promptly and specifically asserted when a claim on the insurance agreement is made. The undisputed facts show that instead of invoking the ground of delay in objecting to petitioner's claim of recovery on the cover note, it took steps clearly indicative that this particular ground for objection to the claim was never in its mind. The nature of this specific ground for resisting a claim places the insurer on duty to inquire when the loss took place, so that it could determine whether delay would be a valid ground upon which to object to a claim against it. As already stated earlier, private respondent's reaction upon receipt of the notice of loss, which was on April 15, 1963, was to set in motion from July 1963 what would be necessary to determine the cause and extent of the loss, with a view to the payment thereof on the insurance agreement. Thus it sent its adjuster to investigate and assess the loss in July, 1963. The adjuster submitted his report on August 23, 1963 and its computation of respondent's liability on September 14, 1963. From April 1963 to July, 1963, enough time was available for private respondent to determine if petitioner was guilty of delay in communicating the loss to respondent company. In the proceedings that took place later in the Office of the Insurance Commissioner, private respondent should then have raised this ground of delay to avoid liability. It did not do so. It must be because it did not find any delay, as this Court fails to find a real and substantial sign

thereof. But even on the assumption that there was delay, this Court is satisfied and convinced that as expressly provided by law, waiver can successfully be raised against private respondent. Thus Section 84 of the Insurance Act provides: Section 84.Delay in the presentation to an insurer of notice or proof of loss is waived if caused by any act of his or if he omits to take objection promptly and specifically upon that ground. From what has been said, We find duly substantiated petitioner's assignments of error. ACCORDINGLY, the appealed decision is set aside and the decision of the Court of First Instance is reinstated in toto with the affirmance of this Court. No special pronouncement as to costs. SO ORDERED. Teehankee (Chairman), Makasiar, Fernandez Guerrero, Melencio-Herrera and Plana, JJ., concur.

G.R. No. L-20853

May 29, 1967

BONIFACIO BROS., INC., ET AL., plaintiffs-appellants, vs. ENRIQUE MORA, ET AL., defendants-appellees. G. Magsaysay for plaintiffs-appellants. Abad Santos and Pablo for defendant-appellee H. E. Reyes, Inc. J. P. Santilla and A. D. Hidalgo, Jr. for other defendant-appellee. CASTRO, J.: This is an appeal from the decision of the Court of First Instance of Manila, Branch XV, in civil case 48823, affirming the decision of the Municipal Court of Manila, declaring the H.S. Reyes, Inc. as having a better right than the Bonifacio Bros., Inc. and the Ayala Auto Parts Company, appellants herein, to the proceeds of motor insurance policy A0615, in the sum of P2,002.73, issued by the State Bonding & Insurance Co. Inc., and directing payment of the said amount to the H. Reyes, Inc. Enrique Mora, owner of Oldsmobile sedan model 1956, bearing plate No. QCmortgaged the same to the H.S. Reyes, Inc., with the condition that the former would insure the automobile with the latter as beneficiary. The automobile was thereafter insured on June 23, 1959 with the State Bonding & Insurance Co., Inc., and motor car insurance policy A-0615 was issued to Enrique Mora, the pertinent provisions of which read: 1. The Company (referring to the State Bonding & Insurance Co., Inc.) will, subject to the Limits of Liability, indemnify the Insured against loss of or damages to the Motor Vehicle and its accessories and spare parts whilst thereon; (a) by accidental collision or overturning or collision or overturning consequent upon mechanical breakdown or consequent upon wear and tear, xxx xxx xxx

2. At its own option the Company may pay in cash the amount of the loss or damage or may repair, reinstate, or replace the Motor Vehicle or any part thereof or its accessories or spare parts. The liability of the Company shall not exceed the value of the parts whichever is the less. The Insured's estimate of value stated in the schedule will be the maximum amount payable by the Company in respect of any claim for loss or damage.1wph1.t xxx xxx xxx

4. The Insured may authorize the repair of the Motor Vehicle necessitated by damage for which the Company may be liable under this Policy provided that: (a) The estimated cost of such repair does not exceed the Authorized Repair Limit, (b) A detailed estimate of the cost is forwarded to the Company without delay, subject to the condition that "Loss, if any is payable to H.S. Reyes, Inc.," by virtue of the fact that said Oldsmobile sedan was mortgaged in favor of the said H.S. Reyes, Inc. and that under a clause in said insurance policy, any loss was made payable to the H.S. Reyes, Inc. as Mortgagee; xxx xxx xxx

During the effectivity of the insurance contract, the car met with an accident. The insurance company then assigned the accident to the Bayne Adjustment Co. for investigation and appraisal of the damage. Enrique Mora, without the knowledge and consent of the H.S. Reyes, Inc., authorized the Bonifacio Bros. Inc. to furnish the labor and materials, some of which were supplied by the Ayala Auto Parts Co. For the cost of labor and materials, Enrique Mora was billed at P2,102.73 through the H.H. Bayne Adjustment Co. The insurance company after claiming a franchise in the amount of P100, drew a check in the amount of P2,002.73, as proceeds of the insurance policy, payable to the order of Enrique Mora or H.S. Reyes,. Inc., and entrusted the check to the H.H. Bayne Adjustment Co. for disposition and delivery to the proper party. In the meantime, the car was delivered to Enrique Mora without the consent of the H.S. Reyes, Inc., and without payment to the Bonifacio Bros. Inc. and the Ayala Auto Parts Co. of the cost of repairs and materials. Upon the theory that the insurance proceeds should be paid directly to them, the Bonifacio Bros. Inc. and the Ayala Auto Parts Co. filed on May 8, 1961 a complaint with the Municipal Court of Manila against Enrique Mora and the State Bonding & Insurance Co., Inc. for the collection of the sum of P2,002.73 The insurance company filed its answer with a counterclaim for interpleader, requiring the Bonifacio Bros. Inc. and the H.S. Reyes, Inc. to interplead in order to determine who has better right to the insurance proceeds in question. Enrique Mora was declared in default for failure to appear at the hearing, and evidence against him was received ex parte. However, the counsel for the Bonifacio Bros. Inc., Ayala Auto Parts Co. and State Bonding & Insurance Co. Inc. submitted a stipulation of facts, on the basis of which are Municipal Court rendered a decision declaring the H.S. Reyes, Inc. as having a better right to the disputed amount and ordering State Bonding & Insurance Co. Inc. to pay to the H. S. Reyes, Inc. the said sum of P2,002.73. From this decision, the appellants elevated the case to the Court of First Instance of Manila which the stipulation of facts was reproduced. On October 19, 1962 the latter court rendered a decision, affirming the decision of the Municipal Court. The Bonifacio Bros. Inc. and the Ayala Auto Parts Co. moved for reconsideration of the decision, but the trial court denied the motion. Hence, this appeal. The main issue raised is whether there is privity of contract between the Bonifacio Bros. Inc. and the Ayala Auto Parts Co. on the one hand and the insurance company on the other. The appellants argue that the insurance company and Enrique Mora are parties

to the repair of the car as well as the towage thereof performed. The authority for this assertion is to be found, it is alleged, in paragraph 4 of the insurance contract which provides that "the insured may authorize the repair of the Motor Vehicle necessitated by damage for which the company may be liable under the policy provided that (a) the estimated cost of such repair does not exceed the Authorized Repair Limit, and (b) a detailed estimate of the cost is forwarded to the company without delay." It is stressed that the H.H. Bayne Adjustment Company's recommendation of payment of the appellants' bill for materials and repairs for which the latter drew a check for P2,002.73 indicates that Mora and the H.H. Bayne Adjustment Co. acted for and in representation of the insurance company. This argument is, in our view, beside the point, because from the undisputed facts and from the pleadings it will be seen that the appellants' alleged cause of action rests exclusively upon the terms of the insurance contract. The appellants seek to recover the insurance proceeds, and for this purpose, they rely upon paragraph 4 of the insurance contract document executed by and between the State Bonding & Insurance Company, Inc. and Enrique Mora. The appellants are not mentioned in the contract as parties thereto nor is there any clause or provision thereof from which we can infer that there is an obligation on the part of the insurance company to pay the cost of repairs directly to them. It is fundamental that contracts take effect only between the parties thereto, except in some specific instances provided by law where the contract contains some stipulation in favor of a third person.1Such stipulation is known as stipulation pour autrui or a provision in favor of a third person not a pay to the contract. Under this doctrine, a third person is allowed to avail himself of a benefit granted to him by the terms of the contract, provided that the contracting parties have clearly and deliberately conferred a favor upon such person.2 Consequently, a third person not a party to the contract has no action against the parties thereto, and cannot generally demand the enforcement of the same.3 The question of whether a third person has an enforcible interest in a contract, must be settled by determining whether the contracting parties intended to tender him such an interest by deliberately inserting terms in their agreement with the avowed purpose of conferring a favor upon such third person. In this connection, this Court has laid down the rule that the fairest test to determine whether the interest of a third person in a contract is a stipulation pour autrui or merely an incidental interest, is to rely upon the intention of the parties as disclosed by their contract.4 In the instant case the insurance contract does not contain any words or clauses to disclose an intent to give any benefit to any repairmen or materialmen in case of repair of the car in question. The parties to the insurance contract omitted such stipulation, which is a circumstance that supports the said conclusion. On the other hand, the "loss payable" clause of the insurance policy stipulates that "Loss, if any, is payable to H.S. Reyes, Inc." indicating that it was only the H.S. Reyes, Inc. which they intended to benefit. We likewise observe from the brief of the State Bonding & Insurance Company that it has vehemently opposed the assertion or pretension of the appellants that they are privy to the contract. If it were the intention of the insurance company to make itself liable to the repair shop or materialmen, it could have easily inserted in the contract a

stipulation to that effect. To hold now that the original parties to the insurance contract intended to confer upon the appellants the benefit claimed by them would require us to ignore the indespensable requisite that a stipulationpour autrui must be clearly expressed by the parties, which we cannot do. As regards paragraph 4 of the insurance contract, a perusal thereof would show that instead of establishing privity between the appellants and the insurance company, such stipulation merely establishes the procedure that the insured has to follow in order to be entitled to indemnity for repair. This paragraph therefore should not be construed as bringing into existence in favor of the appellants a right of action against the insurance company as such intention can never be inferred therefrom. Another cogent reason for not recognizing a right of action by the appellants against the insurance company is that "a policy of insurance is a distinct and independent contract between the insured and insurer, and third persons have no right either in a court of equity, or in a court of law, to the proceeds of it, unless there be some contract of trust, expressed or implied between the insured and third person." 5 In this case, no contract of trust, expressed or implied exists. We, therefore, agree with the trial court that no cause of action exists in favor of the appellants in so far as the proceeds of insurance are concerned. The appellants' claim, if at all, is merely equitable in nature and must be made effective through Enrique Mora who entered into a contract with the Bonifacio Bros. Inc. This conclusion is deducible not only from the principle governing the operation and effect of insurance contracts in general, but is clearly covered by the express provisions of section 50 of the Insurance Act which read: The insurance shall be applied exclusively to the proper interests of the person in whose name it is made unless otherwise specified in the policy. The policy in question has been so framed that "Loss, if any, is payable to H.S. Reyes, Inc.," which unmistakably shows the intention of the parties. The final contention of the appellants is that the right of the H.S. Reyes, Inc. to the insurance proceeds arises only if there was loss and not where there is mere damage as in the instant case. Suffice it to say that any attempt to draw a distinction between "loss" and "damage" is uncalled for, because the word "loss" in insurance law embraces injury or damage. Loss in insurance, defined. The injury or damage sustained by the insured in consequence of the happening of one or more of the accidents or misfortune against which the insurer, in consideration of the premium, has undertaken to indemnify the insured. (1 Bouv. Ins. No. 1215; Black's Law Dictionary; Cyclopedic Law Dictionary, cited in Martin's Phil. Commercial Laws, Vol. 1, 1961 ed. p. 608). Indeed, according to sec. 120 of the Insurance Act, a loss may be either total or partial. Accordingly, the judgment appealed from is hereby affirmed, at appellants' cost.

Concepcion, C.J., Reyes, J.B.L., Dizon, Regala, Makalintal, Bengzon, J.P., Zaldivar, Sanchez and Castro, JJ., concur.

G.R. No. L-23276

November 29, 1968

MELECIO COQUIA, MARIA ESPANUEVA and MANILA YELLOW TAXICAB CO., INC., plaintiffs-appellees, vs. FIELDMEN'S INSURANCE CO., INC., defendant-appellant. Antonio de Venecia for plaintiffs-appellees. Rufino Javier for defendant-appellant. CONCEPCION, C.J.: This is an appeal from a decision of the Court of First Instance of Manila, certified to us by the Court of Appeals, only questions of law being involved therein. Indeed, the

pertinent facts have been stipulated and/or, admitted by the parties at the hearing of the case in the trial court, to dispense with the presentation of evidence therein. It appears that on December 1, 1961, appellant Fieldmen's Insurance Company, Inc. hereinafter referred to as the Company issued, in favor of the Manila Yellow Taxicab Co., Inc. hereinafter referred to as the Insured a common carrier accident insurance policy, covering the period from December 1, 1961 to December 1, 1962. It was stipulated in said policy that: The Company will, subject to the Limits of Liability and under the Terms of this Policy, indemnify the Insured in the event of accident caused by or arising out of the use of Motor Vehicle against all sums which the Insured will become legally liable to pay in respect of: Death or bodily injury to any fare-paying passengerincluding the Driver, Conductor and/or Inspector who is riding in the Motor Vehicle insured at the time of accident or injury. 1 While the policy was in force, or on February 10, 1962, a taxicab of the Insured, driven by Carlito Coquia, met a vehicular accident at Mangaldan, Pangasinan, in consequence of which Carlito died. The Insured filed therefor a claim for P5,000.00 to which the Company replied with an offer to pay P2,000.00, by way of compromise. The Insured rejected the same and made a counter-offer for P4,000.00, but the Company did not accept it. Hence, on September 18, 1962, the Insured and Carlito's parents, namely, Melecio Coquia and Maria Espanueva hereinafter referred to as the Coquias filed a complaint against the Company to collect the proceeds of the aforementioned policy. In its answer, the Company admitted the existence thereof, but pleaded lack of cause of action on the part of the plaintiffs. After appropriate proceedings, the trial court rendered a decision sentencing the Company to pay to the plaintiffs the sum of P4,000.00 and the costs. Hence, this appeal by the Company, which contends that plaintiffs have no cause of action because: 1) the Coquias have no contractual relation with the Company; and 2) the Insured has not complied with the provisions of the policy concerning arbitration. As regards the first defense, it should be noted that, although, in general, only parties to a contract may bring an action based thereon, this rule is subject to exceptions, one of which is found in the second paragraph of Article 1311 of the Civil Code of the Philippines, reading: If a contract should contain some stipulation in favor of a third person, he may demand its fulfillment provided he communicated his acceptance to the obligor before its revocation. A mere incidental benefit or interest of a person is not sufficient. The contracting parties must have clearly and deliberately conferred a favor upon a third person.2 This is but the restatement of a well-known principle concerning contracts pour autrui, the enforcement of which may be demanded by a third party for whose benefit it was

made, although not a party to the contract, before the stipulation in his favor has been revoked by the contracting parties. Does the policy in question belong to such class of contracts pour autrui? In this connection, said policy provides, inter alia: Section I Liability to Passengers. 1. The Company will, subject to the Limits of Liability and under the Terms of this Policy, indemnify the Insured in the event of accident caused by or arising out of the use of Motor Vehicle against all sums which the Insured will become legally liable to pay in respect of: Death or bodily injury to any fare-paying passenger including the Driver ... who is riding in the Motor Vehicle insured at the time of accident or injury. Section II Liability to the Public xxx xxx xxx

3. In terms of and subject to the limitations of and for the purposes of this Section, the Company will indemnify any authorized Driver who is driving the Motor Vehicle.... Conditions xxx xxx xxx

7. In the event of death of any person entitled to indemnity under this Policy, the Company will, in respect of the liability incurred by such person, indemnify his personal representatives in terms of and subject to the limitations of this Policy, provided, that such representatives shall, as though they were the Insured, observe, fulfill and be subject to the Terms of this Policy insofar as they can apply. 8. The Company may, at its option, make indemnity payable directly to the claimants or heirs of claimants, with or without securing the consent of or prior notification to the Insured, it being the true intention of this Policy to protect, to the extent herein specified and subject always to the Terms Of this Policy, the liabilities of the Insured towards the passengers of the Motor Vehicle and the Public. Pursuant to these stipulations, the Company "will indemnify any authorized Driver who is driving the Motor Vehicle" of the Insured and, in the event of death of said driver, the Company shall, likewise, "indemnify his personal representatives." In fact, the Company "may, at its option, make indemnity payable directly to the claimants or heirs of claimants ... it being the true intention of this Policy to protect ... the liabilities of the Insured towards the passengers of the Motor Vehicle and the Public" in other words, third parties.

Thus, the policy under consideration is typical of contracts pour autrui, this character being made more manifest by the fact that the deceased driver paid fifty percent (50%) of the corresponding premiums, which were deducted from his weekly commissions. Under these conditions, it is clear that the Coquias who, admittedly, are the sole heirs of the deceased have a direct cause of action against the Company,3 and, since they could have maintained this action by themselves, without the assistance of the Insured, it goes without saying that they could and did properly join the latter in filing the complaint herein.4 The second defense set up by the Company is based upon Section 17 of the policy reading: If any difference or dispute shall arise with respect to the amount of the Company's liability under this Policy, the same shall be referred to the decision of a single arbitrator to be agreed upon by both parties or failing such agreement of a single arbitrator, to the decision of two arbitrators, one to be appointed in writing by each of the parties within one calendar month after having been required in writing so to do by either of the parties and in case of disagreement between the arbitrators, to the decision of an umpire who shall have been appointed in writing by the arbitrators before entering on the reference and the costs of and incident to the reference shall be dealt with in the Award. And it is hereby expressly stipulated and declared that it shall be a condition precedent to any right of action or suit upon this Policy that the award by such arbitrator, arbitrators or umpire of the amount of the Company's liability hereunder if disputed shall be first obtained. The record shows, however, that none of the parties to the contract invoked this section, or made any reference to arbitration, during the negotiations preceding the institution of the present case. In fact, counsel for both parties stipulated, in the trial court, that none of them had, at any time during said negotiations, even suggested the settlement of the issue between them by arbitration, as provided in said section. Their aforementioned acts or omissions had the effect of a waiver of their respective right to demand an arbitration. Thus, in Kahnweiler vs. Phenix Ins. Co. of Brooklyn,5 it was held: Another well-settled rule for interpretation of all contracts is that the court will lean to that interpretation of a contract which will make it reasonable and just. Bish. Cont. Sec. 400. Applying these rules to the tenth clause of this policy, its proper interpretation seems quite clear. When there is a difference between the company and the insured as to the amount of the loss the policy declares: "The same shall then be submitted to competent and impartial arbitrators, one to be selected by each party ...". It will be observed that the obligation to procure or demand an arbitration is not, by this clause, in terms imposed on either party. It is not said that either the company or the insured shall take the initiative in setting the arbitration on foot. The company has no more right to say the insured must do it than the insured has to say the company must do it. The contract in this respect is neither unilateral nor self-executing. To procure a reference to

arbitrators, the joint and concurrent action of both parties to the contract is indispensable. The right it gives and the obligation it creates to refer the differences between the parties to arbitrators are mutual. One party to the contract cannot bring about an arbitration. Each party is entitled to demand a reference, but neither can compel it, and neither has the right to insist that the other shall first demand it, and shall forfeit any right by not doing so. If the company demands it, and the insured refuses to arbitrate, his right of action is suspended until he consents to an arbitration; and if the insured demands an arbitration, and the company refuses to accede to the demand, the insured may maintain a suit on the policy, notwithstanding the language of the twelfth section of the policy, and, where neither party demands an arbitration, both parties thereby waive it.6 To the same effect was the decision of the Supreme Court of Minnesota in Independent School Dist. No. 35, St. Louis County vs. A. Hedenberg & Co., Inc.7 from which we quote: This rule is not new in our state. In Meyer v. Berlandi, 53 Minn. 59, 54 N.W. 937, decided in 1893, this court held that the parties to a construction contract, having proceeded throughout the entire course of their dealings with each other in entire disregard of the provision of the contract regarding the mode of determining by arbitration the value of the extras, thereby waived such provision. xxx xxx xxx

The test for determining whether there has been a waiver in a particular case is stated by the author of an exhaustive annotation in 117 A.L.R. p. 304, as follows: "Any conduct of the parties inconsistent with the notion that they treated the arbitration provision as in effect, or any conduct which might be reasonably construed as showing that they did not intend to avail themselves of such provision, may amount to a waiver thereof and estop the party charged with such conduct from claiming its benefits". xxx xxx xxx

The decisive facts here are that both parties from the inception of their dispute proceeded in entire disregard of the provisions of the contract relating to arbitration and that neither at any stage of such dispute, either before or after commencement of the action, demanded arbitration, either by oral or written demand, pleading, or otherwise. Their conduct was as effective a rejection of the right to arbitrate as if, in the best Coolidge tradition, they had said, "We do not choose to arbitrate". As arbitration under the express provisions of article 40 was "at the choice of either party," and was chosen by neither, a waiver by both of the right to arbitration followed as a matter of law.

WHEREFORE, the decision appealed from should be as it is hereby affirmed in toto, with costs against the herein defendant-appellant, Fieldmen's Insurance Co., Inc. It is so ordered. Reyes, J.B.L., Dizon, Makalintal, Zaldivar, Sanchez, Castro, Fernando and Capistrano, JJ., concur.

G.R. No. L-22042

August 17, 1967

DIONISIA, EULOGIO, MARINA, GUILLERMO and NORBERTO all surnamed GUINGON, plaintiffs-appellees, vs. ILUMINADO DEL MONTE, JULIO AGUILAR and CAPITAL INSURANCE and SURETY CO., INC., defendants. CAPITAL INSURANCE and SURETY CO., INC., defendant-appellant. Generoso Almario and Associates for plaintiffs-appellees. Achacoso and Associates for defendant-appellant. BENGZON, J.P., J.: Julio Aguilar owned and operated several jeepneys in the City of Manila among which was one with plate number PUJ-206-Manila, 1961. He entered into a contract with the Capital Insurance & Surety Co., Inc. insuring the operation of his jeepneys against accidents with third-party liability. As a consequence thereof an insurance policy was executed by the Capital Insurance & Surety Co., Inc., the pertinent provisions of which in so far as this case is concerned contains the following: Section II LIABILITY TO THE PUBLIC 1. The Company, will, subject to the limits of liability, indemnify the Insured in the event of accident caused by or arising out of the use of the Motor Vehicle/s or in connection with the loading or unloading of the Motor Vehicle/s, against all sums

including claimant's costs and expenses which the Insured shall become legally liable to pay in respect of: a. death of or bodily injury to any person b. damage to property During the effectivity of such insurance policy on February 20, 1961 Iluminado del Monte, one of the drivers of the jeepneys operated by Aguilar, while driving along the intersection of Juan Luna and Moro streets, City of Manila, bumped with the jeepney abovementioned one Gervacio Guingon who had just alighted from another jeepney and as a consequence the latter died some days thereafter. A corresponding information for homicide thru reckless imprudence was filed against Iluminado del Monte, who pleaded guilty. A penalty of four months imprisonment was imposed on him. As a corollary to such action, the heirs of Gervacio Guingon filed an action for damages praying that the sum of P82,771.80 be paid to them jointly and severally by the defendants, driver Iluminado del Monte, owner and operator Julio Aguilar, and the Capital Insurance & Surety Co., Inc. For failure to answer the complaint, Del Monte and Aguilar were declared in default. Capital Insurance & Surety Co., Inc. answered, alleging that the plaintiff has no cause of action against it. During the trial the following facts were stipulated: COURT: The Court wants to find if there is a stipulation in the policy whereby the insured is insured against liability to third persons who are not passengers of jeeps. ALMARIO: As far as I know, in my honest belief, there is no particularization as to the passengers, whether the passengers of the jeep insured or a passenger of another jeep or whether it is a pedestrian. With those, we can submit the stipulation. SIMBULAN: I admit that. (T.s.n., p. 21, Jan. 23, 1962; p. 65 Rec. on Appeal) On August 27, 1962, the Court of First Instance of Manila rendered its judgment with the following dispositive portion: WHEREFORE, judgment is rendered sentencing Iluminado del Monte and Julio Aguilar jointly and severally to pay plaintiffs the sum of P8,572.95 as damages for the death of their father, plus P1,000.00 for attorney's fees plus costs. The defendant Capital Insurance and Surety Co., Inc. is hereby sentenced to pay the plaintiffs the sum of Five Thousand (P5,000.00) Pesos plus Five Hundred (P500.00) Pesos as attorney's fees and costs. These sums of P5,000.00 and

P500.00 adjudged against Capital Insurance and Surety Co., Inc. shall be applied in partial satisfaction of the judgment rendered against Iluminado del Monte and Julio Aguilar in this case. SO ORDERED. The case was appealed to the Court of Appeals which appellate court on September 30, 1963 certified the case to Us because the appeal raises purely questions of law. The issues raised before Us in this appeal are (1) As the company agreed to indemnify the insured Julio Aguilar, is it only the insured to whom it is liable? (2) Must Julio Aguilar first show himself to be entitled to indemnity before the insurance company may be held liable for the same? (3) Plaintiffs not being parties to the insurance contract, do they have a cause of action against the company; and (4) Does the fact that the insured is liable to the plaintiffs necessarily mean that the insurer is liable to the insured? In the discussion of the points thus raised, what is paramount is the interpretation of the insurance contract with the aim in view of attaining the objectives for which the insurance was taken. The Rules of Court provide that parties may be joined either as plaintiffs or defendants, as the right to relief in respect to or arising out of the same transactions is alleged to exist (Sec. 6, Rule 3). The policy, on the other hand, contains a clause stating: E. Action Against Company No action shall lie against the Company unless, as a condition precedent thereto, the Insured shall have fully complied with all of the terms of this Policy, nor until the amount of the Insured's obligation to pay shall have been finally determined either by judgment against the Insured after actual trial or by written agreement of the Insured, the claimant, and the Company. Any person or organization or the legal representative thereof who has secured such judgment or written agreement shall thereafter be entitled to recover under this policy to the extent of the insurance afforded by the Policy. Nothing contained in this policy shall give any person or organization any right to join the Company as a co-defendant in any action against the Insured to determine the Insured's liability. Bankruptcy or insolvency of the Insured or of the Insured's estate shall not relieve the Company of any of its obligations hereunder. Appellant contends that the "no action" clause in the policy closes the avenue to any third party which may be injured in an accident wherein the jeepney of the insured might have been the cause of the injury of third persons, alleging the freedom of contracts. Will the mere fact that such clause was agreed upon by the parties in an insurance

policy prevail over the Rules of Court which authorizes the joining of parties plaintiffs or defendants? The foregoing issues raise two principal: questions: (1) Can plaintiffs sue the insurer at all? (2) If so, can plaintiffs sue the insurer jointly with the insured? The policy in the present case, as aforequoted, is one whereby the insurer agreed to indemnify the insured "against all sums . . . which the Insured shall become legally liable to pay in respect of: a. death of or bodily injury to any person . . . ." Clearly, therefore, it is one for indemnity against liability;1 from the fact then that the insured is liable to the third person, such third person is entitled to sue the insurer. 1wph1.t The right of the person injured to sue the insurer of the party at fault (insured), depends on whether the contract of insurance is intended to benefit third persons also or only the insured. And the test applied has been this: Where the contract provides for indemnity against liability to third persons, then third persons to whom the insured is liable, can sue the insurer. Where the contract is for indemnity against actual loss or payment, then third persons cannot proceed against the insurer, the contract being solely to reimburse the insured for liability actually discharged by him thru payment to third persons, said third persons' recourse being thus limited to the insured alone.2 The next question is on the right of the third person to sue the insurer jointly with the insured. The policy requires, as afore-stated, that suit and final judgment be first obtained against the insured; that only "thereafter" can the person injured recover on the policy; it expressly disallows suing the insurer as a co-defendant of the insured in a suit to determine the latter's liability. As adverted to before, the query is which procedure to follow that of the insurance policy or the Rules of Court. The "no action" clause in the policy of insurance cannot prevail over the Rules of Court provision aimed at avoiding multiplicity of suits. In a case squarely on the point, American Automobile Ins. Co. vs. Struwe, 218 SW 534 (Texas CCA), it was held that a "no action" clause in a policy of insurance cannot override procedural rules aimed at avoidance of multiplicity of suits. We quote: Appellants filed a plea in abatement on the grounds that the suit had been prematurely brought against the insurance company, and that it had been improperly joined with Zunker, as said insurance company, under the terms of the policy, was only liable after judgment had been awarded against Zunker. . . . * * * That plea was properly overruled, because under the laws of Texas a dual suit will always be avoided whenever all parties can have a fair trial when joined in one suit. Appellee, had he so desired, could have prosecuted his claim to judgment as against Zunker and then have sued on that judgment against the insurance company, but the law does not make it imperative that he should do so, but would permit him to dispose of the whole matter in one suit.

The rule has often been announced in Texas that when two causes of action are connected with each other, or grow out of the same transaction, they may be properly joined, and in such suit all parties against whom the plaintiff asserts a common or an alternative liability may be joined as defendants. . . . Even if appellants had presented any plea in abatement as to joinder of damages arising from a tort with those arising from a contract, it could not, under the facts of this case, be sustained, for the rule is that a suit may include an action for breach of contract and one for tort, provided they are connected with each other or grew out of the same transaction. Similarly, in the instant suit, Sec. 5 of Rule 2 on "Joinder of causes of action" and Sec. 6 of Rule 3 on "Permissive joinder of parties" cannot be superseded, at least with respect to third persons not a party to the contract, as herein, by a "no action" clause in the contract of insurance. Wherefore, the judgment appealed from is affirmed in toto. Costs against appellant. So ordered. Reyes, J.B.L., Makalintal, Zaldivar, Sanchez, Castro, Angeles and Fernando, JJ., concur. Concepcion, C.J. and Dizon, J., are on leave.

G.R. No. 128833 April 20, 1998 RIZAL COMMERCIAL BANKING CORPORATION, UY CHUN BING AND ELI D. LAO, petitioners, vs. COURT OF APPEALS and GOYU & SONS, INC., respondents. G.R. No. 128834 April 20, 1998 RIZAL COMMERCIAL BANKING CORPORATION, petitioners, vs. COURT OF APPEALS, ALFREDO C. SEBASTIAN, GOYU & SONS, INC., GO SONG HIAP, SPOUSES GO TENG KOK and BETTY CHIU SUK YING alias BETTY GO, respondents. G.R. No. 128866 April 20, 1998 MALAYAN INSURANCE INC., petitioners, vs. GOYU & SONS, INC. respondent. MELO, J.: The issue relevant to the herein three consolidated petitions revolve around the fire loss claims of respondent Goyu & Sons, Inc. (GOYU) with petitioner Malayan Insurance Company, Inc. (MICO) in connection with the mortgage contracts entered into by and between Rizal Commercial Banking Corporation (RCBC) and GOYU. The Court of Appeals ordered MICO to pay GOYU its claims in the total amount of P74,040,518.58, plus 37% interest per annum commending July 27, 1992. RCBC was ordered to pay actual and compensatory damages in the amount of P5,000,000.00. MICO and RCBC were held solidarily liable to pay GOYU P1,500,000.00 as exemplary damages and P1,500,000.00 for attorney's fees. GOYU's obligation to RCBC was fixed at P68,785,069.04 as of April 1992, without any interest, surcharges, and penalties. RCBC and MICO appealed separately but, in view of the common facts and issues involved, their individual petitions were consolidated. The undisputed facts may be summarized as follows: GOYU applied for credit facilities and accommodations with RCBC at its Binondo Branch. After due evaluation, RCBC Binondo Branch, through its key officers, petitioners Uy Chun Bing and Eli D. Lao, recommended GOYU's application for approval by RCBC's executive committee. A credit facility in the amount of P30 million was initially granted. Upon GOYU's application and Uy's and Lao's recommendation, RCBC's executive committee increased GOYU's credit facility to P50 million, then to P90 million, and finally to P117 million.

As security for its credit facilities with RCBC, GOYU executed two real estate mortgages and two chattel mortgages in favor of RCBC, which were registered with the Registry of Deeds at Valenzuela, Metro Manila. Under each of these four mortgage contracts, GOYU committed itself to insure the mortgaged property with an insurance company approved by RCBC, and subsequently, to endorse and deliver the insurance polices to RCBC. GOYU obtained in its name a total of ten insurance policies from MICO. In February 1992, Alchester Insurance Agency, Inc., the insurance agent where GOYU obtained the Malayan insurance policies, issued nine endorsements in favor of RCBC seemingly upon instructions of GOYU (Exhibits "1-Malayan" to "9-Malayan"). On April 27, 1992, one of GOYU's factory buildings in Valenzuela was gutted by fire. Consequently, GOYU submitted its claim for indemnity on account of the loss insured against. MICO denied the claim on the ground that the insurance policies were either attached pursuant to writs of attachments/garnishments issued by various courts or that the insurance proceeds were also claimed by other creditors of GOYU alleging better rights to the proceeds than the insured. GOYU filed a complaint for specific performance and damages which was docketed at the Regional Trial Court of the National Capital Judicial Region (Manila, Branch 3) as Civil Case No. 93-65442, now subject of the present G.R. No. 128833 and 128866. RCBC, one of GOYU's creditors, also filed with MICO its formal claim over the proceeds of the insurance policies, but said claims were also denied for the same reasons that MICO denied GOYU's claims. In an interlocutory order dated October 12, 1993 (Record, pp. 311-312), the Regional Trial Court of Manila (Branch 3), confirmed that GOYU's other creditors, namely, Urban Bank, Alfredo Sebastian, and Philippine Trust Company obtained their respective writs of attachments from various courts, covering an aggregate amount of P14,938,080.23, and ordered that the proceeds of the ten insurance policies be deposited with the said court minus the aforementioned P14,938,080.23. Accordingly, on January 7, 1994, MICO deposited the amount of P50,505,594.60 with Branch 3 of the Manila RTC. In the meantime, another notice of garnishment was handed down by another Manila RTC sala (Branch 28) for the amount of P8,696,838.75 (Exhibit "22-Malayan"). After trial, Branch 3 of the Manila RTC rendered judgment in favor of GOYU, disposing: WHEREFORE, judgment is hereby rendered in favor of the plaintiff and against the defendant, Malayan Insurance Company, Inc. and Rizal Commercial Banking Corporation, ordering the latter as follows: 1. For defendant Malayan Insurance Co., Inc.:

a. To pay the plaintiff its fire loss claims in the total amount of P74,040,518.58 less the amount of P50,000,000.00 which is deposited with this Court; b. To pay the plaintiff damages by was of interest for the duration of the delay since July 27, 1992 (ninety days after defendant insurer's receipt of the required proof of loss and notice of loss) at the rate of twice the ceiling prescribed by the Monetary Board, on the following amounts: 1) P50,000,000.00 from July 27, 1992 up to the time said amount was deposited with this Court on January 7, 1994; 2) P24,040,518.58 from July 27, 1992 up to the time when the writs of attachments were received by defendant Malayan; 2. For defendant Rizal Commercial Banking Corporation: a. To pay the plaintiff actual and compensatory damages in the amount of P2,000,000.00; 3. For both defendants Malayan and RCBC: a. To pay the plaintiff, jointly and severally, the following amounts: 1) P1,000,000.00 as exemplary damages; 2) P1,000,000.00 as, and for, attorney's fees; 3) Costs of suit. and on the Counterclaim of defendant RCBC, ordering the plaintiff to pay its loan obligations with defendant RCBC in the amount of P68,785,069.04, as of April 27, 1992, with interest thereon at the rate stipulated in the respective

promissory notes (without surcharges and penalties) per computation, pp. 14-A, 14-B & 14-C. FURTHER, the Clerk of Court of the Regional Trial Court of Manila is hereby ordered to release immediately to the plaintiff the amount of P50,000,000.00 deposited with the Court by defendant Malayan, together with all the interest earned thereon. (Record, pp. 478-479.) From this judgment, all parties interposed their respective appeals. GOYU was unsatisfied with the amount awarded in its favor. MICO and RCBC disputed the trial court's findings of liability on their part. The Court of Appeals party granted GOYU's appeal, but sustained the findings of the trial court with respect to MICO and RCBC's liabilities, thusly: WHEREFORE, the decision of the lower court dated June 29, 1994 is hereby modified as follows: 1. FOR DEFENDANT MALAYAN INSURANCE CO., INC: a) To pay the plaintiff its fire loss claim in the total amount of P74,040,518.58 less the amount of P50,505,594.60 (per O.R. No. 3649285) plus deposited in court and damages by way of interest commencing July 27, 1992 until the time Goyu receives the said amount at the rate of thirty-seven (37%) percent per annum which is twice the ceiling prescribed by the Monetary Board. 2. FOR DEFENDANT RIZAL COMMERCIAL BANKING CORPORATION; a) To pay the plaintiff actual and compensatory damages in the amount of P5,000,000.00. 3. FOR DEFENDANTS MALAYAN INSURANCE CO., INC., RIZAL COMMERCIAL BANKING CORPORATION, UY CHUN BING AND ELI D. LAO: a) To pay the plaintiff jointly and severally the following amounts: 1. P1,500,000.00 as exemplary damages;

2. P1,500,000.00 as and for attorney's fees. 4. And on RCBC's Counterclaim, ordering the plaintiff Goyu & Sons, Inc. to pay its loan obligation with RCBC in the amount of P68,785,069.04 as of April 27, 1992 without any interest, surcharges and penalties. The Clerk of the Court of the Regional Trial Court of Manila is hereby ordered to immediately release to Goyu & Sons, Inc. the amount of P50,505,594.60 (per O.R. No. 3649285) deposited with it by Malayan Insurance Co., Inc., together with all the interests thereon. (Rollo, p. 200.) RCBC and MICO are now before us in G.R. No. 128833 and 128866, respectively, seeking review and consequent reversal of the above dispositions of the Court of Appeals. In G.R. No. 128834, RCBC likewise appeals from the decision in C.A. G.R. No. CV48376, which case, by virtue of the Court of Appeals' resolution dated August 7, 1996, was consolidated with C.A. G.R. No. CV-46162 (subject of herein G.R. No. 128833). At issue in said petition is RCBC's right to intervene in the action between Alfredo C. Sebastian (the creditor) and GOYU (the debtor), where the subject insurance policies were attached in favor of Sebastian. After a careful reviews of the material facts as found by the two courts below in relation to the pertinent and applicable laws, we find merit in the submission of RCBC and MICO. The several causes of action pursued below by GOYU gave rise to several related issues which are now submitted in the petitions before us. This Court, however, discerns one primary and central issue, and this is, whether or not RCBC, as mortgagee, has any right over the insurance policies taken by GOYU, the mortgagor, in case of the occurrence of loss. As earlier mentioned, accordant with the credit facilities extended by RCBC to GOYU, the latter executed several mortgage contracts in favor of RCBC. It was expressly stipulated in these mortgage contracts that GOYU shall insure the mortgaged property with any of the insurance companies acceptable to RCBC. GOYU indeed insured the mortgaged property with MICO, an insurance company acceptable to RCBC. Bases on their stipulations in the mortgage contracts, GOYU was supposed to endorse these insurance policies in favor of, and deliver them, to RCBC. Alchester Insurance Agency, Inc., MICO's underwriter from whom GOYU obtained the subject insurance policies, prepared the nine endorsements (see Exh. "1-Malayan" to "9-Malayan"; also Exh. "51RCBC" to "59-RCBC"), copies of which were delivered to GOYU, RCBC, and MICO. However, because these endorsements do not bear the signature of any officer of

GOYU, the trial court, as well as the Court of Appeals, concluded that the endorsements are defective. We do not quite agree. It is settled that a mortgagor and a mortgagee have separated and distinct insurable interests in the same mortgaged property, such that each one of them may insure the same property for his own sole benefit. There is no question that GOYU could insure the mortgaged property for its own exclusive benefit. In the present case, although it appears that GOYU obtained the subject insurance policies naming itself as the sole payee, the intentions of the parties as shown by their contemporaneous acts, must be given due consideration in order to better serve the interest of justice and equity. It is to be noted that nine endorsement documents were prepared by Alchester in favor of RCBC. The Court is in a quandary how Alchester could arrive at the idea of endorsing any specific insurance policy in favor of any particular beneficiary or payee other than the insured had not such named payee or beneficiary been specifically disclosed by the insured itself. It is also significant that GOYU voluntarily and purposely took the insurance policies from MICO, a sister company of RCBC, and not just from any other insurance company. Alchester would not have found out that the subject pieces of property were mortgaged to RCBC had not such information been voluntarily disclosed by GOYU itself. Had it not been for GOYU, Alchester would not have known of GOYU's intention of obtaining insurance coverage in compliance with its undertaking in the mortgage contracts with RCBC, and verily, Alchester would not have endorsed the policies to RCBC had it not been so directed by GOYU. On equitable principles, particularly on the ground of estoppel, the Court is constrained to rule in favor of mortgagor RCBC. The basis and purpose of the doctrine was explained in Philippine National Bank vs. Court of Appeals (94 SCRA 357 [1979]), to wit: The doctrine of estoppel is based upon the grounds of public, policy, fair dealing, good faith and justice, and its purpose is to forbid one to speak against his own act, representations, or commitments to the injury of one to whom they were directed and who reasonably relied thereon. The doctrine of estoppel springs from equitable principles and the equities in the case. It is designed to aid the law in the administration of justice where without its aid injustice might result. It has been applied by this Court wherever and whenever special circumstances of a case so demand. (p. 368.) Evelyn Lozada of Alchester testified that upon instructions of Mr. Go, through a certain Mr. Yam, she prepared in quadruplicate on February 11, 1992 the nine endorsement documents for GOYU's nine insurance policies in favor of RCBC. The original copies of each of these nine endorsement documents were sent to GOYU, and the others were

sent to RCBC and MICO, while the fourth copies were detained for Alchester's file (tsn, February 23, pp. 7-8). GOYU has not denied having received from Alchester the originals of these documents. RCBC, in good faith, relied upon the endorsement documents sent to it as this was only pursuant to the stipulation in the mortgage contracts. We find such reliance to be justified under the circumstances of the case. GOYU failed to seasonably repudiate the authority of the person or persons who prepared such endorsements. Over and above this, GOYU continued, in the meantime, to enjoy the benefits of the credit facilities extended to it by RCBC. After the occurrence of the loss insure against, it was too late for GOYU to disown the endorsements for any imagined or contrived lack of authority of Alchester to prepare and issue said endorsements. If there had not been actually an implied ratification of said endorsements by virtue of GOYU's inaction in this case, GOYU is at the very least estopped from assailing their operative effects. To permit GOYU to capitalize on its non-confirmation of these endorsements while it continued to enjoy the benefits of the credit facilities of RCBC which believed in good faith that there was due endorsement pursuant to their mortgage contracts, is to countenance grave contravention of public policy, fair dealing, good faith, and justice. Such an unjust situation, the Court cannot sanction. Under the peculiar circumstances obtaining in this case, the Court is bound to recognize RCBC's right to the proceeds of the insurance polices if not for the actual endorsement of the policies, at least on the basis of the equitable principle of estoppel. GOYU cannot seek relief under Section 53 of the Insurance Code which provides that the proceeds of insurance shall exclusively apply to the interest of the person in whose name or for whose benefit it is made. The peculiarity of the circumstances obtaining in the instant case presents a justification to take exception to the strict application of said provision, it having been sufficiently established that it was the intention of the parties to designate RCBC as the party for whose benefit the insurance policies were taken out. Consider thus the following: 1. It is undisputed that the insured pieces of property were the subject of mortgage contracts entered into between RCBC and GOYU in consideration of and for securing GOYU's credit facilities from RCBC. The mortgage contracts contained common provisions whereby GOYU, as mortgagor, undertook to have the mortgaged property properly covered against any loss by an insurance company acceptable to RCBC. 2. GOYU voluntarily procured insurance policies to cover the mortgaged property from MICO, no less than a sister company of RCBC and definitely an acceptable insurance company to RCBC. 3. Endorsement documents were prepared by MICO's underwriter, Alchester Insurance Agency, Inc., and copies thereof were sent to GOYU, MICO, and RCBC. GOYU did not assail, until of late, the validity of said endorsements.

4. GOYU continued until the occurrence of the fire, to enjoy the benefits of the credit facilities extended by RCBC which was conditioned upon the endorsement of the insurance policies to be taken by GOYU to cover the mortgaged properties. This Court can not over stress the fact that upon receiving its copies of the endorsement documents prepared by Alchester, GOYU, despite the absence of its written conformity thereto, obviously considered said endorsement to be sufficient compliance with its obligation under the mortgage contracts since RCBC accordingly continued to extend the benefits of its credits facilities and GOYU continued to benefit therefrom. Just as plain too is the intention of the parties to constitute RCBC as the beneficiary of the various insurance policies obtained by GOYU. The intention of the parties will have to be given full force and effect particular case. The insurance proceeds may, therefore, be exclusively applied to RCBC, which under the factual circumstances of the case, is truly the person or entity for whose benefit the polices were clearly intended. Moreover, the law's evident intention to protect the interests of the mortgage upon the mortgaged property is expressed in Article 2127 of the Civil Code which states: Art. 2127. The mortgage extends to the natural accessions, to the improvements, growing fruits, and the rents or income not yet received when the obligation becomes due, and to the amount of the indemnity granted or owing to the proprietor from the insurers of the property mortgaged, or in virtue of expropriation for public use, with the declarations, amplifications and limitations established by law, whether the estate remains in the possession of the mortgagor, or it passes into the hands of a third person. Significantly, the Court notes that out of the 10 insurance policies subject of this case, only 8 of them appear to have been subject of the endorsements prepared and delivered by Alchester for and upon instructions of GOYU as shown below: INSURANCE POLICY PARTICULARS ENDORSEMENT a. Policy Number F-114-07795 None Issue Date March 18, 1992 Expiry Date April 5, 1993 Amount P9,646,224.92 b. Policy Number ACIA/F-174-07660 Exhibit "1-Malayan" Issue Date January 18, 1992 Expiry Date February 9, 1993 Amount P4,307,217.54 c. Policy Number ACIA/F-114-07661 Exhibit "2-Malayan" Issue Date January 18, 1992

Expiry Date February 15, 1993 Amount P6,603,586.43 d. Policy Number ACIA/F-114-07662 Exhibit "3-Malayan" Issue Date January 18, 1992 Expiry Date (not legible) Amount P6,603,586.43 e. Policy Number ACIA/F-114-07663 Exhibit "4-Malayan" Issue Date January 18, 1992 Expiry Date February 9, 1993 Amount P9,457,972.76 f. Policy Number ACIA/F-114-07623 Exhibit "7-Malayan" Issue Date January 13, 1992 Expiry Date January 13, 1993 Amount P24,750,000.00 g. Policy Number ACIA/F-174-07223 Exhibit "6-Malayan" Issue Date May 29, 1991 Expiry Date June 27, 1992 Amount P6,000,000.00 h. Policy Number CI/F-128-03341 None Issue Date May 3, 1991 Expiry Date May 3, 1992 Amount P10,000,000.00 i. Policy Number F-114-07402 Exhibit "8-Malayan" Issue Date September 16, 1991 Expiry Date October 19, 1992 Amount P32,252,125.20 j. Policy Number F-114-07525 Exhibit "9-Malayan" Issue Date November 20, 1991 Expiry Date December 5, 1992 Amount P6,603,586.43 (pp. 456-457, Record; Folder of Exhibits for MICO.) Policy Number F-114-07795 [(a) above] has not been endorsed. This fact was admitted by MICO's witness, Atty. Farolan (tsn, February 16, 1994, p. 25). Likewise, the record shows no endorsement for Policy Number CI/F-128-03341 [(h) above]. Also, one of the endorsement documents, Exhibit "5-Malayan", refers to a certain insurance policy number ACIA-F-07066, which is not among the insurance policies involved in the complaint.

The proceeds of the 8 insurance policies endorsed to RCBC aggregate to P89,974,488.36. Being excessively payable to RCBC by reason of the endorsement by Alchester to RCBC, which we already ruled to have the force and effect of an endorsement by GOYU itself, these 8 policies can not be attached by GOYU's other creditors up to the extent of the GOYU's outstanding obligation in RCBC's favor. Section 53 of the Insurance Code ordains that the insurance proceeds of the endorsed policies shall be applied exclusively to the proper interest of the person for whose benefit it was made. In this case, to the extent of GOYU's obligation with RCBC, the interest of GOYU in the subject policies had been transferred to RCBC effective as of the time of the endorsement. These policies may no longer be attached by the other creditors of GOYU, like Alfredo Sebastian in the present G.R. No. 128834, which may nonetheless forthwith be dismissed for being moot and academic in view of the results reached herein. Only the two other policies amounting to P19,646,224.92 may be validly attached, garnished, and levied upon by GOYU's other creditors. To the extent of GOYU's outstanding obligation with RCBC, all the rest of the other insurance policies above-listed which were endorsed to RCBC, are, therefore, to be released from attachment, garnishment, and levy by the other creditors of GOYU. This brings us to the next issue to be resolved, which is, the extent of GOYU's outstanding obligation with RCBC which the proceeds of the 8 insurance policies will discharge and liquidate, or put differently, the actual amount of GOYU's liability to RCBC. The Court of Appeals simply echoed the declaration of the trial court finding that GOYU's total obligation to RCBC was only P68,785,060.04 as of April 27, 1992, thus sanctioning the trial court's exclusion of Promissory Note No. 421-92 (renewal of Promissory Note No. 908-91) and Promissory Note No. 420-92 (renewal of Promissory Note No. 952-91) on the ground that their execution is highly questionable for not only are these dated after the fire, but also because the signatures of either GOYU or any its representative are conspicuously absent. Accordingly, the Court of Appeals speculated thusly: . . . Hence, this Court is inclined to conclude that said promissory notes were pre-signed by plaintiff in bank terms, as averred by plaintiff, in contemplation of the speedy grant of future loans, for the same practice of procedure has always been adopted in its previous dealings with the bank. (Rollo, pp. 181-182.) The fact that the promissory notes bear dates posterior to the fire does not necessarily mean that the documents are spurious, for it is presumed that the ordinary course of business had been followed (Metropolitan Bank and Trust Company vs. Quilts and All, Inc., 22 SCRA 486 [1993]). The obligor and not the holder of the negotiable instrument has the burden of proof of showing that he no longer owes the obligee any amount (Travel-On, Inc. vs. Court of Appeals, 210 SCRA 351 [1992]).

Even casting aside the presumption of regularity of private transactions, receipt of the loan amounting to P121,966,058.67 (Exhibits 1-29, RCBC) was admitted by GOYU as indicated in the testimony of Go Song Hiap when he answered the queries of the trial court. ATTY. NATIVIDAD Q: But insofar as the amount stated in Exhibits 1 to 29RCBC, you received all the amounts stated therein? A: Yes, sir, I received the amount. COURT He is asking if he received all the amounts stated in Exhibits 1 to 29-RCBC? WITNESS: Yes, Your Honor, I received all the amounts. COURT Indicated in the Promissory Notes? WITNESS A. The promissory Notes they did not give to me but the amount I asked which is correct, Your Honor. COURT Q Your mean to say the amounts indicated in Exhibits 1 to 29-RCBC is correct? A Yes, Your Honor. (tsn, Jan. 14, 1994, p. 26.) Furthermore, aside from its judicial admission of having received all the proceeds of the 29 promissory notes as hereinabove quotes, GOYU also offered and admitted to RCBC that is obligation be fixed at P116,301,992.60 as shown in its letter date March 9, 1993, which pertinently reads: We wish to inform you, therefore that we are ready and willing to pay the current past due account of this company in the amount of

P116,301,992.60 as of 21 January 1993, specified in pars. 15, p. 10, and 18, p. 13 of your affidavits of Third Party Claims in the Urban case at Makati, Metro Manila and in the Zamboanga case at Zamboanga city, respectively, less the total of P8,851,519.71 paid from the Seaboard and Equitable insurance companies and other legitimate deductions. We accept and confirm this amount of P116,301,992.60 as stated as true and correct. (Exhibit BB.) The Court of Appeals erred in placing much significance on the fact that the excluded promissory notes are dated after the fire. It failed to consider that said notes had for their origin transactions consummated prior to the fire. Thus, careful attention must be paid to the fact that Promissory Notes No. 420-92 and 421-92 are mere renewalsof Promissory Notes No. 908-91 and 952-91, loans already availed of by GOYU. The two courts below erred in failing to see that the promissory notes which they ruled should be excluded for bearing dates which are after that of the fire, are mere renewals of previous ones. The proceeds of the loan represented by these promissory notes were admittedly received by GOYU. There is ample factual and legal basis for giving GOYU's judicial admission of liability in the amount of P116,301,992.60 full force and effect. It should, however, be quickly added that whatever amount RCBC may have recovered from the other insurers of the mortgage property will, nonetheless, have to be applied as payment against GOYU's obligation. But, contrary to the lower courts' findings, payments effected by GOYU prior to January 21, 1993 should no longer be deducted. Such payments had obviously been duly considered by GOYU, in its aforequoted letter date March 9, 1993, wherein it admitted that its past due account totaled P116,301,992.60 as of January 21, 1993. The net obligation of GOYU, after deductions, is thus reduced to P107,246,887.90 as of January 21, 1993, to wit: Total Obligation as admitted by GOYU as of January 21, 1993: P116,301,992.60 Broken down as follows: Principal 1 Interest Regular 80,535,946.32 FDU 27,548,025.17 ____________ Total 108,083,971.49 8,218,021.11 2

LESS: 1) Proceeds from Seaboard Eastern Insurance Company 6,095,145.81 2) Proceeds from Equitable Insurance Company 2,756,373.00 3) Payment from foreign department negotiation: 203,584.89 ___________ 9,055,104.70 3 ================ NET AMOUNT as of January 21, 1993 P107,246,887.90 The need for the payment of interest due the principal amount of the obligation, which is the cost of money to RCBC, the primary end and the ultimate reason for RCBC's existence and being, was duly recognized by the trial court when it ruled favorably on RCBC's counterclaim, ordering GOYU "to pay its loan obligation with RCBC in the amount of P68,785,069.04, as of April 27, 1992, with interest thereon at the rate stipulated in the respective promissory notes (without surcharges and penalties) per computation, pp. 14-A, 14-B 14-C" (Record, p. 479). Inexplicably, the Court of Appeals, without even laying down the factual or legal justification for its ruling, modified the trial court's ruling and ordered GOYU "to pay the principal amount of P68,785,069.04 without any interest, surcharges and penalties" (Rollo, p. 200). It is to be noted in this regard that even the trial court hedgingly and with much uncertainty deleted the payment ofadditional interest, penalties, and charges, in this manner: Regarding defendant RCBC's commitment not to charge additional interest, penalties and surcharges, the same does not require that it be embodied in a document or some form of writing to be binding and enforceable. The principle is well known that generally a verbal agreement or contract is no less binding and effective than a written one. And the existence of such a verbal agreement has been amply established by the evidence in this case. In any event, regardless of the existence of such verbal agreement, it would still be unjust and inequitable for defendant RCBC to charge the plaintiff with surcharges and penalties considering the latter's pitiful situation. (Emphasis supplied). (Record, p. 476)

The essence or rationale for the payment of interest or cost of money is separate and distinct from that of surcharges and penalties. What may justify a court in not allowing the creditor to charge surcharges and penalties despite express stipulation therefor in a valid agreement, may not equally justify non-payment of interest. The charging of interest for loans forms a very essential and fundamental element of the banking business, which may truly be considered to be at the very core of its existence or being. It is inconceivable for a bank to grant loans for which it will not charge any interest at all. We fail to find justification for the Court of Appeal's outright deletion of the payment of interest as agreed upon in the respective promissory notes. This constitutes gross error. For the computation of the interest due to be paid to RCBC, the following rules of thumb laid down by this Court inEastern Shipping Lines, Inc. vs. Court of Appeals (234 SCRA 78 [1994]), shall apply, to wit: I. When an obligation, regardless of its source, i.e., law, contracts, quasi-contracts, delicts or quasi-delicts is breached, the contravenor can be held liable for damages. The provisions under Title XVIII on "Damages" of the Civil Code govern in determining the measure of recoverable damages. II. With regard particularly to an award of interest in the concept of actual and compensatory damages, the rate of interest, as well as the actual thereof, is imposed, as follows: 1. When the obligation is breached, and it consists in the payment of a sum of money, i.e., a loan or forbearance of money, the interest due should be that which may have been stipulated in writing. Furthermore, the interest due shall itself earn legal interest from the time it is judicially demanded. In the absence of stipulation, the rate of interest shall be 12% per annum to be computed from default,i.e., from judicial or extrajudicial demand under and subject to the provisions of Article 1169 of the Civil Code. 2. When an obligation, not constituting a loan or forbearance of money, is breached, an interest on the amount of damages awarded may be imposed at the discretion of the court at the rate of 6% per annum. No interest, however, shall be adjudged on unliquidated claims or damages except when or until the demand can be established with reasonable certainty. Accordingly, where the demand is established with reasonable certainty, the interest shall begin to run from the time the claim is made judicially or extrajudicially (Art. 1169, Civil Code) but when such certainty cannot be so reasonably established at the time the demand is made, the interest shall begin to run only from the date of the judgment of the court is made (at which time the quantification of damages may be deemed to have been reasonably ascertained). The actual base for the computation of legal interest shall, in any case, be on the amount finally adjudged.

3. When the judgment of the court awarding a sum of money becomes final and executory, the rate of legal interest, whether the case falls under paragraph 1 or paragraph 2, above, shall be 12% per annum from such finality until its satisfaction, this interim period being deemed to be by then an equivalent to a forbearance of credit. (pp. 95-97). There being written stipulations as to the rate of interest owing on each specific promissory note as summarized and tabulated by the trial court in its decision (pp. 470 and 471, Record) such agreed interest rates must be followed. This is very clear from paragraph II, sub-paragraph 1 quoted above. On the issue of payment of surcharges and penalties, we partly agree that GOYU's pitiful situation must be taken into account. We do not agree, however, that payment of any amount as surcharges and penalties should altogether be deleted. Even assuming that RCBC, through its responsible officers, herein petitioners Eli Lao and Uy Chun Bing, may have relayed its assurance for assistance to GOYU immediately after the occurrence of the fire, we cannot accept the lower courts' finding that RCBC had thereby ipso facto effectively waived collection of any additional interests, surcharges, and penalties from GOYU. Assurances of assistance are one thing, but waiver of additional interests, surcharges, and penalties is another. Surcharges and penalties agreed to be paid by the debtor in case of default partake of the nature of liquidated damages, covered by Section 4, Chapter 3, Title XVIII of the Civil Code. Article 2227 thereof provides: Art. 2227. Liquidated damages, whether intended as a indemnity or penalty, shall be equitably reduced if they are iniquitous and unconscionable. In exercising this vested power to determine what is iniquitous and unconscionable, the Court must consider the circumstances of each case. It should be stressed that the Court will not make any sweeping ruling that surcharges and penalties imposed by banks for non-payment of the loans extended by them are generally iniquitous and unconscionable. What may be iniquitous and unconscionable in one case, may be totally just and equitable in another. This provision of law will have to be applied to the established facts of any given case. Given the circumstance under which GOYU found itself after the occurrence of the fire, the Court rules the surcharges rates ranging anywhere from 9% to 27%, plus the penalty charges of 36%, to be definitely iniquitous and unconscionable. The Court tempers these rates to 2% and 3%, respectively. Furthermore, in the light of GOYU's offer to pay the amount of P116,301,992.60 to RCBC as March 1993 (See: Exhibit "BB"), which RCBC refused, we find it more in keeping with justice and equity for RCBC not to charge additional interest, surcharges, and penalties from that time onward.

Given the factual milieu hereover, we rule that it was error to hold MICO liable in damages for denying or withholding the proceeds of the insurance claim to GOYU. Firstly, by virtue of the mortgage contracts as well as the endorsements of the insurance policies, RCBC has the right to claim the insurance proceeds, in substitution of the property lost in the fire. Having assigned its rights, GOYU lost its standing as the beneficiary of the said insurance policies. Secondly, for an insurance company to be held liable for unreasonably delaying and withholding payment of insurance proceeds, the delay must be wanton, oppressive, or malevolent (Zenith Insurance Corporation vs. CA. 185 SCRA 403 [1990]). It is generally agreed, however, that an insurer may in good faith and honesty entertain a difference of opinion as to its liability. Accordingly, the statutory penalty for vexatious refusal of an insurer to pay a claim should not be inflicted unless the evidence and circumstances show that such refusal was willful and without reasonable cause as the facts appear to a reasonable and prudent man (Bufallo Ins. Co. vs. Bommarito [CCA 8th] 42 F [2d] 53, 70 ALR 1211; Phoenix Ins. Co. vs. Clay, 101 Ga. 331, 28 SE 853, 65 Am St. Rep 307; Kusnetsky vs. Security Ins. Co., 313 Mo. 143, 281 SW 47, 45 ALR 189). The case at bar does not show that MICO wantonly and in bad faith delayed the release of the proceeds. The problem in the determination of who is the actual beneficiary of the insurance policies, aggravated by the claim of various creditors who wanted to partake of the insurance proceeds, not to mention the importance of the endorsement to RCBC, to our mind, and as now borne out by the outcome herein, justified MICO in withholding payment to GOYU. In adjudging RCBC liable in damages to GOYU, the Court of Appeals said that RCBC cannot avail itself of two simultaneous remedies in enforcing the claim of an unpaid creditor, one for specific performance and the other for foreclosure. In doing so, said the appellate court, the second action is deemed barred, RCBC having split a single cause of action (Rollo, pp. 195-199). The Court of Appeals was too accommodating in giving due consideration to this argument of GOYU, for the foreclosure suit is still pending appeal before the same Court of Appeals in CA G.R. CV No. 46247, the case having been elevated by RCBC. In finding that the foreclosure suit cannot prosper, the Fifteenth Division of the Court of Appeals pre-empted the resolution of said foreclosure case which is not before it. This is plain reversible error if not grave abuse of discretion. As held in Pea vs. Court of Appeals (245 SCRA 691 [1995]): It should have been enough, nonetheless, for the appellate court to merely set aside the questioned ordered of the trial court for having been issued by the latter with grave abuse of discretion. In likewise enjoining permanently herein petitioner "from entering in and interfering with the use or occupation and enjoyment of petitioner's (now private respondent) residential house and compound," the appellate court in effect,

precipitately resolved with finality the case for injunction that was yet to be heard on the merits by the lower court. Elevated to the appellate court, it might be stressed, were mere incidents of the principal case still pending with the trial court. In Municipality of Bian, Laguna vs. Court of Appeals, 219 SCRA 69, we ruled that the Court of Appeals would have "no jurisdiction in a certiorari proceeding involving an incident in a case to rule on the merits of the main case itself which was not on appeal before it. (pp. 701-702.) Anent the right of RCBC to intervene in Civil Case No. 1073, before the Zamboanga Regional Trial Court, since it has been determined that RCBC has the right to the insurance proceeds, the subject matter of intervention is rendered moot and academic. Respondent Sebastian must, however, yield to the preferential right of RCBC over the MICO insurance policies. It is basic and fundamental that the first mortgagee has superior rights over junior mortgagees or attaching creditors (Alpha Insurance & Surety Co. vs. Reyes, 106 SCRA 274 [1981]; Sun Life Assurance Co. of Canada vs. Gonzales Diaz, 52 Phil. 271 [1928]). WHEREFORE, the petitions are hereby GRANTED and the decision and resolution of December 16, 1996 and April 3, 1997 in CA-G.R. CV No. 46162 are hereby REVERSED and SET ASIDE, and a new one entered: 1. Dismissing the Complaint of private respondent GOYU in Civil Case No. 93-65442 before Branch 3 of the Manila Trial Court for lack of merit; 2. Ordering Malayan Insurance Company, Inc. to deliver to Rizal Commercial Banking Corporation the proceeds of the insurance policies in the amount of P51,862,390.94 (per report of adjuster Toplis & Harding (Far East), Inc., Exhibits "2" and "2-1"), less the amount of P50,505,594.60 (per O.R. No. 3649285); 3. Ordering the Clerk of Court to release the amount of P50,505,594.60 including the interests earned to Rizal Commercial Banking Corporation; 4. Ordering Goyu & Sons, Inc. to pay its loan obligation with Rizal Commercial Banking Corporation in the principal amount of P107,246,887.90, with interest at the respective rates stipulated in each promissory note from January 21, 1993 until finality of this judgment, and surcharges at 2% and penalties at 3% from January 21, 1993 to March 9, 1993, minus payments made by Malayan Insurance Company, Inc. and the proceeds of the amount deposited with the trial court and its earned interest. The total amount due RCBC at the time of the finality of this judgment shall earn interest at the legal rate of 12% in lieu of all other stipulated interests and charges until fully paid.

The petition of Rizal Commercial Banking Corporation against the respondent Court in CA-GR CV 48376 is DISMISSED for being moot and academic in view of the results herein arrived at. Respondent Sebastian's right as attaching creditor must yield to the preferential rights of Rizal Commercial Banking Corporation over the Malayan insurance policies as first mortgagee. SO ORDERED. Regalado, Puno, Mendoza and Martinez, JJ., concur. G.R. No. 184300 July 11, 2012

MALAYAN INSURANCE CO., INC., Petitioner, vs. PHILIPPINES FIRST INSURANCE CO., INC. and REPUTABLE FORWARDER SERVICES, INC., Respondents. DECISION REYES, J.: Before the Court is a petitiOn for review on certiorari filed by petitioner Malayan Insurance Co., lnc. (Malayan) assailing the Decision1 dated February 29, 2008 and Resolution2 dated August 28, 2008 of the Court of Appeals (CA) in CA-G.R. CV No. 71204 which affirmed with modification the decision of the Regional Trial Court (RTC), Branch 38 of Manila. Antecedent Facts Since 1989, Wyeth Philippines, Inc. (Wyeth) and respondent Reputable Forwarder Services, Inc. (Reputable) had been annually executing a contract of carriage, whereby the latter undertook to transport and deliver the formers products to its customers, dealers or salesmen.3 On November 18, 1993, Wyeth procured Marine Policy No. MAR 13797 (Marine Policy) from respondent Philippines First Insurance Co., Inc. (Philippines First) to secure its interest over its own products. Philippines First thereby insured Wyeths nutritional, pharmaceutical and other products usual or incidental to the insureds business while the same were being transported or shipped in the Philippines. The policy covers all risks of direct physical loss or damage from any external cause, if by land, and provides a limit of P6,000,000.00 per any one land vehicle. On December 1, 1993, Wyeth executed its annual contract of carriage with Reputable. It turned out, however, that the contract was not signed by Wyeths representative/s.4 Nevertheless, it was admittedly signed by Reputables representatives, the terms thereof faithfully observed by the parties and, as previously

stated, the same contract of carriage had been annually executed by the parties every year since 1989.5 Under the contract, Reputable undertook to answer for "all risks with respect to the goods and shall be liable to the COMPANY (Wyeth), for the loss, destruction, or damage of the goods/products due to any and all causes whatsoever, including theft, robbery, flood, storm, earthquakes, lightning, and other force majeure while the goods/products are in transit and until actual delivery to the customers, salesmen, and dealers of the COMPANY".6 The contract also required Reputable to secure an insurance policy on Wyeths goods.7 Thus, on February 11, 1994, Reputable signed a Special Risk Insurance Policy (SR Policy) with petitioner Malayan for the amount of P1,000,000.00. On October 6, 1994, during the effectivity of the Marine Policy and SR Policy, Reputable received from Wyeth 1,000 boxes of Promil infant formula worth P2,357,582.70 to be delivered by Reputable to Mercury Drug Corporation in Libis, Quezon City. Unfortunately, on the same date, the truck carrying Wyeths products was hijacked by about 10 armed men. They threatened to kill the truck driver and two of his helpers should they refuse to turn over the truck and its contents to the said highway robbers. The hijacked truck was recovered two weeks later without its cargo. On March 8, 1995, Philippines First, after due investigation and adjustment, and pursuant to the Marine Policy, paid Wyeth P2,133,257.00 as indemnity. Philippines First then demanded reimbursement from Reputable, having been subrogated to the rights of Wyeth by virtue of the payment. The latter, however, ignored the demand. Consequently, Philippines First instituted an action for sum of money against Reputable on August 12, 1996.8 In its complaint, Philippines First stated that Reputable is a "private corporation engaged in the business of a common carrier." In its answer,9 Reputable claimed that it is a private carrier. It also claimed that it cannot be made liable under the contract of carriage with Wyeth since the contract was not signed by Wyeths representative and that the cause of the loss was force majeure, i.e., the hijacking incident. Subsequently, Reputable impleaded Malayan as third-party defendant in an effort to collect the amount covered in the SR Policy. According to Reputable, "it was validly insured with Malayan for P1,000,000.00 with respect to the lost products under the latters Insurance Policy No. SR-0001-02577 effective February 1, 1994 to February 1, 1995" and that the SR Policy covered the risk of robbery or hijacking. 10 Disclaiming any liability, Malayan argued, among others, that under Section 5 of the SR Policy, the insurance does not cover any loss or damage to property which at the time of the happening of such loss or damage is insured by any marine policy and that the SR Policy expressly excluded third-party liability.

After trial, the RTC rendered its Decision11 finding Reputable liable to Philippines First for the amount of indemnity it paid to Wyeth, among others. In turn, Malayan was found by the RTC to be liable to Reputable to the extent of the policy coverage. The dispositive portion of the RTC decision provides: WHEREFORE, on the main Complaint, judgment is hereby rendered finding [Reputable] liable for the loss of the Wyeth products and orders it to pay Philippines First the following: 1. the amount of P2,133,257.00 representing the amount paid by Philippines First to Wyeth for the loss of the products in question; 2. the amount of P15,650.00 representing the adjustment fees paid by Philippines First to hired adjusters/surveyors; 3. the amount of P50,000.00 as attorneys fees; and 4. the costs of suit. On the third-party Complaint, judgment is hereby rendered finding Malayan liable to indemnify [Reputable] the following: 1. the amount of P1,000,000.00 representing the proceeds of the insurance policy; 2. the amount of P50,000.00 as attorneys fees; and 3. the costs of suit. SO ORDERED.12 Dissatisfied, both Reputable and Malayan filed their respective appeals from the RTC decision. Reputable asserted that the RTC erred in holding that its contract of carriage with Wyeth was binding despite Wyeths failure to sign the same. Reputable further contended that the provisions of the contract are unreasonable, unjust, and contrary to law and public policy. For its part, Malayan invoked Section 5 of its SR Policy, which provides: Section 5. INSURANCE WITH OTHER COMPANIES. The insurance does not cover any loss or damage to property which at the time of the happening of such loss or damage is insured by or would but for the existence of this policy, be insured by any Fire or Marine policy or policies except in respect of any excess beyond the amount

which would have been payable under the Fire or Marine policy or policies had this insurance not been effected. Malayan argued that inasmuch as there was already a marine policy issued by Philippines First securing the same subject matter against loss and that since the monetary coverage/value of the Marine Policy is more than enough to indemnify the hijacked cargo, Philippines First alone must bear the loss. Malayan sought the dismissal of the third-party complaint against it. In the alternative, it prayed that it be held liable for no more than P468,766.70, its alleged pro-rata share of the loss based on the amount covered by the policy, subject to the provision of Section 12 of the SR Policy, which states: 12. OTHER INSURANCE CLAUSE. If at the time of any loss or damage happening to any property hereby insured, there be any other subsisting insurance or insurances, whether effected by the insured or by any other person or persons, covering the same property, the company shall not be liable to pay or contribute more than its ratable proportion of such loss or damage. On February 29, 2008, the CA rendered the assailed decision sustaining the ruling of the RTC, the decretal portion of which reads: WHEREFORE, in view of the foregoing, the assailed Decision dated 29 September 2000, as modified in the Order dated 21 July 2001, is AFFIRMED with MODIFICATION in that the award of attorneys fees in favor of Reputable is DELETED. SO ORDERED.13 The CA ruled, among others, that: (1) Reputable is estopped from assailing the validity of the contract of carriage on the ground of lack of signature of Wyeths representative/s; (2) Reputable is liable under the contract for the value of the goods even if the same was lost due to fortuitous event; and (3) Section 12 of the SR Policy prevails over Section 5, it being the latter provision; however, since the ratable proportion provision of Section 12 applies only in case of double insurance, which is not present, then it should not be applied and Malayan should be held liable for the full amount of the policy coverage, that is, P1,000,000.00.14 On March 14, 2008, Malayan moved for reconsideration of the assailed decision but it was denied by the CA in its Resolution dated August 28, 2008. 15 Hence, this petition. Malayan insists that the CA failed to properly resolve the issue on the "statutory limitations on the liability of common carriers" and the "difference between an other insurance clause and an over insurance clause."

Malayan also contends that the CA erred when it held that Reputable is a private carrier and should be bound by the contractual stipulations in the contract of carriage. This argument is based on its assertion that Philippines First judicially admitted in its complaint that Reputable is a common carrier and as such, Reputable should not be held liable pursuant to Article 1745(6) of the Civil Code.16 Necessarily, if Reputable is not liable for the loss, then there is no reason to hold Malayan liable to Reputable. Further, Malayan posits that there resulted in an impairment of contract when the CA failed to apply the express provisions of Section 5 (referred to by Malayan as over insurance clause) and Section 12 (referred to by Malayan as other insurance clause) of its SR Policy as these provisions could have been read together there being no actual conflict between them. Reputable, meanwhile, contends that it is exempt from liability for acts committed by thieves/robbers who act with grave or irresistible threat whether it is a common carrier or a private/special carrier. It, however, maintains the correctness of the CA ruling that Malayan is liable to Philippines First for the full amount of its policy coverage and not merely a ratable portion thereof under Section 12 of the SR Policy. Finally, Philippines First contends that the factual finding that Reputable is a private carrier should be accorded the highest degree of respect and must be considered conclusive between the parties, and that a review of such finding by the Court is not warranted under the circumstances. As to its alleged judicial admission that Reputable is a common carrier, Philippines First proffered the declaration made by Reputable that it is a private carrier. Said declaration was allegedly reiterated by Reputable in its third party complaint, which in turn was duly admitted by Malayan in its answer to the said third-party complaint. In addition, Reputable even presented evidence to prove that it is a private carrier. As to the applicability of Sections 5 and 12 in the SR Policy, Philippines First reiterated the ruling of the CA. Philippines First, however, prayed for a slight modification of the assailed decision, praying that Reputable and Malayan be rendered solidarily liable to it in the amount of P998,000.00, which represents the balance from the P1,000.000.00 coverage of the SR Policy after deducting P2,000.00 under Section 10 of the said SR Policy.17 Issues The liability of Malayan under the SR Policy hinges on the following issues for resolution: 1) Whether Reputable is a private carrier; 2) Whether Reputable is strictly bound by the stipulations in its contract of carriage with Wyeth, such that it should be liable for any risk of loss or damage,

for any cause whatsoever, including that due to theft or robbery and other force majeure; 3) Whether the RTC and CA erred in rendering "nugatory" Sections 5 and Section 12 of the SR Policy; and 4) Whether Reputable should be held solidarily liable with Malayan for the amount of P998,000.00 due to Philippines First. The Courts Ruling On the first issue Reputable is a private carrier. The Court agrees with the RTC and CA that Reputable is a private carrier. Wellentrenched in jurisprudence is the rule that factual findings of the trial court, especially when affirmed by the appellate court, are accorded the highest degree of respect and considered conclusive between the parties, save for certain exceptional and meritorious circumstances, none of which are present in this case.18 Malayan relies on the alleged judicial admission of Philippines First in its complaint that Reputable is a common carrier.19 Invoking Section 4, Rule 129 of the Rules on Evidence that "an admission verbal or written, made by a party in the course of the proceeding in the same case, does not require proof," it is Malayans position that the RTC and CA should have ruled that Reputable is a common carrier. Consequently, pursuant to Article 1745(6) of the Civil Code, the liability of Reputable for the loss of Wyeths goods should be dispensed with, or at least diminished. It is true that judicial admissions, such as matters alleged in the pleadings do not require proof, and need not be offered to be considered by the court. "The court, for the proper decision of the case, may and should consider, without the introduction of evidence, the facts admitted by the parties."20 The rule on judicial admission, however, also states that such allegation, statement, or admission is conclusive as against the pleader,21 and that the facts alleged in the complaint are deemed admissions of the plaintiff and binding upon him.22 In this case, the pleader or the plaintiff who alleged that Reputable is a common carrier was Philippines First. It cannot, by any stretch of imagination, be made conclusive as against Reputable whose nature of business is in question. It should be stressed that Philippines First is not privy to the SR Policy between Wyeth and Reputable; rather, it is a mere subrogee to the right of Wyeth to collect from Reputable under the terms of the contract of carriage. Philippines First is not in any position to make any admission, much more a definitive pronouncement, as to the nature of Reputables business and there appears no other connection between Philippines First and Reputable which suggests mutual familiarity between them.

Moreover, records show that the alleged judicial admission of Philippines First was essentially disputed by Reputable when it stated in paragraphs 2, 4, and 11 of its answer that it is actually a private or special carrier.23 In addition, Reputable stated in paragraph 2 of its third-party complaint that it is "a private carrier engaged in the carriage of goods."24 Such allegation was, in turn, admitted by Malayan in paragraph 2 of its answer to the third-party complaint.25 There is also nothing in the records which show that Philippines First persistently maintained its stance that Reputable is a common carrier or that it even contested or proved otherwise Reputables position that it is a private or special carrier. Hence, in the face of Reputables contrary admission as to the nature of its o wn business, what was stated by Philippines First in its complaint is reduced to nothing more than mere allegation, which must be proved for it to be given any weight or value. The settled rule is that mere allegation is not proof.26 More importantly, the finding of the RTC and CA that Reputable is a special or private carrier is warranted by the evidence on record, primarily, the unrebutted testimony of Reputables Vice President and General Manager, Mr. William Ang Lian Suan, who expressly stated in open court that Reputable serves only one customer, Wyeth. 27 Under Article 1732 of the Civil Code, common carriers are persons, corporations, firms, or associations engaged in the business of carrying or transporting passenger or goods, or both by land, water or air for compensation, offering their services to the public. On the other hand, a private carrier is one wherein the carriage is generally undertaken by special agreement and it does not hold itself out to carry goods for the general public.28 A common carrier becomes a private carrier when it undertakes to carry a special cargo or chartered to a special person only.29 For all intents and purposes, therefore, Reputable operated as a private/special carrier with regard to its contract of carriage with Wyeth. On the second issue Reputable is bound by the terms of the contract of carriage. The extent of a private carriers obligation is dictated by the stipulations of a contract it entered into, provided its stipulations, clauses, terms and conditions are not contrary to law, morals, good customs, public order, or public policy. "The Civil Code provisions on common carriers should not be applied where the carrier is not acting as such but as a private carrier. Public policy governing common carriers has no force where the public at large is not involved."30 Thus, being a private carrier, the extent of Reputables liability is fully governed by the stipulations of the contract of carriage, one of which is that it shall be liable to Wyeth for the loss of the goods/products due to any and all causes whatsoever, including theft, robbery and other force majeure while the goods/products are in transit and until actual delivery to Wyeths customers, salesmen and dealers.31 On the third issue other insurance vis--vis over insurance.

Malayan refers to Section 5 of its SR Policy as an "over insurance clause" and to Section 12 as a "modified other insurance clause".32 In rendering inapplicable said provisions in the SR Policy, the CA ruled in this wise: Since Sec. 5 calls for Malayans complete absolution in case the other insurance would be sufficient to cover the entire amount of the loss, it is in direct conflict with Sec. 12 which provides only for a pro-rated contribution between the two insurers. Being the later provision, and pursuant to the rules on interpretation of contracts, Sec. 12 should therefore prevail. xxxx x x x The intention of both Reputable and Malayan should be given effect as against the wordings of Sec. 12 of their contract, as it was intended by the parties to operate only in case of double insurance, or where the benefits of the policies of both plaintiff-appellee and Malayan should pertain to Reputable alone. But since the court a quo correctly ruled that there is no double insurance in this case inasmuch as Reputable was not privy thereto, and therefore did not stand to benefit from the policy issued by plaintiffappellee in favor of Wyeth, then Malayans stand should be rejected. To rule that Sec. 12 operates even in the absence of double insurance would work injustice to Reputable which, despite paying premiums for a P1,000,000.00 insurance coverage, would not be entitled to recover said amount for the simple reason that the same property is covered by another insurance policy, a policy to which it was not a party to and much less, from which it did not stand to benefit. Plainly, this unfair situation could not have been the intention of both Reputable and Malayan in signing the insurance contract in question.33 In questioning said ruling, Malayan posits that Sections 5 and 12 are separate provisions applicable under distinct circumstances. Malayan argues that "it will not be completely absolved under Section 5 of its policy if it were the assured itself who obtained additional insurance coverage on the same property and the loss incurred by Wyeths cargo was more than that insured by Philippines Firsts marine policy. On the other hand, Section 12 will not completely absolve Malayan if additional insurance coverage on the same cargo were obtained by someone besides Reputable, in which case Malayans SR policy will contribute or share ratable proportion of a covered cargo loss."34 Malayans position cannot be countenanced. Section 5 is actually the other insurance clause (also called "additional insurance" and "double insurance"), one akin to Condition No. 3 in issue in Geagonia v. CA, 35 which validity was upheld by the Court as a warranty that no other insurance exists. The Court ruled that Condition No. 336 is a condition which is not proscribed by law as its incorporation in the policy is allowed by Section 75 of the Insurance Code. It was also the Courts finding that unlike the other insurance clauses, Condition No. 3 does not

absolutely declare void any violation thereof but expressly provides that the condition "shall not apply when the total insurance or insurances in force at the time of the loss or damage is not more than P200,000.00." In this case, similar to Condition No. 3 in Geagonia, Section 5 does not provide for the nullity of the SR Policy but simply limits the liability of Malayan only up to the excess of the amount that was not covered by the other insurance policy. In interpreting the "other insurance clause" in Geagonia, the Court ruled that the prohibition applies only in case of double insurance. The Court ruled that in order to constitute a violation of the clause, the other insurance must be upon same subject matter, the same interest therein, and the same risk. Thus, even though the multiple insurance policies involved were all issued in the name of the same assured, over the same subject matter and covering the same risk, it was ruled that there was no violation of the "other insurance clause" since there was no double insurance. Section 12 of the SR Policy, on the other hand, is the over insurance clause. More particularly, it covers the situation where there is over insurance due to double insurance. In such case, Section 15 provides that Malayan shall "not be liable to pay or contribute more than its ratable proportion of such loss or damage." This is in accord with the principle of contribution provided under Section 94(e) of the Insurance Code,37 which states that "where the insured is over insured by double insurance, each insurer is bound, as between himself and the other insurers, to contribute ratably to the loss in proportion to the amount for which he is liable under his contract." Clearly, both Sections 5 and 12 presuppose the existence of a double insurance. The pivotal question that now arises is whether there is double insurance in this case such that either Section 5 or Section 12 of the SR Policy may be applied. By the express provision of Section 93 of the Insurance Code, double insurance exists where the same person is insured by several insurers separately in respect to the same subject and interest. The requisites in order for double insurance to arise are as follows:38 1. The person insured is the same; 2. Two or more insurers insuring separately; 3. There is identity of subject matter; 4. There is identity of interest insured; and 5. There is identity of the risk or peril insured against. In the present case, while it is true that the Marine Policy and the SR Policy were both issued over the same subject matter, i.e. goods belonging to Wyeth, and both covered the same peril insured against, it is, however, beyond cavil that the said policies were

issued to two different persons or entities. It is undisputed that Wyeth is the recognized insured of Philippines First under its Marine Policy, while Reputable is the recognized insured of Malayan under the SR Policy. The fact that Reputable procured Malayans SR Policy over the goods of Wyeth pursuant merely to the stipulated requirement under its contract of carriage with the latter does not make Reputable a mere agent of Wyeth in obtaining the said SR Policy. The interest of Wyeth over the property subject matter of both insurance contracts is also different and distinct from that of Reputables. The policy issued by Philippines First was in consideration of the legal and/or equitable interest of Wyeth over its own goods. On the other hand, what was issued by Malayan to Reputable was over the latters insurable interest over the safety of the goods, which may become the basis of the latters liability in case of loss or damage to the property and falls within the contemplation of Section 15 of the Insurance Code.39 Therefore, even though the two concerned insurance policies were issued over the same goods and cover the same risk, there arises no double insurance since they were issued to two different persons/entities having distinct insurable interests. Necessarily, over insurance by double insurance cannot likewise exist. Hence, as correctly ruled by the RTC and CA, neither Section 5 nor Section 12 of the SR Policy can be applied. Apart from the foregoing, the Court is also wont to strictly construe the controversial provisions of the SR Policy against Malayan.1wphi1 This is in keeping with the rule that: "Indemnity and liability insurance policies are construed in accordance with the general rule of resolving any ambiguity therein in favor of the insured, where the contract or policy is prepared by the insurer. A contract of insurance, being a contract of adhesion, par excellence, any ambiguity therein should be resolved against the insurer; in other words, it should be construed liberally in favor of the insured and strictly against the insurer. Limitations of liability should be regarded with extreme jealousy and must be construed in such a way as to preclude the insurer from noncompliance with its obligations."40 Moreover, the CA correctly ruled that: To rule that Sec. 12 operates even in the absence of double insurance would work injustice to Reputable which, despite paying premiums for a P1,000,000.00 insurance coverage, would not be entitled to recover said amount for the simple reason that the same property is covered by another insurance policy, a policy to which it was not a party to and much less, from which it did not stand to benefit. x x x41 On the fourth issue Reputable is not solidarily liable with Malayan. There is solidary liability only when the obligation expressly so states, when the law so provides or when the nature of the obligation so requires.

In Heirs of George Y. Poe v. Malayan lnsurance Company., lnc.,42 the Court ruled that: Where the insurance contract provides for indemnity against liability to third persons, the liability of the insurer is direct and such third persons can directly sue the insurer. The direct liability of the insurer under indemnity contracts against third party[- ]liability does not mean, however, that the insurer can be held solidarily liable with the insured and/or the other parties found at fault, since they are being held liable under different obligations. The liability of the insured carrier or vehicle owner is based on tort, in accordance with the provisions of the Civil Code; while that of the insurer arises from contract, particularly, the insurance policy:43 (Citation omitted and emphasis supplied) Suffice it to say that Malayan's and Reputable's respective liabilities arose from different obligations- Malayan's is based on the SR Policy while Reputable's is based on the contract of carriage. All told, the Court finds no reversible error in the judgment sought to be reviewed. WHEREFORE, premises considered, the petition is DENIED. The Decision dated February 29, 2008 and Resolution dated August 28, 2008 of the Court of Appeals in CA-G.R. CV No. 71204 are hereby AFFIRMED. Cost against petitioner Malayan Insurance Co., Inc. SO ORDERED. BIENVENIDO L. REYES Associate justice WE CONCUR: ANTONIO T. CARPIO Senior Associate Justice Chairperson, Second Division ARTURO D. BRION Associate Justice JOSE PORTUGAL PEREZ Associate Justice

MARIA LOURDES P.A. SERENO Associate justice CERTIFICATION I certify that the conclusions in the above Decision had been reached in consultation before the case was assigned to the writer of the opinion of the Court's Division.

ANTONIO T. CARPIO Senior Associate Justice (Per Section 12, R.A. 296, The Judiciary Act of 1948, as amended)

G.R. No. 85296 May 14, 1990 ZENITH INSURANCE CORPORATION, petitioner, vs. COURT OF APPEALS and LAWRENCE FERNANDEZ, respondents. Vicente R. Layawen for petitioner. Lawrence L. Fernandez & Associates for private respondent.

MEDIALDEA, J.: Assailed in this petition is the decision of the Court of Appeals in CA-G.R. C.V. No. 13498 entitled, "Lawrence L. Fernandez, plaintiff-appellee v. Zenith Insurance Corp., defendant-appellant" which affirmed in toto the decision of the Regional Trial Court of

Cebu, Branch XX in Civil Case No. CEB-1215 and the denial of petitioner's Motion for Reconsideration. The antecedent facts are as follows: On January 25, 1983, private respondent Lawrence Fernandez insured his car for "own damage" under private car Policy No. 50459 with petitioner Zenith Insurance Corporation. On July 6, 1983, the car figured in an accident and suffered actual damages in the amount of P3,640.00. After allegedly being given a run around by Zenith for two (2) months, Fernandez filed a complaint with the Regional Trial Court of Cebu for sum of money and damages resulting from the refusal of Zenith to pay the amount claimed. The complaint was docketed as Civil Case No. CEB-1215. Aside from actual damages and interests, Fernandez also prayed for moral damages in the amount of P10,000.00, exemplary damages of P5,000.00, attorney's fees of P3,000.00 and litigation expenses of P3,000.00. On September 28, 1983, Zenith filed an answer alleging that it offered to pay the claim of Fernandez pursuant to the terms and conditions of the contract which, the private respondent rejected. After the issues had been joined, the pre-trial was scheduled on October 17, 1983 but the same was moved to November 4, 1983 upon petitioner's motion, allegedly to explore ways to settle the case although at an amount lower than private respondent's claim. On November 14, 1983, the trial court terminated the pretrial. Subsequently, Fernandez presented his evidence. Petitioner Zenith, however, failed to present its evidence in view of its failure to appear in court, without justifiable reason, on the day scheduled for the purpose. The trial court issued an order on August 23, 1984 submitting the case for decision without Zenith's evidence (pp. 10-11, Rollo). Petitioner filed a petition for certiorari with the Court of Appeals assailing the order of the trial court submitting the case for decision without petitioner's evidence. The petition was docketed as C.A.-G.R. No. 04644. However, the petition was denied due course on April 29, 1986 (p. 56, Rollo). On June 4, 1986, a decision was rendered by the trial court in favor of private respondent Fernandez. The dispositive portion of the trial court's decision provides: WHEREFORE, defendant is hereby ordered to pay to the plaintiff: 1. The amount of P3,640.00 representing the damage incurred plus interest at the rate of twice the prevailing interest rates; 2. The amount of P20,000.00 by way of moral damages; 3. The amount of P20,000.00 by way of exemplary damages; 4. The amount of P5,000.00 as attorney's fees; 5. The amount of P3,000.00 as litigation expenses; and 6. Costs. (p. 9, Rollo) Upon motion of Fernandez and before the expiration of the period to appeal, the trial court, on June 20, 1986, ordered the execution of the decision pending appeal. The

order was assailed by petitioner in a petition forcertiorari with the Court of Appeals on October 23, 1986 in C.A. G.R. No. 10420 but which petition was also dismissed on December 24, 1986 (p. 69, Rollo). On June 10, 1986, petitioner filed a notice of appeal before the trial court. The notice of appeal was granted in the same order granting private respondent's motion for execution pending appeal. The appeal to respondent court assigned the following errors: I. The lower court erred in denying defendant appellant to adduce evidence in its behalf. II. The lower court erred in ordering Zenith Insurance Corporation to pay the amount of P3,640.00 in its decision. III. The lower court erred in awarding moral damages, attorneys fees and exemplary damages, the worst is that, the court awarded damages more than what are prayed for in the complaint. (p. 12,Rollo) On August 17, 1988, the Court of Appeals rendered its decision affirming in toto the decision of the trial court. It also ruled that the matter of the trial court's denial of Fernandez's right to adduce evidence is a closed matter in view of its (CA) ruling in ACG.R. 04644 wherein Zenith's petition questioning the trial court's order submitting the case for decision without Zenith's evidence, was dismissed. The Motion for Reconsideration of the decision of the Court of Appeals dated August 17, 1988 was denied on September 29, 1988, for lack of merit. Hence, the instant petition was filed by Zenith on October 18, 1988 on the allegation that respondent Court of Appeals' decision and resolution ran counter to applicable decisions of this Court and that they were rendered without or in excess of jurisdiction. The issues raised by petitioners in this petition are: a) The legal basis of respondent Court of Appeals in awarding moral damages, exemplary damages and attomey's fees in an amount more than that prayed for in the complaint. b) The award of actual damages of P3,460.00 instead of only P1,927.50 which was arrived at after deducting P250.00 and P274.00 as deductible franchise and 20% depreciation on parts as agreed upon in the contract of insurance. Petitioner contends that while the complaint of private respondent prayed for P10,000.00 moral damages, the lower court awarded twice the amount, or P20,000.00 without factual or legal basis; while private respondent prayed for P5,000.00 exemplary damages, the trial court awarded P20,000.00; and while private respondent prayed for P3,000.00 attorney's fees, the trial court awarded P5,000.00. The propriety of the award of moral damages, exemplary damages and attorney's fees is the main issue raised herein by petitioner.

The award of damages in case of unreasonable delay in the payment of insurance claims is governed by the Philippine Insurance Code, which provides: Sec. 244. In case of any litigation for the enforcement of any policy or contract of insurance, it shall be the duty of the Commissioner or the Court, as the case may be, to make a finding as to whether the payment of the claim of the insured has been unreasonably denied or withheld; and in the affirmative case, the insurance company shall be adjudged to pay damages which shall consist of attomey's fees and other expenses incurred by the insured person by reason of such unreasonable denial or withholding of payment plus interest of twice the ceiling prescribed by the Monetary Board of the amount of the claim due the insured, from the date following the time prescribed in section two hundred forty-two or in section two hundred forty-three, as the case may be, until the claim is fully satisfied; Provided, That the failure to pay any such claim within the time prescribed in said sections shall be considered prima facie evidence of unreasonable delay in payment. It is clear that under the Insurance Code, in case of unreasonable delay in the payment of the proceeds of an insurance policy, the damages that may be awarded are: 1) attorney's fees; 2) other expenses incurred by the insured person by reason of such unreasonable denial or withholding of payment; 3) interest at twice the ceiling prescribed by the Monetary Board of the amount of the claim due the injured; and 4) the amount of the claim. As regards the award of moral and exemplary damages, the rules under the Civil Code of the Philippines shall govern. "The purpose of moral damages is essentially indemnity or reparation, not punishment or correction. Moral damages are emphatically not intended to enrich a complainant at the expense of a defendant, they are awarded only to enable the injured party to obtain means, diversions or amusements that will serve to alleviate the moral suffering he has undergone by reason of the defendant's culpable action." (J. Cezar S. Sangco, Philippine Law on Torts and Damages, Revised Edition, p. 539) (See also R and B Surety & Insurance Co., Inc. v. IAC, G.R. No. 64515, June 22, 1984; 129 SCRA 745). While it is true that no proof of pecuniary loss is necessary in order that moral damages may be adjudicated, the assessment of which is left to the discretion of the court according to the circumstances of each case (Art. 2216, New Civil Code), it is equally true that in awarding moral damages in case of breach of contract, there must be a showing that the breach was wanton and deliberately injurious or the one responsible acted fraudently or in bad faith (Perez v. Court of Appeals, G.R. No. L-20238, January 30,1965; 13 SCRA 137; Solis v. Salvador, G.R. No. L-17022, August 14, 1965; 14 SCRA 887). In the instant case, there was a finding that private respondent was given a "run-around" for two months, which is the basis for the award of the damages granted under the Insurance Code for unreasonable delay in the payment of the claim. However, the act of petitioner of delaying payment for two months cannot be considered

as so wanton or malevolent to justify an award of P20,000.00 as moral damages, taking into consideration also the fact that the actual damage on the car was only P3,460. In the pre-trial of the case, it was shown that there was no total disclaimer by respondent. The reason for petitioner's failure to indemnify private respondent within the two-month period was that the parties could not come to an agreement as regards the amount of the actual damage on the car. The amount of P10,000.00 prayed for by private respondent as moral damages is equitable. On the other hand, exemplary or corrective damages are imposed by way of example or correction for the public good (Art. 2229, New Civil Code of the Philippines). In the case of Noda v. Cruz-Arnaldo, G.R. No. 57322, June 22,1987; 151 SCRA 227, exemplary damages were not awarded as the insurance company had not acted in wanton, oppressive or malevolent manner. The same is true in the case at bar. The amount of P5,000.00 awarded as attomey's fees is justified under the circumstances of this case considering that there were other petitions filed and defended by private respondent in connection with this case. As regards the actual damages incurred by private respondent, the amount of P3,640.00 had been established before the trial court and affirmed by the appellate court. Respondent appellate court correctly ruled that the deductions of P250.00 and P274.00 as deductible franchise and 20% depreciation on parts, respectively claimed by petitioners as agreed upon in the contract, had no basis. Respondent court ruled: Under its second assigned error, defendant-appellant puts forward two arguments, both of which are entirely without merit. It is contented that the amount recoverable under the insurance policy defendant-appellant issued over the car of plaintiff-appellee is subject to deductible franchise, and . . . . The policy (Exhibit G, pp. 4-9, Record), does not mntion any deductible franchise, . . . (p. 13, Rollo) Therefore, the award of moral damages is reduced to P10,000.00 and the award of exemplary damages is hereby deleted. The awards due to private respondent Fernandez are as follows: 1) P3,640.00 as actual claim plus interest of twice the ceiling prescribed by the Monetary Board computed from the time of submission of proof of loss; 2) P10,000.00 as moral damages; 3) P5,000.00 as attorney's fees; 4) P3,000.00 as litigation expenses; and 5) Costs.

ACCORDINGLY, the appealed decision is MODIFIED as above stated. SO ORDERED. Narvasa, Cruz and Grio-Aquino, JJ., concur. Gancayco, J., is on leave.

G.R. No. 103883 November 14, 1996 JACQUELINE JIMENEZ VDA. DE GABRIEL, petitioner, vs. HON. COURT OF APPEALS and FORTUNE INSURANCE & SURETY COMPANY, INC., respondents.

VITUG, J.: The petition for review on certiorari in this case seeks the reversal of the decision 1 of the Court of Appeals setting aside the judgment of the Regional Trial Court of Manila, Branch 55, which has ordered private respondent Fortune Insurance & Surety Company, Inc., to pay petitioner Jacqueline Jimenez vda. de Gabriel, the surviving spouse and beneficiary in an accident (group) insurance of her deceased husband, the amount of P100,000.00, plus legal interest. Marcelino Gabriel, the insured, was employed by Emerald Construction & Development Corporation ("ECDC") at its construction project in Iraq. He was covered by a personal accident insurance in the amount of P100,000.00 under a group policy 2 procured from private respondent by ECDC for its overseas workers. The insured risk was for "(b)odily injury caused by violent accidental external and visible means which injury (would) solely and independently of any other cause" 3 result in death or disability. On 22 May 1982, within the life of the policy, Gabriel died in Iraq. A year later, or on 12 July 1983, ECDC reported Gabriel's death to private respondent by telephone. 4 Among the documents thereafter submitted to private respondent were a copy of the death certificate 5 issued by the Ministry of Health of the Republic of Iraq which stated REASON OF DEATH: UNDER EXAMINATION NOW NOT YET KNOWN 6 and an autopsy report 7 of the National Bureau of Investigation ("NBI") to the effect that "(d)ue to advanced state of postmortem decomposition, cause of

death (could) not be determined." 8 Private respondent referred the insurance claim to Mission Adjustment Service, Inc. Following a series of communications between petitioner and private respondent, the latter, on 22 September 1983, ultimately denied the claim of ECDC on the ground of prescription. 9 Petitioner went to the Regional Trial Court of Manila. In her complaint against ECDC and private respondent, she averred that her husband died of electrocution while in the performance of his work and prayed for the recovery of P100,000.00 for insurance indemnification and of various other sums by way of actual, moral, and exemplary damages, plus attorney's fees and costs of suit. Private respondent filed its answer, which was not verified, admitting the genuineness and due execution of the insurance policy; it alleged, however, that since both the death certificate issued by the Iraqi Ministry of Health and the autopsy report of the NBI failed to disclose the cause of Gabriel's death, it denied liability under the policy. In addition, private respondent raised the defense of "prescription," invoking Section 384 10 of the Insurance Code. Later, private respondent filed an amended answer, still unverified, reiterating its original defenses but, this time, additionally putting up a counterclaim and a crossclaim. The trial court dismissed the case against ECDC for the failure of petitioner to take steps to cause the service of the fourth alias summons on ECDC. The dismissal was without prejudice. The case proceeded against private respondent alone. On 28 May 1987, the trial court rendered its decision 11 in favor (partly) of petitioner's claim. In arriving at its conclusion, the trial court held that private respondent was deemed to have waived the defense, i.e., that the cause of Gabriel's death was not covered by the policy, when the latter failed to impugn by evidence petitioner's averment on the matter. With regard to the defense of prescription, the court considered the complaint to have been timely filed or within one (1) year from private respondent's denial of the claim. Petitioner and private respondent both appealed to the Court of Appeals. Petitioner contended that the lower court should have awarded all the claims she had asked for. Private respondent asserted, on its part, that the lower court erred in ruling (a) that the insurer had waived the defense that Gabriel's death was not caused by the insured peril ("violent accidental external and visible means") specified in the policy and (b) that the cause of action had not prescribed. The Court of Appeals, on 18 September 1991, reversed the decision of the lower court. The appellate court held that petitioner had failed to substantiate her allegation that her husband's death was caused by a risk insured against. The appellate court observed that the only evidence presented by petitioner, in her attempt to show the circumstances that led to the death of the insured, were her own affidavit and a letter allegedly written by a co-worker of the deceased in Iraq which, unfortunately for her, were held to be both hearsay. 12

The motion for reconsideration was denied.

13

Petitioner's recourse to this Court must also fail. On the issue of "prescription," private respondent correctly invoked Section 384 of the Insurance Code; viz: Sec. 384. Any person having any claim upon the policy issued pursuant to this chapter shall, without any unnecessary delay, present to the insurance company concerned a written notice of claim setting forth the nature, extent and duration of the injuries sustained as certified by a duly licensed physician. Notice of claim must be filed within six months from date of the accident, otherwise, the claim shall be deemed waived. Action or suit for recovery of damage due to loss or injury must be brought, in proper cases, with the Commissioner or the Courts within one year from denial of the claim, otherwise, the claimant's right of action shall prescribe. The notice of death was given to private respondent, concededly, more than a year after the death of petitioner's husband. Private respondent, in invoking prescription, was not referring to the one-year period from the denial of the claim within which to file an action against an insurer but obviously to the written notice of claim that had to be submitted within six months from the time of the accident. Petitioner argues that private respondent must be deemed to have waived its right to controvert the claim, that is, to show that the cause of death is an excepted peril, by failing to have its answers (to the Request for Admission sent by petitioner) duly verified. It is true that a matter of which a written request for admission is made shall be deemed impliedly admitted "unless, within a period designated in the request, which shall not be less than ten (10) days after service thereof, or within such further time as the court may allow on motion and notice, the party to whom the request is directed serves upon the party requesting the admission a sworn statement either denying specifically the matters of which an admission is requested or setting forth in detail the reasons why he cannot truthfully either admit or deny those matters;" 14 however, the verification, like in most cases required by the rules of procedure, is a formal, not jurisdictional, requirement, and mainly intended to secure an assurance that matters which are alleged are done in good faith or are true and correct and not of mere speculation. When circumstances warrant, the court may simply order the correction of unverified pleadings or act on it and waive strict compliance with the rules in order that the ends of justice may thereby be served. 15 In the case of answers to written requests for admission particularly, the court can allow the party making the admission, whether made expressly or deemed to have been made impliedly, "to withdraw or amend it upon such terms as may be just." 16 The appellate court acted neither erroneously nor with grave abuse of discretion when it seconded the court a quoand ruled:

As to the allegation of the plaintiff-appellant that the matters requested by her to be admitted by the defendant-appellant under the Request for Admission were already deemed admitted by the latter for its failure to answer it under oath, has already been properly laid to rest when the lower court in its Order of May 28, 1987 correctly ruled: At the outset, it must be stressed that the defendant indeed filed a written answer to the request for admission, sans verification. The case of Motor Service Co., Inc. vs. Yellow Taxicab Co., Inc., et al. may not therefore be controlling, or actually opposite. In said case, there was an absolute failure on the part of the defendant to answer the request for admission, and thus the court was justified in rendering a summary judgment. Here, however, as clearly intimated elsewhere above, the defendant answered in writing practically every question posed in the request for admission. The Court believes, under the peculiar circumstance, that the more controlling jurisprudence on the mater would be those cited by the defendant in its memorandum, particularly the case of Quimpo vs. de la Victoria, 46 SCRA 139. Prescinding from the foregoing, there is absolutely no basis in fact and in law for the lower court to hold that the appellant insurance company was deemed to have waived the defense, that the death of plaintiff-appellant's husband was not caused by violent accidental external and visible means' as contemplated in the insurance policy. The Death Certificate (Exh. 9) and the Autopsy Report (Exh. 10), more than controverted the allegation of the plaintiff-appellant as to the cause of death of her husband. 17 The insurance policy expressly provided that to be compensable, the injury or death should be caused by "violent accidental external and visible means." In attempting to prove the cause of her husband's death, all that petitioner could submit were a letter sent to her by her husband's co-worker, stating that Gabriel died when he tried to haul water out of a tank while its submerged motor was still functioning, 18 and petitioner's sinumpaang salaysay 19 which merely confirmed the receipt and stated contents of the letter. Said the appellate court in this regard: . . . . It must be noted that the only evidence presented by her to prove the circumstances surrounding her husband's death were her purported affidavit and the letter allegedly written by the deceased co-worker in Iraq. The said affidavit however suffers from procedural infirmity as it was not even testified to or identified by the affiant (plaintiff-appellant) herself. This self-serving affidavit therefore is a mere hearsay under the rules, . . . .

xxx xxx xxx In like manner, the letter allegedly written by the deceased's co-worker which was never identified to in court by the supposed author, suffers from the same defect as the affidavit of the plaintiff-appellant. 20 Not one of the other documents submitted, to wit, the POEA decision, dated 06 June 1984, 21 the death certificate issued by the Ministry of Health of Iraq and the NBI autopsy report, 22 could give any probative value to petitioner's claim. The POEA decision did not make any categorical holding on the specific cause of Gabriel's death. Neither did the death certificate issued by the health authorities in Iraq nor the NBI autopsy report provide any clue on the cause of death. All that appeared to be clear was the fact of Gabriel's demise on 22 May 1982 in Iraq. Evidence, in fine, is utterly wanting to establish that the insured suffered from an accidental death, the risk covered by the policy. In an accident insurance, the insured's beneficiary has the burden of proof in demonstrating that the cause of death is due to the covered peril. Once that fact is established, the burden then shifts to the insurer to show any excepted peril that may have been stipulated by the parties. An "accident insurance" is not thus to be likened to an ordinary life insurance where the insured's death, regardless of the cause thereof, would normally be compensable. The latter is akin in property insurance to an "all risk" coverage where the insured, on the aspect of burden of proof, has merely to show the condition of the property insured when the policy attaches and the fact of loss or damage during the period of the policy and where, thereafter, the burden would be on the insurer to show any "excluded peril." When, however, the insured risk is specified, like in the case before us, it lies with the claimant of the insurance proceeds to initially prove that the loss is caused by the covered peril. While petitioner did fail in substantiating her allegation that the death of her husband was due to an accident, considering, however, the uncertainty on the real cause of death, private respondent might find its way clear into still taking a second look on the matter and perhaps help ease the load of petitioner's loss. WHEREFORE, the decision appealed from is AFFIRMED. No costs. SO ORDERED. Padilla, Bellosillo, Kapunan and Hermosisima, Jr., JJ., concur. G.R. No. L-66935 November 11, 1985 ISABELA ROQUE, doing busines under the name and style of Isabela Roque Timber Enterprises and ONG CHIONG, petitioners, vs. HON. INTERMEDIATE APPELATE COURT and PIONEER INSURANCE AND SURETY CORPORATION,respondent.

GUTIERREZ, JR., J.: This petition for certiorari asks for the review of the decision of the Intermediate Appellate Court which absolved the respondent insurance company from liability on the grounds that the vessel carrying the insured cargo was unseaworthy and the loss of said cargo was caused not by the perils of the sea but by the perils of the ship. On February 19, 1972, the Manila Bay Lighterage Corporation (Manila Bay), a common carrier, entered into a contract with the petitioners whereby the former would load and carry on board its barge Mable 10 about 422.18 cubic meters of logs from Malampaya Sound, Palawan to North Harbor, Manila. The petitioners insured the logs against loss for P100,000.00 with respondent Pioneer Insurance and Surety Corporation (Pioneer). On February 29, 1972, the petitioners loaded on the barge, 811 pieces of logs at Malampaya Sound, Palawan for carriage and delivery to North Harbor, Port of Manila, but the shipment never reached its destination because Mable 10 sank with the 811 pieces of logs somewhere off Cabuli Point in Palawan on its way to Manila. As alleged by the petitioners in their complaint and as found by both the trial and appellate courts, the barge where the logs were loaded was not seaworthy such that it developed a leak. The appellate court further found that one of the hatches was left open causing water to enter the barge and because the barge was not provided with the necessary cover or tarpaulin, the ordinary splash of sea waves brought more water inside the barge. On March 8, 1972, the petitioners wrote a letter to Manila Bay demanding payment of P150,000.00 for the loss of the shipment plus P100,000.00 as unrealized profits but the latter ignored the demand. Another letter was sent to respondent Pioneer claiming the full amount of P100,000.00 under the insurance policy but respondent refused to pay on the ground that its hability depended upon the "Total loss by Total Loss of Vessel only". Hence, petitioners commenced Civil Case No. 86599 against Manila Bay and respondent Pioneer. After hearing, the trial court found in favor of the petitioners. The dispositive portion of the decision reads: FOR ALL THE FOREGOING, the Court hereby rendered judgment as follows: (a) Condemning defendants Manila Bay Lighterage Corporation and Pioneer Insurance and Surety Corporation to pay plaintiffs, jointly and severally, the sum of P100,000.00; (b) Sentencing defendant Manila Bay Lighterage Corporation to pay plaintiff, in addition, the sum of P50,000.00, plus P12,500.00, that the

latter advanced to the former as down payment for transporting the logs in question; (c) Ordering the counterclaim of defendant Insurance against plaintiffs, dismissed, for lack of merit, but as to its cross-claim against its codefendant Manila Bay Lighterage Corporation, the latter is ordered to reimburse the former for whatever amount it may pay the plaintiffs as such surety; (d) Ordering the counterclaim of defendant Lighterage against plaintiffs, dismissed for lack of merit; (e) Plaintiffs' claim of not less than P100,000.00 and P75,000.00 as exemplary damages are ordered dismissed, for lack of merits; plaintiffs' claim for attorney's fees in the sum of P10,000.00 is hereby granted, against both defendants, who are, moreover ordered to pay the costs; and (f) The sum of P150,000.00 award to plaintiffs, shall bear interest of six per cent (6%) from March 25, 1975, until amount is fully paid. Respondent Pioneer appealed to the Intermediate Appellate Court. Manila Bay did not appeal. According to the petitioners, the transportation company is no longer doing business and is without funds. During the initial stages of the hearing, Manila Bay informed the trial court that it had salvaged part of the logs. The court ordered them to be sold to the highest bidder with the funds to be deposited in a bank in the name of Civil Case No. 86599. On January 30, 1984, the appellate court modified the trial court's decision and absolved Pioneer from liability after finding that there was a breach of implied warranty of seaworthiness on the part of the petitioners and that the loss of the insured cargo was caused by the "perils of the ship" and not by the "perils of the sea". It ruled that the loss is not covered by the marine insurance policy. After the appellate court denied their motion for reconsideration, the petitioners filed this petition with the following assignments of errors: I THE INTERMEDIATE APPELLATE COURT ERRED IN HOLDING THAT IN CASES OF MARINE CARGO INSURANCE, THERE IS A WARRANTY OF SEAWORTHINESS BY THE CARGO OWNER. II

THE INTERMEDIATE APPELLATE COURT ERRED IN HOLDING THAT THE LOSS OF THE CARGO IN THIS CASE WAS CAUSED BY "PERILS OF THE SHIP" AND NOT BY "PERILS OF THE SEA." III THE INTERMEDIATE APPELLATE COURT ERRED IN NOT ORDERING THE RETURN TO PETITIONER OF THE AMOUNT OF P8,000.00 WHICH WAS DEPOSITED IN THE TRIAL COURT AS SALVAGE VALUE OF THE LOGS THAT WERE RECOVERED. In their first assignment of error, the petitioners contend that the implied warranty of seaworthiness provided for in the Insurance Code refers only to the responsibility of the shipowner who must see to it that his ship is reasonably fit to make in safety the contemplated voyage. The petitioners state that a mere shipper of cargo, having no control over the ship, has nothing to do with its seaworthiness. They argue that a cargo owner has no control over the structure of the ship, its cables, anchors, fuel and provisions, the manner of loading his cargo and the cargo of other shippers, and the hiring of a sufficient number of competent officers and seamen. The petitioners' arguments have no merit. There is no dispute over the liability of the common carrier Manila Bay. In fact, it did not bother to appeal the questioned decision. However, the petitioners state that Manila Bay has ceased operating as a firm and nothing may be recovered from it. They are, therefore, trying to recover their losses from the insurer. The liability of the insurance company is governed by law. Section 113 of the Insurance Code provides: In every marine insurance upon a ship or freight, or freightage, or upon any thing which is the subject of marine insurance, a warranty is implied that the ship is seaworthy. Section 99 of the same Code also provides in part. Marine insurance includes: (1) Insurance against loss of or damage to: (a) Vessels, craft, aircraft, vehicles, goods, freights, cargoes, merchandise, ... From the above-quoted provisions, there can be no mistaking the fact that the term "cargo" can be the subject of marine insurance and that once it is so made, the implied warranty of seaworthiness immediately attaches to whoever is insuring the cargo whether he be the shipowner or not.

As we have ruled in the case of Go Tiaoco y Hermanos v. Union Insurance Society of Canton (40 Phil. 40): The same conclusion must be reached if the question be discussed with reference to the seaworthiness of the ship. It is universally accepted that in every contract of insurance upon anything which is the subject of marine insurance, a warranty is implied that the ship shall be seaworthy at the time of the inception of the voyage. This rule is accepted in our own Insurance Law (Act No. 2427, sec. 106). ... Moreover, the fact that the unseaworthiness of the ship was unknown to the insured is immaterial in ordinary marine insurance and may not be used by him as a defense in order to recover on the marine insurance policy. As was held in Richelieu and Ontario Nav. Co. v. Boston Marine, Inc., Co. (136 U.S. 406): There was no look-out, and both that and the rate of speed were contrary to the Canadian Statute. The exception of losses occasioned by unseaworthiness was in effect a warranty that a loss should not be so occasioned, and whether the fact of unseaworthiness were known or unknown would be immaterial. Since the law provides for an implied warranty of seaworthiness in every contract of ordinary marine insurance, it becomes the obligation of a cargo owner to look for a reliable common carrier which keeps its vessels in seaworthy condition. The shipper of cargo may have no control over the vessel but he has full control in the choice of the common carrier that will transport his goods. Or the cargo owner may enter into a contract of insurance which specifically provides that the insurer answers not only for the perils of the sea but also provides for coverage of perils of the ship. We are constrained to apply Section 113 of the Insurance Code to the facts of this case. As stated by the private respondents: In marine cases, the risks insured against are "perils of the sea" (Chute v. North River Ins. Co., Minn214 NW 472, 55 ALR 933). The purpose of such insurance is protection against contingencies and against possible damages and such a policy does not cover a loss or injury which must inevitably take place in the ordinary course of things. There is no doubt that the term 'perils of the sea' extends only to losses caused by sea damage, or by the violence of the elements, and does not embrace all losses happening at sea. They insure against losses from extraordinary occurrences only, such as stress of weather, winds and waves, lightning, tempests, rocks and the like. These are understood to be the "perils of the sea" referred in the policy, and not those ordinary perils which every vessel must encounter. "Perils of the sea" has been said to include only

such losses as are of extraordinarynature, or arise from some overwhelming power, which cannot be guarded against by the ordinary exertion of human skill and prudence. Damage done to a vessel by perils of the sea includes every species of damages done to a vessel at sea, as distinguished from the ordinary wear and tear of the voyage, and distinct from injuries suffered by the vessel in consequence of her not being seaworthy at the outset of her voyage (as in this case). It is also the general rule that everything which happens thru the inherent vice of the thing, or by the act of the owners, master or shipper, shall not be reputed a peril, if not otherwise borne in the policy. (14 RCL on Insurance, Sec. 384, pp. 1203- 1204; Cia. de Navegacion v. Firemen's Fund Ins. Co., 277 US 66, 72 L. ed. 787, 48 S. Ct. 459). With regard to the second assignment of error, petitioners maintain, that the loss of the cargo was caused by the perils of the sea, not by the perils of the ship because as found by the trial court, the barge was turned loose from the tugboat east of Cabuli Point "where it was buffeted by storm and waves." Moreover, petitioners also maintain that barratry, against which the cargo was also insured, existed when the personnel of the tugboat and the barge committed a mistake by turning loose the barge from the tugboat east of Cabuli Point. The trial court also found that the stranding and foundering of Mable 10 was due to improper loading of the logs as well as to a leak in the barge which constituted negligence. On the contention of the petitioners that the trial court found that the loss was occasioned by the perils of the sea characterized by the "storm and waves" which buffeted the vessel, the records show that the court ruled otherwise. It stated: xxx xxx xxx ... The other affirmative defense of defendant Lighterage, 'That the supposed loss of the logs was occasioned by force majeure... "was not supported by the evidence. At the time Mable 10 sank, there was no typhoon but ordinary strong wind and waves, a condition which is natural and normal in the open sea. The evidence shows that the sinking of Mable 10 was due to improper loading of the logs on one side so that the barge was tilting on one side and for that it did not navigate on even keel; that it was no longer seaworthy that was why it developed leak; that the personnel of the tugboat and the barge committed a mistake when it turned loose the barge from the tugboat east of Cabuli point where it was buffeted by storm and waves, while the tugboat proceeded to west of Cabuli point where it was protected by the mountain side from the storm and waves coming from the east direction. ..." In fact, in the petitioners' complaint, it is alleged that "the barge Mable 10 of defendant carrier developed a leak which allowed water to come in and that one of the hatches of said barge was negligently left open by the person in charge thereof causing more

water to come in and that "the loss of said plaintiffs' cargo was due to the fault, negligence, and/or lack of skill of defendant carrier and/or defendant carrier's representatives on barge Mable 10." It is quite unmistakable that the loss of the cargo was due to the perils of the ship rather than the perils of the sea. The facts clearly negate the petitioners' claim under the insurance policy. In the case of Go Tiaoco y Hermanos v. Union Ins. Society of Canton, supra, we had occasion to elaborate on the term "perils of the ship." We ruled: It must be considered to be settled, furthermore, that a loss which, in the ordinary course of events, results from the natural and inevitable action of the sea, from the ordinary wear and tear of the ship, or from the negligent failure of the ship's owner to provide the vessel with proper equipment to convey the cargo under ordinary conditions, is not a peril of the sea. Such a loss is rather due to what has been aptly called the "peril of the ship." The insurer undertakes to insure against perils of the sea and similar perils, not against perils of the ship. As was well said by Lord Herschell in Wilson, Sons & Co. v. Owners of Cargo per the Xantho ([1887], 12 A. C., 503, 509), there must, in order to make the insurer liable, be some casualty, something which could not be foreseen as one of the necessary incidents of the adventure. The purpose of the policy is to secure an indemnity against accidents which may happen, not against events which must happen. In the present case the entrance of the sea water into the ship's hold through the defective pipe already described was not due to any accident which happened during the voyage, but to the failure of the ship's owner properly to repair a defect of the existence of which he was apprised. The loss was therefore more analogous to that which directly results from simple unseaworthiness than to that which result from the perils of the sea. xxx xxx xxx Suffice it to say that upon the authority of those cases there is no room to doubt the liability of the shipowner for such a loss as occurred in this case. By parity of reasoning the insurer is not liable; for generally speaking, the shipowner excepts the perils of the sea from his engagement under the bill of lading, while this is the very perils against which the insurer intends to give protection. As applied to the present case it results that the owners of the damaged rice must look to the shipowner for redress and not to the insurer. Neither can petitioners allege barratry on the basis of the findings showing negligence on the part of the vessel's crew.

Barratry as defined in American Insurance Law is "any willful misconduct on the part of master or crew in pursuance of some unlawful or fraudulent purpose without the consent of the owners, and to the prejudice of the owner's interest." (Sec. 171, U.S. Insurance Law, quoted in Vance, Handbook on Law of Insurance, 1951, p. 929.) Barratry necessarily requires a willful and intentional act in its commission. No honest error of judgment or mere negligence, unless criminally gross, can be barratry. (See Vance on Law of Insurance, p. 929 and cases cited therein.) In the case at bar, there is no finding that the loss was occasioned by the willful or fraudulent acts of the vessel's crew. There was only simple negligence or lack of skill. Hence, the second assignment of error must likewise be dismissed. Anent the third assignment of error, we agree with the petitioners that the amount of P8,000.00 representing the amount of the salvaged logs should have been awarded to them. However, this should be deducted from the amounts which have been adjudicated against Manila Bay Lighterage Corporation by the trial court. WHEREFORE, the decision appealed from is AFFIRMED with the modification that the amount of P8,000.00 representing the value of the salvaged logs which was ordered to be deposited in the Manila Banking Corporation in the name of Civil Case No. 86599 is hereby awarded and ordered paid to the petitioners. The liability adjudged against Manila Bay Lighterage Corporation in the decision of the trial court is accordingly reduced by the same amount. SO ORDERED. Teehankee (Chairman), Melencio-Herrera, Plana, De la Fuente and Patajo, JJ., concur. Relova, J., is on leave. G.R. No. 85141 November 28, 1989 FILIPINO MERCHANTS INSURANCE CO., INC., petitioner, vs. COURT OF APPEALS and CHOA TIEK SENG, respondents. Balgos & Perez Law Offices for petitioner. Lapuz Law office for private respondent.

REGALADO, J.:

This is a review of the decision of the Court of Appeals, promulgated on July 19,1988, the dispositive part of which reads: WHEREFORE, the judgment appealed from is affirmed insofar as it orders defendant Filipino Merchants Insurance Company to pay the plaintiff the sum of P51,568.62 with interest at legal rate from the date of filing of the complaint, and is modified with respect to the third party complaint in that (1) third party defendant E. Razon, Inc. is ordered to reimburse third party plaintiff the sum of P25,471.80 with legal interest from the date of payment until the date of reimbursement, and (2) the third-party complaint against third party defendant Compagnie Maritime Des Chargeurs Reunis is dismissed. 1 The facts as found by the trial court and adopted by the Court of Appeals are as follows: This is an action brought by the consignee of the shipment of fishmeal loaded on board the vessel SS Bougainville and unloaded at the Port of Manila on or about December 11, 1976 and seeks to recover from the defendant insurance company the amount of P51,568.62 representing damages to said shipment which has been insured by the defendant insurance company under Policy No. M-2678. The defendant brought a third party complaint against third party defendants Compagnie Maritime Des Chargeurs Reunis and/or E. Razon, Inc. seeking judgment against the third (sic) defendants in case Judgment is rendered against the third party plaintiff. It appears from the evidence presented that in December 1976, plaintiff insured said shipment with defendant insurance company under said cargo Policy No. M-2678 for the sum of P267,653.59 for the goods described as 600 metric tons of fishmeal in new gunny bags of 90 kilos each from Bangkok, Thailand to Manila against all risks under warehouse to warehouse terms. Actually, what was imported was 59.940 metric tons not 600 tons at $395.42 a ton CNF Manila. The fishmeal in 666 new gunny bags were unloaded from the ship on December 11, 1976 at Manila unto the arrastre contractor E. Razon, Inc. and defendant's surveyor ascertained and certified that in such discharge 105 bags were in bad order condition as jointly surveyed by the ship's agent and the arrastre contractor. The condition of the bad order was reflected in the turn over survey report of Bad Order cargoes Nos. 120320 to 120322, as Exhibit C4 consisting of three (3) pages which are also Exhibits 4, 5 and 6- Razon. The cargo was also surveyed by the arrastre contractor before delivery of the cargo to the consignee and the condition of the cargo on such delivery was reflected in E. Razon's Bad Order Certificate No. 14859, 14863 and 14869 covering a total of 227 bags in bad order condition. Defendant's surveyor has conducted a final and detailed survey of the cargo in the warehouse for which he prepared a survey report Exhibit F with the findings on the extent of shortage or loss on the bad order bags totalling 227 bags amounting to 12,148 kilos, Exhibit F-1. Based on said

computation the plaintiff made a formal claim against the defendant Filipino Merchants Insurance Company for P51,568.62 (Exhibit C) the computation of which claim is contained therein. A formal claim statement was also presented by the plaintiff against the vessel dated December 21, 1976, Exhibit B, but the defendant Filipino Merchants Insurance Company refused to pay the claim. Consequently, the plaintiff brought an action against said defendant as adverted to above and defendant presented a third party complaint against the vessel and the arrastre contractor. 2 The court below, after trial on the merits, rendered judgment in favor of private respondent, the decretal portion whereof reads: WHEREFORE, on the main complaint, judgment is hereby rendered in favor of the plaintiff and against the defendant Filipino Merchant's (sic) Insurance Co., ordering the defendants to pay the plaintiff the following amount: The sum of P51,568.62 with interest at legal rate from the date of the filing of the complaint; On the third party complaint, the third party defendant Compagnie Maritime Des Chargeurs Reunis and third party defendant E. Razon, Inc. are ordered to pay to the third party plaintiff jointly and severally reimbursement of the amounts paid by the third party plaintiff with legal interest from the date of such payment until the date of such reimbursement. Without pronouncement as to costs. 3 On appeal, the respondent court affirmed the decision of the lower court insofar as the award on the complaint is concerned and modified the same with regard to the adjudication of the third-party complaint. A motion for reconsideration of the aforesaid decision was denied, hence this petition with the following assignment of errors: 1. The Court of Appeals erred in its interpretation and application of the "all risks" clause of the marine insurance policy when it held the petitioner liable to the private respondent for the partial loss of the cargo, notwithstanding the clear absence of proof of some fortuitous event, casualty, or accidental cause to which the loss is attributable, thereby contradicting the very precedents cited by it in its decision as well as a prior decision of the same Division of the said court (then composed of Justices Cacdac, Castro-Bartolome, and Pronove); 2. The Court of Appeals erred in not holding that the private respondent had no insurable interest in the subject cargo, hence, the marine insurance policy taken out by private respondent is null and void;

3. The Court of Appeals erred in not holding that the private respondent was guilty of fraud in not disclosing the fact, it being bound out of utmost good faith to do so, that it had no insurable interest in the subject cargo, which bars its recovery on the policy. 4 On the first assignment of error, petitioner contends that an "all risks" marine policy has a technical meaning in insurance in that before a claim can be compensable it is essential that there must be "some fortuity, " "casualty" or "accidental cause" to which the alleged loss is attributable and the failure of herein private respondent, upon whom lay the burden, to adduce evidence showing that the alleged loss to the cargo in question was due to a fortuitous event precludes his right to recover from the insurance policy. We find said contention untenable. The "all risks clause" of the Institute Cargo Clauses read as follows: 5. This insurance is against all risks of loss or damage to the subjectmatter insured but shall in no case be deemed to extend to cover loss, damage, or expense proximately caused by delay or inherent vice or nature of the subject-matter insured. Claims recoverable hereunder shall be payable irrespective of percentage. 5 An "all risks policy" should be read literally as meaning all risks whatsoever and covering all losses by an accidental cause of any kind. The terms "accident" and "accidental", as used in insurance contracts, have not acquired any technical meaning. They are construed by the courts in their ordinary and common acceptance. Thus, the terms have been taken to mean that which happens by chance or fortuitously, without intention and design, and which is unexpected, unusual and unforeseen. An accident is an event that takes place without one's foresight or expectation; an event that proceeds from an unknown cause, or is an unusual effect of a known cause and, therefore, not expected. 6 The very nature of the term "all risks" must be given a broad and comprehensive meaning as covering any loss other than a willful and fraudulent act of the insured. 7 This is pursuant to the very purpose of an "all risks" insurance to give protection to the insured in those cases where difficulties of logical explanation or some mystery surround the loss or damage to property. 8 An "all asks" policy has been evolved to grant greater protection than that afforded by the "perils clause," in order to assure that no loss can happen through the incidence of a cause neither insured against nor creating liability in the ship; it is written against all losses, that is, attributable to external causes. 9 The term "all risks" cannot be given a strained technical meaning, the language of the clause under the Institute Cargo Clauses being unequivocal and clear, to the effect that it extends to all damages/losses suffered by the insured cargo except (a) loss or damage or expense proximately caused by delay, and (b) loss or damage or expense proximately caused by the inherent vice or nature of the subject matter insured.

Generally, the burden of proof is upon the insured to show that a loss arose from a covered peril, but under an "all risks" policy the burden is not on the insured to prove the precise cause of loss or damage for which it seeks compensation. The insured under an "all risks insurance policy" has the initial burden of proving that the cargo was in good condition when the policy attached and that the cargo was damaged when unloaded from the vessel; thereafter, the burden then shifts to the insurer to show the exception to the coverage. 10 As we held in Paris-Manila Perfumery Co. vs. Phoenix Assurance Co., Ltd. 11 the basic rule is that the insurance company has the burden of proving that the loss is caused by the risk excepted and for want of such proof, the company is liable. Coverage under an "all risks" provision of a marine insurance policy creates a special type of insurance which extends coverage to risks not usually contemplated and avoids putting upon the insured the burden of establishing that the loss was due to the peril falling within the policy's coverage; the insurer can avoid coverage upon demonstrating that a specific provision expressly excludes the loss from coverage. 12 A marine insurance policy providing that the insurance was to be "against all risks" must be construed as creating a special insurance and extending to other risks than are usually contemplated, and covers all losses except such as arise from the fraud of the insured. 13 The burden of the insured, therefore, is to prove merely that the goods he transported have been lost, destroyed or deteriorated. Thereafter, the burden is shifted to the insurer to prove that the loss was due to excepted perils. To impose on the insured the burden of proving the precise cause of the loss or damage would be inconsistent with the broad protective purpose of "all risks" insurance. In the present case, there being no showing that the loss was caused by any of the excepted perils, the insurer is liable under the policy. As aptly stated by the respondent Court of Appeals, upon due consideration of the authorities and jurisprudence it discussed ... it is believed that in the absence of any showing that the losses/damages were caused by an excepted peril, i.e. delay or the inherent vice or nature of the subject matter insured, and there is no such showing, the lower court did not err in holding that the loss was covered by the policy. There is no evidence presented to show that the condition of the gunny bags in which the fishmeal was packed was such that they could not hold their contents in the course of the necessary transit, much less any evidence that the bags of cargo had burst as the result of the weakness of the bags themselves. Had there been such a showing that spillage would have been a certainty, there may have been good reason to plead that there was no risk covered by the policy (See Berk vs. Style [1956] cited in Marine Insurance Claims, Ibid, p. 125). Under an 'all risks' policy, it was sufficient to show that there was damage occasioned by some accidental cause of any kind, and there is no necessity to point to any particular cause. 14

Contracts of insurance are contracts of indemnity upon the terms and conditions specified in the policy. The agreement has the force of law between the parties. The terms of the policy constitute the measure of the insurer's liability. If such terms are clear and unambiguous, they must be taken and understood in their plain, ordinary and popular sense. 15 Anent the issue of insurable interest, we uphold the ruling of the respondent court that private respondent, as consignee of the goods in transit under an invoice containing the terms under "C & F Manila," has insurable interest in said goods. Section 13 of the Insurance Code defines insurable interest in property as every interest in property, whether real or personal, or any relation thereto, or liability in respect thereof, of such nature that a contemplated peril might directly damnify the insured. In principle, anyone has an insurable interest in property who derives a benefit from its existence or would suffer loss from its destruction whether he has or has not any title in, or lien upon or possession of the property y. 16 Insurable interest in property may consist in (a) an existing interest; (b) an inchoate interest founded on an existing interest; or (c) an expectancy, coupled with an existing interest in that out of which the expectancy arises. 17 Herein private respondent, as vendee/consignee of the goods in transit has such existing interest therein as may be the subject of a valid contract of insurance. His interest over the goods is based on the perfected contract of sale. 18 The perfected contract of sale between him and the shipper of the goods operates to vest in him an equitable title even before delivery or before be performed the conditions of the sale. 19 The contract of shipment, whether under F.O.B., C.I.F., or C. & F. as in this case, is immaterial in the determination of whether the vendee has an insurable interest or not in the goods in transit. The perfected contract of sale even without delivery vests in the vendee an equitable title, an existing interest over the goods sufficient to be the subject of insurance. Further, Article 1523 of the Civil Code provides that where, in pursuance of a contract of sale, the seller is authorized or required to send the goods to the buyer, delivery of the goods to a carrier, whether named by the buyer or not, for, the purpose of transmission to the buyer is deemed to be a delivery of the goods to the buyer, the exceptions to said rule not obtaining in the present case. The Court has heretofore ruled that the delivery of the goods on board the carrying vessels partake of the nature of actual delivery since, from that time, the foreign buyers assumed the risks of loss of the goods and paid the insurance premium covering them. 20 C & F contracts are shipment contracts. The term means that the price fixed includes in a lump sum the cost of the goods and freight to the named destination. 21 It simply means that the seller must pay the costs and freight necessary to bring the goods to the named destination but the risk of loss or damage to the goods is transferred from the seller to the buyer when the goods pass the ship's rail in the port of shipment. 22

Moreover, the issue of lack of insurable interest was not among the defenses averred in petitioners answer. It was neither an issue agreed upon by the parties at the pre-trial conference nor was it raised during the trial in the court below. It is a settled rule that an issue which has not been raised in the court a quo cannot be raised for the first time on appeal as it would be offensive to the basic rules of fair play, justice and due process. 23 This is but a permuted restatement of the long settled rule that when a party deliberately adopts a certain theory, and the case is tried and decided upon that theory in the court below, he will not be permitted to change his theory on appeal because, to permit him to do so, would be unfair to the adverse party. 24 If despite the fundamental doctrines just stated, we nevertheless decided to indite a disquisition on the issue of insurable interest raised by petitioner, it was to put at rest all doubts on the matter under the facts in this case and also to dispose of petitioner's third assignment of error which consequently needs no further discussion. WHEREFORE, the instant petition is DENIED and the assailed decision of the respondent Court of Appeals is AFFIRMED in toto. SO ORDERED. Paras, Padilla and Sarmiento, JJ., concur. Melencio-Herrera (Chairperson), J., is on leave.

G.R. No. 84507 March 15, 1990 CHOA TIEK SENG, doing business under the name and style of SENG'S COMMERCIAL ENTERPRISES,petitioner, vs. HON. COURT OF APPEALS, FILIPINO MERCHANTS' INSURANCE COMPANY, INC., BEN LINES CONTAINER, LTD. AND E. RAZON, INC., respondents. Lapuz Law Office for petitioner. De Santos, Balgoz & Perez for respondent Filipino Merchants' Insurance Company, Inc. Marilyn Cacho-Noe for respondent Ben Lines Container, Ltd.

GANCAYCO, J.: This is an appeal from a decision of the Court of Appeals dated February 18, 1988 in CA-G.R. CV No. 09627 which affirmed the decision of the Regional Trial Court (RTC) of Manila which in turn dismissed the complaint. 1 On November 4, 1976 petitioner imported some lactose crystals from Holland. The importation involved fifteen (15) metric tons packed in 600 6-ply paper bags with polythelene inner bags, each bag at 25 kilos net. The goods were loaded at the port at Rotterdam in sea vans on board the vessel "MS Benalder' as the mother vessel, and thereafter aboard the feeder vessel "Wesser Broker V-25" of respondent Ben Lines Container, Ltd. (Ben Lines for short). The goods were insured by the respondent Filipino Merchants' Insurance Co., Inc. (insurance company for short) for the sum of P98,882.35, the equivalent of US$8,765.00 plus 50% mark-up or US$13,147.50, against all risks under the terms of the insurance cargo policy. Upon arrival at the port of Manila, the cargo was discharged into the custody of the arrastre operator respondent E. Razon, Inc. (broker for short), prior to the delivery to petitioner through his broker. Of the 600 bags delivered to petitioner, 403 were in bad order. The surveys showed that the bad order bags suffered spillage and loss later valued at P33,117.63. Petitioner filed a claim for said loss dated February 16, 1977 against respondent insurance company in the amount of P33,117.63 as the insured value of the loss. Respondent insurance company rejected the claim alleging that assuming that spillage took place while the goods were in transit, petitioner and his agent failed to avert or minimize the loss by failing to recover spillage from the sea van, thus violating the terms of the insurance policy sued upon; and that assuming that the spillage did not occur while the cargo was in transit, the said 400 bags were loaded in bad order, and that in any case, the van did not carry any evidence of spillage. Hence, petitioner filed the complaint dated August 2, 1977 in the Regional Trial Court of Manila against respondent insurance company seeking payment of the sum of P33,117.63 as damages plus attorney's fees and expenses of litigation. In its answer, respondent insurance company denied all the material allegations of the complaint and raised several special defenses as well as a compulsory counterclaim. On February 24, 1978, respondent insurance company filed a third-party complaint against respondents Ben Lines and broker. Respondent broker filed its answer to the third-party complaint denying liability and arguing, among others, that the petitioner has no valid cause of action against it. Similarly, Ben Lines filed its answer denying any liability and a special defense arguing that respondent insurance company was not the proper party in interest and has no connection whatsoever with Ben Lines Containers, Ltd. and that the third-party complaint has prescribed under the applicable provisions of the Carriage of Goods by Sea Act.

On November 6, 1979, respondent Ben Lines filed a motion for preliminary hearing on the affirmative defense of prescription. In an order dated February 28, 1980, the trial court deferred resolution of the aforesaid motion after trial on the ground that the defense of prescription did not appear to be indubitable. After the pre-trial conference and trial on the merits, on March 31, 1986, the court a quo rendered a judgment dismissing the complaint, the counterclaim and the third-party complaint with costs against the petitioner. Hence, the appeal to the Court of Appeals by petitioner which, in due course, as aforestated, affirmed the judgment of the trial court. A motion for reconsideration of said judgment was denied by the appellate court in a resolution dated August 1, 1988. Petitioner now filed this petition for review on certiorari in this Court predicated on the following grounds: I RESPONDENT COURT ERRED IN HOLDING THAT THE INSURED SHIPMENT DID NOT SUSTAIN ANY DAMAGE/LOSS DESPITE ADMISSION THEREOF ON THE PART OF RESPONDENT INSURANCE COMPANY AND THE FINDING OF THE LATTER'S SURVEYORS. II RESPONDENT COURT ERRED IN HOLDING THAT AN "ALL RISKS" COVERAGE COVERS ONLY LOSSES OCCASIONED BY OR RESULTING FROM "EXTRA AND FORTUITOUS EVENTS" DESPITE THE CLEAR AND UNEQUIVOCAL DEFINITION OF THE TERM MADE AND CONTAINED IN THE POLICY SUED UPON. III THE HOLDING OF RESPONDENT COURT THAT AN "ALL RISKS" COVERAGE COVERS LOSSES OCCASIONED BY AND RESULTING FROM "EXTRA AND FORTUITOUS EVENTS" CONTRADICTS THE RULING OF THE SAME COURT IN ANOTHER CASE WHERE THE DEFINITION OF THE TERM "ALL RISKS"/ STATED IN THE POLICY WAS MADE TO CONTROL HENCE THE NEED FOR REVIEW. 2 The petition is impressed with merit. The appellate court, in arriving at the conclusion that there was no damage suffered by the cargo at the time of the devanning thereof, held as follows:

Appellant argued that the cargo in question sustained damages while still in the possession of the carrying vessel, because as his appointed surveyor reported, Worldwide Marine Survey Corporation, at the time of devanning at the pier, 403 bags were already in bad order and condition. Appellant found support to this contention on the basis of the survey report of Worldwide Marine Survey Corporation of the Philippines and of the Adjustment Corporation of the Philippines which were identified by his sole witness, Jose See. It must be pointed out, however, that witness Jose See was incompetent to identify the two survey reports because he was not actually present during the actual devanning of the cargo, which fact was admitted by him, hence, he failed to prove the authenticity of the aforesaid survey reports. On the other hand, the evidence submitted by the appellee would conclusively establish the fact that there was no damage suffered by the subject cargo at the time of the devanning thereof. The cargo, upon discharge from the vessel, was delivered to the custody of the arrastre operator (E. Razon) under clean tally sheet (Exh. 6-FMIC). Moreover, the container van containing the cargo was found with both its seal and lock intact. Article IV, paragraph 4 of the Management Contract (Exh. 5) signed between the Bureau of Customs and the Arrastre Operator provides: 4. Tally Sheets for Cargo Vans or Containers The contractor shall give a clean tally sheet for cargo vans received by it in good order and condition with locks, and seals intact. The same cargo was in turn delivered into the possession of the appellant by the arrastre operator at the pier in good order and condition as shown by the clean gate passes (Exhs. 2 and 3) and the delivery permit (Exh. 4). The clean gate passes were issued by appellee arrastre operator covering the shipment in question, with the conformity of the appellant's representative. The clean gate passes provide in part: . . . issuance of this Gate Pass constitutes delivery to and receipt by consignee of the goods as described above, in good order and condition, unless an accompanying B.O. (Bad Order) Certificate duly issued and noted on the face of this Gate Pass appears. These clean gate passes are undoubtedly important and vital pieces of evidence. They are noted in the dorsal side of another important piece of document which is the permit to deliver (Exh. 4) issued by the Bureau of Customs to effect delivery of the cargo to the consignee. The significance and value of these documents is that they bind the shipping company and the arrastre operator whenever a cargo sustains damage while in their

respective custody. It is worthy of note that there was no turn over survey executed between the vessel and the arrastre operator, indicating any damage to the cargo upon discharge from the custody of the vessel. There was no bad order certificate issued by the appellee arrastre operator, indicating likewise that there was no damage to the cargo while in its custody. It is surprising to the point that one could not believe that if indeed there was really damage affecting the 403 bags out of the 600, with an alleged estimated spillage of 240%, this purportedly big quantity of spillage was never recovered which could have been easily done considering that the shipment was in a container van which was found to be sealed and intact. 3 However, in the same decision of the appellate court, the following evidence of the petitioner on this aspect was summarized as follows: The 600 bags which the original carrier received in apparent good order condition and certified to by the vessel's agent to be weighing 15,300 kg. gross, were unloaded from the transhipment vessel "Wesser Broker" stuffed in one container and turned over to the arrastre operator, third party defendant-appellee E. Razon, Inc. A shipboard surveyor, the Worldwide Marine Cargo Surveyor, as well as a representative of the vessel "Wesser Broker" and a representative of the arrastre operator attended the devanning of the shipment and the said shipboard surveyor certified that 403 bags were in bad order condition with estimated spillage as follows: 65 P/bags each of 20% 78 P/bags each of 35% 79 P/bags each of 45% 87 P/bags each of 65% 94 P/bags each of 75% (Exh. F-1) Defendant and third-party plaintiff-appellee's protective surveyor determined the exact spillage from the bad order bags as found by the shipboard surveyor at the consignee's warehouse by weighing the bad order bags. Said protective surveyor found after weighing the 403 bags in bad order condition that an aggregate of 5,173 kilos were missing therefrom (Exh. F). 4 The assertion of the appellate court that the authenticity of the survey reports of the Worldwide Marine Cargo Survey Corporation and the Adjustment Corporation of the Philippines were not established as Jose See who identified the same was incompetent

as he was not actually present during the actual devanning of the cargo is not well taken. In the first place it was respondent insurance company which undertook the protective survey aforestated relating to the goods from the time of discharge up to the time of delivery thereof to the consignee's warehouse, so that it is bound by the report of its surveyor which is the Adjustment Corporation of the Philippines. 5 The Worldwide Marine Cargo Survey Corporation of the Philippines was the vessel's surveyor. The survey report of the said Adjustment Corporation of the Philippines reads as follows: During the turn-over of the contents delivery from the cargo sea van by the representative of the shipping agent to consignee's representative/ Broker (Saint Rose Forwarders), 403 bags were bursted and/or torn, opened on one end contents partly spilled. The same were inspected by thevessel's surveyor (Worldwide Marine & Cargo Survey Corporation), findings as follows: One (1) Container No. 2987789 Property locked and secured with Seal No. 18880. FOUND: 197-Paper Bags (6-Ply each with One inner Plastic Lining Machine Stitched with cotton Twine on Both ends. Containing Lactose Crystal 25 mesh Sep 061-09-03 in good order. 403-Bags, 6-ply torn and/or opened on one end, contents partly spilled, estimated spillages as follows: 65 P/bags each of 20% 78 P/bags each of 35% 79 P/bags each of 45% 87 P/bags each of 65% 94 P/bags each of 75% (emphasis supplied) 6 The authenticity of the said survey report need not be established in evidence as it is binding on respondent insurance company who caused said protective survey. Secondly, contrary to the findings of the appellate court that petitioner's witness Jose See was not present at the time of the actual devanning of the cargo, what the record shows is that he was present when the cargo was unloaded and received in the warehouse of the consignee. He saw 403 bags to be in bad order. Present then was the surveyor, Adjustment Corporation of the Philippines, who surveyed the cargo by

segregating the bad order cargo from the good order and determined the amount of loss. 7 Thus, said witness was indeed competent to identify the survey report aforestated. Thirdly, in its letter dated May 26, 1977 to petitioner, respondent insurance company admitted in no uncertain terms, the damages as indicated in the survey report in this manner: We do not question the fact that out of the 600 bags shipment 403 bags appeared to be in bad order or in damaged condition as indicated in the survey report of the vessel surveyor. . . . 8 This admission even standing alone is sufficient proof of loss or damage to the cargo. The appellate court observed that the cargo was discharged from the vessel and delivered to the custody of the broker under the clean tally sheet, that the container van containing the cargo was found with both its seal and lock intact; and that the cargo was delivered to the possession of the petitioner by the broker in good order and condition as shown by the clean gate passes and delivery permit. The clean tally sheet referred to by the appellate court covers the van container and not the cargo stuffed therein.9 The appellate court clearly stated that the clean tally sheet issued by the broker covers the cargo vans received by it in good order and condition with lock and seal intact. Said tally sheet is no evidence of the condition of the cargo therein contained. Even the witness of the respondent insurance company, Sergio Icasiano, stated that the clean gate passes do not reflect the actual condition of the cargo when released by the broker as it was not physically examined by the broker. 10 There is no question, therefore, that there were 403 bags in damaged condition delivered and received by petitioner. Nevertheless, on the assumption that the cargo suffered damages, the appellate court ruled: Even assuming that the cargo indeed sustained damage, still the appellant cannot hold the appellee insurance company liable on the insurance policy. In the case at bar, appellant failed to prove that the alleged damage was due to risks connected with navigation. A distinction should be made between "perils of the sea" which render the insurer liable on account of the loss and/or damage brought about thereof and "perils of the ship" which do not render the insurer liable for any loss or damage. Perils of the sea or perils of navigation embrace all kinds of marine casualties, such as shipwreck, foundering, stranding, collision and every specie of damage done to the ship or goods at sea by the violent action of the winds or waves. They do not embrace all loses happening on the sea. A peril whose only connection with the sea is that it arises aboard ship is not

necessarily a peril of the sea; the peril must be of the sea and not merely one accruing on the sea (The Phil. Insurance Law, by Guevarra, 4th ed., 1961, p. 143). In Wilson, Sons and Co. vs. Owners of Cargo per the Xantho(1887) A.C. 503, 508, it was held: There must, in order to make the insurer liable be "some casualty," something which could not be foreseen as one of the necessary incidents of the adventure. The purpose of the policy is to secure an indemnity against accidents which may happen, not against events which must happen. Moreover, the cargo in question was insured in an "against all risk policy." Insurance "against all risk" has a technical meaning in marine insurance. Under an "all risk" marine policy, there must be a general rule be a fortuitous event in order to impose liability on the insurer; losses occasioned by ordinary circumstances or wear and tear are not covered, thus, while an "all risk" marine policy purports to cover losses from casualties at sea, it does not cover losses occasioned by the ordinary circumstances of a voyage, but only those resulting from extra and fortuitous events. It has been held that damage to a cargo by high seas and other weather is not covered by an "all risk" marine policy, since it is not fortuitous, particularly where the bad weather occurs at a place where it could be expected at the time in question. (44 Am. Jur. 2d. 216) In Go Tiaoco y Hermanas vs.Union Insurance Society of Canto, 40 Phil. 40, it was held: In the present case, the entrance of the sea water into the ship's hold through the defective pipe already described was not due to any accident which happened during the voyage, but to the failure of the ship's owner properly to repair a defect of the existence of which he was apprised. The loss was therefore more analogous to that which directly results from simple unseaworthiness than to that whose results, from perils of the sea. 11 The Court disagrees. In Gloren Inc. vs. Filipinas Cia. de Seguros, 12 it was held that an all risk insurance policy insures against all causes of conceivable loss or damage, except as otherwise excluded in the policy or due to fraud or intentional misconduct on the part of the insured. It covers all losses during the voyage whether arising from a marine peril or not, including pilferage losses during the war. In the present case, the "all risks" clause of the policy sued upon reads as follows:

5. This insurance is against all risks of loss or damage to the subject matter insured but shall in no case be deemed to extend to cover loss, damage, or expense proximately caused by delay or inherent vice or nature of the subject matter insured. Claims recoverable hereunder shall be payable irrespective of percentage. 13 The terms of the policy are so clear and require no interpretation. The insurance policy covers all loss or damage to the cargo except those caused by delay or inherent vice or nature of the cargo insured. It is the duty of the respondent insurance company to establish that said loss or damage falls within the exceptions provided for by law, otherwise it is liable therefor. An "all risks" provision of a marine policy creates a special type of insurance which extends coverage to risks not usually contemplated and avoids putting upon the insured the burden of establishing that the loss was due to peril falling within the policy's coverage. The insurer can avoid coverage upon demonstrating that a specific provision expressly excludes the loss from coverage. 14 In this case, the damage caused to the cargo has not been attributed to any of the exceptions provided for nor is there any pretension to this effect. Thus, the liability of respondent insurance company is clear. WHEREFORE, the decision appealed from is hereby REVERSED AND SET ASIDE and another judgment is hereby rendered ordering the respondent Filipinas Merchants Insurance Company, Inc. to pay the sum of P33,117.63 as damages to petitioner with legal interest from the filing of the complaint, plus attorney's fees and expenses of litigation in the amount of P10,000.00 as well as the costs of the suit. SO ORDERED. Narvasa, Cruz, Grio-Aquino and Medialdea, JJ., concur.

G.R. No. 127897

November 15, 2001

DELSAN TRANSPORT LINES, INC., petitioner, vs. THE HON. COURT OF APPEALS and AMERICAN HOME ASSURANCE CORPORATION, respondents. DE LEON, JR., J.:

Before us is a petition for review on certiorari of the Decision1 of the Court of Appeals in CA-G.R. CV No. 39836 promulgated on June 17, 1996, reversing the decision of the Regional Trial Court of Makati City, Branch 137, ordering petitioner to pay private respondent the sum of Five Million Ninety-Six Thousand Six Hundred Thirty-Five Pesos and Fifty-Seven Centavos (P5,096,635.57) and costs and the Resolution 2 dated January 21, 1997 which denied the subsequent motion for reconsideration. The facts show that Caltex Philippines (Caltex for brevity) entered into a contract of affreightment with the petitioner, Delsan Transport Lines, Inc., for a period of one year whereby the said common carrier agreed to transport Caltexs industrial fuel oil from the Batangas-Bataan Refinery to different parts of the country. Under the contract, petitioner took on board its vessel, MT Maysun 2,277.314 kiloliters of industrial fuel oil of Caltex to be delivered to the Caltex Oil Terminal in Zamboanga City. The shipment was insured with the private respondent, American Home Assurance Corporation. On August 14, 1986, MT Maysum set sail from Batangas for Zamboanga City. Unfortunately, the vessel sank in the early morning of August 16, 1986 near Panay Gulf in the Visayas taking with it the entire cargo of fuel oil. Subsequently, private respondent paid Caltex the sum of Five Million Ninety-Six Thousand Six Hundred Thirty-Five Pesos and Fifty-Seven Centavos (P5,096,635.67) representing the insured value of the lost cargo. Exercising its right of subrogation under Article 2207 of the New Civil Code, the private respondent demanded of the petitioner the same amount it paid to Caltex.1wphi1.nt Due to its failure to collect from the petitioner despite prior demand, private respondent filed a complaint with the Regional Trial Court of Makati City, Branch 137, for collection of a sum of money. After the trial and upon analyzing the evidence adduced, the trial court rendered a decision on November 29, 1990 dismissing the complaint against herein petitioner without pronouncement as to cost. The trial court found that the vessel, MT Maysum, was seaworthy to undertake the voyage as determined by the Philippine Coast Guard per Survey Certificate Report No. M5-016-MH upon inspection during its annual dry-docking and that the incident was caused by unexpected inclement weather condition or force majeure, thus exempting the common carrier (herein petitioner) from liability for the loss of its cargo.3 The decision of the trial court, however, was reversed, on appeal, by the Court of Appeals. The appellate court gave credence to the weather report issued by the Philippine Atmospheric, Geophysical and Astronomical Services Administration (PAGASA for brevity) which showed that from 2:00 oclock to 8:oo oclock in the morning on August 16, 1986, the wind speed remained at 10 to 20 knots per hour while the waves measured from .7 to two (2) meters in height only in the vicinity of the Panay Gulf where the subject vessel sank, in contrast to herein petitioners allegation that the waves were twenty (20) feet high. In the absence of any explanation as to what may have caused the sinking of the vessel coupled with the finding that the same was improperly manned, the appellate court ruled that the petitioner is liable on its obligation

as common carrier4 to herein private respondent insurance company as subrogee of Caltex. The subsequent motion for reconsideration of herein petitioner was denied by the appellate court. Petitioner raised the following assignments of error in support of the instant petition, 5 to wit: I THE COURT OF APPEALS ERRED IN REVERSING THE DECISION OF THE REGIONAL TRIAL COURT. II THE COURT OF APPEALS ERRED AND WAS NOT JUSTIFIED IN REBUTTING THE LEGAL PRESUMPTION THAT THE VESSEL MT "MAYSUN" WAS SEAWORTHY. III THE COURT OF APPEALS ERRED IN NOT APPLYING THE DOCTRINE OF THE SUPREME COURT IN THE CASE OF HOME INSURANCE CORPORATION V. COURT OF APPEALS. Petitioner Delsan Transport Lines, Inc. invokes the provision of Section 113 of the Insurance Code of the Philippines, which states that in every marine insurance upon a ship or freight, or freightage, or upon any thin which is the subject of marine insurance there is an implied warranty by the shipper that the ship is seaworthy. Consequently, the insurer will not be liable to the assured for any loss under the policy in case the vessel would later on be found as not seaworthy at the inception of the insurance. It theorized that when private respondent paid Caltex the value of its lost cargo, the act of the private respondent is equivalent to a tacit recognition that the ill-fated vessel was seaworthy; otherwise, private respondent was not legally liable to Caltex due to the latters breach of implied warranty under the marine insurance policy that the vessel was seaworthy. The petitioner also alleges that the Court of Appeals erred in ruling that MT Maysun was not seaworthy on the ground that the marine officer who served as the chief mate of the vessel, Francisco Berina, was allegedly not qualified. Under Section 116 of the Insurance Code of the Philippines, the implied warranty of seaworthiness of the vessel, which the private respondent admitted as having been fulfilled by its payment of the insurance proceeds to Caltex of its lost cargo, extends to the vessels complement. Besides, petitioner avers that although Berina had merely a 2nd officers license, he was qualified to act as the vessels chief officer under Chapter IV(403), Category III(a)(3)(ii)(aa) of the Philippine Merchant Marine Rules and Regulations. In fact, all the

crew and officers of MT Maysun were exonerated in the administrative investigation conducted by the Board of Marine Inquiry after the subject accident.6 In any event, petitioner further avers that private respondent failed, for unknown reason, to present in evidence during the trial of the instant case the subject marine cargo insurance policy it entered into with Caltex. By virtue of the doctrine laid down in the case of Home Insurance Corporation vs. CA,7 the failure of the private respondent to present the insurance policy in evidence is allegedly fatal to its claim inasmuch as there is no way to determine the rights of the parties thereto. Hence, the legal issues posed before the Court are: I Whether or not the payment made by the private respondent to Caltex for the insured value of the lost cargo amounted to an admission that the vessel was seaworthy, thus precluding any action for recovery against the petitioner. II Whether or not the non-presentation of the marine insurance policy bars the complaint for recovery of sum of money for lack of cause of action. We rule in the negative on both issues. The payment made by the private respondent for the insured value of the lost cargo operates as waiver of its (private respondent) right to enforce the term of the implied warranty against Caltex under the marine insurance policy. However, the same cannot be validly interpreted as an automatic admission of the vessels seaworthiness by the private respondent as to foreclose recourse against the petitioner for any liability under its contractual obligation as a common carrier. The fact of payment grants the private respondent subrogatory right which enables it to exercise legal remedies that would otherwise be available to Caltex as owner of the lost cargo against the petitioner common carrier.8 Article 2207 of the New civil Code provides that: Art. 2207. If the plaintiffs property has been insured, and he has received indemnity from the insurance company for the injury or loss arising out of the wrong or breach of contract complained of, the insurance company shall be subrogated to the rights of the insured against the wrongdoer or the person who has violated the contract. If the amount paid by the insurance company does not fully cover the injury or loss, the aggrieved party shall be entitled to recover the deficiency from the person causing the loss or injury. The right of subrogation has its roots in equity. It is designed to promote and to accomplish justice and is the mode which equity adopts to compel the ultimate payment of a debt by one who in justice and good conscience ought to pay. 9 It is not dependent

upon, nor does it grow out of, any privity of contract or upon written assignment of claim. It accrues simply upon payment by the insurance company of the insurance claim.10 Consequently, the payment made by the private respondent (insurer) to Caltex (assured) operates as an equitable assignment to the former of all the remedies which the latter may have against the petitioner. From the nature of their business and for reasons of public policy, common carriers are bound to observe extraordinary diligence in the vigilance over the goods and for the safety of passengers transported by them, according to all the circumstance of each case.11 In the event of loss, destruction or deterioration of the insured goods, common carriers shall be responsible unless the same is brought about, among others, by flood, storm, earthquake, lightning or other natural disaster or calamity.12 In all other cases, if the goods are lost, destroyed or deteriorated, common carriers are presumed to have been at fault or to have acted negligently, unless they prove that they observed extraordinary diligence.13 In order to escape liability for the loss of its cargo of industrial fuel oil belonging to Caltex, petitioner attributes the sinking of MT Maysun to fortuitous even or force majeure. From the testimonies of Jaime Jarabe and Francisco Berina, captain and chief mate, respectively of the ill-fated vessel, it appears that a sudden and unexpected change of weather condition occurred in the early morning of August 16, 1986; that at around 3:15 oclock in the morning a squall ("unos") carrying strong winds with an approximate velocity of 30 knots per hour and big waves averaging eighteen (18) to twenty (20) feet high, repeatedly buffeted MT Maysun causing it to tilt, take in water and eventually sink with its cargo.14 This tale of strong winds and big waves by the said officers of the petitioner however, was effectively rebutted and belied by the weather report15 from the Philippine Atmospheric, Geophysical and Astronomical Services Administration (PAGASA), the independent government agency charged with monitoring weather and sea conditions, showing that from 2:00 oclock to 8:00 oclock in the morning on August 16, 1986, the wind speed remained at ten (10) to twenty (20) knots per hour while the height of the waves ranged from .7 to two (2) meters in the vicinity of Cuyo East Pass and Panay Gulf where the subject vessel sank. Thus, as the appellate court correctly ruled, petitioners vessel, MT Maysun, sank with its entire cargo for the reason that it was not seaworthy. There was no squall or bad weather or extremely poor sea condition in the vicinity when the said vessel sank. The appellate court also correctly opined that the petitioners witnesses, Jaime Jarabe and Francisco Berina, ship captain and chief mate, respectively, of the said vessel, could not be expected to testify against the interest of their employer, the herein petitioner common carrier. Neither may petitioner escape liability by presenting in evidence certificates 16 that tend to show that at the time of dry-docking and inspection by the Philippine Coast Guard, the vessel MT Maysun, was fit for voyage. These pieces of evidence do not necessarily take into account the actual condition of the vessel at the time of the commencement of the voyage. As correctly observed by the Court of appeals:

At the time of dry-docking and inspection, the ship may have appeared fit. The certificates issued, however, do not negate the presumption of unseaworthiness triggered by an unexplained sinking. Of certificates issued in this regard, authorities are likewise clear as to their probative value, (thus): Seaworthiness relates to a vessels actual condition. Neither the granting of classification or the issuance of certificates established seaworthiness. (2-A Benedict on Admiralty, 7-3, Sec. 62). And also: Authorities are clear that diligence in securing certificates of seaworthiness does not satisfy the vessel owners obligation. Also securing the approval of the shipper of the cargo, or his surveyor, of the condition of the vessel or her stowage does not establish due diligence if the vessel was in fact unseaworthy, for the cargo owner has no obligation in relation to seaworthiness. (Ibid.)17 Additionally, the exoneration of MT Maysuns officers and crew by the Board of Marine Inquiry merely concerns their respective administrative liabilities. It does not in any way operate to absolve the petitioner common carrier from its civil liabilities. It does not in any way operate to absolve the petitioner common carrier from its civil liability arising from its failure to observe extraordinary diligence in the vigilance over the goods it was transporting and for the negligent acts or omissions of its employees, the determination of which properly belongs to the courts.18 In the case at bar, petitioner is liable for the insured value of the lost cargo of industrial fuel oil belonging to Caltex for its failure to rebut the presumption of fault or negligence as common carrier19 occasioned by the unexplained sinking of its vessel, MT Maysun, while in transit. Anent the second issue, it is our view and so hold that the presentation in evidence of the marine insurance policy is not indispensable in this case before the insurer may recover from the common carrier the insured value of the lost cargo in the exercise of its subrogatory right. The subrogation receipt, by itself, is sufficient to establish not only the relationship of herein private respondent as insurer and Caltex, as the assured shipper of the lost cargo of industrial fuel oil, but also the amount paid to settle the insurance claim. The right of subrogation accrues simply upon payment by the insurance company of the insurance claim.20 The presentation of the insurance policy was necessary in the case of Home Insurance Corporation v. CA21 (a case cited by petitioner) because the shipment therein (hydraulic engines) passed through several stages with different parties involved in each stage. First, from the shipper to the port of departure; second, from the port of departure to the M/S Oriental Statesman; third, from the M/S Oriental Statesman to the M/S Pacific Conveyor; fourth, from the M/S Pacific Conveyor to the port or arrival; fifth, from the port of arrival to the arrastre operator; sixth, from the arrastre operator to the hauler, Mabuhay Brokerage Co., Inc. (private respondent therein); and lastly, from the hauler to

the consignee. We emphasized in that case that in the absence of proof of stipulations to the contrary, the hauler can be liable only for any damage that occurred from the time it received the cargo until it finally delivered it to the consignee. Ordinarily, it cannot be held responsible for the handling of the cargo before it actually received it. The insurance contract, which was not presented in evidence in that case would have indicated the scope of the insurers liability, if any, since no evidence was adduced indicating at what stage in the handling process the damage to the cargo was sustained. Hence, our ruling on the presentation of the insurance policy in the said case of Home Insurance Corporation is not applicable to the case at bar. In contrast, there is no doubt that the cargo of industrial fuel oil belonging to Caltex, in the case at bar, was lost while on board petitioners vessel, MT Maysun, which sank while in transit in the vicinity of Panay Gulf and Cuyo East Pass in the early morning of August 16, 1986. WHEREFORE, the instant petition is DENIED. The Decision dated June 17, 1996 of the Court of Appeals in CA-G.R. CV No. 39836 is AFFIRMED. Costs against the petitioner. SO ORDERED.1wphi1.nt Bellosillo, Mendoza, Quisumbing, and Buena, JJ., concur.

G.R. No. 177839

January 18, 2012

FIRST LEPANTO-TAISHO INSURANCE CORPORATION (now known as FLT PRIME INSURANCE CORPORATION), Petitioner, vs. CHEVRON PHILIPPINES, INC. (formerly known as CALTEX [PHILIPPINES], INC.), Respondent. DECISION VILLARAMA, JR., J.: Before this Court is a Rule 45 Petition assailing the Decision1 dated November 20, 2006 and Resolution2 dated May 8, 2007 of the Court of Appeals (CA) in CA-G.R. CV No. 86623, which reversed the Decision3 dated August 5, 2005 of the Regional Trial Court (RTC) of Makati City, Branch 59 in Civil Case No 02-857.

Respondent Chevron Philippines, Inc., formerly Caltex Philippines, Inc., sued petitioner First Lepanto-Taisho Insurance Corporation (now known as FLT Prime Insurance Corporation) for the payment of unpaid oil and petroleum purchases made by its distributor Fumitechniks Corporation (Fumitechniks). Fumitechniks, represented by Ma. Lourdes Apostol, had applied for and was issued Surety Bond FLTICG (16) No. 01012 by petitioner for the amount of P15,700,000.00. As stated in the attached rider, the bond was in compliance with the requirement for the grant of a credit line with the respondent "to guarantee payment/remittance of the cost of fuel products withdrawn within the stipulated time in accordance with the terms and conditions of the agreement." The surety bond was executed on October 15, 2001 and will expire on October 15, 2002.4 Fumitechniks defaulted on its obligation. The check dated December 14, 2001 it issued to respondent in the amount of P11,461,773.10, when presented for payment, was dishonored for reason of "Account Closed." In a letter dated February 6, 2002, respondent notified petitioner of Fumitechniks unpaid purchases in the total amount of P15,084,030.30. In its letter-reply dated February 13, 2002, petitioner through its counsel, requested that it be furnished copies of the documents such as delivery receipts.5 Respondent complied by sending copies of invoices showing deliveries of fuel and petroleum products between November 11, 2001 and December 1, 2001. Simultaneously, a letter6 was sent to Fumitechniks demanding that the latter submit to petitioner the following: (1) its comment on respondents February 6, 2002 letter; (2) copy of the agreement secured by the Bond, together with copies of documents such as delivery receipts; and (3) information on the particulars, including "the terms and conditions, of any arrangement that [Fumitechniks] might have made or any ongoing negotiation with Caltex in connection with the settlement of the obligations subject of the Caltex letter." In its letter dated March 1, 2002, Fumitechniks through its counsel wrote petitioners counsel informing that it cannot submit the requested agreement since no such agreement was executed between Fumitechniks and respondent. Fumitechniks also enclosed a copy of another surety bond issued by CICI General Insurance Corporation in favor of respondent to secure the obligation of Fumitechniks and/or Prime Asia Sales and Services, Inc. in the amount of P15,000,000.00.7 Consequently, petitioner advised respondent of the non-existence of the principal agreement as confirmed by Fumitechniks. Petitioner explained that being an accessory contract, the bond cannot exist without a principal agreement as it is essential that the copy of the basic contract be submitted to the proposed surety for the appreciation of the extent of the obligation to be covered by the bond applied for.8 On April 9, 2002, respondent formally demanded from petitioner the payment of its claim under the surety bond. However, petitioner reiterated its position that without the basic contract subject of the bond, it cannot act on respondents claim; pe titioner also contested the amount of Fumitechniks supposed obligation.9

Alleging that petitioner unjustifiably refused to heed its demand for payment, respondent prayed for judgment ordering petitioner to pay the sum of P15,080,030.30, plus interest, costs and attorneys fees equivalent to ten percent of the total obligation. 10 Petitioner, in its Answer with Counterclaim,11 asserted that the Surety Bond was issued for the purpose of securing the performance of the obligations embodied in the Principal Agreement stated therein, which contract should have been attached and made part thereof. After trial, the RTC rendered judgment dismissing the complaint as well as petitioners counterclaim. Said court found that the terms and conditions of the oral credit line agreement between respondent and Fumitechniks have not been relayed to petitioner and neither were the same conveyed even during trial. Since the surety bond is a mere accessory contract, the RTC concluded that the bond cannot stand in the absence of the written agreement secured thereby. In holding that petitioner cannot be held liable under the bond it issued to Fumitechniks, the RTC noted the practice of petitioner, as testified on by its witnesses, to attach a copy of the written agreement (principal contract) whenever it issues a surety bond, or to be submitted later if not yet in the possession of the assured, and in case of failure to submit the said written agreement, the surety contract will not be binding despite payment of the premium. Respondent filed a motion for reconsideration while petitioner filed a motion for partial reconsideration as to the dismissal of its counterclaim. With the denial of their motions, both parties filed their respective notice of appeal. The CA ruled in favor of respondent, the dispositive portion of its decision reads: WHEREFORE, the appealed Decision is REVERSED and SET ASIDE. A new judgment is hereby entered ORDERING defendant-appellant First Lepanto-Taisho Insurance Corporation to pay plaintiff-appellant Caltex (Philippines) Inc. now Chevron Philippines, Inc. the sum of P15,084,030.00. SO ORDERED.12 According to the appellate court, petitioner cannot insist on the submission of a written agreement to be attached to the surety bond considering that respondent was not aware of such requirement and unwritten company policy. It also declared that petitioner is estopped from assailing the oral credit line agreement, having consented to the same upon presentation by Fumitechniks of the surety bond it issued. Considering that such oral contract between Fumitechniks and respondent has been partially executed, the CA ruled that the provisions of the Statute of Frauds do not apply. With the denial of its motion for reconsideration, petitioner appealed to this Court raising the following issues:

I. WHETHER OR NOT THE HONORABLE COURT OF APPEALS ERRED IN ITS INTERPRETATION OF THE PROVISIONS OF THE SURETY BOND WHEN IT HELD THAT THE SURETY BOND SECURED AN ORAL CREDIT LINE AGREEMENT NOTWITHSTANDING THE STIPULATIONS THEREIN CLEARLY SHOWING BEYOND DOUBT THAT WHAT WAS BEING SECURED WAS A WRITTEN AGREEMENT, PARTICULARLY, THE WRITTEN AGREEMENT A COPY OF WHICH WAS EVEN REQUIRED TO BE ATTACHED TO THE SURETY BOND AND MADE A PART THEREOF. II. WHETHER OR NOT THE HONORABLE COURT OF APPEALS ERRED IN NOT STRIKING OUT THE QUESTIONED RESPONDENTS EVIDENCE FOR BEING CONTRARY TO THE PAROL EVIDENCE RULE, IMMATERIAL AND IRRELEVANT AND CONTRARY TO THE STATUTE OF FRAUDS. III. WHETHER OR NOT THE HONORABLE COURT OF APPEALS ERRED IN NOT STRIKING OUT THE RESPONDENTS MOTION FOR RECONSIDERATION OF THE RTC DECISION FOR BEING A MERE SCRAP OF PAPER AND PRO FORMA AND, CONSEQUENTLY, IN NOT DECLARING THE RTC DECISION AS FINAL AND EXECUTORY IN SO FAR AS IT DISMISSED THE COMPLAINT. IV. WHETHER OR NOT THE HONORABLE COURT OF APPEALS ERRED IN REVERSING THE RTC DECISION AND IN NOT GRANTING PETITIONERS COUNTERCLAIM.13 The main issue to be resolved is one of first impression: whether a surety is liable to the creditor in the absence of a written contract with the principal. Section 175 of the Insurance Code defines a suretyship as a contract or agreement whereby a party, called the surety, guarantees the performance by another party, called the principal or obligor, of an obligation or undertaking in favor of a third party, called the obligee. It includes official recognizances, stipulations, bonds or undertakings issued under Act 536,14 as amended. Suretyship arises upon the solidary binding of a person deemed the surety with the principal debtor, for the purpose of fulfilling an obligation.15 Such undertaking makes a surety agreement an ancillary contract as it presupposes the existence of a principal contract. Although the contract of a surety is in essence secondary only to a valid principal obligation, the surety becomes liable for the debt or duty of another although it possesses no direct or personal interest over the obligations nor does it receive any benefit therefrom. And notwithstanding the fact that the surety contract is secondary to the principal obligation, the surety assumes liability as a regular party to the undertaking.16 The extent of a suretys liability is determined by the language of the suretyship contract or bond itself. It cannot be extended by implication, beyond the terms of the contract.17 Thus, to determine whether petitioner is liable to respondent under the surety bond, it becomes necessary to examine the terms of the contract itself.

Surety Bond FLTICG (16) No. 01012 is a standard form used by petitioner, which states: That we, FUMITECHNIKS CORP. OF THE PHILS. of #154 Anahaw St., Project 7, Quezon City as principal and First Lepanto-Taisho Insurance Corporation a corporation duly organized and existing under and by virtue of the laws of the Philippines as Surety, are held firmly bound unto CALTEX PHILIPPINES, INC. of ______ in the sum ofFIFTEEN MILLION SEVEN HUNDRED THOUSAND ONLY PESOS (P15,700,000.00), Philippine Currency, for the payment of which sum, well and truly to be made, we bind ourselves, our heirs, executors, administrators, successors, and assigns, jointly and severally, firmly by these presents: The conditions of this obligation are as follows: WHEREAS, the above-bounden principal, on 15th day of October, 2001 entered into [an] agreement with CALTEX PHILIPPINES, INC. of ________________ to fully and faithfully a copy of which is attached hereto and made a part hereof: WHEREAS, said Obligee__ requires said principal to give a good and sufficient bond in the above stated sum to secure the full and faithful performance on his part of said agreement__. NOW THEREFORE, if the principal shall well and truly perform and fulfill all the undertakings, covenants, terms, conditions, and agreements stipulated in said agreement__ then this obligation shall be null and void; otherwise it shall remain in full force and effect. The liability of First Lepanto-Taisho Insurance Corporation under this bond will expire on October 15, 2002__. x x x x18 (Emphasis supplied.) The rider attached to the bond sets forth the following: WHEREAS, the Principal has applied for a Credit Line in the amount of PESOS: Fifteen Million Seven Hundred thousand only (P15,700,000.00), Philippine Currency with the Obligee for the purchase of Fuel Products; WHEREAS, the obligee requires the Principal to post a bond to guarantee payment/remittance of the cost of fuel products withdrawn within the stipulated time in accordance with terms and conditions of the agreement;

IN NO CASE, however, shall the liability of the Surety hereunder exceed the sum of PESOS: Fifteen million seven hundred thousand only (P15,700,000.00), Philippine Currency. NOW THEREFORE, if the principal shall well and truly perform and fulfill all the undertakings, covenants, terms and conditions and agreements stipulated in said undertakings, then this obligation shall be null and void; otherwise, it shall remain in full force and effect. The liability of FIRST LEPANTO-TAISHO INSURANCE CORPORATION, under this Bond will expire on 10.15.01_. Furthermore, it is hereby understood that FIRST LEPANTO-TAISHO INSURANCE CORPORATION will not be liable for any claim not presented to it in writing within fifteen (15) days from the expiration of this bond, and that the Obligee hereby waives its right to claim or file any court action against the Surety after the termination of fifteen (15) days from the time its cause of action accrues.19 Petitioner posits that non-compliance with the submission of the written agreement, which by the express terms of the surety bond, should be attached and made part thereof, rendered the bond ineffective. Since all stipulations and provisions of the surety contract should be taken and interpreted together, in this case, the unmistakable intention of the parties was to secure only those terms and conditions of the written agreement. Thus, by deleting the required submission and attachment of the written agreement to the surety bond and replacing it with the oral credit agreement, the obligations of the surety have been extended beyond the limits of the surety contract. On the other hand, respondent contends that the surety bond had been delivered by petitioner to Fumitechniks which paid the premiums and delivered the bond to respondent, who in turn, opened the credit line which Fumitechniks availed of to purchase its merchandise from respondent on credit. Respondent points out that a careful reading of the surety contract shows that there is no such requirement of submission of the written credit agreement for the bonds effectivity. Moreover, respondents witnesses had already explained that distributorship accounts are not covered by written distribution agreements. Supplying the details of these agreements is allowed as an exception to the parol evidence rule even if it is proof of an oral agreement. Respondent argues that by introducing documents that petitioner sought to exclude, it never intended to change or modify the contents of the surety bond but merely to establish the actual terms of the distribution agreement between Fumitechniks and respondent, a separate agreement that was executed shortly after the issuance of the surety bond. Because petitioner still issued the bond and allowed it to be delivered to respondent despite the fact that a copy of the written distribution agreement was never attached thereto, respondent avers that clearly, such attaching of the copy of the principal agreement, was for evidentiary purposes only. The real intention of the bond was to secure the payment of all the purchases of Fumitechniks from respondent up to the maximum amount allowed under the bond.

A reading of Surety Bond FLTICG (16) No. 01012 shows that it secures the payment of purchases on credit by Fumitechniks in accordance with the terms and conditions of the "agreement" it entered into with respondent. The word "agreement" has reference to the distributorship agreement, the principal contract and by implication included the credit agreement mentioned in the rider. However, it turned out that respondent has executed written agreements only with its direct customers but not distributors like Fumitechniks and it also never relayed the terms and conditions of its distributorship agreement to the petitioner after the delivery of the bond. This was clearly admitted by respondents Marketing Coordinator, Alden Casas Fajardo, who testified as follows: Atty. Selim: Q : Mr. Fajardo[,] you mentioned during your cross-examination that the surety bond as part of the requirements of [Fumitechniks] before the Distributorship Agreement was approved? A : Yes Sir. xxxx Q : Is it the practice or procedure at Caltex to reduce distributorship account into writing? xxxx A : No, its not a practice to make an agreement. xxxx Atty. Quiroz: Q : What was the reason why you are not reducing your agreement with your client into writing? A : Well, of course as I said, there is no fix pricing in terms of distributorship agreement, its usually with regards to direct service to the customers which have direct fixed price. xxxx Q : These supposed terms and conditions that you agreed with [Fumitechniks], did you relay to the defendant A : Yes Sir. xxxx

Q : How did you relay that, how did you relay the terms and conditions to the defendant? A : I dont know, it was during the time for collection because I collected them and explain the terms and conditions. Q : You testified awhile ago that you did not talk to the defendant First LepantoTaisho Insurance Corporation? A : I was confused with the question. Im talking about Malou Apostol. Q : So, in your answer, you have not relayed those terms and conditions to the defendant First Lepanto, you have not? A : Yes Sir. Q : And as of this present, you have not yet relayed the terms and conditions? A : Yes Sir. Respondent, however, maintains that the delivery of the bond and acceptance of premium payment by petitioner binds the latter as surety, notwithstanding the nonsubmission of the oral distributorship and credit agreement which understandably cannot be attached to the bond. The contention has no merit. The law is clear that a surety contract should be read and interpreted together with the contract entered into between the creditor and the principal. Section 176 of the Insurance Code states: Sec. 176. The liability of the surety or sureties shall be joint and several with the obligor and shall be limited to the amount of the bond. It is determined strictly by the terms of the contract of suretyship in relation to the principal contract between the obligor and the obligee. (Emphasis supplied.) A surety contract is merely a collateral one, its basis is the principal contract or undertaking which it secures.21Necessarily, the stipulations in such principal agreement must at least be communicated or made known to the surety particularly in this case where the bond expressly guarantees the payment of respondents fuel products withdrawn by Fumitechniks in accordance with the terms and conditions of their agreement. The bond specifically makes reference to a written agreement. It is basic that if the terms of a contract are clear and leave no doubt upon the intention of the contracting parties, the literal meaning of its stipulations shall control.22 Moreover, being an onerous undertaking, a surety agreement is strictly construed against the creditor, and every doubt is resolved in favor of the solidary debtor.23 Having accepted the bond,

respondent as creditor must be held bound by the recital in the surety bond that the terms and conditions of its distributorship contract be reduced in writing or at the very least communicated in writing to the surety. Such non-compliance by the creditor (respondent) impacts not on the validity or legality of the surety contract but on the creditors right to demand performance. It bears stressing that the contract of suretyship imports entire good faith and confidence between the parties in regard to the whole transaction, although it has been said that the creditor does not stand as a fiduciary in his relation to the surety. The creditor is generally held bound to a faithful observance of the rights of the surety and to the performance of every duty necessary for the protection of those rights. 24 Moreover, in this jurisdiction, obligations arising from contracts have the force of law between the parties and should be complied with in good faith.25 Respondent is charged with notice of the specified form of the agreement or at least the disclosure of basic terms and conditions of its distributorship and credit agreements with its client Fumitechniks after its acceptance of the bond delivered by the latter. However, it never made any effort to relay those terms and conditions of its contract with Fumitechniks upon the commencement of its transactions with said client, which obligations are covered by the surety bond issued by petitioner. Contrary to respondents assertion, there is no indication in the records that petitioner had actual knowledge of its alleged business practice of not having written contracts with distributors; and even assuming petitioner was aware of such practice, the bond issued to Fumitechniks and accepted by respondent specifically referred to a "written agreement." As to the contention of petitioner that respondents motion for reconsideration filed before the trial court should have been deemed not filed for being pro forma, the Court finds it to be without merit. The mere fact that a motion for reconsideration reiterates issues already passed upon by the court does not, by itself, make it a pro forma motion. Among the ends to which a motion for reconsideration is addressed is precisely to convince the court that its ruling is erroneous and improper, contrary to the law or evidence; the movant has to dwell of necessity on issues already passed upon.261avvphi1 Finally, we hold that the trial court correctly dismissed petitioners counterclaim for moral damages and attorneys fees. The filing alone of a civil action should not be a ground for an award of moral damages in the same way that a clearly unfounded civil action is not among the grounds for moral damages.27 Besides, a juridical person is generally not entitled to moral damages because, unlike a natural person, it cannot experience physical suffering or such sentiments as wounded feelings, serious anxiety, mental anguish or moral shock.28 Although in some recent cases we have held that the Court may allow the grant of moral damages to corporations, it is not automatically granted; there must still be proof of the existence of the factual basis of the damage and its causal relation to the defendants acts. This is so because moral damages, though incapable of pecuniary estimation, are in the category of an award designed to compensate the claimant for actual injury suffered and not to impose a penalty on the

wrongdoer.29 There is no evidence presented to establish the factual basis of petitioners claim for moral damages. Petitioner is likewise not entitled to attorneys fees. The settled rule is that no premium should be placed on the right to litigate and that not every winning party is entitled to an automatic grant of attorneys fees.30 In pursuing its claim on the surety bond, respondent was acting on the belief that it can collect on the obligation of Fumitechniks notwithstanding the non-submission of the written principal contract. WHEREFORE, the petition for review on certiorari is PARTLY GRANTED. The Decision dated November 20, 2006 and Resolution dated May 8, 2007 of the Court of Appeals in CA-G.R. CV No. 86623, are REVERSED and SET ASIDE. The Decision dated August 5, 2005 of the Regional Trial Court of Makati City, Branch 59 in Civil Case No. 02-857 dismissing respondents complaint as well as petitioners counterclaim, is hereby REINSTATED and UPHELD. No pronouncement as to costs. SO ORDERED. MARTIN S. VILLARAMA, JR. Associate Justice WE CONCUR: RENATO C. CORONA Chief Justice Chairperson TERESITA J. LEONARDO-DE CASTRO Associate Justice LUCAS P. BERSAMIN Associate Justice

MARIANO C. DEL CASTILLO Associate Justice CERTIFICATION Pursuant to Section 13, Article VIII of the 1987 Constitution, I certify that the conclusions in the above Decision had been reached in consultation before the case was assigned to the writer of the opinion of the Courts Division. RENATO C. CORONA Chief Justice G.R. No. L-34583 October 22, 1931

THE BANK OF THE PHILIPPINE ISLANDS, administrator of the estate of the late Adolphe Oscar Schuetze,plaintiff-appellant, vs. JUAN POSADAS, JR., Collector of Internal Revenue, defendant-appellee. Araneta, De Joya, Zaragoza and Araneta for appellant. Attorney-General Jaranilla for appellee.

VILLA-REAL, J.: The Bank of the Philippine Islands, as administrator of the estate of the deceased Adolphe Oscar Schuetze, has appealed to this court from the judgment of the Court of First Instance of Manila absolving the defendant Juan Posadas, Jr., Collector of Internal Revenue, from the complaint filed against him by said plaintiff bank, and dismissing the complaint with costs. The appellant has assigned the following alleged errors as committed by the trial court in its judgment, to wit: 1. The lower court erred in holding that the testimony of Mrs. Schuetze was inefficient to established the domicile of her husband. 2. The lower court erred in holding that under section 1536 of the Administrative Code the tax imposed by the defendant is lawful and valid. 3. The lower court erred in not holding that one-half () of the proceeds of the policy in question is community property and that therefore no inheritance tax can be levied, at least on one-half () of the said proceeds. 4. The lower court erred in not declaring that it would be unconstitutional to impose an inheritance tax upon the insurance policy here in question as it would be a taking of property without due process of law. The present complaint seeks to recover from the defendant Juan Posadas, Jr., Collector of Internal Revenue, the amount of P1,209 paid by the plaintiff under protest, in its capacity of administrator of the estate of the late Adolphe Oscar Schuetze, as inheritance tax upon the sum of P20,150, which is the amount of an insurance policy on the deceased's life, wherein his own estate was named the beneficiary. At the hearing, in addition to documentary and parol evidence, both parties submitted the following agreed statement of facts of the court for consideration: It is hereby stipulated and agreed by and between the parties in the aboveentitled action through their respective undersigned attorneys:

1. That the plaintiff, Rosario Gelano Vda. de Schuetze, window of the late Adolphe Oscar Schuetze, is of legal age, a native of Manila, Philippine Islands, and is and was at all times hereinafter mentioned a resident of Germany, and at the time of the death of her husband, the late Adolphe Oscar Schuetze, she was actually residing and living in Germany; 2. That the Bank of the Philippine Islands, is and was at all times hereinafter mentioned a banking institution duly organized and existing under and by virtue of the laws of the Philippine Islands; 3. That on or about August 23, 1928, the herein plaintiff before notary public Salvador Zaragoza, drew a general power appointing the above-mentioned Bank of the Philippine Islands as her attorney-in-fact, and among the powers conferred to said attorney-in-fact was the power to represent her in all legal actions instituted by or against her; 4. That the defendant, of legal age, is and at all times hereinafter mentioned the duly appointed Collector of Internal Revenue with offices at Manila, Philippine Islands; 5. That the deceased Adolphe Oscar Schuetze came to the Philippine Islands for the first time of March 31, 1890, and worked in the several German firms as a mere employee and that from the year 1903 until the year 1918 he was partner in the business of Alfredo Roensch; 6. That from 1903 to 1922 the said Adolphe Oscar Schuetze was in the habit of making various trips to Europe; 7. That on December 3, 1927, the late Adolphe Oscar Schuetze coming from Java, and with the intention of going to Bremen, landed in the Philippine Islands where he met his death on February 2, 1928; 8. That on March 31, 1926, the said Adolphe Oscar Schuetze, while in Germany, executed a will, in accordance with its law, wherein plaintiff was named his universal heir; 9. That the Bank of the Philippine Islands by order of the Court of First Instance of Manila under date of May 24, 1928, was appointed administrator of the estate of the deceased Adolphe Oscar Schuetze; 10. That, according to the testamentary proceedings instituted in the Court of First Instance of Manila, civil case No. 33089, the deceased at the time of his death was possessed of not only real property situated in the Philippine Islands, but also personal property consisting of shares of stock in nineteen (19) domestic corporations;

11. That the fair market value of all the property in the Philippine Islands left by the deceased at the time of his death in accordance with the inventory submitted to the Court of First Instance of Manila, civil case No. 33089, was P217,560.38; 12. That the Bank of the Philippine Islands, as administrator of the estate of the deceased rendered its final account on June 19, 1929, and that said estate was closed on July 16, 1929; 13. That among the personal property of the deceased was found life-insurance policy No. 194538 issued at Manila, Philippine Islands, on January 14, 1913, for the sum of $10,000 by the Sun Life Assurance Company of Canada, Manila branch, a foreign corporation duly organized and existing under and by virtue of the laws of Canada, and duly authorized to transact business in the Philippine Islands; 14. That in the insurance policy the estate of the said Adolphe Oscar Schuetze was named the beneficiary without any qualification whatsoever; 15. That for five consecutive years, the deceased Adolphe Oscar Schuetze paid the premiums of said policy to the Sun Life Assurance Company of Canada, Manila branch; 16. That on or about the year 1918, the Sun Life Assurance Company of Canada, Manila branch, transferred said policy to the Sun Life Assurance Company of Canada, London branch; 17. That due to said transfer the said Adolphe Oscar Schuetze from 1918 to the time of his death paid the premiums of said policy to the Sun Life Assurance Company of Canada, London Branch; 18. That the sole and only heir of the deceased Adolphe Oscar Schuetze is his widow, the plaintiff herein; 19. That at the time of the death of the deceased and at all times thereafter including the date when the said insurance policy was paid, the insurance policy was not in the hands or possession of the Manila office of the Sun Life Assurance Company of Canada, nor in the possession of the herein plaintiff, nor in the possession of her attorney-in-fact the Bank of the Philippine Islands, but the same was in the hands of the Head Office of the Sun Life Assurance Company of Canada, at Montreal, Canada; 20. That on July 13, 1928, the Bank of the Philippine Islands as administrator of the decedent's estate received from the Sun Life Assurance Company of Canada, Manila branch, the sum of P20,150 representing the proceeds of the insurance policy, as shown in the statement of income and expenses of the

estate of the deceased submitted on June 18, 1929, by the administrator to the Court of First Instance of Manila, civil case No. 33089; 21. That the Bank of the Philippine Islands delivered to the plaintiff herein the said sum of P20,150; 22. That the herein defendant on or about July 5, 1929, imposed an inheritance tax upon the transmission of the proceeds of the policy in question in the sum of P20,150 from the estate of the late Adolphe Oscar Schuetze to the sole heir of the deceased, or the plaintiff herein, which inheritance tax amounted to the sum of P1,209; 23. That the Bank of the Philippine Islands as administrator of the decedent's estate and as attorney-in-fact of the herein plaintiff, having been demanded by the herein defendant to pay inheritance tax amounting to the sum of P1,209, paid to the defendant under protest the above-mentioned sum; 24. That notwithstanding the various demands made by plaintiff to the defendant, said defendant has refused and refuses to refund to plaintiff the above mentioned sum of P1,209; 25. That plaintiff reserves the right to adduce evidence as regards the domicile of the deceased, and so the defendant, the right to present rebuttal evidence; 26. That both plaintiff and defendant submit this stipulation of facts without prejudice to their right to introduce such evidence, on points not covered by the agreement, which they may deem proper and necessary to support their respective contentions. In as much as one of the question raised in the appeal is whether an insurance policy on said Adolphe Oscar Schuetze's life was, by reason of its ownership, subject to the inheritance tax, it would be well to decide first whether the amount thereof is paraphernal or community property. According to the foregoing agreed statement of facts, the estate of Adolphe Oscar Schuetze is the sole beneficiary named in the life-insurance policy for $10,000, issued by the Sun Life Assurance Company of Canada on January 14, 1913. During the following five years the insured paid the premiums at the Manila branch of the company, and in 1918 the policy was transferred to the London branch. The record shows that the deceased Adolphe Oscar Schuetze married the plaintiff-appellant Rosario Gelano on January 16, 1914. With the exception of the premium for the first year covering the period from January 14, 1913 to January 14, 1914, all the money used for paying the premiums, i. e., from the second year, or January 16, 1914, or when the deceased Adolphe Oscar

Schuetze married the plaintiff-appellant Rosario Gelano, until his death on February 2, 1929, is conjugal property inasmuch as it does not appear to have exclusively belonged to him or to his wife (art. 1407, Civil Code). As the sum of P20,150 here in controversy is a product of such premium it must also be deemed community property, because it was acquired for a valuable consideration, during said Adolphe Oscar Schuetze's marriage with Rosario Gelano at the expense of the common fund (art. 1401, No. 1, Civil Code), except for the small part corresponding to the first premium paid with the deceased's own money. In his Commentaries on the Civil Code, volume 9, page 589, second edition, Manresa treats of life insurance in the following terms, to wit: The amount of the policy represents the premiums to be paid, and the right to it arises the moment the contract is perfected, for at the moment the power of disposing of it may be exercised, and if death occurs payment may be demanded. It is therefore something acquired for a valuable consideration during the marriage, though the period of its fulfillment, depend upon the death of one of the spouses, which terminates the partnership. So considered, the question may be said to be decided by articles 1396 and 1401: if the premiums are paid with the exclusive property of husband or wife, the policy belongs to the owner; if with conjugal property, or if the money cannot be proved as coming from one or the other of the spouses, the policy is community property. The Supreme Court of Texas, United States, in the case of Martin vs. Moran (11 Tex. Civ. A., 509) laid down the following doctrine: COMMUNITY PROPERTY LIFE INSURANCE POLICY. A husband took out an endowment life insurance policy on his life, payable "as directed by will." He paid the premiums thereon out of community funds, and by his will made the proceeds of the policy payable to his own estate. Held, that the proceeds were community estate, one-half of which belonged to the wife. In In re Stan's Estate, Myr. Prob. (Cal.), 5, the Supreme Court of California laid down the following doctrine: A testator, after marriage, took out an insurance policy, on which he paid the premiums from his salary. Held that the insurance money was community property, to one-half of which, the wife was entitled as survivor. In In re Webb's Estate, Myr. Prob. (Cal.), 93, the same court laid down the following doctrine: A decedent paid the first third of the amount of the premiums on his lifeinsurance policy out of his earnings before marriage, and the remainder from his earnings received after marriage. Held, that one-third of the policy belonged to his separate estate, and the remainder to the community property.

Thus both according to our Civil Code and to the ruling of those North American States where the Spanish Civil Code once governed, the proceeds of a life-insurance policy whereon the premiums were paid with conjugal money, belong to the conjugal partnership. The appellee alleges that it is a fundamental principle that a life-insurance policy belongs exclusively to the beneficiary upon the death of the person insured, and that in the present case, as the late Adolphe Oscar Schuetze named his own estate as the sole beneficiary of the insurance on his life, upon his death the latter became the sole owner of the proceeds, which therefore became subject to the inheritance tax, citing Del Val vs. Del Val (29 Phil., 534), where the doctrine was laid down that an heir appointed beneficiary to a life-insurance policy taken out by the deceased, becomes the absolute owner of the proceeds of such policy upon the death of the insured. The estate of a deceased person cannot be placed on the same footing as an individual heir. The proceeds of a life-insurance policy payable to the estate of the insured passed to the executor or administrator of such estate, and forms part of its assets (37 Corpus Juris, 565, sec. 322); whereas the proceeds of a life-insurance policy payable to an heir of the insured as beneficiary belongs exclusively to said heir and does not form part of the deceased's estate subject to administrator. (Del Val vs. Del Val, supra; 37 Corpus Juris, 566, sec. 323, and articles 419 and 428 of the Code of Commerce.) Just as an individual beneficiary of a life-insurance policy taken out by a married person becomes the exclusive owner of the proceeds upon the death of the insured even if the premiums were paid by the conjugal partnership, so, it is argued, where the beneficiary named is the estate of the deceased whose life is insured, the proceeds of the policy become a part of said estate upon the death of the insured even if the premiums have been paid with conjugal funds. In a conjugal partnership the husband is the manager, empowered to alienate the partnership property without the wife's consent (art. 1413, Civil Code), a third person, therefore, named beneficiary in a life-insurance policy becomes the absolute owner of its proceeds upon the death of the insured even if the premiums should have been paid with money belonging to the community property. When a married man has his life insured and names his own estate after death, beneficiary, he makes no alienation of the proceeds of conjugal funds to a third person, but appropriates them himself, adding them to the assets of his estate, in contravention of the provisions of article 1401, paragraph 1, of the Civil Code cited above, which provides that "To the conjugal partnership belongs" (1) Property acquired for a valuable consideration during the marriage at the expense of the common fund, whether the acquisition is made for the partnership or for one of the spouses only." Furthermore, such appropriation is a fraud practised upon the wife, which cannot be allowed to prejudice her, according to article 1413, paragraph 2, of said Code. Although the husband is the manager of the conjugal partnership, he cannot of his own free will convert the partnership property into his own exclusive property.

As all the premiums on the life-insurance policy taken out by the late Adolphe Oscar Schuetze, were paid out of the conjugal funds, with the exceptions of the first, the proceeds of the policy, excluding the proportional part corresponding to the first premium, constitute community property, notwithstanding the fact that the policy was made payable to the deceased's estate, so that one-half of said proceeds belongs to the estate, and the other half to the deceased's widow, the plaintiff-appellant Rosario Gelano Vda. de Schuetze. The second point to decide in this appeal is whether the Collector of Internal Revenue has authority, under the law, to collect the inheritance tax upon one-half of the life-insurance policy taken out by the late Adolphe Oscar Schuetze, which belongs to him and is made payable to his estate. According to the agreed statement of facts mentioned above, the plaintiffappellant, the Bank of the Philippine Islands, was appointed administrator of the late Adolphe Oscar Schuetze's testamentary estate by an order dated March 24, 1928, entered by the Court of First Instance of Manila. On July 13, 1928, the Sun Life Assurance Company of Canada, whose main office is in Montreal, Canada, paid Rosario Gelano Vda. de Schuetze upon her arrival at Manila, the sum of P20,150, which was the amount of the insurance policy on the life of said deceased, payable to the latter's estate. On the same date Rosario Gelano Vda. de Schuetze delivered the money to said Bank of the Philippine Islands, as administrator of the deceased's estate, which entered it in the inventory of the testamentary estate, and then returned the money to said widow. Section 1536 of the Administrative Code, as amended by section 10 of Act No. 2835 and section 1 of Act No. 3031, contains the following relevant provision: SEC. 1536. Conditions and rate of taxation. Every transmission by virtue of inheritance, devise, bequest, gift mortis causa or advance in anticipation of inheritance, devise, or bequest of real property located in the Philippine Islands and real rights in such property; of any franchise which must be exercised in the Philippine Islands; of any shares, obligations, or bonds issued by any corporation or sociedad anonimaorganized or constituted in the Philippine Islands in accordance with its laws; of any shares or rights in any partnership, business or industry established in the Philippine Islands or of any personal property located in the Philippine Islands shall be subject to the following tax: xxx xxx xxx

In as much as the proceeds of the insurance policy on the life of the late Adolphe Oscar Schuetze were paid to the Bank of the Philippine Islands, as administrator of the deceased's estate, for management and partition, and as such proceeds were turned over to the sole and universal testamentary heiress Rosario Gelano Vda. de Schuetze, the plaintiff-appellant, here in Manila, the situs of said proceeds is the Philippine Islands.

In his work "The Law of Taxation," Cooley enunciates the general rule governing the levying of taxes upon tangible personal property, in the following words: GENERAL RULE. The suits of tangible personal property, for purposes of taxation may be where the owner is domiciled but is not necessarily so. Unlike intangible personal property, it may acquire a taxation situs in a state other than the one where the owner is domiciled, merely because it is located there. Its taxable situs is where it is more or less permanently located, regardless of the domicile of the owner. It is well settled that the state where it is more or less permanently located has the power to tax it although the owner resides out of the state, regardless of whether it has been taxed for the same period at the domicile of the owner, provided there is statutory authority for taxing such property. It is equally well settled that the state where the owner is domiciled has no power to tax it where the property has acquired an actual situs in another state by reason of its more or less permanent location in that state. ... (2 Cooley, The Law of Taxation, 4th ed., p. 975, par. 451.) With reference to the meaning of the words "permanent" and "in transit," he has the following to say: PERMANENCY OF LOCATION; PROPERTY IN TRANSIT. In order to acquire a situs in a state or taxing district so as to be taxable in the state or district regardless of the domicile of the owner and not taxable in another state or district at the domicile of the owner, tangible personal property must be more or less permanently located in the state or district. In other words, the situs of tangible personal property is where it is more or less permanently located rather than where it is merely in transit or temporarily and for no considerable length of time. If tangible personal property is more or less permanently located in a state other than the one where the owner is domiciled, it is not taxable in the latter state but is taxable in the state where it is located. If tangible personal property belonging to one domiciled in one state is in another state merely in transitu or for a short time, it is taxable in the former state, and is not taxable in the state where it is for the time being. . . . . Property merely in transit through a state ordinarily is not taxable there. Transit begins when an article is committed to a carrier for transportation to the state of its destination, or started on its ultimate passage. Transit ends when the goods arrive at their destination. But intermediate these points questions may arise as to when a temporary stop in transit is such as to make the property taxable at the place of stoppage. Whether the property is taxable in such a case usually depends on the length of time and the purpose of the interruption of transit. . . . . . . . It has been held that property of a construction company, used in construction of a railroad, acquires a situs at the place where used for an indefinite period. So tangible personal property in the state for the purpose of

undergoing a partial finishing process is not to be regarded as in the course of transit nor as in the state for a mere temporary purpose. (2 Cooley, The Law of Taxation, 4th ed., pp. 982, 983 and 988, par. 452.) If the proceeds of the life-insurance policy taken out by the late Adolphe Oscar Schuetze and made payable to his estate, were delivered to the Bank of the Philippine Islands for administration and distribution, they were not in transit but were more or less permanently located in the Philippine Islands, according to the foregoing rules. If this be so, half of the proceeds which is community property, belongs to the estate of the deceased and is subject to the inheritance tax, in accordance with the legal provision quoted above, irrespective of whether or not the late Adolphe Oscar Schuetze was domiciled in the Philippine Islands at the time of his death. By virtue of the foregoing, we are of opinion and so hold: (1) That the proceeds of a life-insurance policy payable to the insured's estate, on which the premiums were paid by the conjugal partnership, constitute community property, and belong one-half to the husband and the other half to the wife, exclusively; (2) that if the premiums were paid partly with paraphernal and partly conjugal funds, the proceeds are likewise in like proportion paraphernal in part and conjugal in part; and (3) that the proceeds of a lifeinsurance policy payable to the insured's estate as the beneficiary, if delivered to the testamentary administrator of the former as part of the assets of said estate under probate administration, are subject to the inheritance tax according to the law on the matter, if they belong to the assured exclusively, and it is immaterial that the insured was domiciled in these Islands or outside.1awphil.net Wherefore, the judgment appealed from is reversed, and the defendant is ordered to return to the plaintiff the one-half of the tax collected upon the amount of P20,150, being the proceeds of the insurance policy on the life of the late Adolphe Oscar Schuetze, after deducting the proportional part corresponding to the first premium, without special pronouncement of costs. So ordered. Avancea, C.J., Johnson, Street, Malcolm, Villamor, and Ostrand, JJ., concur.

Separate Opinions

IMPERIAL, J., dissenting: I cannot concur with the majority in holding that one-half of the insurance policy on the life of the late Adolphe Oscar Schuetze, excepting the proportional part corresponding to the first year's premium is community property belonging to the deceased's widow, named Rosario Gelano, and as such is not subject to the inheritance tax.

There is no question in regard to the facts: It is admitted that Schuetze insured himself in the Sun Life Insurance Company of Canada in Manila, and that the policy was issued on January 14, 1913, payable to his estate after death. He died in Manila on February 2, 1928, leaving his widow as his sole testamentary heiress. The appellant, the Bank of the Philippine Islands, as administrator of the late Schuetze's testamentary estate, received from the insurer the amount of this policy, or the net sum of P20,150. It is an established and generally recognized principle that in a life-insurance policy where the insured has named a beneficiary, the proceeds belong to said beneficiary, and to him alone. "Vested Interest of Beneficiary. In practically every jurisdiction it is the rule that in an ordinary life insurance policy made payable to a beneficiary, and which does not authorize a change of beneficiary, the named beneficiary has an absolute, vested interest in the policy from the date of its issuance, delivery and acceptance, and this is true of a policy payable to the children of the insured equally, without naming them, or their executors, administrators or assigns." (14 R.C.L., 1376.) (Del Val vs. Del Val, 29 Phil., 534 et seq.; Gercio vs. Sun Life Assurance Co. of Canada, 48 Phil., 53 et seq.) When in a life-insurance policy the insured's estate is named beneficiary, the proceeds must be delivered not to the decedent's heirs, but to his administrator or legal representative. "Policy Payable to Insured, His Estate, or Legal Representatives. ... Ordinarily the proceeds of a life insurance policy are payable to the executor or administrator of insured as assets of his estate where by the terms of the policy the proceeds are payable to insured, his estate, his legal representatives, his executors or administrators, his "executors, administrators, or assigns," or even his "heirs, executors, administrators, or assigns." ..." (37 C.J., 565.) "Personal Representatives or Legal Representatives. While there is some authority to the effect that "legal representatives" means the persons entitled to the estate of the insured, and not his executor or administrator, the better view is that ordinarily the proceeds of such a policy pass to his executor or administrator." (14 R.C.L., 1372.) If the foregoing are the principles which should govern life-insurance policies with reference to beneficiaries and the right to the proceeds of such policies, it is evident that Schuetze's estate, and not his widow or the conjugal partnership, is entitled to the proceeds of said policy exclusively, and may receive them from the insurer. The parties must have so understood it when the insurer delivered the net amount of the policy to the Bank of the Philippine Islands, as judicial administrator of the insured. It is stated in the majority opinion that the money with which the premiums were paid during the marriage of the Schuetzes is presumed to have been taken from the conjugal funds, according to article 1407 of the Civil Code, which provides that "All the property of the spouses shall be deemed partnership property in the absence of proof that it belongs exclusively to the husband or to the wife." This is the very argument which led to the settlement of the point of law raised. The provisions of the Civil Code on conjugal property have been improperly applied without considering that a lifeinsurance contract is a peculiar contract governed by special laws, such as Act No. 2427 with its amendments, and the Code of Commerce, which is still in force. In Del Val, supra, it was already held:

We cannot agree with these contentions. The contract of life insurance is a special contract and the destination of the proceeds thereof is determined by special laws which deal exclusively with that subject. The Civil Code has no provisions which relate directly and specially to life insurance contracts or to the destination of life insurance proceeds. That subject is regulated exclusively by the Code of Commerce which provides for the terms of the contract, the relations of the parties and the destination of the proceeds of the policy. The main point to be decided was not whether the premiums were paid out of conjugal or personal funds of one of the spouses, but whether or not the proceeds of the policy became assets of the insured's estate. If it be admitted that the estate is the sole owner of the aforesaid proceeds, which cannot be denied, inasmuch as the policy itself names the estate as the beneficiary, it is beside the point to discuss the nature and origin of the amounts used to pay the premiums, as the title to the proceeds of the policy is vested in the insured's estate, and any right the widow might have should be vindicated in another action. In such a case she might be entitled to reimbursement of her share in the conjugal funds, but not in the present case, for she has been instituted the sole testamentary heiress. From the foregoing, it follows that as the proceeds of the policy belong to Schuetze's estate, and inasmuch as the inheritance tax is levied upon the transmission of a deceased person's estate upon, or, on the occasion of his death, it is clear that the whole proceeds, and not one-half thereof, are subject to such tax. In my opinion the judgment appealed from should have been affirmed in its entirely. Romualdez, J., concurs.

G.R. No. L-9374

February 16, 1915

FRANCISCO DEL VAL, ET AL., plaintiffs-appellants, vs. ANDRES DEL VAL, defendant-appellee. Ledesma, Lim and Irureta Goyena for appellants. O'Brien and DeWitt for appellee. MORELAND, J.: This is an appeal from a judgment of the Court of First Instance of the city of Manila dismissing the complaint with costs. The pleadings set forth that the plaintiffs and defendant are brother and sisters; that they are the only heirs at law and next of kin of Gregorio Nacianceno del Val, who died in Manila on August 4, 1910, intestate; that an administrator was appointed for the estate of the deceased, and, after a partial administration, it was closed and the administrator discharged by order of the Court of First Instance dated December 9, 1911; that during the lifetime of the deceased he took out insurance on his life for the sum of P40,000 and made it payable to the defendant as sole beneficiary; that after his death the defendant collected the face of the policy; that of said policy he paid the sum of P18,365.20 to redeem certain real estate which the decedent had sold to third persons with a right to repurchase; that the redemption of said premises was made by the attorney of the defendant in the name of the plaintiff and the defendant as heirs of the deceased vendor; that the redemption of said premises they have had the use and benefit thereof; that during that time the plaintiffs paid no taxes and made no repairs. It further appears from the pleadings that the defendant, on the death of the deceased, took possession of most of his personal property, which he still has in his possession, and that he has also the balance on said insurance policy amounting to P21,634.80. Plaintiffs contend that the amount of the insurance policy belonged to the estate of the deceased and not to the defendant personally; that, therefore, they are entitled to a partition not only of the real and personal property, but also of the P40,000 life insurance. The complaint prays a partition of all the property, both real and personal, left

by the deceased; that the defendant account for P21,634.80, and that that sum be divided equally among the plaintiffs and defendant along with the other property of deceased. The defendant denies the material allegations of the complaint and sets up as special defense and counterclaim that the redemption of the real estate sold by his father was made in the name of the plaintiffs and himself instead of in his name alone without his knowledge or consent; and that it was not his intention to use the proceeds of the insurance policy for the benefit of any person but himself, he alleging that he was and is the sole owner thereof and that it is his individual property. He, therefore, asks that he be declared the owner of the real estate redeemed by the payment of the P18,365.20, the owner of the remaining P21,634.80, the balance of the insurance policy, and that the plaintiff's account for the use and occupation of the premises so redeemed since the date of the redemption. The learned trial court refused to give relief to either party and dismissed the action. It says in its opinion: "This purports to be an action for partition, brought against an heir by his coheirs. The complaint, however, fails to comply with Code Civ., Pro. sec. 183, in that it does not 'contain an adequate description of the real property of which partition is demanded.' Because of this defect (which has not been called to our attention and was discovered only after the cause was submitted) it is more than doubtful whether any relief can be awarded under the complaint, except by agreement of all the parties." This alleged defect of the complaint was made one of the two bases for the dismissal of the action. We do not regard this as sufficient reason for dismissing the action. It is the doctrine of this court, set down in several decisions, Lizarraga Hermanos vs. Yap Tico, 24 Phil. Rep., 504, that, even though the complaint is defective to the extent of failing in allegations necessary to constitute a cause of action, if, on the trial of the cause, evidence is offered which establishes the cause of action which the complaint intended to allege, and such evidence is received without objection, the defect is thereby cured and cannot be made the ground of a subsequent objection. If, therefore, evidence was introduced on the trial in this case definitely and clearly describing the real estate sought to be partitioned, the defect in the complaint was cured in that regard and should not have been used to dismiss the action. We do not stop to inquire whether such evidence was or was not introduced on the trial, inasmuch as this case must be turned for a new trial with opportunity to both parties to present such evidence as is necessary to establish their respective claims. The court in its decision further says: "It will be noticed that the provision above quoted refers exclusively to real estate. . . . It is, in other words, an exclusive real property action, and the institution thereof gives the court no jurisdiction over chattels. . . . But no relief could possibly be granted in this action as to any property except the last (real estate), for the law contemplated that all the personal property of an estate be

distributed before the administration is closed. Indeed, it is only in exceptional cases that the partition of the real estate is provided for, and this too is evidently intended to be effected as a part of the administration, but here the complaint alleges that the estate was finally closed on December 9, 1911, and we find upon referring to the record in that case that subsequent motion to reopen the same were denied; so that the matter of the personal property at least must be considered res judicata (for the final judgment in the administration proceedings must be treated as concluding not merely what was adjudicated, but what might have been). So far, therefore, as the personal property at least is concerned, plaintiffs' only remedy was an appeal from said order." We do not believe that the law is correctly laid down in this quotation. The courts of the Islands have jurisdiction to divide personal property between the common owners thereof and that power is as full and complete as is the power to partition real property. If an actual partition of personal property cannot be made it will be sold under the direction of the court and the proceeds divided among the owners after the necessary expenses have been deducted. The administration of the estate of the decedent consisted simply, so far as the record shows, in the payment of the debts. No division of the property, either real or personal, seems to have been made. On the contrary, the property appears, from the record, to have been turned over to the heirs in bulk. The failure to partition the real property may have been due either to the lack of request to the court by one or more of the heirs to do so, as the court has no authority to make a partition of the real estate without such request; or it may have been due to the fact that all the real property of decedent had been sold under pacto de retro and that, therefore, he was not the owner of any real estate at the time of his death. As to the personal property, it does not appear that it was disposed of in the manner provided by law. (Sec. 753, Code of Civil Procedure.) So far as this action is concerned, however, it is sufficient for us to know that none of the property was actually divided among the heirs in the administration proceeding and that they remain coowners and tenants-in- common thereof at the present time. To maintain an action to partition real or personal property it is necessary to show only that it is owned in common. The order finally closing the administration and discharging the administrator, referred to in the opinion of the trial court, has nothing to do with the division of either the real or the personal property. The heirs have the right to ask the probate court to turn over to them both the real and personal property without division; and where that request is unanimous it is the duty of the court to comply with it, and there is nothing in section 753 of the Code of Civil Procedure which prohibits it. In such case an order finally settling the estate and discharging the administrator would not bar a subsequent action to require a division of either the real or personal property. If, on the other hand, an order had been made in the administration proceedings dividing the personal or the real property, or both, among the heirs, then it is quite possible that, to a subsequent action brought by one of the heirs for a partition of the real or personal property, or both, there could have been interposed a plea of res judicata based on such order. As the matter

now stands, however, there is no ground on which to base such a plea. Moreover, no such plea has been made and no evidence offered to support it. With the finding of the trial court that the proceeds of the life-insurance policy belong exclusively to the defendant as his individual and separate property, we agree. That the proceeds of an insurance policy belong exclusively to the beneficiary and not to the estate of the person whose life was insured, and that such proceeds are the separate and individual property of the beneficiary, and not of the heirs of the person whose life was insured, is the doctrine in America. We believe that the same doctrine obtains in these Islands by virtue of section 428 of the Code of Commerce, which reads: The amount which the underwriter must deliver to the person insured, in fulfillment of the contract, shall be the property of the latter, even against the claims of the legitimate heirs or creditors of any kind whatsoever of the person who effected the insurance in favor of the former. It is claimed by the attorney for the plaintiffs that the section just quoted is subordinate to the provisions of the Civil Code as found in article 1035. This article reads: An heir by force of law surviving with others of the same character to a succession must bring into the hereditary estate the property or securities he may have received from the deceased during the life of the same, by way of dowry, gift, or for any good consideration, in order to compute it in fixing the legal portions and in the account of the division. Counsel also claim that the proceeds of the insurance policy were a donation or gift made by the father during his lifetime to the defendant and that, as such, its ultimate destination is determined by those provisions of the Civil Code which relate to donations, especially article 819. This article provides that "gifts made to children which are not betterments shall be considered as part of their legal portion." We cannot agree with these contentions. The contract of life insurance is a special contract and the destination of the proceeds thereof is determined by special laws which deal exclusively with that subject. The Civil Code has no provisions which relate directly and specifically to life- insurance contracts or to the destination of life insurance proceeds. That subject is regulated exclusively by the Code of Commerce which provides for the terms of the contract, the relations of the parties and the destination of the proceeds of the policy. The proceeds of the life-insurance policy being the exclusive property of the defendant and he having used a portion thereof in the repurchase of the real estate sold by the decedent prior to his death with right to repurchase, and such repurchase having been made and the conveyance taken in the names of all of the heirs instead of the defendant alone, plaintiffs claim that the property belongs to the heirs in common and not to the defendant alone.

We are not inclined to agree with this contention unless the fact appear or be shown that the defendant acted as he did with the intention that the other heirs should enjoy with him the ownership of the estate in other words, that he proposed, in effect, to make a gift of the real estate to the other heirs. If it is established by the evidence that that was his intention and that the real estate was delivered to the plaintiffs with that understanding, then it is probable that their contention is correct and that they are entitled to share equally with the defendant therein. If, however, it appears from the evidence in the case that the conveyances were taken in the name of the plaintiffs without his knowledge or consent, or that it was not his intention to make a gift to them of the real estate, then it belongs to him. If that facts are as stated, he has two remedies. The one is to compel the plaintiffs to reconvey to him and the other is to let the title stand with them and to recover from them the sum he paid on their behalf. For the complete and proper determination of the questions at issue in this case, we are of the opinion that the cause should be returned to the trial court with instructions to permit the parties to frame such issues as will permit the settlement of all the questions involved and to introduce such evidence as may be necessary for the full determination of the issues framed. Upon such issues and evidence taken thereunder the court will decide the questions involved according to the evidence, subordinating his conclusions of law to the rules laid down in this opinion. We do not wish to be understood as having decided in this opinion any question of fact which will arise on the trial and be there in controversy. The trial court is left free to find the facts as the evidence requires. To the facts as so found he will apply the law as herein laid down. The judgment appealed from is set aside and the cause returned to the Court of First Instance whence it came for the purpose hereinabove stated. So ordered. Arellano, C.J., and Carson, J., concur. Torres, J., concurs in the result.

Separate Opinions ARAULLO, J., concurring:

I concur in the result and with the reasoning of the foregoing decision, only in so far as concerns the return of the record to the lower court in order that it fully and correctly decide all the issues raised therein, allow the parties to raise such questions as may help to decide all those involved in the case, and to present such evidence as they may deem requisite for a complete resolution of all the issues in discussion, because it is my opinion that it is inopportune to make, and there should not be made in the said majority decision the findings therein set forth in connection with articles 428 of the Code of Commerce and 1035 of the Civil Code, in order to arrive at the conclusion that the amount of the insurance policy referred to belongs exclusively to the defendant, inasmuch a this is one of the questions which, according to the decision itself, should be decided by the lower court after an examination of the evidence introduced by the parties; it is the lower court that should make those findings, which ought afterwards to be submitted to this court, if any appeal be taken from the judgment rendered in the case by the trial court in compliance with the foregoing decision.

G.R. No. L-44059 October 28, 1977 THE INSULAR LIFE ASSURANCE COMPANY, LTD., plaintiff-appellee, vs. CARPONIA T. EBRADO and PASCUALA VDA. DE EBRADO, defendants-appellants.

MARTIN, J.: This is a novel question in insurance law: Can a common-law wife named as beneficiary in the life insurance policy of a legally married man claim the proceeds thereof in case of death of the latter? On September 1, 1968, Buenaventura Cristor Ebrado was issued by The Life Assurance Co., Ltd., Policy No. 009929 on a whole-life for P5,882.00 with a, rider for Accidental Death for the same amount Buenaventura C. Ebrado designated T. Ebrado as the revocable beneficiary in his policy. He to her as his wife. On October 21, 1969, Buenaventura C. Ebrado died as a result of an t when he was hit by a failing branch of a tree. As the policy was in force, The Insular Life Assurance Co., Ltd. liable to pay the coverage in the total amount of P11,745.73, representing the face value of the policy in the amount of P5,882.00 plus the additional benefits for accidental death also in the amount of P5,882.00 and the refund of P18.00 paid for the premium due November, 1969, minus the unpaid premiums and interest thereon due for January and February, 1969, in the sum of P36.27. Carponia T. Ebrado filed with the insurer a claim for the proceeds of the Policy as the designated beneficiary therein, although she admits that she and the insured Buenaventura C. Ebrado were merely living as husband and wife without the benefit of marriage. Pascuala Vda. de Ebrado also filed her claim as the widow of the deceased insured. She asserts that she is the one entitled to the insurance proceeds, not the common-law wife, Carponia T. Ebrado. In doubt as to whom the insurance proceeds shall be paid, the insurer, The Insular Life Assurance Co., Ltd. commenced an action for Interpleader before the Court of First Instance of Rizal on April 29, 1970. After the issues have been joined, a pre-trial conference was held on July 8, 1972, after which, a pre-trial order was entered reading as follows: +.wph!1 During the pre-trial conference, the parties manifested to the court. that there is no possibility of amicable settlement. Hence, the Court proceeded to have the parties submit their evidence for the purpose of the pre-trial

and make admissions for the purpose of pretrial. During this conference, parties Carponia T. Ebrado and Pascuala Ebrado agreed and stipulated: 1) that the deceased Buenaventura Ebrado was married to Pascuala Ebrado with whom she has six (legitimate) namely; Hernando, Cresencio, Elsa, Erlinda, Felizardo and Helen, all surnamed Ebrado; 2) that during the lifetime of the deceased, he was insured with Insular Life Assurance Co. Under Policy No. 009929 whole life plan, dated September 1, 1968 for the sum of P5,882.00 with the rider for accidental death benefit as evidenced by Exhibits A for plaintiffs and Exhibit 1 for the defendant Pascuala and Exhibit 7 for Carponia Ebrado; 3) that during the lifetime of Buenaventura Ebrado, he was living with his common-wife, Carponia Ebrado, with whom she had 2 children although he was not legally separated from his legal wife; 4) that Buenaventura in accident on October 21, 1969 as evidenced by the death Exhibit 3 and affidavit of the police report of his death Exhibit 5; 5) that complainant Carponia Ebrado filed claim with the Insular Life Assurance Co. which was contested by Pascuala Ebrado who also filed claim for the proceeds of said policy 6) that in view ofthe adverse claims the insurance company filed this action against the two herein claimants Carponia and Pascuala Ebrado; 7) that there is now due from the Insular Life Assurance Co. as proceeds of the policy P11,745.73; 8) that the beneficiary designated by the insured in the policy is Carponia Ebrado and the insured made reservation to change the beneficiary but although the insured made the option to change the beneficiary, same was never changed up to the time of his death and the wife did not have any opportunity to write the company that there was reservation to change the designation of the parties agreed that a decision be rendered based on and stipulation of facts as to who among the two claimants is entitled to the policy. Upon motion of the parties, they are given ten (10) days to file their simultaneous memoranda from the receipt of this order. SO ORDERED. On September 25, 1972, the trial court rendered judgment declaring among others, Carponia T. Ebrado disqualified from becoming beneficiary of the insured Buenaventura Cristor Ebrado and directing the payment of the insurance proceeds to the estate of the deceased insured. The trial court held: +.wph!1 It is patent from the last paragraph of Art. 739 of the Civil Code that a criminal conviction for adultery or concubinage is not essential in order to establish the disqualification mentioned therein. Neither is it also necessary that a finding of such guilt or commission of those acts be made in a separate independent action brought for the purpose. The guilt of the donee (beneficiary) may be proved by preponderance of evidence

in the same proceeding (the action brought to declare the nullity of the donation). It is, however, essential that such adultery or concubinage exists at the time defendant Carponia T. Ebrado was made beneficiary in the policy in question for the disqualification and incapacity to exist and that it is only necessary that such fact be established by preponderance of evidence in the trial. Since it is agreed in their stipulation above-quoted that the deceased insured and defendant Carponia T. Ebrado were living together as husband and wife without being legally married and that the marriage of the insured with the other defendant Pascuala Vda. de Ebrado was valid and still existing at the time the insurance in question was purchased there is no question that defendant Carponia T. Ebrado is disqualified from becoming the beneficiary of the policy in question and as such she is not entitled to the proceeds of the insurance upon the death of the insured. From this judgment, Carponia T. Ebrado appealed to the Court of Appeals, but on July 11, 1976, the Appellate Court certified the case to Us as involving only questions of law. We affirm the judgment of the lower court. 1. It is quite unfortunate that the Insurance Act (RA 2327, as amended) or even the new Insurance Code (PD No. 612, as amended) does not contain any specific provision grossly resolutory of the prime question at hand. Section 50 of the Insurance Act which provides that "(t)he insurance shag be applied exclusively to the proper interest of the person in whose name it is made" 1 cannot be validly seized upon to hold that the mm includes the beneficiary. The word "interest" highly suggests that the provision refers only to the "insured" and not to the beneficiary, since a contract of insurance is personal in character. 2 Otherwise, the prohibitory laws against illicit relationships especially on property and descent will be rendered nugatory, as the same could easily be circumvented by modes of insurance. Rather, the general rules of civil law should be applied to resolve this void in the Insurance Law. Article 2011 of the New Civil Code states: "The contract of insurance is governed by special laws. Matters not expressly provided for in such special laws shall be regulated by this Code." When not otherwise specifically provided for by the Insurance Law, the contract of life insurance is governed by the general rules of the civil law regulating contracts. 3 And under Article 2012 of the same Code, "any person who is forbidden from receiving any donation under Article 739 cannot be named beneficiary of a fife insurance policy by the person who cannot make a donation to him. 4 Common-law spouses are, definitely, barred from receiving donations from each other. Article 739 of the new Civil Code provides: +.wph!1 The following donations shall be void: 1. Those made between persons who were guilty of adultery or concubinage at the time of donation;

Those made between persons found guilty of the same criminal offense, in consideration thereof; 3. Those made to a public officer or his wife, descendants or ascendants by reason of his office. In the case referred to in No. 1, the action for declaration of nullity may be brought by the spouse of the donor or donee; and the guilt of the donee may be proved by preponderance of evidence in the same action. 2. In essence, a life insurance policy is no different from a civil donation insofar as the beneficiary is concerned. Both are founded upon the same consideration: liberality. A beneficiary is like a donee, because from the premiums of the policy which the insured pays out of liberality, the beneficiary will receive the proceeds or profits of said insurance. As a consequence, the proscription in Article 739 of the new Civil Code should equally operate in life insurance contracts. The mandate of Article 2012 cannot be laid aside: any person who cannot receive a donation cannot be named as beneficiary in the life insurance policy of the person who cannot make the donation.5 Under American law, a policy of life insurance is considered as a testament and in construing it, the courts will, so far as possible treat it as a will and determine the effect of a clause designating the beneficiary by rules under which wins are interpreted. 6 3. Policy considerations and dictates of morality rightly justify the institution of a barrier between common law spouses in record to Property relations since such hip ultimately encroaches upon the nuptial and filial rights of the legitimate family There is every reason to hold that the bar in donations between legitimate spouses and those between illegitimate ones should be enforced in life insurance policies since the same are based on similar consideration As above pointed out, a beneficiary in a fife insurance policy is no different from a donee. Both are recipients of pure beneficence. So long as manage remains the threshold of family laws, reason and morality dictate that the impediments imposed upon married couple should likewise be imposed upon extra-marital relationship. If legitimate relationship is circumscribed by these legal disabilities, with more reason should an illicit relationship be restricted by these disabilities. Thus, in Matabuena v. Cervantes, 7 this Court, through Justice Fernando, said: +.wph!1 If the policy of the law is, in the language of the opinion of the then Justice J.B.L. Reyes of that court (Court of Appeals), 'to prohibit donations in favor of the other consort and his descendants because of and undue and improper pressure and influence upon the donor, a prejudice deeply rooted in our ancient law;" por-que no se enganen desponjandose el uno al otro por amor que han de consuno' (According to) the Partidas (Part IV, Tit. XI, LAW IV), reiterating the rationale 'No Mutuato amore invicem spoliarentur' the Pandects (Bk, 24, Titl. 1, De donat, inter virum et uxorem); then there is very reason to apply the same prohibitive policy to persons living together as husband and wife without the benefit of

nuptials. For it is not to be doubted that assent to such irregular connection for thirty years bespeaks greater influence of one party over the other, so that the danger that the law seeks to avoid is correspondingly increased. Moreover, as already pointed out by Ulpian (in his lib. 32 ad Sabinum, fr. 1), 'it would not be just that such donations should subsist, lest the condition 6f those who incurred guilt should turn out to be better.' So long as marriage remains the cornerstone of our family law, reason and morality alike demand that the disabilities attached to marriage should likewise attach to concubinage. It is hardly necessary to add that even in the absence of the above pronouncement, any other conclusion cannot stand the test of scrutiny. It would be to indict the frame of the Civil Code for a failure to apply a laudable rule to a situation which in its essentials cannot be distinguished. Moreover, if it is at all to be differentiated the policy of the law which embodies a deeply rooted notion of what is just and what is right would be nullified if such irregular relationship instead of being visited with disabilities would be attended with benefits. Certainly a legal norm should not be susceptible to such a reproach. If there is every any occasion where the principle of statutory construction that what is within the spirit of the law is as much a part of it as what is written, this is it. Otherwise the basic purpose discernible in such codal provision would not be attained. Whatever omission may be apparent in an interpretation purely literal of the language used must be remedied by an adherence to its avowed objective. 4. We do not think that a conviction for adultery or concubinage is exacted before the disabilities mentioned in Article 739 may effectuate. More specifically, with record to the disability on "persons who were guilty of adultery or concubinage at the time of the donation," Article 739 itself provides: +.wph!1 In the case referred to in No. 1, the action for declaration of nullity may be brought by the spouse of the donor or donee; and the guilty of the donee may be proved by preponderance of evidence in the same action. The underscored clause neatly conveys that no criminal conviction for the offense is a condition precedent. In fact, it cannot even be from the aforequoted provision that a prosecution is needed. On the contrary, the law plainly states that the guilt of the party may be proved "in the same acting for declaration of nullity of donation. And, it would be sufficient if evidence preponderates upon the guilt of the consort for the offense indicated. The quantum of proof in criminal cases is not demanded. In the caw before Us, the requisite proof of common-law relationship between the insured and the beneficiary has been conveniently supplied by the stipulations between the parties in the pre-trial conference of the case. It case agreed upon and stipulated therein that the deceased insured Buenaventura C. Ebrado was married to Pascuala

Ebrado with whom she has six legitimate children; that during his lifetime, the deceased insured was living with his common-law wife, Carponia Ebrado, with whom he has two children. These stipulations are nothing less thanjudicial admissions which, as a consequence, no longer require proof and cannot be contradicted. 8 A fortiori, on the basis of these admissions, a judgment may be validly rendered without going through the rigors of a trial for the sole purpose of proving the illicit liaison between the insured and the beneficiary. In fact, in that pretrial, the parties even agreed "that a decision be rendered based on this agreement and stipulation of facts as to who among the two claimants is entitled to the policy." ACCORDINGLY, the appealed judgment of the lower court is hereby affirmed. Carponia T. Ebrado is hereby declared disqualified to be the beneficiary of the late Buenaventura C. Ebrado in his life insurance policy. As a consequence, the proceeds of the policy are hereby held payable to the estate of the deceased insured. Costs against Carponia T. Ebrado. SO ORDERED. Teehankee (Chairman), Makasiar, Mu;oz Palma, Fernandez and Guerrero, JJ., concur.1wph1.t

G.R. No. 198588

July 11, 2012

UNITED MERCHANTS CORPORATION, Petitioner, vs. COUNTRY BANKERS INSURANCE CORPORATION, Respondent. DECISION CARPIO, J.:

The Case This Petition for Review on Certiorari1 seeks to reverse the Court of Appeals Decision2 dated 16 June 2011 and its Resolution3 dated 8 September 2011 in CA-G.R. CV No. 85777. The Court of Appeals reversed the Decision4of the Regional Trial Court (RTC) of Manila, Branch 3, and ruled that the claim on the Insurance Policy is void. The Facts The facts, as culled from the records, are as follows: Petitioner United Merchants Corporation (UMC) is engaged in the business of buying, selling, and manufacturing Christmas lights. UMC leased a warehouse at 19-B Dagot Street, San Jose Subdivision, Barrio Manresa, Quezon City, where UMC assembled and stored its products. On 6 September 1995, UMCs General Manager Alfredo Tan insured UMCs stocks in trade of Christmas lights against fire with defendant Country Bankers Insurance Corporation (CBIC) for P15,000,000.00. The Fire Insurance Policy No. F-HO/95-576 (Insurance Policy) and Fire Invoice No. 12959A, valid until 6 September 1996, states: AMOUNT OF INSURANCE: FIFTEEN MILLION PESOS PHILIPPINE CURRENCY xxx PROPERTY INSURED: On stocks in trade only, consisting of Christmas Lights, the properties of the Assured or held by them in trust, on commissions, or on joint account with others and/or for which they are responsible in the event of loss and/or damage during the currency of this policy, whilst contained in the building of one lofty storey in height, constructed of concrete and/or hollow blocks with portion of galvanized iron sheets, under galvanized iron rood, occupied as Christmas lights storage. 5 On 7 May 1996, UMC and CBIC executed Endorsement F/96-154 and Fire Invoice No. 16583A to form part of the Insurance Policy. Endorsement F/96-154 provides that UMCs stocks in trade were insured against additional perils, to wit: "typhoon, flood, ext. cover, and full earthquake." The sum insured was also increased toP50,000,000.00 effective 7 May 1996 to 10 January 1997. On 9 May 1996, CBIC issued Endorsement F/96-157 where the name of the assured was changed from Alfredo Tan to UMC. On 3 July 1996, a fire gutted the warehouse rented by UMC. CBIC designated CRM Adjustment Corporation (CRM) to investigate and evaluate UMCs loss by reason of the fire. CBICs reinsurer, Central Surety, likewise requested the National Bureau of

Investigation (NBI) to conduct a parallel investigation. On 6 July 1996, UMC, through CRM, submitted to CBIC its Sworn Statement of Formal Claim, with proofs of its loss. On 20 November 1996, UMC demanded for at least fifty percent (50%) payment of its claim from CBIC. On 25 February 1997, UMC received CBICs letter, dated 10 January 1997, rejecting UMCs claim due to breach of Condition No. 15 of the Insurance Policy. Condition No. 15 states: If the claim be in any respect fraudulent, or if any false declaration be made or used in support thereof, or if any fraudulent means or devices are used by the Insured or anyone acting in his behalf to obtain any benefit under this Policy; or if the loss or damage be occasioned by the willful act, or with the connivance of the Insured, all the benefits under this Policy shall be forfeited.6 On 19 February 1998, UMC filed a Complaint7 against CBIC with the RTC of Manila. UMC anchored its insurance claim on the Insurance Policy, the Sworn Statement of Formal Claim earlier submitted, and the Certification dated 24 July 1996 made by Deputy Fire Chief/Senior Superintendent Bonifacio J. Garcia of the Bureau of Fire Protection. The Certification dated 24 July 1996 provides that: This is to certify that according to available records of this office, on or about 6:10 P.M. of July 3, 1996, a fire broke out at United Merchants Corporation located at 19-B Dag[o]t Street, Brgy. Manresa, Quezon City incurring an estimated damage of Fifty-Five Million Pesos (P55,000,000.00) to the building and contents, while the reported insurance coverage amounted to Fifty Million Pesos (P50,000,000.00) with Country Bankers Insurance Corporation. The Bureau further certifies that no evidence was gathered to prove that the establishment was willfully, feloniously and intentionally set on fire. That the investigation of the fire incident is already closed being ACCIDENTAL in nature.8 In its Answer with Compulsory Counterclaim9 dated 4 March 1998, CBIC admitted the issuance of the Insurance Policy to UMC but raised the following defenses: (1) that the Complaint states no cause of action; (2) that UMCs claim has already prescribed; and (3) that UMCs fire claim is tainted with fraud. CBIC alleged that UMCs claim was fraudulent because UMCs Statement of Inventory showed that it had no stocks in trade as of 31 December 1995, and that UMCs suspicious purchases for the year 1996 did not even amount to P25,000,000.00. UMCs GIS and Financial Reports further revealed that it had insufficient capital, which meant UMC could not afford the allegedP50,000,000.00 worth of stocks in trade. In its Reply10 dated 20 March 1998, UMC denied violation of Condition No. 15 of the Insurance Policy. UMC claimed that it did not make any false declaration because the

invoices were genuine and the Statement of Inventory was for internal revenue purposes only, not for its insurance claim. During trial, UMC presented five witnesses. The first witness was Josie Ebora (Ebora), UMCs disbursing officer. Ebora testified that UMCs stocks in trade, at the time of the fire, consisted of: (1) raw materials for its Christmas lights; (2) Christmas lights already assembled; and (3) Christmas lights purchased from local suppliers. These stocks in trade were delivered from August 1995 to May 1996. She stated that Straight Cargo Commercial Forwarders delivered the imported materials to the warehouse, evidenced by delivery receipts. However, for the year 1996, UMC had no importations and only bought from its local suppliers. Ebora identified the suppliers as Fiber Technology Corporation from which UMC bought stocks worth P1,800,000.00 on 20 May 1996; Fuze Industries Manufacturer Philippines from which UMC bought stocks worth P19,500,000.00 from 20 January 1996 to 23 February 1996; and Tomco Commercial Press from which UMC bought several Christmas boxes. Ebora testified that all these deliveries were not yet paid. Ebora also presented UMCs Balance Sheet, Income Statement and Statement of Cash Flow. Per her testimony, UMCs purchases amounted to P608,986.00 in 1994;P827,670.00 in 1995; and P20,000,000.00 in 1996. Ebora also claimed that UMC had sales only from its fruits business but no sales from its Christmas lights for the year 1995. The next witness, Annie Pabustan (Pabustan), testified that her company provided about 25 workers to assemble and pack Christmas lights for UMC from 28 March 1996 to 3 July 1996. The third witness, Metropolitan Bank and Trust Company (MBTC) Officer Cesar Martinez, stated that UMC opened letters of credit with MBTC for the year 1995 only. The fourth witness presented was Ernesto Luna (Luna), the delivery checker of Straight Commercial Cargo Forwarders. Luna affirmed the delivery of UMCs goods to its warehouse on 13 August 1995, 6 September 1995, 8 September 1995, 24 October 1995, 27 October 1995, 9 November 1995, and 19 December 1995. Lastly, CRMs adjuster Dominador Victorio testified that he inspected UMCs warehouse and prepared preliminary reports in this connection. On the other hand, CBIC presented the claims manager Edgar Caguindagan (Caguindagan), a Securities and Exchange Commission (SEC) representative, Atty. Ernesto Cabrera (Cabrera), and NBI Investigator Arnold Lazaro (Lazaro). Caguindagan testified that he inspected the burned warehouse on 5 July 1996, took pictures of it and referred the claim to an independent adjuster. The SEC representatives testimony was dispensed with, since the parties stipulated on the existence of certain documents, to wit: (1) UMCs GIS for 1994-1997; (2) UMCs Financial Report as of 31 December 1996; (3) SEC Certificate that UMC did not file GIS or Financial Reports for certain years; and (4) UMCs Statement of Inventory as of 31 December 1995 filed with the BIR. Cabrera and Lazaro testified that they were hired by Central Surety to investigate UMCs claim. On 19 November 1996, they concluded that arson was committed based from their interview with barangay officials and the pictures showing that blackened surfaces were present at different parts of the warehouse. On cross-examination,

Lazaro admitted that they did not conduct a forensic investigation of the warehouse, nor did they file a case for arson. For rebuttal, UMC presented Rosalinda Batallones (Batallones), keeper of the documents of UCPB General Insurance, the insurer of Perfect Investment Company, Inc., the warehouse owner. When asked to bring documents related to the insurance of Perfect Investment Company, Inc., Batallones brought the papers of Perpetual Investment, Inc. The Ruling of the Regional Trial Court On 16 June 2005, the RTC of Manila, Branch 3, rendered a Decision in favor of UMC, the dispositive portion of which reads: WHEREFORE, judgment is hereby rendered in favor of plaintiff and ordering defendant to pay plaintiff: a) the sum of P43,930,230.00 as indemnity with interest thereon at 6% per annum from November 2003 until fully paid; b) the sum of P100,000.00 for exemplary damages; c) the sum of P100,000.00 for attorneys fees; and d) the costs of suit. Defendants counterclaim is denied for lack of merit. SO ORDERED.11 The RTC found no dispute as to UMCs fire insurance contract with CBIC. Thus, the RTC ruled for UMCs entitlement to the insurance proceeds, as follows: Fraud is never presumed but must be proved by clear and convincing evidence. (see Alonso v. Cebu Country Club, 417 SCRA 115 [2003]) Defendant failed to establish by clear and convincing evidence that the documents submitted to the SEC and BIR were true. It is common business practice for corporations to have 2 sets of reports/statements for tax purposes. The stipulated documents of plaintiff (Exhs. 2 8) may not have been accurate. The conflicting findings of defendants adjuster, CRM Adjustment [with stress] and that made by Atty. Cabrera & Mr. Lazaro for Central Surety shall be resolved in favor of the former. Definitely the formers finding is more credible as it was made soon after the fire while that of the latter was done 4 months later. Certainly it would be a different situation as the site was no longer the same after the clearing up operation which is normal after a fire incident. The Christmas lights and parts could have been swept away. Hence the finding of the latter appears to be speculative to benefit the reinsurer and which defendant wants to adopt to avoid liability.

The CRM Adjustment report found no arson and confirmed substantial stocks in the burned warehouse (Exhs. QQQ) [underscoring supplied]. This is bolstered by the BFP certification that there was no proof of arson and the fire was accidental (Exhs. PPP). The certification by a government agency like BFP is presumed to be a regular performance of official duty. "Absent convincing evidence to the contrary, the presumption of regularity in the performance of official functions has to be upheld." (People vs. Lapira, 255 SCRA 85) The report of UCPB General Insurances adjuster also found no arson so that the burned warehouse owner PIC was indemnified. 12 Hence, CBIC filed an appeal with the Court of Appeals (CA). The Ruling of the Court of Appeals On 16 June 2011, the CA promulgated its Decision in favor of CBIC. The dispositive portion of the Decision reads: WHEREFORE, in view of the foregoing premises, the instant appeal is GRANTED and the Decision of the Regional Trial Court, of the National Judicial Capital Region, Branch 3 of the City of Manila dated June 16, 2005 in Civil Case No. 98-87370 is REVERSED and SET ASIDE. The plaintiff-appellees claim upon its insurance policy is deemed avoided. SO ORDERED.13 The CA ruled that UMCs claim under the Insurance Policy is void. The CA found that the fire was intentional in origin, considering the array of evidence submitted by CBIC, particularly the pictures taken and the reports of Cabrera and Lazaro, as opposed to UMCs failure to explain the details of the alleged fire accident. In addition, it found that UMCs claim was overvalued through fraudulent transactions. The CA ruled: We have meticulously gone over the entirety of the evidence submitted by the parties and have come up with a conclusion that the claim of the plaintiff-appellee was indeed overvalued by transactions which were fraudulently concocted so that the full coverage of the insurance policy will have to be fully awarded to the plaintiff-appellee. First, We turn to the backdrop of the plaintiff-appellees case, thus, [o]n September 6, 1995 its stocks-in-trade were insured for Fifteen Million Pesos and on May 7, 1996 the same was increased to 50 Million Pesos. Two months thereafter, a fire gutted the plaintiff-appellees warehouse. Second, We consider the reported purchases of the plaintiff-appellee as shown in its financial report dated December 31, 1996 vis--vis the testimony of Ms. Ebora thus: 1994 - P608,986.00 1995 - P827,670.00

1996 - P20,000,000.00 (more or less) which were purchased for a period of one month. Third, We shall also direct our attention to the alleged true and complete purchases of the plaintiff-appellee as well as the value of all stock-in-trade it had at the time that the fire occurred. Thus: Exhibit Exhs. "P"-"DD", inclusive Source Fuze Industries Manufacturer Phils. Amount (pesos) 19,550,400.00 Dates Covered January 20, 1996 January 31, 1996 February 12, 1996 February 20, 1996 February 23, 1996 December 19, 1995 January 24, 1996 February 21, 1996 November 24, 1995 January 13, 1996 January 19, 1996 January 26, 1996 February 3, 1996 February 13, 1996 February 20, 1996 February 27, 1996 April 3, 1996 April 12,

Exhs. "EE""HH", inclusive

Tomco Commercial Press

1,712,000.00

Exhs. "II"-"QQ", inclusive

Precious Belen Trading

2,720,400.00

Exhs. "RR""EEE", inclusive

Wisdom Manpower

361,966.00

Services

1996 April 19, 1996 April 26, 1996 May 3, 1996 May 10, 1996 May 17, 1996 May 24, 1996 June 7, 1996 June 14, 1996 June 21, 1996 June 28, 1996 July 5, 1996 15,159,144.71 May 29, 1995 June 15, 1995 July 5, 1995 September 4, 1995 October 2, 1995 October 27, 1995 January 8, 1996 March 19, 1996 June 15, 1995 June 28, 1995 August 1, 1995 September 4, 1995 September 8, 1995 September 11, 1995 October 30,

Exhs. "GGG"Costs of Letters "NNN", inclusive of Credit for imported raw materials

Exhs. "GGG-11" SCCFI - "GGG-24", statements of "HHH-12", account "HHH-22", "III11", "III-14", "JJJ-13", "KKK11", "LLL-5"

384,794.38

199[5] November 10, 1995 December 21, 1995 TOTAL 44,315,024.31

Fourth, We turn to the allegation of fraud by the defendant-appellant by thoroughly looking through the pieces of evidence that it adduced during the trial. The latter alleged that fraud is present in the case at bar as shown by the discrepancy of the alleged purchases from that of the reported purchases made by plaintiff-appellee. It had also averred that fraud is present when upon verification of the address of Fuze Industries, its office is nowhere to be found. Also, the defendant-appellant expressed grave doubts as to the purchases of the plaintiff-appellee sometime in 1996 when such purchases escalated to a high 19.5 Million Pesos without any contract to back it up. 14 On 7 July 2011, UMC filed a Motion for Reconsideration,15 which the CA denied in its Resolution dated 8 September 2011. Hence, this petition. The Issues UMC seeks a reversal and raises the following issues for resolution: I. WHETHER THE COURT OF APPEALS MADE A RULING INCO[N]SISTENT WITH LAW, APPLICABLE JURISPRUDENCE AND EVIDENCE AS TO THE EXISTENCE OF ARSON AND FRAUD IN THE ABSENCE OF "MATERIALLY CONVINCING EVIDENCE." II. WHETHER THE COURT OF APPEALS MADE A RULING INCONSISTENT WITH LAW, APPLICABLE JURISPRUDENCE AND EVIDENCE WHEN IT FOUND THAT PETITIONER BREACHED ITS WARRANTY.16 The Ruling of the Court At the outset, CBIC assails this petition as defective since what UMC ultimately wants this Court to review are questions of fact. However, UMC argues that where the findings of the CA are in conflict with those of the trial court, a review of the facts may be made. On this procedural issue, we find UMCs claim meritorious. A petition for review under Rule 45 of the Rules of Court specifically provides that only questions of law may be raised. The findings of fact of the CA are final and conclusive

and this Court will not review them on appeal,17subject to exceptions as when the findings of the appellate court conflict with the findings of the trial court.18Clearly, the present case falls under the exception. Since UMC properly raised the conflicting findings of the lower courts, it is proper for this Court to resolve such contradiction. Having settled the procedural issue, we proceed to the primordial issue which boils down to whether UMC is entitled to claim from CBIC the full coverage of its fire insurance policy. UMC contends that because it had already established a prima facie case against CBIC which failed to prove its defense, UMC is entitled to claim the full coverage under the Insurance Policy. On the other hand, CBIC contends that because arson and fraud attended the claim, UMC is not entitled to recover under Condition No. 15 of the Insurance Policy. Burden of proof is the duty of any party to present evidence to establish his claim or defense by the amount of evidence required by law,19 which is preponderance of evidence in civil cases.20 The party, whether plaintiff or defendant, who asserts the affirmative of the issue has the burden of proof to obtain a favorable judgment.21Particularly, in insurance cases, once an insured makes out a prima facie case in its favor, the burden of evidence shifts to the insurer to controvert the insureds prima facie case.22 In the present case, UMC established a prima facie case against CBIC. CBIC does not dispute that UMCs stocks in trade were insured against fire under the Insurance Policy and that the warehouse, where UMCs stocks in trade were stored, was gutted by fire on 3 July 1996, within the duration of the fire insurance. However, since CBIC alleged an excepted risk, then the burden of evidence shifted to CBIC to prove such exception.1wphi1 An insurer who seeks to defeat a claim because of an exception or limitation in the policy has the burden of establishing that the loss comes within the purview of the exception or limitation.23 If loss is proved apparently within a contract of insurance, the burden is upon the insurer to establish that the loss arose from a cause of loss which is excepted or for which it is not liable, or from a cause which limits its liability. 24 In the present case, CBIC failed to discharge its primordial burden of establishing that the damage or loss was caused by arson, a limitation in the policy. In prosecutions for arson, proof of the crime charged is complete where the evidence establishes: (1) the corpus delicti, that is, a fire caused by a criminal act; and (2) the identity of the defendants as the one responsible for the crime.25 Corpus delicti means the substance of the crime, the fact that a crime has actually been committed. 26This is satisfied by proof of the bare occurrence of the fire and of its having been intentionally caused.27 In the present case, CBICs evidence did not prove that the fire was intentionally caused by the insured. First, the findings of CBICs witnesses, Cabrera and Lazaro, were based on an investigation conducted more than four months after the fire. The testimonies of

Cabrera and Lazaro, as to the boxes doused with kerosene as told to them by barangay officials, are hearsay because the barangay officials were not presented in court. Cabrera and Lazaro even admitted that they did not conduct a forensic investigation of the warehouse nor did they file a case for arson.28 Second, the Sworn Statement of Formal Claim submitted by UMC, through CRM, states that the cause of the fire was "faulty electrical wiring/accidental in nature." CBIC is bound by this evidence because in its Answer, it admitted that it designated CRM to evaluate UMCs loss. Third, the Certification by the Bureau of Fire Protection states that the fire was accidental in origin. This Certification enjoys the presumption of regularity, which CBIC failed to rebut. Contrary to UMCs allegation, CBICs failure to prove arson does not mean that it also failed to prove fraud. Qua Chee Gan v. Law Union29 does not apply in the present case. In Qua Chee Gan,30 the Court dismissed the allegation of fraud based on the dismissal of the arson case against the insured, because the evidence was identical in both cases, thus: While the acquittal of the insured in the arson case is not res judicata on the present civil action, the insurers evidence, to judge from the decision in the criminal case, is practically identical in both cases and must lead to the same result, since the proof to establish the defense of connivance at the fire in order to defraud the insurer "cannot be materially less convincing than that required in order to convict the insured of the crime of arson" (Bachrach vs. British American Assurance Co., 17 Phil. 536). 31 In the present case, arson and fraud are two separate grounds based on two different sets of evidence, either of which can void the insurance claim of UMC. The absence of one does not necessarily result in the absence of the other. Thus, on the allegation of fraud, we affirm the findings of the Court of Appeals. Condition No. 15 of the Insurance Policy provides that all the benefits under the policy shall be forfeited, if the claim be in any respect fraudulent, or if any false declaration be made or used in support thereof, to wit: 15. If the claim be in any respect fraudulent, or if any false declaration be made or used in support thereof, or if any fraudulent means or devices are used by the Insured or anyone acting in his behalf to obtain any benefit under this Policy; or if the loss or damage be occasioned by the willful act, or with the connivance of the Insured, all the benefits under this Policy shall be forfeited. In Uy Hu & Co. v. The Prudential Assurance Co., Ltd.,32 the Court held that where a fire insurance policy provides that "if the claim be in any respect fraudulent, or if any false declaration be made or used in support thereof, or if any fraudulent means or devices are used by the Insured or anyone acting on his behalf to obtain any benefit under this Policy," and the evidence is conclusive that the proof of claim which the insured submitted was false and fraudulent both as to the kind, quality and amount of the goods

and their value destroyed by the fire, such a proof of claim is a bar against the insured from recovering on the policy even for the amount of his actual loss. In the present case, as proof of its loss of stocks in trade amounting to P50,000,000.00, UMC submitted its Sworn Statement of Formal Claim together with the following documents: (1) letters of credit and invoices for raw materials, Christmas lights and cartons purchased; (2) charges for assembling the Christmas lights; and (3) delivery receipts of the raw materials. However, the charges for assembling the Christmas lights and delivery receipts could not support its insurance claim. The Insurance Policy provides that CBIC agreed to insure UMCs stocks in trade. UMC defined stock in trade as tangible personal property kept for sale or traffic.33 Applying UMCs definition, only the letters of credit and invoices for raw materials, Christmas lights and cartons may be considered. The invoices, however, cannot be taken as genuine. The invoices reveal that the stocks in trade purchased for 1996 amounts to P20,000,000.00 which were purchased in one month. Thus, UMC needs to prove purchases amounting to P30,000,000.00 worth of stocks in trade for 1995 and prior years. However, in the Statement of Inventory it submitted to the BIR, which is considered an entry in official records,34 UMC stated that it had no stocks in trade as of 31 December 1995. In its defense, UMC alleged that it did not include as stocks in trade the raw materials to be assembled as Christmas lights, which it had on 31 December 1995. However, as proof of its loss, UMC submitted invoices for raw materials, knowing that the insurance covers only stocks in trade. Equally important, the invoices (Exhibits "P"-"DD") from Fuze Industries Manufacturer Phils. were suspicious. The purchases, based on the invoices and without any supporting contract, amounted to P19,550,400.00 worth of Christmas lights from 20 January 1996 to 23 February 1996. The uncontroverted testimony of Cabrera revealed that there was no Fuze Industries Manufacturer Phils. located at "55 Mahinhin St., Teachers Village, Quezon City," the business address appearing in the invoi ces and the records of the Department of Trade & Industry. Cabrera testified that: A: Then we went personally to the address as I stated a while ago appearing in the record furnished by the United Merchants Corporation to the adjuster, and the adjuster in turn now, gave us our basis in conducting investigation, so we went to this place which according to the records, the address of this company but there was no office of this company. Q: You mentioned Atty. Cabrera that you went to Diliman, Quezon City and discover the address indicated by the United Merchants as the place of business of Fuze Industries Manufacturer, Phils. was a residential place, what then did you do after determining that it was a residential place? A: We went to the owner of the alleged company as appearing in the Department of Trade & Industry record, and as appearing a certain Chinese name Mr. Huang, and the address as appearing there is somewhere in Binondo. We went personally there

together with the NBI Agent and I am with them when the subpoena was served to them, but a male person approached us and according to him, there was no Fuze Industries Manufacturer, Phils., company in that building sir.35 In Yu Ban Chuan v. Fieldmens Insurance, Co., Inc.,36 the Court ruled that the submission of false invoices to the adjusters establishes a clear case of fraud and misrepresentation which voids the insurers liability as per condition of the policy. Their falsity is the best evidence of the fraudulent character of plaintiffs claim. 37 InVerendia v. Court of Appeals,38 where the insured presented a fraudulent lease contract to support his claim for insurance benefits, the Court held that by its false declaration, the insured forfeited all benefits under the policy provision similar to Condition No. 15 of the Insurance Policy in this case. Furthermore, UMCs Income Statement indicated that the purchases or costs of sales are P827,670.00 for 1995 and P1,109,190.00 for 1996 or a total of P1,936,860.00.39 To corroborate this fact, Ebora testified that: Q: Based on your 1995 purchases, how much were the purchases made in 1995? A: The purchases made by United Merchants Corporation for the last year 1995 is P827,670.[00] sir Q: And how about in 1994? A: In 1994, its P608,986.00 sir. Q: These purchases were made for the entire year of 1995 and 1994 respectively, am I correct? A: Yes sir, for the year 1994 and 1995.40 (Emphasis supplied) In its 1996 Financial Report, which UMC admitted as existing, authentic and duly executed during the 4 December 2002 hearing, it had P1,050,862.71 as total assets and P167,058.47 as total liabilities.41 Thus, either amount in UMCs Income Statement or Financial Reports is twenty-five times the claim UMC seeks to enforce. The RTC itself recognized that UMC padded its claim when it only allowed P43,930,230.00 as insurance claim. UMC supported its claim of P50,000,000.00 with the Certification from the Bureau of Fire Protection stating that "x x x a fire broke out at United Merchants Corporation located at 19-B Dag[o]t Street, Brgy. Manresa, Quezon City incurring an estimated damage of Fifty- Five Million Pesos (P55,000,000.00) to the building and contents x x x." However, this Certification only proved that the estimated damage of P55,000,000.00 is shared by both the building and the stocks in trade.

It has long been settled that a false and material statement made with an intent to deceive or defraud voids an insurance policy.42 In Yu Cua v. South British Insurance Co.,43 the claim was fourteen times bigger than the real loss; in Go Lu v. Yorkshire Insurance Co,44 eight times; and in Tuason v. North China Insurance Co.,45 six times. In the present case, the claim is twenty five times the actual claim proved. The most liberal human judgment cannot attribute such difference to mere innocent error in estimating or counting but to a deliberate intent to demand from insurance companies payment for indemnity of goods not existing at the time of the fire. 46 This constitutes the so-called "fraudulent claim" which, by express agreement between the insurers and the insured, is a ground for the exemption of insurers from civil liability. 47 In its Reply, UMC admitted the discrepancies when it stated that "discrepancies in its statements were not covered by the warranty such that any discrepancy in the declaration in other instruments or documents as to matters that may have some relation to the insurance coverage voids the policy."48 On UMCs allegation that it did not breach any warranty, it may be argued that the discrepancies do not, by themselves, amount to a breach of warranty. However, the Insurance Code provides that "a policy may declare that a violation of specified provisions thereof shall avoid it."49 Thus, in fire insurance policies, which contain provisions such as Condition No. 15 of the Insurance Policy, a fraudulent discrepancy between the actual loss and that claimed in the proof of loss voids the insurance policy. Mere filing of such a claim will exonerate the insurer.50 Considering that all the circumstances point to the inevitable conclusion that UMC padded its claim and was guilty of fraud, UMC violated Condition No. 15 of the Insurance Policy. Thus, UMC forfeited whatever benefits it may be entitled under the Insurance Policy, including its insurance claim. While it is a cardinal principle of insurance law that a contract of insurance is to be construed liberally in favor of the insured and strictly against the insurer company,51 contracts of insurance, like other contracts, are to be construed according to the sense and meaning of the terms which the parties themselves have used. 52 If such terms are clear and unambiguous, they must be taken and understood in their plain, ordinary and popular sense. Courts are not permitted to make contracts for the parties; the function and duty of the courts is simply to enforce and carry out the contracts actually made.53 WHEREFORE, we DENY the petition. We AFFIRM the 16 June 2011 Decision and the 8 September 2011 Resolution of the Court of Appeals in CA-G.R. CV No. 85777. SO ORDERED.

ANTONIO T. CARPIO Senior Associate Justice WE CONCUR: ARTURO D. BRION Associate Justice JOSE PORTUGAL PEREZ Associate Justice MARIA LOURDES P. A. SERENO Associate Justice

BIENVENIDO L. REYES Associate Justice CERTIFICATION I certify that the conclusions in the above Decision had been reached in consultation before the case was assigned to the writer of the opinion of the Courts Division. ANTONIO T. CARPIO Senior Associate Justice (Per Section 12, R.A. 296, The Judiciary Act of 1948, as amended)

G.R. No. L-33131

December 13, 1930

EMILIO GONZALES LA O, plaintiff-appellee, vs. THE YEK TONG LIN FIRE AND MARINE INSURANCE CO., LTD., defendantappellant. Araneta and Zaragosa for appellant. Feria and La O for appellee.

VILLAMOR, J.: This is an action to recover of the defendant the Yek Tong Lin Fire & Marine Insurance Co., Ltd., the amount of two insurance policies totaling P100,000 upon leaf tobacco belonging to the plaintiff, which was damaged by the fire that destroyed the building on Soler Street No. 188, where said tobacco was stored, on January 11, 1928.

The defendant filed a general and specific denial of each and every allegation of the complaint, set up three special defenses, and prayed to be absolved from the complaint with costs against the plaintiff. After the case was tried, the court below rendered judgment as follows: In this case and in Nos. 334568, and 33480 of this court, which, by agreement of the interested parties, were jointly tried, the plaintiff demands P290,000 from the defendant assurance companies, alleging that to be the amount of the insurance on his leaf tobacco which was damaged by the fire that destroyed the warehouse at No. 188 Soler Street, Manila, where it was stored, on January 11, 1928, the plaintiff's claim against the herein defendant, the Yek Tong Lin Fire & Marine Insurance Co. being for P100,000, and against the defendants in the three other cases mentioned above, for P190,000. After the plaintiff had presented his evidence, the defendant companies in cases Nos. 33458, 33868, and 33480, offered to compromise with him by paying eighty-five per cent of his claim against them. In view of the fact that said defendants had in their answer raised the question of warranties A and G of the plaintiff's policies, providing that the building used for the effects insured would not be occupied by any other lessee, nor would be used for the deposit of other goods, without the consent of said defendants, and inasmuch as the latter alleged in their answer that the owner of the burnt building had leased the warehouse to several persons for the storage of sundry articles, the plaintiff had to accept the proposed compromise, and in consequence thereof, the three cases aforesaid were dismissed. The present case followed the usual course of procedure because the plaintiffs refused to accept the compromise which, in the same terms as those made by the defendants in the three cases mentioned, was proposed to him by the defendant the Yek Tong Lin Fire & Marine Insurance Company, the plaintiff contending that said defendant did not, nor could, raise the question of warranties A and G heretofore mentioned for the simple reason that it was the defendant itself, as owner, who had leased the building which later was destroyed by fire, to another person after having already ceded a portion of it to said plaintiff. The only question to be determined, having been raised in the defendant's answer both parties agreeing that the plaintiff insured his leaf tobacco with the defendant assurance company, and that said goods were damaged by the fire which destroyed the warehouse where they were stored, on January 11, 1928 is whether said

goods were worth what the plaintiff claims, that is, about equal to the amount for which they were insured in the four above mentioned assurance companies, including the defendant in this case. The plaintiff has conclusively shown by the Official Register Book (Exhibit 1) and the Official Guide (Exhibit J), furnished by the Bureau of Internal Revenue, and kept under the supervision thereof in the usual form, in accordance with articles 10, 34 to 38 of the Regulations of the same promulgated under No. 17, by the Secretary of Finance; the Stock Book for recording the quantity of tobacco, Exhibit K, kept by the plaintiff and presented as part of the testimony of witnesses Claveria, Bonete, and Leoncio Jose; the testimony of Estanislao Lopez, Inspector of Internal Revenue, and the latter's report (Exhibit N), submitted to the Collector of Internal Revenue in pursuance of article 33 of the aforementioned Regulations; the tobacco invoices of stock damaged by the fire, Exhibits L and L-1 to L-20; and by the testimony of Clemente Uson who went over the plaintiff's books as auditor and public accountant, and also prepared Exhibits T and U, attached to the record, that the plaintiff had in the warehouse at No. 188 Soler at the time of the fire, not less, but rather more, than 6,200 bales of leaf tobacco worth over P300,000, which is of course more than the sum total of all the insurances taken out with the defendant herein and the defendants in the three aforementioned cases Nos. 33458, 33868, and 33480.lawphi1>net The reason why the entry showing that 258 bales of tobacco had been removed from the warehouse, appearing in the Official Register Book, Exhibit I, was not posted in the Stock Book, Exhibit K, has been satisfactorily explained by the plaintiff's witnesses, who stated that it was due to the fact that there was no time to post it in the Stock Book, because the fire took place and the plaintiff told them not to touch, and to make no further entries in the books. Witness White, the defendant company's adjuster, who carefully examined then plaintiff's books not only immediately after the fire, but also during the hearing of this case, seems not to have found any irregularity therein; at least he said nothing on the point when he took the witness stand. On the contrary, in his report Exhibit UU sent to the defendant herein in his capacity as adjuster, appointed by the latter, and in Exhibits WW and XX, admitted by the Yek Tong Lin Ins. Co., Ltd., he admitted that the leaf tobacco belonging to the plaintiff in the warehouse when the fire took place exceeded, in quantity and value, the amount of the insurance. The defendant did not present evidence to rebut the plaintiff's evidence, but only presented witness Rowlands, whose testimony or opinion as to the probable number of bales of tobacco in the

warehouse at the date of the fire does not deserve serious consideration, not only because of the plaintiff's evidence, but because his opinion or estimate is based solely upon photographs of the place taken after the fire. In view of the foregoing, the court hereby sentences the defendant the Yek Tong Lin Fire and Marine Insurance Company, Ltd., to pay the plaintiff Emilio Gonzales La O, the amount of one hundred thousand pesos (P100,000), for which it had accepted the insurance on the leaf tobacco belonging to said plaintiff, damaged by the fire which destroyed the warehouse at No. 188 Soler Street, where it was stored, on January 11, 1928, and legal interest upon said amount from June 27, 1928, when the complaint was filed in this case, plus the costs. So ordered. Manila, P. I., this 24th day of December, 1929. ANACLETO DIAZ Judge. The defendant duly appealed from this judgment, alleging that the trial court erred in making reference to the settlement arrived at by the plaintiff and other insurance companies, and in declaring that the only question involved in the case is whether or not the tobacco damaged by the fire is worth at least P290,000. There is no merit in these assignments of error. Since the settlement between the plaintiff and the other defendant companies was reached after the plaintiff had presented his evidence, and as those three cases were tried jointly with the instant case, there is no valid reason why the trial court should not refer to it in deciding this case. Furthermore, the court's holding here assigned as error, granting there were other incidental matters to be decided by the court, does not in itself constitute a reversible error. In the third assignment of error, the defendant contends that the plaintiff cannot recover under the policy as he has failed to prove that the Bank of the Philippine Islands, to whom the policy was made payable, no longer has any rights and interests in it. It should be noted that the defendant did not in its answer allege defect of parties plaintiff, and, besides, it does not appear that the plaintiff ceded to the bank all his rights or interests in the insurance, the note attached to the policies merely stating: "There shall be paid to the Bank of the Philippine Islands an indemnity for any loss caused by fire, according to the interest appearing in its favor." And the fact that the plaintiff himself presented in evidence the policies mortgaged to the Bank of the Philippine Islands gives rise to the presumption that the debt thus secured has been paid, in accordance with article 1191 of the Civil Code.

Corpus Juris, volume 26, pages 483 et seq., states: Insured, being the person with whom the contract was made, is primarily the proper person to bring suit thereon. Subject to some exceptions, insured may thus sue, although the policy is taken wholly or in part for the benefit of another person named or unnamed, and although it is expressly made payable to another as his interest may appear or otherwise. Although a policy issued to a mortgagor is taken out for the benefit of the mortgagee and is made payable to him, yet the mortgagor may sue thereon in his own name, especially where the mortgagee's interest is less than the full amount recoverable under the policy, . . . . And in volume 33, page 82, of the same work, we read the following: Insured may be regarded as the real party in interest, although he has assigned as collateral security any judgment he may obtain. It is also contended that the trial court erred in not declaring that in as much as the plaintiff failed to notify the defendant corporation in writing, of other insurance policies obtained by him, he has violated article 3 of the conditions of the policies in question, thereby rendering these policies null and void. Article 3 of the conditions of the policies in question prescribes: ART. 3. Any insurance in force upon all or part of the things insured must be declared in writing by the insured and he should cause the company to insert or mention it in the policy, and without such requisite said policy will be regarded as null and void, and the assured deprived of all rights of indemnity in case of loss. The following clause has been inserted with a typewriter in the policies: "Subject to clauses G and A and other insurances with a special short period attached to this policy." And attached to said policies issued by the defendant there is a sheet of "Other insurances" with the amount and the assurance companies in blank, which, according to the appellee, constitutes a notification that there were other insurances existing at the time. In the case of Benedict vs. Ocean Insurance Co. (31 N.Y., 391-393), the construction of the clause, "privilege for $4,500 additional insurance," was discussed. One of the printed clauses of the policy reads as follows: If said assured, or his assigns, shall hereafter make any other insurance upon the same property, and shall not, with all reasonable diligence, give notice to this corporation, and have the same indorsed on this instrument, or otherwise acknowledged by them, in writing, this policy shall cease and be of no further effect.

The Supreme Court of New York held that the words "Privilege for $4,500 additional insurance" made it unnecessary for the assured to inform the insurer of any other policy up to that amount. In the case cited the same goods insured by the defendant company were reinsured to the amount of $4,500 in accordance with the clause "privilege for $4,500 additional insurance;" but in the instant case it may be said that the tobacco insured in the other companies was different from that insured with the defendant, since the number of bales of tobacco in the warehouse greatly exceeded that insured with the defendant and the other companies put together. And according to the doctrine enunciated in 26 Corpus Juris, 188, "to be insurance of the sort prohibited the prior policy must have been insurance upon the same subject matter, and upon the same interest therein. Furthermore, the appellant cannot invoke the violation of article 3 of the conditions of the insurance policies for the first time on appeal, having failed to do so in its answer; besides, as the appellee correctly contends in his brief, Guillermo Cu Unjieng, who was then president and majority shareholder of the appellant company, the Yek Tong Lin Fire & Marine Insurance Co., knew that there were other insurances, at least from the attempt to raise the insurance premium on the warehouse and the appellee's tobacco deposited therein to 1 per centum, and it was later reduced upon petition of the appellant itself and other assurance companies to 0.75 per centum presented to the association of assurance companies in the year 1927, and notwithstanding this, said appellant did not rescind the insurance policies in question, but demanded and collected from the appellee the increased premium. That the defendant had knowledge of the existence of other policies obtained by the plaintiff from other insurance companies, is specifically shown by the defendant's answer wherein it alleges, by way of special defense, the fact that there exist other policies issued by the companies mentioned therein. If, with the knowledge of existence of other insurances which the defendant deemed violations of the contract, it has preferred to continue the policy, its action amounts to a waiver of the annulment of the contract, in accordance with the following doctrine in 19 Cyc., 791, 792:. FAILURE TO ASSERT FORFEITURE IN GENERAL. While the weight of authority is that a policy conditioned to become void upon a breach of a warranty is void ipso facto upon such a breach without formal proceedings on the part of the insurer, yet it is true that such conditions are inserted for the benefit of the insurer and may be waived, and that the insurer may elect to continue the policy despite the breach. If it does the policy is revived and restored. Its failure to assert a forfeiture therefore is at least evidence tending to show a waiver thereof. Many authorities go further, however, and hold that the failure to assert a forfeiture after knowledge of a ground thereof will amount of itself to waiver. . . . The fifth and sixth assignments of error refer to the quantity of tobacco in the Soler warehouse at the time of the fire, which, according to the appellant, did not exceed 4,930 bales. As may be seen, these assignments of error by the appellant involved

purely questions of fact, and it is for this court to decide whether the findings of the trial court are supported by the evidence. The judgment appealed from sets forth clearly the evidence presented to the court in order to determine the quantity of tobacco in the warehouse at the time of the fire. We have studied the evidence aforesaid, are fully convinced that the court's findings are well supported by the same. Inasmuch as it has not, in our opinion, been shown that the trial judge overlooked any fact, which, if duly considered would have change the result of the case, we do not feel justified in altering of modifying his findings. Finally, the appellant contends that the trial court erred in arriving at the damages that plaintiff may recover under the policies in question by the cost price of the tobacco damaged by the fire, instead of computing the same on the market price of the said tobacco at the time of the fire; and in declaring that the tobacco damaged was worth more than P300,000. This error is not well taken, for it is clear that the cost price is competent evidence tending to show the value of the article in question. And it was so held the case of Glaser vs. Home Ins. Co. (47 Misc. Rep., 89; 93 N. Y. Supp., 524; Abbott's Proof of Facts, 3d ed., p. 847), where it was declared that the cost of the goods destroyed by fire is some evidence of value, in an action against the insurance company. Exhibits L to L-20, which are invoices for tobacco purchased by the appellee, and the testimony of the public accountant Clemente Uson, who went over them and the rest of the appellee's books after the fire, taken in connection with reports T and Z, adduced as part of his testimony, show that the cost price of each bale of tobacco belonging to the appellee, damaged by the fire, was P51.8544, which, multiplied by 6,264, the number of bales, yields a total of over P320,000. The adjusters of the appellant, White & Page, in ascertaining the market price of the plaintiff's tobacco deposited in the burnt warehouse, taking the information furnished by the Tabacalera and by M. Pujalte, S. en C., as a basis, thus conclude their report: "We therefore are obliged to the conclusion that the value of the tobacco destroyed was not less than P290,000." And, indeed, said adjusters, in behalf of the appellant, appraised the appellee's tobacco assured and damaged by the fire at P303,052.32, collecting from the proceeds of the sale of the tobacco saved from the fire P3,000, the appellants share in proportion to the to the insurance of P100,000 belonging to it, and P190,000 belonging to the other assurance companies, and considered the appellee himself as his own assurer in the amount of P13,052.32 which was the difference between the total value of the tobacco damaged and the total amount of the insurance, P290,000, for which reason the appellee received P129.21, as his proportionate share of the tobacco saved, as shown by Exhibits UU, WW, and XX. Hence the last assignment of error is without merit. Wherefore, the judgment appealed from is in accordance with law, and must be, as it is hereby, affirmed, with costs against the appellant. So ordered. Johnson, Street, Malcolm, Ostrand, Johns, Romualdez and Villa-Real, JJ., concur.

G.R. No. 20341

September 1, 1923

DOMINGO GARCIA and THE PHILIPPINE NATIONAL BANK, plaintiffs-appellees, vs. THE HONGKONG FIRE & MARINE INSURANCE CO., LTD., defendant-appellant. William and Ferrier for appellant. Roman Lacson for the appellee Bank. Vicente de Vera for the other appellee. STATEMENT After formal pleas, the plaintiff's allege that on the 19th of March, 1918, in the City of Manila, the plaintiff, Domingo Garcia, then a merchant and owner of a bazaar known as "Las Novedades" in the district of Legaspi, municipality and Province of Albay, entered into a contract with the defendant whereby it insured his merchandise in the sum of P15,000 at a premium of P300 per annum; that in consideration of such premium, the defendant issued its fire insurance policy No. 1951 in favor of the plaintiff, not on the merchandise in the building, but on the building which contained the merchandise; that for such reason the policy does not contain the true agreement and intent of the parties; that the plaintiff was not the owner of, and did not have any interest in, the building; and that the policy was so issued through error, carelessness and negligence of the defendant.

That on august 30, 1919, Garcia executed a mortgage to the plaintiff Bank on the merchandise insured by the defendant, and that with the consent of the defendant, the plaintiff endorsed the policy to the Bank; that on February 6, 1920, and while the policy was in force and effect, a fire took place which destroyed the merchandise in the building of the value of P20,000, together with the building itself; that demand was made upon the defendant for the payment of P15,000, as provided for in the policy, and that payment was refused. Wherefore, plaintiffs pray judgment for that amount, with legal interest from the date of filing of the complaint, and costs. For answer, the defendant admits the formal allegations of the complaint, and denies generally and specifically all other allegations. As a result of the trial, the lower court rendered judgment for the plaintiff, as prayed for in the complaint, from which the defendant appeals and contends that the lower court erred in denying its motion to make the complaint more definite and certain; in permitting Garcia over its objection to testify to the contents of certain documents; in refusing to strike them from the record; in finding that the defendant, through its agent, knew that it was the merchandise which was insured and not the building; in failing to find the plaintiffs, and Garcia in particular, guilty of negligence; in finding that the defendant committed error in making out the policy to cover the building rather than the merchandise; in rendering the judgment; and in denying defendant's motion for a new trial.

JOHNS, J.: It appears that the policy was in the English language, of which the plaintiff Garcia is ignorant. When he received it he noticed that the amount P15,000 was correct, and never personally made a further investigation. He was the exclusive owner of the merchandise in the building which, at the time of the fire, was of the probable value of P20,000. He did not own or claim any interest in the building. Desiring to have his merchandise insured for P15,000, he wrote a letter to "El Pilar," requesting that firm to have it insured, as a result of which, the policy in questions was issued and delivered to him, and it was issued on the building with Garcia did not own, and did not cover the merchandise which he did own. Desiring to obtain a loan from the Philippine National Bank, Garcia later delivered and assigned the policy to the plaintiff Bank as collateral security for a loan. Upon receipt of the policy, and as one of the conditions for the making of the loan, the Bank, through its manager, addressed the following letter to the agents of the defendant on August 6, 1919: We beg to advise that the merchandise insured by you against fire in favor of Mr. Domingo Garcia of Legaspi, Albay, P. I., for P15,000 for which you issued policy No. 1951, has been mortgaged to this bank together with the policy to secure a credit and loans not to exceed P6,000 in all.

We would appreciate very much if you have our claims against the property and policy covering it, on account of the mortgage, entered in your records and advise us accordingly. Hoping to hear from you soon, we are, Very truly yours, This was answered by the agents August 14, 1919, as follows: We beg to acknowledge receipt of your esteemed favor of the 6th inst., informing us that the Hongkong Fire Insurance Company, Ltd.'s Policy in the name of Mr. Domingo Garcia, for the sum of P15,000 has been mortgaged to your goodselves. In order that this transaction made by officially recorded, it will be necessary to make an endorsement upon the original policy, and we shall be glad, therefore, if you will return this document to us as soon as convenient. We are, Dear Sirs, Yours faithfully. August 18, 1919, the Bank wrote the following letter to the agents: Complying with your request of the 13th ultimo, we beg to inclose herewith policy No. 1951 in favor of Mr. Domingo Garcia, Legaspi, Albay, for P15,000, which has been mortgaged to this Bank to secure a credit and loan of not exceed P6,000 in all, for your proper indorsement. Trusting to have your prompt action in this matter, we are, Very respectfully yours. September 1, 1919, the agents wrote the Bank as follows: We beg to acknowledge receipt of your favour of the 18th ultimo, enclosing Hongkong Fire Insurance Fire Insurance Co., Ltd.'s Policy No. 1951, in the name of Mr. Domingo Garcia, and in accordance with your request have endorsed same in your favour, and beg to return the document herewith. Please be good enough to acknowledge safe receipt in due course and oblige. Yours faithfully. It clearly appears that where the word "merchandise" was written in the letter of August 6th above quoted, some other word had been previously written and erased, and the word "merchandise" was the written, as it now appears.

It is contended that when the letter was written, the Bank, which then had the possession of the policy, knew that it covered the building and did not insure the merchandise. That, having such knowledge, it was the duty of the Bank to notify the defendant, and having failed to do so, it cannot now contend that the policy was issued through a mistake. The fact remains that the defendant, through its agents, received this letter, and that it recites: We beg to advise that the merchandise insured by you against fire in favor of Mr. Domingo Garcia, etc. That was a personal notice to the defendant of the fact that the policy was on the merchandise. It is pointed out that the Bank and not the defendant then had the policy, and, for such reason, the Bank did not have notice of the error. Although the policy was in possession of the Bank, the defendant had among its own records all of the data and information upon which the policy was issued, and, as a matter of fact, its agents knew or should have known the kind of property insured. It is possible that when the Bank wrote the letter, it knew of the error in the issuance of the policy. But that is a matter of inference or conjecture only. Outside of the appearance of the letter itself, there is no evidence that the Bank had any acknowledge of the error. Garcia had his dealings with the officials of the branch Bank at Legaspi where he was doing business as a merchant, of which the officials of that Bank had knowledge. Under such facts, the presumption of knowledge, if any, on the part of the Bank would be that the policy was on the merchandise. Be that as it may, when the defendant received the letter from the Bank, it knew from its own records that the policy was issued on the building, and, as a matter of fair dealing, it should have notified the Bank that the policy was on the building. It will be noted that the letters in question were all written several months before the fire. In the final analysis, Garcia wanted insurance upon a stock of goods, which he owned, and he received and paid for a policy on a building, which he did not own, and while the policy was in force and effect, both the building, which he did not own, and the stock of merchandise, which he did own, were completely destroyed by fire. Garcia was a well known merchant, and his merchandise was in the building described in the policy. For some unknown reason, the party who applied for the insurance at the instance and request of Garcia was not called as a witness, and, as stated, that answer of the defendant is confined to general denial, and it did not offer any evidence. In a well-written opinion, the trial court analyzed the evidence and made findings of fact upon which it rendered judgment for the plaintiff. It is claimed that the letters and the copy of the telegram introduced in evidence were hearsay and not competent. If for no other purpose, they were competent to show that Garcia wanted insurance on his merchandise and the reason why he wanted it.

The defense is purely technical, and is founded upon the contention that plaintiff cannot recover, because the policy covers loss on a building, and does not cover loss of merchandise. It is very apparent that a mistake was made in the issuance of the policy. In its opinion the trial court says: Under these circumstances it seems clear and manifest that the insured, as well as the manager of the National Bank at Legaspi, who was interested in the policy, because the same secured a loan of P6,000 made to Domingo Garcia, and the corporation of Wise & Co., Ltd., which represented the insurance company, have been in the belief that it was not the building but the merchandise that was insured, for the reason that none of them paid attention to the context of the policy. The opinion of the trial court further points out that, under the pleadings and proof, there is ground for the contention that the plaintiff would be entitled to recover on the policy for the loss of the building. All things considered, the judgment of the lower court is affirmed, with costs. So ordered. Araullo, C.J., Johnson, Malcolm, Avancea, Villamor and Romualdez, JJ., concur. Street, J., dissents.

G.R. No. L-60887 November 13, 1991 PERLA COMPANIA DE SEGUROS, INC., petitioner, vs.

HON. JOSE R. RAMOLETE, PRIMITIVA Y. PALMES, HONORATO BORBON, SR., OFFICE OF THE PROVINCIAL SHERIFF, PROVINCE OF CEBU, respondents. Hector L. Fernandez for petitioner. Domingo Quibranza and Vicente A. Quibranza for private respondents.

FELICIANO, J.:p The present Petition for Certiorari seeks to annul: (a) the Order dated 6 August 1979 1 which ordered the Provincial Sheriff to garnish the third-party liability insurance policy issued by petitioner Perla Compania de Seguros, Inc. ("Perla") in favor of Nelia Enriquez, judgment debtor in Civil Case No. R-15391; (b) the Order dated 24 October 1979 2 which denied the motion for reconsideration of the 6 August 1979 Order; and (c) the Order dated 8 April 1980 3 which ordered the issuance of an alias writ of garnishment against petitioner. In the afternoon of 1 June 1976, a Cimarron PUJ owned and registered in the name of Nelia Enriquez, and driven by Cosme Casas, was travelling from Cebu City to Danao City. While passing through Liloan, Cebu, the Cimarron PUJ collided with a private jeep owned by the late Calixto Palmes (husband of private respondent Primitiva Palmes) who was then driving the private jeep. The impact of the collision was such that the private jeep was flung away to a distance of about thirty (30) feet and then fell on its right side pinning down Calixto Palmes. He died as a result of cardio-respiratory arrest due to a crushed chest. 4 The accident also caused physical injuries on the part of Adeudatus Borbon who was then only two (2) years old. On 25 June 1976, private respondents Primitiva Palmes (widow of Calixto Palmes) and Honorato Borbon, Sr. (father of minor Adeudatus Borbon) filed a complaint 5 against Cosme Casas and Nelia Enriquez (assisted by her husband Leonardo Enriquez) before the then Court of First Instance of Cebu, Branch 3, claiming actual, moral, nominal and exemplary damages as a result of the accident. The claim of private respondent Honorato Borbon, Sr., being distinct and separate from that of co-plaintiff Primitiva Palmes, and the amount thereof falling properly within the jurisdiction of the inferior court, respondent Judge Jose R. Ramolete ordered the Borbon claim excluded from the complaint, without prejudice to its being filed with the proper inferior court. On 4 April 1977, the Court of First Instance rendered a Decision 6 in favor of private respondent Primitiva Palmes, ordering common carrier Nelia Enriquez to pay her P10,000.00 as moral damages, P12,000.00 as compensatory damages for the death of Calixto Palmes, P3,000.00 as exemplary damages, P5,000.00 as actual damages, and P1,000.00 as attorney's fees.

The judgment of the trial court became final and executory and a writ of execution was thereafter issued. The writ of execution was, however, returned unsatisfied. Consequently, the judgment debtor Nelia Enriquez was summoned before the trial court for examination on 23 July 1979. She declared under oath that the Cimarron PUJ registered in her name was covered by a third-party liability insurance policy issued by petitioner Perla. Thus, on 31 July 1979, private respondent Palmes filed a motion for garnishment 7 praying that an order of garnishment be issued against the insurance policy issued by petitioner in favor of the judgment debtor. On 6 August 1979, respondent Judge issued an Order 8 directing the Provincial Sheriff or his deputy to garnish the third-party liability insurance policy. Petitioner then appeared before the trial court and moved for reconsideration of the 6 August 1979 Order and for quashal of the writ of garnishment, 9 alleging that the writ was void on the ground that it (Perla) was not a party to the case and that jurisdiction over its person had never been acquired by the trial court by service of summons or by any process. The trial court denied petitioner's motion.10 An Order for issuance of an alias writ of garnishment was subsequently issued on 8 April 1980. 11 More than two (2) years later, the present Petition for Certiorari and Prohibition was filed with this Court on 25 June 1982 alleging grave abuse of discretion on the part of respondent Judge Ramolete in ordering garnishment of the third-party liability insurance contract issued by petitioner Perla in favor of the judgment debtor, Nelia Enriquez. The Petition should have been dismissed forthwith for having been filed way out of time but, for reasons which do not appear on the record, was nonetheless entertained. In this Petition, petitioner Perla reiterates its contention that its insurance contract cannot be subjected to garnishment or execution to satisfy the judgment in Civil Case No. R-15391 because petitioner was not a party to the case and the trial court did not acquire jurisdiction over petitioner's person. Perla further argues that the writ of garnishment had been issued solely on the basis of the testimony of the judgment debtor during the examination on 23 July 1979 to the effect that the Cimarron PUJ was covered by a third-party liability insurance issued by Perla, without granting it the opportunity to set up any defenses which it may have under the insurance contract; and that the proceedings taken against petitioner are contrary to the procedure laid down in Economic Insurance Company, Inc. v. Torres, et al., 12 which held that under Rule 39, Section 45, the Court "may only authorize" the judgment creditor to institute an action against a third person who holds property belonging to the judgment debtor. We find no grave abuse of discretion or act in excess of or without jurisdiction on the part of respondent Judge Ramolete in ordering the garnishment of the judgment debtor's third-party liability insurance. Garnishment has been defined as a species of attachment for reaching any property or credits pertaining or payable to a judgment debtor. 13 In legal contemplation, it is a

forced novation by the substitution of creditors: 14the judgment debtor, who is the original creditor of the garnishee is, through service of the writ of garnishment, substituted by the judgment creditor who thereby becomes creditor of the garnishee. Garnishment has also been described as a warning to a person having in his possession property or credits of the judgment debtor, not to pay the money or deliver the property to the latter, but rather to appear and answer the plaintiff's suit. 15 In order that the trial court may validly acquire jurisdiction to bind the person of the garnishee, it is not necessary that summons be served upon him. The garnishee need not be impleaded as a party to the case. All that is necessary for the trial court lawfully to bind the person of the garnishee or any person who has in his possession credits belonging to the judgment debtor is service upon him of the writ of garnishment. The Rules of Court themselves do not require that the garnishee be served with summons or impleaded in the case in order to make him liable. Rule 39, Section 15 provides: Sec. 15. Execution of money judgments. The officer must enforce an execution of a money judgment by levying on all the property, real or personal of every name and nature whatsoever, and which may be disposed of for value, of the judgment debtor not exempt from execution . . . Real property, stocks, shares, debts, credits, and other personal property, or any interest in either real or personal property, may be levied on in like manner and with like effect as under a writ of attachment. (Emphasis supplied). Rule 57, Section 7(e) in turn reads: Sec. 7. Attachment of real and personal property; recording thereof. Properties shall be attached by the officer executing the order in the following manner: xxx xxx xxx (e) Debts and credits, and other personal property not capable of manual delivery, by leaving with the person owing such debts, or having his possession or under his control such credits or other personal property, or with his agent, a copy of the order, and notice that the debts owing by him to the party against whom attachment is issued, and the credits and other personal property in his possession, or under his control, belonging to said party, are attached in pursuance of such order; xxx xxx xxx

(Emphasis supplied) Through service of the writ of garnishment, the garnishee becomes a "virtual party" to, or a "forced intervenor" in, the case and the trial court thereby acquires jurisdiction to bind him to compliance with all orders and processes of the trial court with a view to the complete satisfaction of the judgment of the court. In Bautista v. Barredo, 16 the Court, through Mr. Justice Bautista Angelo, held: While it is true that defendant Jose M. Barredo was not a party in Civil Case No. 1636 when it was instituted by appellant against the Philippine Ready Mix Concrete Company, Inc., however, jurisdiction was acquired over him by the court and he became a virtual party to the case when, after final judgment was rendered in said case against the company, the sheriff served upon him a writ of garnishment in behalf of appellant. Thus, as held by this Court in the case of Tayabas Land Company vs. Sharruf, 41 Phil. 382, the proceeding by garnishment is a species of attachment for reaching credits belonging to the judgment debtor and owing to him from a stranger to the litigation. By means of the citation, the stranger becomes a forced intervenor; and the court, having acquired jurisdiction over him by means of the citation, requires him to pay his debt, not to his former creditor, but to the new creditor, who is creditor in the main litigation. (Emphasis supplied). In Rizal Commercial Banking Corporation v. De Castro, 17 the Court stressed that the asset or credit garnished is thereupon subjected to a specific lien: The garnishment of property to satisfy a writ of execution operates as an attachment and fastens upon the property a lien by which the property is brought under the jurisdiction of the court issuing the writ. It is brought into custodia legis, under the sole control of such court. 18 (Emphasis supplied) In the present case, there can be no doubt, therefore, that the trial court actually acquired jurisdiction over petitioner Perla when it was served with the writ of garnishment of the third-party liability insurance policy it had issued in favor of judgment debtor Nelia Enriquez. Perla cannot successfully evade liability thereon by such a contention. Every interest which the judgment debtor may have in property may be subjected to execution.19 In the instant case, the judgment debtor Nelia Enriquez clearly had an interest in the proceeds of the third-party liability insurance contract. In a third-party liability insurance contract, the insurer assumes the obligation of paying the injured third party to whom the insured is liable. 20 The insurer becomes liable as soon as the liability of the insured to the injured third person attaches. Prior payment by the insured to the injured third person is not necessary in order that the obligation of the insurer may arise. From the moment that the insured became liable to the third person, the insured

acquired an interest in the insurance contract, which interest may be garnished like any other credit. 21 Petitioner also contends that in order that it may be held liable under the third-party liability insurance, a separate action should have been commenced by private respondents to establish petitioner's liability. Petitioner invokesEconomic Insurance Company, Inc. vs. Torres, 22 which stated: It is clear from Section 45, Rule 39 that if a persons alleged to have property of the judgment debtor or to be indebted to him claims an interest in the property adverse to him or denies the debt, the court may only authorize the judgment creditor to institute an action against such person for the recovery of such interest or debt. Said section does not authorize the court to make a finding that the third person has in his possession property belonging to the judgment debtor or is indebted to him and to order said third person to pay the amount to the judgment creditor. It has been held that the only power of the court in proceedings supplemental to execution is to niake an order authorizing the creditor to sue in the proper court to recover an indebtedness due to the judgment debtor. The court has no jurisdiction to try summarily the question whether the third party served with notice of execution and levy is indebted to defendant when such indebtedness is denied. To make an order in relation to property which the garnishee claimed to own in his own right, requiring its application in satisfaction of judgment of another, would be to deprive the garnishee of property upon summary proceeding and without due process of law. (Emphasis supplied) But reliance by petitioner on the case of Economic Insurance Company, Inc. v. Torres (supra) is misplaced. The Court there held that a separate action needs to be commenced when the garnishee "claims an interest in the property adverse to him (judgment debtor) or denies the debt." In the instant case, petitioner Perla did not deny before the trial court that it had indeed issued a third-party liability insurance policy in favor of the judgment debtor. Petitioner moreover refrained from setting up any substantive defense which it might have against the insured-judgment debtor. The only ground asserted by petitioner in its "Motion for Reconsideration of the Order dated August 6, 1979 and to Quash Notice of Garnishment" was lack of jurisdiction of the trial court for failure to implead it in the case by serving it with summons. Accordingly, Rule 39, Section 45 of the Rules of Court is not applicable in the instant case, and we see no need to require a separate action against Perla: a writ of garnishment suffices to hold petitioner answerable to the judgment creditor. If Perla had any substantive defenses against the judgment debtor, it is properly deemed to have waived them by laches. WHEREFORE, the Petition for Certiorari and Prohibition is hereby DISMISSED for having been filed out of time and for lack of merit. The assailed Orders of the trial court

are hereby AFFIRMED. Costs against petitioner. This Decision is immediately executory. SO ORDERED. Narvasa, CJ., Cruz, Grio-Aquino and Medialdea, JJ., concur.

G.R. No. 115278 May 23, 1995 FORTUNE INSURANCE AND SURETY CO., INC., petitioner, vs. COURT OF APPEALS and PRODUCERS BANK OF THE PHILIPPINES, respondents.

DAVIDE, JR., J.: The fundamental legal issue raised in this petition for review on certiorari is whether the petitioner is liable under the Money, Security, and Payroll Robbery policy it issued to the private respondent or whether recovery thereunder is precluded under the general exceptions clause thereof. Both the trial court and the Court of Appeals held that there should be recovery. The petitioner contends otherwise.

This case began with the filing with the Regional Trial Court (RTC) of Makati, Metro Manila, by private respondent Producers Bank of the Philippines (hereinafter Producers) against petitioner Fortune Insurance and Surety Co., Inc. (hereinafter Fortune) of a complaint for recovery of the sum of P725,000.00 under the policy issued by Fortune. The sum was allegedly lost during a robbery of Producer's armored vehicle while it was in transit to transfer the money from its Pasay City Branch to its head office in Makati. The case was docketed as Civil Case No. 1817 and assigned to Branch 146 thereof. After joinder of issues, the parties asked the trial court to render judgment based on the following stipulation of facts: 1. The plaintiff was insured by the defendants and an insurance policy was issued, the duplicate original of which is hereto attached as Exhibit "A"; 2. An armored car of the plaintiff, while in the process of transferring cash in the sum of P725,000.00 under the custody of its teller, Maribeth Alampay, from its Pasay Branch to its Head Office at 8737 Paseo de Roxas, Makati, Metro Manila on June 29, 1987, was robbed of the said cash. The robbery took place while the armored car was traveling along Taft Avenue in Pasay City; 3. The said armored car was driven by Benjamin Magalong Y de Vera, escorted by Security Guard Saturnino Atiga Y Rosete. Driver Magalong was assigned by PRC Management Systems with the plaintiff by virtue of an Agreement executed on August 7, 1983, a duplicate original copy of which is hereto attached as Exhibit "B"; 4. The Security Guard Atiga was assigned by Unicorn Security Services, Inc. with the plaintiff by virtue of a contract of Security Service executed on October 25, 1982, a duplicate original copy of which is hereto attached as Exhibit "C"; 5. After an investigation conducted by the Pasay police authorities, the driver Magalong and guard Atiga were charged, together with Edelmer Bantigue Y Eulalio, Reynaldo Aquino and John Doe, with violation of P.D. 532 (Anti-Highway Robbery Law) before the Fiscal of Pasay City. A copy of the complaint is hereto attached as Exhibit "D"; 6. The Fiscal of Pasay City then filed an information charging the aforesaid persons with the said crime before Branch 112 of the Regional Trial Court of Pasay City. A copy of the said

information is hereto attached as Exhibit "E." The case is still being tried as of this date; 7. Demands were made by the plaintiff upon the defendant to pay the amount of the loss of P725,000.00, but the latter refused to pay as the loss is excluded from the coverage of the insurance policy, attached hereto as Exhibit "A," specifically under page 1 thereof, "General Exceptions" Section (b), which is marked as Exhibit "A-1," and which reads as follows: GENERAL EXCEPTIONS The company shall not be liable under this policy in report of xxx xxx xxx (b) any loss caused by any dishonest, fraudulent or criminal act of the insured or any officer, employee, partner, director, trustee or authorized representative of the Insured whether acting alone or in conjunction with others. . . . 8. The plaintiff opposes the contention of the defendant and contends that Atiga and Magalong are not its "officer, employee, . . . trustee or authorized representative . . . at the time of the robbery. 1 On 26 April 1990, the trial court rendered its decision in favor of Producers. The dispositive portion thereof reads as follows: WHEREFORE, premises considered, the Court finds for plaintiff and against defendant, and (a) orders defendant to pay plaintiff the net amount of P540,000.00 as liability under Policy No. 0207 (as mitigated by the P40,000.00 special clause deduction and by the recovered sum of P145,000.00), with interest thereon at the legal rate, until fully paid; (b) orders defendant to pay plaintiff the sum of P30,000.00 as and for attorney's fees; and (c) orders defendant to pay costs of suit.

All other claims and counterclaims are accordingly dismissed forthwith. SO ORDERED. 2 The trial court ruled that Magalong and Atiga were not employees or representatives of Producers. It Said: The Court is satisfied that plaintiff may not be said to have selected and engaged Magalong and Atiga, their services as armored car driver and as security guard having been merely offered by PRC Management and by Unicorn Security and which latter firms assigned them to plaintiff. The wages and salaries of both Magalong and Atiga are presumably paid by their respective firms, which alone wields the power to dismiss them. Magalong and Atiga are assigned to plaintiff in fulfillment of agreements to provide driving services and property protection as such in a context which does not impress the Court as translating into plaintiff's power to control the conduct of any assigned driver or security guard, beyond perhaps entitling plaintiff to request are replacement for such driver guard. The finding is accordingly compelled that neither Magalong nor Atiga were plaintiff's "employees" in avoidance of defendant's liability under the policy, particularly the general exceptions therein embodied. Neither is the Court prepared to accept the proposition that driver Magalong and guard Atiga were the "authorized representatives" of plaintiff. They were merely an assigned armored car driver and security guard, respectively, for the June 29, 1987 money transfer from plaintiff's Pasay Branch to its Makati Head Office. Quite plainly it was teller Maribeth Alampay who had "custody" of the P725,000.00 cash being transferred along a specified money route, and hence plaintiff's then designated "messenger" adverted to in the policy. 3 Fortune appealed this decision to the Court of Appeals which docketed the case as CAG.R. CV No. 32946. In its decision 4 promulgated on 3 May 1994, it affirmed in toto the appealed decision. The Court of Appeals agreed with the conclusion of the trial court that Magalong and Atiga were neither employees nor authorized representatives of Producers and ratiocinated as follows: A policy or contract of insurance is to be construed liberally in favor of the insured and strictly against the insurance company (New Life Enterprises vs. Court of Appeals, 207 SCRA 669; Sun Insurance Office, Ltd. vs. Court of Appeals, 211 SCRA 554). Contracts of insurance, like other contracts, are to be construed according to the sense and meaning of the terms which the parties themselves have used. If such terms are clear and unambiguous, they must be taken and understood in their plain, ordinary

and popular sense (New Life Enterprises Case, supra, p. 676; Sun Insurance Office, Ltd. vs. Court of Appeals, 195 SCRA 193). The language used by defendant-appellant in the above quoted stipulation is plain, ordinary and simple. No other interpretation is necessary. The word "employee" must be taken to mean in the ordinary sense. The Labor Code is a special law specifically dealing with/and specifically designed to protect labor and therefore its definition as to employeremployee relationships insofar as the application/enforcement of said Code is concerned must necessarily be inapplicable to an insurance contract which defendant-appellant itself had formulated. Had it intended to apply the Labor Code in defining what the word "employee" refers to, it must/should have so stated expressly in the insurance policy. Said driver and security guard cannot be considered as employees of plaintiff-appellee bank because it has no power to hire or to dismiss said driver and security guard under the contracts (Exhs. 8 and C) except only to ask for their replacements from the contractors. 5 On 20 June 1994, Fortune filed this petition for review on certiorari. It alleges that the trial court and the Court of Appeals erred in holding it liable under the insurance policy because the loss falls within the general exceptions clause considering that driver Magalong and security guard Atiga were Producers' authorized representatives or employees in the transfer of the money and payroll from its branch office in Pasay City to its head office in Makati. According to Fortune, when Producers commissioned a guard and a driver to transfer its funds from one branch to another, they effectively and necessarily became its authorized representatives in the care and custody of the money. Assuming that they could not be considered authorized representatives, they were, nevertheless, employees of Producers. It asserts that the existence of an employer-employee relationship "is determined by law and being such, it cannot be the subject of agreement." Thus, if there was in reality an employer-employee relationship between Producers, on the one hand, and Magalong and Atiga, on the other, the provisions in the contracts of Producers with PRC Management System for Magalong and with Unicorn Security Services for Atiga which state that Producers is not their employer and that it is absolved from any liability as an employer, would not obliterate the relationship. Fortune points out that an employer-employee relationship depends upon four standards: (1) the manner of selection and engagement of the putative employee; (2) the mode of payment of wages; (3) the presence or absence of a power to dismiss; and (4) the presence and absence of a power to control the putative employee's conduct. Of the four, the right-of-control test has been held to be the decisive factor. 6 It asserts that the power of control over Magalong and Atiga was vested in and exercised by Producers. Fortune further insists that PRC Management System and Unicorn Security

Services are but "labor-only" contractors under Article 106 of the Labor Code which provides: Art. 106. Contractor or subcontractor. There is "labor-only" contracting where the person supplying workers to an employer does not have substantial capital or investment in the form of tools, equipment, machineries, work premises, among others, and the workers recruited and placed by such persons are performing activities which are directly related to the principal business of such employer. In such cases, the person or intermediary shall be considered merely as an agent of the employer who shall be responsible to the workers in the same manner and extent as if the latter were directly employed by him. Fortune thus contends that Magalong and Atiga were employees of Producers, following the ruling in International Timber Corp. vs. NLRC 7 that a finding that a contractor is a "labor-only" contractor is equivalent to a finding that there is an employer-employee relationship between the owner of the project and the employees of the "labor-only" contractor. On the other hand, Producers contends that Magalong and Atiga were not its employees since it had nothing to do with their selection and engagement, the payment of their wages, their dismissal, and the control of their conduct. Producers argued that the rule in International Timber Corp. is not applicable to all cases but only when it becomes necessary to prevent any violation or circumvention of the Labor Code, a social legislation whose provisions may set aside contracts entered into by parties in order to give protection to the working man. Producers further asseverates that what should be applied is the rule in American President Lines vs. Clave, 8 to wit: In determining the existence of employer-employee relationship, the following elements are generally considered, namely: (1) the selection and engagement of the employee; (2) the payment of wages; (3) the power of dismissal; and (4) the power to control the employee's conduct. Since under Producers' contract with PRC Management Systems it is the latter which assigned Magalong as the driver of Producers' armored car and was responsible for his faithful discharge of his duties and responsibilities, and since Producers paid the monthly compensation of P1,400.00 per driver to PRC Management Systems and not to Magalong, it is clear that Magalong was not Producers' employee. As to Atiga, Producers relies on the provision of its contract with Unicorn Security Services which provides that the guards of the latter "are in no sense employees of the CLIENT." There is merit in this petition.

It should be noted that the insurance policy entered into by the parties is a theft or robbery insurance policy which is a form of casualty insurance. Section 174 of the Insurance Code provides: Sec. 174. Casualty insurance is insurance covering loss or liability arising from accident or mishap, excluding certain types of loss which by law or custom are considered as falling exclusively within the scope of insurance such as fire or marine. It includes, but is not limited to, employer's liability insurance, public liability insurance, motor vehicle liability insurance, plate glass insurance, burglary and theft insurance, personal accident and health insurance as written by non-life insurance companies, and other substantially similar kinds of insurance. (emphases supplied) Except with respect to compulsory motor vehicle liability insurance, the Insurance Code contains no other provisions applicable to casualty insurance or to robbery insurance in particular. These contracts are, therefore, governed by the general provisions applicable to all types of insurance. Outside of these, the rights and obligations of the parties must be determined by the terms of their contract, taking into consideration its purpose and always in accordance with the general principles of insurance law. 9 It has been aptly observed that in burglary, robbery, and theft insurance, "the opportunity to defraud the insurer the moral hazard is so great that insurers have found it necessary to fill up their policies with countless restrictions, many designed to reduce this hazard. Seldom does the insurer assume the risk of all losses due to the hazards insured against." 10 Persons frequently excluded under such provisions are those in the insured's service and employment. 11 The purpose of the exception is to guard against liability should the theft be committed by one having unrestricted access to the property. 12 In such cases, the terms specifying the excluded classes are to be given their meaning as understood in common speech. 13 The terms "service" and "employment" are generally associated with the idea of selection, control, and compensation. 14 A contract of insurance is a contract of adhesion, thus any ambiguity therein should be resolved against the insurer, 15 or it should be construed liberally in favor of the insured and strictly against the insurer. 16 Limitations of liability should be regarded with extreme jealousy and must be construed in such a way, as to preclude the insurer from non-compliance with its obligation. 17 It goes without saying then that if the terms of the contract are clear and unambiguous, there is no room for construction and such terms cannot be enlarged or diminished by judicial construction. 18 An insurance contract is a contract of indemnity upon the terms and conditions specified therein. 19 It is settled that the terms of the policy constitute the measure of the insurer's liability. 20 In the absence of statutory prohibition to the contrary, insurance companies have the same rights as individuals to limit their liability and to impose whatever conditions they deem best upon their obligations not inconsistent with public policy.

With the foregoing principles in mind, it may now be asked whether Magalong and Atiga qualify as employees or authorized representatives of Producers under paragraph (b) of the general exceptions clause of the policy which, for easy reference, is again quoted: GENERAL EXCEPTIONS The company shall not be liable under this policy in respect of xxx xxx xxx (b) any loss caused by any dishonest, fraudulent or criminal act of the insured or any officer, employee, partner, director, trustee or authorized representative of the Insured whether acting alone or in conjunction with others. . . . (emphases supplied) There is marked disagreement between the parties on the correct meaning of the terms "employee" and "authorized representatives." It is clear to us that insofar as Fortune is concerned, it was its intention to exclude and exempt from protection and coverage losses arising from dishonest, fraudulent, or criminal acts of persons granted or having unrestricted access to Producers' money or payroll. When it used then the term "employee," it must have had in mind any person who qualifies as such as generally and universally understood, or jurisprudentially established in the light of the four standards in the determination of the employeremployee relationship, 21 or as statutorily declared even in a limited sense as in the case of Article 106 of the Labor Code which considers the employees under a "laboronly" contract as employees of the party employing them and not of the party who supplied them to the employer. 22 Fortune claims that Producers' contracts with PRC Management Systems and Unicorn Security Services are "labor-only" contracts. Producers, however, insists that by the express terms thereof, it is not the employer of Magalong. Notwithstanding such express assumption of PRC Management Systems and Unicorn Security Services that the drivers and the security guards each shall supply to Producers are not the latter's employees, it may, in fact, be that it is because the contracts are, indeed, "labor-only" contracts. Whether they are is, in the light of the criteria provided for in Article 106 of the Labor Code, a question of fact. Since the parties opted to submit the case for judgment on the basis of their stipulation of facts which are strictly limited to the insurance policy, the contracts with PRC Management Systems and Unicorn Security Services, the complaint for violation of P.D. No. 532, and the information therefor filed by the City Fiscal of Pasay City, there is a paucity of evidence as to whether the contracts between Producers and PRC Management Systems and Unicorn Security Services are "labor-only" contracts.

But even granting for the sake of argument that these contracts were not "labor-only" contracts, and PRC Management Systems and Unicorn Security Services were truly independent contractors, we are satisfied that Magalong and Atiga were, in respect of the transfer of Producer's money from its Pasay City branch to its head office in Makati, its "authorized representatives" who served as such with its teller Maribeth Alampay. Howsoever viewed, Producers entrusted the three with the specific duty to safely transfer the money to its head office, with Alampay to be responsible for its custody in transit; Magalong to drive the armored vehicle which would carry the money; and Atiga to provide the needed security for the money, the vehicle, and his two other companions. In short, for these particular tasks, the three acted as agents of Producers. A "representative" is defined as one who represents or stands in the place of another; one who represents others or another in a special capacity, as an agent, and is interchangeable with "agent." 23 In view of the foregoing, Fortune is exempt from liability under the general exceptions clause of the insurance policy. WHEREFORE , the instant petition is hereby GRANTED. The decision of the Court of Appeals in CA-G.R. CV No. 32946 dated 3 May 1994 as well as that of Branch 146 of the Regional Trial Court of Makati in Civil Case No. 1817 are REVERSED and SET ASIDE. The complaint in Civil Case No. 1817 is DISMISSED. No pronouncement as to costs. SO ORDERED. Bellosillo and Kapunan, JJ., concur. Padilla, J., took no part. Quiason, J., is on leave. G.R. No. L-20853 May 29, 1967 BONIFACIO BROS., INC., ET AL., plaintiffs-appellants, vs. ENRIQUE MORA, ET AL., defendants-appellees. G. Magsaysay for plaintiffs-appellants. Abad Santos and Pablo for defendant-appellee H. E. Reyes, Inc. J. P. Santilla and A. D. Hidalgo, Jr. for other defendant-appellee. CASTRO, J.: This is an appeal from the decision of the Court of First Instance of Manila, Branch XV, in civil case 48823, affirming the decision of the Municipal Court of Manila, declaring the H.S. Reyes, Inc. as having a better right than the Bonifacio Bros., Inc. and the Ayala Auto Parts Company, appellants herein, to the proceeds of motor insurance policy A-

0615, in the sum of P2,002.73, issued by the State Bonding & Insurance Co. Inc., and directing payment of the said amount to the H. Reyes, Inc. Enrique Mora, owner of Oldsmobile sedan model 1956, bearing plate No. QCmortgaged the same to the H.S. Reyes, Inc., with the condition that the former would insure the automobile with the latter as beneficiary. The automobile was thereafter insured on June 23, 1959 with the State Bonding & Insurance Co., Inc., and motor car insurance policy A-0615 was issued to Enrique Mora, the pertinent provisions of which read: 1. The Company (referring to the State Bonding & Insurance Co., Inc.) will, subject to the Limits of Liability, indemnify the Insured against loss of or damages to the Motor Vehicle and its accessories and spare parts whilst thereon; (a) by accidental collision or overturning or collision or overturning consequent upon mechanical breakdown or consequent upon wear and tear, xxx xxx xxx

2. At its own option the Company may pay in cash the amount of the loss or damage or may repair, reinstate, or replace the Motor Vehicle or any part thereof or its accessories or spare parts. The liability of the Company shall not exceed the value of the parts whichever is the less. The Insured's estimate of value stated in the schedule will be the maximum amount payable by the Company in respect of any claim for loss or damage.1wph1.t xxx xxx xxx

4. The Insured may authorize the repair of the Motor Vehicle necessitated by damage for which the Company may be liable under this Policy provided that: (a) The estimated cost of such repair does not exceed the Authorized Repair Limit, (b) A detailed estimate of the cost is forwarded to the Company without delay, subject to the condition that "Loss, if any is payable to H.S. Reyes, Inc.," by virtue of the fact that said Oldsmobile sedan was mortgaged in favor of the said H.S. Reyes, Inc. and that under a clause in said insurance policy, any loss was made payable to the H.S. Reyes, Inc. as Mortgagee; xxx xxx xxx

During the effectivity of the insurance contract, the car met with an accident. The insurance company then assigned the accident to the Bayne Adjustment Co. for investigation and appraisal of the damage. Enrique Mora, without the knowledge and consent of the H.S. Reyes, Inc., authorized the Bonifacio Bros. Inc. to furnish the labor and materials, some of which were supplied by the Ayala Auto Parts Co. For the cost of labor and materials, Enrique Mora was billed at P2,102.73 through the H.H. Bayne Adjustment Co. The insurance company after claiming a franchise in the amount of P100, drew a check in the amount of P2,002.73, as proceeds of the insurance policy,

payable to the order of Enrique Mora or H.S. Reyes,. Inc., and entrusted the check to the H.H. Bayne Adjustment Co. for disposition and delivery to the proper party. In the meantime, the car was delivered to Enrique Mora without the consent of the H.S. Reyes, Inc., and without payment to the Bonifacio Bros. Inc. and the Ayala Auto Parts Co. of the cost of repairs and materials. Upon the theory that the insurance proceeds should be paid directly to them, the Bonifacio Bros. Inc. and the Ayala Auto Parts Co. filed on May 8, 1961 a complaint with the Municipal Court of Manila against Enrique Mora and the State Bonding & Insurance Co., Inc. for the collection of the sum of P2,002.73 The insurance company filed its answer with a counterclaim for interpleader, requiring the Bonifacio Bros. Inc. and the H.S. Reyes, Inc. to interplead in order to determine who has better right to the insurance proceeds in question. Enrique Mora was declared in default for failure to appear at the hearing, and evidence against him was received ex parte. However, the counsel for the Bonifacio Bros. Inc., Ayala Auto Parts Co. and State Bonding & Insurance Co. Inc. submitted a stipulation of facts, on the basis of which are Municipal Court rendered a decision declaring the H.S. Reyes, Inc. as having a better right to the disputed amount and ordering State Bonding & Insurance Co. Inc. to pay to the H. S. Reyes, Inc. the said sum of P2,002.73. From this decision, the appellants elevated the case to the Court of First Instance of Manila which the stipulation of facts was reproduced. On October 19, 1962 the latter court rendered a decision, affirming the decision of the Municipal Court. The Bonifacio Bros. Inc. and the Ayala Auto Parts Co. moved for reconsideration of the decision, but the trial court denied the motion. Hence, this appeal. The main issue raised is whether there is privity of contract between the Bonifacio Bros. Inc. and the Ayala Auto Parts Co. on the one hand and the insurance company on the other. The appellants argue that the insurance company and Enrique Mora are parties to the repair of the car as well as the towage thereof performed. The authority for this assertion is to be found, it is alleged, in paragraph 4 of the insurance contract which provides that "the insured may authorize the repair of the Motor Vehicle necessitated by damage for which the company may be liable under the policy provided that (a) the estimated cost of such repair does not exceed the Authorized Repair Limit, and (b) a detailed estimate of the cost is forwarded to the company without delay." It is stressed that the H.H. Bayne Adjustment Company's recommendation of payment of the appellants' bill for materials and repairs for which the latter drew a check for P2,002.73 indicates that Mora and the H.H. Bayne Adjustment Co. acted for and in representation of the insurance company. This argument is, in our view, beside the point, because from the undisputed facts and from the pleadings it will be seen that the appellants' alleged cause of action rests exclusively upon the terms of the insurance contract. The appellants seek to recover the insurance proceeds, and for this purpose, they rely upon paragraph 4 of the insurance contract document executed by and between the State Bonding & Insurance Company, Inc. and Enrique Mora. The appellants are not mentioned in the contract as parties thereto nor is there any clause or provision thereof from which we can infer that there is an obligation on the part of the insurance company to pay the cost of repairs directly to

them. It is fundamental that contracts take effect only between the parties thereto, except in some specific instances provided by law where the contract contains some stipulation in favor of a third person.1Such stipulation is known as stipulation pour autrui or a provision in favor of a third person not a pay to the contract. Under this doctrine, a third person is allowed to avail himself of a benefit granted to him by the terms of the contract, provided that the contracting parties have clearly and deliberately conferred a favor upon such person.2 Consequently, a third person not a party to the contract has no action against the parties thereto, and cannot generally demand the enforcement of the same.3 The question of whether a third person has an enforcible interest in a contract, must be settled by determining whether the contracting parties intended to tender him such an interest by deliberately inserting terms in their agreement with the avowed purpose of conferring a favor upon such third person. In this connection, this Court has laid down the rule that the fairest test to determine whether the interest of a third person in a contract is a stipulation pour autrui or merely an incidental interest, is to rely upon the intention of the parties as disclosed by their contract.4 In the instant case the insurance contract does not contain any words or clauses to disclose an intent to give any benefit to any repairmen or materialmen in case of repair of the car in question. The parties to the insurance contract omitted such stipulation, which is a circumstance that supports the said conclusion. On the other hand, the "loss payable" clause of the insurance policy stipulates that "Loss, if any, is payable to H.S. Reyes, Inc." indicating that it was only the H.S. Reyes, Inc. which they intended to benefit. We likewise observe from the brief of the State Bonding & Insurance Company that it has vehemently opposed the assertion or pretension of the appellants that they are privy to the contract. If it were the intention of the insurance company to make itself liable to the repair shop or materialmen, it could have easily inserted in the contract a stipulation to that effect. To hold now that the original parties to the insurance contract intended to confer upon the appellants the benefit claimed by them would require us to ignore the indespensable requisite that a stipulationpour autrui must be clearly expressed by the parties, which we cannot do. As regards paragraph 4 of the insurance contract, a perusal thereof would show that instead of establishing privity between the appellants and the insurance company, such stipulation merely establishes the procedure that the insured has to follow in order to be entitled to indemnity for repair. This paragraph therefore should not be construed as bringing into existence in favor of the appellants a right of action against the insurance company as such intention can never be inferred therefrom. Another cogent reason for not recognizing a right of action by the appellants against the insurance company is that "a policy of insurance is a distinct and independent contract between the insured and insurer, and third persons have no right either in a court of equity, or in a court of law, to the proceeds of it, unless there be some contract of trust, expressed or implied between the insured and third person." 5 In this case, no contract of trust, expressed or implied exists. We, therefore, agree with the trial court that no cause of action exists in favor of the appellants in so far as the proceeds of insurance are

concerned. The appellants' claim, if at all, is merely equitable in nature and must be made effective through Enrique Mora who entered into a contract with the Bonifacio Bros. Inc. This conclusion is deducible not only from the principle governing the operation and effect of insurance contracts in general, but is clearly covered by the express provisions of section 50 of the Insurance Act which read: The insurance shall be applied exclusively to the proper interests of the person in whose name it is made unless otherwise specified in the policy. The policy in question has been so framed that "Loss, if any, is payable to H.S. Reyes, Inc.," which unmistakably shows the intention of the parties. The final contention of the appellants is that the right of the H.S. Reyes, Inc. to the insurance proceeds arises only if there was loss and not where there is mere damage as in the instant case. Suffice it to say that any attempt to draw a distinction between "loss" and "damage" is uncalled for, because the word "loss" in insurance law embraces injury or damage. Loss in insurance, defined. The injury or damage sustained by the insured in consequence of the happening of one or more of the accidents or misfortune against which the insurer, in consideration of the premium, has undertaken to indemnify the insured. (1 Bouv. Ins. No. 1215; Black's Law Dictionary; Cyclopedic Law Dictionary, cited in Martin's Phil. Commercial Laws, Vol. 1, 1961 ed. p. 608). Indeed, according to sec. 120 of the Insurance Act, a loss may be either total or partial. Accordingly, the judgment appealed from is hereby affirmed, at appellants' cost. Concepcion, C.J., Reyes, J.B.L., Dizon, Regala, Makalintal, Bengzon, J.P., Zaldivar, Sanchez and Castro, JJ., concur.

G.R. No. 77397 April 3, 1990 PEOPLE OF THE PHILIPPINES, plaintiff-appellee, vs. RONALDO JOMAO-AS y PADILLA and ALEXANDER SPANDONIS alias "BRUTUS", defendants, ALEXANDER SPANDONIS alias "BRUTUS", defendantappellant. The Office of the Solicitor General for plaintiff-appellee. Dominador G. Suga for defendant-appellant.

PADILLA, J.: The defendant-appellant Alexander Spandonis alias "Brutus" has come to this Court seeking the reversal of the decision rendered by Judge Milagros C. Nartatez in Criminal Case No. 12447 of the Regional Trial Court of Davao City, the dispositive portion of which reads, as follows: WHEREFORE, finding the guilt of ALEXANDER SPANDONIS alias Brutus of Violation of Section 4 of Republic Act No. 6425 as amended by Batas Pambansa Bilang 179, as charged, proven beyond reasonable doubt, he is hereby sentenced to LIFE IMPRISONMENT and to pay a fine of TWENTY THOUSAND (P20,000.00) PESOS, and to pay a proportionate part of the costs. 1 The records of the case show that about 4:15 o'clock in the afternoon of 29 June 1985, PC NARCOM agents arrested one Ronaldo Jomao-as y Padilla while selling five (5) matchboxes of marijuana to a civilian informer posing as a buyer, at the Food Plaza situated at the corner of Claveria and San Pedro Streets in Davao City. He was brought to the PC NARCOM headquarters and when questioned, he pointed to the herein appellant Alexander Spandonis as the source of the marijuana confiscated from

him. 2 As a result, Ronaldo Jomao-as and the appellant Alexander Spandonis were charged with violation of the Dangerous Drugs Act (Rep. Act No. 6425 as amended), committed, as follows: That on or about June 29, 1985, in the City of Davao, Philippines, and within the jurisdiction of this Honorable Court, the above-named accused, conspiring and confederation together and helping one another, did then and wilfully, unlawfully and feloniously without having been authorized by law dealt with the sale of five (5) matchboxes of Marijuana leaves, which is prohibited drug. 3 When arraigned, both accused pleaded "not guilty" to the commission of the crime. The accused Ronaldo Jomao-as, however, subsequently asked for a reinvestigation of the case after which he withdrew his plea of "not guilty." After a re-arraignment, he pleaded "guilty" to a lesser offense and was sentenced accordingly. 4 The accused, Alexander Spandonis, upon the other hand, moved to dismiss the case against him on the ground that no preliminary investigation was conducted against him prior to the filing of the information, 5 but his motion to dismiss was denied for the reason that he did not file said motion to dismiss before he entered his plea so that he was deemed to have waived it. 6 Trial of the case then proceeded against Alexander Spandonis who denied that the marijuana leaves found in the possession of his coaccused Ronaldo Jomao-as by NARCOM agents in a "buy-bust" operation of 29 June 1985 came from him since he was not in the business of selling marijuana, and that on 29 June 1985 he was at Ecoland, painting the house of his sister. He further stated that his co-accused may have pointed to him as the source of the marijuana leaves because of the grudge Ronaldo Jomao-as had against him as a result of an altercation between them a month before, in May 1985, when he refused to give money to Jomao-as with which to buy some drinks. 7 The trial court gave no credence to his defense. In support of his appeal, the appellant Alexander Spandonis, through counsel, claims that the trial court erred: (1) in giving credence to the testimony of his co-accused Ronaldo Jomao-as, and that of Joseph Cagas who had been accused of violation of the Dangerous Drugs Act in another case; and (2) in not dismissing the case for lack of preliminary investigation. While the rile is that great weight and respect should be accorded to the trial court's findings, the severity and harshness of the penalty imposed by law on violators of the Dangerous Drugs Act (Rep. Act No. 6425, as amended) calls for a meticulous evaluation of the evidence whether incriminating or exculpating an accused. 8 The Court has also said that "Courts must be extra vigilant in trying drug charges lest an innocent person is made to suffer unusually severe penalties for drug offenses. 9 We have gone over the records of the case and find that the evidence presented by the prosecution is not sufficient to support a conviction. It would appear that the trial court relied mainly, if not entirely, on the uncorroborated testimony of the co-accused Ronaldo

Jomao-as in finding the appellant Alexander Spandonis guilty of the offense charged. In fact, without the testimony of the said Ronaldo Jomao-as implicating the appellant, the latter would have been acquitted, as there is no other evidence that would link the appellant to the marijuana leaves found in the possession of the said Ronaldo Jomaoas during the "buy-bust" operation conducted by NARCOM agents on 29 June 1985. While there may be no law which requires that the testimony of a witness has to be corroborated, except in treason where the testimony of at least two (2) witnesses is needed to prove the same overt act, it has been held that the testimony of a selfconfessed accomplice or co-conspirator imputing the blame to or implicating his coaccused cannot, by itself and without corroboration, be considered as proof to a moral certainty that the latter committed or participated in the commission of the crime; it is required that the testimony be substantially corroborated by other evidence in all its material points. 10 We cannot consider the testimony of Joseph Cagas that he got from the appellant the marijuana leaves confiscated from him by NARCOM agents in a "buy-bust" operation conducted on 9 July 1985, to be corroborative of the testimony of Ronaldo Jomao-as because it appears to be incompatible with the testimony of another prosecution witness on a material point and, therefore, unreliable. Sgt. Adilhaman Asnawi, a member of the NARCOM team which arrested Ronaldo Jomao-as during the "buy-bust" operation on 29 June 1985, declared that his commanding officer decided to have a close surveillance on the appellant Alexander Spandonis after Ronaldo Jomao-as had told them that he (Jomao-as) got the marijuana leaves from the appellant. 11 There being a "close surveillance" on the appellant, it was to be expected that, at least, a report be made on every transaction subsequently entered into by the appellant and a buyer, or that the buyer be arrested together with the appellant. But it would appear that Cagas was not arrested together with the appellant. Nor was a report made on the transaction entered into by them. In fact, the appellant was not included in the criminal case filed against Joseph Cagas. It is thus possible that Joseph Cagas was not telling the truth when he said that he got marijuana leaves from the appellant on 9 July 1985. The testimony of Ronaldo Jomao-as, coming from a polluted source without being corroborated in its material points by other evidence, is not, therefore, sufficient to convict the appellant of the offense with which he is charged. Accordingly, the acquittal of the appellant is called for. In view of the foregoing, we no longer find it necessary to discuss the procedural issue raised by the appellant in his brief that the case should be dismissed for the additional ground of lack of preliminary investigation. WHEREFORE, the judgment appealed from is hereby REVERSED and SET ASIDE and another one entered acquitting the appellant Alexander Spandonis, on reasonable doubt, from the charge against him. With costs de oficio. SO ORDERED.

Melencio-Herrera, Paras, Sarmiento and Regalado, JJ., concur.

You might also like